Index

S.No

Chapter Name

Page No.

1. 2. 3. 4. 5. 6. 7. 8. 9. 10. 11. 12. 13. 14. 15.

Inequality Alphanumeric Series Syllogism Order & Ranking Coding & Decoding Blood Relation Direction & Distance Machine Input Output Puzzle Seating Arrangement Logical Reasoning Decision Making Word Problems Data Sufficiency Mixed Reasoning Questions

4 27 59 101 139 193 224 278 336 444 532 572 601 641 939

Join Our Social groups and support us

1. Facebook Page : https://www.facebook.com/governmentadda/ 2. Facebook Group : https://www.facebook.com/groups/governmentadda/ 3. Telegram Channel : https://telegram.me/GA_Buzz https://telegram.me/jobalert 4. Telegram Groups : Banking - https://telegram.me/IbpsZone 1. SSC - https://telegram.me/SscAdda 2. RBI - https://telegram.me/RbiZone 3. Railways - https://telegram.me/RailwayZone 4. IT Officer - https://telegram.me/IT_Officer 5. Insurance - https://telegram.me/InsuranceZone 5. Twitter Page - https://twitter.com/GovtAdda 6. Instagram Page - https://www.instagram.com/governmentadda/ 7. Google Group - https://plus.google.com/communities/101179260272404032205 8. Pinterest Page - https://www.pinterest.com/governmentadda/ 9. Website - https://www.GovernmentAdda.com

Visit Daily :

[GOVERNMENTADDA.COM]

120+ Reasoning Inequality Questions With Answers Governmentadda.com

GovernmentAdda.com | IBPS SBI SSC RBI RRB FCI RAILWAYS

1

Visit Daily :

[GOVERNMENTADDA.COM]

1. Statements: A ≥ B; C > G; A ≥ H;

Conclusions:

B ≥ C; I = B

I. G≥A

Conclusions:

II. G≥O

I. C > H

III. H>M

II. H > B

IV. H≤G

III. B > G

A. Only I, II and III are true

IV. I > A

B. Only II is true

A. Only I is true

C. Only IV is true

B. Only II is true

D. Only I and IV are true

C. Either I or II true

E. None is true

D. Neither I nor II is true E. Only III is true Answer & Explanation

Answer & Explanation Answer – D. Only I and IV are true

Answer – E. Only III is true 4. Statement : P ≥ Q > R < S ≤ T 2. Statements: A=B; C≤F; G>C; B
Conclusion:

Conclusions:

I. T > R

I. F < B

II. T > Q

II. F > G

III. R < P

III. A > G

IV. Q > P

IV. A > C

A. Only I is true

A. Only II is true

B. Only II is true

B. None is true

C. Only I and III are true

C. Only I and II are true

D. Only I and IV are true

D. Only II and III are true

E. All I, II, III and IV are true

E. Only IV is true

Answer & Explanation

Answer & Explanation Answer – C. Only I and III are Answer – B. None is true 3. Statements: A≤H, G≥H; G>M; O≤M

true 5. Statement : P < Q ≥ R > S ≤ T Conclusion: GovernmentAdda.com | IBPS SBI SSC RBI RRB FCI RAILWAYS

2

Visit Daily :

[GOVERNMENTADDA.COM]

I. T ≥ R II. P < R

Answer & Explanation Answer – A.Only I is true

III. Q > S IV. S < P

8. Statements: M≥N≥O>P≤Q≤R

A. Only I is true

Conclusions: i. M>Q, ii. N≤R

B. Only III is true

A.Only I is true

C. Only II is true

B.Only II is true

D. Only IV is true

C.Either I or II true

E. Both I and II are true

D.Neither I nor II is true

Answer & Explanation Answer – B. Only III is true 6. Statements: MM, ii. Q≤O

E.Both I and II are true Answer & Explanation Answer – D.Neither I nor II is true 9. Statements: M
A.Only I is true

H≥F
B.Only II is true

Conclusions: P≤B, M
C.Either I or II true

A.Only I is true

D.Neither I nor II is true

B.Only II is true

E.Both I and II are true

C.Either I or II true

Answer & Explanation

D.Neither I nor II is true E.Both I and II are true

Answer – E.Both I and II are true 7. Statements: A>N=I≥W
Answer & Explanation Answer – B.Only II is true

O>S; TS, ii. A
10. Statements: M
A.Only I is true

H≥F
B.Only II is true

Conclusions: H≥J, B
C.Either I or II true

A.Only I is true

D.Neither I nor II is true

B.Only II is true

E.Both I and II are true

C.Either I or II true

GovernmentAdda.com | IBPS SBI SSC RBI RRB FCI RAILWAYS

3

Visit Daily :

[GOVERNMENTADDA.COM]

D.Neither I nor II is true E.Both I and II are true Answer & Explanation

2. Statements: A * E, E $ F, F # O, O @L Conclusions: I. L # F

Answer – B.Only II is true

II. E @ O III. A # O

Direction(1-5): Study the following

IV. E @ L

information to answer the given questions

A. None is true

A$B means A is not smaller than B

B. Only I is true

A@B means A is neither smaller than nor

C. Only II is true

equal to B

D. Only III is true

A#B means A is neither greater than nor

E. Only IV is true

equal to B A&B means A is neither greater than nor smaller than B

Answer Answer – A. None is true

A*B means A is not greater than B 3. Statements: B @ Q, Q # A, A & L, 1. Statements: O & A, A $ R, R # S, S

L*N

*Q

Conclusions:

Conclusions:

I. N $ A

I. Q @ R

II. L @ Q

II. S @ O

III. B @ N

III. R & O

IV. Q # N

IV. R # O

A. I, II and III are true

A. Only I is true

B. I, II and IV are true

B. Only III is true

C. I, III and IV are true

C. Only IV is true

D. I, III and IV are true

D. Either III or IV is true

E. All are true

E. Either III or IV and I are true

Answer

Answer Answer – B. I, II and IV are true Answer – E. Either III or IV and I are true GovernmentAdda.com | IBPS SBI SSC RBI RRB FCI RAILWAYS

4

Visit Daily :

[GOVERNMENTADDA.COM]

4. Statements: E # M, M * N, N @

Directions (Q.No – 6-10) In these questions,

O, O $ P

relationships between different elements is

Conclusions:

shown in the statements. These statements

I. P # M

are followed by two conclusions.

II. P # N III. M # O

Give Answer

IV. N @ E

A. If only Conclusion I follows

A. II and III are true

B. If only Conclusion II follows

B. II and IV are true

C. If either Conclusion I or II follows

C. III and IV are true

D. If neither Conclusion I nor II follows

D. I, and IV are true

E. If both Conclusions I or II follow

E. All are true Answer

6. Statement: B≥E
Answer – D. II, and IV are true

I. S>E II.Q
5. Statements: A $ E, E @ F, F * G, G#H

Answer

Conclusions:

Answer – A. If only Conclusion I

I. H @ E

follows

II. A $ G III. E @ H

7. Statement: P≥Q>RN

IV. A @ F

Conclusions:

A. None is true

I. P>G

B. Only I is true

II. R>N

C. Only II is true D. Only III is true

Answer

E. Only IV is true

Answer – D. If neither Conclusion

Answer

I nor II follows

Answer – E. Only IV is true

8. Statement: A>S>P>O=E Conclusions:

GovernmentAdda.com | IBPS SBI SSC RBI RRB FCI RAILWAYS

5

Visit Daily :

[GOVERNMENTADDA.COM]

I. P≥E II. S>E

Answer & Explanation Answer – B. PT

Answer 2. Which of the following symbols Answer – B. If only Conclusion II

should be placed in the blank

follows

spaces respectively(in the same order from left to right) in order

9. Statement: A=B≥C, DD Answer

to complete the given expression in such a manner that both „N≥Q‟ as well as „Q≤M‟ definitely holds true? M _ N _ P _ Q _ R A. >, ≥, <, = B. >, >, ≥, <

Answer – B. If only Conclusion II

C. ≥, ≥, ≤,≤

follows

D. ≥, =, ≥,< E. Other than those given as options

10. Statement: P≥RE>O Conclusions: I. P>E II. Q>O Answer

Answer & Explanation Answer – D. ≥, =, ≥,< 3. In Which of the following expressions does the expression

Answer – B. If only Conclusion II

„D=V‟ to definitely hold true?

follows

A. K ≥ D ≤ R = P < S ≤ V B. U ≥ V ≥ M = F ≤ A ≥ D

1. In which of these expressions „U > W‟ be definitely false? A. U>P≥Q=G≥R>W

C. D ≥ C > Q ≥ B = N ≤ V D. G ≥ D = A < B ≤ S ≤ V E. V ≥ E = G ≥ W = Y ≥ D

B. PT C. W≤A≤L=RC>=F≤H; WT=O≥P; W
Answer & Explanation Answer – E. V ≥ E = G ≥ W = Y ≥ D GovernmentAdda.com | IBPS SBI SSC RBI RRB FCI RAILWAYS

6

Visit Daily :

[GOVERNMENTADDA.COM]

4. Which of the following expressions

B. >, >, ≥, <, >, =

is true if the expression

C. ≥, ≥, ≥, ≤, >, >

PS>M>=A is definitely

D. >, =, ≥, =, ≤, =

true?

E. Other than those given as options

A. A ≤ P B. S < P C. M > P

Answer & Explanation Answer – D. >, =, ≥, =, ≤, =

D. A < B E. T ≤ M

7. In Which of the following expressions does the expression

Answer & Explanation Answer – D. A < B

„L=T‟ and “E≥W” to definitely hold true? A. E ≥ W ≤ R = P < S ≤ T

5. In which of these expressions „S >

B. U ≥ T ≥ M = W ≤ E ≥ L

V‟ and „V > B‟ be definitely false?

C. L ≥ C > E ≥ W = N ≤ T

A. S>P≥Q=G≥R>V>B

D. E ≥ W = A < B ≤ S ≤ T

B. PT
E. T ≥ E = G ≥ W = Y ≥ L

C. B>V≤A≤L=RC>=F≤H; B>V
Answer & Explanation

E. S>T=O≥P; B
Answer – E. T ≥ E = G ≥ W = Y ≥

Answer & Explanation

L

Answer – B. PT
8. Which of the following expressions is true if the expression

6. Which of the following symbols

P= R ≥ S>M>=W>A = R

should be placed in the blank

is definitely true?

spaces respectively(in the same

A. W ≤ P

order from left to right) in order

B. S < P

to complete the given expression in

C. M < R

such a manner that both „B>S‟ as

D. W > Q

well as „E≤F‟ definitely holds true?

E. T ≤ M

B_A_S_E_D_F_G A. >, ≥, <, =, <, <

Answer & Explanation GovernmentAdda.com | IBPS SBI SSC RBI RRB FCI RAILWAYS

7

Visit Daily :

[GOVERNMENTADDA.COM]

Answer – C. M < R 9. In which of these expressions „P > R‟ and „P = R‟ be definitely true? A. S>P≥Q=G≥R>V B. PT C. V≤A≤L=RS>C>=F≤H; VT=O≥P; VR Answer & Explanation Answer – A. S>P≥Q=G≥R>V 10. In which of these expressions „T > P‟ and „T = P‟ be definitely false? A. T≥S≥P≥Q=G≥R>V B. PT C. V≤A≤L=RC>=F≤H=P≤Q=T; VT=O≥P; VT In which of these expressions „I > K‟ be definitely false? A. I>P≥Q=G≥R>K B. PT C. K≤A≤L=RC>=F≤H; KT=O≥P; KT Which of the following symbols should be placed in the blank spaces respectively(in the same order from left to right) in order to

complete the given expression in such a manner that both „F>N‟ as well as „N≤B‟ definitely holds true? B _ A _ N _ E _ F A. >, ≥, <, = B. >, >, ≥, < C. ≥, ≥, ≥,≤ D. ≥, =, ≤,< E. Other than those given as options Answer & Explanation Answer – D. ≥, =, ≤,< In Which of the following expressions does the expression „I≥D‟ to definitely hold true? A. K ≥ I ≤ R = P < S ≤ D B. U ≥ D ≥ M = F ≤ A ≥ I C. I ≥ C ≥ Q ≥ B = N ≥ D D. G ≥ I = A < B ≤ S ≤ D E. D ≥ E = G ≥ W = Y ≥ I Answer & Explanation Answer – C. I ≥ C ≥ Q ≥ B = N ≥ D Which of the following expressions is true if the expression PS>M≥E is definitely true? A. E ≤ P B. S < P C. M > P D. E < S E. T ≤ M Answer & Explanation Answer – D. E < S Statements: Y ≤ K < D = S; D < B < O; A ≥ D B, ii. Y < Z A.Only I is true B.Only II is true C.Either I or II true D.Neither I nor II is true E.Both I and II are true Answer & Explanation Answer – B.Only II is true Statements: HH, ii. B≤I A.Only I is true B.Only II is true C.Either I or II true D.Neither I nor II is true E.Both I and II are true Answer & Explanation GovernmentAdda.com | IBPS SBI SSC RBI RRB FCI RAILWAYS

8

Visit Daily :

[GOVERNMENTADDA.COM]

Answer – E.Both I and II are true Statements: A>Z=R≥NF; KF, ii. AC≤B≤M Conclusions: i. U>B, ii. J≤M A.Only I is true B.Only II is true C.Either I or II true D.Neither I nor II is true E.Both I and II are true Answer & Explanation Answer – D.Neither I nor II is true Statements: G M; D ≥ O; E ≥ F; N = E Conclusions: I. F > O II. O > E

III. E > M IV. N > D A. Only I is true B. Only II is true C. Either I or II true D. Neither I nor II is true E. Only III is true Answer & Explanation Answer – E. Only III is true 2. Statements: P=Q; R≤D; E>R; Q E III. P > E IV. P > R A. Only II is true B. None is true C. Only I and II are true D. Only II and III are true E. Only IV is true Answer & Explanation Answer – B. None is true 3. Statements: B≤P, E≥P; E>Q; L≤Q Conclusions: I. E≥B II. E≥L III. P>Q IV. B≤E GovernmentAdda.com | IBPS SBI SSC RBI RRB FCI RAILWAYS

9

Visit Daily :

[GOVERNMENTADDA.COM]

A. Only I, II and III are true

B. Only III is true

B. Only II is true

C. Only II is true

C. Only IV is true

D. Only IV is true

D. Only I and IV are true

E. Both I and II are true

E. None is true

Answer & Explanation

Answer & Explanation Answer – B. Only III is true Answer – D. Only I and IV are true

6. Statement: P≥M>FQ Conclusions:

4. Statement : A ≥ N > M < B ≤ L

I. P>B

Conclusion:

II. F>Q

I. L > M

A. If only Conclusion I follows

II. L > N

B. If only Conclusion II follows

III. M < A

C. If either Conclusion I or II

IV. B > A

follows

A. Only I is true

D. If neither Conclusion I nor II

B. Only II is true

follows

C. Only I and III are true

E. If both Conclusions I or II follow

D. Only I and IV are true E. All I, II, III and IV are true Answer & Explanation

Answer & Explanation Answer – D. If neither Conclusion I nor II follows

Answer – C. Only I and III are true

7. Statement: B≥D
5. Statement : B < A ≥ W > V ≤ X

I. E>D

Conclusion:

II.P
I. X ≥ W

A. If only Conclusion I follows

II. B < W

B. If only Conclusion II follows

III. A > V

C. If either Conclusion I or II

IV. V < B

follows

A. Only I is true

D. If neither Conclusion I nor II GovernmentAdda.com | IBPS SBI SSC RBI RRB FCI RAILWAYS

10

Visit Daily :

[GOVERNMENTADDA.COM]

follows

follows

E. If both Conclusions I or II follow

E. If both Conclusions I or II follow

Answer & Explanation

Answer & Explanation

Answer – A. If only Conclusion I

Answer – A. If only Conclusion II

follows

follows

8. A>T>Q>R=E

10. Statement: A≥MF>B

Conclusions:

Conclusions:

I. Q≥E

I. A>F

II. T>E

II. Y>B

A. If only Conclusion I follows

A. If only Conclusion I follows

B. If only Conclusion II follows

B. If only Conclusion II follows

C. If either Conclusion I or II

C. If either Conclusion I or II

follows

follows

D. If neither Conclusion I nor II

D. If neither Conclusion I nor II

follows

follows

E. If both Conclusions I or II follow

E. If both Conclusions I or II follow

Answer & Explanation

Answer & Explanation

Answer – B. If only Conclusion II

Answer – B. If only Conclusion II

follows

follows

9. Statement: S=R≥A, PP A. If only Conclusion II follows B. If only Conclusion I follows C. If either Conclusion I or II follows D. If neither Conclusion I nor II

In which of these expressions „B > E‟ be definitely false? A. B>P≥Q=G≥R>E B. PT C. E≤A≤L=RC>=F≤H; ET=O≥P; ET Which of the following symbols should be placed in the blank spaces respectively(in the same order from left to right) in order to complete the given expression in such a manner that both „O>N‟ as well as „G≤M‟ GovernmentAdda.com | IBPS SBI SSC RBI RRB FCI RAILWAYS

11

Visit Daily :

[GOVERNMENTADDA.COM]

definitely holds true? M _ A _ N _ G _ O A. >, ≥, <, = B. >, >, ≥, < C. ≥, ≥, ≥,≤ D. ≥, =, ≥,< E. Other than those given as options Answer & Explanation Answer – D. ≥, =, ≥,< In Which of the following expressions does the expression „Z≥D‟ to definitely hold true? A. K ≥ Z ≤ R = P < S ≤ D B. U ≥ D ≥ M = F ≤ A ≥ Z C. Z ≥ C > Q ≥ B = N ≥ D D. G ≥ Z = A < B ≤ S ≤ D E. D ≥ E = G ≥ W = Y ≥ Z Answer & Explanation Answer – C. Z ≥ C > Q ≥ B = N ≥ D Which of the following expressions is true if the expression PS>M≥A is definitely true? A. A ≤ P B. S < P C. M > P D. A < B E. T ≤ M Answer & Explanation Answer – D. A < B

Statements: B>Z=R≥MQ; KQ, ii. BC≤A≤L Conclusions: i. V>A, ii. I≤L A.Only I is true B.Only II is true C.Either I or II true D.Neither I nor II is true E.Both I and II are true Answer & Explanation Answer – D.Neither I nor II is true Statements: C
Statements: Y ≤ K < D = S; D < B < O; A ≥ D B, ii. Y < Z A.Only I is true B.Only II is true C.Either I or II true D.Neither I nor II is true E.Both I and II are true Answer & Explanation Answer – B.Only II is true

Statements: C
Statements: GG, ii. A≤F A.Only I is true B.Only II is true C.Either I or II true D.Neither I nor II is true E.Both I and II are true Answer & Explanation Answer – E.Both I and II are true

Statements: D ≥ E; F > A; D ≥ C; E ≥ F; B=E Conclusions: I. F > C II. C > E III. E > A IV. B > D A. Only I is true B. Only II is true C. Either I or II true GovernmentAdda.com | IBPS SBI SSC RBI RRB FCI RAILWAYS

12

Visit Daily :

[GOVERNMENTADDA.COM]

D. Neither I nor II is true E. Only III is true Answer & Explanation Answer – E. Only III is true Statements: A ≥ B; C = B; E > F; A ≤ D; B ≥F Conclusions: I. C < E II. D ≥ B III. A ≥ F IV. E > D A. Only II is true B. Only III is true C. II and III are true D. Either I or II true E. I and II are true Answer & Explanation Answer – C. II and III are true Statements: A > E; K ≥ M; E ≤ L; L = K; GM; K Q III. I > Q IV. I > M A. Only IV is true B. Only II is true C. Only I and II are true D. Only II and III are true E. None is true Answer & Explanation Answer – E.None is true Statements: C ≥ A; E = G; K < I; C< E; C< M; G ≥ I

Conclusions: I. G < K II. A ≥ E III. I < M IV. A < E A. Only I is true B. Only IV is true C. Only III is true D. None is true E. All are true Answer & Explanation Answer – B. Only IV is true Statements: S = T; U < O; S ≤ U ; P ≥ Q; Q >T Conclusions: I. P > S II. Q > S III. O > T IV. T ≤ U A. Only I is true B. Only II is true C. Only I and II are true D. All are true E. Only III and IV are true Answer & Explanation Answer – D. All are true Statements: A>C; G>E; G≤C; R≤I; K≤I Conclusions: I. A>G II. C>E III. I≥K IV. C≤K A. Only I, II and III are true B. Only II is true C. Only I, II and IV are true D. Neither I nor II is true E. None is true Answer & Explanation Answer – A. Only I, II and III are true Statements: M≤Q, O≥Q; O>R; P≤R Conclusions: I. O≥M II. O≥P III. Q>R IV. M≥O A. Only I, II and III are true B. Only II is true C. Only I is true GovernmentAdda.com | IBPS SBI SSC RBI RRB FCI RAILWAYS

13

Visit Daily :

[GOVERNMENTADDA.COM]

D. Neither I nor II is true E. None is true Answer & Explanation Answer – C. Only I is true Statements: BH; H=A; J>L; L≥F Conclusions: I. J>A II. C=H III. HR, P = O ≥ T, S ≥ U>Q Conclusions: I.S≥R II.PR A.Only I is true B.Only II is true C.Only III is true D.Only IV is true E.All are true Answer & Explanation Answer –D.Only IV is true Explanation : S≥U>Q≥RC, DB III.E>C IV.B>C

A.Only I is true B.Only II is true C.Only III is true D.Only IV is true E.All are true Answer & Explanation Answer – B.Only II is true Explanation : EC≥A=B Statements: O=R, P=N>I, M≥R, N >M Conclusions: I.OI Statements: P
S, B=Q>T, S≤MS≤M G Conclusions: I.HG≤J< I Statements: D < A ≤ E < L, A = K > N, L > K≥M GovernmentAdda.com | IBPS SBI SSC RBI RRB FCI RAILWAYS

14

Visit Daily :

[GOVERNMENTADDA.COM]

Conclusions: I.D>M II.L>M III.K ≤ N IV.M ≤ A A.Only I is true B.Only II is true C.Only I and II are true D.Only II and IV are true E.All are true Answer & Explanation Answer – D.Only II and IV are true Explanation : D
N L>A=K≥M Statements: X > S = T ≤ U, Y = Z > V ≥ U, S=W>O Conclusions: I.S=W II.TO IV.T≤V A.Only I and IV are true B.Only II is true C.Only II and III are true D.Either I or III and I are true E.All are true Answer & Explanation Answer – E.All are true Explanation : X > S=T=W>O X > S = T ≤ U ≤ VK=G Conclusions: I.A≥T II.T=G III.KT A.Only I and II are true B.Only I and III are true C.Either I or III are true D.Either I or IV and III are true E.All are true Answer & Explanation Answer – B.Only I and III are true Explanation : A ≥ Q = B≥T≤H≥F F ≤ H ≥ T>K=G A ≥ Q = B≥T>K=G What should come in place of question mark to make S>Q always true? R≤T>P=S>R ? O >Q A.> B.= C.≥

D.All of these E.None of these Answer & Explanation Answer – D.All of these Explanation : R≤T>P=S>R = O >Q R≤T>P=S>R > O >Q R≤T>P=S>R ≥ O >Q What should come in place of question mark to make E≤I always true? D>E≤F=M?K≤C ? I A.≤, ≤ B.≤, < C.=, ≥ D.≥, ≤ E.None of these Answer & Explanation Answer – A.≤, ≤ Explanation : D>E≤F=M≤K≤C≤I 1. Statements: M≤V; PT; R≤T Conclusions: I. S>M II. V=P III. P
GovernmentAdda.com | IBPS SBI SSC RBI RRB FCI RAILWAYS

15

Visit Daily :

[GOVERNMENTADDA.COM]

Explanation:

D. II and III are true

P=M
E. I and II are true

2. Statements: B
Answer & Explanation Answer –D. II and III are true D≥A≥B=C≥FQ;

iii. Q≥M

M
iv. F
Conclusions:

A. Only I, II and III are true

I. M < R

B. Only II is true

II. R > T

C. Only II,III and IV are true

III. P > T

D. Neither I nor II is true

IV. P > Q

E. None is true

A. None is true

Answer & Explanation

B. Only I is true C. Only I and II are true

Answer – C. Only II,III and IV are

D. Only II and III are true

true

E. Only IV is true

Explanation: B
Answer & Explanation Answer – B. Only I is true Explanation: T>Q≤R>M=P

Conclusions: I. C < E

5. Statements: S = A; R < T; R ≥ S ;

II. D ≥ B

P ≥ Q; Q > A

III.A ≥ F

Conclusions:

IV.E > D

I. P > S

A. Only II is true

II. Q > S

B. Only III is true

III. T < A

C. Either I or II true

IV. A ≥ R A. Only I is true GovernmentAdda.com | IBPS SBI SSC RBI RRB FCI RAILWAYS

16

Visit Daily :

[GOVERNMENTADDA.COM]

B. Only II is true C. Only I and II are true D. Only III and IV are true E. All are true Answer & Explanation

Answer & Explanation Answer – D. Only III is true Explanation: $=≥ @=> #=<

Answer – C. Only I and II are true

&==

Explanation:

*=≤

P≥Q>A=S≤R
KL≤A≥M 7. Statements: W*N, K * V, Y @ V, W@K Conclusions: I. Y @ K II. W $ N

P#Q means P is neither greater than nor equal to Q

III.W @ Y IV.W @ V

P&Q means P is neither greater than nor smaller

A. None is true

than Q

B. Only I is true

P*Q means P is not greater than Q

C. Only II is true D. Only III is true

6. Statements: A $ M, P @ L, K # P,

E. Only IV is true

A$L Conclusions: I. K # L II. A @ P III.L*A

Answer & Explanation Answer – B. Only I is true Explanation: N≥W>K≤V
IV.M # P A. None is true

8. Statements: T * Y, S # M, Y $ S, M

B. Only I is true

@K

C. Only II is true

Conclusions:

D. Only III is true

I. K # S

E. Only IV is true

II. Y @ M GovernmentAdda.com | IBPS SBI SSC RBI RRB FCI RAILWAYS

17

Visit Daily :

[GOVERNMENTADDA.COM]

III.T # M

Conclusions:

IV.Y @ K

I. M # P

A. None is true

II. M @ L

B. Only I is true

III.P # E

C. Only II is true

IV.L # C

D. Only III is true

A. None is true

E. Only IV is true

B. Only I is true

Answer & Explanation Answer – A. None is true Explanation:

C. Only II is true D. Only III is true E. Only IV is true Answer & Explanation

T ≤Y ≥ SK Answer – A. None is true 9. Statements: S @ L, L # M, M & B, B*Q

Explanation: CE≥M

Conclusions: I. Q $ M II. B @ L III.S @ Q IV.L @ Q A. I, II and III are true B. I, II are true C. I, III are true D. I, III and IV are true E. All are true Answer & Explanation Answer – B. I, II are true Explanation: S>L
Statements: P ≥ Q; R > M; P ≥ O; Q ≥ R; N = Q Conclusions: I. R > O II. O > Q III. Q > M IV. N > P A. Only I is true B. Only II is true C. Either I or II true D. Neither I nor II is true E. Only III is true Answer & Explanation Answer – E. Only III is true Statements: A ≥ B; C = B; E > F; A ≤ D; B ≥F Conclusions: I. C < E II. D ≥ B III. A ≥ F IV. E > D A. Only II is true B. Only III is true C. Either I or II true GovernmentAdda.com | IBPS SBI SSC RBI RRB FCI RAILWAYS

18

Visit Daily :

[GOVERNMENTADDA.COM]

D. II and III are true E. I and II are true Answer & Explanation Answer – D. II and III are true Statements: P > Q; S ≥ U; Q ≤ T; R = S; R C; B E III. A > E IV. A > C A. None is true B. Only II is true C. Only I and II are true D. Only II and III are true E. Only IV is true Answer & Explanation Answer – A.None is true Statements: Q ≥ P; R = S; U < T; Q< R; Q< V; S ≥ T Conclusions: I. S < U II. P > R III. T < V IV. P < R A. Only I is true B. Only IV is true C. Only III is true D. None is true E. All are true Answer & Explanation Answer – B. Only IV is true Statements: W = X; R < C; W ≤ R ; A ≥ B; B>X

Conclusions: I. A > W II. B > W III. C > X IV. X ≤ R A. Only I is true B. Only II is true C. Only I and II are true D. Only III and IV are true E. All are true Answer & Explanation Answer – E. All are true Statements: T>O; S>R; S≤O; R≤F; U≤F Conclusions: I. T>S II. O>R III. F≥U IV. O≤U A. Only I, II and III are true B. Only II is true C. Only I, II and IV are true D. Neither I nor II is true E. None is true Answer & Explanation Answer – A. Only I, II and III are true Statements: B≤U, E≥U; E>V; L≤V Conclusions: I. E≥B II. E≥L III. U>V IV. B≤E A. Only I, II and III are true B. Only II is true C. Only I and IV are true D. Neither I nor II is true E. None is true Answer & Explanation Answer – C. Only I and IV are true Statements: C
19

Visit Daily :

[GOVERNMENTADDA.COM]

D. Neither I nor II is true E. None is true Answer & Explanation Answer – C. Only II and IV are true

B) only II follows C) either I or II follows D) neither I nor II follow E) both I and II follow

Statements: N≤Q; R>P; P=N; S>T; T≥R Conclusions: I. S>N II. Q=P III. P
View Answer

Direction (1-6): Relationship between different elements is shown in the statements. Find if the conclusions also follow or not. 1. Statements: A > L ≥ H > M = D < G ≤ F; U ≤ K = P > M; N= K < S Conclusions: I. A > S II. A = S A) only I follows B) only II follows C) either I or II follows D) neither I nor II follow E) both I and II follow

Option E 3. Statements: V < L = D < K ≥ H > S; P ≤ K = D > E; K = F ≤ Z Conclusions: I. Z > L II. E < V A) only I follows B) only II follows C) either I or II follows D) neither I nor II follow E) both I and II follow View Answer Option B Solution: Z ≥ K = D = L, So Z ≥ L 4. Statements: K = L < D ≤ G = P; U > E = D ≥ X ≤ B; P > O ≥ V < S Conclusions: I. V < X II. P ≥ X A) only I follows B) only II follows C) either I or II follows D) neither I nor II follow E) both I and II follow

View Answer View Answer Option D Solution: A>M, = sign should be present in conclusion 2. Statements: B < G ≤ V = F > H; K = L > F ≤ N = P; D > N = S Conclusions: I. H < S II. B < P A) only I follows

Option B 5. Statements: K = L < D ≤ G = P; U > E = D ≥ X ≤ B; P > O ≥ V < S Conclusions: I. U > K II. P > B A) only I follows B) only II follows C) either I or II follows D) neither I nor II follow E) both I and II follow View Answer GovernmentAdda.com | IBPS SBI SSC RBI RRB FCI RAILWAYS

20

Visit Daily :

[GOVERNMENTADDA.COM]

Option A 6. Statements: B < G ≤ V = F > H; K = L > F ≤ N = P; D > N = S Conclusions: I. L > G II. D > K A) only I follows B) only II follows C) either I or II follows D) neither I nor II follow E) both I and II follow View Answer Option A

7. Which of the following would replace & and # in the following expression so that ‘A > N’ holds true? A > L ≥ H & M = D < G ≤ F; U ≤ K = M > P; N # K < S A) =, = B) ≥, > C) ≥, ≥ D) <, = E) None of these View Answer Option A Solution: A > L ≥ H = M = K = N, So A > N 8. In which of the following expressions does the expression ‘G < S’ definitely hold true? A) A = S < F ≥ H = K > G > D B) D > A = G ≥ B = F ≤ S < H C) A < O > G < H = H < S ≥ B D) G = U ≤ B = S ≤ H = O < A E) None of these View Answer Option C 9. In which of these expression ‘L ≥ R’ is definitely false?

A) W < R ≥ S ≥ Q < N> A ≥ L > V B) N > L > M = D ≥ B = A > P = R C) M ≤ A > L > W ≥ V ≤ B = P < R D) S > L = C ≥ H = V ≥ P ≤ R = T E) B > R ≤ A = M = Q ≤ T = L < G View Answer Option B Solution: in B) – L > R so ‘L ≥ R’ is definitely false In A), C) and D) – relation between L and R cannot be determined, so cannot be told that whether ‘L ≥ R’ is definitely false or not. In E) – L ≥ R is true 10. Which of the following expressions is definitely false if the expression K > O = G ≤ D > F = P ≥ Q < T is definitely true? A) K > D B) F ≥ T C) F < G D) D = Q E) P < O View Answer Option D Solution: D < Q in given expression, so D = Q is definitely false Direction (1-6): Relationship between different elements is shown in the statements. Find if the conclusions also follow or not. 1. Statements: H ≥ O = U ≥ B < L = P; D < N =B≥S>K Conclusions: I. K < L II. H ≥ K A) only I follows B) only II follows C) either I or II follows D) neither I nor II follow E) both I and II follow View Answer

GovernmentAdda.com | IBPS SBI SSC RBI RRB FCI RAILWAYS

21

Visit Daily :

[GOVERNMENTADDA.COM]

Option A 2. Statements: B < N = T ≥ G > H = F; G > L = D > V; L > W = A Conclusions: I. A < H II. V < B A) only I follows B) only II follows C) either I or II follows D) neither I nor II follow E) both I and II follow View Answer Option D 3. Statements: F ≥ V = T ≥ G < L ≤ D = S; E = Q < T ≤ N; Q > P = W Conclusions: I. D > N II. F > W A) only I follows B) only II follows C) either I or II follows D) neither I nor II follow E) both I and II follow View Answer Option B 4. Statements: N > D ≥ F > J; E < L ≤ G < S < P < F; G > W Conclusions: I. W < J II. J ≤ W A) only I follows B) only II follows C) either I or II follows D) neither I nor II follow E) both I and II follow View Answer Option C Solution: W < G < S < P < F > J. So no relation between W and J, so 1) either W > J or W ≤ J follows OR 2) either W < J or ≥ J follows —— which is given case

5. Statements: H > L = G ≥ S < L ≤ W; S > W > P = R ≤ V; P < X = O Conclusions: I. W > R II. O > R A) only I follows B) only II follows C) either I or II follows D) neither I nor II follow E) both I and II follow View Answer Option E 6. Statements: V < E = D = W ≥ L; F ≥ S = D < K; L ≥ R = H ≥ B Conclusions: I. B < S II. B = S A) only I follows B) only II follows C) either I or II follows D) neither I nor II follow E) both I and II follow View Answer Option C Solution: B ≤ L ≤ W = D = S, so B ≤ S

7. Which of the following would replace % and # in the following expression so that A ≤ B holds true? Q < D % S ≥ A = W; B ≥ P # D = Z > X A) >, ≤ B) ≥, > C) ≥, ≥ D) >, = E) None of these View Answer Option C Solution: A≤S≤D≤P≤B 8. In which of the following expressions does the expression ‘B ≤ H’ and ‘A > G’ GovernmentAdda.com | IBPS SBI SSC RBI RRB FCI RAILWAYS

22

Visit Daily :

[GOVERNMENTADDA.COM]

definitely hold true? A) A = B < F ≥ H = K > G > D B) D > A = G ≥ B = F ≤ G < H C) A < O > G < H = H ≥ S ≥ B D) G = U ≤ B = E ≤ H = O < A E) None of these View Answer Option D 9. In which of these expression ‘L > P’ is definitely false? A) W < P ≥ S ≥ Q < N> A ≥ L > V B) N > L > M = D ≥ B = A > P = R C) M ≤ A > L > W ≥ V ≤ B = P < S D) S > L = C ≥ H = H ≥ P ≤ Q = T E) B > L ≤ A = M < Q ≤ T = P < G View Answer Option E Solution: In A, B, C – relation between L and P

cannot be determined, so cannot be told that whether ‘A ≤ P’ is definitely false or not. In D) – L ≥ P, so cannot be said that whether L > P or L = P, so cannot be said that ‘L > P’ is definitely false In E) – L < P, so ‘L > P’ is definitely false 10. In which of these expression ‘A ≤ P’ is definitely false? A) W < P ≥ S ≥ Q < N> A ≥ R > V B) N > A > M = D ≥ B = L > P = R C) M ≤ A > L > W ≥ V ≤ B = P < S D) S > A > = C ≤ H = P ≤ Q = T = K E) B > L ≤ A > M ≥ Q < T > P < G View Answer Option B Solution: In B) – A > P so ‘A ≤ P’ is definitely false. In other option, relation between A and P cannot be determined, so cannot be told that whether ‘A ≤ P’ is definitely false or not.

GovernmentAdda.com | IBPS SBI SSC RBI RRB FCI RAILWAYS

23

Daily Visit:

[GOVERNMENTADDA.COM]

120+ Alphanumeric Series Questions With solutions

Governmentadda.com

IBPS SBI RBI SSC RRB FCI RAILWAYS SSB |

1

Daily Visit:

[GOVERNMENTADDA.COM]

Directions (1-5): In each question below is given a group of letters followed by four combinations of digits/symbols given in (A), (B), (C) and (D). You have to find out which of the combinations correctly represents the group of digits based on the rules and mark the answer accordingly. If none of the combinations correctly represents the group of letters, mark (E) as your answer. (i) If the first two letters are vowels, the codes for first and last letters are to be interchanged. (ii) If the first letter is a consonant, the codes for the second and third letters are to be interchanged. (iii) If the last letter is a vowel then the second letter is to be coded as the code for the vowel.

1. UEPMOQ A) 3^70@# B) 4^70!# C) 4^70@# D) 4^70©# E) None of these View Answer

Option C Explanation: The given number satisfies rule (i) 2. OPHDLU A) #@286# B) @#286# C) @#246 D) @#286 E) None of these View Answer

Option B Explanation: The given number satisfies rule (iii) 3. MIRKNQ A) O©#9*4 B) 9©%9*4 C) O©%9*4 D) O©%9@4 E) None of these View Answer

IBPS SBI RBI SSC RRB FCI RAILWAYS SSB | Governmentadda.com

2

Daily Visit:

[GOVERNMENTADDA.COM]

Option C Explanation: The given number satisfies rule (ii) 4. HOLUIB A) 26@!%3 B) 26^#%3 C) 26@#%7 D) 24@#%3 E) None of these View Answer

Option E Explanation: The given number satisfies rule (ii) 26@#%3 5. OINRBK A) 9@*©3@ B) 9%6©3@ C) 7%*©3@ D) 9%*©3@ E) None of these View Answer

Option D Explanation: The given number satisfies rule (i) Directions (6-10): Study the following arrangement and answer questions given: 4

8 @ B D E ! Y I 7 3 N ( M $

*

K

W 6

A

L 5

#

9

O

2

U

^

6. Which of the following is the 9th letter to the left of the 16th letter from the right end? A) E B) B C) D D) * E) @ View Answer

Option C Left to right = 16+9 = 25th from right 7. How many such letters are there in the above arrangement each of which is immediately preceded by a symbol and immediately followed by a consonant? A) Four IBPS SBI RBI SSC RRB FCI RAILWAYS SSB | Governmentadda.com

3

©

Daily Visit:

[GOVERNMENTADDA.COM]

B) Three C) None D) One E) Five View Answer

Option B Solution: B, K, and M 8. Complete the series: 4$V (BD !©^ 27* ? A) K#5 B) W95 C) W#5 D) W9# E) K9# View Answer

Option C Solution: 4$V – 4=first element, $= first element from last, V = previous letter of $ (BD – ( = leaving one letter from V, ( = 4th element from last, B= 4th element from starting, D = next letter to B !©^ – ! = leaving one letter from D, ! = 7th from starting, © = 7th from last, ^ = previous to © 27* – 2 = leaving one letter from ^, 2 = 10th from last, 7 = 10th from starting, * = next to 7 9. How many such numbers are there each of which is immediately preceded by a symbol? A) None B) One C) Two D) Three E) More than three View Answer

Option C Solution: 9, and 3 10. Four of the five are alike in a certain way based on arrangement, which does not belong to the group? A) KW* B) BD@ C) MV( D) V$( E) ©3^ View Answer

IBPS SBI RBI SSC RRB FCI RAILWAYS SSB | Governmentadda.com

4

[GOVERNMENTADDA.COM]

Daily Visit:

Option D Directions (1-5): Study the following arrangement and answer questions given: F +

N S

$

3 U

2 #

* L

W T

E

%

!

B

M

P

(

A

V

~

9

F

6

1. If the first fifteen elements in the above passage sequence are written in reverse order then which of the following will be fifteenth from the right end? A) $ B) 3 C) P D) ( E) 2 View Answer

Option B 2. If the above sequence is written in reverse order then which of the following will be seventh to the right of sixteenth element from the right end? A) ! B) A C) E D) % E) ~ View Answer

Option D 16-7 = 9th from right end So in given series, 9th from left end will be 9th from right end in reverse series 3. What should come in place of question mark in the following on the basis of the above sequence ? 32$ : WE*

P(M : V~A

%!E : ?

A) P(M B) AV( C) BM! D) Other than given in options. E) MPB

View Answer

Option E IBPS SBI RBI SSC RRB FCI RAILWAYS SSB | Governmentadda.com

5

Daily Visit:

[GOVERNMENTADDA.COM]

4. How many such numbers are there in the above sequence, each of which is immediately preceeded by a consonant and immediately followed by a symbol? A) None B) One C) Two D) Three E) More than three View Answer

Option A 5. Which of the following is fourth to the left of sixteenth element from the left? A) ~ B) P C) ( D) % E) M View Answer

Option E Directions (6-10): Study the following arrangement and answer questions given: 7 4 Y 3 I

* +

# D < =

E G

9

L

0

1

&

~

M

B

)

6

S

A

X

J

6. Which of the following is the 5th letter to the right of the 16th letter from the right end? A) & B) # C) 6 D) S E) ) View Answer

Option C Solution: Right to right = 16-5 = 11th from right 7. How many such symbols are there in the above arrangement each of which is immediately preceded by a consonant and immediately followed by a number? A) Four B) Three C) None IBPS SBI RBI SSC RRB FCI RAILWAYS SSB | Governmentadda.com

6

Daily Visit:

[GOVERNMENTADDA.COM]

D) One E) Five View Answer

Option D Only one – ) 8. Which of the following is the 9th letter to the left of the 18th letter from the left end? A) 9 B) L C) 0 D) ~ E) & View Answer

Option B Solution: Left to left = 18-9 = 9th from left 9. How many such numbers are there each of which is immediately preceded by a consonant? A) None B) One C) Two D) Three E) More than three View Answer

Option C Solution: 0 and 3 10. Four of the five are alike in a certain way based on arrangement, which does not belong to the group? A) 1&9 B) 6SM C) 3I6 D) GJ+ E) 9L# View Answer

Option C Directions (1-5): Study the following arrangement and answer questions given: IBPS SBI RBI SSC RRB FCI RAILWAYS SSB | Governmentadda.com

7

Daily Visit:

2 8

[GOVERNMENTADDA.COM]

0 4 5 3 6 8 4 2 5 1 7 9 2 5 7 3 1 7 4 2 9 0

4

8

5

6

9

3

2

0

4

7

4

5

1. Which of the following is the 9th letter to the left of the 21st letter from the right end? A) 4 B) 8 C) 6 D) 2 E) 5 View Answer

Option B Solution: Left from Right = 21+9 = 30th from right 2. How many such 4’s are there in the above arrangement, each of which is immediately preceded by an odd digit and immediately followed by an even digit? (0 is neither odd nor even) A) Four B) Three C) Two D) One E) Five View Answer

Option C Solution: 7 4 8, 7

4 2

3. How many such 7’s are there in the above arrangement, each of which is immediately preceded by a perfect square? A) Four B) Three C) Two D) One E) Five View Answer

Option B Solution: 1 7, 4 7, 1

7

4. How many such 5’s are there in the above arrangement, each of which is immediately followed by a digit which has a numerical value greater than 6? A) None B) One C) Two D) Three E) More than three IBPS SBI RBI SSC RRB FCI RAILWAYS SSB | Governmentadda.com

8

Daily Visit:

[GOVERNMENTADDA.COM]

View Answer

Option C Solution: 5 8, 5 7 5. If all the odd numbers are deleted from the above arrangement, then which of the following will be 9th from the right end? A) 2 B) 6 C) 4 D) 0 E) 8 View Answer

Option A Directions (6-10): Study the following arrangement and answer questions given: R % # D E W 3 O H 1 2 $ B

@ 0 M I )

!

L 6

*

A

G

U 8

^

V

&

S

7

6. Which of the following is the 5th letter to the right of the 16th letter from the right end? A) S B) ^ C) V D) & E) * View Answer

Option D Solution: Right to right = 16-5 = 11th from right 7. How many such symbols are there in the above arrangement each of which is immediately preceded by a consonant and immediately followed by a number? A) Four B) Three C) None D) One E) Five View Answer

Option C 8. Which of the following is the 9th letter to the left of the 18th letter from the left end? A) @ IBPS SBI RBI SSC RRB FCI RAILWAYS SSB | Governmentadda.com

9

Daily Visit:

[GOVERNMENTADDA.COM]

B) 0 C) M D) 8 E) & View Answer

Option B Solution: Left to left = 18-9 = 9th from left 9. How many such numbers are there each of which is immediately preceded by a consonant? A) None B) One C) Two D) Three E) More than three View Answer

Option E Solution: 3, 6, 7, 1 10. Four of the five are alike in a certain way based on arrangement, which does not belong to the group? A) @3! B) GA^ C) OS2 D) D#3 E) 8U& View Answer

Option C Directions (1-5): In each question below is given a group of letters followed by four combinations of digits/symbols given in (A), (B), (C) and (D). You have to find out which of the combinations correctly represents the group of digits based on the rules and mark the answer accordingly. If none of the combinations correctly represents the group of letters, mark (E) as your answer. (i) If the first letter is a vowel and the last letter is a consonant both are to be coded as *. (ii) If the first letter is a consonant and the last letter is a vowel their codes are to be interchanged. (iii) If both the first and the last letters are vowels both are to be coded as the first letter.

IBPS SBI RBI SSC RRB FCI RAILWAYS SSB | Governmentadda.com

10

Daily Visit:

[GOVERNMENTADDA.COM]

1. IFNVDE A) 26%#92 B) 46%#92 C) 46%#94 D) *%#9* E) None of these View Answer

Option A Solution: From rule (iii) 2. KEIMPD A) 9423$1 B) 1423$9 C) 9423$9 D) 1423$1 E) None of these View Answer

Option B Solution: No rule is implied here. So all codes written as it is 3. UBTIZN A) 7!©25% B) %!©257 C) *!©25* D) 7!©257 E) None of these View Answer

Option C Solution: From rule (i) 4. BAEFNP A) !@46%! B) $!46%! C) *@46%* D) $@46%$ E) None of these View Answer

IBPS SBI RBI SSC RRB FCI RAILWAYS SSB | Governmentadda.com

11

Daily Visit:

[GOVERNMENTADDA.COM]

Option E Solution: No rule is implied here. So all codes written as it is BAEFNP becomes — !@46%$ 5. RKPZFI A) 81$562 B) 81$568 C) 21$562 D) 21$568 E) None of these View Answer

Option D Solution: From rule (ii) Directions (6-10): Study the following arrangement and answer questions given: P 7 @ I R W 3 9 T E $ 4 H D 5 © M A 2 % B * 8 ! U Q N 1 F 6 # K V 6. How many such consonants are there in the above arrangement, each of which is immediately preceded by a consonant and immediately followed by a symbol? A) None B) One C) Two D) Three E) More than three View Answer

Option A 7. If all the symbols are dropped from the above arrangement, which of the following will be the 15th from the left end? A) A B) 5 C) D D) 2 E) None of these View Answer

Option A

IBPS SBI RBI SSC RRB FCI RAILWAYS SSB | Governmentadda.com

12

Daily Visit:

[GOVERNMENTADDA.COM]

8. How many such numbers are there in the above arrangement, each of which is immediately preceded by a symbol and immediately followed by a consonant? A) None B) One C) Two D) Three E) More than three View Answer

Option B 9. Four of the following five are alike in a certain way based on their positions in the above arrangement and so form a group. Which is the one that does not belong to that group? A) B*2 B) QN! C) 39R D) 6FK E) %BA View Answer

Option D 10. Which of the following is the 9th to the right of the 22nd from the right end of the above arrangement? A) H B) # C) B D) @ E) None of these View Answer

Option C Directions (1-5): Study the following arrangement and answer questions given: (

F $

E ~ 3 ^

6 2 % S P 4 C

A V

8 #

B

)

*

1

L

O

9

@

W

U

1. How many such vowels are there in the above arrangement, each of which is immediately followed by a symbol and not preceded by a letter? A) Four B) Three C) Two IBPS SBI RBI SSC RRB FCI RAILWAYS SSB | Governmentadda.com

13

Daily Visit:

[GOVERNMENTADDA.COM]

D) One E) None View Answer

Option E 2. What is the sum of all numbers which are immediately preceded by a symbol? A) There are no such numbers B) 10 C) 12 D) 9 E) 11 View Answer

Option B Explanation: 6+1+3 = 10 3. What will come in place of questions mark (?) based on the above arrangement? 3U^ $W3 U@$ W9U ? A) 9OW B) Other than those given in options C) @9W D) @OW E) U@W View Answer

Option D 4. Four of the following five are alike in a certain way based on their positions in the above arrangement and so form a group. Which is the one that does not belong to that group? A) (#V B) EC4 C) 2^P D) S$U E) *OL View Answer

Option C 5. Which of the following is the 9th to the left of the 17th from the left end of the above arrangement? IBPS SBI RBI SSC RRB FCI RAILWAYS SSB | Governmentadda.com

14

Daily Visit:

[GOVERNMENTADDA.COM]

A) @ B) S C) 9 D) % E) O View Answer

Option B Explanation: Left of left – 17-9 = 8th from left = S Directions (6-10): Study the following arrangement and answer questions given: K I

J 2 ^ U % N P

! 4

A E

S 3 * M

C 7 + W

$

9

L

#

D

O

8

X

6. How many such numbers are there in the above arrangement, each of which is immediately preceded by a consonant and immediately followed by a vowel? A) One B) Three C) Two D) Five E) None View Answer

Option A Only 4 — P 4 E 7. How many such consonants are there in the above arrangement, each of which is immediately preceded by a symbol and immediately followed by a letter? A) Four B) Three C) Two D) Five E) None View Answer

Option C Explanation: D, N 8. Which of the following is the 8th to the right of the 19th from the right end of the above arrangement? IBPS SBI RBI SSC RRB FCI RAILWAYS SSB | Governmentadda.com

15

Daily Visit:

[GOVERNMENTADDA.COM]

A) % B) # C) 8 D) X E) None of these View Answer

Option D Explanation: Right to right = 19-8=11th from right = X 9. Four of the following five are alike in a certain way based on their positions in the above arrangement and so form a group. Which is the one that does not belong to that group? A) $97 B) U!^ C) XI8 D) +WM E) C7S View Answer

Option E 10. Which of the following is the 5th to the left of the 13th from the right end of the above arrangement? A) 7 B) 9 C) S D) $ E) ! View Answer

Option D Explanation: Left of right = 13+5=18th from right Directions (1-5): In each question below is given a group of digits followed by four combinations of digits/symbols given in (A), (B), (C) and (D). You have to find out which of the combinations correctly represents the group of digits based on the rules and mark the answer accordingly. If none of the combinations correctly represents the group of letters, mark (E) as your answer. (i) If the first digit is odd and last is even, then interchange the codes for both digits (ii) If there is a digit which is getting repeated in the given number, then code that digit as the code of 5 (iii) If the third digit is followed by an even number and is preceded by an odd number, then skip its IBPS SBI RBI SSC RRB FCI RAILWAYS SSB | Governmentadda.com

16

Daily Visit:

[GOVERNMENTADDA.COM]

code. (iv) If the middle digit is odd then interchange the codes of middle digit and last number. (v) 0 is neither odd nor even

1. 53678 A) #©DH! B) ©!DH# C) !©DH# D) !©HD# E) None of these View Answer

Option C Solution: The given number satisfies rule (i) 2. 12375 A) @L©H# B) @L#H© C) @H#L© D) @L!H© E) None of these View Answer

Option B Solution: The given number satisfies rule (iv) 3. 36014 A) ©DE@O B) OED@© C) ODE%© D) ODE@© E) None of these View Answer

Option D Solution: The given number satisfies rule (i) IBPS SBI RBI SSC RRB FCI RAILWAYS SSB | Governmentadda.com

17

[GOVERNMENTADDA.COM]

Daily Visit:

4. 81624 A) !@LDO B) !@DLO C) !@LO D) !@OL E) None of these View Answer

Option C Solution: The given number satisfies rule (iii) 5. 60456 A) *O#H# B) #EO## C) *©#H# D) OB©# # E) None of these View Answer

Option B Solution: The given number satisfies rule (ii) Directions (6-10): Study the following arrangement and answer questions given: H N 4 & U 7 !

T E 3 (

@ 5

R 1

* J

S 2

%

9

M 6 A L O

# V

1. Which of the following is the 6th letter to the right of the 20th letter from the right end? A) J B) S C) 2 D) % E) @ View Answer

Option B Solution: Right to right = 20-6 = 14th from right 2. How many such numbers are there in the above arrangement each of which is immediately preceded by a symbol and immediately followed by a letter? A) None B) One C) Two D) Three E) More than three IBPS SBI RBI SSC RRB FCI RAILWAYS SSB | Governmentadda.com

18

Daily Visit:

[GOVERNMENTADDA.COM]

View Answer

Option C Solution: 5, and 9 3. Complete the series: T&3 A) %26 B) %29 C) %SM D) %2M E) None of these

(35

R5*

J*2 ?

View Answer

Option D 4. How many such consonants are there each of which is immediately preceded by a number? A) None B) One C) Two D) Three E) More than three View Answer

Option C Solution: R and M 5. Four of the five are alike in a certain way based on arrangement, which does not belong to the group? A) 4T& B) 51R C) *SJ D) 29M E) OV# View Answer

Option D Directions (1-5): Study the following arrangement and answer questions given: J % K W ! 2

7 D

E $

4 9 H I 3 ( @ A M 8 ^ F Q

O U +

& B P

1. How many such vowels are there in the above arrangement, each of which is immediately followed by a symbol? A) Four IBPS SBI RBI SSC RRB FCI RAILWAYS SSB | Governmentadda.com

19

Daily Visit:

[GOVERNMENTADDA.COM]

B) Three C) Two D) Five E) None View Answer

Option C Explanation: E, U 2. If all the symbols are dropped from the above arrangement, how many such numbers will be there in the above arrangement, each of which is immediately followed by a consonant? A) Four B) Three C) Two D) Five E) None View Answer

Option B Explanation: 7, 9, 8 3. What will come in place of questions mark (?) based on the above arrangement? DK$ E74 $D9 4EH ? A) 94H B) Other than those given in options C) H4I D) H43 E) 9$I View Answer

Option E 4. Four of the following five are alike in a certain way based on their positions in the above arrangement and so form a group. Which is the one that does not belong to that group? A) K%$ B) H$( C) U^+ D) O^+ E) P+!

IBPS SBI RBI SSC RRB FCI RAILWAYS SSB | Governmentadda.com

20

[GOVERNMENTADDA.COM]

Daily Visit: View Answer

Option E Explanation: All options carry a letter followed by 2 symbols. Observe that the first symbol is the symbol which is before the letter and second symbol after letter as: % K 7 D E $ $ 4 9 H I 3 ( ^ F Q O U + ^ F Q O U + & B P W ! 2 So with P should be P&! 5. Which of the following is the 6th to the left of the 17th from the left end of the above arrangement? A) 3 B) ( C) H D) I E) 9 View Answer

Option D Explanation: Left of left – 17-6 = 11th from left = I Directions (6-10): Study the following arrangement and answer questions given: R 4 % ! Y

D )

A 5

T

I

3

#

1

W

E

2

@

*

9

^

B

H

O

6. How many such numbers are there in the above arrangement, each of which is immediately preceded by a consonant and immediately followed by a vowel? A) Four B) Three C) Two D) Five E) None View Answer

Option E

IBPS SBI RBI SSC RRB FCI RAILWAYS SSB | Governmentadda.com

21

5

Daily Visit:

[GOVERNMENTADDA.COM]

7. How many such consonants are there in the above arrangement, each of which is immediately preceded by a number and immediately followed by a letter? A) Four B) Three C) Two D) Five E) None View Answer

Option C Explanation: T, W 8. Which of the following is the 5th to the right of the 19th from the right end of the above arrangement? A) 3 B) # C) W D) 1 E) None of these View Answer

Option D Explanation: Right to right = 19-5=14th from right = 1 9. Four of the following five are alike in a certain way based on their positions in the above arrangement and so form a group. Which is the one that does not belong to that group? A) D)4 B) 5T) C) 1W3 D) @*2 E) HO^ View Answer

Option D 10. Which of the following is the 8th to the left of the 9th from the right end of the above arrangement? A) 3 B) T C) I D) )

IBPS SBI RBI SSC RRB FCI RAILWAYS SSB | Governmentadda.com

22

Daily Visit:

[GOVERNMENTADDA.COM]

E) A View Answer

Option C Explanation: Left of right = 9+8=17th from right Directions (1-5): Study the following arrangement and answer questions given: 2 D I 4 $ K J 7 % L A T 3 8 @ F 6 U # V P 1 E * 9 H 5 1. How many such vowels are there in the above arrangement, each of which is immediately followed by a symbol? A) Four B) Three C) Two D) Five E) None View Answer

Option C Explanation: U, E 2. If all the numbers are dropped from the above arrangement, which of the following will be the 10th from the right end of above arrangement? A) T B) @ C) # D) I E) A View Answer

Option A 3. What will come in place of questions mark (?) based on the above arrangement? D4$ K7% LT3 ? A) F6# B) Other than those given in options C) 8@6 D) 8F6 E) FU# View Answer

Option D

IBPS SBI RBI SSC RRB FCI RAILWAYS SSB | Governmentadda.com

23

Daily Visit:

[GOVERNMENTADDA.COM]

4. Four of the following five are alike in a certain way based on their positions in the above arrangement and so form a group. Which is the one that does not belong to that group? A) J%7 B) D4I C) T38 D) E9* E) FU6 View Answer

Option C 5. Which of the following is the 12th to the right of the 6th from the left end of the above arrangement? A) 1 B) # C) 6 D) U E) L View Answer

Option D Directions (6-10): Study the following arrangement and answer questions given: R D A K 5 B I 2 M J E N 9 7 U Z V 1 W 3 H

4 F Y 8 P 6 T G

6. How many such numbers are there in the above arrangement, each of which is immediately preceded by a consonant and immediately followed by a vowel? A) Four B) Three C) Two D) Five E) None View Answer

Option E 7. How many such consonants are there in the above arrangement, each of which is immediately preceded by a number and immediately followed by a consonant? A) Four B) Three C) Two D) Five E) None IBPS SBI RBI SSC RRB FCI RAILWAYS SSB | Governmentadda.com

24

Daily Visit:

[GOVERNMENTADDA.COM]

View Answer

Option B Explanation: M, F, T 8. Which of the following is the 6th to the right of the 19th from the right end of the above arrangement? A) 5 B) Z C) V D) 1 E) None of these View Answer

Option C 9. Four of the following five are alike in a certain way based on their positions in the above arrangement and so form a group. Which is the one that does not belong to that group? A) E9J B) Z1U C) HW4 D) Y48 E) B2K View Answer

Option E 10. Which of the following is the 8th to the left of the 17th from the left end of the above arrangement? A) 8 B) J C) M D) 5 E) None of these View Answer

Option C Directions (1-5): In each question below is given a group of digits followed by four combinations of digits/symbols given in (A), (B), (C) and (D). You have to find out which of the combinations correctly represents the group of digits based on the rules and mark the answer

IBPS SBI RBI SSC RRB FCI RAILWAYS SSB | Governmentadda.com

25

Daily Visit:

[GOVERNMENTADDA.COM]

accordingly. If none of the combinations correctly represents the group of letters, mark (E) as your answer. (i) If the first digit is odd and the last digit is even their codes are to be swapped (ii) If the first as well as the last digits are even, both are to be coded by the code for the first digit. (iii) If the first digit is even and the last digit is odd, both are top be coded by the code for the odd digit.

1. 431068 A) %FT@$# B) %FT$@# C) %FT@$% D) %FT@$# E) None of these View Answer

Option C Solution: From rule (iii) 2. 803279 A) E@FLWE B) #@FLWE C) #@FLW# D) E@FLW# E) None of these View Answer

Option A Solution: No rule is implied here. So all codes written as it is 3. 384695 A) F#%$EF B) F#%$EQ C) Q#%$EQ D) Q#%$EF E) None of these IBPS SBI RBI SSC RRB FCI RAILWAYS SSB | Governmentadda.com

26

Daily Visit:

[GOVERNMENTADDA.COM]

View Answer

Option B Solution: From rule (i) 4. 584632 A) Q#%$FL B) L#%$FQ C) L#%$QF D) L#%$FQ E) None of these View Answer

Option D Solution: No rule is implied here. So all codes written as it is BAEFNP becomes — !@46%$ 5. 765984 A) W$QE#% B) W$QE#W C) %$QE#% D) %$QE#L E) None of these View Answer

Option E Solution: From rule (ii) Directions (6-10): Study the following arrangement and answer questions given: Q 4 N S F # K 7 Y $ C 8 3 P I T 6 % 9 H U 2 R J * V 5 W A 1 B D G 6. How many such consonants are there in the above arrangement, each of which is immediately preceded by either a symbol or number but nit immediately followed by a letter? A) Four B) Three C) Two D) Five E) None View Answer

IBPS SBI RBI SSC RRB FCI RAILWAYS SSB | Governmentadda.com

27

Daily Visit:

[GOVERNMENTADDA.COM]

Option A Explanation: K, Y C, V 7. If all the numbers are dropped from the above arrangement, which of the following will be the 12th from the right end of aboev arrangement? A) T B) H C) % D) I E) * View Answer

Option C 8. What will come in place of questions mark (?) based on the above arrangement? 1AG *JW H9R IP% ? A) Y$# B) Other than those given in options C) 7KC D) $J8 E) $Y3 View Answer

Option E 9. Four of the following five are alike in a certain way based on their positions in the above arrangement and so form a group. Which is the one that does not belong to that group? A) KY# B) %H6 C) J*R D) NF4 E) 1DA View Answer

Option C 10. Which of the following is the 7th to the right of the 15th from the left end of the above arrangement? A) H B) R C) B IBPS SBI RBI SSC RRB FCI RAILWAYS SSB | Governmentadda.com

28

Daily Visit:

[GOVERNMENTADDA.COM]

D) U E) None of these View Answer

Option E Directions (1-5): Study the following arrangement and answer questions given: BL$4#GA8PXY7%βN*S2@DM=9*5CF6 1. How many such numbers are there which are immediately followed by a symbol and immediately preceded by letter? A) Four B) Two C) None D) One E) Three View Answer

Option B Solution: Y 7 %, S 2 @ 2. If all the symbols are dropped from the above arrangement, then which is ninth from the right end? A) N B) 2 C) 7 D) S E) M View Answer

Option D Solution: The series become: BL4GA8PXY7NS2DM95CF6 3. If all the numbers immediately after symbols are added, what will be the sum? A) 26 B) 20 C) 15 D) 22 E) 18 IBPS SBI RBI SSC RRB FCI RAILWAYS SSB | Governmentadda.com

29

Daily Visit:

[GOVERNMENTADDA.COM]

View Answer

Option E Solution: 4 + 9 + 5 = 18 4. If every fourth element in the above series from the left end is omitted then which among the following is fourth to the right of ninth from the right end? A) 5 B) 9 C) M D) = E) @ View Answer

Option D Solution: BL$#GAPXY%βNS2@M=95CF 5. How many such vowels are there which is immediately followed by consonants and immediately preceded by number? A) Four B) Two C) None D) One E) Three View Answer

Option C

Directions (6-10): Study the following arrangement and answer questions given: E3*LJ8#R14%SAδ6K9M$X7βBZ@2L 6. How many such symbols are there which is immediately followed by number and immediately followed by letter? A) Four B) Two C) None D) One E) Three View Answer

IBPS SBI RBI SSC RRB FCI RAILWAYS SSB | Governmentadda.com

30

Daily Visit:

[GOVERNMENTADDA.COM]

Option B Solution: A δ 6 and Z @ 2 7. How many such numbers are there in the above arrangement each of which is immediately preceded by a symbol and immediately followed by a letter ? A) None B) One C) Two D) Three E) More than three View Answer

Option C Solution: δ 6 K, @ 2 L 8. Complete the series: K9 X A) %AX B) 4%A C) SA% D) 4%δ E) None of these

δ6M SAK ?

View Answer

Option D

9. If all the numbers are dropped from the above sequence, then which element is eleventh from the right end? A) M B) % C) S D) δ E) A View Answer

Option E Solution: The series becomes: E*LJ#R%SAδKM$XβBZ@L

IBPS SBI RBI SSC RRB FCI RAILWAYS SSB | Governmentadda.com

31

Daily Visit:

[GOVERNMENTADDA.COM]

10. Four of the five are alike in a certain way based on arrangement, which does not belong to the group? A) LJ* B) SA% C) K96 D) M9$ E) BZβ View Answer

Option D

IBPS SBI RBI SSC RRB FCI RAILWAYS SSB | Governmentadda.com

32

[1]

SYLLOGISM - 300 Directions (Q. 1-5): In each of the questions below are given two/three statements followed by two conclusions numbered I and II. You have to take the given statements to be true even if they seem to be at variance with commonly known facts. Read all the conclusions and then decide which of the given conclusions logically follows from the given statements, disregarding commonly known facts. Give answer if only conclusion I follows. if only conclusion II follows. if either conclusion I or conclusion II follows. if neither conclusion I nor conclusion II foUows. if both conclusion I and conclusion II follow. Statements: All blades are sharp. Some blades are blunt. No blunt is a sword. Conclusions: I. All sword being blades is a possibility. At least some blades may be swords. Statements: All flowers are white. Some whites are beautiful. Conclusions: I. All flowers being beautiful is a possibility. At least some white may not be flowers. Statements: All fruits are tasty. No tasty is good. All goods are yummy. Conclusions: I. All yummy things being tasty is a possibility. All fruits being yummy is a possibility. Statements: All mangoes are grapes. Some grapes are black. Conclusions: I. Some mangoes being black is a possibility.

GovernmentAdda.com There is a possibility that some mangoes are not black.

Statements: No book is a pencil. All pencils are pens. No pen is blue. Conclusions: I. No blue is a pencil. II. All books being pen is a possibility. Directions (Q. 6-10): In each question below are given two or three statements followed by two conclusions numbered I and II. You have to take the given statements to be true even if they seem to be at variance with commonly known facts. Read all the conclusions and decide which of the given conclusions logically follows from the given statements, disregarding commonly known facts. Give answer if only conclusion I follows. if if if if

only conclusion II follows. either conclusion I or conclusion II follows. neither conclusion I nor conclusion II follows. both conclusion I and conclusion II follow.

Statements: Some tigers are panthers. No lion is a tiger. Conclusions I. Some panthers being lions is a possibility. Panthers, which are not tigers, being lions is a possibility. Statements: All doctors are perfect. All engineers are perfect. Conclusions I. There is a possibility that some doctors are engineers

[2] All perfects are either doctors or engineers. Statements: All eggs are tomatoes. No tomato is a potato. All potatoes are goods. Conclusions I.

Some tomatoes may be goods.

All eggs being goods is a possibility. Statements: Some dogs are cats. All animals are cats. All cats are whites. Conclusions I. All those dogs which are cats are also whites. All the animals may or may not be dogs. Statements: Some golds are silver. All silvers are white. No white is a yellow. Conclusions I. Some golds which are white are not yellow. II. Some golds being yellow is a possibility. Directions (Q. 11 - 15): In each question below are given three statements followed by two conclusions numbered I and II. You have to take the given statements to be true even if they seem to be at variance with commonly known facts. Read all the conclusions and then decide which of the given conclusions logically follows from the given statements, disregarding commonly known facts. Give answer if only conclusion I follows. if only conclusion II follows. if either conclusion I or conclusion II follows. if neither conclusion I nor conclusion II follows. if both conclusion I and conclusion II follow. (11-12): Statements: All stars are moons. All moons are planets. All planets are round. Conclusions: I. All moons being stars is a possibility. All stars are round. Conclusions: I. At least some planets are stars. All stars if they are moon will be planet. (13-14): Statements: All shirts are pants. No pant is trousers. Some jackets are pants.

GovernmentAdda.com

Conclusions: I. All shirts being jackets is a possibility. No trousers are shirt. Conclusions: I. There is a possibility that all shirts are trousers. All pants being jackets is a possibility. Statements: Some cakes are pastries. Some breads are pastries. Conclusions: I. All pastries being cakes is a possibility. There is a possibility that some cakes are breads. Directions (Q. 16-20): In each of the questions below are given two or three statements followed by two conclusions numbered I and II. You have to take the given statements to be true even if they seem to be at variance with commonly known facts and then decide which of the given conclusions logically follows from the given statements, disregarding commonly known facts. Give answer if only conclusion I follows. if only conclusion II follows. if either conclusion I or II follows. if neither conclusion I nor II follows. if both conclusions I and II follow. (16-20): 16.

Statements: All princes are kings. All braves are princes. Conclusions: I. All princes being brave is a possibility.

No queen is a king.

[3] At least some kings are brave. Conclusions: I. A prince can never be a queen. All those princes who are kings are queens. (18-20): Statements: All squares are circles. All circles are triangles. Some rectangles are triangles. Conclusions: I. All circles being rectangles is a possibility. Some triangles are squares. Conclusions: I. All squares being rectangle is a possibility. At least some rectangles are circles. Statements: Some flowers are white. No flower is black. Conclusions: I. No black is white. All whites being flowers is a possibility. Directions (Q.21-25) In each question below are two statements followed by two conclusions numbered I and II. You have to take the two given statements to be true even.if they seem to be at variance from commonly known facts and then decide which of the given conclusions logically follows from the given statements disregarding commonly known facts. Give answer (1) if only conclusion I follows. Give’answer (2) if only conclusion II follows. Give answer (3) if either conclusion I or II follows. Give answer (4) if neither conclusion I nor II follows. Give answer (5) if both conclusions I and II follow Statements : All graphics are designs. All patterns are designs. Conclusions : I. Atleast some designs are graphics. No pattern is a graphic. Statements : All leaves are roots. Some leaves are branches. Conclusions : I. Some branches are not leaves. Atleast some roots are branches. Statements : All toys are gifts. No toy is a reward. Conclusions : I. Atleast some rewards are gifts. All gifts are toys. Statements : Some kings are emperors. No ruler is a king. Conclusions : I. No ruler is an emperor. Atleast some rulers are emperors. Statements : No wish is a demand. All wishes are claims. Conclusions : I. No demand is claim. All claims are wishes. Directions (Q. 26-30): In each question below are given two/three statements followed by two conclusions numbered I and II. You have to take the given statements to be true even if they seem to be at variance with commonly known facts and then decide which of the two conclusions logically follows from the given statements disregarding commonly known facts. Give answer if only conclusion I follows.

GovernmentAdda.com

if if if if

only conclusion II follows. either conclusion I or II follows. neither conclusion I nor II follows. both conclusions I and II follow.

(26-27): Statements:

All bags are purses. No purse is black. All blacks are beautiful.

Conclusions: I. Some bags being black is a possibility. At least some purses are bags. Conclusions: I. All purses being beautiful is a possibility. Some bags are not black. (28-29): Statements: Some fishes are cats. Some dogs are cats. No fish is black. Conclusions: I. At least some cats are not black. There is a possibility that some fishes are dogs.

[4] Conclusions: I. No dog is black. Some cats are black. Statements: No rose is red. No flower is a rose. Conclusions: I. At least some flowers are red. All red are flowers. Directions (Q. 31-35): In each question below are given two or three statements followed by two conclusions numbered I and II. You have to take the given statements to be true even if they seem to be at variance with commonly known facts. Read the following statements and then decide which of the given conclusions logically follows from the given statements, disregarding commonly known facts. Give answer if if if if if

only conclusion I follows. only conclusion II follows. either conclusion I or II follows. neither conclusion I nor II follows. both conclusions I and II follow.

(31-32): All blacks are dogs. Statements: Some cats are black. Conclusions: I. No black is an animal. Some cats which are dogs are not animals. Conclusions: I. Some blacks being animals is a possibility. All dogs being cats is a possibility. (33-34): Statements: Some roses are red.

All flowers are red.

No dog is an animal.

All reds are beautiful.

Conclusions: I. All those roses which are red are also beautiful. Some flowers are beautiful. Conclusions: I. There is a possibility that some roses are flowers. All flowers are roses.

GovernmentAdda.com

35. Statements: No river is a lake. No lake is a sea. Conclusions: I. No river is an ocean. II. At least some lakes are oceans.

Some seas are oceans.

Directions (Q. 36-40): In each question below are two/three statements followed by two conclusions numbered I and II. You have to take the two/three given statements to be true even if they seem to be at variance from commonly known facts and then decide which of the given conclusions logically follows from the given statements disregarding commonly known facts. Give answer (1) if only conclusion I follows. Give answer (2) if only conclusion II follows. Give answer (3) if either conclusion I or conclusion II follows. Give answer (4) if neither conclusion I nor conclusion II follows. Give answer (5) if both conclusion I and conclusion II follow. (36-40). Statements Some colours are paints. All colours are varnishes. No varnish is dye. Conclusions: I. No paint-is dye. All paints Being varnishes is a possibility. Conclusions: I. Some varnishes are paints. No dye is colour. (38-39). Statements: All squares are triangles. No triangle is circle. All circles are rectangles. Conclusions: I. No rectangle is square. All rectangles being square is a possibility. Conclusions: I. No square is circle. At least some circles are square. Statements: No paper is book. Some books are libraries. Conclusions: I. All libraries being books is a possibility. II. No library is paper.

[5] Directions (Q. 41-45): In each question below are given two or three statements followed by two conclusions numbered I and II. You have to take the given statements to be true even if they seem to be at variance with commonly known facts. Read the following statements and then decide which of the given conclusions logically follows from the given statements, disregarding commonly known facts. Give answer if only conclusion I follows. if if if if

only conclusion II follows. either conclusion I or II follows. neither conclusion I nor II follows. both conclusions I and II follow.

(41-42): Statements: All boys are doctors. All doctors are usually not intelligent. Some intelligent are smart. Conclusions: I. All boys being intelligent is a possibility. At least some smarts are doctors. Conclusions: I. No doctor is smart. Some boys may be intelligent. (43-44): Statements: Some ice are candies. No candy is tasty. Conclusions: I. Some ice can never be yummy. No ice is tasty.

All tasty things are yummy.

Conclusions: I. Some ice being yummy is a possibility. Some yummy things are not candy. 45. Statements: No apple is red. All apples are tasty. Conclusions: I. Some red things being tasty is a possibility II. All tasty things are not red. Directions (Q. 46-50): In each question below are given three statements followed by two conclusions numbered I and II. You have to take the given two statements to be true even if they seem to be at variance with commonly known facts and then decide which of the given conclusions logically follows from the statements, disregarding commonly known facts. Give answer if only conclusion I follows. if only conclusion II follows.

GovernmentAdda.com if either conclusion I or II follows. if neither conclusion I nor II follows.

46.

47.

48.

49.

if both conclusions I and II follow. Statements: Some bananas are soaps.

Some powders are soaps.

Conclusions: I. Some soaps are detergents. II. Some detergents are not soaps. Statements: Some mobiles are pots. Some pots are animals. Conclusions: I. Some pots are cows. II. Some mobiles are cows. Statements: Some papers are bottles. No perfume is a paper. Conclusions: I. Some bottles are not perfumes. II. Some perfumes are not files. Statements: All books are magazines. All magazines are ice.

Only detergents are powders.

Some animals are cows.

All bottles are files.

Some ice are drops.

Conclusions: I. There is a possibility that all drops are books. II. At least some ice are books. 50.

Statements: Some balls are tables. Conclusions: I. Some chairs are balls.

Some tables are ipods.

All ipods are chalks.

II. Some balls are not chairs. Directions (Q. 51-55): In each question below are given two or three statements followed by two conclusions numbered I and II. You have to take the given two statements to be true even if they seem

[6] to be at variance with commonly known facts. Read all the statements and then decide which of the given conclusions logically follows from the statements, disregarding commonly known facts. Give answer if only conclusion I follows. if only conclusion II follows. if either conclusion I or II follows. if neither conclusion I nor II follows. if both conclusions I and II follow. (51-52): Some diamonds are precious. Some glasses are precious. Statements: All stones are diamonds. Conclusions: I. Some stones being precious is a possibility. All stones are precious. Conclusions: I. Some diamonds can never be stones. At least some glasses are diamonds. (53-54): Statements: No chocolate is tasty. Every chocolate is brown. Some browns are good. Conclusions: I. All chocolates are possibly good. Some chocolates are good. Conclusions: I. No brown is tasty. There is a possibility that some chocolates are not good. 55. Statements: All pencils are papers. All papers are black. Conclusions: I. All black things being pencils is a possibility. II. Some papers are not pencils. Directions (Q. 56-60): In each question below are two statements followed by two conclusions numbered I and n. You have to take the two given statements to be true even if they seem to be at variance with commonly known facts. Read all the conditions and then decide which of the given conclusions logically follows from the given statements, disregarding commonly known facts and give answer if only conclusion I follows.

GovernmentAdda.com

56.

57.

if only conclusion II follows. if either conclusion I or II follows. if neither conclusion I nor II follows. if both conclusions I and II follow. Statements: Some trees are bushes. All flowers are bushes. Conclusions: I. At least some bushes are trees. II. At least some flowers are trees. Statements: All colours are paints.

No paint is a brush.

Conclusions: I. At least some brushes are colours. 58.

59.

II. No brush is a colour. Statements: Some chemicals are organics. All organics are fertilisers. Conclusions: I. At least some fertilizers are chemicals. II. All fertilizers are organics. Statements: No air is solid. Some solids are liquids. Conclusions: I.

No liquid is air.

II. Some air is definitely not liquid. 60. Statements: All gems are diamonds. All diamonds are rocks. Conclusions: I. At least some rocks are gems. II. All gems are rocks. Directions (Q. 61-65): In each question below are given two or three statements followed by two conclusions numbered I and II. You have to take the given statements to be true even if they seem to be at variance with commonly known facts and then decide which of the given conclusions logically follows from the given statements, disregarding commonly known facts. Give answer if only conclusion I follows if only conclusion II follows. if either conclusion I or II follows.

[7] if neither conclusion I nor II follows. if, both conclusions I and II follow. (61-62): Statements: Some colleges are universities

All colleges are schools.

All schools are hostels.

Conclusions: I. Some colleges are not hostels. All schools being universities is a possibility. Conclusions: I. All universities can never be hostels. Any university which is a college is a hostel. (63-64): Statements: All pens are pencils.

No pencil is a paper.

All papers are notebooks.

Conclusions: I. All pens being notebooks is a possibility. All notebooks being pencils is a possibility. Conclusions: I. All notebooks if they are pencils are also pens. No pen is a paper. 65. Statements: Some bikes are motors. All machines are bikes. Conclusions: I. All machines are motors. II. Some machines being motors is a possibility. Directions (Q. 66-70): In each question below are given two/three statements followed by two conclusions numbered I and II. You have to take the given statements to be true even if they seem to be at variance with commonly known facts and then decide which of the given conclusions logically follows from the given statements, disregarding commonly known facts. Give answer if only conclusion I follows. if only conclusion II follows. if either conclusion I or II follows. if neither conclusion I nor II follows. if both the conclusions I and II follow.

GovernmentAdda.com

(66-67):

Statements: All tables are chairs.

No chair is a cooler.

All coolers are fans.

Conclusions: I. No fan is a table.

All chairs being fans is a possibility. Conclusions: I. No table is a cooler. All tables being fans is a possibility. 68.

Statements: Some erasers are pens.

All boxes are pens.

Conclusions: I. All erasers being boxes is a possibility. II. No box is an eraser. (69-70) Statements: No weak is strong.

Some strong are winners.

All winners are modest.

Conclusions: I. No modest is strong. All modest being strong is a possibility. Conclusions: I. All weak being modest is a possibility. No weak is a winner. Directions (Q. 71-75): In each question below are given two or three statements followed by two conclusions numbered I and II. You have to take the given statements to be true even if they seem to be at variance with commonly known facts and then decide which of the given conclusions logically follows from the given statements, disregarding commonly known facts. Give answer if only conclusion I follows. if only conclusion II follows. if either conclusion I or II follows. if neither conclusion I nor II follows. if both conclusions I and II follow.

[8] (71-72): Statements: All silver is gold. All gold is metal. Conclusions: I. All metals being silver is a possibility. At least some silver is precious. Conclusions: I. Some gold is metal as well as precious. All gold is precious. (73-74): Statements: All plants are trees. No tree is a tole. Conclusions: I. Some toles being plants is a possibility. No tole is a plant. Conclusions: I. All plants being green is a possibility. No green is a tree. 75.

All metals are precious.

All toles are green.

Statements: Some kites are papers. Some papers are black. Conclusions: I. Some kites being black is a possibility. II. All papers are not necessarily kites.

Directions (Q. 76-80): In each question below are given two or three statements followed by two conclusions numbered I and II. You have to take the given statements to be true even if they seem to be at variance with commonly known facts and then decide which of the given conclusions logically follows from the given statements, disregarding commonly known facts. Give answer if only conclusion I follows. if only conclusion II follows. if either conclusion I or II follows. if neither conclusion I nor II follows. if both conclusions I and II follow. (76-77) : Some roses are flowers. Statements: Some flowers are red. Conclusions: I. All those flowers which are roses are red.

GovernmentAdda.com

No rose is red. Conclusions: I. All roses being red is a possibility. Some flowers can never be roses. (78-79): Statements: All princesses are queens. All girls are princesses. Conclusions: I. Some princesses being women is a possibility.

No woman is a queen.

At least some queens are girls. Conclusions: I. No queen is a girl. All princesses being girls is a possibility. Statements: Some hills are mountains. All mountains are high. Conclusions: I. All hills being high is a possibility. Some mountains can never be hills. Directions (Q.81-85): In each of the questions below are given three statements followed by two conclusions numbered I and II. You have to take the given statements to be true even if they seem to be at variance with commonly known facts. Read all the conclusions and then decide which of the given conclusions logically follows from the given statements, disregarding commonly known facts. Give answer (1) (2) (3) (4) (5) 81.

if if if if if

only conclusion I follows. only conclusion II follows. either conclusion I or conclusion II follows. neither conclusion I nor conclusion II follow. both conclusions I and II follow

Statements: Some apartments are huts. All buildings are cottages. Conclusions: I.

All apartments are buildings.

Some apartments are not cottages.

II. All buildings being huts is a possibility.

82.

83.

Statements: All staplers are printers. No printer is a computer. All computers are machines. Conclusions: I. All staplers being machines is a possiblity. II. No stapler is a computer. Statements: Some fans are moons. All planets are fans. Conclusions: I.

No planet is a universe.

Some moons, if they are planets, are also fans.

II. All universes being fans is a possibility. Statements: All professors are scientists. Some professors are intelligent. No intelligent is honest. Conclusions: I. At least some professors being honest is a possibility. 85.

II. Statements: No Conclusions: I. II.

All professors, if they are intelligent, are also honest pin is a tin. All tins are copper. All metals being tins is a possibility. Some pins are not copper.

No copper is a metal.

Directions (Q. 86-90): In each of the questions below are given three statements followed by two conclusions numbered I and II. You have to take the given statements to be true even if they seem to be at variance with commonly known facts. Read all the conclusions and then decide which of the given conclusions logically follows from the given statements, disregarding commonly known facts. Give answer if only conclusion I follows. if only conclusion II follows. if only conclusion I or II follows. if neither conclusion I nor II follows. if both conclusions I and II follow. (86-87): Statements: Some balls are locks.

All locks are keys.

Some doors are keys.

Conclusions: I. At least some balls are doors. All doors being locks is a possibility. Conclusions: I. All doors being keys is a possibility. Some balls are keys.

GovernmentAdda.com

(88-89): Statements: All lizards are birds. Some ants are birds. Conclusions: I. All cobras being lizards is a possiblity. At least some lizards are ants.

All ants are cobras.

Conclusions: I. All lizards being cobras is a possibility. Some cobras are birds. No debenture is an equity. Statements: Some shares are debentures. Some equities are capital. Conclusions: I. All debentures being equities is a possibility. II. All shares being equities is a possibility.

(90):

Directions (Q. 91-97): In each question below are given three statements followed by two conclusions numbered I and II. You have to take the given statements to be true even if they seem to be at variance with commonly known facts. Read all the conclusions and then decide which of the given conclusions logically follows from the given statements, disregarding commonly known facts. Give answer if only conclusion I follows. if only conclusion II follows. if either conclusion I or conclusion II follows. if neither conclusion I nor conclusion II follows. if both conclusion I and conclusion II follow. (91-92): Statements: No planet is a universe. All moons are planets. Some stars are moons. Conclusions: I. All moons being stars is a possibility. No universe is a star.

[10] Conclusions: I. At least some planets are stars. No moon is a universe. (93-94(: Statements: All insects are amphibians All sticks are plants. All plants are insects. Conclusions: I. At least some amphibians are plants. All sticks are insects. Conclusions: I. All amphibians are sticks. All plants are amphibians. (95-97): Statements: All apartments are huts. No hut is a building. All buildings are cottages. Conclusions: I. No cottage is an apartment. Some cottages are apartments. Conclusions: I. No apartment is a cottage. Some buildings being apartments is a possibility. Conclusions: I. Some cottages being apartments is a possibility. No cottage is a hut. Directions (Q. 98-100) : In each question below are two statements followed by two conclusions numbered I and II. You have to take the two given statements to be true even if they seem to be at variance with commonly known facts and then decide which of the given conclusions logically follows from the given statements, disregarding commonly known facts. Give answer if only conclusion I follows. if if if if

only conclusion II follows. either conclusion I or II follows. neither conclusion I nor II follows. both conclusions 1 and II follow.

Statements: All exams are tests. No test is a question. Conclusions: I. At least some exams are questions. No exam is a question.

GovernmentAdda.com

Statements: No bangle is an earring. Some earrings are rings. Conclusions: I. No ring is a bangle. Some rings are definitely not earrings.

Statements: Some banks are colleges. All colleges are schools. Conclusions: I. At least some banks are schools. All schools are colleges. Directions (Q. 101-107) : In each group of questions below are two/three statements followed by two conclusions numbered I and II. You have to take the given statements to be true even if they seem to be at variance with commonly known facts and then decide which of the given conclusions logically follows from the two/three statements, disregarding commonly known facts. Give answer if only conclusion I follows. if only conclusion II follows. if either conclusion I or conclusion II follows. if neither conclusion I nor conclusion II follows. if both conclusion I and conclusion II follow. Statements: All books are magazines. No magazine is a newspaper. Conclusions: I. No book is a newspaper. II. Some magazines are books. (102-103) : Statements: Some mobiles are telephones. All telephones are pagers. No pager is a camera. Conclusions: I. Some mobiles are definitely not cameras. No mobile is a camera. Conclusions: I. All telephones are cameras. All mobiles being telephones is a possibility. (104-105) : Statements: No cube is a cuboid.

Some cuboids are rectangles. All squares are cubes. Conclusions: I. No cuboid is a square. All cubes being rectangle is a possibility. Conclusions: I. No rectangle is a square. No cube is a rectangle. Statements: All fruits are vegetables. Some vegetables are pulses. Some pulses are not cereals. Conclusions: I. Some vegetables are not cereals. All vegetables being fruits is a possibility. Statements: All schools are colleges All schools are universities No university is a campus. Conclusions: I. All colleges are universities II. All schools being campus is a possibility. Directions (Q.108-114): In each group of questions below are two/three statements followed by two conclusions numbered I and II. You have to take the given statements to be true even if they seem to be at variance from commonly known facts and then decide which of the given conclusions logically follows from the two/three statements disregarding commonly known facts give answer. Give answer (1) if only conclusion I follows; Give answer (2) if only conclusion II follows; Give answer (3) if either conclusion I or conclusion II follows; Give answer (4) if neither conclusion I nor conclusion II follows; Give answer (5) if both conclusion I and conclusion II follow. 108. Statements: Some exams are tests. No exam is a question. Conclusions: I. No question is a test. II. Some tests are definitely not exams. (109-110) Statements: All forces are energies. All energies are powers. No power is heat. Conclusions: I. Some forces are definitely not powers. No heat is force. Conclusions : I. No energy is heat. Some forces being heat is a possibility. (111-112) Statements: No note is a coin. Some coins are metals. All plastics are notes. Conclusions: I. No coin is plastic. All plastics being metals is a possibility. Conclusions: I. No metal is plastic. All notes are plastics. Statements: Some symbols are figures. All symbols are graphics. No graphic is a picture. Conclusions: I. Some graphics are figures. No symbol is a picture. 114. Statements: All vacancies are jobs. Some jobs are occupations. Conclusions: I. All vacancies are occupations. II. All occupations being vacancies is a possibility. Direction (Q. 115-119) In each question of below are two/three statements followed by two conclusions numbered I and II. You have to take the two/three given statements to be true even if they seem to be at variance from commonly known facts and then decide which of the given conclusions logically follows from the given statements disregarding commonly known facts. Give answer (1) if only conclusion I follows. Give answer (2) if only conclusion II follows. Give answer (3) if either conclusion I or conclusion II follows. Give answer (4) if neither conclusion I nor conclusion II follows. Give answer (5) if both conclusion I and conclusion II follow. 115. Statements: All kites are birds. All aeroplanes are kites. No bird is a fish. Conclusions: I. No fish is a kite. II. All aeroplanes are birds. 116. Statements: Some wires are fires. All fires are tyres. Conclusions: I. Atleast some tyres are wires. II. Some fires are definitely not wires. Statements: No clip is a pin. All badges are pins. Conclusions: I. No badge is a clip.

GovernmentAdda.com

All pins are badges. Statements: No colour is a paint. No paint is a brush. Conclusions: I. No colour is a brush. All brushes are colours. Statements: All stars are planets. All planets are galaxies. Conclusions: I. All galaxies are planets. All stars are galaxies. Direction (Q.120-124). In each question below are two/three statements followed by two conclusions numbered I and II. You have to take the two/three given statements to be true even if they seem to be at variance from commonly known facts and then decide which of the given conclusions logically follows from the given statements disregarding commonly known facts. Give answer (1) if only conclusion I follows. Give answer (2) if only conclusion II follows. Give answer (3) if either conclusion I or conclusion II follows. Give answer (4) if neither conclusion I nor conclusion II follows. Give answer (5) if both conclusion I and conclusion II follows. 120. Statements: No toffee is coffee. No sweet is toffee. Conclusions: I. No coffee is sweet ; II. All sweets are coffee 121. Statements: All medals are awards. All rewards are medals. Conclusions: I. All rewards are awards II. All awards are medals 122. Statements: Some trees are plants. All bushes are plants. Conclusions: I. Atleast some trees are bushes II. Some trees are definitely not bushes 123. Statements: All bottles are glasses. No cup is a glass. Conclusions: I. No bottle is a cup II. Atleast some glasses are bottles 124. Statements: All windows are doors. All entrances are windows. No gate is a door. Conclusions: I. Atleast some windows are gates II. No gate is an entrance Directions (Q. 125-129) : In each question below are given two/three statements followed by two conclusions numbered I and II. You have to take the given statements to be true even if they seem to be at variance with commonly known facts. Read all the conclusions and then decide which of the given conclusions logically follows from the given statements, disregarding commonly known facts. Give answer: if only conclusion I follows. if only conclusion II follows. if either conclusion I or II follows. if neither conclusion I nor II follows. if both conclusions I and II follow. (125-126) : Some books are colleges. Statements: All notes are books. All books are pages. Conclusions: I. All books can never be colleges. II. Any book which is a note must be a page. 126. Conclusions: I. All books being notes is a possibility. II. No note is a college. (127-128) : Statements: No needle is a tree. Some trees are threads. All threads are boxes. Conclusions: I. No needle is a thread. No boxes are trees. Conclusions: I. All boxes being threads is a possibility. All trees being threads is a possibility., Statements: Some players are spectators. No player is a theatre. Conclusions: I. No spectators are theatres. All spectators being theatres is a possibility. Directions (Q. 130-134) : In each of the questions given below three statements are followed by two conclusions numbered I and II. You have to take the given statements to be true even if they seem to be at variance with commonly known facts. Read all the conclusions and then decide which of the given conclusions logically follows from the given statements, disregarding commonly known facts. Give answers:

GovernmentAdda.com

if only conclusion I follows. if only conclusion II follows. if either conclusion I or conclusion II follows. if neither conclusion I nor conclusion II follows. if both conclusion I and II follow. 130. Statements: Some reds are blues. All blues are pinks. Some pinks are not reds. Conclusions: I. All blues are reds. II. Some reds are not pinks is a possibility. 131. Statements: Some tables are chairs. All chairs are pens. All pens are copies. Conclusions: I. Some pens are tables. II. All copies being chairs is a possibility. 132. Statements: Some moons are stars. All stars are skies. No moon is a planet. Conclusions: I. Some moons are skies. II. No star is a planet. 133. Statements: All milk is cold drink. Some water is milk. Some water is not cold drink. Conclusions: I. All milk is water. II. All water being milk is a possibility. 134. Statements: All trees are flowers. All flowers are coins. Some coins are notes. Conclusions: I. All notes being trees is a possibility. II. Some coins are flowers. Directions (Q. 135-139) : In each question below are given two/three statements followed by two conclusions numbered I and II. You have to take the given two statements to be true even if they seem to be at variance with commonly known facts. Read all the conclusions and then decide which of the given conclusions logically follows from the given statements, disregarding commonly known facts. Give answer if only conclusion I follows. if only conclusion II follows. if either conclusion I or II follows. if neither conclusion I nor II follows. if both conclusions I and II follow. 135. Statements: All erasers are pens. All pens are boxes. No eraser is a pencil. Conclusions: I. No box is a pencil. Some pens being pencils is a possibility. Statements: Some tests are unpleasant. All unpleasant things need to be avoided. Conclusions: I. Some tests need to be avoided. All tests being unpleasant is a possibility. Statements: Some forests are dark and horrible. Dark and horrible places are filled with Freudian imagery. Conclusions: I. At least some horrible places are forests. All horrible forests being dark is a possibility. Statements: Some firefighters are fit. All firefighters are courageous. All courageous people are army men. Conclusions: I. All firefighters are army men. A11 courageous people can never be fit. Statements: No lawyers are enthusiastic. All enthusiastic people are positive. No positive people are honest. Conclusions: I. No honest people are enthusiastic. II. All positive people being lawyers is a possibility. Directions (Q. 140-144) : In each question below are given two/three statements followed by two conclusions numbered I and If, You have to take the given two statements to be true even if they seem to be at variance with commonly known facts. Read all the conclusions and then decide which of the given conclusions logically follows from the given statements, disregarding commonly known facts. Give answer if only conclusion I follows. if only conclusion II follows. if either conclusion I or II follows. if neither conclusion I nor II follows. if both conclusions I and II follow. Statements: Some cats are kittens. All dogs are kittens.

GovernmentAdda.com

No kittens are black. Conclusions: I. All kittens being cats is a possibility. Some dogs can never be black. Statements: 60% of the government teachers went on strike. Miss Rani is a government teacher. Conclusions: I. That Miss Rani went on strike is a possibility. Miss Rani did not participate in the strike. Statements: All scholars are eccentric. No woman is eccentric. All eccentrics are studies. Conclusions: I. No woman is a scholar. All studies being scholar is a possibility. Statements: Some eggs are hard-boiled. No eggs are uncrackable. Conclusions: I. Some hard-boiled are uncrackable. No hard-boiled are uncrackable. Statements: Some perfumes reek badly. All perfumes are expensive. All expensive things are unique. Conclusions: I. There is a possibility that all unique things are perfumes. II. Unique things can never reek badly. Directions (Q. 145-149) :In each of the question-sets below are two/three statements followed by two conclusions numbered I and II. You have to take the given statements to be true even if they seem to be at variance with commonly known facts and then decide which of the given conclusions logically follows from the given statements, disregarding commonly known facts. Give answer if only conclusion I follows. if only conclusion II follows, if either conclusion I or conclusion II follows. if neither conclusion I nor conclusion II follows. if both conclusion I and conclusion II follow. (145-147) : Statements: All stars are bottles. Some bottles are papers. No paper is a calendar. Conclusions: I. All stars being papers is a possibility. No calendar is a bottle. Conclusions: I. All calendars being stars is a possibility At least some bottles are stars. Conclusions: I. At least some calendars are bottles. No calendar is a star. (148-149) : Statements: Some pencils are blankets. All blankets are erasers. Conclusions: I. At least some pencils are erasers. All erasers being pencils is a possibility. Conclusions: I. No eraser is a pencil. All blankets being pencils is a possibility. Directions (Q. 150-154) : In each question below are given two/three statements followed by two conclusions numbered I and II. You have to take the given statements to be true even if they seem to be at variance with commonly known facts. Read all the conclusions and then decide which of the given conclusions logically follows from the given statements, disregarding commonly known facts. Give answer if only conclusion I follows. if only conclusion II follows. if either conclusion I or II follows. if neither conclusion I nor II follows. if both conclusions I and II follow. (150-151) :

GovernmentAdda.com

Statements: No copy is a book. Some books are magazines. All papers are copies. Conclusions: I. No copy is a magazine. No book is a paper. Conclusions: I. All copies being magazines is a possibility. No magazine is a paper. 152. Statements: All cars are motors. All cars are bikes. No bike is an auto. Conclusions: I. All motors are bikes. II. All cars being autos is a possibility. (153-154) : Statements: Some jobs are occupations. All jobs are vacancies. No vacancy is an unemployment. Conclusions: I. Some vacancies are occupations. No job is an unemployment. Conclusions: I. All unemployment being occupation is a possibility. That some occupations are not unemployment is a possibility. Directions (Q. 155-161) : In each question below are given two/three statements followed by two conclusions numbered I and II. You have to take the given two statements to be true even if they seem to be at variance with commonly known facts. Read all the conclusions and then decide which of the given conclusions logically follows from the given statements, disregarding commonly known facts. Give answer if only conclusion I follows. if only conclusion II follows. if either conclusion I or II follows. if neither conclusion I nor II follows.if both conclusions I and II follow. (155-156) : Statements: All stars are galaxies. No planet is sun. Some galaxies are suns. Conclusions: I. All sun being stars is a possibility. Some stars are galaxies. Conclusions: I. No galaxy is a planet. Some planets are galaxies (157-158) : Statements: No RNA is DNA. Some cells are tissues. All DNA are cells. Conclusions: I. All DNAs are tissues. Some cells are definitely not RNA. Conclusions: I. All RNAs being cells is a possibility. All cells being RNA is a possibility. Statements: Some stains are dirts. All dirts are shines. No stain is dull. Conclusions: I. All dull being dirts is a possibility. All shines are stains. (160-161) : Statements: Some coffee are cups. Some cups are not copies. All colds are coffee. Conclusions: I. Some coffee are not copies. All colds are cups. Conclusions: I. There is a possibility that no cups are coffee. There is a possibility that all coffee are cold. Directions (Q. 162-167) : In each of these questions, there are two conclusions I and II. Assume the statements to be true even if they vary with facts. Choose your answer as if only conclusion I follows. if only conclusion II follows. if either conclusion I or conclusion II follows. if neither conclusion I nor II follows. if both conclusions I and II follow (162-163) : Statements: Statements: Some weeks are years. All months are Weeks. All years are decades. No hour is a month. Conclusions: I. All hours being decades is a possibility. Some hours can never be years. Conclusions: I. All decades being weeks is a possibility. All hours, if they are months, cannot be decades.

GovernmentAdda.com

[16] (164-167) : Statements: All elections are dramas. All elections are stories Some dramas are not films. Conclusions: I. No film is a story. Some films which are dramas are also stories. Conclusions: I. All films being elections is a possibility. Some stories which are not elections are dramas. Statements: No spade is a club. All clubs are diamonds. All spades are hearts Conclusions: I. Some spades which are diamonds may also be clubs. Some hearts, if they are clubs, must be diamonds. Statements: All roads are drivers Some drivers are pilots Some walkers are not drivers. Conclusions: I. Some roads being pilots is a possibility. II. All walkers are pilots. Directions (Q. 168-172) : In each question/group of questions below are two/three statements followed by two conclusions numbered I and II. You have to take the two/ three given statements to be true even if they seem to be at variance with commonly known facts and then decide which of the given conclusions logically follows from the given statements disregarding commonly known facts. Give answer if only conclusion I follows. if only conclusion II follows. if either conclusion I or conclusion II follows. if neither conclusion I nor conclusion II follows. if both conclusion I and conclusion II follow. 168. Statements: No bank is a school. Some schools are colleges. Conclusions: I. Some colleges are definitely not schools. II. All banks being colleges is a possibility. 169. Statements: Some carts are trolleys. All baskets are trolleys. Conclusions: I. At least some baskets are carts. II. All trolleys are baskets. (170-171) : Statements: All fruits are vegetables. All vegetables are plants. No plant is a root. Conclusions: I. All fruits are plants. No root is a vegetable. Conclusions: I. No fruit is a root. At least some roots are vegetables. 172. Statements: Some calculators are phones. No phone is an eraser. Conclusions: I. No calculator is an eraser. II. Some calculators are definitely not phones. Directions (Q. 173-177) : In each question below are given two/three statements followed by two conclusions numbered I and II You have to take the given two statements to be true even if they seem to beat variance with commonly known facts. Read all the conclusions and then decide which of the given conclusions logically follows from the given statements, disregarding commonly known facts. Give answer if only conclusion I follows. if only conclusion II follows. if either conclusion I or II follows. if neither conclusion I nor II follows. if both conclusions I and II follow. (173-174) : Statements: No statement is a conclusion. Some conclusions are assumptions: All assumptions are arguments. Conclusions: I. No argument is a conclusion. No statement is an assumption. ; Conclusions: I. All arguments being assumptions is a possibility. All assumptions being conclusions is a possibility.

GovernmentAdda.com

[17] (175-176) : Statements: Some fabrics are cloths. All fabrics are garments. All plants are fabrics. Conclusions: I. All fabrics being plants is a possibility. No plant is a cloth. Conclusions: I. All fabrics can never be cloths. Any fabric which is a plant must be a garment. Statements: Some villages are towns. All towns are states. No state is a world. Conclusions: I. No town is a world. II. Some villages which are states are not worlds. Directions (Q. 178-182) : In each question below are given three statements followed by two conclusions numbered I and II. You have to take the given two statements to be true even if they seem to be at variance with commonly known facts. Read all the conclusions and then decide which of the given conclusions logically follows from the given statements, disregarding commonly known facts. Give answer if only conclusion I follows. if only conclusion II follows. if either conclusion I or II follows. if neither conclusion I nor II follows. if both conclusions I and II follow. Statements: All statements are conclusions. Some answers are conclusions. All conclusions are options. Conclusions: I. There is a possibility that some answers are statements. All statements are answers. Conclusions: I. Some options are statements. All those answers which are conclusions are also options. Statements: No sweet is namkeen. All coffees are sweets. Some drinks are coffees. Conclusions: I. No coffee is namkeen. Some drinks which are sweets are not namkeens. Conclusions: I. All sweets being drinks is a possibility. Some coffees being namkeen is a possibility. Statements: No eraser is a sharpener. Some scales are pencils. No sharpener is a scale. Conclusions: I. At least some sharpners are pencils. II. No eraser is a pencil. Directions (Q. 183-187) : In each question below are given two/three statements followed by two conclusions numbered I and II. You have to take the given two statements to be true even if they seem to be at variance with commonly known facts. Read all the conclusions and then decide which of the given conclusions logically follows from the given statements, disregarding commonly known facts. Give answer if only conclusion I follows.

GovernmentAdda.com

if if if if

only conclusion II follows. either conclusion I or II follows. neither conclusion I nor II follows. both conclusions I and II follow.

183. Statements:

All papers are books.

All books are files.

Conclusions: I. Some books being novels is a possibility. II. No file is a novel. 184. Statements: No key is a lock. All locks are buttons. Conclusions: I. No clock is a lock.

No paper is a novel.

No button is a clock.

[18] II. All buttons being

locks is a possibility.

185. Statements: Some clips are watches. All clips are telephones. Conclusions: I. All telephones can never be watches. II. All clips are pins. (186-187) : Statements: Some benches are trees. Some trees are flowers.

All telephones are pins.

All flowers are fruits.

Conclusions: I. Some fruits are benches. All trees being benches is a possibility. Conclusions: I. All fruits can never be trees. All flowers being benches is a possibility. Directions (Q. 188-192) : In each question below are given two/three statements followed by two conclusions numbered I and II. You have to take the given statements to be true even if they seem to be at variance with commonly known facts. Read all the conclusions and then decide which of the given conclusions logically follows from the given statements, disregarding commonly known facts. Give answer if only conclusion I follows. if only conclusion II follows. if either conclusion I or II follows. if neither conclusion I nor II follows. if both conclusions I and II follow. 188. Statements: No mango is a tomato. All tomatoes are potatoes. No potato is sugarcane. Conclusions: I. No mango is sugarcane. II. All potatoes being sugarcane is a possibility. 189. Statements: All mangoes are tangos. Some tangos are charlies. No charlie is a sweet. Conclusions: I. No sweet is a tango. II. Some tangos being sweets is a possibility. 190. Statements: No book is a paper. Some papers are pens. Conclusions: I. Some books being pens is a possibility. II. All pens being books is a possibility. 191. Statements: No book is a rough. No rough is a pen. Conclusions: I. No book is a pen. II. Some pens being books is a possibility. 192. Statements: All pens are columns. Some columns are inks. No ink is a copy. Conclusions: I. Some columns are copies. II. No copy is a column. Directions (Q. 193-197): In each question below are given three/four statements followed by two conclusions numbered I and II. You have to take the given statements to be true even if they seem to be at variance with commonly known facts. Read all the conclusions and then decide which of the given conclusions logically follows from the given statements, disregarding commonly known facts. Give answer 1) if only conclusion I follows. 2) if only conclusion II follows. 3) if either conclusion I or II follows. 4) if neither conclusion I nor II follows. 5) if both conclusions I and II follow. (193-194): Statements: Some balls are footballs. All bats are balls. All footballs are volleyballs. No fund is a bat. Conclusions: I. All volleyballs being balls is a possibility. All funds are balls. Conclusions: I. All funds being volleyballs is a possibility. Some funds can never be footballs. 195. Statements: No star is a sun. All suns are planets. All stars are moons. Conclusions: I. Some moons if they are suns must be planets. II. Some stars which are planets may also be suns. (196-197): Statements: Some stores are not products. All businesses are stores. All businesses are capital.

GovernmentAdda.com

[19] Conclusions: I. Some products which are capital are also stores. No product is a store. Conclusions: I. All products being businesses is a possibility. Some stores which are not business are capital. Directions (Q. 198-202) : In each question below are given three statements followed by two conclusions numbered I and II. You have to take the given two statements to be true even if they seem to be at variance with commonly known facts. Read all the conclusions and then decide which of the given conclusions logically follows from the given statements, disregarding commonly known facts. Give answer if only conclusion I follows. if only conclusion II follows. if either conclusion 1 or II follows. if neither conclusion I nor II follows. if both conclusions I and II follow. No paper is a marker. 198. Statements: All pens are markers. All copies are pens. Conclusions: I. At least some pens are papers. II. No paper is a copy. (199-200) : Statements: No chair is a table. Some tables are doors. All plastics are chairs. Conclusions: I. No table is a plastic. All chairs being doors is a possibility. Conclusions: I. No door is a plastic. No chair is a door. (201-202) : Statements: Some bags are laptops. All laptops are mobiles. No mobile is a page. Conclusions: I. Some bags are not pages. No bag is a page. Conclusions: I. All laptops are pages. All bags being laptops is a possibility. Directions (Q. 203-208) : In each question below are given two/three statements followed by two conclusions numbered I and II. You have to take the given statements to be true even if they seem to be at variance with commonly known facts. Read all the conclusions and then decide which of the given conclusions logically follows from the given statements, disregarding commonly known facts. Give answer if only conclusion I follows. if only conclusion II follows. if either conclusion I or II follows. if neither conclusion I nor II follows. if both conclusions I and II follow. (203-204) : All classes are Statements: No carrom is a corner. Some corridors are carroms. Conclusions: I. All corridors being corners is a possibility. All carroms being classes is a possibility. Conclusions: I. Some corners are not corridors. No class is a carrom. 205. Statements: Some oceans are lakes. Some ponds are rivers. All rivers are lakes. Conclusions: I. All oceans being rivers is a possibility. II. Some lakes are not ponds is a possibility. 206. Statements: All blue are brown. All black are bags. Some brown are not bags. Conclusions: I. Some brown are not black. II. No blue is a bag. 207. Statements: No train is a truck. Some trains are tumblers. All trucks are toys. Conclusions: I. All trucks being tumblers is a possibility. II. Some toys are not trains. 208. Statements: Some voyages are journeys. No journey is a trekking. Conclusions: I. No voyage is trekking. II. Some trekkings are voyages. Directions (Q. 209-214): In each question below are given two/three statements followed by two conclusions numbered I and II. You have to take the given statements to be true even if they seem

GovernmentAdda.com

[20] to be at variance with commonly known facts. Read all the conclusions and then decide which of the given conclusions logically follows from the given statements, disregarding commonly known facts. Give answer if only conclusion I follows.

209.

if only conclusion II follows. if either conclusion I or II follows. if neither conclusion I nor II follows. if both conclusions I and II follow. Statements: Statements: All boats are berries.

No brake is a belt.

All belts are boats.

Conclusions: I. No brake is a boat. Some berries are belts. Conclusions: I. All berries being belts is a possibility. All boats being brakes is a possibility. 211. Statements: Some teas are tasty. Some toasters are toys. Conclusions: I. Some toasters are tables. II. All tea being toasters is a possibility. 212.

213.

Statements: All coffee is cold.

Some tables are not toys.

Some cold are cups.

Conclusions: I. Some cups being coffee is a possibility. II. All cups being cold is a possibility. Statements: No rack is a rod. Some rats are red. Conclusions: I. Some red are not rods.

All red are racks.

All rats are racks. Statements: Some parrots are pigeons. Some pigeons are penguins. No parrot is pink. Conclusions: I. No pigeon is pink. Some pink are pigeons. Directions (Q. 215-220): In each of the questions given below three statements are followed by two conclusions numbered I and II. You have to take the given statements to be true even if they seem to be at variance with commonly known facts. Read all the conclusions and then decide which of the given conclusions logically follows from the given statements, disregarding commonly known facts. Give answer if only conclusion I follows. if only conclusion II follows. if either conclusion I or conclusion II follows. if neither conclusion I nor conclusion II follows. if both conclusions I and II follow. 215. Statements: All rights are wrong. Some rights are false. All false are true. Conclusions: I. Some false are wrong.

GovernmentAdda.com

II. All true being wrong is 216.

217.

218.

a possibility.

Statements: Some bags are baggages. No baggage is a basket.

All bags are buckets.

Conclusions: I. All baggages are buckets. II. Some bags are not baskets. Statements: All papers are plastics. No packet is a plant. All plastics are panels. Conclusions: I. Some panels are papers. II. All panels are plastics. Statements: Some glasses are not goggles. All glasses are gases. No glass is a goose.

Conclusions: I. Some gases are geese. II. Some gases are not goggles. (219-220): Statements: No toy is a tree.

Some toys are tough.

All trees are trains.

219. Conclusions: I. Some trains are toys. II. No train is a toy. Conclusions: I. All trees being tough is a possibility. All trains being toys is a possibility.

Directions (Q. 221-226): In each question below are given three/four statements followed by two conclusions numbered I and II. You have to take the given statements to be true even if they seem to be at variance with commonly known facts. Read all the conclusions and then decide which of the given conclusions logically follows from the given statements, disregarding commonly known facts. Give answer (1) if only conclusion I follows. (2) (3) (4) (5)

if if if if

only conclusion II follows. either conclusion I or II follows. neither conclusion I nor II follows. both conclusions I and II follow.

(221-223): Statements: All trousers are tracks. Some travels are not trains.

All tours are travels. No track is a travel.

Conclusions: I. No track is a tour. All tours are trains. Conclusions: I. Some trains are not tracks. No travel is a trouser. 223. Statements: No book is a ball. Some books are boys.

224.

Conclusions: I. All baskets being books is a possibility. II. All balls being boys is a possibility. Statements: Some foods are forests. Some floods are flutes. Conclusions: I. Some foods are not frozen is a possibility.

All boys are baskets.

All forests are frozen

II. Some frozen are forests. 225.

Statements: All pipes are piders. All pillars are paints. Conclusions: I. Some piders are not pyramids. II. All paints are pillars.

No pipe is a pyramid.

226.

Statements: All keys are kites. Some kites are carroms. Conclusions: I. Some carroms are keys. II. No kite is a colony.

No carrom is a colony.

GovernmentAdda.com

Directions (Q. 227-233): In each question below are given three statements followed by two conclusions numbered I and n. You have to take the given statements to be true even if they seem to be at variance with commonly known facts. Read all the conclusions and then decide which of the given conclusions logically follows from the given statements, disregarding commonly known facts. Give answer if only conclusion I follows. if only conclusion II follows. if either conclusion I or II follows. if neither conclusion I nor II follows. if both conclusions I and II follow. 227. Statements: All states are countries.

No country is a town.

Conclusions: I. No village is a state. II. Some states are not towns. 228. Statements: All ports are airports. Some airports are harbours. Conclusions: I. All docks being airports is a possibility. II. All ports are harbours. 229. Statements: All riddles are puzzles. All magics are trivia. Conclusions: I. II. 230. Statements: No Conclusions: I.

Some riddles being difficult is a possibility. All trivia being magic is a possibility. bucket is a boat. Some buckets are black. All blue being buckets is a possibility.

All boats being black is a possibility. Statements: Some pumpkins are potatoes. Some potatoes are papayas.

Some towns are villages.

Some harbours are docks.

No puzzle is difficult.

Some blue are boats.

All peanuts are pumpkins. Conclusions: I. Some papayas being pumpkins is a possibility. II. Some potatoes are not peanuts. (232-233) : Statements: No tree is a root. Some roots are plants.

All plants are saplings.

Conclusions: I. Some saplings are roots. Some plants are not trees. Conclusions: I. All plants being trees is a possibility. All saplings are plants. Directions (Q. 234-238) : In each question below are given two/three statements followed by two conclusions numbered I and II. You have to, take the given statements to be true even if they seem to be at variance with commonly known facts. Read all the conclusions and then decide which of the given conclusions logically follows from the given statements, disregarding commonly known facts. Give answer if only conclusion I follows. if only conclusion II follows. if either conclusion I or II follows. if neither conclusion I nor II follows. if both conclusions I and II follow. (234-235) : Statements: All books are novels.

No novel is a pen.

All pens are pencils.

Conclusions: I. No book is a pen. All books being pencils is a possibility. Conclusions: I. No pencil is a book. All novels being pencils is a possibility. 236. Statements: Some cats are dogs. All rats are dogs. Conclusions: I. All cats being rats is a possibility.

GovernmentAdda.com II. No rat is a cat.

(237-238) :

Statements: No boy is a postgraduate. Some postgraduates are girls. All girls are intelligent. Conclusions: I. All boys being intelligent is a possibility. No boy is a girl. Conclusions: I. No intelligent is a postgraduate. At least some intelligent are postgraduates. Directions (Q. 239-243): In each question below are given two/three statements followed by two conclusions numbered I and II. You have to take the given statements to be true even if they seem to be at variance with commonly known facts. Read all the conclusions and then decide which of the given conclusions logically follows from the given statements, disregarding commonly known facts. Give answer if only conclusion I follows. if only conclusion II follows. if either conclusion I or II follows. if neither conclusion I nor II follows. if both conclusions I and II follow. Statements: No guide is a software. Some softwares are antivirus. All applications are softwares. Conclusions: I. No application is a guide. II. Some guides are applications. 240. Statements: All wings are plastic. No metal is a wing. All metals are fibres. Conclusions: I. Some fibres are metals. II. Some fibres if they are wings must be plastic. 241. Statements: Some frames are circular. Some rectangles are frames. Conclusions: I. Some frames being circular is a possibility. II. Some rectangles are circular. 242. Statements: All seniors are experienced. All seniors are educated. Conclusions: I. Some experienced are educated.

II. Some seniors being not educated is a possibility. Statements: No logic is right. Some wrongs are right. No solution is wrong. Conclusions: I. No logic is a solution. II. No right is a solution. Directions (Q. 244-248) : In each of the questions below, two/three statements are given followed by conclusions/ group of conclusions numbered 1 and II. You have to assume all the statements to be true even if they seem to be at variance with the commonly known facts and then decide which of the given two conclusions logically follows from the information given in the statements. Give answer

243.

if only conclusion I follows. if only conclusion II follows. if either conclusion I or conclusion II follows. if neither conclusion I nor conclusion II follows. if both conclusion I and II follow. (244-245) : Statements: Some squares are circles. No circle is a triangle.

No line is a square.

Conclusions: I. All squares can never be triangles. Some lines are circles. Conclusions: I. No triangle is a square. No line is a circle. (246-247) : Statements: All songs are poems.

All poems are rhymes.

No rhyme is a paragraph.

Conclusions: I. No song is a paragraph. No poem is a paragraph. Conclusions: I. All rhymes are poems. 248.

All songs are rhymes. Statements: Some dews are drops. All drops are stones. Conclusions: I. At least some dews are stones. II. At least some stones are drops.

GovernmentAdda.com

Directions (Q. 249-253): In each group of questions below are given three statements followed by two conclusions numbered I and II. You have to take the given statements to be true even if they seem to be at variance with commonly known facts. Read all the conclusions and then decide which of the given conclusions logically follows from the given statements, disregarding commonly known facts. Give answer if only conclusion I follows.

249.

if only conclusion II follows. if either conclusion I or II follows. if neither conclusion I nor II follows. if both conclusions I and II follow. Statements: All books are magazines. Some notebooks are papers.

Some magazines are notebooks.

Conclusions: I. Some books are notebooks. 250.

II. Some magazines are papers. Statements: Some fruits are mangoes. Some mangoes are red.

All red are vegetables.

251.

Conclusions: I. No fruit is red. II. Some fruits are red; Statements: All windows are doors.

All doors are boats.

All buildings are doors.

Conclusions: I. All windows are boats. 252.

Statements: Conclusions:

253.

Statements: Conclusions:

II. All buildings are boats. Some sheets are roads. No sheet is a roll. I. Some sheets are marbles II. Some rolls are roads. 100% heads are caps. 29% caps are shoes. I. Some caps are pens.

All roads are marbles

99% shoes are pens.

II. All heads are shoes. Directions (Q. 254-258): In each of the questions given below three statements are followed by two conclusions numbered I and IL You have to take the given statements to be true even if they seem to be at variance with commonly known facts. Read all the conclusions and then-decide which of the given conclusions logically follows from the given statements, disregarding commonly known facts. Give answer: if only conclusion I follows. if only conclusion II follows. if either conclusion I or conclusion II follows. if neither conclusion I nor conclusion II follows. if both conclusions I and II follow. (254-255): Statements: Some ties are belts. All belts are shirts. Conclusions: I. All ties being shirts is a possibility. Some shirts are not belts. Conclusions: I. All T-shirts being ties is a possibility. No belt is a T-shirt. (256-257): Statements: All crows are birds. All birds are parrots. Conclusions: I. All crows are parrots. All parrots being sparrows is a possibility. Conclusions: I. Some parrots are not birds.

No shirt is a T-shirt.

Some birds are sparrows.

All sparrows are birds. 258. Statements: Some knives are pins. All pins are keys. No key is a lock. Conclusions: I. Some locks are knives. II. No lock is a knife. Directions (Q. 259-264): In each question below are given three statements followed by two conclusions numbered I and II. You have to take the given statements to be true even if they seem to be at variance with commonly known facts and then decide which of the given conclusions logically follows from the given statements, disregarding commonly known facts. Give answer if only conclusion I follows. if only conclusion II follows. if either conclusion I or II follows. if neither conclusion I nor II follows. if both conclusions I and II follow. (259-260): Some wise are experienced. Statements: All politicians are men. Every man is wise. Conclusions: I. Some politicians are experienced. Some men being experienced is a possibility. Conclusions: I. At least some wise are experienced. At least some men are experienced. (261-262): Statements: All actors are dancers. No dancer is a singer. Some singers are musicians. Conclusions: I. No musician is a dancer. Some dancers being musicians is a possibility. Conclusions: I. Some actors being musicians is a possibility. Some singers being actors is a possibility. (263-264): Statements: Some newspapers are magazines. All newspapers are books. Some books are journals. Conclusions: I. No journal is a magazine. Some newspapers being journal is a possibility. Conclusions: I. Some journals are newspapers. All magazines are books. Directions (Q. 265-269): In each group of questions below are given three statements followed by two conclusions numbered I and II. You have to take the given statements to be true even if they

GovernmentAdda.com

seem to be at variance with commonly known facts. Read all the conclusions and then decide which of the given conclusions logically follows from the given statements, disregarding commonly known facts. Give answer if only conclusion I follows. if only conclusion II follows. if either conclusion I or II follows. if neither conclusion I nor II follows. if both conclusions I and II follow. (265-266): Statements: All petals are flowers. No flower is a colour. All colours are fruits. Conclusions: I. No colour is a petal. All fruits being colours is a possibility. Conclusions: I. At least some petals are colours. Some flowers being fruits is a possibility. 267. Statements: All dolls are toys. Some toys are gems. Some gems are boxes. Conclusions: I. Some toys are dolls. II. Some boxes are toys. 268. Statements: Some days are nights. Some nights are weeks. All weeks are months. Conclusions: I. Some months are nights. II. All weeks being days is a possibility. 269. Statements: Some rivers are roads. Some jungles are roads Some roads are cars. Conclusions: I. All cars being jungles is a possibility. II. All rivers being cars is a possibility. Directions (Q. 270-274): In each question below are given three statements followed by two conclusions numbered I and II. You have to take the given statements to be true even if they seem to be at variance with commonly known facts. Read all the conclusions and then decide which of the given conclusions logically follows from the given statements, disregarding commonly known facts. Give answer if only conclusion I follows. if only conclusion II follows. if either conclusion 1 or II follows. if neither conclusion I nor II follows. if both conclusions I and II follow (270-271): No necessary is a result. Statements: Some inputs are outputs. All outputs are necessary. Conclusions: I. All inputs being necessary is a possibility. All results being inputs is a possibility. Conclusions: I. No output is a result. Some necessary are not outputs. (272-273): Statements: All symbols are blanks. All blanks are spaces. Some blanks are marks. 272. Conclusions: I. All symbols are spaces. II. All spaces being marks is a possibility. 273. Conclusions: I. Some spaces are not blanks. II. All marks are blanks. 274. Statements: Some demands are public. No demand is extensive. All public are central. Conclusions: I. Some demands are central. II. Some extensive are public. Directions (Q. 275-279): In each question below are given three statements followed by two conclusions numbered I and II. You have to take the given statements to be true even if they seem to be at variance with commonly known facts and then decide which of the given conclusions logically follows from the given statements, disregarding commonly known facts. Give answer if only conclusion I follows. if only conclusion II follows. if either conclusion I or II follows. if neither conclusion I nor II follows. if both conclusions I and II follow.

GovernmentAdda.com

275. Statements: All bricks are stones. Some stones are rocks. All rocks are mountains. Conclusions: I. Some mountains are stones. II. All stones being bricks is a possibility. Statements: Some radios are telephones. All telephones are mirrors. All mirrors are desks. Conclusions: I. No telephone is a desk. II. All radios being mirrors is a possibility. 277. Statements: All furniture are jungles. No jungle is a road. Some roads are hills. Conclusions: I. Some hills are definitely not jungles. II. No hill is a jungle. 278. Statements: All windows are boats. All boats are doors. All doors are buildings. Conclusions: I. All buildings being boats is a possibility. II. At least some doors are windows. 279. Statements: All houses are wheels. Some wheels are trees. No tree is a plant. Conclusions: I. No plant is a tree. II. All houses are trees. Directions (Q. 280-284): In each question below are given three statements followed by two conclusions numbered I and II. You have to take the given statements to be true even if they seem to be at variance with commonly known facts. Read all the conclusions and then decide which of the given conclusions logically follows from the given statements, disregarding commonly known facts. Give answer if only conclusion I follows. if only conclusion II follows. if either conclusion I or II follows. if neither conclusion I nor II follows. if both conclusions I and II follow. 280. Statements: All animals are cats. Some cats are lions. Conclusions: I. No animal is a tiger.

No tiger is a cat.

Some tigers are animals.

GovernmentAdda.com

Statements: Some places are cities. Some villages are places. Some places are cool. Conclusions: I. Some villages being Cool is a possibility. Some villages are cities. (282-283): Statements: All coals are diamonds. All diamonds are graphite. No graphite is carbon. Conclusions: I. No coal is carbon. No diamond is carbon. Conclusions: I. All graphites are diamonds. All coals are graphite. 284.

Statements: Some fruits are plants. No plant is a tree. No flower is a fruit. Conclusions: I. All fruits can never be trees. II. Some flowers are plants.

Directions (Q. 285-290): In each question below are given three statements followed by two conclusions numbered I and II. You have to take the given statements to be true even if they seem to be at variance with commonly known facts and then decide which of the given conclusions logically follows from the given statements, disregarding commonly known facts. Give answer if if if if if

only conclusion I follows. only conclusion II follows. either conclusion I or II follows. neither conclusion I nor II follows. both conclusions I and II follow.

(285-286): Statements: Some doors are handles. All handles are threads. Conclusions: I. Some doors are threads. Some handles are windows. Conclusions: I. All handles being windows is a possibility.

Some threads are windows.

II. All windows being doors is a possibility. (287-288): Statements: Some states are villages. No village is a forest. Conclusions: I. Some villages are definitely not towns. Some forests are not states. Conclusions: I. Some forests being villages is a possbility. Some towns are forests. (289-290): Statements: All milk is butter. All butter is curd. Conclusions: I. At least some butter is milkshake. Some milkshakes are definitely not curd.

All forests are towns.

No curd is milkshake.

Conclusions: I. All curd being milk is a possibility. All butter being milkshakes is a possibility. Directions (Q. 291-295): In each question below are given three statements followed by two conclusions numbered I and II. You have to take the given statements to be true even if they seem to be at variance with commonly known facts. Read all the conclusions and then decide which of the given conclusions logically follows from the given statements, disregarding commonly known facts. Give answer if only conclusion I follows. if only conclusion II follows. if either conclusion I or II follows. if neither conclusion I nor II follows. if both conclusions I and II follow. 291. Statements: All equals are questions. All questions are symbols. Some questions are answerable. Conclusions: I. All answerable being symbols is a possibility. II. No equal is answerable. 292. Statements: No game is an application. Some applications are easy. All easy are dynamics. Conclusions: I. No game is dynamic. II. No dynamic is application. 293. Statements: Some vitamins are calcium. All calcium are minerals. Some minerals are not vitamins. Conclusions: I. All calcium are vitamins. II. Some vitamins are not minerals is a possibility. 294. Statements: Some details are beneficial. All beneficial are helpful. Some informations are not details. Conclusions: I. Some informations are helpful. II. All helpful being beneficial is a possibility. 295. Statements: No word is a statement. Some statements are paragraphs. No paragraph is a chapter. Conclusions: I. All statements can never be chapters. II. All words being chapters is a possibility. Directions (Q. 296-300): In each question below are given three statements followed by two conclusions numbered I and II. You have to take the given statements to be true even if they seem to be at variance with commonly known facts and then decide which of the given conclusions logically follows from the given statements, disregarding commonly known facts. Give answer if only conclusion I follows. if only conclusion II follows. if either conclusion I or II follows. if neither conclusion I nor II follows. if both conclusions I and II follow. (296-297): Statements: No building is white. All whites are blacks. Some blacks are huts. Conclusions: I. No building is black. At least some whites are huts.

GovernmentAdda.com

Conclusions: I. All blacks being whites is a possibility. Some blacks are not buildings. (298-299): Statements: Some mangoes are apples. Some apples are bananas. 298. Conclusions: I. Some bananas are definitely not mangoes. II. Some grapes are apples. 299. Conclusions: I. All mangoes being grapes is a possibility. II. All bananas being apples is a possibility. 300. Statements: All flowers are rooms. Some rooms are windows. Conclusions: I. Some rooms are cards. II. At least some windows are flowers.

Some bananas are grapes.

All cards are windows.

GovernmentAdda.com

SHORT ANSWER 1. 9. 17 . 25. 33. 41. 49. 57. 65. 73. 81. 89. 97. 105. 113. 121. 129. 137. 145. 153. 161. 169. 177. 185. 193. 201. 209. 217. 225. 233. 241. 249. 257. 265. 273. 281. 289. 297.

(5) (5) (1) (4) (5) (1) (5) (2) (2) (2) (2) (5) (1) (4) (5) (1) (4) (5) (1) (5) (2) (4) (5) (2) (1) (1) (2) (1) (1) (4) (1) (4) (4) (5) (4) (1) (2) (5)

2. 10. 18. 26. 34. 42. 50. 58. 66. 74. 82. 90. 98. 106. 114. 122. 130. 138. 146. 154. 162. 170. 178. 186. 194. 202. 210. 218. 226. 234. 242. 250. 258. 266. 274. 282. 290. 298.

(5) (5) (5) (2) (1) (2) (3) (1) (2) (1) (5) (4) (2) (2) (2) (3) (2) (1) (5) (5) (1) (5) (1) (2) (1) (2) (1) (4) (4) (5) (1) (3) (3) (2) (1) (5) (1) (4)

3. 11. 19. 27. 35. 43. 51. 59. 67. 75. 83. 91. 99. 107. 115. 123. 131. 139. 147. 155. 163. 171. 179. 187. 195. 203. 211. 219. 227. 235. 243. 251. 259. 267. 275. 283. 291. 299.

(2) (5) (1) (5) (4) (4) (1) (4) (5) (1) (5) (1) (4) (4) (5) (5) (5) (1) (4) (5) (1) (1) (5) (2) (5) (4) (2) (3) (2) (2) (4) (5) (2) (1) (5) (2) (1) (5)

4. 12. 20. 28. 36. 44. 52. 60. 68. 76. 84. 92. 100. 108. 116. 124. 132. 140. 148. 156. 164. 172. 180. 188. 196. 204. 212. 220. 228. 236. 244. 252. 260. 268. 276. 284. 292. 300.

(5) (5) (2) (5) (2) (5) (4) (5) (1) (4) (1) (5) (1) (4) (1) (2) (1) (5) (5) (3) (4) (4) (5) (4) (4) (2) (5) (1) (1) (1) (1) (1) (4) (5) (2) (1) (4) (4)

5. 13. 21. 29. 37. 45. 53. 61. 69. 77. 85. 93. 101. 109. 117. 125. 133. 141. 149. 157. 165. 173. 181. 189. 197. 205. 213. 221. 229. 237. 245. 253. 261. 269. 277. 285. 293.

(5) (5) (1) (4) (5) (5) (1) (2) (2) (1) (4) (5) (5) (2) (1) (2) (4) (1) (2) (2) (1) (4) (1) (2) (1) (5) (1) (1) (2) (1) (4) (4) (2) (5) (1) (1) (2)

6. 14. 22. 30. 38. 46. 54. 62. 70. 78. 86. 94. 102. 110. 118. 126. 134. 142. 150. 158. 166. 174. 182. 190. 198. 206. 214. 222. 230. 238. 246. 254. 262. 270. 278. 286. 294.

(5) (2) (2) (4) (4) (1) (2) (2) (1) (2) (2) (2) (1) (1) (4) (1) (5) (5) (2) (1) (2) (5) (4) (1) (2) (1) (3) (2) (2) (2) (5) (1) (1) (3) (5) (5) (2)

7. 15. 23. 31. 39. 47. 55. 63. 71. 79. 87. 95. 103. 111. 119. 127. 135. 143. 151. 159. 167. 175. 183. 191. 199. 207. 215. 223. 231. 239. 247. 255. 263. 271. 279. 287. 295.

(1) (5) (4) (5) (1) (4) (1) (1) (5) (2) (5) (4) (2) (5) (2) (4) (2) (3) (1) (1) (1) (1) (1) (2) (5) (5) (5) (1) (1) (1) (2) (5) (2) (1) (1) (4) (5)

8. 16. 24. 32. 40. 48. 56. 64. 72. 80. 88. 96. 104. 112. 120. 128. 136. 144. 152. 160. 168. 176. 184. 192. 200. 208. 216. 224. 232. 240. 248. 256. 264. 272. 280. 288. 296.

(5) (5) (3) (2) (1) (1) (1) (2) (5) (1) (1) (4) (5) (4) (4) (5) (5) (1) (4) (4) (2) (2) (5) (3) (4) (3) (2) (5) (5) (5) (5) (5) (4) (5) (1) (2) (4)

GovernmentAdda.com

[30]

ANSWERS 10. 5; Sharp Blades 1.

Blunt Sword

5;

White

Yellow

Silver

Gold

OR White

Gold White

2.

5;

Flowers

Yellow

Silver

Beautiful

OR White

Tasty yummy

Yellow

Silver

2;fruit good

Gold

So, II follows.

From the above possible venn diagrams, both I and II follow. 5; Possible Venn diagrams are

grapes

5;mango black Moon 5. 5;

pen book pencil

5;TigerPanther

Star Blue

or

Star Moon

Thus, I follows, Round Planets Moon Star

Lion

Perfect

GovernmentAdda.com

1;Engineer

8.

5;

Doctor

Tomato

Goods

Egg

Potato

Thus, II follows. 12. 5; From the Venn diagram (c), both I and II follow. 5; A possible Venn diagram is Pants

Hence, I follows.

Jackets Trouser

Shirts

Goods Egg Tomato

Potato

Thus, II follows

So, both I and II follow. 2; From the above possible diagram, I does not follow. Only II follows. 5;

Dogs

Cake

Pastry

Cats

9.

5;

Animals

White

Breads

Thus, II follows. Another possible Venn diagram is

OR White

Cake Breads Pastry

Cats Animals

Dogs

Thus, I follows. 16. 5; In both the possible cases above, I follows.

[31] king princes

Queen

24. 3;

Brave

Thus, II follows. Another possible Venn diagram is 25. 4;

king

Queen

Brave Prince

C

D

C D W

W

Thus, I follows. 17. 1; From the above two Venn diagrams II does not follow. Only I follows. (18-19):

(26 - 277): Possible Venn-diagram are

Purse

Beautiful

Bag

2; Thus, only II follows. 5; Another possible Venn - diagram is

Triangles Circle Rectangle

Square

Black

Purse Beautiful Bag

Black

Dash lines indicate possibilities. Both I and II follow. (28-29): Possible Venn - diagram is

5 1 2;

Fish cat

Flowers White

Dog

Black

or

or

Fish cat Dog Black

White

5; Both I and II follow. 4 4; Possible Venn-diagram -

GovernmentAdda.com Flowers

Black

Thus, I doesn’t follow. Another possible Venn diagram is Flowers

red

flower

Black

White

rose

Thus neither I nor II follows.

Thus, II follows.

Dog

31. 5;

Cats

Cats Black

I follows II also follows. 2; From the above Venn-diagram in Qustion 161, I doesn’t follow. Also,

Dogs

Animals

Animal

White

From the above two venn diagrams, II follows. 1; Possible Venn-diagrams are

Black

Dog Cat

Thus, II follows Thus only I follows. doesn’t follow as it is not always true. 2; Possible Venn-diagrams are

33. 5; Beautiful Red

Roots Leaves

Branches

or

Branches

Flower

Rose

Leaves Roots So, only II follows. 23. 4;

Both I and II follow. 1; From the above Venn - diagram only I follows. 4; A possible Venn - diagram is

[32] Sea Ocean

Some soaps are powders + All powders are detergents = I + A = I = Some soaps are detergents. Hence, I follows. But its conversion does not lead to II. 4; I + I = No conclusion. 1; No perfume is a paper + Some papers are bottles = E + I O* = Some bottles are not perfumes. Hence, I follows. But we can’t proceed further. Hence, II does not follow. 5; The possibility in I exists as there is negative statement. Again, All books are magazines + All magazines are ice

River

Lake Thus, neither I nor II follows. 2;

J

= A + A = A = All books are ice conversion ice are books (I). Hence, II follows. 3; They form an I - E complementary pair.

5 4;

J

Some

1; Stone

Precious

Diamond

1;

Thus, only I follows. Stone

40. 1;

52. 4;

Diamond

Thus, I doesn’t follows. Diamond Precious

(41-42): Possible Venn-diagrams are

Glass

Thus, doesn’t II follow. 53. 1;

Doctor Boy

Smart Good Brown

OR Doctor Boy

Or Smart

Good

GovernmentAdda.com Brown

1; Only I follows. 2; Only II follows. (43-44): Possible venn diagrams are Yummy

Thus, only I follows.

54. 2;

Tasty

Ice

Tasty

Thus, only II follows. 55. 1; A possible Venn-diagram is

OR

Yummy

Good Brown

Ice

43. 4 Tasty 5 5; Possible Venn-diagrams are

Pencil

Candy

Paper

Black

Thus only I follows 56. 1;



 

Con

ver

sion

o

Some trees are bushes Some bushes are trees. It means at least some bushes are trees. Hence, conclusion I follows. But conclusion II does not follow Tasty Apple

because A + I o No conclusion. 2; All colours are paints (A) + No paint is a brush (E) = A +

Red

E = E o No colour is a brush

Conversiono No brush, is a colour.

OR Tasty Apple

Possible Venn-diagrams are J

46. 1; Some powders are soaps (I) powders (I).

Hence, conclusion II follows. But conclusion I does not follow. 1; Some chemicals are organics (I) + All organics are

Red

conversion

Again, only detergents are powders powders are detergents.

J

J

Some soaps are

conversion

J

All

fertilizers (A) = I + A = I o Some chemicals are fertilizers. It means At least some fertilizers are chemicals. Hence, conclusion I follows but conclusion IIdoes not follow. 59. 4; No air is solid (E) + Some solids are liquids (I) = E + I =

[33] O*. Some liquids are not air. Neither conclusion I nor II follows. 5; All gems are diamonds (A) + All diamond are rocks (A) = A + A = A o All gems are rocks. Thus, conclusion II follows.

Thus 1 follows. But II doesn’t follow. 75. 1; Black Kites

Conversion

Now, All gems are rocks o Some rocks are gems. It means, At least some rocks are gems. Hence, both conclusion I and II follow. 2; I does not follow because there is no negative statement.

J

Thus I follows. (76-77): Possible Venn diagrams are:

J

I + A = I (Some colleges are universities conversion Some universities are colleges + All colleges are schools = Some universities are schools. Hence, probability of II exists.

62. 2; Some universities are colleges + All colleges are schools (I + A = I) = Some universities are schools. This does not lead us to I. II follows because All colleges are hostels.

J

Red or Red

1; No pencil is a paper + All papers are notebooks (E + A O*) Some notebooks are not pencils. Hence, possibility of II is ruled out. All pens are pencils + No pencil is a paper (A + E = E) No pen is a paper + All papers are

Flowers

J

2; 2; 2;

5; 1; 2;

1;

notebooks (E + A O*) Some notebooks are not pens This does not rule out the possibility of I II follows (A + E = E) but I does not follow unless All pencils are pens Since there is no negative statement, possibility exists but certainty does not follow. All tables are chairs + No chair is a cooler (A + E = E) = No table is a cooler + All cooler are fans (E + A = O*) = Some fans are not tables. Hence, I does not follow. But No chair is a cooler + All coolers are fans = Some fans are not chairs. This leaves us with the possibility of II. A + E = E. So, I follows. We also get Some fans are not tables. This leaves the possibility of II. There is no negative statement. Hence II does not follow and I follows, I + A = I. Hence, Some strong are winners + All winners are modest = Some strong are modest. This leaves us with the possibility of II. No weak is strong + Some strong are modest = E + I = O* Some modest are not weak. This leaves us with the possibility of 1. While No weak is strong + Some strong

OR

Red

Flowers

4; Neither I nor II follows. 1; Only I follows. (78-79):

GovernmentAdda.com J

O*). So, II does not follow. are winners (E + I 5; A possible Venn-diagram is

Queen

Princess Girl

Woman Tole

(a)

Silver Gold

Metal

Thus I follows Precious

(b)

Thus II also follows. 5; From the Venn-diagram (b), both I and II follows 2; Tree

Tol e Plant

74. 1; A possible Venn-diagram is Green

2; Only conclusion II follows. 2; A possible Venn diagram is

Girl

Prince

ss

Thus, only II follows. 1; A possible Venn diagram is High Hills

Thus, only I follows. 2; All apartments are buildings + All buildings are cottages J A + A A = All apartments are cottages. Hence, I does not follow.

Some apartments are huts (I) conversion

J

J

Some huts

GovernmentAdda.com

are apartments (I) + All apartments are buildings (I + A I) = Some huts are buildings. Hence the possibility of

[34]

J

exists. 5; All staplers are printers + No printer is a computer = A + = E No stapler is a computer + All computers are

J

machines. = E + A O* = Some machines are not staplers. This does not rule out the possibility of I. All staplers are printers + No printer is a computer = A +

J

E

E = No stapler is a computer. Hence, II also follows.

83. 5; I is true because All planets are fans. For II: All

planets

J J are fans Conversion Some fans are planets + No planet is a J

Some fans are not universe. This does not rule out universe. the possibility of II.

1; Some professors are intelligent (I) + No intelligent is

J

O = Some professors are not honest. honest = I + E This leaves us with the possibility of I. While II does not follow because ‘No Intelligent is an honest.’ J

4; All tins are copper + No copper is a metal = A + E E = No tin is a metal. So the possibility of I is ruled out. No pin is a tin + All tins are copper = (E + A II does not follow.

J

O* = Some coppers are not pins. So,

2; Some balls are locks. + All locks are keys (I + A

J

I) = Some

J

balls are keys + Some doors are keys (I + I no conclusion). So, I does not follow. As there is no negative statement, so possibility case is true.

J

5; Some balls are locks + All locks are keys (I + A I). So, II follows. Some doors are keys can be derived from ‘All doors are keys’. So, possibility case follows. 1; Some ants are birds

J

conversion J

All ants are cobras (I + A

J

Some birds are ants

I) Some birds are cobras.

J

All lizards are birds. + Some birds are cobras (A + 1 no conclusion). But in case of possibility All cobras being lizards can be true. 5 4; ‘No’, statement can never lead to ‘All’. So, I does not follow. And Some shares are debentures + No debenture is an

J

O) = Some shares are not equities. So, equity (I + E possibility of II is also ruled out.

Hence, neither conclusion I nor II follows. 1; Some banks are colleges (I) + All colleges are schools (A) J

I+A I = Some banks are schools. It means, at least some banks are schools.

Hence, only conclusion I follows, but conclusion II does not follow. 101. 5; All books are magazines ( A) + No magazine is a newspaper (E) = A + E = E = No book is a newspaper. Hence, conclusion I follows. conversion

Again, All books are magazines o Some magazines are books, Thus conclusion II follows. So, both conclusion I and conclusion II follow. 102. 1; Some mobiles are telephones (I) + All telephones are pagers (A) = I + A = I = Some mobile are pagers (I) + No pagers is a camera (E) = I + E = O = Some mobiles are not cameras. Hence, I follows. 103. 2; All telephones are pagers (A) + No pager is a camera (E) = A + E = E = No telephone is a camera. Hence conclusion I does not follow. But, Some mobiles are telephones means, All mobiles being telephone is a possibility. 104. 5; All squares are cubes (A) + No cube is a cuboid (E) = A + conversion

E = E = No square is a cuboid o No cuboid is a

square. Hence, conclusion I follows. No cube is a cuboid (E) + Some cuboids are rectangles (I) = E + I = O* = Some rectangles are not cubes. It means All cubes being rectangle is a possibility. Both conclusion I and II follow. 105. 4; All squares are cubes (A) + No cube is a cuboid (E) = A + E = No square is a cuboid (E) + Some cuboids are rectangles (I) = E + I = O* = Some rectangle are not squares, Hence, conclusion I does not follow. Similarly conclusion II does not follow. 106. 2; All fruits

are vegetables (A) conversion oSome vegetables are fruits. It means, All vegetables being fruits is a possibility. Hence, conclusion II follows. But, Some vegetables are pulses (I) + Some pulses are not cereals (O) = I + O = No conclusion. Hence, conclusion I does not follow.

GovernmentAdda.com

(91-92): 1; One of the possible Venn-diagrams is-

conversion

Star Planet

Thus only I follows. 92. 5; From the above Venn diagram both I and II follow. (93-94):

94. 2

hut

95. 4 98. 2;

conversion

No force is heat o No heat is force Hence,

cottage

96. 4 97. 1 All exams are tests (A) + No test is a question (E) = A + E J E = No exam is a question.

Hence, only conclusion II follows. But conclusion I does not follow. 4; No bangle is an earring (E) + Some earrings are rings (I) B+I

J

conversion

108. 4; Some exams are tests o Some tests are exams (I) + No exam is a question (E) = I + E = O = Some tests are not questions. Hence, neither conclusion I nor II follows. 109. 2; All forces are energies (A) + All energies are powers (A) = A + A = A = All forces are powers (A) + No power is heat (E) = A + E = E

Amphibian insect Plant

93. 5 (95-97) :

107. 4; All schools are colleges o Some colleges are schools (I) + All schools are universities (A) = I + A = I = Some colleges are universities. Hence, conclusion I does not follow. All schools are universities (A) + No university is a campus (E) = A + E = E = No school is a campus. Hence, conclusion II does not follow. Thus, neither conclusion I nor II follows.

O* = Some rings are not bangles.

conclusion II follows but conclusion I does not follow. 110. 1; ALL energies are powers (A) + No power is heat (E) = A + E = E = No energy is heat. Hence, conclusion I follows. But, All forces are power (A) + No power is heat (E) = A + E = E = No force is heat. So, conclusion II does not follow. 111. 5; All plastics are notes (A) + No note is a coin (E) = A + E = E = No plastic is a coin

conversiono No coin is plastic.

Hence, conclusion I follows. Again, No plastic is coin (E) + Some coins are metals (I) = E + I = O* = Some metals are not plastics. It means All plastics being metals is a possibility. 112. 4; All plastics are notes (A) conversiono Some notes are

[35] plastics. Hence, conclusion II does not follow. Again, All plastics are notes (A) + No note is a coin (E) = A + E = E = No plastic is a coin (E) + Some coins are metals (I) = E + I = O* = Some metals are not plastics. Hence, conclusion I does not follow. Thus neither conclusion I nor conclusion II follows.

124. 2; All entrances are windows (A) + All windows are (E) = A + E = E = No entrance is a gate conversiono No gate is an entrance. Hence, conclusion II follows. Again, All windows are doors (A) + [No gate is a door (E)

conversiono ] No door is a gate (E) = A + E = E = No

conversion

113. 5; Some symbols are figures o Some figures

window is a gate. Thus, conclusion I does not follow.

are symbols (I) + All symbols are graphics (A) = I + A = I = Some f igures are graphics

conversion

oSome

125. 2; Some books are colleges does not necessarily lead us to Conclusion I. Again, All notes are books (A) + All books are pages (A) = A + A = A. All notes are pages. Hence, conclusion II follows. 1; The possibility in conclusion I exists in the first statement. Conclusion II does not follow because there is no negative statement. 4; Some trees are threads (I) + All threads are boxes - I + A I. Some tees are boxes. No needle is a tree (E) + Some trees are threads I = E + I = O* = Some threads are not needles. Hence, neither conclusion I nor II follows. 5; The possibility stated in conclusion I is inherent in the third statement while that in conclusion II is inherent in the second statement. J J

graphics are figures. Hence, conclusion I follows. Again, All symbols are graphics (A) + No graphic is a picture (E) = A + E = E = No symbol is a picture. Hence, conclusion II follows. Thus, both conclusion I and II follow. 114. 2; All vacancies are jobs (A) + Some jobs are occupations (I) = A + I = No conclusion. Hence, I does not follow.

All vacancies are jobs



con



vers

ion



oSome jobs are

vacancies (I) + Some jobs are occupation (I) = I + I = I = No conclusion However, All occupations being vacancies is a possibility. Thus, conclusion II follows. 115. 5; All aeroplanes are kites (A) + All kites are birds (A) = A + A

J

doors

(A) + [No gate is a door (E) conversiono ] No door is a gate

4; Some players are spectators (I) conversion are players (I) + No player is a theatre = I + E

A = All aeroplanes are birds.

O = Some spectators are not theatres. 130. 2; Some reds are blues (I) + All blues are pinks (A) = I + A = I = Some reds are pinks. Hence, Some reds are not pinks is a possibility. Conclusion II follows.

Hence, conclusion II follows. Again, All kites are birds (A) + No bird is a fish (E) = A +

J

E = No kite is a fish conversiono No fish is E a kite. So, conclusion I follows. Hence both conclusion I and conclusion II follow.

J

116. 1; Some wires are fires (I) + All fires are tyres (A) = I + A = Some wires are tyres conversiono Some tyres are wires. Hence, conclusion I follows but II doesn’t follow.

Some spectators

I

But, Some reds are blues is a particular positive statement. So,~conclusion I does not follow.

5; Some table are chairs (I) + All chairs are pens (A) = I + A I Some tables are pens tables.

J

conversion

J

Some pens are

GovernmentAdda.com

117. 1; All badges are pins (A) + [No clip is a pin

conversiono No pin is a clip (E)] = A + E No badge is a clip. Hence conclusion I follows.

J

E=

But, All badges are pins conversiono Some pins are badges. Hence, conclusion II does not follow. 4; No colour is a paint (E) + No paint is a brush (E) = E + E

J

No conclusion Hence, neither conclusion I nor conclusion II follows.

2; All stars are planets (A) + All planets are galaxies (A) = A + A

J

A = All stars are galaxies.

Hence, conclusion II follows. But, conclusion I does not follow. 120. 4; E + E = No conclusion. Thus, neither conclusion I nor conclusion II follows. 121. 1; All rewards are medals (A) + All medals are awards (A) = A + A = A = All rewards are awards. Hence, conclusion I follows. But, conclusion II does not follow. 122. 3; All bushes are plants + Some plants are trees = A + I = No conclusion. Hence, neither conclusion I nor II follows. However, since they make a complementary pair I-O, either conclusion I or conclusion II follows. 123. 5; All bottles are glasses (A) + [No cup is a glass (E) conversiono ] No glass is a cup (E)

= A+E = E = No bottle is a cup. Hence conclusion I follows. Again, All bottles are glasses conversiono Some glasses are bottles. It means, At least some glasses are bottles. Hence conclusion II follows.

Thus, both conclusion I and II follow.

Hence, conclusion I follows. Again, All chairs are pens (A) + All pens are copies (A) = A + A = A = All chairs are copies. It means All copies being chairs is a possibility. Thus, conclusion II follows. Hence, both conclusion I and II foliow. 1; Some moons are stars (I) + All stars are skies (A) = I + A I = Some moons are skies. Hence, conclusion I follows.

J

J

Again, Some moons are stars conversion Some stars are moons (I) + No moon is a planet (E) = I + E = O = Some stars are not planets. Hence, conclusion II does not follow. 133. 4; Conclusion I need not be true. Some water is milk means some milk is water (conversion) Now, for II, suppose All water is milk. Then All water is milk + All milk is cold drinks = A + A = A = All water is cold drink. But the third statement contradicts this. Hence II does not follow. J J 134. 5; All flowers are coins conversions Some coins are flowers. Hence, conclusion II follows.

Again, All trees are flowers (A) + All flowers are coins (A) = A + A = A = All trees are coins (A) + Some coins are notes (I) = A + I = No conclusions. But there is no negative statement. So, All notes being trees is a possibility. Thus, conclusion I follows. Hence, both conclusion I and II follow. 135. 2; All erasers are pens (A) + All pens are boxes (A) = A + A =

J

J

A = All erasers are boxes conversion Some boxes are erasers (I) + No eraser is pencil (E) = I + E = O = Some boxes are not pencils. Hence, conclusion I does not follow. Again, All erasers are

J

J

pens (A) conversion Some pens are erasers (I) + No eraser is a pencil (E) = I + E = O = Some pens are not pencils. It means Some pens being pencils is a possibility.

[36] Thus conclusion II follows. 136. 5; Some tests are unpleasant (I) + All unpleasant things need to be avoided (A) = I + A = I = Some tests need to be avoided. Thus, conclusion I follows. The possibility stated in conclusion II is inherent in the first statement. 5; The conclusions I and II are inherent in the first statement.

J

J

conversion Some fit are 1; Some firefighters are fit (I) firefighters (I) + All firefighters are courageous (A) = I + A = I = Some fits are courageous. Hence conclusion II does not follow. Again, All firefighters are courageous (A) + All courageous people are armymen (A) = A + A = A All firefighters are armymen. Hence, conclusion I follows. 139. 1; All enthusiatic people are positive (A) + No positive people are honest (E) = A + E = E = No enthusiatic people are

J

J

honest conversion No honest people are enthusiatic. Hence, conclusion I follows. Again, No lawyers are enthusiatic (E) + All enthusiatic people are positive (A) = E + A = O* = Some positive people are not lawyers. Hence II does not follow. 140. 5; Conclusion I is inherent in the first statement. Again, All dogs are kittens (A) + No kittens are black (E) = A + E = E = No dog is black Hence, conclusion II follows. 1; There is no negative statement. Hence, Conclusion I follows. But conclusion II is a negative conclusion. Hence, II does not follow. 5; All scholar are eccentric (A) + Conversion of No woman is

J

J

eccentric conversion No woman is a scholar. Hence, conclusion I follows. Again, All scholars are eccentric (A) + All eccentrics are studies (A) = A + A = A. All scholars are studies. It means. All studies being scholar is a possibility. Hence, conclusion II follows.

(A) + No copy is a book (E) = A + E = E = No paper is a

J

J

conversion No book is a paper. Thus, book (E) conclusion II follows. 151. 1; No copy is a book (E) + Some books are magazines. (I) = E + I = O* = Some magazines are not copies. It mean All copies being magazines is a possibility, Hence, conclusion I follows. Again, All papers are copies (A) + No copy is a book (E) = A + E = E = No paper is a book (E) + Some books are magazines (I) = E + I = O* = Some magazine are not papers. Hence, conclusion II does not follow.

J

J

152. 4; All cars are motors conversion Some motors are cars (I) + All cars are bikes (A) = I + A = I = Some motors are bikes. Hence conclusion I does not follow. Again, All cars are bikes (A) + No bike is an auto (E) = A + E = E = No car is an auto (E). Thus, neither conclusion I nor II follows.

J

J

153. 5; Some jobs are occupations conversion Some occupations are jobs (I) + All jobs are vacancies (A) = I + A =

J

J

I = Some occupations are vacancies conversion Some vacancies are occupations. Hence, Conclusion I follows.

Again, All jobs are vacancies (A) + No vacancy is an unemployment ( E) = A + E = E = No job is an unemployment. Hence, conclusion II follows. Thus, both conclusion I and II follow.

J

J

154. 5; Some jobs are occupations conversion Some occupations are jobs (I) + All jobs are vacancies (A) = I + A = I = Some occupations are vacancies (I) + No vacancy is an unemployment (E) = I + E = O Some occupations are not unemployment. Hence, conclusion II follows. Also, conclusion I may follow. J J 155. 5; All stars are galaxies implication Hence conclusion II follows.

Some stars are galaxies.

Again, All stars are galaxies (A) + Some galaxies are sun (I) = A + I = No conclusion. But there can be a possible relation between Star and Sun. Thus, conclusion 1 follows. Some galaxies are suns (I) + conversion of No planet is sun (E) = Some galaxies are not planets(O). However, conclusions 1 and II make a complementary pair (I-E). Thus, either conclusion I or 1! follows. All DNAs are cells (A) + Some cells are tissues (I) = A + I = No conclusion. Thus, conclusion I does not follow. Again, No RNA is a DNA (E) + All DNAs are cells (A) = E + A = O* = Some cells are not RNAs. Thus, conclusion II follows. No RNA is a DNA(E) + All DNAs are cells (A) = E + A = O* Some cells are not RNAs. But All RNA being cells is a possibility. Hence, conclusion I follows. But conclusion II does not follow. Some stains are dirts (I) + All dirts are shines (A) = I + A I = Some stains are shines, Hence, II does not follow. J J

GovernmentAdda.com J

J

143. 3; Some eggs are hard-boiled conversion Some hardboiled are eggs (I) + No eggs are uncrackable (E) = I + E = O = Some hard-boiled are not uncrackable. But, conclusion I and II make a complementary pair (I-E). 144. 1; All perfumes are expensive (A) + All expensive things are unique (A) = A + A = All perfumes are unique. Hence, All unique thing being perfumes is a possibility. Thus, conclusion I follows. But II does not follow. 1; All stars are bottles (A) + Some bottles are papers (I) = A A = No conclusion. However, neither statement is negative. Hence. All stars being papers is a possibility. Hence conclusion I follows. Again, Some bottles are papers (I) + No paper is a calendar (E) = I + E = O = Some bottles are not calendars. Hence conclusion II does not follow. J J

146. 5; All stars are bottles

conversion

Some bottles are

stars. Hence conclusion II follows. Only, Some bottles are not calendars. Hence conclusion II follows. 147. 4; Some bottles are papers (I) + No paper is a calendar (E) = I + E = O = Some bottles are not calendars. Hence conclusion I does not follow. And there is no definite positive or negative relation between star and calendar. Hence conclusion II does not follow. 148. 5; Some pencils are blankets (I) + All blankets are erasers (A) = I + A = I = Some pencils are erasers. Hence conclusion I follows. Now, Some pencils are erasers, it means All erasers being pencils is a possibility. Hence conclusion II follows.

2; Because there is no negative statement. Hence conclusion I does not follow. But conclusion II is inherent in the first statement. 2; No copy is a book (E) + Some books are magazines (I) = E + I = O* = Some magazines are not copies. Hence, conclusion 1 does not follow. Again, All papers are copies

156. 3;

157. 2;

1;

1;

Again, Some stains are dirts (I)

conversion

Some

dirts are stains (I) + No stain is dull (E) = I + E = O = Some dirts are not dull. However, All dull being dirts is a possibility. Thus, conclusion 1 follows. 160. 4; Some coffee are cups (I) + Some cups are not copies (O) = I + O = No conclusion. Thus, conclusion I does not follow. Again, All colds are coffee (A) + Some coffee are cups (I) = A + 1 = No conclusion. Thus, II does not follow.

J

J

2; Some coffee are cups (I) conversion Some cups are coffee (I). Hence conclusion I is not true. Conclusion II follows because this possibility exists in the third statement. 1; No hour is a month (E) + All months are weeks (A) = E + A = O* = Some weeks are not hours. Again Some weeks are years (I) + All years are decades (A) = I + A = I = Some weeks are decades (I). Now, Some weeks are decades

J

conversion

J

Some

[37] decades are weeks (I) + Some weeks are not hours (O*) = No conclusion. But, a relation is possible between hour and decades. It means All hours being decades is a possibility. Hence, conclusion I follows. But, II does not follow. 1; Some weeks are years (I) + All years are decades (A) = I + = I = Some weeks are decades, means All decads being weeks is a possibility. Hence, conclusion I follows. But,

does not follow.

J

J

J

J

4; All elections are dramas conversion Some dramas are elections (I) + All elections are stories (A) = I + A = I = Some dramas are stories (I) conversion Some stories are dramas (I) + Some dramas are not films (O) = I + O = No conclusion. Hence, I does not follow.

also does not follow. 1; All elections are dramas (A) + Some dramas are not films (O) = A + O = No conclusion But All film being elections is a possibility, Hence, conclusion I follows. II does not follow. 2; I is false because No spade is a club. II is true because All clubs are diamonds. 1; All roads are drivers + Some drivers are pilots = A + I = No conclusion. But the possibility of I exists because neither of these statements is negative. Some walkers are not drivers + Some drivers are pilots = O + A = No conclusion. Hence, II does not follow. 2; No bank is a school (E) + Some schools are colleges (I) = + I = O* = Some colleges are not bank, it means All banks being colleges is a possibility Hence, conclusion follows. But conversion of 2nd statement does not lead to conclusion I 4; All baskets are trolleys (A) + conversion of some carts are trolleyes (I) = A + I = No conclusion Hence. I does not follow. II does not follow by converting the second statement Hence, neither I nor II follows. 170. 5; All fruits are vegatables (A) + All vegetables are plants (A) = A + A = A = AI1 fruits are plants Hence, conclusion I follows. Again, All vegetables are plants (A) + No plant is a root (E) = A + E = E = No vegetable is a root conversions No root is a vegetable. Hence, conclusion II follows. 1; All fruits are vegetables (A) + All vegetables are plant (A)

conclusions are answer, Now, All statements are conclusions (A) + Some conclusions are answers (I) = A + I = No conclusion, but a possible positive relation exists between statements and answers. Thus, conclusion I follows. However, a definite conclusion can’t be obtained. Hence, conclusion II does not follow. 179. 5; All statements are conclusions (A) + All conclusions are options (A) = A + A = A = All statements are options

J

conversion Some options are statements. Hence, conclusion 1 follows. Conclusion II follows from second and third statement. 180. 5; All coffees are sweets (A) + No sweet is namkeen (E) = A + E = E = No coffee is namkeen. Thus, conclusion I follows. Again, Some drinks are coffees (I) + All coffees are sweets (A) = I + A = I = Some drinks are sweets + No sweet is namkeen (E) = I + E = O = Some drinks are not namkeens. Thus, conclusion II follows. 181. 1; Some drinks are coffees (I) + All coffees are sweets (A) = I + A = I = Some drinks are sweets. It means All sweets being drinks is a possibility. Hence, conclusion I follows.

Again, All coffees are sweets (A) + No sweet is namkeen (E) = A + E = E = No coffee is namkeen. Hence, conclusion

II does not follow. 4; No sharpener is a scale (E) + Some scales are pencils (I) = E + I = O* = Some pencils are not sharpeners. Thus, conclusion I does not follow. We can’t find any definite positive or negative relation between eraser and pencil.

J

J

183. 1; All papers are books conversion Some books are papers (I) + No paper is a novel (E) = 1 + E = O = Some books are not novels. But there is a possibility that some books are novels. Thus conclusion I follows. Again, All papers are books (A) + All books are files (A)

J

J

conversion = A + A = A = All papers are files (A) Some files are papers (I) + No paper is a novel (E) = I + E = O = Some files are not novels. Thus, conclusion II does not follow. 184. 5; All locks are buttons (A) + No button is a clock (E) = A +

GovernmentAdda.com A + A = A = All fruits are plants (A) + No plant is a root

= A + E = E = No fruit is a root Hence, conclusion I follows. But All vegetable are plants (A) + No plant is a root (E) = A + E = E = No vegetable is a root. Hence, II does not follow. 4; Some calculators are phones (I) + No phone is an eraser = I + E = O = Some calculators are not erasers, Neither conclusion I nor II follows. 173. 4; Some conclusions are assumptions (I) + All assumptions are arguments (A) = I + A = I = Some conclusions are arguments. Hence, I does not follow. Again, No statement is a conclusion (E) + Some conclusions are assumptions (I) = E + I = O* = Some assumptions are not statements. Hence, conclusion II does not follow. 5; The possibility of conclusion I can be obtained from the third statement while that of conclusion II from the second statements. 1; The possibility of conclusion I can be obtained from the third statement. But II does not follow because there is no negative statement. 2; Conclusion I can’t be obtained from first statement. Again, All plants are fabrics (A) + All fabrics are garments

(A) = A + A = A = All plants are garments. Thus, Conclusion II follows from second and third statements. 177. 5; All towns are states (A) + No state is a world (E) = A + E = E = No town is a world. Hence, Conclusion I follows. Again, Some villages are towns (I) + All towns are states (A) = Some villages are states. Since No state is a world, Conclusion II follows. 178. 1; Some answers are conclusions (I)

J

conversion

J

Some

J

conversion E = E = No lock is a clock lock (E). Thus, conclusion I follows.

J

J

No clock is a

J

All locks are buttons (A) conversion Some buttons are locks (I). Here, All buttons being locks is a possibility. Hence, both I and II follow.

J

J

185. 2; Some clips are watches (I) conversion Some watches are clips (I) + All clips are telephones (A) = I + A = I = Some watches are telephones. From this conclusion I does not follow. All clips are telephones (A) + All telephones are pins (A) = A + A = A = All clips are pins. Thus, conclusion II follows.

2; Some trees are flowers (I) + All flowers are fruits (A) = I = Some trees are fruits (I). Now, Some benches are trees (I) + Some trees are fruits (I) = I + I = No conclusion. From this, conclusion I does not follow. Now, Some benches are- trees. So All trees being benches is a possibility. Hence, conclusion II follows. 187. 2; Some, trees are flowers (I) + All flowers are fruits (A) = I + A = I = Some trees are fruits (I). Hence, conclusion I does not follow. Some benches are trees (I) + Some trees are flowers (I) = No conclusion. But some possible relation can be established between them. Hence, ‘All flowers being benches is a possibility. Therefore, only conclusion

II follows. 4; No mango is a tomato (E) + All tomatoes are potatoes (A) = E + A = O* = Some potatoes are not mangoes (O*) We can’t proceed further. Thus, conclusion I does not follow. Again, conclusion II does not follow because of the 3rd statement. 189. 2; Some tangos are Charlies (I) + No charlie is a sweet (E) = I + E = Some, tangos are not sweets. Means Some tangos being sweets is a possibility. Hence, conclusion II follows. But conclusion I does not follow. ; 190. 1; No book is a paper (E) + Some papers are pens (I) = E +

[38] I = O* = Some pens are not books (O*) This rules out the posibility of II but I follows. 191. 2; No book is a rough (E) + No rough is a pen (E) = E + E = No conclusion. But some possible relation can be established between book and pens. Hence, conclusion II follows. 192. 3; Some columns are inks (I) + No ink is a copy (E) = I + E = O = Some columns are not copies. But conclusions I and II make a complementary pair (I E). Thus, either conclusion I or II follows. 1; Some balls are footballs (I) + All footballs are volleyballs = I + A = 1 = Some balls are volleyballs. It means, All Volleyballs being balls is a possibility. Hence, conclusion I follows. Again No fund is a bat (E) + All bats are balls

= E + A = O* = Some balls are not funds Hence, II does not follow. 1; Some balls are footballs (I) + All footballs are volleyballs J

= I + A = I = Some balls are volleyballs (I)

J

conversion

Some volleyballs are balls (I) + Some balls are not fund (O) = No conclusion.

But, possibility of 1 can be established between fund and volleyball. It means All funds being volleyball is a possibility. Hence, conclusion I follows. Again, No fund is a bat (E) + All bats are balls (A) = E + A = O* = Some balls are not funds. So, Some balls are footballs -»

J

conversion Some footballs are balls (I) + Some balls are not funds (O) = I + O = No conclusion. Hence, II does not follow.

J

J

195. 5; No star is a sun (E) conversion No sun is a star (E) + All stars are moons (A) = E + A = O* = Some moons are not suns. But All sun are planets. Hence conclusion I follows. Again, No star is a sun (E) + All suns are planets

= E + A = O* = Some planets are not stars. But All suns are planets. Thus, conclusion II follows.

J

J

4; All businesses are stores conversion Some stores are businesses (I) + All businesses are capital (A) = I + A = I = Some stores are capital But, Some stores are not products (O). It leads us nowhere. Hence conclusion 1 does not follow. Again, conclusion II need not follow from first statement. 1; Suppose conclusion I follows. Then, All products are businesses + All businesses are stores = A + A = A = All products are stores. Now, in this situation, Some stores are not products can be true. Hence conclusion I follows. As for II, nothing can be said with certainty.

(E) = I + E = O = Some corridors are not corners. Thus, All corridors being corners is a possibility does not hold true.

J

J

Again, No carrom is a corner (E) conversion No corner is a carrom (E). Now, All class are corners (A) + No corner is a carrom (E) = A + E = E = No classes is a carrom. Hence, All carroms being classes is a possibility is not true. Thus, neither conclusion I nor II follows. 204. 2; Some corridors are carroms (I) + No carrom is a corner (E) = I + E = O = Some corridors are not corners. Thus, conclusion I does not follow. Conclusion II follows as we have seen in the explanation to Q. 157 5; Some ponds are rivers (I) + All rivers are lakes (A) = I + A = I = Some ponds are lakes It means Some lakes are not ponds is a possibility. Hence, conclusion II follows.

J

J

Again, Some oceans are lakes conversion Some lakes are oceans. Now, All rivers are lakes (A) + Some lakes are oceans (I) = A + I = No conclusion, but a possible positive relation exists between ‘river’ and ‘ocean’. Thus, conclusion I follows. 1; Suppose ‘Some brown are not black’ is not true. It means All brown are black. Now, All brown are black + All black are bags = A + A = A = All brown are bags. But this can’t be true because ‘Some brown are not bags’ (given). Hence, our assumption is wrong and conclusion I is true. All blue are brown + Some brown are not bags = A + O = No conclusion. Hence, II does not follow. -

J

J

5; No train is a truck (E) conversion No truck is a train (E) + Some trains are tumblers (I) = E + I = O* = Some tumblers are not trucks it means All trucks being tumblers is a possibility. Hence, conclusion I follows. Again, No train is a truck (E) + All trucks are toys (A) = E + A = O* = Some toys are not trains. Thus conclusion II follows. 3; Some voyages are journeys (I) + No journeys is a trekking (E) = I + E = O = Some voyages are not trekking. But conclusion I and II make a complementary pair of I - E. Thus, either I or II follows. 2; No brake is a belt (E) + All belts are boats (A) = E + A = O* = Some boats are not brakes. Hence, conclusion I does not follow. Again, All belts are boats (A) + All boats are

GovernmentAdda.com

2; All copies are pens (A) + All pens are markers (A) = A + A = A = All copies are markers (A) + (No paper is a marker

J

conversion

No marker is a paper (E) = A + E = E = No copy is a paper

J ) J

J

conversion No paper is a copy. Hence, conclusion II follows. Again, All pens are markers

+ No marker is a paper (E) = A + E = E = No pen is a paper Thus, conclusion I does not follow. 5; All plastics are chairs (A) + No chair is a table (E) = A + E

J

J

conversion No table is a = E = No plastic is a table plastic. Hence, conclusion I follows. No chair is a table (E) + Some tables are doors (I) = E + I O* = Some doors are not chairs. It means All chairs being doors is a possibility. Both conclusions I and II follow. 4; All plastics are chairs (A).+ No chair is a table (E) = A + E E = No plastic is a table (E) + Some tables are doors (I) E + I + O* = Some doors are not plastic. Hence, conclusion I does not follow. Again, No chair is a table (E) + Some tables are doors (I) = E + I = O* = Some doors are not chairs. Hence conclusion II does not follow. 201. 1; Some bags are laptops (I) + All laptops are mobiles (A) = I + A = I = Some bags are mobiles (I) + No mobile is a page (E) = I + E = O = Some bags are not pages. Hence, conclusion I follows. But, conclusion II does not follow. 202. 2; All laptops are mobiles (A) + No mobile is a page (E) = A + E = E = No laptop is a page. Hence conclusion I does not follow. But, Some bags are laptops means, All bags being laptops is a possibility. Hence, conclusion II follows. 203. 4; Some corridors are carroms (I) + No carrom is a corner

berries (A) = A + A = A = All belts are berries conversion II follows.

J

J

Some berries are belts. Hence, conclusion

1; All belts are berries (A). So, there is a possibility that All berries are belts. Hence, conclusion I follows. Again, Some boats are not brakes. Hence, All boats being brakes is not a possibility. Hence, conclusion II does not follow.

2; I- and O-type statements can’t be combined. Hence conclusion I does not follow. Again, since ‘tea’ is not related to either ‘table’ or ‘toy’ - the only terms that are negatively related - the possibility of II exists. 5; Since there is no negative statement, the possibilities of both conclusions I and II exist. 1; All red are racks (A) + No rack is a rod (E) = A + E = E =

J

J

No red is a rod (E) implication Some red are not rods. Hence conclusion I follows. Some rats are red (I) + All red are racks (A) = I + A = I - Some rats are racks. Thus, conclusion II also does not follow.

J

J

3; Some parrots are pigeons conversion Some pigeons are parrots (I) + No parrot is pink (E) = I + E = O = Some pigeons are not pink (O*). Hence, neither conclusion I nor conclusion II follows. But conclusion I and II form a complementary pair (I-E). Thus, either conclusion I or II follows.

J

J

conversion Some false are 5; Some rights are false (I) right (I) + All rights are wrong (A) = I + A = I = Some false are wrong (I). Hence, conclusion I follows. Since there is no negative statement, conclusion II also follows.

J

J

216. 2; Some bags are baggages (I) conversion Some baggage are bags (I) + All bags are buckets (A) = I + A = I = Some baggages are buckets. Hence, conclusion I does not follow. Some bags are baggages (I) + No baggage is a basket (E) = I + E = O = Some bags are not baskets (O). Hence,

[39] conclusion II follows. 217. 1; All papers are plastics (A) + All plastics are panels (A) = A

J

conversion + A = A = All papers are panels (A) Some panels are papers. Hence, conclusion I follows. J J

J

Again, All plastics are panels conversion Some panels are plastics. Hence, conclusion II does not follow.

J

J

218. 4; All glasses are gases conversion Some gases are glasses (I) + No glass is a goose (E) = 1 + E = O = Some gases are not geese. Hence, conclusion I does not follow. Some gases are glasses + Some glasses are not goggles = I + O = No conclusion. Hence conclusion II does not follow. 3; No toy is a tree (E) + All trees are trains (A) = E + A = O* Some trains are not toys (O*). Hence, conclusion I does not follow. Conclusion II also does not follow. But, conclusion I and II form a complementary (I - E) pair. Thus, either of the two conclusions I or II follows.

J

J

220. I; No toy is a tree conversion No tree is a toy (E) + Some toys are tough (I) = E + I = O* = Some tough are not trees (O*). Thus, there is a possibility that All trees are tough. Hence, conclusion I follows. Again, No toy is a tree (E) + All trees are trains (A) = E + A = O* = Some trains are not toys. So, there is no possibility that All trains are toys. Hence, conclusion II does not follow.

J

J

1; No track is a travel (E) conversion No’travel is a track (E). Now, All tours are travels (A) + No travel is a track (E) =

J

J

A + E = E = No tour is a track conversion No track is a tour. Hence, conclusion I follows. Again, All tours are travels (A) + Some travels are not trains (O)

A + O = No conclusion. Hence, conclusion II does not follow. 2; No track is a travel (E) + Some travels are not trains (O)

E + O = No conclusion. Hence, conclusion I does not follow. Again, All trousers are tracks (A) + No track is a travel (E) = A + E = E = No trouser is a travel

J

J

conversion

No travel is a trouser. Hence, conclusion II follows.,

230. 2; No bucket is a boat (E)

J

conversion

J

No boat is a bucket (E)

Now, Some blue are boats (I) + No boat is a bucket (E) = I + E = O = Some blue are not buckets. Hence, conclusion I does not follow. Again, No boat is a bucket (E) + Some buckets are black (I) = E + I = O* = Some black are not boats. Hence, All boats being black is a possibility. Thus, conclusion II follows. I; Since all statements are positive conclusion I follows. Since there is no negative statement, conclusion II does not follow. 5; Some roots are plants (I) + All plants are saplings (A) = I J J A = I = Some roots are saplings conversion saplings are roots. Hence, conclusion I follows.

Some

Again, No tree is a root (E) + Some roots are plants (I) = E + I = O* = Some plants are not trees. Hence, conclusion

follows. 4; Some plants are not trees. Hence, All plants being trees is not possible. Hence, conclusion I does not follow. Again, conclusion II does not follow from the third statement. 5; All books are novels (A) + No novel is a pen (E) = A + E = = No book is a pen. Hence, conclusion I follows. Again, No book is a pen (E) + All pens are pencils (A) = E

A = O* = Some pencils are not books. It means, All books being pencils is a possibility. Thus, conclusion II follows. 2; No novel is a pen (E) + All pens are pencils (A) = E + A = O* = Some pencils are not novels. It means, All novels being pencils is a possibility. Hence, conclusion II follows. Now, No book is a pen (E) + All pens are pencils (A) = E

A = O* = Some books are not pencils. Hence, conclusion I does not follow. J J

1; Some cats are dogs conversion Some dogs are cats. Now, All rats are dogs (A) + Some dogs are cats (I) = A+1

= No conclusion. Thus we can’t reach any definite conclusion by combining the statements. However, note that these is no negative statement. So II gets rejected outright while the possibility of conclusion I exists. 237. 1; Some postgraduates are girls (I) All girls are intelligent (A) = I + A = I = Some postgraduates are intelligent (I). Now, No boy is a postgraduate (E) + Some postgraduates are intelligents (I) = E + 1 = O* = Some intelligent are not boys. It means All boys being intelligent is a possibility. Hence, conclusion I follows. Again, No boy is a postgraduate (E) + Some postgraduates are girls (I) = E + I = O* = Some girls are not boys. Thus conclusion II does not follow. 2; Some postgraduates are girls (I) + All girls are intelligent

GovernmentAdda.com

1; Some books are boys (I) + All boys are baskets (A) = I + A I = Some books are baskets. Hence, All baskets being books is a possibility. Thus, conclusion I follows”.

J

J

Some books are boys conversion Some boys are books (I) + No book is a ball (E) = I + E ,= O = Some boys are not balls. Thus, All boys being balls is not a possibility. Hence, conclusion II does not follow. 224. 5; Some foods are forests (I) + All forests are frozen (A) = 1 + A = I?= Some foods are frozen. So, there is a possibility that some foods are not frozen. Thus, conclusion I follows. J J Again, All forests are frozen (A) conversion Some frozen are forests (I). Thus, conclusion II also follows.

J

J

1; All pipes are piders (A) conversion Some piders are pipes (I) + No pipe is a pyramid (E) = I + E = O = Some piders are not pyramids. Hence, conclusion I follows. Again, All pillars

J

J

conversion Some paints are pillars (I). are paints (A) Hence, conclusion II does not follow.

4; All keys are kites (A) + Some kites are carroms (I) = A + I = No conclusion. Thus, conclusion I does not follow. Again, Some kites are carroms (I) + No carrom is a colony

= I + E = O = Some kites are not colonies (O). Thus, conclusion II does not follow. 2; All states are countries (A) + No country is a town (E) = A + E = E = No state is a town (E) + Some towns are villages

= E + I = O* = Some villages are not states. Hence, conclusion I does not follow. J J

Again, No state is a town (E) implication not towns (O). Hence, conclusion II follows.

Some states are

228. 1; Since there is no negative statement, All docks being airports is a possibility. Hence, conclusion I follows. Again, All ports are airports (A) + Some airports are harbours (I) = No conclusion. Hence, conclusion, II does not follow. 229. 2; All riddles are puzzles (A) + No puzzle is difficult (E) = A + E = E = No riddle is difficult. Hence, conclusion I does not follow. Conclusion II follows from second statement.

= I + A = I = Some postgraduates are intelligent

J

J

conversion Some intelligent are postgraduates. Hence, conclusion II follows. But conclusion I does not follow.

J

J

1; No guide is a software conversion No software is a guide. Now, All applications are softwares (A) + No software is a guide (E) = A + E = E = No application is a guide. Hence-conclusion I follows. But conclusion II does not follow.

J

J

5; All metals are fibres conversion Some fibres are metals. Hence, conclusion I follows. Conclusion II follows because All wings are plastic. 1; Conclusion I follows from the first statement. Now, Some rectangles are frames + Some frames are circular = I + I = No conclusion. Hence, II does not follow. J J 1; All seniors are educated seniors.

conversion

Some educated are

Now, Some educated are seniors (I) + All seniors are experienced ( A) = I + A= I = Some educated are experienced. Hence, conclusion I follows. Conclusion II does not follow because of the second statement. 243. 4; No solution is wrong (E) + Some wrongs are right (I) = E + I = O* = Some rights are not solutions. Hence, conclusion II does not follow.

J

J

Again, No logic is right conversion No right is logic. Now, Some wrongs are right (I) + No right is logic (E) = I + E = O = Some wrongs are not logic. So, No solution is wrong (E) + Some wrongs are not logic

(o)’= E + O = No conclusion. Thus, conclusion I does not

[40] follow. 244. 1; Some squares are circles (I) + No circle is a triangle (E) = I + E = O = Some squares are not triangles. Hence, conclusion I follows. Again, No line is a square (E) + Some squares are circles (I) = E = I = O* = Some circles are not lines. Hence, conclusion II does not follow. 245. 4; Some squares are circles (I) + No circle is a triangle (E) = I + E = 0 = Some squares are not triangles. Hence, conclusion I does not follow. Again, No line is a square + Some squares are circles (I) = E + I = O* = Some circles are not lines. Hence, conclusion II does not follow. 5; All songs are poems (A) + All poems are rhymes (A) = A + A = A = All songs are rhymes. Now, All songs are rhymes

+ No rhyme is a paragraph (E) = A + E = E = No song is a paragraph. Hence, conclusion I follows. Again, All poems are rhymes (A) + No rhyme is a paragraph (E) = A + E = E = No poem is a paragraph. Hence, conclusion II follows. J J

2; All poems are rhymes conversion Hence, conclusion I does not follow.

Some rhymes are poems.

But both conclusions make a complementary pair (I-E). Hence, either conclusion I or II follows. 259. 2; All politicians are men (A) + Every man is wise (A) = A + A = A = All politicians are wise. Now, All politicians are wise (A) + Some wise are experienced (I) = A + I = No conclusion. Hence, conclusion I does not follow. But conclusion II follows from second and third statements. 260. 4; Conclusion I does not follow because it is a restatement. Again, Every man is wise (A) + Some wise are experienced

261. 2;

1;

Again, All songs are poems (A) + All poems are rhymes (A) = A + A = A = All songs are rhymes. Hence, conclusion

follows. 5; Some dews are drops (I) + All drops are stones (A) = I + A = I = Some dews are stones. It means at least Some dews are stones. Hence, conclusion I follows. Conclusion II follows by converting the second statement. 4; All books are magazines (A) + Some magazines are notebooks (I) = A + I = No conclusion. Hence, Conclusion

does not follow. Again, Some magazines are notebooks (I) + Some notebooks are papers (I) = I + I = No conclusion. Hence, Conclusion II does not follow. 250. 3; Some fruits are mangoes (I) + Some mangoes are red (I) I + I = No conclusion. However, either I or II follows as they form a complementary (I-E) pair. 5; All windows are doors (A) + All doors are boats (A) = A + A = A = All windows are boats. Hence, conclusion I follows. Again, All buildings are doors (A) + All doors are boats (A) =A + A = A = All buildings are boats. Hence, conclusion II follows. 1; Some sheets are roads (I). +All roads are marbles (A) = I + A = I = Some sheets are marbles. Hence, conclusion I

263. 2;

4;

(I) = A + I = No conclusion. Hence, conclusion II does not follow. No dancer is a singer (E) + Some singers are musicians (I) = E +1 = O* = Some musicians are not dancers. Hence, conclusion I does not follow. But a possible relation may exist between ‘dancers’ and ‘musicians’. Hence, conclusion II follows. All actors are dancers (A) + No dancer is a singer (E) = A E = E = No actor is a singer. Hence, conclusion II does not follow. Now, No actor is a singer (E) + Some singers Since there is no negative statement a negative conclusion can not exist. Again, All newspapers are books (A) + Some books are journals (I) =A+ I = No conclusion. But a possible relation exists between ‘newspapers’ and ‘journals’. Thus, conclusion II follows. All newspapers are books (A) + Some books are journals = A + I = No conclusion. Hence, conclusion I does not follow.

J

J

Again, Some newspapers are magazines conversion Some magazines are newspapers (I) + All newspapers are books (A) = I + A = I = Some magazines are books. Hence, conclusion II does not follow. 265. 5; All petals are flowers (A) + No flower is a colour (E) = A + E = E = No petal is a colour. Hence, conclusion I follows.

J

J

Again, All colours are fruits conversion Some fruits are colours. But the possibility in II exists. Hence conclusion II follows. 2; All petals are flowers (A) + No flower is a colour (E) = A + E = E = No petal is a colour. Hence, conclusion I does not follow. Again, No flower is a colour (E) + All colours are fruits (A) = E + A = O* = Some fruits are not flowers. But the possibility in II exists between ‘flower’ and ‘fruits’. Hence, conclusion II follows. J J

GovernmentAdda.com J

J

follows. Again, Some sheets are roads conversion Some roads are sheets (I) + No sheet is a roll (E) = I + E O = Some roads are not rolls. Hence, conclusion II does not follow. 4; 29% (some) caps are shoes (I) + 99% (some) shoes are pens (I) = I + I = No conclusion. Hence, conclusion I does not follow. 100% (All) heads are caps (A) + 29% (Some) caps are shoes (I) = A + I = No conclusion. Hence, conclusion II does not follow. 1; Some ties are belts (I) + All belts are shirts (A) = I + A = I Some ties are shirts. Hence a positive relation exists between ties and shirts. Thus, conclusion I follows.

J

J

Again, All belts are shirts conversion Some shirts are belts. Hence, conclusion II does not follow. 255. 5; Some ties are shirts (I) + No shirt is a T-shirt (E) = I + E = O = Some ties are not T-shirts. But All T-shirts being ties is a possibility. Hence, conclusion I follows. Again, All belts are shirts (A) + No shirt is T-shirt (E) = A + E = E = No belt is a T-shirt. Hence, conclusion II follows.

5; All crows are birds (A) + All birds are parrots (A) = A + A = A = All crows are parrots. Hence, conclusion I follows. Again, there is no negative statements. Hence, the possibility exist. Thus, conclusion II follows. J J

257. 4; All birds are parrots conversion Hence, conclusion I does not follow.

J

Some parrots are birds.

J

Again, Some birds are sparrows conversion Some sparrows are birds. Hence, conclusion II does not follow.

258. 3; Some knives are pins (I) + All pins are keys (A) = I + A = I = Some knives are keys (I) + No keyis a lock (E) = I + E = O = Some knives are not locks. Thus, conclusion I and II do not follow.

1; All dolls are toys conversion conclusion I follows.

Some toys are dolls. Hence,

Again, Some toys are gems (I) + Some gems are boxes (I) = I + I = No conclusion.. Hence, conclusion II does not follow. 5; Some nights are weeks (I) + All weeks are months (A) = I J J A = I = Some nights are months conversion months are nights. Hence, conclusion I follows.

Some

Again, Some days are nights (I) + Some nights are weeks (I) = I + I = No conclusion, but a possibility exists. Hence, conclusion II follows. 269. 5; There is no negative statement. Hence, conclusion I follows from the second and third statements. Again, conclusion II follows from first and third statements. Hence, conclusion I and II both follow. 270. 3; Some inputs are outputs (I) + All outputs are necessary (A) = I + A = I = Some inputs are necessary. Thus, conclusion I follows. Again, Some inputs are necessary (I) + No necessary is a result (E) = O = Some inputs are not results. Still, conclusion II follows. However, if we take the two conclusions together, All results are inputs + All inputs are necessary = A + A = A = All results are necessary. This contradicts the given statement. Hence, either I or follows.’ 1; All outputs are necessary (A) + No necessary is a result’ (E) = A + E = E = No output is a result. Hence, conclusion follows. Again, All outputs are necessary

J

J

conversion

Some necessary- are outputs. Thus, conclusion II does not follow.

[41] 5; All symbols are blanks (A) + All blanks are spaces (A) = A = All symbols are spaces. Hence, conclusion I follows. Again, there is no negative statement. Hence, the possibility exists between space and mark. Hence, conclusion II follows. J J 4; All blanks are spaces conversion . Thus, conclusion I does not follow.

Some spaces are blanks.

J

J

Again, Some blanks are marks conversion Some marks are blanks.,Hence, conclusion II does not follow.

274. 1; Some demands are public (I) + All public are central (A) = I + A = I = Some demands are central (I). Thus, conclusion I follows.

J

Again, Some demands are public conversion Some public are demands (I) + No demand is extensive (E) = 1 + E = 0 = Some public are not extensive. Thus, conclusion II does not follow. 275. 5; Some stones are rocks (I) + All rocks are mountains (A) I + A = I = Some stones are mountains. Hence, conclusion 1 follows. Again, there is no negative statement. Hence, the possibility in second statement exists. Hence, conclusion II follows. 2; All telephones are mirrors (A) + All mirrors are desks (A) A + A = All telephones are desks. Hence, conclusion I does not follow. Again, there is no negative statement. Hence the given possibility exists between radios and mirrors on the basis of first and second statements. Hence conclusion II follows. 1; No jungle is a road (E) + Some roads are hills (I) = E + I = O* = Some hills are not jungles. Hence, conclusion I follows. But conclusion II does not follow. 5; There is no negative statement. So the possibility in I exists. Hence; conclusion 1 follows. Again, All windows are boats (A) + All boats are doors (A)

J

J

= A + A = A = All windows are doors conversion Some doors are windows. Hence, conclusion II follows. J J

279. 1; No tree is a plant conversion conclusion I follows.

No plant is a tree. Hence,

Again, All houses are wheels (A) + Some wheels are trees (I) = A + I = No conclusion. Hence, conclusion II does not follow.

Again, Some states are villages (I) + No village is a forest (E) = I + E = O = Some states are not forests. Hence, conclusion II does not follow. J J

288. 2; No village is a forest conversion conclusion I does not follow.

J

No forest is a village. Hence,

Again, All forests are towns conversion forests. Hence, conclusion II follows.

J

Some towns are

289. 2; All butter is curd (A) + No curd is milkshake (E) = A + E = E = No butter is milkshake. Hence conclusion I does not follow,

J

Again, No curd is milkshake conversion is curd. Hence, conclusion II follows.

J

No milkshake

290. 1; All milk is butter (A) + All butter is curd (A) = A + A = A = All rnilk is curd. Sois the possibility in I exists. Hence, conclusion I follows. Again, All butter is curd (A) + No curd is milkshake (E) = A + E = E = No butter is milkshake. Hence, conclusion II does not follow. J J 291. 1; Some questions are answerable I conversion Some answerable are questions (I) + All questions are symbols

(A) = 1 + A = I = Some answerable are symbols. Hence, the possibility in 1 exists. Hence, conclusion I follows. But there is no negative statement. Thus any negative conclusion does not follow. Hence, conclusion II does not follow. 4; Some applications are easy I + All easy are dynamics (A) 1 + A = I = Some applications are dynamics. Hence, conclusion II does not follow. Again, No game is an application (E) + Some applications are dynamics (I) = E + 1 = O* = Some dynamics are not games. Hence, conclusion I does not follow.

2; Conclusion I does not follow from first statement. But the possibility in conclusion II exists from third statements. Hence, conclusion II follows. 2; Some details are beneficial (I) + All beneficial are helpful = I + A = I = Some details are helpful (1). Now, Some informations are not details (O) + Some details are helpful I = O + I = No conclusion. Hence, conclusion I does not follow. Again, the possibility in II exists from second statement. 295. 5; Some statements are paragraphs (I) + No paragraph is a chapter (E) = I + E = O = Some statements are not chapters. Hence, conclusion I follows.

GovernmentAdda.com J

J

1; No tiger is a cat conversion No cat is a tiger (E). Now, All animals are cats (A) + No cat is a tiger (E) = A + E = E = No animal is a tiger. Hence, conclusion I follows. But conclusion II does not follow. 1; Conclusion I follows because there is no negative statement. Again, Some villages are places (I) + Some places are cities (I) = No conclusion. Hence, II does not follow. 282. 5; All coals are diamonds + All diamonds are graphite = A + A = A = All coals are graphite (A) + No graphite is carbon

(E) = A + E = E = No coal is carbon. Hence, conclusion I follows. All diamonds are graphite (A) + No graphite is carbon ( E) = E = No diamond is carbon. Hence, conclusion II follows.

J

J

283. 2; All diamonds are graphite conversion Some graphites are diamonds. Hence, conclusion I does not follow. Again, All coals are diamonds (A) + All diamonds are graphite (A) = A + A = All coals are graphite. Hence, conclusion II follows. 284. 1; Some fruits are plants (I) + No plant is a tree (E) = I + E = O = Some fruits are not trees. Hence, conclusion I follows. Again, No flower is a fruit (E) + Some fruits are plants (I) E + I = O* = Some plants are not flowers. Hence, conclusion II does not follow. 1; Some doors are handles (I) + All handles are threads (A) I + A = I = Some doors are threads. Hence, conclusion I follows. Again, All handles are threads (A) + Some threads are windows (I) = A+1 = No conclusion. Hence, conclusion II does not folllow. 5; There are no negative statements. So, the possibilities exist. Hence, both conclusions I and II follow. 4; No village is a forest (E) + All forests are towns (A) = E + A = O* = Some towns arc not villages. Hence, conclusion I does not follow. .

Now, No word is a statement (E) + Some statements are paragraphs (I) = E + I = O* = Some paragraphs are not

J

J

conversion words. Again, No paragraph is a chapter No chapter is a paragraph (E) + Some paragraphs are not words (O) = E + O = No conclusion.

But the possibility in conclusion II exists. Hence, conclusion II follows. 296. 4; No building is white (E) + All whites are blacks (A) = E + A = O* = Some blacks are not buildings. Hence, conclusion I does not follow. Again, All whites are blacks (A) + Some blacks are huts I = A + 1 = No conclusion. Hence, conclusion II does not follow.

J

J

5; All whites are blacks conversion Some blacks are whites. But the possibility in 1 exists. Hence, conclusion I follows. Again, No building is white (E) + All whites are blacks (A) = E + A = O* = Some blacks are not buildings. Hence, conclusion II follows. 4; Some mangoes are apples (I) + Some apples are bananas I + I = No conclusion. Hence, conclusion I does not follow. Again, Some apples are bananas (I) + Some bananas are grapes (I) = I + 1 = No conclusion. Hence, conclusion

does not follow. 5; There is no negative statement. Hence, both conclusions and II follow. 4; Some rooms are windows (I) + All cards are windows

J

J

conversion

Some windows are cards (I) = I + I = No conclusion.

Hence, conclusion I does not follow. Again, All flowers are rooms (A) + Some rooms are windows (I) = A + I = No conclusion. Hence, conclusion II does not follow.

Daily Visit :

[GOVERNMENTADDA.COM]

100+ Order & Ranking Questions with Answers

GovernmentAdda.com

Governmentadda.com | IBPS SSC SBI RBI RRB FCI RAILWAYS

1

Daily Visit :

[GOVERNMENTADDA.COM]

If Anil finds that he is fourteenth from the left end and 7 from the right end, then how many boys must be added to the line such that there are 30 boys in the line? a) 8 b) 10 c) 12 d) 14 e) None of these Answer & Explanation Answer – b) 10 Explanation : 13 boys Anil 6 boys = 20 boys so number of boys to be added = 10 In a class of 90 students, numbers of boys are twice the number of girls. Rani is 58 th from the left end and there are 20 boys to the right of Rani, then the number of girls to the left of Rani? a) 15 b) 16 c) 17 d) 19 e) None of these Answer & Explanation Answer – c) 17 Explanation : Number of boys = 60 and girls are = 30 (57 students) Rani (20 boys) (12 girls) So number of girls to the left of Rani = 30 – 12 -1 = 17 In a row of 50 students, A is fourteenth from the left end and B is tenth from the right end. How many students are there in between A and C if C is eight to the left of B? a) 14 b) 16 c) 18 d) 20 e) None of these Answer & Explanation Answer – c) 18 Explanation : 13 students A (18 Students) C (7 students) B (9 students) A number of students are standing in a row facing north is such a way that a particular student is nineteenth from both the ends. So find the number of students in the class. a) 36 b) 37 c) 38 d) 39 e) None of these Answer & Explanation

Governmentadda.com | IBPS SSC SBI RBI RRB FCI RAILWAYS

2

Daily Visit :

[GOVERNMENTADDA.COM]

Answer – b) 37 Explanation : (18 students) BOY (18 students) = 18+18+1 = 37 In a row of 25 girls, when Neha was shifted by four places towards the left, she become 10th from the left end. What was her earlier position from the right end of the row? a) 10th b) 11th c) 12th d) 13th e) None of these Answer & Explanation Answer – c) 12th Explanation : (9 students) Neha ***(initial position) (11 students) In a queue, P is seventeenth from the front while Q is nineteenth form the last. If R is twenty forth from the front and is exactly in the middle of P and Q. Then find the number of people in the queue. a) 47 b) 48 c) 49 d) 50 e) None of these Answer & Explanation Answer – c) 49 Explanation : (16 people) P and Q (18 people). Since R is exactly in the middle and also 24th from the front so, number of people between P and R is 6. Similarly between R and Q is 6. So total people = 16 + P + 6 +R + 6 + Q+ 18 = 49 If A ranks seventh in the class. B is tenth from the last. If P is fifth after A and just in the middle of A and B, then how many students are there in the class? a) 26 b) 28 c) 24 d) 30 e) None of these Answer & Explanation Answer – a) 26 Explanation : (6 students) A (4 Students) P (4 students) B (9 students) = 26 In a row facing north, A is tenth to the left of B, who is 19th from the right end. If C who is 16th from the left end, is fourth to the right of A, how many girls are there in the row? a) 38 b) 40 c) 42

Governmentadda.com | IBPS SSC SBI RBI RRB FCI RAILWAYS

3

Daily Visit :

[GOVERNMENTADDA.COM]

d) 44 e) None of these Answer & Explanation Answer – b) 40 Explanation : (11 girls) A *** C (5 girls) B (18 girls) = 40 A is eight from the left end and B is sixteenth from the right end. C who is fourth to the right of A is sixth to the left of B. Find the total number of people in the row. a) 31 b) 32 c) 33 d) 34 e) None of these Answer & Explanation Answer – c) 33 Explanation : (7 people) A – – – C – – – – – B (15 people) = 33 In a row of 40 boys, when Rajesh was shifted to his left by 4 places his number from the left end of the row becomes 10. What was the number of Suresh from the right end of the row if Suresh was three places to the right of Rajesh’s original position? a) 23 b) 25 c) 26 d) 27 d) 21 e) None of these Answer & Explanation Answer – e) None of these Explanation : 24th (9 boys) Rajesh *** (original position) ** (Suresh) (23 boys) I. Directions(Q.No: 1-5)Study the following arrangement to answer the given questions Y#OU7D$V@BE8©X1AC%P24QIN6M*Z5

1. If all the numbers are dropped from the above arrangement, which of the following will be the ninth from the right end? 1.A 2.C 3.X 4.Y 5.O Governmentadda.com | IBPS SSC SBI RBI RRB FCI RAILWAYS

4

Daily Visit :

[GOVERNMENTADDA.COM]

Answer 2.C Explanation : Y#OU7D$V@BE8©X1AC%P24QIN6M*Z5 Ninth from the right end – C 2. Which of the following will be the sixth to the left of the fourteenth from the left end of the above arrangement? 1.Y 2.O 3.U 4.V 5.None of these Answer 4.V Explanation: 6th to the left of the 14th from the left end –> 8th from the left end Y#OU7D$V@BE8©X1AC%P24QIN6M*Z5 3. Which of the following is the fifth to the right of the thirteenth to the left of Q in the given arrangement? 1.X 2.@ 3.U 4.* 5.None of these Answer 1.X Explanation : Y#OU7D$V@BE8©X1AC%P24QIN6M*Z5 Governmentadda.com | IBPS SSC SBI RBI RRB FCI RAILWAYS

5

Daily Visit :

[GOVERNMENTADDA.COM]

13th to the left of Q – @ 5th to the right of – X 4. How many such vowels are there in the above arrangement, each of which is immediately preceded by a consonant and immediately followed by a number? 1.None 2.One 3.Two 4.Three 5.Four Answer 2.One Explanation : BE8 5. How many such symbols are there in the above arrangement, each of which is immediately preceded and also immediately followed by a consonant? 1.One 2.Two 3.Three 4.Four 5.None Answer 4Four Explanation : D$V, V@B, C%P, M*Z II. Directions(Q.No: 6-10) Study the following arrangement to answer the given questions R6J#L@3EDTAC*G©UM2FIV$4Z5Y

Governmentadda.com | IBPS SSC SBI RBI RRB FCI RAILWAYS

6

Daily Visit :

[GOVERNMENTADDA.COM]

6. If all the numbers are dropped from the above arrangement, which of the following will be the seventeenth from the right end? 1.E 2.L 3.I 4.C 5.@ Answer 5.@ Explanation : R6J#L@3EDTAC*G©UM2FIV$4Z5Y 7. Which of the following is the sixth to the left of the fifth to the left of “V”? 1.3 2.A 3.D 4.T 5.None of these Answer 4.T Explanation : sixth to the left of the fifth to the left of “V” – 11th to the left of V R6J#L@3EDTAC*G©UM2FIV$4Z5Y 8. Which of the following is the eighth to the right of the fourteenth from the left end of the above arrangement? 1.V 2.4 3.$ 4.E 5.None of these Governmentadda.com | IBPS SSC SBI RBI RRB FCI RAILWAYS

7

Daily Visit :

[GOVERNMENTADDA.COM]

Answer 3.$ Explanation : 8th to the right of 14th from the left – 6th from the left end – $ 9. Four of the following five are alike in a certain way based on their positions in the above arrangement and so form a group. Which is the one that does not belong to that group? 1.IM$ 2.CDG 3.3#D 4.UGC 5.GAU Answer 4.UGC 10. How many such symbols are there in above arrangement, each of which is immediately preceded by an alphabet and immediately followed by a number? 1.One 2.Two 3.Three 4.More than three 5.None Answer 2.Two Explanation : L@3, V$4

A is shorter than B but taller than C. D is taller than A. E is shorter than C. Who amongst the following is the tallest? a) A b) B c) D Governmentadda.com | IBPS SSC SBI RBI RRB FCI RAILWAYS

8

Daily Visit :

[GOVERNMENTADDA.COM]

d) Either B or D e) None of these Answer & Explanation Answer – d) Either B or D Explanation : B,D > A> C >E (we can’t say anything about B and D) In a row of 30 children, Mahesh is 12th from the left end. Rakesh a friend of Mahesh is 3 to the left of Mahesh. Find the position of rakesh from the left end. a) 8th b) 9th c) 10th d) 5th e) None of these Answer & Explanation Answer – b) 9th Explanation : (8 persons) (rakesh) ** Mahesh (18 person) A is fifteenth from the left end and B is eight from the right end. If there are 4 boys between them and B is to the right of A then the total number of student sitting in the row. a) 26 b) 27 c) 28 d) 29 e) None of these Answer & Explanation Answer – b) 27 Explanation : (14 person) A **** B (7 persons) P is fifteenth from the left end in a row of boys and Q is eighteenth from the right end. If R is tenth from P towards the right end and fourth from Q towards right end. How many boys are there in the row? a) 35 b) 36 c) 38 d) 40 e) None of these Answer & Explanation Answer – c) 38 Explanation : (14 person)P * * * * * Q * * * R (13 person) Akash is 5 ranks above sumit in a class of 30. If sumit rank is 15th from the last. What is akash rank from the start? a) 10 b) 11 Governmentadda.com | IBPS SSC SBI RBI RRB FCI RAILWAYS

9

Daily Visit :

[GOVERNMENTADDA.COM]

c) 12 d) 13 e) None of these Answer & Explanation Answer – b) 11 Explanation : (10 students) AKASH * * * * sumit (14 person) In a row of girls facing north, A is fourteenth from the left and B is seventeenth from the right. C who is third to the right of A is also sixth to the left of B in the row. How many girls are there in the row? a) 36 b) 37 c) 38 d) 39 e) None of these Answer & Explanation Answer – d) 39 Explanation : (13 person) A * * C * * * * * B (16 person) In a group of 6 students P, Q, R, S, T and U each one having different height. P is taller than T but not as tall as U. Q and U are not the tallest and also R is the shortest. Who is the tallest among them? a) P b) S c) Q d) U e) None of these Answer & Explanation Answer – b) S Explanation : S > (Q,U) > P > T > R There are 40 students in the class. Priya ranks 6th in the class among the girls and pankaj ranks 5th among the boys in the class. Priya is 2 ranks below pankaj in the class. Find the rank of pankaj in the class. a) 8 b) 9 c) 10 d) 11 e) None of these Answer & Explanation Answer – b) 9 Explanation : GGGGG priya and BBBB pankaj. (4 boys and 4 girls above pankaj)Pankaj * priya, so rank is 9th

Governmentadda.com | IBPS SSC SBI RBI RRB FCI RAILWAYS

10

Daily Visit :

[GOVERNMENTADDA.COM]

In a rows of students, P is 12th from the left end and Q is 16th from the right end. If they interchange their position then Q becomes 23rd from the right end. Find the number of students in the row. a) 32 b) 33 c) 34 d) 35 e) None of these Answer & Explanation Answer – c) 34 Explanation : s(11 students) P ………..Q (15 students). After they interchange their position, 11 students Q (22 students) – so total students = 11 + 1 + 22 = 34 If amit finds that it is seventeenth from the right and eighteenth from the left in line facing north. How many persons should be added to the line such that there are 40 people in the line. a) 5 b) 6 c) 8 d) 9 e) None of these Answer & Explanation Answer – b) 6 Explanation : 17 + Amit + 16 + X = 40, X = 6 I. Directions(Q.No: 1-5)Study the following arrangement to answer the given questions R#TU3D$J@BE9©W1AF%P24QIN6M*Z5

1. If all the numbers are dropped from the above arrangement, which of the following will be the ninth from the right end? 1.A 2.F 3.W 4.R 5.T Answer 2.F Explanation :

Governmentadda.com | IBPS SSC SBI RBI RRB FCI RAILWAYS

11

Daily Visit :

[GOVERNMENTADDA.COM]

R#TU3D$J@BE9©W1AF%P24QIN6M*Z5 Ninth from the right end – F 2. Which of the following will be the sixth to the left of the fourteenth from the left end of the above arrangement? 1.R 2.T 3.U 4.J 5.None of these Answer 4.J Explanation: 6th to the left of the 14th from the left end –> 8th from the left end R#TU3D$J@BE9©W1AF%P24QIN6M*Z5 3. Which of the following is the fifth to the right of the thirteenth to the left of Q in the given arrangement? 1.W 2.@ 3.U 4.* 5.None of these Answer 1.W Explanation : R#TU3D$J@BE9©W1AF%P24QIN6M*Z5 13th to the left of Q – @ 5th to the right of – W

Governmentadda.com | IBPS SSC SBI RBI RRB FCI RAILWAYS

12

Daily Visit :

[GOVERNMENTADDA.COM]

4. How many such vowels are there in the above arrangement, each of which is immediately preceded by a consonant and immediately followed by a number? 1.None 2.One 3.Two 4.Three 5.Four Answer 3.Two Explanation : TU3, BE9 5. How many such symbols are there in the above arrangement, each of which is immediately preceded and also immediately followed by consonant? 1.One 2.Two 3.Three 4.More than four 5.None Answer 4.More than four Explanation : R#T, D$J, J@B, F%P, M*Z II. Directions(Q.No: 6-10) Study the following arrangement to answer the given questions Q9K#P@3ENSAC*G©UM7FIV%4Z8Y

6. If all the numbers are dropped from the above arrangement, which of the following will be the seventeenth from the right end? 1.E 2.P Governmentadda.com | IBPS SSC SBI RBI RRB FCI RAILWAYS

13

Daily Visit :

[GOVERNMENTADDA.COM]

3.I 4.C 5.@ Answer 5.@ Explanation : Q9K#P@3ENSAC*G©UM7FIV%4Z8Y 7. Which of the following is the sixth to the left of the fifth to the left of “V”? 1.3 2.A 3.N 4.S 5.None of these Answer 4.S Explanation : sixth to the left of the fifth to the left of “V” – 11th to the left of V Q9K#P@3ENSAC*G©UM7FIV%4Z8Y 8. Which of the following is the eighth to the right of the fourteenth from the left end of the above arrangement? 1.V 2.4 3.% 4.E 5.None of these Answer

Governmentadda.com | IBPS SSC SBI RBI RRB FCI RAILWAYS

14

Daily Visit :

[GOVERNMENTADDA.COM]

3.% Explanation : 8th to the right of 14th from the left – 6th from the left end – % 9. Four of the following five are alike in a certain way based on their positions in the above arrangement and so form a group. Which is the one that does not belong to that group? 1.IM% 2.CNG 3.3#N 4.UGC 5.GAU Answer 4.UGC 10. How many such symbols are there in above arrangement, each of which is immediately preceded by an alphabet and immediately followed by a number? 1.One 2.Two 3.Three 4.More than three 5.None Answer 2.Two Explanation : P@3, V%4 In a row of girls facing north, neha is 9th to the left of niharika, who is 19thfrom the right end. If nisha who is 15th from the left end is 3rd to the right of neha, how many girls are there in the row? a) 34 b) 37 c) 39 d) 41 e) None of these Governmentadda.com | IBPS SSC SBI RBI RRB FCI RAILWAYS

15

Daily Visit :

[GOVERNMENTADDA.COM]

Answer & Explanation Answer – c) 39 Explanation : (11 girls) Neha – – Nisha (5 girls) Niharika (18 girls) 11 girls means before neha there are 11 girls, 5 girls means 5 girls between nisha and niharika and similarly 18 girls In a class of 75 students, the number of girls are twice the number of boys, Pankaj ranked 19th from the top. If there are 10 girls ahead of pankaj, then the number of boys after him in rank. a) 15 b) 16 c) 17 d) 18 e) None of these Answer & Explanation Answer – b) 16 Explanation : Number of girls are 50 and number of boys are 25. If 10 girls ahead of pankaj, means only 8 boys are ahead of him so number of boys after him = 25 – 8 -1 = 16 Sumit is 8th rank ahead of ravi in a class of 45. If ravi rank from bottom is 19 ththen find the rank of sumit from beginning? a) 17 b) 18 c) 19 d) 20 e) None of these Answer & Explanation Answer – c) 19 Explanation : Number of students between sumit and ravi is 7. Number of students after ravi is 18th. So rank of sumit from beginning = 45 – 7 – 18 – 1 (ravi) = 19th In a row of students facing north A is fifteenth from the left end and B is seventh from the right end. If they interchange their positions, B would be 17thfrom the right end. Find the number of students in the row. a) 30 b) 31 c) 32 d) 33 e) None of these Answer & Explanation Answer – b) 31 Explanation : Initially there are 14 students to the left of A and after changing the position, there are 16 students to the right of B so total students = 14 +16 +1 = 31

Governmentadda.com | IBPS SSC SBI RBI RRB FCI RAILWAYS

16

Daily Visit :

[GOVERNMENTADDA.COM]

In a row of 21 boys when akash is shifted four places to the right, he becomes 12 th from the left end. What was akash earlier position from the right end a) 10 b) 11 c) 13 d) 14 e) None of these Answer & Explanation Answer – d) 14 Explanation : after shifting – (11 students) akash (9 students) Before shifting – (7 students) akash (13 students) A number of students are standing in a row facing north is such a way that a particular student is nineteenth from both the ends. So find the number of students in the class. a) 36 b) 37 c) 38 d) 39 e) None of these Answer & Explanation Answer – b) 37 Explanation : (18 students) BOY (18 students) = 18+18+1 = 37 Prakash is 10 ranks above Nikhil who ranks 26th in the class of 45. What is prakash rank in the class from the beginning? a) 14 b) 15 c) 16 d) 17 e) None of these Answer & Explanation Answer – c) 16 Explanation : Final arrangement => (15 students) prakash (9 students) Nikhil In a queue, P is seventeenth from the front while Q is nineteenth form the last. If R is twenty forth from the front and is exactly in the middle of P and Q. Then find the number of people in the queue. a) 47 b) 48 c) 49 d) 50 e) None of these Answer & Explanation Answer – c) 49 Explanation : (16 people) P and Q (18 people). Since R is exactly in the middle and also 24th from the front so, number Governmentadda.com | IBPS SSC SBI RBI RRB FCI RAILWAYS

17

Daily Visit :

[GOVERNMENTADDA.COM]

of people between P and R is 6. Similarly between R and Q is 6. So total people = 16 + P + 6 +R + 6 + Q+ 18 = 49 If arun finds that it is seventeenth from the right and eighteenth from the left in line facing north. How many persons should be added to the line such that there are 50 people in the line. a) 15 b) 16 c) 18 d) 21 e) None of these Answer & Explanation Answer – b) 16 Explanation : 17 + Arun + 16 + X = 50, X = 16 A is eight from the left end and B is sixteenth from the left end. C who is fourth to the right of A is sixth to the left of B. Find the total number of people in the row. a) 31 b) 32 c) 33 d) 34 e) None of these Answer & Explanation Answer – c) 33 Explanation : (7 people) A – – – C – – – – – B (15 people) = 33 Directions(Q.No: 1-5)Study the following arrangement to answer the given questions

F 8Z*Q A $ K 2 P 5 E © N 1 W @ Y3M # 6 & V %9 RU !4D

1. How many such vowels are there in the above arrangement, each of which is immediately preceded by consonant and immediately followed by symbol ? 1.One 2.Two 3.Three 4.More than four 5.None Answer & Explanation

Governmentadda.com | IBPS SSC SBI RBI RRB FCI RAILWAYS

18

Daily Visit :

[GOVERNMENTADDA.COM]

Answer – 2.Two Explanation : Q A $, RU ! 2. If all the symbols are dropped from the above arrangement, which of the following will be the eighteenth from the left end? 1.9 2.R 3.V 4.3 5.6 Answer & Explanation Answer – 3.V Explanation : F 8ZQ A K 2 P 5 E N 1 W Y3M 6 V 9 R U 4D 3. Four of the following five are alike in a certain way based on their positions in the above arrangement and so form a group. Which is the one does not belong to that group? 1.2Q8 2.15K 3.31E 4.631 5.963 Answer & Explanation Answer – 4.631 Explanation : 2 -4 Q -3 8 1 -4 5 -3 K 3 -41 -3 E 6 -4 Y -3 1

Governmentadda.com | IBPS SSC SBI RBI RRB FCI RAILWAYS

19

Daily Visit :

[GOVERNMENTADDA.COM]

4. Which of the following is the seventh to the right of the fourteenth from the right end of the above arrangement? 1.V 2.R 3.% 4.9 5.U Answer & Explanation Answer – 3.% Explanation : 14 R – 7 R = 7 R F8Z*QA$K2P5E©N1W@Y3M#6&V%9RU!4D 5. Which of the following is fourth to the left of the eighth to the right of ‘W’? 1.6 2.3 3.M 4.E 5.None of these Answer & Explanation Answer – 3.M Explanation : 4th to the left of the 8th to the right of ‘Z’ –>4th to the right of W – i.e “M” Direction: Q(6 –10) Answer the following questions based on the arrangement given below:586 425 632 364 218 813

6. What will be the resultant number if the second digit of the second lowest number is divided by the third digit of the highest number? 1.2 2.3 Governmentadda.com | IBPS SSC SBI RBI RRB FCI RAILWAYS

20

Daily Visit :

[GOVERNMENTADDA.COM]

3.5 4.10 5.None of these Answer & Explanation Answer – 1.2 Explanation : The highest number is 813. The third digit of the highest number is 3. The second lowest number is 364. The second digit of the second lowest number is 6. Now, the required resultant = 6 ÷ 3 = 2. 7. If the position of the first and second digits are interchanged then what is the difference between the highest and second lowest number? 1.420 2.582 3.673 4.545 5.None of these Answer & Explanation Answer – 3.673 Explanation : 856 245 362 634 128 183 856 – 183 = 673 8. If all the digits in each of the numbers are arranged in ascending order within the number, what will be the difference between the second highest and second lowest number? 1.218 2.220 3.208

Governmentadda.com | IBPS SSC SBI RBI RRB FCI RAILWAYS

21

Daily Visit :

[GOVERNMENTADDA.COM]

4.225 5.None of these Answer & Explanation Answer – 3.208 Explanation : 568 245 236 346 128 138 346 – 138 = 208 9. If 1 is added to the first digit and 2 is subtracted from the last digit of each of the numbers and then the digits in each of the numbers are arranged in descending order within the number, then what will be the difference between the second digit of the second highest number and the third digit of the lowest number? 1.2 2.4 3.6 4.8 5.10 Answer & Explanation Answer – 2.4 Explanation : 1 is added to 1st digit and 2 sub from last digit :684 523 730 462 316 911 Descending Order :864 532 730 642 631 911 Difference = 6 – 2 = 4 10. If 1 is added to the last digit of each of the numbers then how many numbers thus formed will be divisible by three?

1.One 2.Two 3.Three

Governmentadda.com | IBPS SSC SBI RBI RRB FCI RAILWAYS

22

Daily Visit :

[GOVERNMENTADDA.COM]

4.Four 5.More than four Answer & Explanation Answer – 3.Three Explanation : 587 426 633 365 219 814 426, 633, 219 are divisible by three. I. Directions(Q.No: 1-5)Study the following arrangement to answer the given questions D3M%RA$K2P5E©N4W@F8Z#6&V*9QU!1Y

1. If all the symbols are dropped from the above arrangement, which of the following will be the seventeenth from the left end? 1.U 2.F 3.V 4.9 5.6 Answer 5.6 Explanation : D3MRAK2P5EN4WF8Z6V9QU1Y Seventeenth from the left end – 6 2. Which of the following is fourth to the left of the eighth to the left of ‘Z’? 1.D 2.3 3.K 4.R 5.None of these Answer-4.S Governmentadda.com | IBPS SSC SBI RBI RRB FCI RAILWAYS

23

Daily Visit :

[GOVERNMENTADDA.COM]

Answer 3.K Explanation: 4th to the left of the 8th to the left of ‘Z’ –> 12th to the left of Z – i.e “K” D3M%RA$K2P5E©N4W@F8Z#6&V*9QU!1Y 3. Which of the following is the eighth to the right of the fourteenth from the right end of the above arrangement? 1.5 2.E 3.R 4.9 5.P Answer 4.9 Explanation : 8th to the right of the 14th from the right end —> 6th from the right end D3M%RA$K2P5E©N4W@F8Z#6&V*9QU!1Y 4. Four of the following five are alike in a certain way based on their positions in the above arrangement and so form a group. Which is the one does not belong to that group? 1.54# 2.DAE 3.2©N 4.@#9 5.$5@ Answer 4.@#9 Explanation : 5 -4 4 +6 # Governmentadda.com | IBPS SSC SBI RBI RRB FCI RAILWAYS

24

Daily Visit :

[GOVERNMENTADDA.COM]

D -4 A +6 E 2 -4 © +6 N @ -4 # +5 9 $ -4 5 +6 @ 5. How many such vowels are there in the above arrangement, each of which is immediately preceded by consonant and immediately followed by symbol ? 1.One 2.Two 3.Three 4.More than four 5.None Answer 2.Two Explanation : R A $, Q U ! II. Directions(Q.No: 6-10) Study the following arrangement to answer the given questions E3M%RA$L8O5F©I4G@W7Z#6&B*9QU41S

6. Which of the following is third to the left of the ninth to the right of ‘I’? 1.6 2.# 3.9 4.Q 5.None of these Answer 5.None of these Explanation : 3th to the left of the 9th to the right of ‘I’ –> 6th to the right of I –> Z E3M%RA$L8O5F©I4G@W7Z#6&B*9QU41S Governmentadda.com | IBPS SSC SBI RBI RRB FCI RAILWAYS

25

Daily Visit :

[GOVERNMENTADDA.COM]

7. Which of the following is the eighth to the right of the fourteenth from the left end of the above arrangement? 1.6 2.# 3.9 4.Q 5.None of these Answer 1.6 Explanation : 8th to the right of the 14th from the left end – 22nd from left end E3M%RA$L8O5F©I4G@W7Z#6&B*9QU41S 8. How many such numbers are there in the above arrangement, each of which is immediately preceded by a vowel and immediately followed by a consonant? 1.One 2.Two 3.Three 4.More than four 5.None Answer 3.Three Explanation : E 3 M, O 5 F, I 4 G 9. What should come in the following series based on the above arrangement ? 3$5 ©@Z ? 1.E3M 2.6*U 3.6*Q

Governmentadda.com | IBPS SSC SBI RBI RRB FCI RAILWAYS

26

Daily Visit :

[GOVERNMENTADDA.COM]

4.691 5.None Answer 3.6*Q 10. Insert “T” after every third letter in the above series then what will be the 19 thletter from the right end of the series ? 1.W 2.T 3.@ 4.7 5.None Answer 1.W Explanation : E3MT%RAT$L8TO5FT©I4TG@WT7Z#T6&BT*9QTU41TS 19th letter from the right end – W

In a row of 20 girls, Shruti is sitting 5th from left end of the row and is also sitting 10th to left of Kareena. Pankhuri is sitting 8th from the right end of row. How many girls are sitting between Pankhuri and Kareena? A) None B) 1 C) 5 D) 3 E) 6 Answer & Explanation B) 1 Explanation: Shruti is 5th from left end, and Kareena is 10th to right of Shruti, so Kareena is 15th from left end or 6th from right end of row. Pakhuri is 8th from right end. So there is 1 girl between them. In a class of 45 students, Veena is placed at 31th position from the bottom and Kashish is 4 places above Veena. If Surbhi is 5 places above Kashish, then what is the rank of Surbhi in the class? A) 3 B) 5 Governmentadda.com | IBPS SSC SBI RBI RRB FCI RAILWAYS

27

Daily Visit :

[GOVERNMENTADDA.COM]

C) 6 D) 7 E) None of these Answer & Explanation C) 6 Explanation: Veena is 31 from bottom, Kashish 4 places above so she is 35 from bottom. Surbhi is 40 from bottom, so 6th from top Rita is sitting 5th from the left end of row and Sita is 11th to right of Rita with Tina being 4th to left of Sita. Madhuri is 8th to right of Tina. What is the total number of students in the row if Madhuri is sitting at the extreme end? A) 12 B) 20 C) 28 D) 23 E) 33 Answer & Explanation B) 20 Explanation: Rita is 5th from left, Sita 11th to right of Rita, so 16th from left end, Tina is 4th to left of Sita so Tina is 12th from left end. Now Madhuri is 8th to right of Tina, this means 20th from left, so 20 students Karuna is sitting 25th from the left end and Preeti is sitting 26th from the right end. Preeti is at 20th to the left of Karuna. What is the total number of students sitting in the row? A) 28 B) 30 C) 21 D) 32 E) 26 Answer & Explanation B) 30 Explanation: Karuna is 25th from left end and Preeti is 20th to left of Karuna so Preeti is 5th from left end and given 26th from right end, so total = (5+26) – 1 In a class of 20 students, Shreya is 5 from the top and Annie is 7 ranks below Shreya. Find Annie’s rank from bottom. A) 3 B) 5 C) 6 D) 9 E) None of these Answer & Explanation D) 9 Explanation: Annie is 12 ranks from top, so from bottom = 20-12 + 1

Governmentadda.com | IBPS SSC SBI RBI RRB FCI RAILWAYS

28

Daily Visit :

[GOVERNMENTADDA.COM]

In a row of 30 children, A is 11th from the right end of row. If there are 4 children between A and B, What is the position of B from the left end of row? A) 4 B) 6 C) 5 D) 8 E) Cannot be determined Answer & Explanation E) Cannot be determined Explanation: Since it is not given that B is left of A or right of A, cant be determined. Prerna is 5th from the left end and Charu is 4th from the right of row. Charu interchanges her position with the one who is sitting 3rd to the right of Prerna and now Charu is 10th from the right end. How many children are there in the row? A) 17 B) 18 C) 20 D) 15 E) 16 Answer & Explanation A) 17 Explanation: Let A is 3rd to right of Prerna, so Charu comes to his place so now charu is (5+3) = 8th from the left end and also she is 10th from the right end, so in total (8+10)-1 Garima interchanges her position with the one who is 3 places away from Garima. Now Chinu is 5th to right of Garima and is 3rd from the right of the row. What is the position of Garima from the right end of row? A) 9 B) 8 C) 10 D) 7 E) Data inadequate Answer & Explanation B) 8 Explanation: It is not given that Garima interchanges her position with left person or right person from her. So take with both the cases, we will get same answer. Shikha is 10th from the top in a class with Ruhani being 16th from the bottom. If there are 5 students between Shikha and Ruhani, how many total students are there in the class when no two students share the same rank? A) 30 B) 31 C) 35 D) Data inadequate E) None of these Governmentadda.com | IBPS SSC SBI RBI RRB FCI RAILWAYS

29

Daily Visit :

[GOVERNMENTADDA.COM]

Answer & Explanation D) Data inadequate Explanation: It is not given that Shikha is below Ruhani or above in rank, if we take both cases we get different answers so cant be determined. Ranveer is 5th from left end of row and Ranbir is 6th from right of row. If they interchange their positions, Ranbir becomes 16th from the right end. What is the total number of people in the row? A) 21 B) 22 C) 20 D) 24 E) Data inadequate Answer & Explanation C) 20 Explanation: Ranbir becomes 16th from right and also this place is 5th from left end, so total (16+5) – 1

Direction: Q(1 –5) Answer the following questions based on the arrangement given below:235 762 198 438 623 911

1. If the position of the first and second digits are interchanged then what is the difference between the highest and second lowest number? a) 555 b) 655 c) 455 d) 755 e) None of these Answer & Explanation Answer – b) 655 Explanation : 325 672 918 348 263 191 so, difference = 918 – 263 = 655 2. If all the digit in the numbers are written in reverse order then which number is the third largest number? a) 762 Governmentadda.com | IBPS SSC SBI RBI RRB FCI RAILWAYS

30

Daily Visit :

[GOVERNMENTADDA.COM]

b) 135 c) 235 d) 623 e) None of these Answer & Explanation Answer – c) 235 Explanation : 532 267 891 834 326 119 3. If all the numbers are arranged in ascending order then, then position of how many numbers remains unchanged? a) none b) one c) two d) three e) None of these Answer & Explanation Answer – b) one Explanation : only 911 4. If the digits of all numbers are added then which number is the largest among them? a) 235 b) 762 c) 198 d) 911 e) None of these Answer & Explanation

Governmentadda.com | IBPS SSC SBI RBI RRB FCI RAILWAYS

31

Daily Visit :

[GOVERNMENTADDA.COM]

Answer – c) 198 Explanation : 198 = 18 highest among all 5. If 100 is subtracted from all the numbers and then the number obtained are written in reverse order, then the lowest number isa) 235 b) 198 c) 911 d) 623 e) None of these Answer & Explanation Answer – c) 911 Explanation : After subtracting hundred we get – 135 662 098 338 523 811 And after arranging in reverse order531 266 890 833 325 118 118 – Lowest number so answer is 911 Q(6-10) Study the following pattern and answer the questions below: –

1WE3$RT%M94LSC&FJ2@UP7D5*0ZC?

6. How many such numbers are there which are immediately followed by letter and immediately preceded by symbol? a) more than three b) one b) two d) three e) None of these Answer & Explanation Governmentadda.com | IBPS SSC SBI RBI RRB FCI RAILWAYS

32

Daily Visit :

[GOVERNMENTADDA.COM]

Answer – b) one Explanation : *0Z 7. How many such symbols are there which are immediately followed by number and immediately preceded by letter? a) none b) one c) two d) three e) more than three Answer & Explanation Answer – a) none Explanation : No such arrangement is possible 8. If all the numbers and vowels are dropped in the above pattern then which is 9th form the left end? a) C b) L c) S d) & e) None of these Answer & Explanation Answer – a) C Explanation : W$RT%MLSC&FJ @ P D * ZC? 9. Which among the following is third to the right of ninth from the right end? a) * b) D Governmentadda.com | IBPS SSC SBI RBI RRB FCI RAILWAYS

33

Daily Visit :

[GOVERNMENTADDA.COM]

c) 5 d) 0 e) None of these Answer & Explanation Answer – c) 5 Explanation : 1WE3$RT%M94LSC&FJ2@UP7D5*0ZC? Ninth from the right end = P, so third to the right of P = 5 10. Four of the following are alike in some manner, find the odd one? a) 1EW b) 50* c) M49 d) SCF e) PD7 Answer & Explanation Answer – d) SCF Explanation : +2 and then -1 pattern is followed

Q(1 – 5) Answer the following questions based on the arrangement given below:YX%4@CD6PSA9$βN*L2#BM+7δ5GF8

1. How many such numbers are there which are immediately followed by a symbol and immediately preceded by letter? a) one b) two c) three d) more than three e) None of these Governmentadda.com | IBPS SSC SBI RBI RRB FCI RAILWAYS

34

Daily Visit :

[GOVERNMENTADDA.COM]

Answer & Explanation Answer – b) two Explanation : A9$ and L2# 2. If all the symbols are dropped from the above arrangement, then which is ninth from the right end? a) L b) N c) B d) 2 e) None of these Answer & Explanation Answer – a) L Explanation : after symbols are removed- YX4CD6PSA9NL2BM75GF8 3. If every third element in the above series from the left end is omitted then which among the following is third to the left of seventh from the right end? a) S b) A c) β d) $ e) None of these Answer & Explanation Answer – c) β Explanation : Y X 4 @ D 6 S A $ β * L # B + 7 5 G 8 after omission. 4. If all the numbers immediately after symbols are multiplied by the number itself then what is the sum of these number? Governmentadda.com | IBPS SSC SBI RBI RRB FCI RAILWAYS

35

Daily Visit :

[GOVERNMENTADDA.COM]

a) 70 b) 80 C) 90 d) 100 e) None of these Answer & Explanation Answer – C) 90 Explanation : 4,5,7 are such numbers. So addition = 16 + 49 + 25 = 90 5. How many such vowels are there which is immediately followed by consonants and immediately preceded by number? a) none b) one c) two d) three e) more than three Answer & Explanation Answer – a) none Explanation : there is no such vowel Q(6 – 10) Answer the following questions based on the arrangement given below:A3$MK2@L94%SE#6J1L*X7βBZδ8R

6. How many such symbols are there which is immediately followed by number and immediately followed by letter a) none b) one c) two

Governmentadda.com | IBPS SSC SBI RBI RRB FCI RAILWAYS

36

Daily Visit :

[GOVERNMENTADDA.COM]

d) three e) Four Answer & Explanation Answer – c) two Explanation : E#6 and Zδ8 7. How many numbers are there which is immediately followed by letter and preceded by symbol? a) none b) two c) three d) Four e) more than three Answer & Explanation Answer – b) two Explanation : #6J and δ8R 8. Which among the following is fifth to the right of the ninth from the left end? a) 6 b) S c) E d) # e) None of these Answer & Explanation Answer – d) # Explanation : Ninth from the left end = 9, so fifth to the right of 9 = #

Governmentadda.com | IBPS SSC SBI RBI RRB FCI RAILWAYS

37

Daily Visit :

[GOVERNMENTADDA.COM]

9. What will come in place of question mark? A$2 M29 @9S 4S6 ? a) E6L b) E#L c) EL6 d) #J* e) None of these Answer & Explanation Answer – a) E6L Explanation : +2 and then +3 within word and +3 between two successive pair like A and M in A$2 and M29 10. If all the numbers are dropped from the above sequence, then which element is eleventh from the right end? a) S b) E c) # d) * e) None of these Answer & Explanation Answer – b) E Explanation : A$MK@L%SE#JL*XβBZδR

Governmentadda.com | IBPS SSC SBI RBI RRB FCI RAILWAYS

38

Daily Visit

[GOVERNMENTADDA.COM]

New & Old Pattern Coding Decoding GovernmentAdda.com

GovernmentAdda.com | IBPS SBI RBI SSC FCI RRB RAILWAYS SSB

1

Daily Visit

[GOVERNMENTADDA.COM]

Directions (1-5): Study the following information to answer the given questions: In a certain code, ′Media Letter Reply Profile′ is written as ′Simple Camp Views Much′ is written as ′Improve Tool Worker Draft′ is written as ′Output Hundred List Greater′ is written as

′18e% ′21e$ ′6t@ ′10d%

14r$ 24s@ 25r$ 17t$

20y@ 5p# 11e% 9r%

15a@′ 15h#′ 22l#′ 14t#′

1. What is the code for ‗Post Adverts‘? A) 18t@ 3s% B) 18t@ 3s$ C) 18t# 3s% D) 16t# 3s$ E) Cannot be determined View Answer

Option C Solution: Symbol is according to number of letters in word 4 – #, 5 – @, 6 –$, 7 – % Number is = (number represented by first letter of word) + 2 Letter – last letter of word So Profile – P = 16, 16 +2 = 18, 7 letter word so % and last letter e – so code is 18e% 2. What is the code for ‗Green Signal‘? A) 21l@ 9n@ B) 21l$ 8n# C) 22l# 9n@ D) 21l$ 9n@ E) Cannot be determined View Answer

Option D 3. What is the code for ‗Delete Image‘? A) 6e# 9e$ B) 6e$ 11e@ C) 6e@ 11e# D) 8e@ 11e$ E) Cannot be determined View Answer

Option B

GovernmentAdda.com | IBPS SBI RBI SSC FCI RRB RAILWAYS SSB

2

Daily Visit

[GOVERNMENTADDA.COM]

4. What is the code for ‗Share Content‘? A) 7t% 21e@ B) 3t% 21e% C) 5t% 21e# D) 5t% 21e@ E) Cannot be determined View Answer

Option D 5. What is the code for ‗Current Quiz‘? A) 7t% 19z# B) 5t% 19z# C) 5t% 19z# D) 5t% 20z# E) Cannot be determined View Answer

Option B Directions (6-10): Study the following information to answer the given questions: In a certain code, ‗Session Passive Area Happy‘ is written as ‗Stimulus Hut Puzzle Herbal‘ is written as ‗Affirm Sound Particular Account‘ is written as ‗Admire Potential Helmet Swap‘ is written as

‗@a2 &n4 ‗&s4 %e3 ‗@t4 &d3 ‗&p2 !t3

!y3 !l3 %r5 %l5

%e4‘, !t2 ‘, @m3‘, and @e3‘.

6. What is the code for ‗Approximate‘? A)#l6 B) %e5 C) @e5 D) @e6 E) Cannot be determined View Answer

Option D & for first letter s, % for first letter p, @ for first letter a, and ! for first letter h Number represents number of alphabets in word divided by 2. And is it comes in decimal, the number is rounded off. Example crucial= 7 = 7/2 = 4. sour = 4 = 4/2 = 2 Alphabet represents the last letter of word. So Passive = % for P, e for last letter e and 4 for passive= 7 = 7/2 = 4

GovernmentAdda.com | IBPS SBI RBI SSC FCI RRB RAILWAYS SSB

3

Daily Visit

[GOVERNMENTADDA.COM]

7. What is the code for ‗Humble Adequate‘? A) !e3 @e5 B) !e4 &e4 C) %e3 @e5 D) !e3 @e4 E) Cannot be determined View Answer

Option D

8. What is the code for ‗Safety Proportional‘? A) %a6 !y3 B) %l6 &y3 C) !l5 &y3 D) %l6 !y4 E) Cannot be determined View Answer

Option B remedy – @d3 9. What is code for ‗Part Habitat‘? A) %t3 !t5 B) %t2 !a5 C) %t2 !t4 D) %t3 !t4 E) Cannot be determined View Answer

Option C

10. Which of the following will represent ‗Abroad Setting‘? A) @d3 !g3 B) @d3 &g4 C) !d3 &g4 D) @d3 !g5 E) Cannot be determined View Answer

GovernmentAdda.com | IBPS SBI RBI SSC FCI RRB RAILWAYS SSB

4

Daily Visit

[GOVERNMENTADDA.COM]

Option B Directions (1-5): Study the following information to answer the given questions: In a certain code, ‗Condition Publish Media Paragraph‘ is written as ‗Feedback Setting Show Pages‘ is written as ‗Table Number Prefer College‘ is written as and ‗Game Split Urban Links‘ is written as

‗OM36 AS36 EZ20 US28‘, ‗AH20 ET28 HD16 EP32‘, ‗RI24 UI24 AV20 OV28‘, ‗AV16

IH20

RM20 PG20‘.

1. Which is the code for ‗Letter Insert‘? A) EI28 NG28 B) EI24 NG28 C) EI28 NG24 D) EI24 NG24 E) None of these View Answer

Option D Explanation: Numbers: Number of letters in word multiplied by 4 First letter in code is second letter of respective word. Second letter in code is reverse of last letter of respective word. So Condition – O for second letter O, M for reverse of N, 9 letter word so 9*4 = 36 makes OM36 2. What is the code for ‗Preview Draft‘? A) RG24 RD28 B) RG24 RE28 C) RG20 RD28 D) RF20 RE28 E) None of these View Answer

Option C 3. What is the code for ‗Finance Content‘? A) IW28 OG24 B) IV28 OG28 C) IV24 OG28 D) FV28 OG28 E) None of these View Answer

GovernmentAdda.com | IBPS SBI RBI SSC FCI RRB RAILWAYS SSB

5

Daily Visit

[GOVERNMENTADDA.COM]

Option B 4. What is the code for ‗Discussion Zone‘? A) IN40 OV12 B) IM36 OV16 C) IM40 OV16 D) OV40 OV12 E) None of these View Answer

Option C 5. Which is the code for ‗Featured Aptitude‘? A) EW36 PW32 B) PV32 FW36 C) QV36 EW32 D) PV32 EW32 E) None of these View Answer

Option D Directions (6-10): Study the following information to answer the given questions: In a certain code, ‗Civilized Aroma Priceless Outlook‘ is written as ‗Media Break Emphasis Thousand‘ is written as ‗Remains Camera Blank Download‘ is written as and ‗Invalid Umbrella Embark Fireless‘ is written as

‗N%24 ‗R#21 ‗B#21

Y@12 Z$12 P%18

‗ S@21

D%21

M$18 A#24‘, K@12 C%21‘, A@15 Z$12‘, C$15

G#18‘.

6. Which is the code for ‗Malicious Streak‘? A) P$15 K#24 B) Q$15 K%24 C) R$15 K@24 D) Q$15 K%24 E) Cannot be Determined View Answer

Option D Explanation: Observe all numbers are multiple of 3. Number: (Number of letters in word – 1) * 3 GovernmentAdda.com | IBPS SBI RBI SSC FCI RRB RAILWAYS SSB

6

Daily Visit

[GOVERNMENTADDA.COM]

Observe 4 symbols are used and there are words ending in 4 letters only – u , r, d, i So d – #, a – @, s – %, k – $ Alphabet: first letter of word – 2. So Civilized will be coded as: C – C-2 => A, last letter d => #, and number of letters in 9 so (9-1) * 3 = 24 So A#24 7. What is the code for ‗Outwalk Discard‘? A) C#18 M@18 B) B#18 M$18 C) C#15 M@18 D) B#15 M$18 E) Cannot be determined View Answer

Option B 8. What is the code of ‗Formula Surplus‘? A) D@18 Q%18 B) E@21 Q#18 C) D@18 Q%21 D) E@21 Q#18 E) Cannot be determined View Answer

Option A 9. What is the code of ‗Phobia Educated‘? A) N$18 D#21 B) N@15 C#21 C) M@15 C#24 D) N$15 D#21 E) Cannot be determined View Answer

Option B 10. Which is the code for ‗Compass Replica‘? A) B%18 P@15 B) Z%18 Q@15 C) A%18 P@18 D) A%15 P@18 GovernmentAdda.com | IBPS SBI RBI SSC FCI RRB RAILWAYS SSB

7

[GOVERNMENTADDA.COM]

Daily Visit

E) Cannot be determined View Answer

Option C Directions (1-5): Study the following information to answer the given questions: In a certain code ‗Basic material is available‘ is written as ‗de kl ce dp‘, ‗Basic questions are solved‘ is written as ‗sa kc bk de‘, ‗Almost questions available solved‘ is written as ‗dp fc bk ‗are material good enough‘ is written as ‗kl ts kc mt‘.

sa‘ and

1. Which of the following is the code for ‗is are‘? A) kl dp B) sa fc C) ce de D) ce kc E) None of these View Answer

Option D 2. Which of the following can be coded as ‗mt‘? A) material B) good C) are D) enough E) enough or good View Answer

Option E 3. If ‗questions make difference‘ is coded as ‗cl km sa‘, then which of the following is the code for ‗solved are available‘? A) kc bk sa B) dp mt kl C) bk kc dp D) de fc dp E) bk ce fc View Answer

GovernmentAdda.com | IBPS SBI RBI SSC FCI RRB RAILWAYS SSB

8

Daily Visit

[GOVERNMENTADDA.COM]

Option C questions or solved is ‗sa‘ or ‗bk‘. From given question, questions – ‗sa‘, so solved – ‗bk‘ 4. Which of the following is the code for ‗basic good enough‘? A) kl ts mt B) mt de ts C) mt ke de D) ts kc dp E) Either A or B View Answer

Option B 5. What can be the code of ‗almost material unique‘? A) fc bk op B) fc de kl C) kl op mt D) fc kl jf E) kl ts sa View Answer

Option D unique and ‗jf‘ not present anywhere in codes – so unique can be coded as ‗jf‘ almost – fc, material – kl Directions (6-10): Study the following information to answer the given questions: In a certain code, ‗people are earning money‘ is written as ‗ck pd ta sp‘, ‗companies are paying more‘ is written as ‗kl sp ps qr‘, ‗earning with more money‘ is written as ‗pd kl sa ck‘ and ‗getting work paying money‘ is written as ‗op ck al ps‘. 6. What is the code for ‗more money‘? A) kl sa B) ck kl C) op ta D) sa ck E) Cannot be determined View Answer

GovernmentAdda.com | IBPS SBI RBI SSC FCI RRB RAILWAYS SSB

9

Daily Visit

[GOVERNMENTADDA.COM]

Option B 7. Which of the following can be coded as ‗sa‘? A) with B) earning C) getting D) paying E) with or earning View Answer

Option A 8. Which could be the code for ‗people work hard‘? A) ta ck hj B) sa kl op C) al pd ta D) Cannot be determined E) ta op hk View Answer

Option E there – na, meaning – tif, code for title is not given, so it can be given any code other than present in above given coding 9. What is the code for ‗are with paying‘? A) ck pd ps B) sp pd ck C) kl sp ps D) sp ps sa E) None of these View Answer

Option C 10. Which is the code for ‗companies‘? A) qr B) sp C) ta D) kl E) pd

GovernmentAdda.com | IBPS SBI RBI SSC FCI RRB RAILWAYS SSB

10

Daily Visit

[GOVERNMENTADDA.COM]

View Answer

Option A Directions (1-5): Study the following information to answer the given questions: In a certain code ‗Rain is coming again‘ is written as ‗hi pa le ter‘, ‗again always water‘ is written as ‗hi mne ki‘, ‗water is coming‘ is written as ‗pa le mne‘ and ‗it coming always‘ is written as ‗pa ki riz‘. 1. Which of the following can be coded as ‗pa ter ki le‘? A) always coming water is B) water is again always C) rain is always coming D) Rain is water always E) None of these View Answer

Option C 2. Which of the following is the code for ‗always water‘? A) le pa B) hi ki C) mne riz D) ki mne E) None of these View Answer

Option D 3. How will ‗water is again‘ be written in this code? A) le hi mne B) le ter mne C) pa mne riz D) ki hi mne E) mne ter riz View Answer

Option A 4. Which of the following may represent ‗it‘? A) le GovernmentAdda.com | IBPS SBI RBI SSC FCI RRB RAILWAYS SSB

11

Daily Visit

[GOVERNMENTADDA.COM]

B) mne C) pa D) ter E) riz View Answer

Option E 5. What does ‗le ta‘ represent in the code? A) Rain always B) coming it C) always is D) water coming E) is rain View Answer

Option E Directions (6-10): Study the following information to answer the given questions: In a certain code, ‗many ideas are there‘ is written as ‗fo kla fp na‘, ‗there format are also same‘ is written as ‗jo hke kla si na‘, ‗many name same meaning‘ is written as ‗ya si fp tif‘ and ‗name with there format‘ is written as ‗na ya go hke‘. 6. What does ‗hke‘ stand for? A) also B) are C) there D) name E) format View Answer

Option E 7. What is the code for ‗ideas‘? A) fo B) kla C) fp D) na E) Either fp or na GovernmentAdda.com | IBPS SBI RBI SSC FCI RRB RAILWAYS SSB

12

Daily Visit

[GOVERNMENTADDA.COM]

View Answer

Option A 8. Which could be the code for ‗there title meaning‘? A) tif kla na B) jo si ya C) na tif df D) fp ya hke E) ya ki tif View Answer

Option C there – na, meaning – tif, code for title is not given, so it can be given any code other than present in above given coding 9. What is the code for ‗many‘? A) na B) hke C) fp D) fo E) kla View Answer

Option C 10. Which is the code for ‗name are also same‘? A) ya kla hke si B) jo Si ya Ia C) si jo hke na D) fp ya hke kla E) ya si jo kla View Answer

Option E Directions (1-5): Study the following information to answer the given questions: In a certain code, ‗superstring park avail prelim‘ is written as ‗cereal revolving crook newsroom‘ is written as ‗training system unequal monk‘ is written as ‗payback dream using ethical‘ is written as

‗#22T %8Q $10B @12Q‘, ‗#10S @16O $12D %10D‘, ‗$14V %bN @12T #16U‘, and ‗%14Q $14F #10V @10E‘.

GovernmentAdda.com | IBPS SBI RBI SSC FCI RRB RAILWAYS SSB

13

Daily Visit

[GOVERNMENTADDA.COM]

1. What is the code for ‗shooting upstream‘? A) @16V %16T B) @16V #18T C) @16V #16T D) @16U %16T E) Cannot be determined View Answer

Option C Solution: Symbols are according to last letter in words: g – #, k – %, l – $, m – @ Number represents double the number of letters in words Alphabet represents the next letter in alphabetical series to first letter of word. So superstring — # for last letter g, 22 for 2*11 (11 letters in superstring), and T for next letter to s So superstring – #22T 2. What is the code for ‗making framework‘? A) #12N %18G B) #12M $18G C) #12M %20G D) #12N @18G E) Cannot be determined View Answer

Option A

3. What is the code for ‗logical coding‘? A) @12M $12E B) #14M $12D C) @14M $12C D) #14M $12D E) Cannot be determined View Answer

Option B 4. What is code for ‗environmental petroleum‘? A) @18Q $24F B) @16Q $26E C) @18Q $26F GovernmentAdda.com | IBPS SBI RBI SSC FCI RRB RAILWAYS SSB

14

[GOVERNMENTADDA.COM]

Daily Visit

D) #16Q $26E E) Cannot be determined View Answer

Option C 5. What is code for ‗maximum cashback‘? A) #14N %16D B) @16N %16D C) @14N %16E D) @14N %16D E) Cannot be determined View Answer

Option D Directions (6-10): Study the following information to answer the given questions: In a certain code, ‘letter found option result’ is written as ‘input sneak never final’ is written as ‘travel pirate height office’ is written as ‘spark mutual inward quantum’ is written as

‘w#6 m#15 i#12 g@18’, ‘o#6 p@19 i@14 g#9’, ‘g#8 v@16 o@20 v@15’, and ‘w#9 n@17 o#13 p@19’.

6. What is the code for ‘store gallery’? A) u#19 b#8 B) v@18 c#7 C) u@18 d#7 D) v@19 b#7 E) Cannot be determined View Answer

Option D Solution: There are only 2 symbols in the above coding. They have been set according to first letter of word. Words having numerical value less than equal to 13 are kept in one group and are denoted by #. and from 14-26 by @. (i.e. for first half letter (a-m) – #, and for last 13 letters (n-z) – @ Number represents the numerical value of first letter Alphabet represents the reverse of last letter according to their number in alphabetical series, a = 26, b = 25, …….z = 1 So letter — r -> i, # for l and 12 for l GovernmentAdda.com | IBPS SBI RBI SSC FCI RRB RAILWAYS SSB

15

Daily Visit

[GOVERNMENTADDA.COM]

So letter – i#12 7. What is the code for ‘inshorts phone’? A) g#9 v@18 B) h#9 v@16 C) h@10 v@18 D) h#10 v#16 E) Cannot be determined View Answer

Option B

8. What is the code for ‘message internet’? A) v@13 g#8 B) v#13 g#9 C) v#12 g@8 D) v#13 g#8 E) Cannot be determined View Answer

Option B 9. What is code for ‘setting clock’? A) p#13 t@19 B) p#3 t@18 C) p#3 t@19 D) p@3 t#19 E) Cannot be determined View Answer

Option C 10. What is code for ‘word money’? A) w@24 c#14 B) v@22 b#13 C) w@24 b#13 D) w#24 c@13 E) Cannot be determined View Answer

GovernmentAdda.com | IBPS SBI RBI SSC FCI RRB RAILWAYS SSB

16

Daily Visit

[GOVERNMENTADDA.COM]

Option C Directions (1-5): Study the following information to answer the given questions. ‗to make search down‘ is written as ‗es vst re pt‘ ‗normal in city make‘ is written as ‗ch dsr mo vst‘ ‗normal to head on‘ is written as ‗re dqv ch gi‘ and ‗head refer to make‘ is written as ‗re gi vst wey‘ 1. What is the code for ‗city‘ in the given code language? A) mo B) wey C) ch D) Either ‗dsr‘ or ‗mo‘ E) Other than those given as options View Answer

Option D 2. In the given code language, what does the code ‗pt‘ stand for? A) head B) Either ‗search‘ or ‗down‘ C) city D) make E) Either ‗city‘ or ‗in‘ View Answer

Option B 3. What may be the code for ‗Make us‘ in the given code language? A) dqv iq B) iq gi C) iq vst D) gi es E) vst dqv View Answer

Option C 4. What is the code for ‗to‘ in the given code language? A) mo B) vst GovernmentAdda.com | IBPS SBI RBI SSC FCI RRB RAILWAYS SSB

17

Daily Visit

[GOVERNMENTADDA.COM]

C) gi D) dqv E) re View Answer

Option E 5. If ‗head on wheels‘ is coded as ‗dqv wz gi‘ in the given code language, then what is the code for ‗Refer on wheels‘? A) wz ch es B) dqv wz wey C) wey mo wz D) vst es wz E) ch wey vst View Answer

Option B Directions (6-10): Study the following information to answer the given questions In a certain code language, ‗format head for footer‘ is written as ‗kop mi bo ge‘, ‗refer for style footer‘ is written as ‗seo li ge mi‘ ‗footer space to give‘ is written as ‗ge frw wr ct‘ ‗give and refer head‘ is written as ‗bo seo ct lko‘ 6. What is the code for ‗give‘ in the given code language? A) seo B) ge C) ct D) mu E) None of the Above View Answer

Option C 7. In the given code language, what does ‗and‘ stands for? A) seo B) ge C) ct D) mu GovernmentAdda.com | IBPS SBI RBI SSC FCI RRB RAILWAYS SSB

18

Daily Visit

[GOVERNMENTADDA.COM]

E) lko View Answer

Option E 8. What is the code for ‗footer‘ in the given code language? A) seo B) ge C) ct D) mu E) lko View Answer

Option B 9. What is the code for ‗space‘ in the given code language? A) Either seo or ct B) ge C) Either frw or wr D) mu E) other than those given as options View Answer

Option C 10. What may be the possible code for ‗Refer heading‘ in the given code language? A) seo frw B) ge seo C) ct bo D) seo mu E) None of the Above View Answer

Option D Directions (1-5): Study the following information to answer the given questions: In a certain code language ‘heading and footer style’ is written as ‘gh sif wms st’ ‘footer of document writable’ is written as ‘qui qwe ak gh‘ GovernmentAdda.com | IBPS SBI RBI SSC FCI RRB RAILWAYS SSB

19

Daily Visit

[GOVERNMENTADDA.COM]

‘along style document readable’ is written as ‘dj wms qwe er‘ ‘readable to revive heading’ is written as ‘dh ht dj st‘ 1. What does the code ‘qui’ stand for in the given code language? A) and B) to C) along D) Either ‘of’ or ‘writable’ E) document View Answer

Option D 2. Which of the following may possibly represent ‘revive to’ in the given code? A) qwe ht B) bt qwe C) ht co D) dh ht E) dh dj View Answer

Option D 3. What is the code for ‘heading’ in the given code language? A) st B) Either ‘sif’ or ‘wms’ C) gh D) dj E) sif View Answer

Option A 4. What is the code for ‘style’ in the given code language? A) st B) gh C) wms D) ak E) qwe View Answer

GovernmentAdda.com | IBPS SBI RBI SSC FCI RRB RAILWAYS SSB

20

Daily Visit

[GOVERNMENTADDA.COM]

Option C 5. Which of the following may represent ‘readable copied footer’ in the given code language? A) fs gh dj B) dj sif gh C) gh er st D) gh fs sif E) xs dj ak View Answer

Option A Directions (6-10): Study the following information to answer the given questions: In a certain code, ‘group factor content over’ is written as ‘distant heart prove listen’ is written as ‘goal noble policy discount’ is written as ‘narrow express journal sign’ is written as

‘#p20 %t24 ‘@n15 %t23 ‘@y11 $l20 ‘%s22 %l17

@r21 $r12’, #e11 #t19’, %t23 #l13’, and @w13 $n8’.

6. What is the code for ‘best coding’? A) %t25 @g23 B) %t23 @g24 C) $t25 @g24 D) $t25 #g24 E) Cannot be determined View Answer

Option C Solution: Symbols are number of digits in word 4 – $, 5 – #, 6 – @, 7 – % Number represents the number represented by first letter but from reverse Alphabet represents the last letter of word. So content – 7 letters so %, t for last letter t, c is 24 from reverse in z – a So content- %t24 7. What is the code for ‘update draft’? A) %e6 #t25 B) @e6 #t23 C) @f6 #d23 D) #e6 #t24 GovernmentAdda.com | IBPS SBI RBI SSC FCI RRB RAILWAYS SSB

21

[GOVERNMENTADDA.COM]

Daily Visit

E) Cannot be determined View Answer

Option B

8. What is the code for ‘preview status’? A) %w11 @s9 B) %w10 @s8 C) %w12 @s9 D) %w11 @s8 E) Cannot be determined View Answer

Option D 9. What is code for ‘public feature’? A) @c11 %e21 B) @c12 %e22 C) %c12 #e21 D) %d11 %e21 E) Cannot be determined View Answer

Option A 10. What is code for ‘browse trash’? A) @e24 #h8 B) @e25 #h8 C) @e24 #h7 D) @e25 #h7 E) Cannot be determined View Answer

Option D Directions (1-5): Study the following information to answer the given questions: In a certain code, ′Team Made Best Solve′ is written as ′Solve Keep Test Team′ is written as

′P17 C21 K28 N22′ ′M18 C21 N22 J20′

GovernmentAdda.com | IBPS SBI RBI SSC FCI RRB RAILWAYS SSB

22

Daily Visit

[GOVERNMENTADDA.COM]

′Best Beep Wait Node′ is written as ′Wait Taste Care Main′ is written as

′K28 W15 B12 K20′ ′S17 F23 W15 D27′

1. What can be the code for ‗Wait Merit‘? A) C21 W15 B) W15 D27 C) N22 J20 D) C21 D27 E) None of these View Answer

Option E Wait – W15 Merit can be given any code other than already given above. But all options contain the above codes 2. Which of the following can be used to find out the code for ‗Solve‘? (i) C21 K28 M18 D27 (ii) P17 K20 C21 K28 (iii) C21 K20 S17 N22 (iv) C12 N22 J20 K28 A) (ii) or (iii) or (iv) B) (ii) or (iv) C) (i) or (ii) or (iv) D) (ii) or (iii) E) None of these View Answer

Option C Explanation: From 1st 2 lines Solve – is either C21 or N22, so if we know the words for codes given in (1), (ii) and (iv) [in which either C21 or N22 is present – not both], code for Solve can be found 3. Which of the following cannot be used to find the code for ‗Node‘? A) K20 F23 J13 L09 B) C21 B12 W15 L22 C) P23 B12 M12 N12 D) B12 K28 K20 M18 E) M18 K20 N22 O34 View Answer

Option D Node can be B12 or K20. and these both are present in D) option so if we get to know the words for code in D), then also we cannot find code for Node GovernmentAdda.com | IBPS SBI RBI SSC FCI RRB RAILWAYS SSB

23

[GOVERNMENTADDA.COM]

Daily Visit

4. If ‗Care‘ is coded as ‗F23‘, then what can be used to find code for ‗Taste‘? A) D27 P17 S17 W15 B) H17 M19 D27 L25 C) G19 E22 D27 S17 D) S17 C22 D27 L04 E) M18 S17 D27 C21 View Answer

Option B If Care – F23, then Taste will be D27 or S17, so the one code in which only one is present can be used to find code for Taste. 5. If the code for ‗Beep is ‗B12‘, then which of the following can be used to find the code for ‗Node Test‘? A) K20 M18 B) J20 M18 C) J20 K20 D) P17 K20 E) A or C View Answer

Option E If Beep – B12, then Node will be K20 Test is M18 or J20 Directions (6-10): Study the following information to answer the given questions: In a certain code, ‗example hijack picnic classroom‘ is written as ‗recheck puzzle excuse hygienic‘ is written as ‗victim jumble lock chronic‘ is written as ‗hardwork terrific beam prism‘ is written as

‗!k3 %v3 $x4 #s3 ‘, ‗!s4 #i3 %v3 %k3‘, ‗%q3 #o2 !x3 $e3 ‘, and ‗$y2 #s4 $k2 !g4‘.

6. What is the code for ‗proclaim analytic‘? A) $j4 !z4 B) $k4 !z4 C) %k4 !y4 D) #k4 !z4 E) Cannot be determined View Answer

Option B Solution: Symbols are last letter e – %, k – #, c – !, m – $ GovernmentAdda.com | IBPS SBI RBI SSC FCI RRB RAILWAYS SSB

24

Daily Visit

[GOVERNMENTADDA.COM]

Number represents number of alphabets in word divided by 2. And if it comes in decimal, the number is rounded off to previous integer. Example example= 7 = 7/2 = 3. hijack = 6 = 6/2 = 3 Alphabet represents the reverse of first letter. So example – % for last letter e, reverse of e is v and 7 letter word, 7/2 = 3 So example – %v3 7. What is the code for ‗ringworm outkick‘? A) $i4 #l3 B) $h4 #l4 C) $i3 #k3 D) !i4 #l3 E) Cannot be determined View Answer

Option A

8. What is the code for ‗autism displace‘? A) $z4 %w3 B) $z3 %x4 C) $y3 %w4 D) $z3 %w4 E) Cannot be determined View Answer

Option D 9. What is code for ‗pacific struck‘? A) !k3 %h3 B) $j3 #h3 C) !k3 #h3 D) !k3 #i3 E) Cannot be determined View Answer

Option C 10. What is code for ‗invasive turmeric‘? A) !d3 %r4 B) !g4 %r4 C) !j4 %r3 GovernmentAdda.com | IBPS SBI RBI SSC FCI RRB RAILWAYS SSB

25

[GOVERNMENTADDA.COM]

Daily Visit

D) !g4 !y4 E) Cannot be determined View Answer

Option B invasive – %r4 turmeric – !g4 Directions (1-5): Study the following information to answer the given questions: In a certain code, ′Oject Indefinite Interbank Login′ is written as ′I@11 E%14 E$17 J#11′ ′Park Abatement Edible Green ′ is written as ′D@18 F#7 U$3 E%9′ ′Countercheck Carbon Illicit Pipeline′ is written as ′H#18 E%5 G$11 L@5′ ′Expectant Uncollectible Panick Hometown′ is written as ′I$7 F@18 H%10 M#23′ 1. What will be the code for ‗Absorption Mixture‘? A) H#15 J#3 B) G@15 J%3 C) G@15 J$3 D) G#15 J%3 E) None of these View Answer

Option D Explanation: There are 4 symbol, and observe that each line contains 4 words, one each ending with either t or e or k or n In 1st line — There are 2 codes with 11 in it and 2 words starting with I, I = 9, 9+2 = 11, check if that makes sense in other words also. It makes O = 15, 15+2 = 17, which is in E$17 So with this get the symbol and number. t – $, e – #, k – @, n – % Next letter: There are two E‘s in 1st line, 1 each in 2nd and 3rd lines of code. also 2 words with 5 letters in 1st line, and 1 each in 2nd and 3rd line, and E = 5. Check with others. So Indefinite – end with e so #, I = 9, 9+2 = 11, and 10 letter word, J = 10 So Indefinite – J#11 2. What will be the code for ‗Unopen Celebrative‘? A) K#5 F%23 B) I#5 F#23 C) L#5 F@23 D) J#5 F%23 GovernmentAdda.com | IBPS SBI RBI SSC FCI RRB RAILWAYS SSB

26

Daily Visit

[GOVERNMENTADDA.COM]

E) None of these View Answer

Option A 3. What will be the code for ‗Maiden Timework‘? A) E%16 H@22 B) F%15 H#22 C) F%15 H@22 D) F%16 H#22 E) Cannot be determined View Answer

Option C 4. What will be the code for ‗Trademark Economist‘? A) IJ7 I@22 B) I$7 I@22 C) J$7 I%22 D) I$7 I$22 E) None of these View Answer

Option B 5. What will be the code for ‗Fastest A) H$9 N#15 B) H$7 N%14 C) G$8 N%15 D) G$7 N#15 E) Cannot be determined

Monopolisation‘?

View Answer

Option C Directions (6-10): Study the following information to answer the given questions: In a certain code language ―title makes art beautiful‖ is written as ―makes baby very happy‖ is written as

‗ds wdr ms te‘ ‗gi te fid ho‘

GovernmentAdda.com | IBPS SBI RBI SSC FCI RRB RAILWAYS SSB

27

Daily Visit

[GOVERNMENTADDA.COM]

―Very title Based stories‖ is written as ―Based like beautiful picture‖ is written as

‗ms opr ho je‘ and ‗cb ds dr opr‘

6. What is the code for ‗Based baby happy‘ in the given code language? A) gi fid ms B) opr ms gi C) ho opr gi D) dr ds wdr E) fid opr gi View Answer

Option E 7. What is the code for ‗stories‘ in the given code language? A) opr B) Other than those given in options C) je D) ms E) dr View Answer

Option C 8. If ‗art very risk‘ is coded as ‗ho wdr zy‘ in the given code language, then how will ‗title risk beautiful‘ be coded as? A) ho dr ds B) ds ms dr C) zy ms ho D) wdr zy ds E) Other than those given in options View Answer

Option E 9. Which of the following may represents the code ‗makes improve‘ in the given code language? A) le dr B) cs ds C) ds nq D) cs te E) dr cs View Answer

GovernmentAdda.com | IBPS SBI RBI SSC FCI RRB RAILWAYS SSB

28

Daily Visit

[GOVERNMENTADDA.COM]

Option D 10. In the given code language, what does the code ‗cb‘ stands for? A) makes B) very C) either ‗picture‘ or ‗like‘ D) either ‗beautiful‘ or developed E) title View Answer

Option C Directions (1-5): Study the following information to answer the given questions: In a certain code, ′Produce Land Shares Total′ is written as ′World Puzzles Estate Favour′ is written as ′Matter Quicker Sale First′ is written as ′Gate Owner Complex Meet′ is written as

′H1% ′K1@ ′U2# ′N2$

O4$ U9% H5$ T5$

K5@ D4# N9% L9#

G3#′ V5%′ J9@′ X6@′

1. What will be the code for ‗Goods Service‘? A) T2# G5@ B) T1$ G5@ C) T1# H5@ D) T2# H5% E) None of these View Answer

Option C Explanation: In 2nd and 4th code there are 2 codes with % and $ respectively. And also there are 2 words with 6 and 4 letters in these respectively. So by this it can be found that symbols are used for no. of letters Symbols for number of letters: 4 – $, 5 – #, 6 – %, 7 – @ For letters: Reverse in English Alphabet For numbers – see that each lines of code contains 1 code with 5 digit – K5@, V5%, H5$, T5$ . So find relation between words to find the pattern There is one word in each line having ‗e‘ letter at last of word. and e is 5. Check for others too if it works. So you get is that the last digits of number of last letter are added to obtain number in coding. So Puzzles – P -> K, S = 19, 1+9=10 =1+0 = 1 and 7 letters in word so @ So Puzzles – K1@ 2. What will be the code for ‗Major Share‘? A) N2% G5# GovernmentAdda.com | IBPS SBI RBI SSC FCI RRB RAILWAYS SSB

29

[GOVERNMENTADDA.COM]

Daily Visit

B) M9% H4# C) N2# G5# D) N9# H5# E) None of these View Answer

Option D 3. What will be the code for ‗Download‘? A) W4@ B) X4@ C) T4@ D) V4@ E) Cannot be determined View Answer

Option E Download is 8 letter word, so no symbol for 8 letter word is specified 4. What will be the code for ‗Free Games‘? A) U1$ U5# B) T1# U5$ C) S2% U5$ D) T1% V4$ E) None of these View Answer

Option B 5. What will be the code for ‗Angry Birds‘? A) Y1# Z7# B) Y2$ X6# C) Y1% Z7$ D) Z1$ Z8# E) Cannot be determined View Answer

Option A Directions (6-10): Study the following information to answer the given questions: In a certain code language GovernmentAdda.com | IBPS SBI RBI SSC FCI RRB RAILWAYS SSB

30

Daily Visit

[GOVERNMENTADDA.COM]

‗Replace the text‘ is written as ‗He is getting replace‘ is written as ‗Outside material in text‘ is written as ‗He is standing outside‘ is written as

‗dsf ki pa‘ ‗hf sa dsf ta‘ ‗ra ja pa sd‘ ‗hf sa mu sd‘

6. How does ‗mu‘ stand for ? A) is B) standing C) he D) outside E) None of these View Answer

Option B 7. How is ‗getting‘ written in that code language? A) ki B) ta C) pa D) dsf E) None of these View Answer

Option B 8. What is the code for ‗he‘ ? A) hf B) sa C) either hf or sa D) mu E) None of these View Answer

Option C 9. What is the code for ‗material‘? A) ja B) ra C) ta D) either ra or ja E) None of these View Answer

GovernmentAdda.com | IBPS SBI RBI SSC FCI RRB RAILWAYS SSB

31

Daily Visit

[GOVERNMENTADDA.COM]

Option D 10. Which of the following may code for ‗he is material in text‘? A) ra ja hf sa pa B) hf dsf ta ki pa C) ra pa dsf mu ta D) Can‘t be determined E) None of these View Answer

Option A Directions (1-5): Study the following information to answer the given questions: In a certain code, ‗mixed chamber incidence vacuum‘ is written as ‗jammer corporate frozen weird‘ is written as ‗comedian magnitude webcam jumbled‘ as ‗maximum revealed union editor‘ is written as

‘88$B 12@C ‘24©B 40@B ‗40%C 12©D ‗72%D 52$C

36#E 52%A‘, 92%A 12#E ‘, 92$B 52#E ‘, and 20@B 84©A ‘.

1. What is the code for ‗cardamom‘? A) 16#D B) 12%E C) 12$D D) 14$D E) Cannot be determined View Answer

Option C Explanation: Observe the numbers. They all are divisible by 4. Number which represents first letter * 4. Symbol is according to last letter of word: D to %, N – ©, R – @, E – #, M – $ letters are only A, B, C, D, and E. They are according to number of letters in each word: 5 – A, 6 – B, 7 – C, 8 – D, 9 – E Example: INCIDENCE I = 9, so 9*4 = 36 last letter E so # Number of letters in incidence = 9, so E So incidence is coded as – 36#E 2. What is the code for ‗pager style‘? A) 64@A 76#A B) 64#A 76@A GovernmentAdda.com | IBPS SBI RBI SSC FCI RRB RAILWAYS SSB

32

[GOVERNMENTADDA.COM]

Daily Visit

C) 60@C 76#B D) 64$A 76#A E) Cannot be determined View Answer

Option A 3. What is the code for ‗September December‘? A) 16@E 76@D B) 72@E 16@D C) 76@E 16@D D) 76@E 16#D E) Cannot be determined View Answer

Option C

4. What is code for ‗fusion clown‘? A) 12©B 24©A B) 24©B 12©A C) 24©D 12@A D) 24©C 12©A E) Cannot be determined View Answer

Option B

5. What is the code for ‗hundred folder‘? A) 32%D 24#B B) 32©C 24@B C) 32%D 24#B D) 32%C 24@B E) Cannot be determined View Answer

Option D

GovernmentAdda.com | IBPS SBI RBI SSC FCI RRB RAILWAYS SSB

33

Daily Visit

[GOVERNMENTADDA.COM]

Directions (6-10): Study the following information to answer the given questions: In a certain code, ‗home means happy house‘ is written as ‗ne dl vl og ‘, ‗brave means so bold‘ is written as ‗rep og du be‘, ‗bold words happy users‘ is written as ‗dl uj tr rep‘, and ‗More house so users‘ is written as ‗ki ne be uj‘. 6. Which is the code for ‗more home‘? A) dl uj B) ki ne C) og dl D) vl ki E) Cannot be determined View Answer

Option D ‗home‘ and ‗vl‘ are present in only first code. Also other words in first code are present in below codes. so ‗home‘ – ‗vl‘. Similarly for ‗more‘ and ‗ki‘ in fourth code 7. What is the code for ‗Bold house‘? A) ne rep B) ne tr C) rep og D) tr og E) Cannot be determined View Answer

Option A

8. What does ‗brave so words‘ stands for? A) rep be vl B) be tr du C) du rep ki D) be du og E) Cannot be determined View Answer

Option B

GovernmentAdda.com | IBPS SBI RBI SSC FCI RRB RAILWAYS SSB

34

Daily Visit

[GOVERNMENTADDA.COM]

9. What does ‗tr rep ty‘ could mean in the given code language? A) Words happy Letters B) Bold often house C) Bold makes words D) Words so Bold E) Cannot be determined View Answer

Option C ty not present anywhere and also ‗makes‘ so ‗ty‘ could be for ‗makes‘. 10. Which is the code for ‗home happy brave‘? A) og ne rep B) dl du vl C) dl vl og D) du ki be E) Cannot be determined View Answer

Option B Directions (1-5): Study the following information to answer the given questions: In a certain code, ′Right Previous Courage Money′ is written as ′P@28 R#16 ′Practice Target Justify Mocks′ is written as ′J©24 P%28 ′Market Enzymes Awake Twenty′ is written as ′T©20 A%16 ′Jokers Crazy Object Globe′ is written as ′O#20 J@20

M©16 C%24′ T#20 M@16′ E@24 M#20′ G%16 C©16′

1. What will be the code for ‗Courage Settlement‘? A) C©24 S#32 B) C%24 S@36 C) C%20 S#36 D) C%24 S#36 E) None of these View Answer

Option D Explanation: Symbols for last letters: t – #, s – @, y – ©, e – % For letters: First letter of word For numbers – 4* (number of letters in word – 1) GovernmentAdda.com | IBPS SBI RBI SSC FCI RRB RAILWAYS SSB

35

Daily Visit

[GOVERNMENTADDA.COM]

So Courage – C -> C, E =% and 7 letters in word so (7-1)*4 = 24 So Courage – C%24 2. What will be the code for ‗Allotment‘? A) A@32 B) A©32 C) A#28 D) A%32 E) None of these View Answer

Option C Allotment is A#32 3. What will be the code for ‗Target Qualify‘? A) P©24 T%20 B) Q©24 T#20 C) Q©24 T#24 D) Q©20 T#20 E) None of these View Answer

Option B 4. What will be the code for ‗Copy Limit‘? A) L#16 C%16 B) L@16 C©12 C) L#20 C@12 D) L#16 C©12 E) None of these View Answer

Option D 5. What will be the code for ‗Powers Vide‘? A) V©12 P@24 B) V#16 P@24 C) V%12 P@20 D) V%16 P%20 E) None of these View Answer

GovernmentAdda.com | IBPS SBI RBI SSC FCI RRB RAILWAYS SSB

36

[GOVERNMENTADDA.COM]

Daily Visit

Option C Directions (1-5): Study the following information to answer the given questions: In a certain code ‗developed record test precise‘ is coded as ‗precise record clock theory‘ is coded as ‗theory test light reality‘ is coded as ‗developed reality each sharp‘ is coded as

‗st rp cg rt‘, ‗bk st rp vw‘, ‗rt bk pr ab‘, and ‗cg ab no cd‘.

6. What is the code for ‗pr‘? A) light B) test C) theory D) reality E) None of these View Answer

Option A 7. What is the code for ‗rt bk rp‘? A) test theory record B) test theory precise C) theory precise developed D) Either A or B E) None of these View Answer

Option D 8. Which of the following is the code for ‗developed‘? A) rp B) st C) cg D) vw E) None of these View Answer

Option C 9. What is the code for ‗Reality test precise‘? A) ab rt cg B) ab cg no C) ab rt st D) st rp cg E) st pr vw

GovernmentAdda.com | IBPS SBI RBI SSC FCI RRB RAILWAYS SSB

37

Daily Visit

[GOVERNMENTADDA.COM]

View Answer

Option C 10. Which of the following is the code for ‗sharp‘? A) ab B) cg C) no D) bk E) None of these View Answer

Option E no or cd Directions (1-5): Study the following information to answer the given questions: In a certain code, ‗it is raining heavily today‘ is written as ‗to ga di gi ni‘, ‗today is make it set‘ is written as ‗ru to ni di zi‘, ‗come heavily it‘ is written as ‗ga ni la‘, and ‗is make set gone‘ is written as ‗ru zi mu to‘. 1. What is the code for ‘heavily’? A) ni B) ga C) to D) la E) di View Answer

Option B Decoding: From ‗it is raining heavily today‘ is written as ‗to ga di gi ni‘, and ‗today is make it set‘ is written as ‗ru to ni di zi‘, – ‗it‘ is ni, so from ‗it is raining heavily today‘ is written as ‗to ga di gi ni‘, and ‗come heavily it‘ is written as ‗ga ni la‘, ‗heavily‘ is ga 2. What does the code ‘ru’ stand for in the given code language? A) come raining gone B) make C) come raining it D) set E) None of these View Answer

GovernmentAdda.com | IBPS SBI RBI SSC FCI RRB RAILWAYS SSB

38

Daily Visit

[GOVERNMENTADDA.COM]

Option A Decoding: mu and gone only present ‗is make set gone‘ is written as ‗ru zi mu to‘. come and la only present in ‗come heavily it‘ is written as ‗ga ni la‘ raining and gi only in ‗it is raining heavily today‘ is written as ‗to ga di gi ni‘ 3. What does the code ‘mu la gi’ stand for in the given code language? A) come raining gone B) set raining gone C) come raining it D) some gone heavily E) None of these View Answer

Option A Decoding: mu and gone only present ‗is make set gone‘ is written as ‗ru zi mu to‘. come and la only present in ‗come heavily it‘ is written as ‗ga ni la‘ raining and gi only in ‗it is raining heavily today‘ is written as ‗to ga di gi ni‘ 4. What may the code ‘ru la di’ stand for in the given code language? A) make set today B) come heavily today C) come make gone D) come make today E) gone is come View Answer

Option D Decoding: ‗la‘ is ‗come‘ ‗is‘ is ‗to‘ from ‗today is make it set‘ is written as ‗ru to ni di zi‘ and ‗is make set gone‘ is written as ‗ru zi mu to‘ – make set – ru zi 5. What may be the code for ‘come this raining’ in the given code language? A) gi la ru B) la gi zi C) jo la di D) jo gi la E) jo gi ni View Answer

GovernmentAdda.com | IBPS SBI RBI SSC FCI RRB RAILWAYS SSB

39

[GOVERNMENTADDA.COM]

Daily Visit

Option D Directions (6-10): Study the following information to answer the given questions: With a certain code language, ‗she is a little girl‘ is written as ‗me bu da jo ka‘, ‗a girl was there‘ is written as ‗pu da sha ka‘, ‗there exists a little tree‘ is written as ‗bu pe te sha ka‘, and ‗little is exists‘ is written as ‗me te bu‘. 6. What is the code for ‘girl’? A) me B) da or ka C) jo D) da E) Cannot be determined View Answer

Option D 7. What is the code for ‘is was exists’? A) te da ka B) bu pu te C) pu te me D) pu jo bu E) Cannot be determined View Answer

Option C 8. What does ‘bu’ stand for? A) is B) little C) she D) is or little E) girl View Answer

Option B 9. What could be the code for ‘she a boy’? A) sha bu ka GovernmentAdda.com | IBPS SBI RBI SSC FCI RRB RAILWAYS SSB

40

Daily Visit

[GOVERNMENTADDA.COM]

B) jo me pu C) ka jo te D) mu ka jo E) None of these View Answer

Option D 10. What could be the code for ‘she tree there each’? A) me pu ha jo B) sha pe bu ut C) jo ut sha pe D) pe jo yu te E) jo sha me ka View Answer

Option C Directions (1-5): Study the following information to answer the given questions: With a certain code language, ‗care adjust found under‘ is written as ‗#6a @5u $5f %4c‘, ‗damage part road order‘ is written as ‗#4p $4r @5o %6d‘, ‗emperor entire attempt sad‘ is written as ‗#7a $3s @7e %6e‘, and ‗proud under online adjust‘ is written as ‗%6o @5u $5p #6a‘. 1. What is the code for ‗road‘? A) @5o B) #4p C) $4r D) %6d E) Cannot be determined View Answer

Option C road – $4r 4 for 4 letters in road, $ for last letter d and ‗r‘ for first letter ‗r‘. 2. What is the code for ‗attempt under entire‘? A) #6a @7e #7a B) @5u @5o $3s C) %6e #7a @5u D) None of these GovernmentAdda.com | IBPS SBI RBI SSC FCI RRB RAILWAYS SSB

41

[GOVERNMENTADDA.COM]

Daily Visit

E) Cannot be determined View Answer

Option C attempt – #7a. # for last letter t, 7 for 7 letters in attempt, ‗a‘ for first letter ‗a‘ under – @5u. @ for last letter r, 5 for 5 letters in attempt, ‗u‘ for first letter ‗u‘ entire – %6e. % for last letter e, 6 for 6 letters in attempt, ‗e‘ for first letter ‗e‘ 3. What does ‗$5p %4c @7e‘ stand for? A) under found attempt B) care proud emperor. C) care sad adjust D) entire proud emperor E) Cannot be determined View Answer

Option B

4. What will be the code for ‗butter court used‘? A) %6b #5r @6c B) #3y @6d %4r C) %8b $5g @4t D) $4d #5c @6b. E) None of these View Answer

Option D

5. What is the code for ‗damage sad online‘? A) @6o $3s %6d B) #3s @6d %4r C) %6o $3s @6d D) $4d #5c @6b E) None of these View Answer

GovernmentAdda.com | IBPS SBI RBI SSC FCI RRB RAILWAYS SSB

42

Daily Visit

[GOVERNMENTADDA.COM]

Option C

Directions (6-10): Study the following information to answer the given questions: With a certain code language, ‗hole create black bestows‘ is written as ‗f $h l#b t!b f%c‘, ‗report letters till civil‘ is written as ‗m#c t!l u%r m$t‘, ‗guard also failure junked‘ is written as ‗e%j e#g p$a f!f‘, and ‗into export every meeting‘ is written as ‗z#e u%e p$i h!m‘. 6. What is the code for ‗letters‘? A) u%r B) m#c C) t!l D) m $t E) Cannot be determined View Answer

Option C $ for four letter words, # for five letter words, ! for seven letter words, % for six letter words, hole – f$h. f for next letter of last letter ‗e‘, $ for four letter words and h for first letter of hole. letters – t!l. t for next letter of last letter ‗s‘, ! for seven letter words and l for first letter of letters. 7. What is the code for ‗report create meeting‘? A) t!l u%e f$h B) h!m u%r f%c C) l#b u%r m $t D) None of these E) Cannot be determined View Answer

Option B

8. What does ‗t!b u$l u#c‘ stand for? A) under bestows attempt B) court last bestows C) care hole adjust D) black proud emperor E) Cannot be determined

GovernmentAdda.com | IBPS SBI RBI SSC FCI RRB RAILWAYS SSB

43

Daily Visit

[GOVERNMENTADDA.COM]

View Answer

Option B

9. What will be the code for ‗butter would used‘? A) t$r y#d e#w B) s%b g!r e$u C) t!g f%b e#v D) e$u s%b e#w E) None of these View Answer

Option D

10. What does ‗e$t i#b f%c‘ stand for? A) tallest fish into B) could best earn C) centre told bench D) ice earn calorie E) None of these View Answer

Option C Directions (1 – 5): Study the following information arrangement carefully and answer the questions given below: With a certain code language, ‗alarm forest cuddle morning‘ is written as ‗%f6 !m7 #a5 @c6‘, ‗sight fire making criticism‘ is written as ‗#c9 @f4 %s5 !m6‘, ‗raising centre recent alarm‘ is written as ‗@c6 %r6 #a5 !r7‘, and ‗strike arm ignoring sight‘ is written as ‗!i8 %s5 @s6 #a3‘. 1. What is the code for ‘raising’? A) !r7 B) @c6 C) #a5 D) %r6 E) Cannot be determined View Answer

GovernmentAdda.com | IBPS SBI RBI SSC FCI RRB RAILWAYS SSB

44

Daily Visit

[GOVERNMENTADDA.COM]

A) !r7 Explanation: # for last letter m, % for last letter t, @ for last letter e, and ! for last letter g Number represents number of alphabets in word Alphabet represents the first letter of word. So raising = ! for g, r for first r in raising and 7 for raising 2. What is the code for ‘fire arm morning’? A) @c6 !m6 %s5 B) #a3 !i8 @c6 C) @f4 !m7 #a3 D) None of these E) Cannot be determined View Answer

C) @f4 !m7 #a3 Explanation: # for last letter m, % for last letter t, @ for last letter e, and ! for last letter g Number represents number of alphabets in word Alphabet represents the first letter of word. 3. What does ‘@s6 %s5 !m6’ stand for? A) ignoring cuddle forest B) sight morning arm C) making strike sight D) strike raising fire E) Cannot be determined View Answer

C) making strike sight 4. What could be the code for ‘surfeit attempt alarm’? A) %a6 #a5 @s6 B) #a5 %s7 %a7 C) %s8 #a5 @s4 D) #a5 #a3 !m4 E) None of these View Answer

B) #a5 %s7 %a7 Explanation: surfeit – % for t, s for s, 7 for surfeit GovernmentAdda.com | IBPS SBI RBI SSC FCI RRB RAILWAYS SSB

45

Daily Visit

[GOVERNMENTADDA.COM]

5. What is the code for ‘making centre forest’? A) !m7 #a5 @c6 B) %r6 %f6 #c9 C) !m6 @s6 #a3 D) %f6 @c6 !m6 E) Cannot be determined View Answer

D) %f6 @c6 !m6 Directions (6-10): Study the following information to answer the given questions: In a certain code, ‗commit also make policy‘ is written as ‗%e4 !y6 #t6 @o4‘, ‗policy craze anger mobile‘ is written as ‗!y6 @r5 %e6 #e5‘, ‗allow mild course prize‘ is written as ‗!e5 %d4 #e6 @w5‘, and ‗craze manner pump artist‘ is written as ‗%r6 #e5 !p4 @t6‘. 6. What is the code for ‘mild’ ? A) @w5 B) %d4 C) %e4 D) %e6 E) Can‘t be determined View Answer

B) %d4 Explanation: # for first letter c, % for first letter m, @ for first letter a, and ! for first letter p Number represents number of alphabets in word Alphabet represents the last letter of word. So mild = % for m, d for last d in mild and 4 for mild 7. What does ‘#e6 #e5 @04’ stand for? A) also make course B) craze also course C) commit course mobile D) artist mild craze E) Either also make course or prize also course View Answer

B) craze also course Explanation: #e6 – course GovernmentAdda.com | IBPS SBI RBI SSC FCI RRB RAILWAYS SSB

46

Daily Visit

[GOVERNMENTADDA.COM]

#e5 – craze @04 – also 8. Which could be the code for ‘peace protect pump’? A) #t4 %d6 !r4 B) !e4 !r4 !e2 C) !p4 @w3 !r4 D) !e5 !t7 !p4 E) None of these View Answer

D) !e5 !t7 !p4 Explanation: peace – !e5 protect – !t7 pump – !p4 9. ’!l8 %n6 @e7’ could be a code for which of the following? A) mobile charge victory B) peaceful modern advance C) against modern effort D) mild peaceful west E) None of these View Answer

B) peaceful modern advance Explanation: tis – lists since da and arise both not present so ‗da‘ can be ‗arise‘ tip – regime 10. Which of the following is the code for ‘anger prize commit’? A) !e5 %d4 %e4 B) !e5 @r5 #t6 C) @r5 #e6 #t6 D) #t6 @o4 %e6 E) None of these View Answer

B) !e5 @r5 #t6 Directions (1-5): Study the following information to answer the given questions: In a certain code, ′Search box to find′ is written as

′K21

S22

P15

T16′

GovernmentAdda.com | IBPS SBI RBI SSC FCI RRB RAILWAYS SSB

47

Daily Visit

[GOVERNMENTADDA.COM]

′Find is first form′ is written as ′L31 ′Table is looking good′ is written as ′R19 ′Search text not command′ is written as ′S17

N20 O19 E25

R19 F34 K21

T16′ E15′ U35′

1. What text be the code for ‗find girl‘? A) T16 K21 B) T16 U35 C) A19 T16 D) O19 T16 E) None of these View Answer

Option C Find – T16 Girl is not present anywhere in codes and so is A19 2. Which of the following can be used to find out the code for ‗box‘? (i) S22 K21 P15 U38 (ii) N20 S22 K67 U35 (iii) V20 K21 K67 P15 (iv) N20 P20 L31 U35 A) (ii) B) (ii) or (iv) C) (ii) or (iii) D) (i) or (ii) E) None of these View Answer

Option C Explanation: From 1st code, search – K21 and find – T16 So ‗box‘ is either S22 or P15 so if given that what codes (ii) and (iii) stand for, the code for ‗box‘ can be found. [because they contain either S22 or P15 in (i) both are given, so cannot be found. In (iv) – neither is given so cant be found So either (ii) or (iii) can be used 3. Which of the following cannot be used to find the code for ‗command‘? A) N20 S22 K67 U35 B) S17 K21 E25 P15 C) S22 E25 P15 U38 D) N20 P20 L31 S17 E) None of these View Answer

GovernmentAdda.com | IBPS SBI RBI SSC FCI RRB RAILWAYS SSB

48

Daily Visit

[GOVERNMENTADDA.COM]

Option B Command can be S17 or E25 or U35. But B) contains both S17 and E25 4. If ‗first‘ is coded as ‗L31‘, then what text be the code for ‗not form‘ A) N20 K21 B) U35 N20 C) O19 N20 D) N20 S22 E) F34 N20 View Answer

Option B In ′Find is first form′ is written as ′L31 N20 R19 T16′. Find – T16, is – R19. Now if first – L31, then form – N20. Now in ′Search text not command′ is written as ′S17 E25 K21 U35′ Search – K21, so A cannot be the answer. not can be S17 or E25 or U35, none of which is present in B, C or D. In B U35 is there. So B) text be the code. 5. If the code for ‗looking‘ is ‗F34‘, then which of the following can be used to find the code for ‗good‘? A) O19 E15 K21 P15 B) S17 O19 R19 E15 C) O19 P15 F34 E15 D) L31 U35 E15 P15 E) None of these View Answer

Option D ‗is‘ – R19. So if ‗looking‘ is R34, then ‗good‘ can be coded as O19 or E15. Only in D option, one of them is written. In other options both O19 and E15 are written so will not be useful to find the code for ‗good‘. Directions (6-10): Study the following information to answer the given questions: In a certain code, ‗crucial major radical sour‘ is written as ‗%a4 @a4 ‗right calm safety mutual‘ is written as ‗!t3 @h3 ‗close micro school regime‘ is written as ‗@m3 &r3 ‗mystery reside sour crucial‘ is written as ‗%a4 &r4

!u2 &a3 !o3 !u2

&o3‘, %l2‘, %s3‘, and @d3‘.

6. What is the code for ‗mutual‘? A) !n3 B) @c2 C) &a3 D) &n2 GovernmentAdda.com | IBPS SBI RBI SSC FCI RRB RAILWAYS SSB

49

[GOVERNMENTADDA.COM]

Daily Visit

E) Cannot be determined View Answer

Option C & for first letter m, % for first letter c, @ for first letter r, and ! for first letter s Number represents number of alphabets in word divided by 2. And is it comes in decimal, the number is rounded off. Example crucial= 7 = 7/2 = 4. sour = 4 = 4/2 = 2 Alphabet represents the second last letter of word. So crucial = % for c, a for second last a in crucial and 4 for crucial= 7 = 7/2 = 4 7. What does ‗!o3 @h3‘ stand for? A) calm regime B) right school C) school calm D) close mutual E) Cannot be determined View Answer

Option B

8. ‘!a2 @n3 &n4‘ could be a code for which of the following? A) morbid raising shake B) morning star rising C) carry style sour D) spacing rising radical E) Cannot be determined View Answer

Option B remedy – @d3 9. What is code for ‗close safety‘? A) %r2 &r4 B) %l3 @c2 C) !n3 %r2 D) !t3 %s3 E) Cannot be determined View Answer

GovernmentAdda.com | IBPS SBI RBI SSC FCI RRB RAILWAYS SSB

50

Daily Visit

[GOVERNMENTADDA.COM]

Option D

10. Which of the following will represent ‗carry spacing righteous‘? A) @m2 !a5 &t3 B) %r3 !a4 @t3 C) %m3 &s5 &r2 D) %r3 !n4 @u5 E) Cannot be determined View Answer

Option D multi – &t3 satirical – !a5 ramp – @m2 Directions (1-5): Study the following information to answer the given questions: In a certain code, ′Field Panel joint Guide′ is written as ′K28@ U10% Q18# ′Since Valid Worst Ideal′ is written as ′D36# H28© E24% ′Mount Local Trend Three′ is written as ′N42# G36© O06@ ′Mixed loose Final Sport′ is written as ′U28@ O30© N48%

T18©′ R10@′ G12%′ H30#′

1. What will be the code for ‗Spend Noise‘? A) H10% N18© B) H10% M18@ C) H12# M18© D) H10% M18© E) None of these View Answer

Option D Explanation: Symbols for last letters: t – #, l – @, e – ©, d – % For letters: Reverse of 1st letter is taken. A -> Z, B -> Y………. and Z -> A For numbers – 2* (number represented by middle letter of word) So Field – F -> U, E =5 so 5*2 = 10, last letter d – % So Field – U10% 2. What will be the code for ‗Prize Level Ahead‘? A) Z10% K20© M44@ B) Z10% K18© O44# GovernmentAdda.com | IBPS SBI RBI SSC FCI RRB RAILWAYS SSB

51

[GOVERNMENTADDA.COM]

Daily Visit

C) Z10% K18© O44@ D) Z10# J18© O44@ E) None of these View Answer

Option C 3. What will be the code for ‗Rural Prove‘? A) K30© I34@ B) J30© I36@ C) K28© J36@ D) K30© I36@ E) None of these View Answer

Option D 4. What could ‗Y42% W28%‘ stand for? A) Noise Solid B) Dance Bound C) Close Broad D) Bound Arise E) None of these View Answer

Option B 5. What will be the code for ‗Doubt Craft‘? A) W42# Y02# B) Z42# X02# C) W44# Y02# D) W42# X02# E) None of these View Answer

Option D Directions (6-10): Study the following information to answer the given questions: In a certain code language ―review paper style manage‖ is written as ‗def fa cde te‘ GovernmentAdda.com | IBPS SBI RBI SSC FCI RRB RAILWAYS SSB

52

Daily Visit

[GOVERNMENTADDA.COM]

―paper of known country‖ is written as ‗gi te ku ad‘ ―Known review view reference‖ is written as ‗cde bp ad bgs‘ ―View like manage layout‖ is written as ‗kx def dr bp‘ 6. If ‗style known page‘ is coded as ‗ad fa zy‘ in the given code language, then what will ‗review risk manage‘ be coded as? A) ad dr def B) def cde dr C) zy cde ad D) fa zy def E) None of these View Answer

Option E

7. What is the code for ‗view of post‘ in the given code language? A) gi ku cde B) bp cde gi C) ku bp gi D) dr def fa E) None of these View Answer

Option E

8. Which of the following may represents the code ‗paper improve‘ in the given code language? A) le dr B) cs def C) cs te D) def nq E) dr cs View Answer

Option C

9. In the given code language, what does the code ‗kx‘ stands for? A) paper B) known GovernmentAdda.com | IBPS SBI RBI SSC FCI RRB RAILWAYS SSB

53

Daily Visit

[GOVERNMENTADDA.COM]

C) either ‗layout‘ or ‗like‘ D) either ‗manage‘ or developed E) review View Answer

Option C

10. What is the code for ‗reference‘ in the given code language? A) bp B) dr C) bgs D) cde E) None of these View Answer

Option C

GovernmentAdda.com | IBPS SBI RBI SSC FCI RRB RAILWAYS SSB

54

Daily Visit :

[GOVERNMENTADDA.COM]

Blood Relation Questions With Solution

GovernmentAdda.com

GovernmentAdda.com | IBPS SBI RBI SSC RRB FCI RAILWAYS SSB

1

Daily Visit :

[GOVERNMENTADDA.COM]

Directions (1-3): Study the following information carefully to answer the questions that follow: A is father of B and C is mother of A. E is sister of F whose daughter is G. S, the husband of C is the grandfather of G. P is father of E and brother of R. S has only two children, both of opposite sex. 1. What is the relation between F and S? A) F is daughter of S B) F is sister of S C) F is son of S D) F is daughter in law of S E) Either A option or D option View Answer Option D Explanation: C and S have 2 children, one is A who is male (A is father of B). since C is the grandfather of G so F should be wife of A to have the relation C is the grandfather of G. If F was daughter of C and s, so C would have been maternal grandfather of her daughter G. 2. What is the relation between E and B? A) E is sister of B B) E is brother of B C) E is aunt of B D) E is maternal grandmother of B E) None of these View Answer Option C Explanation: E is sister of F and B is child of F

E) None of these View Answer Option E Explanation: Since gender of B is not known, either sister or brother of G Directions (4-6): A is mother of D who is father of G. B is grandfather of E and husband of A. D who has only two children is brother of C. A has two children both of same gender. J is aunt of H who is sister of G. 4. What is the relation between J and D? A) J is sister of D B) J is mother of D C) J is aunt of D D) Cannot be determined E) None of these View Answer Option E Explanation: H is sister of G and G is child of D. so H and G children of D. J is aunt of H. so J can be wife of D‘s brother C or J can be sister of D‘s wife. In both cases J will be sister in law of D. 5. What is the relation between C and E? A) C is brother of E B) C is father of E C) C is uncle of E D) Cannot be determined E) None of these View Answer

3. What is the relation between B and G? A) B is sister of G B) B is brother of G C) B is aunt of G D) There is no relation

Option B Explanation: B and A are husband wife, who have 2 children of same sex. A is mother of D who is father of G. this means both children are

GovernmentAdda.com | IBPS SBI RBI SSC RRB FCI RAILWAYS SSB

2

Daily Visit :

[GOVERNMENTADDA.COM]

males. D is brother of C, so C and D both are sons of A and B. D also has two children – G and H. if B is grandfather of E then C must be father of E. 6. At least how many male members can be predicted by the given relations? A) 2 B) 3 C) 4 D) 5 E) None of these View Answer Option B Explanation: B, C, and D are certainly males H and J are females. Gender of E and G not known. Directions (7-8): A has two sons. E is the daughter of G and B is the mother of C. F, the brother of E is the son of C who is the son of A. A is grandfather of J who is not a sibling of E. B has a child named D. 7. What is the relationship between D and J? A) D is father of J B) D is uncle of J C) J is son of D D) Cannot be determined E) None of these View Answer Option D Explanation: A has two sons, but it is not mentioned that A has only 2 sons. So A and B can have daughters also Also it is not known that D is son of A or daughter of A, so J can be child of D or D can be uncle/aunt of J.

A) 1 : 1 B) 1 : 3 C) 5 : 3 D) 3 : 5 E) Cannot be determined View Answer Option E Explanation: J‘s and D‘s gender not known. 9. If ‗P # Q‘ means ‗P is the sister of Q‘, ‗P + Q‘ means ‗P is the wife of Q‘, ‗P @ Q‘ means ‗P is the son of Q‘, and ‗P $ Q‘ means ‗P is the mother of Q‘, then what will come in the place of question mark, if it is provided that ‗D is the daughter-in-law of T‘ in the expression ‗D $ R ? L @ T‘? A) + B) # C) $ D) @ E) + or @ View Answer Option D Explanation: L is son of T, D is mother of R, so R should be child of D and L, for D to be daughter in law of T. 10. F is the Father of D, who is the father of S. M is the grandmother of R, who is the son of O.N is paternal uncle of R, who is brother of S. What is the relationship between O and R? A) Mother and son B) uncle and nephew C) Father and son D) No relation E) Cannot be determined View Answer

8. What is the ratio of males to females in the family? GovernmentAdda.com | IBPS SBI RBI SSC RRB FCI RAILWAYS SSB

3

Daily Visit :

[GOVERNMENTADDA.COM]

Option A Explanation: R is brother of S means D is father of R. R is son of O also, so O is mother of R. Directions (1 – 3): B is the mother of C who is the sister of G and H only. I is the son of H. D is the father of E. Among the children of A and B, only 1 is unmarried. G is the uncle of E who is the sister of F. C has only 2 children. 1. How is G related to B? A) son B) daughter C) son-in-law D) daughter-in-law E) Cannot be determined View Answer Option A Explanation: G is the uncle so is male. so is son of B 2. Who is the unmarried child of A and B? A) C B) G C) H D) E E) Cannot be determined View Answer Option B Explanation: I is son of H and C has 2 children, so G is left. Also A and B have only 3 children because C is sister on only H and G. 3. Who is the mother of F? A) C B) H C) B D) Cannot be determined E) None of these

View Answer Option D Explanation: E is sister of F, D being their father. But no relation is given between C and D or H and D. C can be F‘s mother or H can also be F‘s mother because not specified that H has how many children. Directions (4 – 5): Consider the following relationships and answer based on the relationships given below: ‗P % Q‘ means ‗P is daughter of Q‘ ‗P # Q‘ means ‗P is wife of Q‘ ‗P $ Q‘ means ‗P is father of Q‘ ‗P @ Q‘ means ‗P is brother of Q‘ 4. N @ B $ M % D indicates what relationship between N and D? A) D is sister of N B) D is nephew of N C) D is sister in law of N D) N is wife of D E) None of these View Answer Option C Explanation: N is brother of B, B is father of M and M is daughter of D, so L is daughter of B and D, B is father so D is female, and so sister in law of N. 5. The expression ―M # Q @ D % S‖ indicates which of the following relationship? A) Q is father of S B) D is brother-in-law of M C) M is sister of S D) S is mother-in-law of M E) None of above View Answer Option E Explanation:

GovernmentAdda.com | IBPS SBI RBI SSC RRB FCI RAILWAYS SSB

4

Daily Visit :

[GOVERNMENTADDA.COM]

M is wife of Q, Q is brother of D, so M is sister-in-law of D D is daughter of S, so M must be daughterin-law of S – but gender of S not known, and D sister-in-law of M 6. Looking at a lady in a photograph, Tanya said, ―She is my mother‘s mother‘s daughter‖. How is the lady related to Tanya? A) Aunt B) Sister C) Sister in law D) Daughter E) Cannot be determined View Answer Option E Explanation: Since it is not specified that mother‘s mother have how many daughters, so lady can be mother or aunt of Tanya. 7. If ‗P $ Q‘ means ‗P is mother of Q‘, ‗P # Q‘ means ‗P is father of Q‘, and ‗P @ Q‘ means ‗P is husband of Q‘, then in the expression ―B @ K $ W # M‖ what is the relationship of B with M? A) Paternal Grandmother B) Maternal Grandmother C) Paternal Grandfather D) Maternal Grandfather E) None of these View Answer Option C Explanation: B is husband of K, K is mother of W, so W is child of B and K W is father of M, so M is grandchild of B and K, B being male here Directions (8 – 10): P is the brother of Q. C is the daughter of P and also sister of J. A is mother of J. S is father of A.

D who is son of T is brother of A. B has only one son and is married to E, mother of P. 8. How is D related to J? A) son B) father C) brother-in-law D) uncle E) Cannot be determined View Answer Option D Explanation: D is brother of A and A is mother of J. So D is uncle of J. 9. If G is married to Q, how is B related to G? A) father B) father-in-law C) son-in-law D) uncle E) Cannot be determined View Answer Option B Explanation: E is mother of P, so B is his father, Q is child of B, so B is father-in-law of someone (here G) who is married to Q. 10. If A relates to S in the same way as J relates to P, then how is J related to Q? A) daughter B) nephew C) niece D) Cannot be determined E) None of these View Answer Option C Explanation: A is daughter of S. So J is daughter of P hence is niece of Q.

GovernmentAdda.com | IBPS SBI RBI SSC RRB FCI RAILWAYS SSB

5

[GOVERNMENTADDA.COM]

Daily Visit :

Direction(1-5) Read the following information carefully to answer the following questions

3. Which of the following can be the correct conclusion drawn from the expression

‗A # B‘ means ‗A is the daughter of B‘

„A = P # Q © V * R‟?

‗A © B‘ means ‗B is the brother of A‘

A. R is the grandson of P

‗A = B‘ means ‗B is the sister of A‘

B. A is the uncle of Q

‗A & B‘ means ‗A is the son of B‘

C. Q is the uncle of R

‗A * B‘ means ‗A is the father of B‘

D. R is the niece of Q

‗A @ B‘ means ‗A is the mother of B‘

E. None of these

1. What does the expression „P @ R = Q © S & T‟?

Answer Answer E. None of these

A. T is the husband of P B. R is the son of T

4. Which of the following can be

C. R is the daughter of T

correct conclusion drawn from the

D. T is the wife of P

expression

E. None of these

„E & C @ D © G = F‟? A. D is the brother of F

Answer

B. C has two sons and two daughters

Answer- A. T is the husband of P

C. D is the sister of F D. F is the sister of E

2. Which of the following indicates that „A is the paternal uncle of B‟? A. A & U # Q @ R © B B. A & U & R @ Q © B

E. None of these Answer Answer D. F is the sister of E

C. B & R & Q @ U © A D. B & Q # U @ R © A

5. Which of the following indicates „M is the daughter of N‟?

E. None of these

A. Q * P # C @ N @ V Answer

B. N * D # R @ M @ B

Answer – C. B & R & Q @ U © A

C. F @ N # R * M D. F © M = B # N E. None of these

GovernmentAdda.com | IBPS SBI RBI SSC RRB FCI RAILWAYS SSB

6

[GOVERNMENTADDA.COM]

Daily Visit : Answer

Answer – B. P

Answer B. N * D # R @ M @ B

8. How is R related to T? A. Father-in-law

Directions(6-8): Study the following

B. Mother-in-law

information carefully and answer the

C. Father

questions given below:

D. Mother

P is the daughter of Q. R has only two

E. Brother

children – P and S. T is the brother of U. S is married to V. R has only two daughters. W is

Answer

the mother of Q. T is married to P. W is Answer A. Father-in-law

married to X. R is the son of Y. 9. 6. Who among the following is the father of Q?

Directions(Q.No: 9 & 10): Study the following

A. W

information carefully and answer the

B. Y

questions given below:

C. R

P is the brother of Q. Q is the mother of R. R

D. X

is the sister of S. S is married to T. U is the

E. None of these

father of Q. U has only one daughter. U is married to V. P is the brother of W. X is the

Answer

father-in-law of W. Answer – D. X 6. 7. Who among the following is the

9. How is W related to R?

sister-in-law of U?

A. Cannot be determined

A. S

B. Mother

B. P

C. Uncle

C. Q

D. Father

D. W

E. Aunt

E. Y Answer Answer Answer C. Uncle GovernmentAdda.com | IBPS SBI RBI SSC RRB FCI RAILWAYS SSB

7

[GOVERNMENTADDA.COM]

Daily Visit :

10. If X is the father of M, then how is W related

of K – P must be son or daughter of Q. so both × and +

to M? A. Cousin

2. Which of the following relation is

B. Husband

true with regard to expression „B

C. Wife

÷ P × Z – K + O‟?

D. Cannot be determined

A) P is brother of O

E. Brother

B) B is daughter-in-law of K C) B is daughter-in-law of O

Answer Answer B. Husband

D) O is daughter of Z E) None of these Answer & Explanation

1 – 4: ‗P × Q‘ means ‗P is son of Q‘.

C) B is daughter-in-law of O

‗P + Q‘ means ‗P is daughter of Q‘.

Explanation:

‗P ’ Q‘ means ‗P is wife of Q‘.

B wife of P, P son of Z, Z father of

‗P – Q‘ means ‗P is father of Q‘.

K – so P and K siblings, K daughter of O

1. In the expression „K + H – P ? Q‟,

So Z and O husband wife, P and K

what will come in place of ? if Q is

brother sister, B wife of P

mother of K?

So B is daughter-in-law of O

A) + 3. Which of the following pairs

B) – C) ÷

represent the first cousins in the

D) ×

expressions – „L † V – J + P‟ and

E) None of these

„S × A – D + F – E + K‟ – if it is given that A is the sister of J?

Answer & Explanation

A) LP B) SP

E) None of these

C) SK

Explanation: K daughter of H, H father of P, so KandP siblings, now Q to be mother

D) SF E) Cannot be determined Answer & Explanation

GovernmentAdda.com | IBPS SBI RBI SSC RRB FCI RAILWAYS SSB

8

[GOVERNMENTADDA.COM]

Daily Visit :

B) SP

A) J is sister of P

Explanation:

B) J is nephew of P

J father of P, and S son of A now

C) J is niece of P

given that A is the sister of J,

D) Cannot be determined

Now A and J siblings. P is J‘s child,

E) None of these

and S is A‘s, so both first cousins 4. If it is provided that M is grandmother of P, then what will come in place of ? in expression – „P – H ÷ T ? M‟?

Answer & Explanation C) J is niece of P Explanation: P is brother of R, R is father of J and J is wife of T

A) +

Since J is wife, so is a female, so she

B) –

is niece of her father‘s brother

C) ÷ D) ×

6. The expression “P # O @ G % F”

E) None of these

indicates which of the following

Answer & Explanation

relationship? A) O is father of F

E) None of these

B) G is brother-in-law of P

Explanation:

C) P is sister of F

P father of H, H wife of T.

D) F is mother-in-law of P

Now for M to be grandmother of P,

E) Cannot be determined

there is no relation given between T and M.

Answer & Explanation E) Cannot be determined

5 – 6: ‗A $ B‘ means ‗A is father of B‘ ‗A # B‘ means ‗A is wife of B‘ ‗A @ B‘ means ‗A is brother of B‘ ‗A % B‘ means ‗A is daughter of B‘

Explanation: P is wife of O, O is brother of G, so P is sister-in-law of G G is daughter of F, so P must be daughter-in-law of F – but gender of F not known, and G sister-in-law of

5. P @ R $ J # T indicates what

P (as G being a female)

relationship between P and J? GovernmentAdda.com | IBPS SBI RBI SSC RRB FCI RAILWAYS SSB

9

[GOVERNMENTADDA.COM]

Daily Visit :

7. A told to B, “Yesterday I met the

9. Pointing to a lady, Arun said “The

only daughter of my

son of her only brother is the

grandmother.” Whom did A

brother of my wife”. How is the

meet?

lady related to Arun?

A) Cousin

A) Mother‘s sister

B) Mother

B) Grandmother

C) Sister

C) Mother-in-law

D) Nephew

D) Sister of father-in-law

E) None of these

E) Maternal aunt

Answer & Explanation

Answer & Explanation

E) None of these

D) Sister of father-in-law

Explanation:

Explanation:

He met his aunt, i.e. his father‘s

Her brother is Arun‘s father-in-law,

sister

so she is sister of father-in-law

8. Which of the following means that

10. Pointing to a person, Arun said to

M is grandmother of N, if „P $ Q‟

Sarika, “His mother the only

means P is father of Q‟, „P*Q‟

daughter of your father.” How is

means P is mother of Q, „P @ Q‟

Sarika related to that person?

means P is wife of Q‟?

A) Aunt

A) M * R $ T @ N

B) Daughter

B) M * R @ T @ N

C) Mother-in-law

C) M * T $ N $ R

D) Mother

D) M * T $ R @ N

E) Sister

E) Both C and D

Answer & Explanation

Answer & Explanation D) Mother C) M * T $ N $ R

Explanation:

Explanation:

Sarika‘s father‘s only daughter

Check all options

means Sarika herself, so Sarika is

C) = M is mother of T, T father of N

mother to that person (pointed to by His)

GovernmentAdda.com | IBPS SBI RBI SSC RRB FCI RAILWAYS SSB

10

[GOVERNMENTADDA.COM]

Daily Visit :

Direction(1-2) Read the following information

c) grandmother

carefully to answer the following questions-

d) father

A * B means A is the sister of B

e) None of these

A ÷ B means A is the brother of B A + B means A is the father of B A – B means A is the mother of B

Answer & Explanation Answer –c) grandmother Explanation :

1. What is the relation between Q and R in „P + Q † R – S‟ a) Q is the aunt of R b) Q is the brother of R c) Q is the mother of R d) Q is the father of R e) None of these Answer & Explanation Answer –b) Q is brother of R Explanation : Direction(Q3 – Q5) There is family of 6 members P, Q, R, S, T and U in which two are married couples. S is the grandfather of P and father of T. Q is the wife of T and mother of R. P is the granddaughter of S. Explanation Explanation –

2. In P – Q + R ÷ S, what is P to Ra) Grandfather b) mother SU and TQ are couples GovernmentAdda.com | IBPS SBI RBI SSC RRB FCI RAILWAYS SSB

11

Daily Visit :

[GOVERNMENTADDA.COM]

3. How many males are there in the family?

Answer Answer –c) SU

a) two b) three

Direction(Q6 – Q10) In a family there are 6

c) four

members P, Q, R, X, Y and Z. R is the sister

d) can‘t be determined

of Z. X is the father of P and grandfather of

e) None of these

Z. Q is the brother of Y‟s husband. There are

Answer & Explanation

three brothers, one mother and two fathers.

Answer –d) can‟t be determined

Explanation Explanation :

Explanation :Gender of R is unknown 4. What is the relation between U and T? a) U is the father of T b) U is the mother of T c) U is the mother in law of T d) There is no relation between U and T e) None of these 6. Who is the mother of R? Answer

a) Q

Answer –b) U is the mother of T

b) Z c) Y

5. Which among the following is one of the couple? a) TU

d) X e) None of these Answer

b) SQ c) SU

Answer – c) Y

d) TP e) None of these

7. Who is Y‟s husband ? a) Q GovernmentAdda.com | IBPS SBI RBI SSC RRB FCI RAILWAYS SSB

12

[GOVERNMENTADDA.COM]

Daily Visit : b) P

10. How is Q related to Y?

c) X

a) sister in law

d) Can‘t be determined

b) Brother in law

e) None of these

c) Father in law d) Mother in law

Answer

e) None of these

Answer – b) P

Answer

8. How many females are there in the

Answer – b) Brother in law

family? a) One

Direction(1-5) Read the following information

b) two

carefully to answer the following questions

c) three ‗P # Q‘ means ‗P is the daughter of Q‘

d) four

‗P © Q‘ means ‗Q is the brother of P‘

Answer

‗P = Q‘ means ‗Q is the sister of P‘ ‗P & Q‘ means ‗P is the son of Q‘

Answer – b) two

‗P * Q‘ means ‗P is the father of Q‘ 9. Which of the following is the

‗P @ Q‘ means ‗P is the mother of Q‘

group of brothers ? 1. What does the expression „A @

a) PQZ b) PRZ

C = B © D & E‟?

c) XZP

A. E is the husband of A

d) QRZ

B. C is the son of E

e) None of these

C. C is the daughter of E

Answer & Explanation

D. E is the wife of A E. None of these

Answer – a) PQZ

Answer

Explanation : P and Q are brothers and Z is the

Answer- A. E is the husband of A

brother of R 2. Which of the following indicates that „W is the paternal uncle of GovernmentAdda.com | IBPS SBI RBI SSC RRB FCI RAILWAYS SSB

13

Daily Visit :

[GOVERNMENTADDA.COM] Answer

P‟? A. W & U # Q @ R © P

Answer D. H is the sister of A.

B. W & U & R @ Q © P C. P & R & Q @ U © W

5. Which of the following indicates

D. P & Q # U @ R © W

„A is the daughter of E‟?

E. None of these

A. Q * P # C @ N @ V B. E * D # R @ A @ B

Answer

C. F @ E # R * A Answer – C. P & R & Q @ U © W

D. F © A = B # E E. None of these

3. Which of the following can be the correct conclusion drawn from the expression „O = P # Q © V * W‟?

Answer Answer B. E * D # R @ A @ B

A. W is the grandson of P

Directions(6-8): Study the following

B. O is the uncle of Q

information carefully and answer the

C. Q is the uncle of W

questions given below:

D. W is the niece of Q

A is the daughter of E. F has only two

E. None of these

children – A and G. H is the brother of I. G is

Answer

married to K. F has only two daughters. M is the mother of E. H is married to A. M is

Answer E. None of these 4. Which of the following can be

married to N. F is the son of O.

6. Who among the following

correct conclusion drawn from the

is the father of E?

expression

A. M

„A & C @ D © G = H‟?

B. O

A. D is the brother of H

C. F

B. C has two sons and two daughters

D. N

C. D is the sister of H

E. None of these

D. H is the sister of A E. None of these

Answer

GovernmentAdda.com | IBPS SBI RBI SSC RRB FCI RAILWAYS SSB

14

[GOVERNMENTADDA.COM]

Daily Visit :

Answer – D. N

9. How is I related to C? A. Cannot be determined

7. Who among the following

B. Mother

is the sister-in-law of I?

C. Uncle

A. G

D. Father

B. A

E. Aunt

C. E Answer

D. M E. O

Answer C. Uncle

Answer 10. If L is the father of M, Answer – B. A

then how is I related to M? A. Cousin

8. How is F related to H?

B. Husband

A. Father-in-law

C. Wife

B. Mother-in-law

D. Cannot be determined

C. Father

E. Brother

D. Mother E. Brother

Answer

Answer

Answer B. Husband

Answer A. Father-in-law

Direction(1-4) Read the following information carefully to answer the following questions

Directions(Q.No: 9 & 10): Study the following

There are six persons P, Q, R, S, T and U in a

information carefully and answer the

family. There are two fathers, three brothers

questions given below:

and a mother in the family. R is the sister of

A is the brother of B. B is the mother of C.

U. Q is the brother of T‟s husband. S is the

C is the sister of E. E is married to F. G is the

father of P and grandfather of U.

father of B. G has only one daughter. G is married to H. A is the brother of I. L is the

Explanation

father-in-law of I.

GovernmentAdda.com | IBPS SBI RBI SSC RRB FCI RAILWAYS SSB

15

Daily Visit :

[GOVERNMENTADDA.COM] 3. How is U related to T? a) son b) daughter c) niece d) nephew e) None of these Answer Answer – a) son

1. Who is the mother in the family?

4. Which of the following is the group of brothers?

a) P

a) SUQ

b) R

b) SPQ

c) T

c) PQU

d) S

d) SQU

e) None of these

e) None of these Answer Answer Answer – c) T

Answer – c) PQU

2. How many males are there in the family? a) one b) two c) three d) four e) None of these

5. Pointing to a woman in the picture, Rajesh said her mother has only one grandchild whose mother is my wife. How is the woman in the picture related to Rajesh? a) daughter

Answer

b) niece c) wife

Answer – d) four

d) sister-in-law e) None of these

GovernmentAdda.com | IBPS SBI RBI SSC RRB FCI RAILWAYS SSB

16

Daily Visit :

[GOVERNMENTADDA.COM]

Answer & Explanation

Direction: Q(7 – 8) Read the following information carefully to answer the following

Answer – c) wife

questions

Explanation :

A * B means A is the sister of B A ÷ B means A is the brother of B A + B means A is the father of B A – B means A is the mother of B

7. What is the relation between Q and S in „P + Q † R – S‟ a) Q is the aunt of S b) Q is the uncle of S 6. Pointing to a photograph a lady tells prakash I am the only daughter of this lady and her son

c) Q is the mother of S d) Q is the father of S e) None of these

is your maternal uncle. How the speaker related to prakash father? a) wife

Answer & Explanation Answer – b) Q is the uncle of S

b) daughter

Explanation :

c) sister-in-law d) mother-in-law e) None of these Answer & Explanation Answer – a) wife Explanation :

8. What is the relation between T and Q in „P*Q†R+S*T‟ a) T is the niece of Q b) T is the son of Q GovernmentAdda.com | IBPS SBI RBI SSC RRB FCI RAILWAYS SSB

17

Daily Visit :

[GOVERNMENTADDA.COM]

c) T is the nephew of Q

Place of R is not fixed, it may be the

d) either a and b

wife of S or may be the brother/sister

e) None of these

of P. But the Relation between P

Answer & Explanation

and Q can be derived from the above information

Answer – d) either a and b 10. Rakesh introduced Priya as the

Explanation :

sister of the father of my mother‟s son . How is Priya related to Rakesh. a) Aunt b) sister c) sister-in-law d) wife 9. S, the son-in-law of Q, is the brother-in-law of P who is the

e) None of these Answer & Explanation

brother of R. How is P is related to Q?

Answer – a) Aunt

a) Brother

Explanation :

b) son c) father d) data inadequate e) None of these Answer & Explanation Answer – b) son Explanation :

Direction(1-5) Read the following information carefully to answer the following questions ‗P # Q‘ means ‗P is the daughter of Q‘ ‗P © Q‘ means ‗Q is the brother of P‘ ‗P = Q‘ means ‗Q is the sister of P‘ ‗P & Q‘ means ‗P is the son of Q‘ ‗P * Q‘ means ‗P is the father of Q‘ ‗P @ Q‘ means ‗P is the mother of Q‘ GovernmentAdda.com | IBPS SBI RBI SSC RRB FCI RAILWAYS SSB

18

Daily Visit :

[GOVERNMENTADDA.COM]

1. Which of the following indicates that „A is the paternal uncle of S‟? 1.A & M # P @ K © S 2.A & M & K @ P © S

Answer Answer – 2.S * A # R @ M @T 4. Which of the following can

3.S & K & P @ M © A

be correct conclusion

4.S & P # M @ K © A

drawn from the expression

5.None of these

„E&L @ C © T = R‟? 1.C is the brother of R

Answer

2.L has two sons and two Answer – 3.S & K & P @

daughters

M©A

3.C is the sister of E 4.R is the sister of E

2. What does the expression

5.None of these

„A @ M = L © D &N‟? 1.N is the wife of A 2.M is the son of N 3.M is the daughter of N 4.N is the husband of A 5.None of these

Answer Answer – 4.R is the sister of E 5. Which of the following can be the correct conclusion

Answer

drawn from the expression Answer – 4.N is the

„T = R # U © S * K‟?

husband of A

1.K is the grandson of R 2.T is the uncle of U

3. Which of the following

3.U is the uncle of K

indicates „M is the

4.K is the niece of U

daughter of S‟?

5.None of these

1.M * A # R @ S @ T 2.S * A # R @ M @ T 3.Y @ S # O * M

Answer Answer – 5.None of these

4.Y © M = T # S 5.None of these GovernmentAdda.com | IBPS SBI RBI SSC RRB FCI RAILWAYS SSB

19

Daily Visit :

[GOVERNMENTADDA.COM]

Direction: Q(6-8)

8. A is the daughter of F. H is

K is sister of S. S is married to M. M is father

married to F. F is brother of B. Q

of R. Q is son of P. S is mother-in-law of P. M

is married to A. How H related to

has only one son and no daughter. K is

Q?

married to T. N is daughter of K.

1.Mother-in-law 2.Mother

6. How is P related to M?

3.Aunt

1.Uncle

4.Daughter

2.Son

5.None of these

3.Daughter in law

Answer

4.Grandson 5.None of these

Answer – 1.Mother-in-law

Answer & Explanation

Direction: Q(9-10)

Answer – 3.Daughter in law

M+N means M is Sister of N

Explanation :

M@N means M is Father of N M÷N means M is Son of N M%N means M is Mother of N

9. If L is grandson of M is to be true in 7. How is K related to R ? 1.Mother 2.Sister 3.Aunt 4.Mother-in-law 5.None of these Answer

the given expression,what will come in the place of ? M % G @ L ?R 1.+ 2.@ 3.% 4.÷

Answer – 3.Aunt

5.None of these Answer & Explanation GovernmentAdda.com | IBPS SBI RBI SSC RRB FCI RAILWAYS SSB

20

Daily Visit :

[GOVERNMENTADDA.COM]

Answer – 4.÷

grandfather, one grand mother and one

Explanation :

grand daughter.

M%G@L÷R M mother G Father L Son R

1. How many married couples are there in the family ? 1.Three

10. In which of the

2.One

following pairs is

3.Two

the first person

4.Four

daughter of the

5.None of these

second person with regard to the expression K @ R ÷ M +T ÷ N 1.R, T

Answer & Explanation Answer – 3.Two Explanation :

2.K, N 3.N, R 4.M, N 5.None of these Answer & Explanation Answer – 4.M, N Explanation :

2. How many female members in their family ? 1.Three

K- Father –R- son-

2.Two

M- sister –T- son-N

3.Four 4.One 5.None of these

Direction Question(1-3):

Answer

In a family there are two fathers, two mothers, one sister, one brother, two

– 3.Four

daughter, one mother-in-law, one daughterin-law, two sister-in-law, one son, one GovernmentAdda.com | IBPS SBI RBI SSC RRB FCI RAILWAYS SSB

21

[GOVERNMENTADDA.COM]

Daily Visit :

3. What is the maximum number of people in the family ?

U son :M father: R sister: T father S

1.8 5. How is P related to O in

2.7 3.5

the given expression ?

4.6

V$O#Y@Z%P

5.None of these

1.Grandmother 2.Sister

Answer

3.Wife 4.Mother

– 4.6

5.None of these Direction Question(4-6): Study the following

Answer & Explanation

information carefully to answer the given questions

Answer – 3.Wife

P @ Q means P is sister of Q

Explanation :

P % B means P is son of Q

V mother O father Y sister Z

P $ Q means P is mother of Q

son P

P # Q means P is father of Q

Hence OP => Husband &

Which of the following should come in place

wife => YZ => Children of

of ? to make the expression M is grandfather

OP

of S ? 6. A @ I % D $ F # G, what 4. U % M # R ? T # S

relationship can exist between D and G ?

1.$

1.Aunty

2.% 3.@

2.Grand Father

4.#

3.Mother

5.None of these

4.Grand Mother 5.None of these

Answer & Explanation Answer & Explanation Answer – 3.@ Explanation :

Answer – 4.Grand Mother Explanation :

GovernmentAdda.com | IBPS SBI RBI SSC RRB FCI RAILWAYS SSB

22

[GOVERNMENTADDA.COM]

Daily Visit :

A sister I son D mother F

A–B×C+D–E

father G

1.E is mother-in-law of C 2.D is brother of A

7. Z is father of M. M is

3.E is wife of B

married to B. B is mother

4.A is brother of C

of T. B has only one

5.None of these

daughter, T is married to O. R is son of O. How is B related to R ? 1.Mother

Answer & Explanation Answer – 4.A is brother of C

2.Father

Explanation :

3.Uncle

A son B father C wife D son

4.Grand mother

E

5.None of these Answer & Explanation

9. What will come in the place of ? to establish that

Answer – 4.Grand mother

M is the nephew of N in

Explanation :

the expression M ? K ÷ L × N 1.+ 2.÷ 3.× 4.-

Direction Question(8-10): P × Q means P is father of Q

5.None of these Answer & Explanation

P + Q means P is wife of Q P ÷ Q means P is daughter of Q

Answer – 4.-

P – Q means P is son of Q

Explanation : 8. Which of the following relations are true based upon the relation given in

M–K÷L×N M son K daughter L father N

the equation ? GovernmentAdda.com | IBPS SBI RBI SSC RRB FCI RAILWAYS SSB

23

[GOVERNMENTADDA.COM]

Daily Visit :

10. How is A related to E in the expression A ÷ H × G –

Answer Answer A. V is the husband of P

B÷E? 1.Daughter-in-law

2. Which of the following indicates

2.Granddaughter

that „C is the paternal uncle of D‟?

3.Daughter

A. C & V # N @ L © D

4.Grandson

B. C & V & L @ N © D

5.None of these

C. D & L & N @ V © C D. D & N # V @ L © C

Answer

E. None of these Answer – 2.Granddaughter

Answer

Explanation : A daughter H father G son B

Answer C. D & L & N @ V © C

daughter E 3. Which of the following can be the Direction(1-5) Read the following information

correct conclusion drawn from the

carefully to answer the following questions

expression „L = M # N © P * Q‟?

‗P # Q‘ means ‗P is the daughter of Q‘

A. Q is the grandson of M

‗P © Q‘ means ‗Q is the brother of P‘

B. L is the uncle of N

‗P = Q‘ means ‗Q is the sister of P‘

C. N is the uncle of Q

‗P & Q‘ means ‗P is the son of Q‘

D. Q is the niece of N

‗P * Q‘ means ‗P is the father of Q‘

E. None of these

‗P @ Q‘ means ‗P is the mother of Q‘ Answer 1. What does the expression „P @ R = S © T & V‟? A. V is the husband of P

Answer E. None of these 4. Which of the following can be

B. R is the son of V

correct conclusion drawn from the

C. R is the daughter of V

expression

D. V is the wife of P

„Q & N @ S © M = P‟?

E. None of these

A. S is the brother of P B. N has two sons and two daughters GovernmentAdda.com | IBPS SBI RBI SSC RRB FCI RAILWAYS SSB

24

[GOVERNMENTADDA.COM]

Daily Visit :

C. S is the sister of Q

D. L

D. P is the sister of Q

E. None of these

E. None of these

Answer

Answer Answer D. L Answer D. P is the sister of Q 7. Who among the following 5. Which of the following indicates

is the sister-in-law of B?

„Q is the daughter of N‟?

A. Y

A. Q * P # C @ N @ V

B. S

B. N * P # C @ Q @ V

C. U

C. M @ N # R * Q

D. J

D. M © Q = V # N

E. T

E. None of these

Answer

Answer Answer B. S Answer B. N * P # C @ Q @ V 8. How is V related to P? Directions(6-8): Study the following

A. Father-in-law

information carefully and answer the

B. Mother-in-law

questions given below:

C. Father

S is the daughter of U. V has only two

D. Mother

children – S and Y. P is the brother of B. Y is

E. Brother

married to D. V has only two daughters. J is

Answer

the mother of U. P is married to S. J is married to L. V is the son of T.

6. Who among the following

Answer A. Father-in-law Directions(Q.No: 9 & 10): Study the following

is the father of U?

information carefully and answer the

A. J

questions given below:

B. T

K is the brother of J. J is the mother of Y. Y

C. V

is the sister of T. T is married to Q. S is the father of J. S has only one daughter. S is GovernmentAdda.com | IBPS SBI RBI SSC RRB FCI RAILWAYS SSB

25

[GOVERNMENTADDA.COM]

Daily Visit :

married to R. K is the brother of D. U is the

4.Sister-in-law

father-in-law of D.

5.None of these Answer & Explanation

9. How is D related to Y? A. Cannot be determined

Answer – 4.Sister-in-law

B. Mother

Explanation :

C. Uncle D. Father E. Aunt Answer Answer C. Uncle

2. If M is brother of C then how is G related to M ?

10. If U is the father of C, then how is D related to C? A. Cousin

1.Cousin 2.Uncle 3.Brother

B. Husband C. Wife

4.Can‘t be determined 5.None of these

D. Cannot be determined E. Brother

Answer & Explanation

Answer

Answer – 4.Can‟t be determined Explanation :

Answer B. Husband 1. Q(1-3)A is sister of B. E is brother of B. E is son of H. H is married to F. F is daughter of G. F has only one daughter. G is married to C. E is brother of D. D is married to I. How is I related to A ? 1.Sister

Gender of G is not known 3. If R is daughter of I then how B is related to R ? 1.Uncle 2.Brother 3.Father 4.Can‘t be determined 5.None of these

2.Mother 3.Daughter-in-law

Answer & Explanation GovernmentAdda.com | IBPS SBI RBI SSC RRB FCI RAILWAYS SSB

26

[GOVERNMENTADDA.COM]

Daily Visit : Answer – 1.Uncle

Answer – 1.F+J-G

Explanation :

Explanation :

R daughter I brother B

F father J mother G

4. Q(4-5)A×B means A is brother of

6. Micael said, pointing to Mahavir

B

“He is son of my granfather‟s

A÷B means A is sister of B

daughter‟”.How is Micael related

A+B means A is father of B

to Mahavir ?

A-B means A is mother of B

1.Grandfather

Which of the following means P is

2.Cousin

nephew of K ?

3.Uncle

1.P×M÷K×T

4.Brother

2.T+M-P×K

5.None of these

3.K-M÷T+P

Answer & Explanation

4.K÷M-P×T 5.None of these

Answer – 2.Cousin

Answer & Explanation

Explanation : Grand father‘s daughter‘ son – son

Answer – 4.K÷M-P×T

=> Cousin

Explanation : K sister M mother P brother T

7. Q(7-8)M+N means M is Sister of N M@N means M is Father of N

5. Which of the following means F is

M÷N means M is Son of N

grandfather of G ?

M%N means M is Mother of N

1.F+J-G

What will come in the place of ?. If

2.F-J+G

P is grandson of Q is to be true in

3.H*F+G

the given expression Q % T @ P ?

4.G÷J*F

S

5.None of these

1.+

Answer & Explanation

2.÷ 3.% 4.@ 5.None of these

GovernmentAdda.com | IBPS SBI RBI SSC RRB FCI RAILWAYS SSB

27

Daily Visit :

[GOVERNMENTADDA.COM]

Answer & Explanation

Answer & Explanation

Answer – 2.÷

Answer -3.Niece

Explanation :

Explanation :

Q%T@P÷S

Grandfather –son-daughter of

Q mother T Father P Son S

daughter His sister‘s daughter

8. In which of the following pairs is the first person daughter of the

10. Pointing to a lady in photograph,

second person with regard to the

Nitesh said, “Her mother is the

expression F @ G ÷ I + J ÷ H

only daughter of my mother‟s

1.G, J

mother.” How is Nitesh related to

2.F, H

the lady?

3.H, G

1.Nephew

4.I, H

2.Uncle

5.None of these

3.Maternal uncle

Answer & Explanation Answer -4.I, H

4.Brother 5.None of these Answer & Explanation

Explanation : F- Father –G- son- I- sister –J- son-

Answer – 4.Brother

H

Explanation : Nitesh‘s mothe‘r sister‘s son =>

9. Pointing to a lady in the

brother

photograph Kiran said, “She is the daughter of the daughter of the

1. Q(1-2)M is daughter of N and

only son of my grand father.” How

sister of L. K is daughter of L and

is the lady related to Kiran ?

granddaughter of G. F is sister-in-

1.Sister

law of M. L is only sibling of M.

2.Maternal aunt

How is K related to N ?

3.Niece

1.Grandson

4.Cousin

2.Granddaughter

5.None of these

3.Daughter

GovernmentAdda.com | IBPS SBI RBI SSC RRB FCI RAILWAYS SSB

28

Daily Visit :

[GOVERNMENTADDA.COM]

4.Daughter-in-law

Answer – 1.Grandfather

5.None of these

Explanation :

Answer & Explanation Answer – 2.Granddaughter Explanation : 4. How is V related to T ? 1.Son 2.Daughter 3.Brother 2. How is F related to G ? 1.Daughter

4.Son-in-law 5.None of these

2.Mother Answer

3.Daughter-in-law 4.Mother-in-law

Answer – 4.Son-in-law

5.None of these

5. Q(5-8).A is sister of B. B is

Answer

married to D. D is father of C. H is Answer – 3.Daughter-in-law

son of G. B is mother-in-law of G. D has only one son and no

3. Q(3-4)X is married to Y. Y is mother of Z. Z is sister of V. Y has only one daughter. V is married to W. R is son of W. T is mother of W.

daughter. A is married to E. F is daughter of A. How is H related to D ? 1.Uncle 2.Son

How is X related to R ? 1.Grandfather

3.Brother 4.Grandson

2.Father

5.None of these

3.Uncle Answer & Explanation

4.Father-in-law 5.None of these Answer & Explanation

GovernmentAdda.com | IBPS SBI RBI SSC RRB FCI RAILWAYS SSB

29

Daily Visit :

[GOVERNMENTADDA.COM]

Answer – 4.Grandson Explanation :

Answer Answer – 1.Mother-in-law 9. If P+Q means P is the sister of Q, P-Q means P is brother of Q, P*Q means P is daughter of Q. Which

6. How is A related to C ?

of the following shows the relation that K is a uncle of O.

1.Mother

1.O*F-K

2.Sister

2.K*F+O

3.Aunt

3.F*K+O

4.Mother-in-law

4.O+K*F

5.None of these

5.None of these Answer – 3.Aunt

Answer – 5.None of these

7. How is F related to B ? 10. M and N are sisters. X and Y are

1.Mother

brothers. M‟s daughter is X‟s

2.Sister

sister. what is N‟s relation to Y?

3.Daughter

1.Aunt

4.Aunt

2.Sister

5.None of these

3.Mother-in-law Answer

4.Daughter 5.None of these

Answer – 4.Aunt Answer & Explanation 8. K is the daughter of I. J is married to I. I is brother of L. H is married

Answer – 1.Aunt

to K. How J related to H ?

Explanation :

1.Mother-in-law

M— N—sister

2.Mother

A(M‘s daughter – X‘s sister)

3.Aunt

X—Y—brother

4.Daughter 5.None of these GovernmentAdda.com | IBPS SBI RBI SSC RRB FCI RAILWAYS SSB

30

Daily Visit :

[GOVERNMENTADDA.COM]

GovernmentAdda.com | IBPS SBI RBI SSC RRB FCI RAILWAYS SSB

31

Daily Visit :

[GOVERNMENTADDA.COM]

120+ Direction Sense Questions With Answers Governmentadda.com

GovernmentAdda.com | IBPS SBI RBI SSC FCI RRB RAILWAYS

1

Daily Visit :

[GOVERNMENTADDA.COM]

1. From a point, Sahil starts walking in east direction. After walking for 15 m he takes a right turn. Now he walks for 12 m before turning to his right again. Next he walks 5 m and again turns in same direction as before. He now walks for 20 m before stopping at a point. How far is this point from the point where Sahil started? A) 4 √10 m B) 3 √22 m C) 7 m D) 2√41 m E) 12 m View Answer Option D Required distance = √(102 + 82) = 2√41 m 2. Abhi and Asha start cycle race from point A. They both start in east direction. After cycling for 7 m, Abhi continues straight while Asha takes a left turn. They both cycle for 6 m before turning right and left directions respectively. Next (1) Asha cycles for 8 m and takes a right turn. Now she cycles for 5 m before turning to right again. (2) Abhi cycles for 4 m and takes a left turn. Now he cycles for 6m before turning to left again. If both stop at these points, how much respective distance they have to travel to meet each other on their current paths? A) 10 m, 15 m B) 13 m, 17 m C) 15 m, 20 m D) 18 m, 24 m E) Cannot be determined View Answer

GovernmentAdda.com | IBPS SBI RBI SSC FCI RRB RAILWAYS

2

Daily Visit :

[GOVERNMENTADDA.COM]

Option C Solution:

3. From point A, Swati started walking in south direction. She walked for 4 m and took a right turn. Next she walked 5 m and turned to her left. Next she walked for 3 m and turned to her right. Next she walked 4 m and turned to her right again. Next she walked 15 m and turned to her right again and stopped at point B after walking 7 m. Find distance AB. A) 2√22 m B) 3√21 m C) 2√19 m D) 4√17 m E) None of these View Answer Option E Solution:

GovernmentAdda.com | IBPS SBI RBI SSC FCI RRB RAILWAYS

3

Daily Visit :

[GOVERNMENTADDA.COM]

AB = √(82 +22) = 2√17 m 4. Point P is 10 m west of point Q. Point R is 4 m north of point P. Point T is 3 m east of point S and point S is 5 m south of point Q. What is the direction of point R with respect to point T? A) South-east B) South C) North-east D) North-west E) West View Answer Option D 5. Anaya started from a point in some direction. After walking for some time, she turned to her right and continued walking. Now walking for some distance she turned to her left and after this finally to her right. If now she is walking in west direction, in which direction did she started her journey? A) North B) West C) East D) South E) East or west View Answer Option D Solution: Start from back and direct towards starting direction She is walking in west and before that she turned to her right, so she must be travelling in GovernmentAdda.com | IBPS SBI RBI SSC FCI RRB RAILWAYS

4

Daily Visit :

[GOVERNMENTADDA.COM]

south before turning right. Now she was walking in south and before that she turned to her left, so she must be travelling in west before turning left. Now she was walking in west and before that she turned to her right, so she must be travelling in south before turning right. At last south direction. 6. Sheetal started from point in South direction. After walking for 5 km she took a right turn. Now she walked another 5 km and took a left turn. Then after walking for 2 km she took a right turn. After covering more 2 km she turned 45o in clockwise direction. She is facing which direction now? A) South West B) South East C) North East D) North West E) None of these View Answer Option B Solution:

Directions (7-8): Point P is 5 m south of point A. Point T is 8 m east of point Q. Point Z is 4 m west of point V. Point P is 6 m west of point B. Point V is 6 m south of point T. Point Q is 4 m south of point B. 7. Find distance AZ. A) 5√13 m B) 6√13 m C) 4√14 m D) 7√15 m E) 3√11 m View Answer GovernmentAdda.com | IBPS SBI RBI SSC FCI RRB RAILWAYS

5

Daily Visit :

[GOVERNMENTADDA.COM]

Option A Solution:

Vertical distance = 5+4+6 = 15 m Horizontal distance = (PB) + (QT – CV) = 6 + (8-4) = 10 m So AZ = √(152 + 102) = 5√13 m 8. A person starts from point B in north direction. Walks for 6 m and reaches point C, takes a right turn walks for 5 m reaches point F. Again he takes a right turn, walks for 3 m, reaches point H, now takes a left turn, reaches point K, now takes a final right turn to reach point T. Find the area enclosed by points B, Q, T, K, H, F and C. A) 58m2 B) 65m2 C) 71m2 D) 76m2 E) None of these View Answer Option C Solution:

GovernmentAdda.com | IBPS SBI RBI SSC FCI RRB RAILWAYS

6

Daily Visit :

[GOVERNMENTADDA.COM]

The area will be = Ar. of rectangle QCFD + Ar. of DHKT = 10*5 + 3*7 = 71m2 Directions (9-10): Point A is 8 m west of point B. Point E is 2 m east of point F. Point G is 3 m east of point H. Point E is 3 m north of point of point D. Point C is 9 m west of point D. Point G is 9 m north of point F. Point C is 6 m south of point B. 9. Find distance AH. A) 7√6 m B) 7√5 m C) 6√6 m D) 6√5 m E) None of these View Answer Option D Solution:

GovernmentAdda.com | IBPS SBI RBI SSC FCI RRB RAILWAYS

7

Daily Visit :

[GOVERNMENTADDA.COM]

Vertical distance = GF – (CB – ED) = 9 – (6-3) = 6 Horizontal distance = AB + (CD – (HG + FE)) = 8 + (9 – (3+2) = 12 Required distance AH = √(62 + 122) = 6√5 m 10. A person starts from point G in east direction. Walks for 6 m, takes a right turn, now walks for 5 m. Now he takes a left turn, walks for 3 m, then after two consecutive right turns he reaches point E. Find the distance travelled by him to reach point E. A) 27 m B) 25 m C) 23 m D) 24 m E) 28 m View Answer Option B Solution:

6 + 5+ 3 + (9-5) + (3+4) = 25 m 1. A man walk 30 meters toward north direction, then turn to his left and walk 10 meter. Again he turn his left and walk 10 meter. How far is he from his initial point and in which direction? A) 10√5 m, south-west B) 5√5 m, north-west C) 10√2 m, south-west D) 10√5 m, north-west E) None of these View Answer Option D 2. Starting from point A, Sachin walk 20 km towards south direction. He turn left and walk 30 km, again he turn left and walk 20 km, and last he turn left and walk 40 km and reached at GovernmentAdda.com | IBPS SBI RBI SSC FCI RRB RAILWAYS

8

Daily Visit :

[GOVERNMENTADDA.COM]

point B. If point C is 20 km south of point B, then C is which direction with respect of point A? A) North-east B) North-west C) South-west D) South-east E) None of these View Answer Option C 3. B is 15 km east of point A. Point C is 15 km south of B and 7 km east of D is point C. 20 km north of point D is point E, and 13 Km east of point G is point E. What is shortest distance between A and G, and also point G is which direction respect of point A? A) 2√5 km, north-east B) 5√2 km, north-west C) 4√2 km, south-west D) 5√5 km, south-east E) None of these View Answer Option B Explanation: AG = √(52 + 52) = √50 = 5√2 km

4. A man leave for his office from his house. After moving distance of 20 km, he turn south and walked 10 km then he turn his right and after walking further 35 km, again turned his right and move for 10 km, and at last he turned his right and walked 5 km to reach his office. He is facing east direction now. If straight distance between his house and office is GovernmentAdda.com | IBPS SBI RBI SSC FCI RRB RAILWAYS

9

Daily Visit :

[GOVERNMENTADDA.COM]

30 km, then he started walking in which direction before his first south turn? A) East B) West C) North-west D) South-west E) None of these View Answer Option B Explanation: Only probable solution is the one drawn below. [If his house was 30 km west of office, he would have taken some other path to reach office] So before turning to south, he was travelling in west direction.

5. Point M is 10 km south of point N. Point O is 10 km east from point N. Point P is in west of point O at a distance of 22km. Point R is 5 km south of point P. what is shortest distance between point R and point N, also point O is which direction respect of point R? A) 13 km, north-west B) 12√2 km, south-east C) 13 km, north-east D) 13√2 km, north-east E) 14 km, south-west View Answer Option C 6. Village A is south of village B and north of village D, which is in east of village E. Village E is west of village C. Village F is between C and D anywhere. Village F is north west of village G which direction of village G respect of village B? A) North-east B) North-west C) South-east D) South-west GovernmentAdda.com | IBPS SBI RBI SSC FCI RRB RAILWAYS

10

Daily Visit :

[GOVERNMENTADDA.COM]

E) Cannot be determined View Answer Option E Directions (7-10): Read the following information to answer the questions that follow: (1) A+B = B is south of A, (2) A&B = A is west of B, (3) A*B = B is east of A, (4) A-B = A is north of B, (5) A@B = A is north-west of B, (6) A#B = B is south west of A, (7) A%B = A is north East of B, and (8) A$B = A is south east of B 7. Given that, A*B-C+D&E#F, C-F, Point A is which direction respect of point F? A) South-west B) North-east C) South D) North-west E) None of these View Answer Option D 8. Given that, P*Q-R, P@T*R, U$T, R+U, which three points are in a straight line? A) P, Q, T B) Q, R, U C) P, Q, U D) T, R, U E) None of these View Answer

GovernmentAdda.com | IBPS SBI RBI SSC FCI RRB RAILWAYS

11

Daily Visit :

[GOVERNMENTADDA.COM]

Option B Explanation:

9. Given that, A&B+D*E, E#C, B+C, point C is which direction respect of point A? A) North-west B) North-east C) North D) None of these E) Cannot be determined View Answer Option A 10. Given that, P-Q*R, R$S+T, Q&V, S+V, which three points are in a straight line? A) Q, R, T B) P, V, R C) S, V, R D) Q, V, R E) None of the 3 points together are in a straight line View Answer

GovernmentAdda.com | IBPS SBI RBI SSC FCI RRB RAILWAYS

12

Daily Visit :

[GOVERNMENTADDA.COM]

Option E Explanation:

Directions (1-5): Study the following information carefully and answer the questions that follow. A country has the following types of traffic signals. 3 green lights = go at 60 kmph speed 2 green lights = go at 40 kmph speed 1 green light = go at 20 kmph speed 3 red lights = stop 2 red lights = turn left 1 red light = turn right A person starts driving from a point in West direction and he encounters the following traffic signals: Starting point – 1 green light; After 15 minutes, 1st signal – 2 red & 2 green lights; After 24 minutes, 2nd signal – 1 red & 3 green lights; After 45 minutes, 3rd signal – 1 red & 2 green lights; After 18 minutes, 4th signal – 3 red lights; 1. Find the total distance he covered up to the last signal. A) 76 km B) 78 km C) 70 km D) 75 km E) 79 km View Answer Option B Starting in West direction: 1 green light means 20 km/hr GovernmentAdda.com | IBPS SBI RBI SSC FCI RRB RAILWAYS

13

Daily Visit :

[GOVERNMENTADDA.COM]

So 15 minutes at 20km/hr, means covers 15/60 * 20 = 5 km Next, 2 red & 2 green lights, means turn left and 24 minutes at 40 km/hr So now distance covered = 24/60 * 40 = 16 km Now for next 45 minutes, 1 red & 3 green lights, means turned right and drove at 60 km/hr, so covered 45/60 * 60 = 45 Km Next – 1 red & 2 green lights, turned right and 18 minutes at 40 km/hr So covered = 18/60 * 40 = 12 km. Next 3 red lights so stopped So total distance = 5+16+45+12 = 78 km 2. After passing the third signal if the person encounters fourth signal after half an hour, then what is his final position with respect to the starting point? A) 4 km to the south and 50 km to the east B) 55 km directly to the north-west C) 4 km to the north and 50 km to the west D) 4 km to the north and 45 km to the west E) None of these View Answer Option C Solution: So at last 1 red & 2 green lights, turned right and 30 minutes at 40 km/hr So covered 30/60 * 40 = 20 km We get:

3. If instead of starting in West direction, the man starts in South direction, then what is his position with respect to the starting point? A) 50 km to the south and 4 km to the west B) 54 km directly to the north-west C) 50 km to the north and 4 km to the west D) 50 km to the south and 4 km to the east E) None of these View Answer

GovernmentAdda.com | IBPS SBI RBI SSC FCI RRB RAILWAYS

14

Daily Visit :

[GOVERNMENTADDA.COM]

Option D Solution:

4. If after the first signal, 2nd signal: 2 red and 2 green lights, and 3rd signal: 1 red and 3 green lights, then what is the distance covered up to the last signal? A) 69 km B) 60 km C) 68 km D) 67 km E) 65 km View Answer Option A Solution: Starting in West direction: 1 green light means 20 km/hr So 15 minutes at 20km/hr, means covers 15/60 * 20 = 5 km Next, 2 red & 2 green lights, means turn left and 24 minutes at 40 km/hr So now distance covered = 24/60 * 40 = 16 km Now for next 45 minutes, 2 red & 2 green lights, means turned left and drove at 40 km/hr, so covered 45/60 * 40 = 30 Km Next – 1 red & 3 green lights, turned right and 18 minutes at 60 km/hr So covered = 18/60 * 60 = 18 km. Next 3 red lights so stopped So total distance = 5+16+30+18 = 69 km 5. If the person stops at 3rd signal, then what is his final position with respect to his starting position? A) 50 km to the north-west B) 52.5 km to the south-west C) 52.5 km to the north-east D) 50.5 km to the south-west GovernmentAdda.com | IBPS SBI RBI SSC FCI RRB RAILWAYS

15

Daily Visit :

[GOVERNMENTADDA.COM]

E) 50.5 km to the south-east View Answer Option B Solution: Starting in West direction: 1 green light means 20 km/hr So 15 minutes at 20km/hr, means covers 15/60 * 20 = 5 km Next, 2 red & 2 green lights, means turn left and 24 minutes at 40 km/hr So now distance covered = 24/60 * 40 = 16 km Now for next 45 minutes, 1 red & 3 green lights, means turned right and drove at 60 km/hr, so covered 45/60 * 60 = 45 Km Stopped here We get:

Directions (6-8): Point D is 2 km to the north of point C. Point G is 8 km to the north of point H. Point A is 15 km to the south of point B. Point C is 8 km to the east of point B. Point E is 10 km to the north of point F which is 4 km to the west of point G. Point D is 4 km to the west of point E. 6. Find shortest distance BH. A) 16√4 km B) 32 km C) 15 km D) 16√2 km E) None of these View Answer

GovernmentAdda.com | IBPS SBI RBI SSC FCI RRB RAILWAYS

16

Daily Visit :

[GOVERNMENTADDA.COM]

Option D Solution:

7. If a person after taking 2 turns reaches to point B from point F via point A, then what is the distance that he covered? A) 32 km B) 34 km C) 30 km D) 35 km E) 36 km View Answer Option B Solution:

8. If a person starts from point H and reaches point S which is south of point C, then find distance CS + HS – EF. A) 12 km B) 13 km GovernmentAdda.com | IBPS SBI RBI SSC FCI RRB RAILWAYS

17

Daily Visit :

[GOVERNMENTADDA.COM]

C) 15 km D) 14 km E) None of these View Answer Option D Solution: Point S is south of point C, so CS = 8+8 = 16 km, HS = 4+4 = 8 km and EF is 10 km So required answer = 16 +8 – 10 = 14 km 9. Priya started from point A. after walking for some time, she turned to her right and continued walked, then after some time turned to her right again. Now walking for some distance she turned to her left and after this finally to her right. If now she is walking in west direction, in which direction did she started her journey from point A? A) West B) East C) South D) North E) Cannot be determined View Answer Option B Solution: For this, start from back: She is walking in west and before that she turned to her right, so she must be travelling in south before turning right. Now she was walking in south and before that she turned to her left, so she must be travelling in west before turning left. Now she was walking in west and before that she turned to her right, so she must be travelling in south before turning right. Now she was walking in south and before that she turned to her right, so she must be travelling in east before turning right. At last east direction. 10. Tiya started from her home to office. She started in east direction. After walking for 4 km she turned to her left and walked 8 km, now she turned left and walked 2 km. After this she turned to right walked 4 km. Now after turning to her right she walked 13 km and reached office. Find the shortest distance between her office and home. A) 3√43 m B) 3√41 km C) 4√41 m D) 5√38 m E) None of these

GovernmentAdda.com | IBPS SBI RBI SSC FCI RRB RAILWAYS

18

Daily Visit :

[GOVERNMENTADDA.COM]

View Answer Option B Solution: √152 + 122 = 3√41 km Direction Sense Questions for SBI PO, IBPS PO/Clerk, IBPS RRB, NIACL, NICL, RBI Assistant, OICL, UIICL, BoB and other competitive exams. 1. Suman walks 10km towards the South. Turning to the left, she walks 20km and then moves to her right. After moving a distance of 20km, she turns to the right and walks 20km. Finally, she turns to the right and moves a distance of 10km. How far and in which direction is she from the starting point? A) 10km North B) 20km South C) 20km North D) 10km South E) None of these View Answer Option B 2. Manish walked 6km facing towards East, then he took a right turn and walked a distance of 9km. he then took a left turn and walked a distance of 6km. How far is he from the starting point? A) 15 km B) 21km C) 18km D) 15km E) None of these View Answer Option A 3. Rohit walked 30 metres towards South, took a left turn and walked 50 metres, again he took a left and walked 30 metres. How far is he and in which direction from the starting point? A) 80m south B) 50m west C) 130m east D) 50m east E) None of these

GovernmentAdda.com | IBPS SBI RBI SSC FCI RRB RAILWAYS

19

Daily Visit :

[GOVERNMENTADDA.COM]

View Answer Option D 4. Sharma walked 30 metres towards South, took a left turn and walked 15 metres. He then took a right turn and walked 20 metres. He again took a right turn and walked 15 metres. How far is he from the starting point? A) 95 metres B) 50 metres C) 70 metres D) Cannot be determined E) None of these View Answer Option B 5. Ruhi is facing South. She turn right and walk 20 m. Then she turn right again and walk 10 m. Then she turn left and walk 10 m and then turning right and walk 20 m. Then she turn right again and walk 60 m. She is in which direction from her starting point? A) North B) North-west C) East D) North-east E) None of these View Answer Option D Directions (6-7): A is 20 m away from C in east direction.B is standing in south of A and is facing south direction and distance between A and B is 30m. Now to the right of B covering 32 m towards west,D is standing.D is eating burger standing on its position and facing north. After eating burger D starts moving towards south east direction covering 13 m and reaches to the position of E. 6. What is the distance between D and C and C is in which direction with respect to D? A) 31.2m, north-east B) 29.4m, north-west C) 32.3m, north-east D) 40.8m, north-west E) Cannot be determined

GovernmentAdda.com | IBPS SBI RBI SSC FCI RRB RAILWAYS

20

Daily Visit :

[GOVERNMENTADDA.COM]

View Answer Option C

7. In which direction C is with respect to B? A) South – West B) North – East C) South D) North – West E) None of these View Answer Option D Direction (8-10): Read the given information carefully and answer below Question.There are 7 family members P, Q, R, S, T, U and V standing in ground in which there are 2 married couples. P is sister of Q who is maternal grandson of T. Maternal grandfather of Q is standing 3m to the right of Q who is facing north. The father of S has 2 maternal grandchildren. V is facing north. V is standing 4m to the south of maternal grandson of U. S is 2m to the right of V. P is 1m south of S and 1m west of U. R is sister-in-law of V and standing 9m to the north of her mother. V is father of P. U is a Female. 8. Maternal grand daughter is standing in which direction w.r.t his husband? A) south-east B) south C) north-west D) south-west GovernmentAdda.com | IBPS SBI RBI SSC FCI RRB RAILWAYS

21

Daily Visit :

[GOVERNMENTADDA.COM]

E) None of these View Answer Option D

9. What is direction and distance and relationship of S with respect to P? A) 1m north, Mother B) 1m south, Sister C) 1m north, Mather – in law D) 1m north, Daughter

GovernmentAdda.com | IBPS SBI RBI SSC FCI RRB RAILWAYS

22

Daily Visit :

[GOVERNMENTADDA.COM]

E) None of these. View Answer Option A 10. What is a minimum distance between V and His father – in – Law? A) 3m B) 2m C) 4m D) 6m E) None of these View Answer Option E 1. A person starts from point T in east direction. Walks 6 m and turns right. Next walks 4 m and turns left. Next walks 3m and turns right. Now cycles for 8 km and stops. Find his distance from T. A) 17 m B) 2√31 m C) 15 m D) 7√21 m E) 12 m View Answer Option C Solution: Distance = √[(6+3)2 + (4+8)2] = 15 m Directions (2-4): Point A is 8 m north of point B. Point B is 10 m west of point C. Point C is 11 m north of point D. Point F is 4 m north of point E which is 6m west of point D. 2. A person starts from point F, reaches point G, then takes a left and then a right turn to reach point B. Find FG + BC. A) 13 km B) 12 km C) 15 km D) 14 km E) None of these View Answer GovernmentAdda.com | IBPS SBI RBI SSC FCI RRB RAILWAYS

23

Daily Visit :

[GOVERNMENTADDA.COM]

Option D Solution: Since he takes a left and a right turn to reach B, so the figure is like:

So FG = 10 – 6 = 4m and then FG + BC = 4 + 10 3. Point T is 5m north of point C. Find AT. A) 29 m B) 2√29 m C) 5√26 m D) 29√2 m E) None of these View Answer Option B Solution:

AT = √(102 + 42) = 2√29 m 4. Point H is 6 m west of point F. Point J is 3 m south of point D. A person starts from point H in south direction, reaches a point K, takes a left turn and reaches point J. Find KE. A) 15 m B) 3√10 m C) 2√5 m GovernmentAdda.com | IBPS SBI RBI SSC FCI RRB RAILWAYS

24

Daily Visit :

[GOVERNMENTADDA.COM]

D) 3√2 m E) 3√5 m View Answer Option E Solution: We get figure as:

So KE = √(62 + 32) = 3√5 m 5. A person starts from a point in east direction. He walks 10 m and takes a right turn. Now he walks 5m and again takes a right turn. Next he walks 3 m and takes a left turn. Now he walks 6 m and takes a right turn. He finally stops after walking 8m. In which direction is the starting point with respect to the ending point? A) South-east B) South-west C) North D) North-east E) None of these View Answer

GovernmentAdda.com | IBPS SBI RBI SSC FCI RRB RAILWAYS

25

Daily Visit :

[GOVERNMENTADDA.COM]

Option D Solution:

6. A person starts from point A, walks 10 m in south direction. Now he takes a left turn and walks 8m before turning left again. Next he walks 16 m and takes a right turn. Now he walks 4 m and takes a right turn again. He stops after walking 10 m. Find his distance from point A. A) 20 km B) 3√10 km C) 4√10 km D) 4√5 km E) 10 km View Answer Option C Solution:

Required distance = √((8+4)2 + 42) = 4√10 km Directions (7-9): Point A is 10 m west of point B. Point B is 6 m south of point C and also 7 m north of point D. Point E is 4 m west of point D. Point C is 6 m east of point F. 7. A person starts from point F, walks 2 m in south direction and reaches a point T. He takes a right turn and reaches point K, north of point A. Find TK + BD – DE. A) 8 m B) 7 m GovernmentAdda.com | IBPS SBI RBI SSC FCI RRB RAILWAYS

26

Daily Visit :

[GOVERNMENTADDA.COM]

C) 9 m D) 6 m E) 5 m View Answer Option B Solution: TK = 10 – 6 = 4 m So TK + BD – DE = 4 + 7 – 4 8. Find distance AE. A) 7√5 m B) 8 m C) 10 m D) 8√3 m E) 5√8 m View Answer Option A Solution: AE = √((10+4)2 + 72) = 4√10 km 9. A person starts from point D, walks 10 m in north direction and reaches point S. He takes a right turn, walks 2 m and reaches point X. Find distance XE. A) 2√24 m B) 5√13 m C) 2√13 m D) 3√29 m E) 2√26 m View Answer Option E Solution:

GovernmentAdda.com | IBPS SBI RBI SSC FCI RRB RAILWAYS

27

Daily Visit :

[GOVERNMENTADDA.COM]

XE = √(102 + 22) = 2√26 m 10. A person starts his journey by walking in West direction. He walks for 7 m and takes a left turn. Next after travelling a distance of 10 m, he turned to his right and travelled 4 m. Next he walks for 3 m towards North direction and turns 45o in clockwise direction. In what direction is he travelling now? A) South-west B) North-east C) North-west D) South-east E) East View Answer Option D

Directions (1-2): Point A is 10 km south of point B. Point C is 7 km east of point B. Point A is 4 km west of point D. Point D is 5 km north of point E. 1. A person starts from point E, goes to point F in east direction. After that he takes a left turn and reaches point C. Find distance DF. A) 17 km B) √34 km C) 5√31 km D) 7√24 km E) 2√34 km GovernmentAdda.com | IBPS SBI RBI SSC FCI RRB RAILWAYS

28

Daily Visit :

[GOVERNMENTADDA.COM]

View Answer Option B

After left turn he reaches point C, this means point F is in south of point C. So EF= 3km so DF = √(DE2 + EF2) = √(52 + 32) = √34 km 2. A person starts from point G which is 10 km west of point E. Then he goes towards north and after taking a right turn he reaches point A. Now he turns to his right and reaches a point T on line EG. Find distance GT. A) 8 km B) 12 km C) 5 km D) 6 km E) None of these View Answer

GovernmentAdda.com | IBPS SBI RBI SSC FCI RRB RAILWAYS

29

Daily Visit :

[GOVERNMENTADDA.COM]

Option D

GT = 10 – 4 = 6km 3. A person starts from a point and goes 6 km in north direction. Now he takes a right turn and moves 7 km. Next he takes a left turn and moves 10 km. Next he turns right and moves 5 km. Finally he turns right and moves 12 km to reach his destination. Find the distance from his starting point. A) 12 km B) 4√10 km C) 5√5 km D) 6√2 km E) None of these View Answer Option B Required distance is √(122 + 42) = 4√10 km Directions (4-5): Point B is 9 km east of point A. Point T is 6 km south of point B. Point S is 4 km west of point T. Point T is 8 km west of point G. 4. A person starts from point S, walks 3 km towards south and reaches point D. Next he takes two right turns and reaches point A. How much distance has he travelled? A) 18 km B) 20 km C) 17 km D) 14 km E) 11 km GovernmentAdda.com | IBPS SBI RBI SSC FCI RRB RAILWAYS

30

Daily Visit :

[GOVERNMENTADDA.COM]

View Answer Option A Let after first right turn he is at point K. Since after turn from K he reaches at point A so point K is in south of point A. So he has travelled SD + DK + KA = 3 + (9-4) + (3+6) = 17 km 5. A person is standing on midpoint of line TG. Find his shortest distance from point B? A) 3√15 km B) 5√14 km C) 6√3 km D) 2√13 km E) None of these View Answer Option D Let that point be Y. So Midpoint of TG means TY = 8/2 = 4 km So BY = √(BT2 + TY2) = √(62 + 42) = 2√13 km 6. A person starts from a point in east direction. He walks 6 km and turns to his left. Next he walks 4 km and turns to his left again. Next he walks 2 km and turns towards south direction. He stops after walking 8 km. Find his distance from the starting point. A) 16 km B) 3√2 km C) 4√2 km D) 4√4 km E) 8 km View Answer Option C Required distance = √(42 + 42) = 4√2 km 7. From a point, a person starts walking in south direction. He takes a right turn, then taken 2 lefts turns and then takes two right turns and stops after walking 3 km. In which direction he is standing with respect to the starting point? A) West B) East C) North D) South E) Cannot be determined GovernmentAdda.com | IBPS SBI RBI SSC FCI RRB RAILWAYS

31

Daily Visit :

[GOVERNMENTADDA.COM]

View Answer Option E

Since we do not know that how much distances has he travelled after each turn, his position with respect to starting point cannot be determined. There can be many cases as: Directions (8-10): Point B is 7 km north of point A. Point A is 8 km east of point G. Point B is 12 km east of point C. Point E is 9 km north of point D which is 12 km east of point A. 8. A person starts from point S, walks 9 km and reaches point T mid-way between points A and D. Then he goes 2 km towards south, takes 2 left turns and reaches point D. How much distance has he travelled? A) 15 km B) 18 km C) 20 km D) 19 km E) 21 km View Answer

GovernmentAdda.com | IBPS SBI RBI SSC FCI RRB RAILWAYS

32

Daily Visit :

[GOVERNMENTADDA.COM]

Option D

He has travelled = 9 + 2 + 6 + 2 = 19 km 9. A person starts from point D in south direction. He walks 5 km and reaches point L and then turns towards west and reaches point K which is south of point C. Find distance LK – BC. A) 13 km B) 12 km C) 24 km D) 16 km E) Cannot be determined View Answer Option B LK = 12+12 = 24 km. BC = 12 km, so required ans = 24-12 = 12 km 10. If point M is 4 km north of point G and point O is south if point G such that point G is mid way between points M and N. Find distance MN + AE. A) 23 km B) 25 km C) 19 km D) Cannot be determined E) None of these View Answer GovernmentAdda.com | IBPS SBI RBI SSC FCI RRB RAILWAYS

33

Daily Visit :

[GOVERNMENTADDA.COM]

Option A MN = 4+4 = 8 km AE = √(AD2 + DE2) = √(122 + 92) = 15 km so required ans = 8 + 15 = 23 km Direction: Q(1-5) There are 5 friends A, B, C, D and E standing randomly. B is to the northeast of E. D is 2km to the east of E, who is 6km to the west of A. C is to the northwest of D and in the line of EB. D is 4km the south of B. 1. In which direction is C with respect to A ? A.South west B.South east C.Northeast D.Northwest E.None of these Answer & Explanation Answer – D.Northwest

Explanation : 2. In which direction is A with respect to B ? A.Southeast B.Southwest C.Northwest D.Northeast E.None of these Answer Answer – A.Southeast

GovernmentAdda.com | IBPS SBI RBI SSC FCI RRB RAILWAYS

34

Daily Visit :

[GOVERNMENTADDA.COM]

3. What is the distance between D and A ? A.5km B.4km C.6km D.3km E.None of these Answer Answer – B. 4km 4. What is the shortest distance between B and A? A.5√7km B.4√2km C.6√2km D.3√5km E.None of these Answer & Explanation Answer – B. 4√2km Explanation : X2 = 42 + 42 X = 4√2 5. What is the shortest distance between between B and E? A.2√7km B.5 √2km C.7√2km D.2√5km E.None of these Answer & Explanation Answer – D. 2√5km Explanation : GovernmentAdda.com | IBPS SBI RBI SSC FCI RRB RAILWAYS

35

Daily Visit :

[GOVERNMENTADDA.COM]

X2 = 42 + 22 X = 2√5 Directions : Q(6-7) – Bala walked 25km towards west, took a left turn and walked 15km. He again took a left turn and walked 30km. He then took a right turn and stopped. 6. Now he was facing which direction ? A.West B.East C.South D.North E.None of these Answer & Explanation Answer – C.South Explanation :

7. Instead of turning right at the end if he took left and walked 20km, what is the shortest distance to his starting point? A.3√7km B.2√5km C.7√2km D.5√2 km E.None of these Answer & Explanation

GovernmentAdda.com | IBPS SBI RBI SSC FCI RRB RAILWAYS

36

Daily Visit :

[GOVERNMENTADDA.COM]

Answer – D. 5√2 km Explanation : X2 = 52 + 52 X = √50 = 5√2 8. Raghav starts walking in south direction and walks a distance of 7 meters. Now he tooks a left turn and walk 6m. Again he takes a left turn and walk 15m and reached a point P. Find the distance between starting point and P and in which direction is the person from the initial point. A.10m, south east B.10m, north east C.20m, north west D.20m, south west E.None of these Answer & Explanation Answer – B.10m, north east

Explanation : X2 = 62 + 82 X = √100 = 10 9. Dheepthi started from point A in south direction. After walking for 4 m she turned to her right and walked 5 m. Now she turned to her left and walked 3 m after which she turned to her right. Now she walked 4 m and turned to her right GovernmentAdda.com | IBPS SBI RBI SSC FCI RRB RAILWAYS

37

Daily Visit :

[GOVERNMENTADDA.COM]

again and walked 15 m. Now finally she turned to her right and after walking for 7 m, she stopped at point B. What is the distance AB? A.2√34 m B.34 m C.3√17 m D.2√17 m E.None of these Answer & Explanation Answer – D. 2√17 m Explanation :

X^2 = 8^2 + 2^2 X = √68= 2√17 10. Riya started from her home to office. She started in east direction. After walking for 4 m she turned to her left and walked 8 m, now she turned left and walked 2 m. After this she turned to right walked 4 m. Now after turning to her right she walked 13 m and reached office. Find the shortest distance between her office and home. A.87 m B.9√41 m C.26 m

GovernmentAdda.com | IBPS SBI RBI SSC FCI RRB RAILWAYS

38

Daily Visit :

[GOVERNMENTADDA.COM]

D.3√41 m E.None of these Answer & Explanation Answer – D. 3√41 m Explanation :

X2 = 152 + 122 X = √369= 3√41

Riya starts walking in the north direction and after walking some distance she took a left turn followed by a right turn. After that she took two consecutive left turn, now she is walking in which direction? a) south b) north c) east d) west e) None of these Answer & Explanation Answer – a) south

Explanation :

GovernmentAdda.com | IBPS SBI RBI SSC FCI RRB RAILWAYS

39

Daily Visit :

[GOVERNMENTADDA.COM]

Rahul walks a distance of 10 km towards south, then he turn to his left and walks 5 km. From here he took a right turn and walks 6 km and stops at a point A. Find the distance between the starting point and A and A is in which direction with respect to starting point. a) 17km, north east b) 17km south east c) 17km north west d) 17km south west e) None of these Answer & Explanation Answer – b) 17km south east

Explanation : (approx.)

Distance = 16^2 + 5^2 = √281 = 17km

Neha travelled from a point X straight to point Y at a distance of 50 meters. He turned to his right and walks 50 meter more, then again turned right and walks 60 meter. Finally, he turned to right and walks 50 meters. How far is he from the starting point? a) 10 b) 20 c) 30 d) 40 e) None of these Answer & Explanation Answer – a) 10

Explanation : Nishant walks 30 meter in the north direction, after that he took a right turn and walks 40 meter. After that he took a right turn and walks 40 meter more and finally he took a right turn and stop after walking 40 meter. Find the distance of nishant from the initial position? GovernmentAdda.com | IBPS SBI RBI SSC FCI RRB RAILWAYS

40

Daily Visit :

[GOVERNMENTADDA.COM]

a) 5 b) 10 c) 15 d) 20 e) None of these Answer & Explanation Answer – b) 10

Explanation : From his house, Ram went 15 kms to the north. Then he turns west and covered 20 km. Then he turned south and covered 5 km. Finally turning to east, he covered 25 km. In which direction is he from his house? a) north west b) north east c) south east d) south west e) None of these Answer & Explanation Answer – b) north east

Explanation : A man walks 40 meters towards north. Then turning to his right, he walks 50 meter. Then turning to his left, he walks 30 meters. Again he turns to his left and walks 40 meters. How far is he from initial position? a) 40√2 b) 50√2 c) 60√2 d) 50√3 e) None of these Answer & Explanation

GovernmentAdda.com | IBPS SBI RBI SSC FCI RRB RAILWAYS

41

Daily Visit :

[GOVERNMENTADDA.COM]

Answer – b) 50√2

Distance = √(70^2 + 10^2) = √5000

Explanation : = 50√2

Riya goes 30 km towards North from a fixed point, then after turning to her right she goes 15 km. After this she goes 30 km after turning to her right. How far and in what direction is she from her starting point? a) 10m east b) 15m east c) 20m east d) 25m east e) None of these Answer & Explanation

Answer – b) 15m eastExplanation : A person starts walking from his home in west direction and after walking 20 meter he took a left turn and walk 30 meters. Now he took a right turn and walks 10 meter to reach the bus stand. Find the distance between home and stand a) 20√2 b) 30√2 c) 40√2 d) 50√2 e) None of these Answer & Explanation

GovernmentAdda.com | IBPS SBI RBI SSC FCI RRB RAILWAYS

42

Daily Visit :

[GOVERNMENTADDA.COM]

Answer – b) 30√2

Explanation : m

Distance = √(30^2 + 30^2) = 30√2

A girl rides her bicycle southwards, then turned right and rode 2 km and again turned right and rode 4 km. She found himself exactly 2 km east from the starting point. How far did she ride southwards initially? a) 2km b) 3 km c) 4 km d) 6 km e) None of these Answer & Explanation Answer – c) 4 km

Explanation : One day raj left home and walked 5 km northwards, turned right and walked for 10km and turned left and walked 5 km more and finally turned left and walked 10km. How many kilometres will he have to walk to reach his home straight? a) 15 km b) 10 km c) 5 km d) 20 km e) None of these Answer & Explanation

GovernmentAdda.com | IBPS SBI RBI SSC FCI RRB RAILWAYS

43

Daily Visit :

[GOVERNMENTADDA.COM]

Answer – b) 10 km

Explanation : A person starts walking from a point A in north direction and after covering 20 meter, he took a right turn and walk 20 meter more. After that he turns to his right and walks 40 meter before turning to left and walks 20 meters more. After that he took a left turn and walks 10 meter and finally took a right turn and walk 50 meters and stopped at point B. Now he is facing which direction? a) east b) west c) north d) south e) None of these Answer & Explanation Answer – a) east

Explanation : A person starts walking in south and after walking 20 meters he took a left turn and walks 30 meter and finally took a right turn and stopped after walking 40 meters. Find the distance between his initial position to final position? a) 20√5 b) 30√5 c) 40√5 d) 55√5 e) None of these Answer & Explanation

GovernmentAdda.com | IBPS SBI RBI SSC FCI RRB RAILWAYS

44

Daily Visit :

[GOVERNMENTADDA.COM]

Answer – b) 30√5

Explanation : sqrt(4500) = 30√5

distance = 60^2 + 30^2 =

A dog run 20m towards East and Turns to right runs 10m and turns to right runs 10m and again turns to left run 5m and then turns to left runs 12m and finally turns to left and runs 5m .Now which direction cat facing? a) south b) north c) east d) west e) None of these Answer & Explanation Answer –b) north

Explanation : Two persons A and B are at a distance of 10 meters from each other in west-east direction respectively. A starts walking in north and B starts walking south and move 5 meter respectively. Then A and B takes right and left turn respectively and stopped after travelling 5 meter each. Find the distance between both of them a) 10√3 b) 10√5 c) 10√2 d) 10√7 e) None of these Answer & Explanation

GovernmentAdda.com | IBPS SBI RBI SSC FCI RRB RAILWAYS

45

Daily Visit :

[GOVERNMENTADDA.COM]

Answer – c) 10√2

Explanation : Amit starts walking in east direction and after travelling some distance he took a right turn and then a left turn followed by another left turn. Now he again took a right turn and finally took a left turn. In which direction is Amit walking. a) south b) north c) east d) west e) None of these Answer & Explanation Answer – b) north

Explanation : Rina goes 30 km towards North from a fixed point, then after turning to her right she goes 15 km. After this she goes 30 km after turning to her right. How far and in what direction is she from her starting point? a) 10m east b) 15m east c) 20m east d) 25m east e) None of these Answer & Explanation

GovernmentAdda.com | IBPS SBI RBI SSC FCI RRB RAILWAYS

46

Daily Visit :

[GOVERNMENTADDA.COM]

Answer –b) 15m east

Explanation : A person starts walking in south direction and walks a distance of 7 meters. Now he took a left turn and walk 6m. Again he takes a left turn and walk 15m and reached a point P. Find the distance between starting point and P and in which direction is the person from the initial point. a) 10m, south east b) 10m North West c) 10m, north east d) 10m, south west e) None of these Answer & Explanation Answer – c) 10m, north east

Explanation : Nikhil starts walking in east direction and after 10 m he took a right turn and walks 10 meter to reach stationery. From the stationary he took a left turn and walks for 20 meter to reach his aunt home. After this he took a right turn and walks 10 meter to reach his school. In which direction is his school from the starting point? a) north east b) south west c) south east d) north east e) None of these Answer & Explanation GovernmentAdda.com | IBPS SBI RBI SSC FCI RRB RAILWAYS

47

Daily Visit :

[GOVERNMENTADDA.COM]

Answer – c) south east

Explanation : Priya starts running in west direction and after some distance she turns to her left and cover some distance. After this she took a left turn and then again left turn and run some distance. After that she took a right turn and finally a left turn. Now in which direction she is running. a) north b) south c) east d) west e) None of these Answer & Explanation Answer – a) north

Explanation : Neha starts walking in a direction then she took a left turn and after walking some distance he took right turn. After walking for some distance she took two consecutive right turn and finally a left turn. Now if she is walking in north direction, in which direction it starts? a) north b) south c) west d) east e) None of these Answer & Explanation

GovernmentAdda.com | IBPS SBI RBI SSC FCI RRB RAILWAYS

48

Daily Visit :

[GOVERNMENTADDA.COM]

Answer – c) west

Explanation : A person starts walking in north to his house and walks 50m. Now he took a right turn and walks 20m and after that he took another right and walks 20m. Now he is moving towards his house. In which direction he is walking? a) south east b) south west c) north east d) north west e) None of these Answer & Explanation Answer – b) south west

Explanation : A person starts walking in east direction and walks 20m. After that he turn to his right and walks 10m and then turn to his left and walks 15m and reached at a point A. Find the distance between A and initial point a) 5√51 b) 5√53 c) 5√57 d) 5√59 e) None of these Answer & Explanation

GovernmentAdda.com | IBPS SBI RBI SSC FCI RRB RAILWAYS

49

Daily Visit :

[GOVERNMENTADDA.COM]

Answer – b) 5√53

Explanation : A person starts walking in north direction and after walking some distance he turns to 45 degree in clockwise direction. After that he turns 180 degree anti clockwise and again 45 degree in clockwise direction. Now in which direction he is walking a) west b) east c) north d) south e) None of these Answer & Explanation Answer – a) west

Explanation : Anil starts walking in east direction and after travelling some distance he took a right turn and then a left turn followed by another left turn. Now he again took a right turn and finally took a left turn. In which direction is anil walking. a) south b) north c) east d) west e) None of these Answer & Explanation Answer – b) north

Explanation : GovernmentAdda.com | IBPS SBI RBI SSC FCI RRB RAILWAYS

50

Daily Visit :

[GOVERNMENTADDA.COM]

A person starts walking in south direction and walks a distance of 7 meters. Now he tooks a left turn and walk 6m. Again he takes a left turn and walk 15m and reached a point P. Find the distance between starting point and P and in which direction is the person from the initial point. a) 10m, south east b) 10m, north west c) 10m, north east d) 10m, south west e) None of these Answer & Explanation Answer – c) 10m, north east

Explanation : Riya starts running in west direction and after some distance she turns to her left and cover some distance. After this she took a left turn and then again left turn and run some distance. After that she took a right turn and finally a left turn. Now in which direction she is running. a) north b) south c) east d) west e) None of these Answer & Explanation Answer – a) north

Explanation : GovernmentAdda.com | IBPS SBI RBI SSC FCI RRB RAILWAYS

51

Daily Visit :

[GOVERNMENTADDA.COM]

Two Person P and Q are separated by a distance of 20 meter in west –east direction respectively. Now P and Q start walking in north and south direction respectively and walked for 5 meter. Now P and Q took a right turn and walked 10m each. Now P and Q took left turn and after walking 5 meter both of them stopped. Find the distance between them a) 15 b) 25 c) 30 d) 35 e) None of these Answer & Explanation Answer – e) None of these

Explanation :

20 meter.

P walks 8m to the south ,then he turn to his left and walks 15m then he turn to his right and walk 12m again he turns to his right and walk 15m and turn right and stopped how far and in which direction from the starting point a) 10 north b) 20 south c) 20 east d) 20 west e) None of these Answer & Explanation

GovernmentAdda.com | IBPS SBI RBI SSC FCI RRB RAILWAYS

52

Daily Visit :

[GOVERNMENTADDA.COM]

Answer – b) 20 south

Explanation : A boy is facing south direction. He turns 60 degree in anti-clockwise direction and then 120 degree in clockwise direction. In which direction is he facing? a) south east b) north east c) south west d) north west e) None of these Answer & Explanation Answer – c) south west

Explanation : A postman was returning to the post office which was in front of him to the south. When the post office is 80m away from him, he turned to the right and walks 50m to deliver the last letter. He continues walks in the same direction for 20 meter and then took a left turn and walks 80 meter. How many meters was he away from the post office a) 30 b) 40 c) 60 d) 70 e) None of these Answer & Explanation

GovernmentAdda.com | IBPS SBI RBI SSC FCI RRB RAILWAYS

53

Daily Visit :

[GOVERNMENTADDA.COM]

Answer – d) 70

Explanation :

GovernmentAdda.com | IBPS SBI RBI SSC FCI RRB RAILWAYS

54

Daily Visit

[GOVERNMENTADDA.COM]

New Pattern Input Output / Machine Input Questions With Solution Governmentadda.com

GovernmentAdda.com | IBPS SBI RBI SSC RRB FCI RAILWAYS

1

Daily Visit

[GOVERNMENTADDA.COM]

I. Study the following information carefully and answer the given questions. A number arrangement machine arranges two digit numbers into a typical manner. Each step takes gives output taking input from the previous step. The following is an illustration of Input and rearrangement. Using the illustration answer the question given below.

Example:

Input:

Explanation

GovernmentAdda.com | IBPS SBI RBI SSC RRB FCI RAILWAYS

2

Daily Visit

[GOVERNMENTADDA.COM]

Step I: Multiply the first digit of first number with second digit of fourth Number. Multiply the second digit of first number with first digit fourth number.

Step II: Add the first digit of all numbers in Step I for the first number and second digit of all numbers in Step I for the second number

Step III: Divide second digit by first digit

Step IV: First number is subtracted from the second number. 1. If the value “5” is subtracted from the final output then what will be the resultant value? A. -7 B. 3 C. -3 D. 4 E. None of these Answer Answer – A. -7 2. If in the first step the first digit of every number is added and multiplied by 5 then which will be the resultant value? A. 50 B. 60 C. 55 D. 65 E. None of these Answer Answer – B. 60 3. Which of the following combinations represent the first digit of the second value and the second digit of the first value in Step I of the given input? GovernmentAdda.com | IBPS SBI RBI SSC RRB FCI RAILWAYS

3

Daily Visit

[GOVERNMENTADDA.COM]

A. 6, 4 B. 4, 6 C. 6, 2 D. 2, 8 E. 2, 4 Answer Answer – C. 6, 2 4. Which of the following represents the sum of the first digit of the second value and the second digit of the first value in Step II of the given input? A. 5 B. 2 C. 6 D. 4 E. 3 Answer Answer – B. 2 5. Which of the following represents the difference between the first value and the second value of Step II of the given input? A. 8 B. 7 C. 9 D. 4 E. 6 Answer Answer – D. 4 I. Study the following information carefully and answer the given questions. A number arrangement machine arranges two digit numbers into a typical manner. Each step takes gives GovernmentAdda.com | IBPS SBI RBI SSC RRB FCI RAILWAYS

4

Daily Visit

[GOVERNMENTADDA.COM]

output taking input from the previous step. The following is an illustration of Input and rearrangement. Using the illustration answer the question given below.

Example:

Input:

Explanation

GovernmentAdda.com | IBPS SBI RBI SSC RRB FCI RAILWAYS

5

Daily Visit

[GOVERNMENTADDA.COM]

Step I: Multiply the first digit of first number with second digit of fourth Number. Multiply the second digit of first number with first digit fourth number.

Step II: Add the first digit of all numbers in Step I for the first number and second digit of all numbers in Step I for the second number and then multiply by 2.

Step III: Divide second digit by first digit

Step IV: Add two numbers. 6. If the value “6” is added to the final output then what will be the resultant value? A. 12 B. 18 C. 10 D. 11 E. None of these Answer Answer – B. 18 7. If in the first step the second digit of every number is added and divided by 2 then which will be the resultant value? A. 5 B. 6 C. 7 D. 4 E. None of these Answer Answer – C. 7 8. Which of the following combinations represent the first digit of the third value and the second digit of the first value in Step I of the given input? GovernmentAdda.com | IBPS SBI RBI SSC RRB FCI RAILWAYS

6

Daily Visit

[GOVERNMENTADDA.COM]

A. 4, 1 B. 1, 4 C. 2, 6 D. 4, 6 E. 4, 4 Answer Answer – D. 4, 6 9. Which of the following represents the sum of the second digit of the second value and the first digit of the first value in Step II of the given input? A. 8 B. 7 C. 6 D. 4 E. 9 Answer Answer – E. 9 10. Which of the following represents the difference between the first value and the second value of Step III of the given input? A. 1 B. 2 C. 0 D. 4 E. 5 Answer Answer – D. 4 I. Study the following information carefully and answer the given questions.

GovernmentAdda.com | IBPS SBI RBI SSC RRB FCI RAILWAYS

7

Daily Visit

[GOVERNMENTADDA.COM]

The following is an illustration of Input and rearrangement. Using the illustration answer the question given below.

Step-I: Interchange the Alphabets/Numbers(follow the same pattern as shown in Figure.) Step-II: (a) If both letters are Vowel and number is less than 6, then vowels change to next letter in English alphabetical series and add 2 to the number (b) If both letters are consonant and number is greater than 6 or equal to, then consonants change to the previous letter in English alphabetical series and subtract 3 from the number (c) If both letters are Vowel and number is greater than 6 or equal to, then vowels change to the previous letter in English alphabetical series and subtract 3 from the number (d) If both letters are consonant and number is less than 6, then consonants change to next letter in English alphabetical series and add 3 to the number (e) If there are one vowel and one consonant, then vowel change to next letter and consonant change to the previous letter and add 2 to the number. (f) If there is single consonant, then consonant change to the previous letter and Subtract 3 from the number. (g) If there is a single vowel, then vowel change to next letter and add 3 to the number. Step-III: Follow Both Steps I and II

Example:

GovernmentAdda.com | IBPS SBI RBI SSC RRB FCI RAILWAYS

8

Daily Visit

[GOVERNMENTADDA.COM]

Input:

Explanation

GovernmentAdda.com | IBPS SBI RBI SSC RRB FCI RAILWAYS

9

Daily Visit

[GOVERNMENTADDA.COM]

1. In Step III, what is the sum of numbers in the first row? A. 15 B. 11 C. 12 D. 13 E. None of these Answer Answer – D. 13 2. In Step III, what is the difference between the sum of numbers in the first row and the sum of numbers in the third row? A. 5 B. 6 C. 3 D. 4 E. None of these Answer Answer – C. 3 3. In Step II, what is the product of the sum of numbers in the first column and the sum of numbers in the third column? A. 245 B. 285 C. 275 D. 255 E. 235 Answer Answer – D. 255

GovernmentAdda.com | IBPS SBI RBI SSC RRB FCI RAILWAYS

10

Daily Visit

[GOVERNMENTADDA.COM]

4. In Step II, If the sum of the numbers in the third row is divided by the sum of numbers in the second row then what will be the resultant? A. 8 B. 7 C. 6 D. 4 E. 2 Answer Answer – E. 2 5. In Step I, which of the following letter/number occur more than twice? A. E B. U C. 7 D. 4 E. 8 Answer Answer – D. 4 II. Study the following information carefully and answer the given questions.

The following is an illustration of Input and rearrangement. Using the illustration answer the question given below.

Step-I: Interchange the Alphabets/Numbers(follow the same pattern as shown in Figure.) Step-II: (a) If both letters are Vowel and number is less than 5, then vowels change to next letter in English alphabetical series and add 2 to the number (b) If both letters are consonant and number is greater than 5 or equal to, then consonants change to the previous letter in English alphabetical series and subtract 3 from the number (c) If both letters are Vowel and number is greater than 5 or equal to, then vowels change to the previous GovernmentAdda.com | IBPS SBI RBI SSC RRB FCI RAILWAYS

11

Daily Visit

[GOVERNMENTADDA.COM]

letter in English alphabetical series and subtract 3 from the number (d) If both letters are consonant and number is less than 5, then consonants change to next letter in English alphabetical series and add 3 to the number (e) If there are one vowel and one consonant, then vowel change to next letter and consonant change to the previous letter and add 2 to the number. (f) If there is single consonant, then consonant change to the previous letter and Subtract 3 from the number. (g) If there is a single vowel, then vowel change to next letter and add 3 to the number. Step-III: Follow Both Steps I and II

Example:

GovernmentAdda.com | IBPS SBI RBI SSC RRB FCI RAILWAYS

12

Daily Visit

[GOVERNMENTADDA.COM]

Input:

Explanation

GovernmentAdda.com | IBPS SBI RBI SSC RRB FCI RAILWAYS

13

Daily Visit

[GOVERNMENTADDA.COM]

6. In Step III, what is the sum of numbers in the first row? A. 5 B. 1 C. 2 D. 8 E. None of these Answer Answer – D. 8 7. In Step III, what is the difference between the sum of numbers in the second row and the sum of numbers in the third row? A. 5 B. 6 C. 1 D. 4 E. None of these Answer Answer – C. 1 8. In Step II, what is the product of the sum of numbers in the first column and the sum of numbers in the third column? A. 218 B. 288 C. 278 D. 256 E. 236 Answer Answer – B. 288

GovernmentAdda.com | IBPS SBI RBI SSC RRB FCI RAILWAYS

14

Daily Visit

[GOVERNMENTADDA.COM]

9. If the sum of the numbers in the second column of Step I is divided by sum of the numbers in the second column of Step II then what will be the resultant? A. 8 B. 7 C. 6 D. 3 E. 1 Answer Answer – E. 1 10. In Step I, which of the following letter/number occur more than twice? A. E B. K C. 7 D. 4 E. 8 Answer Answer – A. E Directions (1-5): A string of numbers is given as input. The further steps given are obtained by applying certain logic. Numbers of step II have been obtained by using at least 1 digit of each number in step I. Each step is a resultant of previous step only.

GovernmentAdda.com | IBPS SBI RBI SSC RRB FCI RAILWAYS

15

Daily Visit

[GOVERNMENTADDA.COM]

Input:

1. Which number is greatest in step 1? A) 53 B) 25 C) 63 D) 60 E) Other than options given View Answer Option C Solution: For given input::: Step I: 63…..…25……..21 — Subtraction Step II: 10……14 (6*2 – 2 = 10) Step III: 1……..5 –Addition Step IV: 3 — Average 2. What is the second smallest number obtained in any step of given input? A) 8.5 B) 7.5 C) 3 D) 10.5 E) 7 View Answer Option C 3. Find the difference between sum of numbers obtained in 1st step and sum of numbers obtained in all other steps. A) 61 B) 67 C) 89 D) 72 E) 76 View Answer Option E Solution: Required difference = (63+25+21) – (10+14+1+5+3) GovernmentAdda.com | IBPS SBI RBI SSC RRB FCI RAILWAYS

16

Daily Visit

[GOVERNMENTADDA.COM]

4. What is the difference between the second largest number and the smallest number obtained in any steps? A) 24 B) 18 C) 15 D) 29 E) 21 View Answer Option A Solution: Required difference = 25 – 1 = 24 5. What is the average of numbers obtained in last 2 steps? A) 4 B) 5 C) 9 D) 7 E) None of these View Answer Option E Solution: Required average =(1+3+5)/3 = 3 Directions (6-10): A string of numbers is given as input. The further steps given are obtained by applying certain logic. Numbers of step II have been obtained by using at least 1 digit of each number in step I. Each step is a resultant of previous step only.

GovernmentAdda.com | IBPS SBI RBI SSC RRB FCI RAILWAYS

17

Daily Visit

[GOVERNMENTADDA.COM]

Input:

6. What is the average of numbers obtained in last 2 steps? A) 8 B) 5 C) 3 D) 6 E) 4 View Answer Option E Solution: Step 1: 85……….76……..96 —- Multiplication Step 2: 24……….17 —– Addition (8+7+9 = 24, 5+6+6 = 17) Step 3: 2………..6 —— Subtraction Step 4: 4 —– Average Required Average = (2+6+4)/3 = 12/3 = 4 7. What is the smallest number obtained in any step of given input? A) 6 B) 4 C) 3 D) 2 E) 7 View Answer Option D 8. Find the difference between sum of numbers obtained in 1st step and sum of numbers obtained in all other steps. A) 204 B) 217 C) 189 D) 222 E) 176 View Answer Option A Solution: Required difference = (85+76+96) – (24+17+2+6+4)

GovernmentAdda.com | IBPS SBI RBI SSC RRB FCI RAILWAYS

18

Daily Visit

[GOVERNMENTADDA.COM]

9. What is the difference between the largest and the second smallest numbers obtained in any steps? A) 92 B) 108 C) 115 D) 69 E) 81 View Answer Option A Solution: Required difference = 96 – 4 = 92 10. Digit 7 repeats how much times in any numbers obtained in all steps? A) Three B) Five C) One D) Two E) None View Answer Option D Directions (1-5): A string of numbers is given as input. The further steps given are obtained by applying certain logic. Numbers of step II have been obtained by using at least 1 digit of each number in step I. Each step is a resultant of previous step only.

GovernmentAdda.com | IBPS SBI RBI SSC RRB FCI RAILWAYS

19

Daily Visit

[GOVERNMENTADDA.COM]

Input:

1. What is the average of numbers obtained in step 2? A) 53 B) 66 C) 75 D) 60 E) Other than options given View Answer Option D Solution: For given example: Step I : follow the arrows 9-6=3, 3-1 =2, 32 4-3=1, 5-1=4, 14… So step I: 32………14…..85 Step II: Numbers of step II have been obtained by using at least 1 digit of each number in step I. 3*1*5 = 15, 2*4*8 = 64 So step II: 15……..64 Step III: Average, (1+5)/2 = 3………..(6+4)/2 =5 Step IV: Add- 3+5 = 8 For given input::: Step I: 41…..…43……..86 Step II: 96……24 Step III: 7.5……..3 Step IV: 10.5 2. What is the second smallest number obtained in any step of given input? A) 8.5 B) 7.5 C) 3 D) 10.5 E) 7 View Answer Option B 3. Find the difference between sum of numbers obtained in 1st step and sum of numbers obtained in all other steps. A) 28 B) 25 C) 29 D) 26 E) 27 GovernmentAdda.com | IBPS SBI RBI SSC RRB FCI RAILWAYS

20

Daily Visit

[GOVERNMENTADDA.COM]

View Answer Option C Solution: Required difference = (41+43+86) – (96+24+7.5+3+10.5) 4. What is the difference between the second largest and the third smallest numbers obtained in any steps? A) 75.5 B) 74.5 C) 76.5 D) 71.5 E) 73.5 View Answer Option A Solution: Required difference = 86 – 10.5 5. What is the average of numbers obtained in last 2 steps? A) 4 B) 5 C) 9 D) 7 E) 6 View Answer Option D Solution: Required average =(7.5+3+10.5)/3 = 7 Directions (6-10): A string of numbers is given as input. The further steps given are obtained by applying certain logic. Numbers of step II have been obtained by using at least 1 digit of each number in step I. Each step is a resultant of previous step only.

GovernmentAdda.com | IBPS SBI RBI SSC RRB FCI RAILWAYS

21

Daily Visit

[GOVERNMENTADDA.COM]

Input:

6. What is the average of numbers obtained in step II and III? A) 28.5 B) 26 C) 25.5 D) 29 E) Other than options given View Answer Option A Solution: For given example: Step I : follow the arrows 2*4= 8, 1*9 = 9, 89 3*2=6, 3*1= 3, 63…. So step I: 89………63…..94 Step II: Numbers of step II have been obtained by using at least 1 digit of each number in step I. 8*6+9 = 57…….9*3+4 = 31 So step II: 57……..31 Step III: Average, (5+7)/2 = 6………..(3+1)/2 =2 Step IV: Subtract- 6-2 = 4 For given input::: Step I: 86…..…59……..66 Step II: 46……..60 Step III: 5……..3 Step IV: 2 7. What is the second largest number obtained in any step of given input? A) 86 B) 59 C) 66 D) 60 E) 67 View Answer Option C 8. Find the difference between sum of numbers obtained in step I and sum of numbers obtained in all other steps. A) 98 B) 85 C) 109 D) 86 E) 95 GovernmentAdda.com | IBPS SBI RBI SSC RRB FCI RAILWAYS

22

Daily Visit

[GOVERNMENTADDA.COM]

View Answer Option E Solution: Required difference = (86+59+66) – (46+60+5+3+2) 9. What is the difference between the third largest and the second smallest numbers obtained in any steps? A) 45.5 B) 64 C) 49 D) 57 E) 53.5 View Answer Option D Solution: Required difference = 60 – 3 10. How many numbers obtained in any steps are greater than 50? A) Six B) Four C) Three D) None E) Five View Answer Option B Solution: 86, 59, 66, and 60 Directions (1-5): A string of numbers is given as input. The further steps given are obtained by applying certain logic. Numbers of step II have been obtained by using at least 1 digit of each number in step I. Each step is a resultant of previous step only. None of the exact logic is repeated in any step.

GovernmentAdda.com | IBPS SBI RBI SSC RRB FCI RAILWAYS

23

Daily Visit

[GOVERNMENTADDA.COM]

Input:

1. What is the average of numbers obtained in step II and step IV? A) 33.4 B) 16.3 C) 22.3 D) 23.6 E) Other than options given View Answer Option C Solution: For given example: Step I: See arrows – multiplication is done for digits in blocks. 1st digit with 1st digit, 2nd with 2nd. 2*3 = 6, 3*3 = 9 so 69 in 1st place. 1*5 = 5, 7*1 = 7, so 57, 4*2 = 8, 2*3 = 6 Step II: Instructions say Numbers of step II have been obtained by using at least 1 digit of each number in step I. So 6*5 – 8 = 22, 9*7 – 6 = 57 Step III: (2+2)/2 = 2, (5+7)/2 = 6 Step IV: 6-2 = 4, it could have been (2+6)/2 = 4 but given that (None of the exact logic is repeated in any step.) For given input::: Step I: 4*1, 3*2 = 46…………8*1, 2*4 = 88……..1*6, 3*3 = 69 Step II: 4*8- 6= 26………………6*8-9 = 39 Step III: (2+6)/2 = 4………………(3+9)/2 = 6 Step IV: 6-4 = 2 So Step I: 46……88…..69 Step II: 26……39 Step III: 4…….6 Step IV: 2 2. What is the second smallest number obtained in any step of given input? A) 2 B) 4 C) 6 D) 5 E) 7 View Answer

GovernmentAdda.com | IBPS SBI RBI SSC RRB FCI RAILWAYS

24

Daily Visit

[GOVERNMENTADDA.COM]

Option B 3. Find the difference between sum of numbers obtained in 1st step and sum of numbers obtained in all other steps. A) 118 B) 102 C) 112 D) 173 E) 126 View Answer Option E Solution: Required difference = (46+88+69) – (26+39+4+6+2) 4. What is the difference between the second largest and the third smallest numbers obtained in any steps? A) 63 B) 40 C) 74 D) 61 E) 55 View Answer Option A Solution: Required difference = 69-6 = 63 5. What is the average of numbers obtained in last 2 steps? A) 6 B) 2 C) 5 D) 4 E) None of these View Answer Option D Solution: Required average =(4+6+2)/3 = 4 Directions (1-5): A string of alphabets is given as input. The further steps given are obtained by applying certain logic. Alphabets of step II have been obtained by using at least 1 digit of each number in step I. Each step is a resultant of previous step only. None of the exact logic is repeated in any step.

GovernmentAdda.com | IBPS SBI RBI SSC RRB FCI RAILWAYS

25

Daily Visit

[GOVERNMENTADDA.COM]

Input:

1. Which alphabet occurs exactly 3 times in any steps? A) L B) F C) R D) U E) Other than options given View Answer Option B Solution: For given example: Step I: See arrows, takes first alphabet with 1st and 2nd with 2nd. C = 3, H = 8, 3*8 = 24=X, S = 19, A = 1, 19*1 = 19 = S so XS Next B= 2, K = 11, 2*11 = 22 = V, ….and so on VZ and NT Step II: X = 24, V = 22, N = 14 — 24+22 – 14 = 32, 3 = C, 2=B S= 19, Z = 26, T = 20 — 19+26-20 = 25, 2 = B, 5 = E Step III: CB = 3+2 = 5 = E GovernmentAdda.com | IBPS SBI RBI SSC RRB FCI RAILWAYS

26

Daily Visit

[GOVERNMENTADDA.COM]

BE = 2+5 = 7 = G Step IV: G= 7,E = 5 — 7-5 = 2 = B For given input::: Step I: See arrows, takes first alphabet with 1st and 2nd with 2nd. D = 4, C = 3, 4*3 = 12=L, R = 18, A = 1, 18*1 = 18 = R so LR Next E= 5, D = 4, 5*4 = 20 = T, ….and so on TU and PM Step II: L = 12, T = 20, P = 16 — 12+20 – 16 = 16, 1 = A, 6=F R= 18, U = 21, M = 24 — 18+21-13 = 26, 2 = B, 6 = F Step III: AF = 1+6 = 7 = G BF = 2+6 = 8 = H Step IV: H= 8, G = 7 — 8-7 = 1 = A So Step I: LR….…TU……..PM Step II: AF……BF Step III: G…….H Step IV: A 2. What is the sum of numbers corresponding to each alphabet in step II? (Taking A = 1, B = 2, …..Z = 26) A) 18 B) 19 C) 15 D) 13 E) 11 View Answer Option C Solution: Step II: A+F+B+F = 1+6+2+6 = 15 3. Let @ is the sum of numbers corresponding to each alphabet in step III. What is the alphabet corresponding to @? (Taking A = 1, B = 2, …..Z = 26) A) L B) M C) P D) O E) Q View Answer Option D Solution: Step III: G+H = 7+8 = 15 = O 4. How many alphabets appear more than once in any step? A) 3 B) 2 C) 4 GovernmentAdda.com | IBPS SBI RBI SSC RRB FCI RAILWAYS

27

Daily Visit

[GOVERNMENTADDA.COM]

D) 1 E) 5 View Answer Option B Solution: A and F 5. In step I, what is the number corresponding to the alphabet which is also present in ‘PACT’? A) 16 B) 3 C) 1 D) 20 E) None of P, A, C, T is present View Answer Option D Solution: Out of P, A, C, T, — T is present in step I, T = 20 Directions (1-5): A string of numbers is given as input. The further steps given are obtained by applying certain logic. Numbers of step II have been obtained by using at least 1 digit of each number in step I. Each step is a resultant of previous step only.

Input: GovernmentAdda.com | IBPS SBI RBI SSC RRB FCI RAILWAYS

28

Daily Visit

[GOVERNMENTADDA.COM]

1. What is the average of numbers obtained in step 2? A) 23 B) 26 C) 25 D) 35 E) Other than options given View Answer Option A Solution: For given example:

For given input::: Step I: 89……78…..86 Step II: 23……23 Step III: 1.5……..1.5 Step IV: 0 2. What is the second largest number obtained in any step of given input? A) 89 B) 86 C) 78 D) 23 E) 45 View Answer Option B GovernmentAdda.com | IBPS SBI RBI SSC RRB FCI RAILWAYS

29

Daily Visit

[GOVERNMENTADDA.COM]

3. Find the difference between sum of numbers obtained in 1st step and sum of numbers obtained in all other steps. A) 211 B) 221 C) 225 D) 204 E) 215 View Answer Option D Solution: Required difference = (89+78+86) – (23+23+1.5+1.5+0) 4. What is the difference between the largest and the third smallest numbers obtained in any steps? A) 84 B) 70 C) 66 D) 61 E) 58 View Answer Option C Solution: Required difference = 89 – 23 5. What is the average of numbers obtained in last 2 steps? A) Other than given in options B) 0.5 C) 2 D) 1.5 E) 1 View Answer Option E Solution: Required average =(1.5+1.5+0)/3 = 1 Directions (6-10): Consider the following steps for given input and read the instructions to reach to the last step.

GovernmentAdda.com | IBPS SBI RBI SSC RRB FCI RAILWAYS

30

Daily Visit

[GOVERNMENTADDA.COM]

Instructions: Step-I: Interchange the alphabets in input as arrows mentioned Step-II: (i) If both letters are consonant and number is less than 6, then consonants change to previous letter in English alphabetical series. (ii) If there is one vowel and one consonant, then add 4 to the number. (iii) It there is single consonant, then consonant changes to next letter in English alphabetical series. Step-III: has been derived using a special pattern taking similar (but not exactly) patterns of both step I and step II.

6. What is the sum of all numbers in step II of given input? A) 42 B) 40 GovernmentAdda.com | IBPS SBI RBI SSC RRB FCI RAILWAYS

31

Daily Visit

[GOVERNMENTADDA.COM]

C) 33 D) 36 E) 43 View Answer Option A Solution: In step III, we see that same number of alphabets are at same position, A changes to B, UM to TM, T to U, so if we take same pattern of step I, i.e. keeping letters constant and exchanging, we get the second figure as in below::

Now try some application in this second figure as were given for step-II: We see when there is single letter, letter change to next letter. When both are consonants, then number is (number-2) e.g QF6 –> QF4 And when one is vowel and other consonant, then vowel changes to previous letter and number to (number+1). like in BI8 B is B, I -> H and 8->(8+1)9 For given input:

GovernmentAdda.com | IBPS SBI RBI SSC RRB FCI RAILWAYS

32

Daily Visit

[GOVERNMENTADDA.COM]

7. In last step, which of the following letter/s occur more than 1 time? A) C B) P C) N D) Both T and F E) Both N and T View Answer Option E 8. Which of the following represents the second element in 3rd row in step III? A) L4 B) M4 C) L6 D) N4 E) M3 View Answer Option B 9. What is the addition of numbers which are with L and LR in step II? A) 7 B) 5 C) 6 D) 3 E) 4 View Answer Option A 10. What is the addition of numbers which are with N, BT and TC in step III? A) 17 B) 13 C) 16 D) 15 E) 19 View Answer Option D Directions (1-5): A string of numbers is given as input. The further steps given are obtained by applying certain logic. Numbers of step II have been obtained by using at least 1 digit of each number in step I. Each step is a resultant of previous step only.

GovernmentAdda.com | IBPS SBI RBI SSC RRB FCI RAILWAYS

33

Daily Visit

[GOVERNMENTADDA.COM]

Input:

1. What is the average of numbers obtained in step 2? A) 52 B) 56 C) 45 D) 65 E) Other than options given View Answer Option C Solution: For given example:

For given input::: Step I: 78……86…..86 Step II: 48……42 GovernmentAdda.com | IBPS SBI RBI SSC RRB FCI RAILWAYS

34

Daily Visit Step III: Step IV:

[GOVERNMENTADDA.COM]

2……..0.5 2.5

2. What is the smallest number obtained in any step of given input? A) 1 B) 0.5 C) 2 D) 3 E) 1.5 View Answer Option B 3. Find the difference between sum of numbers obtained in 1st step and sum of numbers obtained in all other steps. A) 175 B) 195 C) 125 D) 115 E) 155 View Answer Option E Solution: Required difference = (78+86+86) – (48+42+2+0.5+2.5) = 250 – 95 = 155 4. What is the difference between the largest and the second smallest numbers obtained in any steps? A) 84 B) 70 C) 60 D) 61 E) 58 View Answer Option A Solution: Required difference = 86 – 2 5. What is the average of numbers obtained in steps II and III? A) 26.135 B) 21.145 C) 23.125 D) 24.175 E) 27.155 View Answer Option C Solution: Required average =(48+42+2+0.5)/4 = 23.125 GovernmentAdda.com | IBPS SBI RBI SSC RRB FCI RAILWAYS

35

Daily Visit

[GOVERNMENTADDA.COM]

Directions (6-10): A string of numbers is given as input. The further steps given are obtained by applying certain logic. Numbers of step II have been obtained by using at least 1 digit of each number in step I. Each step is a resultant of previous step only.

Input:

6. What is the average of numbers obtained in step II and step IV? A) 327 B) 462 C) 535 D) 342 E) 442 View Answer Option D Solution: For given example:

GovernmentAdda.com | IBPS SBI RBI SSC RRB FCI RAILWAYS

36

Daily Visit

[GOVERNMENTADDA.COM]

For given input::: Step I: 79…..…99……..68 Step II: 378…….648 Step III: 80……..80 Step IV: 0 Required average = (378+648+0)/3 = 342 7. What is the ratio of numbers obtained in Step III respectively? A) 3 : 7 B) 1 : 3 C) 4 : 1 D) 4 : 5 E) 1 : 1 View Answer Option E Solution: 80/80 = 1/1 8. Find the difference between sum of numbers obtained in step II and sum of numbers obtained in all other steps. A) 620 B) 770 C) 650 D) 710 E) 690 View Answer Option A Solution: Required difference = (378+648) – (79+99+68+80+80+0) GovernmentAdda.com | IBPS SBI RBI SSC RRB FCI RAILWAYS

37

Daily Visit

[GOVERNMENTADDA.COM]

9. What is the difference between the largest and the smallest numbers obtained in step I? A) 47 B) 40 C) 30 D) 31 E) 28 View Answer Option D Solution: Required difference = 99 – 68 10. If number in step IV of given example was 34 instead of 6, then what would be the number obtained in step IV of asked input? A) 130 B) 160 C) 120 D) 160 E) Cannot be determined View Answer Option B Solution: 20-14 = 6, but instead 20+14 = 34 So for given input 80+80 = 160 Directions (1-5): A string of numbers is given as input. The further steps given are obtained by applying certain logic. Numbers of step II have been obtained by using at least 1 digit of each number in step I. Each step is a resultant of previous step only.

GovernmentAdda.com | IBPS SBI RBI SSC RRB FCI RAILWAYS

38

Daily Visit

[GOVERNMENTADDA.COM]

Input:

Directions (1-5): A string of numbers is given as input. The further steps given are obtained by applying certain logic. Numbers of step II have been obtained by using at least 1 digit of each number in step I. Each step is a resultant of previous step only. Input: 1. What is the average of numbers obtained in step II? A) 88 B) 66 C) 75 D) 51 E) Other than options given View Answer Option D Solution: For given example: Step I : follow the arrows 8/2= 4, 9/3= 3, 43 6/1=6, 8/2 = 4, 64…. So step I: 43………64…..37 Step II: Numbers of step II have been obtained by using at least 1 digit of each number in step I. 4*6*3 = 72…….3*4*7 = 84 So step II: 72……..84 Step III: Subtract, 5………..4 Step IV: Add- 5+4 = 9 For given input::: Step I: 47…..…32……..53 Step II: 60……..42 Step III: 6……..2 Step IV: 8 2. If 5 is subtracted from the number obtained in last step, what is the resultant? A) 6 B) 3 C) 5 D) -2 E) 0 View Answer Option B 3. What is the total sum of the digits of all numbers in step I? A) 18 GovernmentAdda.com | IBPS SBI RBI SSC RRB FCI RAILWAYS

39

Daily Visit

[GOVERNMENTADDA.COM]

B) 45 C) 24 D) 26 E) 35 View Answer Option C Solution: Required sum = 4+7+3+2+5+3 4. Which of the following is one of the factors of the difference obtained by subtracting the smallest number from the second largest number in any steps? A) 4 B) 17 C) 9 D) 11 E) 18 View Answer Option B Solution: Required difference = 53 – 2 = 51 51 = 1*3*17 5. The digit ‘4’ is seen how many times in any of the steps? A) None B) One C) Three D) Five E) Two View Answer Option E Solution: In 47 and 42 Directions (6 – 10): Answer the questions on the basis of the information given below. A number arrangement machine when given an input of words/numbers, rearranges them following a particular rule in each step. The following is an illustration of input and steps of rearrangement. Input: 12 exotic large 56 37 online 19 unique cross 61 paint 42 Step 1: exotic large 56 37 online 19 unique cross 61 paint 42 12 Step 2: 19 exotic large 56 37 online unique 61 paint 42 12 cross Step 3: online 19 exotic large 56 37 unique 61 paint 12 cross 42 Step 4: 37 online 19 exotic 56 unique 61 paint 12 cross 42 large Step 5: unique 37 online 19 exotic 61 paint 12 cross 42 large 56 Step 6: 61 unique 37 online 19 exotic 12 cross 42 large 56 paint This is the final arrangement and step 6 is the last step for this input. GovernmentAdda.com | IBPS SBI RBI SSC RRB FCI RAILWAYS

40

Daily Visit

[GOVERNMENTADDA.COM]

Input: admin 47 51 upscale daily safe 13 36 ideal 18 mail 62 6. What is the position of ‘upscale’ in step 4 from right end? A) 3rd B) 4th C) 7th D) 6th E) 8th View Answer Option C Solution: Words are taken in increasing order of first alphabets. Numbers are taken in ascending order. First -2, then +2, -2, +2 and -2 Input: admin 47 51 upscale daily safe 13 36 ideal 18 mail 62 Step 1: admin 47 51 upscale daily safe 13 36 ideal mail 62 18 Step 2: 13 admin 47 51 upscale safe 36 ideal mail 62 18 daily Step 3: ideal 13 admin 47 51 upscale safe mail 62 18 daily 36 Step 4: 47 ideal 13 admin 51 upscale safe 62 18 daily 36 mail Step 5: upscale 47 ideal 13 admin 51 safe 18 daily 36 mail 62 Step 6: 51 upscale 47 ideal 13 admin 18 daily 36 mail 62 safe 7. In step 6, if ‘51’ is related to ideal’ and ‘47’ is related to ‘admin’ in a certain way, then ‘daily’ is related to which of the following? A) safe B) 62 C) 13 D) mail E) None of these View Answer Option B 8. How many words/numbers are there between ‘ideal’ and ’18’ in step 5? A) None B) One C) Two D) Three E) More than three View Answer Option E 9. Which of the following is second to right of sixth word/number from right end in step 3? A) admin B) safe C) mail D) 62 E) 51 GovernmentAdda.com | IBPS SBI RBI SSC RRB FCI RAILWAYS

41

Daily Visit

[GOVERNMENTADDA.COM]

View Answer Option D Only words are asked – so loan and part 10. Which of the following is the last but one step of given input? A) 51 upscale 47 ideal 13 admin 18 daily 36 mail 62 safe B) upscale 47 ideal 13 admin 51 safe 18 daily 36 mail 62 C) upscale 13 ideal 47 admin 51 safe 18 daily 36 mail 62 D) 47 upscale ideal 13 admin 51 safe 18 daily 36 mail 62 E) None of these View Answer Option B Directions (1-5): A string of numbers is given as input. The further steps given are obtained by applying certain logic. Numbers of step II have been obtained by using at least 1 digit of each number in step I. Each step is a resultant of previous step only.

Input:

1. What is the average of numbers obtained in last two steps? A) 1.5 B) 2 C) 1 D) 2.5

GovernmentAdda.com | IBPS SBI RBI SSC RRB FCI RAILWAYS

42

Daily Visit

[GOVERNMENTADDA.COM]

E) Other than options given View Answer Option C Solution: For given example: Step I : follow the arrows 2+3= 5, 8+1= 9, 59 4+4=8, 3+2= 5, 85…. So step I: 59………85…..76 Step II: Numbers of step II have been obtained by using at least 1 digit of each number in step I. (5+8)*7 = 91…….(9+5)*6= 84 So step II: 91……..84 Step III: Subtract, 8………..4 Step IV: Average- (8+4)/2 = 6 For given input::: Step I: 57…..…69……..84 Step II: 88……..64 Step III: 0……..2 Step IV: 1 2. If 7 is added to the number obtained in last step, what is the resultant? A) 10 B) 13 C) 15 D) 8 E) 9 View Answer Option D 3. What is the difference between the largest and the smallest number in step I and II respectively? A) 18 B) 45 C) 20 D) 26 E) 35 View Answer Option C Solution: Required difference = 84 – 64 = 20 4. Find the HCF of numbers obtained in all steps. A) 4 B) 5 GovernmentAdda.com | IBPS SBI RBI SSC RRB FCI RAILWAYS

43

Daily Visit

[GOVERNMENTADDA.COM]

C) 0 D) 3 E) 1 View Answer Option E Solution: HCF is highest common factor among all numbers. All numbers have highest common factor as 1. 0 = 1*0 5. The digit ‘8’ is seen how many times in any of the steps? A) None B) One C) Three D) Five E) Two View Answer Option C Solution: In 84 and 88 Directions (6 – 10): Answer the questions on the basis of the information given below. A number arrangement machine when given an input of numbers/words, rearranges them following a particular rule in each step. The following is an illustration of input and steps of rearrangement. Input: 44 First Engine 22 17 Product Mania 25 Illicit Outer 60 41 Step 1: Engine 44 First 22 17 Product Mania 25 Illicit Outer 41 60 Step 2: Engine 17 44 First 22 Mania 25 Illicit Outer 41 Product 60 Step 3: Engine 17 Illicit First 22 Mania 25 Outer 41 44 Product 60 Step 4: Engine 17 Illicit 25 First 22 Outer 41 Mania 44 Product 60 Step 5: Engine 17 Illicit 25 Outer First 41 22 Mania 44 Product 60 Step 6: Engine 17 Illicit 25 Outer 41 First 22 Mania 44 Product 60 Step 7: Engine 64 Illicit 49 Outer 25 First 16 Mania 64 Product 36 This is the final arrangement and step 7 is the last step for this input. Input: 18 cotton interest 42 43 access 27 unique replace 65 58 lamp 6. Which word/number is third to right of ninth element from right end in step 4? A) cotton B) interest C) 65 D) 43 E) None of these View Answer GovernmentAdda.com | IBPS SBI RBI SSC RRB FCI RAILWAYS

44

Daily Visit

[GOVERNMENTADDA.COM]

Option E In last step::: starting with Vowel, smallest odd number, then vowel, then next odd number…..then word with consonant, next smallest even number In the same way, steps are arranged. Numbers in Step 7: Square of sum of digits of the respective number. Input: 18 cotton interest 42 43 access 27 unique replace 65 58 lamp Step 1: access 18 cotton interest 42 43 27 unique replace 65 lamp 58 Step 2: access 27 18 cotton interest 42 43 unique 65 lamp replace 58 Step 3: access 27 interest 18 cotton 43 unique 65 lamp 42 replace 58 Step 4: access 27 interest 43 18 cotton unique 65 lamp 42 replace 58 Step 5: access 27 interest 43 unique cotton 65 18 lamp 42 replace 58 Step 6: access 27 interest 43 unique 65 cotton 18 lamp 42 replace 58 Step 7: access 81 interest 49 unique 121 cotton 81 lamp 36 replace 169 7. In step 5, if ‘27’ interchanges position with ‘cotton’ and ‘unique’ with ‘42’, then which word is exactly cotton ‘27’ and ‘unique’? A) 18 B) 65 C) interest D) lamp E) None of these View Answer Option A 8. How many words are there in cotton words ‘18’ and ‘65’ in step 3? A) Four B) Three C) Two D) One E) None of these View Answer Option B 9. Which step number is following step? Step: access 27 interest 43 unique cotton 18 65 lamp 42 replace 58 A) 5 B) 2 C) 7 D) 1 E) There is no such step View Answer GovernmentAdda.com | IBPS SBI RBI SSC RRB FCI RAILWAYS

45

Daily Visit

[GOVERNMENTADDA.COM]

Option E 10. Find the difference in numbers which is 5th from right end in step 3 and 4th from left end in step 7. A) 9 B) 31 C) 16 D) 28 E) 22 View Answer Option D 65 – 49 = 16 Directions (1-5): A string of numbers is given as input. The further steps given are obtained by applying certain logic. Numbers of step II have been obtained by using at least 1 digit of each number in step I. Each step is a resultant of previous step only.

Input:

GovernmentAdda.com | IBPS SBI RBI SSC RRB FCI RAILWAYS

46

Daily Visit

[GOVERNMENTADDA.COM]

1. What is the average of numbers obtained in step II and III? A) 23 B) 26 C) 26.5 D) 28.5 E) Other than options given View Answer Option C Solution: For given input Step I: 9-6 = 3, 5-3 = 2, 32 7-2 = 5, 6-2 = 4, 54 3-2 = 1, 8-5 = 3, 13 So 32……….54………..13 Step II: Numbers of step II have been obtained by using at least 1 digit of each number in step I. 3*5*1 = 15 2*4*3 = 24 Step III: 5/1 = 5…….4/2 = 2 Step IV: 5+2 = 7 Answer steps::: Step I: 35…….24……..34 Step II: 18……..80 Step III: 8……..0 Step IV: 8 2. What is the second largest obtained in any step of given input? A) 18 B) 44 C) 34 D) 24 E) 35 View Answer Option E 3. Find the difference between sum of numbers obtained in step II and sum of numbers obtained in all other steps. A) -11 B) 15 C) 12 D) -13 E) 11 View Answer

GovernmentAdda.com | IBPS SBI RBI SSC RRB FCI RAILWAYS

47

Daily Visit

[GOVERNMENTADDA.COM]

Option A Solution: Required difference = (18+80) – (35+24+34+8+0+8) 4. What is the difference between the largest and the fourth smallest numbers obtained in any steps? A) 47 B) 50 C) 69 D) 56 E) 58 View Answer Option D Solution: Required difference = 80 – 24 5. What is obtained in last step? A) 5 B) 7 C) 2 D) 4 E) 8 View Answer Option E Directions (6 – 10): Answer the questions on the basis of the information given below. A number arrangement machine when given an input of words/numbers, rearranges them following a particular rule in each step. The following is an illustration of input and steps of rearrangement. Input: 29 google 55 microsoft amazon 46 nokia 13 samsung 34 Step 1: amazon 29 google 55 microsoft 46 nokia 13 samsung 34 Step 2: amazon 29 google 55 microsoft 46 nokia samsung 34 11 Step 3: amazon google 29 55 microsoft 46 nokia samsung 34 11 Step 4: amazon google 55 microsoft 46 nokia samsung 34 11 31 Step 5: amazon google microsoft 55 46 nokia samsung 34 11 31 Step 6: amazon google microsoft 55 46 nokia samsung 11 31 32 Step 7: amazon google microsoft nokia 55 46 samsung 11 31 32 Step 8: amazon google microsoft nokia 55 samsung 11 31 32 48 Step 9: amazon google microsoft nokia samsung 55 11 31 32 48 Step 10: amazon google microsoft nokia samsung 11 31 32 48 53 This is the final arrangement and step 10 is the last step for this input. Input: loan 55 part copy 18 43 gain 48 allow 22 6. What is the position of ‘part’ in step 4 from right end? A) 3rd B) 4th C) 5th

GovernmentAdda.com | IBPS SBI RBI SSC RRB FCI RAILWAYS

48

Daily Visit

[GOVERNMENTADDA.COM]

D) 6th E) 8th View Answer Option D Solution: Words are taken in increasing order of first alphabets. Numbers are taken in ascending order. First -2, then +2, -2, +2 and -2 Input: loan 55 part copy 18 43 gain 48 allow 22 Step 1: allow loan 55 part copy 18 43 gain 48 22 Step 2: allow loan 55 part copy 43 gain 48 22 16 Step 3: allow copy loan 55 part 43 gain 48 22 16 Step 4: allow copy loan 55 part 43 gain 48 16 24 Step 5: allow copy gain loan 55 part 43 48 16 24 Step 6: allow copy gain loan 55 part 48 16 24 41 Step 7: allow copy gain loan 55 part 16 24 41 50 Step 8: allow copy gain loan part 55 16 24 41 50 Step 9: allow copy gain loan part 16 24 41 50 53 7. In step 6, if ‘copy’ is related to ‘loan’ and ‘55’ is related to ‘48’ in a certain way, then ‘part’ is related to which of the following? A) loan B) 55 C) 16 D) 48 E) None of these View Answer Option C 8. How many steps are done to complete the output? A) 6 B) 7 C) 8 D) 9 E) 10 View Answer Option D 9. How many words are between ‘gain’ and ‘16’ in step 7? A) 3 B) 4 C) 2 D) 5 E) There is no such step View Answer

GovernmentAdda.com | IBPS SBI RBI SSC RRB FCI RAILWAYS

49

Daily Visit

[GOVERNMENTADDA.COM]

Option C Only words are asked – so loan and part 10. Which of the following is the last but one step of given input? A) allow copy gain loan part 16 24 41 50 53 B) allow copy gain loan 55 part 16 24 41 50 C) allow copy gain loan part 55 16 24 41 50 D) allow copy gain loan part 55 18 24 41 50 E) None of these View Answer Option C Directions (1 – 5): Answer the questions on the basis of the information given below. A number arrangement machine when given an input of words/numbers, rearranges them following a particular rule in each step. The following is an illustration of input and steps of rearrangement. Input: fog 13 angle post 75 26 lamp 31 earn 58 outer 79 Step 1: 131 fog angle post 75 lamp 31 iearn 58 outer 79 262 Step 2: angle 131 post 75 lamp 31 iearn 58 outer 79 262 fog Step 3: 313 angle 131 post 75 lamp earn outer 79 262 fog 585 Step 4: earn 313 angle 131 post 75 outer 79 262 fog 585 lamp Step 5: 757 earn 313 angle 131 post outer 262 fog 585 lamp 797 Step 5: outer 757 earn 313 angle 131 262 fog 585 lamp 797 post This is the final arrangement and step 6 is the last step for this input. Input: 28 hut under 75 out 45 12 break 21 pot 63 east 1. What is the position of ‘212’ in step 5 from right end? A) 7th B) 9th C) 5th D) 4th E) 8th View Answer Option C Explanation: Words: consonants and vowels taken in alternate steps. Vowel at first place, consonant at last. Numbers taken in ascending order – and added the first digit at the end while arrangement. like 13 > 131, 26 -> 262, 31 ->313, 58 -> 585….. Now:: Input: 28 hut under 75 out 45 12 break 21 pot 63 east Step 1: 121 28 hut under 75 out 45 break pot 63 east 212 Step 2: east 121 28 hut under 75 out 45 pot 63 212 break Step 3: 282 east 121 hut under 75 out pot 63 212 break 454 Step 4: out 282 east 121 under 75 pot 63 212 break 454 hut Step 5: 636 out 282 east 121 under pot 212 break 454 hut 757 GovernmentAdda.com | IBPS SBI RBI SSC RRB FCI RAILWAYS

50

Daily Visit

[GOVERNMENTADDA.COM]

Step 6: under 636 out 282 east 121 212 break 454 hut 757 pot 2. In step 6, if ‘out’ is related to ‘212’ in the same way as in step 2, ‘75’ is related to ‘63’ in a certain way, then in step 3, ‘under’ is related to which of the following in the same way? A) pot B) 212 C) 121 D) break E) 63 View Answer Option E 3. What is the sum of numbers – 6th from the left end in step 3 and 4th from the right end in step 4? A) 258 B) 287 C) 232 D) 265 E) 239 View Answer Option B 75 + 212 = 287 4. In which of the following step number do the words/numbers ‘121 212 break 454’ occur together? A) 2 B) 4 C) 3 D) 6 E) 5 View Answer Option D 5. How many words/numbers are there between ‘121’ and ’63’ in step 4? A) 2 B) 5 C) 3 D) 4 E) None of these View Answer GovernmentAdda.com | IBPS SBI RBI SSC RRB FCI RAILWAYS

51

Daily Visit

[GOVERNMENTADDA.COM]

Option C Directions (6-10): Consider the following steps for given input and read the instructions to reach to the last step.

Instructions: Step-I: Interchange the alphabets in input as arrows mentioned Step-II: (i) If both letters are consonant and number is less than 5, then 1st consonant changes to next letter and 2nd consonant changes to previous letter in English alphabetical series. (ii) If there is one vowel and one consonant, then consonant changes to next letter in English alphabetical series. (iii) It there is single consonant, then add 2 to the number. (iv) It there is single vowel, then vowel changes to previous letter in English alphabetical series. Step-III: has been derived using a special pattern.

Input:

GovernmentAdda.com | IBPS SBI RBI SSC RRB FCI RAILWAYS

52

Daily Visit

[GOVERNMENTADDA.COM]

6. What is the sum of all numbers in step II of given input? A) 42 B) 40 C) 33 D) 36 E) 38 View Answer Option E Solution: Try to derive rules for step III, same as given for step II We see, when there are both consonants like ST8, it changes to TS7, means when both consonants are there, (1st consonant + 1), (2nd consonant – 1) , and no changes (no – 1) Next, when there is single consonant like Z8, it changes to X8, so (consonant – 2) Next, when there is one vowel and one consonant, like NE1 -> NF4, given (vowel + 1) and (no +3) So rules for step III are: (i) If both letters are consonant, then 1st consonant changes to next letter, 2nd consonant changes to previous letter in English alphabetical series and subtract 2 from number. (ii) If there is one vowel and one consonant, then vowel changes to next letter and add 3 to the number. (iii) It there is single consonant, then consonant changes to next to next letter in English alphabetical series. (iv) If there are two vowels, then 2nd vowel changes to next letter and add 3 to the number. For given input:

7. In last step, which of the following letter/s occur more than 2 times? A) D B) A GovernmentAdda.com | IBPS SBI RBI SSC RRB FCI RAILWAYS

53

Daily Visit

[GOVERNMENTADDA.COM]

C) J D) Both J and D E) None View Answer Option C 8. Which of the following represents the second element in 3rd row in step III? A) T2 B) R2 C) DJ4 D) S2 E) CK5 View Answer Option B 9. In step III, how many letters are there which represent number less than 13 according to English alphabetical series? (A = 1, B = 2, ….., Z = 26) A) 3 B) 5 C) 6 D) 1 E) 4 View Answer Option B D, J, A, B and I 10. From step II to III, numbers in how many boxes change? A) 3 B) 5 C) 2 D) 1 E) 4 View Answer Option A CK5 -> DJ4. UA1 -> UB4 and NI3 -> NJ6 Directions (1-5): A string of numbers is given as input. The further steps given are obtained by applying certain logic. Numbers of step II have been obtained by using at least 1 digit of each number in step I. Each step is a resultant of previous step only. GovernmentAdda.com | IBPS SBI RBI SSC RRB FCI RAILWAYS

54

Daily Visit

[GOVERNMENTADDA.COM]

Input:

1. Which number is greatest in step 1? A) 53 B) 67 C) 88 D) 60 E) Other than options given View Answer Option C Solution: Step I: Multiply consecutive digits according to arrows.. 2*4 = 8, 4*1 = 4 so 84……79….65 Step II: (8+7) * 6 = 90…….(4+9) * 5 = 65 Step III: 9-0=9…..6-5=1 Step IV: Average — (9+1)/2 = 5 So, now Step I: 88…..…67……..65 Step II: 84……75 Step III: 4……..2 Step IV: 3 2. What is the second smallest number obtained in any step of given input? A) 2 B) 2.5 C) 4 GovernmentAdda.com | IBPS SBI RBI SSC RRB FCI RAILWAYS

55

Daily Visit

[GOVERNMENTADDA.COM]

D) 3 E) 3.5 View Answer Option D 3. Find the difference between sum of numbers obtained in 1st step and sum of numbers obtained in all other steps. A) 52 B) 67 C) 89 D) 72 E) 76 View Answer Option A Solution: Required difference = (88+67+65) – (84+75+4+2+3) 4. What is the difference between the second largest number and the smallest number obtained in any steps? A) 64 B) 108 C) 95 D) 82 E) 71 View Answer Option D Solution: Required difference = 84 – 2 = 82 5. What is the average of numbers obtained in last 2 steps? A) 4 B) 5 C) 3 D) 7 E) None of these View Answer Option C Solution: Required average =(4+2+3)/3 = 3 GovernmentAdda.com | IBPS SBI RBI SSC RRB FCI RAILWAYS

56

Daily Visit

[GOVERNMENTADDA.COM]

Directions (6 – 10): Answer the questions on the basis of the information given below. A number arrangement machine when given an input of words/numbers, rearranges them following a particular rule in each step. The following is an illustration of input and steps of rearrangement. Input: ancient 16 draft upper 52 39 earn portal 63 32 hence 21 Step 1: upper ancient draft 52 39 earn portal 63 32 hence 21 18 Step 2: 22 upper ancient draft 52 39 earn 63 32 hence 18 portal Step 3: earn 22 upper ancient draft 52 39 63 hence 18 portal 34 Step 4: 40 earn 22 upper ancient draft 52 63 18 portal 34 hence Step 5: ancient 40 earn 22 upper draft 63 18 portal 34 hence 54 Step 6: 64 ancient 40 earn 22 upper 18 portal 34 hence 54 draft This is the final arrangement and step 6 is the last step for this input. Input: 68 occupy 23 55 factor early 22 kind 34 ideal 17 safe 6. How many words/numbers are between ‘ideal’ and ‘factor’ in step 5? A) 3 B) 5 C) 2 D) 6 E) 4 View Answer Option A Explanation: In the given example: Words: 3 start with vowel, 3 with consonants. Step 1: vowel at first, step 2 : consonant at last and so on. Numbers: 3 odd numbers, 3 even numbers. Step 1: smallest even (+2) at last. Step 2: smallest odd (+1) at first and so on. Now:: Input: 68 occupy 23 55 factor early 22 kind 34 ideal 17 safe Step 1: occupy 68 23 55 factor early kind 34 ideal 17 safe 24 Step 2: 18 occupy 68 23 55 factor early kind 34 ideal 24 safe Step 3: ideal 18 occupy 68 23 55 factor early kind 24 safe 36 Step 4: 24 ideal 18 occupy 68 55 factor early 24 safe 36 kind Step 5: early 24 ideal 18 occupy 55 factor 24 safe 36 kind 70 Step 6: 56 early 24 ideal 18 occupy 24 safe 36 kind 70 factor 7. Which word/number is second to left of fifth element from the right end in step 4? A) safe B) factor C) 68 D) 55 E) early View Answer Option D second to left of fifth from right = 7th from right GovernmentAdda.com | IBPS SBI RBI SSC RRB FCI RAILWAYS

57

Daily Visit

[GOVERNMENTADDA.COM]

8. In step 6, ‘early’ is related to ‘18’ in the same way as in step 3, ‘68’ is related to ‘factor’. Now in step 5, ‘factor’ is related to what in the same way? A) kind B) safe C) 24 D) occupy E) None of these View Answer Option E 9. In which of the following step numbers do words ‘68 55 factor 24’ occur together? A) 9 B) 4 C) 6 D) 5 E) There is no such step View Answer Option E 10. What is the sum of numbers which is 4th from left end in step 3 and which is 5th from right end in step 5? A) 71 B) 84 C) 67 D) 92 E) None of these View Answer Option D 68 + 24 = 92

GovernmentAdda.com | IBPS SBI RBI SSC RRB FCI RAILWAYS

58

Daily Visit

[GOVERNMENTADDA.COM]

200+ New Pattern Puzzle PDF Governmentadda.com

GovernmentAdda.com | IBPS SBI SSC RBI RRB FCI RAILWAYS

1

Daily Visit

[GOVERNMENTADDA.COM]

Directions (1 – 3): Answer the questions on the basis of the information given below. 8 boxes – A, B, C, D, E, F, G and H are placed one above the another but not necessarily in the same order. Three boxes are placed between D and B. Two boxes are placed between E and B. Two boxes are placed between A and H. H is placed immediately below B. Two boxes are placed between C and G. Two boxes are placed between A and F. 1. How many boxes are placed between D and C? A) Two B) None C) Three D) Five E) One Option A Arrangement: Three boxes are placed between D and B. Two boxes are placed between E and B. H is placed immediately below B. 4 possibilities: – – – E – – – – – – – – D D B B E – H H – – – – – – E – B B D D H H – – – E Two boxes are placed between A and H. Two boxes are placed between A and F. – – – E – – A A F – – – D D B B E – H H A A – –

– – E – B B D D H H – – – E Only 1st possibility is possible. And now Two boxes are placed between C and G. So from top to bottom – F > D > E > A > C/G > B > H > C/G 2. If C is placed above G, which box is at bottom most position? A) B B) C C) H D) G E) Cannot be determined Option D 3. Which box is placed just above box H? A) A B) D C) G D) B E) C Option D Directions (4 – 6): Answer the questions on the basis of the information given below. 8 boxes – A, B, C, D, E, F, G and H are placed one above the another but not necessarily in the same order. Two boxes are placed between F and E. F is placed above E. One box is placed between F and G. Three boxes are placed between A and H. A is placed immediately below F. Two boxes are placed between C and H. B is placed somewhere above D. 4. How many boxes are placed between F and H? A) Four B) None C) Three

GovernmentAdda.com | IBPS SBI SSC RBI RRB FCI RAILWAYS

2

Daily Visit

[GOVERNMENTADDA.COM]

D) Five E) One Option A Arrangement: Two boxes are placed between F and E. F is placed above E. One box is placed between F and G. A is placed immediately below F. – G – – F F A A G – E E Three boxes are placed between A and H. Two boxes are placed between C and H. 1st possibility cancels out . H H . – G G . – – – F F F F A A A A G G C – E E E E – – H H 1st, 2nd, 4th possibility cancels out. B is placed somewhere above D.. So from top to bottom – G > B > F > A > C > E > D > H 5. Which box is placed at top? A) B B) C C) H D) G E) Cannot be determined Option D 6. Which box is placed just below box A? A) E B) D C) G D) B E) C Option E Directions (7 – 10): Answer the questions on the basis of the information given below.

8 boxes – A, B, C, D, E, F, G and H are placed one above the another but not necessarily in the same order. There are four boxes placed between D and G. Two boxes are placed between B and G. Number of boxes between A and G is same as between H and B. A is placed above G. Two boxes are placed between A and H. C is placed just above G. There are at least 2 boxes between E and B. 7. Which box is at top most position? A) C B) A C) E D) H E) None of these Option D Arrangement: 4 between D and G. and 2 between B and G.C just above G. So 2 possibilities as: – C D G – – B – – B C – G D Now — Number of boxes between A and G is same as between H and B. A is placed above G. Two boxes are placed between A and H. Gives H A – C D G A H B – – B C – G D Now: There are at least 2 boxes between E and B.. So 2nd possibility cancels out From top to bottom – H > W > D > A > B > F > C>G 8. How many boxes are between boxes E and A? A) None

GovernmentAdda.com | IBPS SBI SSC RBI RRB FCI RAILWAYS

3

Daily Visit

[GOVERNMENTADDA.COM]

B) Three C) One D) Five E) Four

immediately above an empty flat. Two of A, B and F share same flat number. 6. Who lives on flat number 2 of floor number 4? A) Empty B) D C) B D) Cannot be determined E) E

Option C 9. Which box is placed just above box B? A) A B) C C) E D) H E) D

Option C Arrangement:

Option A 10. How many boxes are below box F? A) Three B) Four C) None D) Two E) Six Option D Directions: Answer the questions on the basis of the information given below. There are 8 people – A, B, C, D, E, F, G and H who stay on 5 floors (numbered 1 to 5) of a building. There are two flats on each of the five floors out of which two flats are vacant. The flats are numbered 1 and 2 on each of the floors and are left to right on the floor respectively. Flat no. 1 of floor no. 2 is exactly above the flat no. 1 of floor no. 1 and so on. So when it is said that A lives above B means they share same flat number. Flats which are empty do not have same flat number. H lives on flat number 1 of floor number 1. There is one floor between floors of H and C. C lives above H. B lives on floor which is immediately above C’s. E lives immediately above G. A and E share same floor. There are 2 floors between A and D and they live in same flat number. D lives on one of the flats which is

7. H shares floor with which of the following? A) D B) F C) G D) No one E) Cannot be determined Option C 8. If flat number 2 of floor number 3 is empty, then who shares same floor with F? A) D B) C C) H D) No one E) Cannot be determined Option A 9. Four of the following are similar in a certain way, and so form a group. Find the odd one out.

GovernmentAdda.com | IBPS SBI SSC RBI RRB FCI RAILWAYS

4

Daily Visit

[GOVERNMENTADDA.COM]

A) A – E B) D-B C) G-C D) C-D E) B-H

Option C

Option D All others live on different flat numbers. C and D on same flat number 10. How many floors are there between the floors of B and E? A) None B) One C) Two D) Three E) Cannot be determined

2. A was born in which year? A) 1965 B) 1993 C) 1997 D) 1986 E) 1984 Option B

Option B All others live on different flat numbers. C and D on same flat number Directions (1 – 3): Answer the questions on the basis of the information given below. There are eight persons A, B, C, D, E, F, G and H. They are born in different years – 1965, 1972, 1978, 1984, 1986, 1993, 1997, and 2002. There ages are with respect to the year 2017. There is a difference of 6 years between F and G. G is younger than F. D was born before 1984. E is 15 years old. C is not 24 years old and he was born after H. The age difference between H and D is a multiple of 3. A is younger than B. B was born after F. 1. Who is 33 years old? A) G B) C C) B D) A E) D

3. Find the odd one out from the following pairs? A) B – 1984 B) E – 2002 C) G – 1978 D) D – 1972 E) C – 1997 Option D Directions (4 – 6): Answer the questions on the basis of the information given below. There are eight persons A, B, C, D, E, F, G and H. They are born in different years – 1953, 1958, 1968, 1974, 1980, 1985, 1993, and 1999. There ages are with respect to the year 2017. Only 2 people were born before H. One person was born between H and D. There is a difference of 5 years between D and G. B is not 18 years old. Two people were born between A and B. Neither A nor B is the oldest among all. A was born before B. There is a difference of 21 years between C and A. E is younger than F. 4. Who is born is year 1980? A) A

GovernmentAdda.com | IBPS SBI SSC RBI RRB FCI RAILWAYS

5

Daily Visit

[GOVERNMENTADDA.COM] G and B have seminar in same month. K’s seminar is just before G’s. There is one seminar between that of H and C. D’s seminar is after L’s seminar. L’s seminar is on 22nd of any month.

B) D C) C D) F E) None of these

7. Who has seminar on 22 May? A) A B) G C) B D) K E) D

Option B Arrangement:

Option A Arrangement:

5. What is the age of F? A) 64 years B) 53 years C) 59 years D) 49 years E) 32 years Option C

8. How many people have seminars between G and F? A) One B) Four C) None D) Two E) Three

6. How many persons are born after A? A) Four B) One C) None D) Three E) Two

Option D

Option A Directions (7 – 10): Answer the questions on the basis of the information given below. Ten persons – A, B, C, D, E, F, G, H, K and L have seminars in January, April, May, July and September with two seminars in each month. The seminar is scheduled either on 22 or 28th of the month. There are 2 seminars after E’s seminar. There is one seminar between that of E and F. A’s seminar is just before F’s. H has seminar in a month having 30 days.

9. E has seminar on? A) 22 July B) 28 May C) 22 April D) 28 July E) None of these Option D

GovernmentAdda.com | IBPS SBI SSC RBI RRB FCI RAILWAYS

6

Daily Visit

[GOVERNMENTADDA.COM]

10. How many seminars are scheduled before B’s seminar? A) One B) Four C) None D) Two E) Three

Option B

Option E Directions (1 – 5): Answer the questions on the basis of the information given below. Eight children – A, B, C, D, E, F G and H live on seven different floors (numbered 1 to 8) of a building but not necessarily in same order. Each one of them has got different marks in a test – 9, 14, 16, 23, 35, 41, 48 and 57. E lives on an even numbered floor below floor numbered 6. Two children live between E and one who got 41 marks. Same number of children live above E as below the floor of A. The one who got 23 marks lives on a floor immediately above A. One child lives between the ones who got 23 and 16 marks respectively. Two children lives between the floors of G and one who got 35 marks such that G is above the one who got 35 marks. Neither A nor E got 35 marks. One child lives between the ones who got 35 and 9 marks respectively. D lives on one of the floors below the one who got 35 marks. H got 9 marks and lives on an odd numbered floor. Difference between the marks of F and C is 21. F lives above C. A did not get highest marks. 1. Who got 35 marks? A) D B) C C) B D) E E) F

2. How many children live between A and H? A) Three B) Four C) One D) None E) Five Option A 3. Who got the highest marks? A) D B) B C) G D) E E) None of these Option A 4. Which of the following combination of floor no – child – marks is true as per given arrangement? A) 7 – B – 16 B) 8 – F – 41 C) 4 – E – 23 D) 5 – A – 48 E) 3 – D – 57 Option D 5. Who among the following lives on the floor numbered 4? A) B B) The one who got 48 marks C) The one who got 16 marks D) D E) F Option C

GovernmentAdda.com | IBPS SBI SSC RBI RRB FCI RAILWAYS

7

Daily Visit

[GOVERNMENTADDA.COM]

Directions (6 – 10): Answer the questions on the basis of the information given below.

Option D Arrangement:

Seven persons – A, B, C, D, E, F and G live on seven different floors (numbered 1 to 7) of a building but not necessarily in same order. Each one of them like different colors, namely Red, Blue, Green, Black, Yellow, Orange and White (but not necessarily in same order). All of them decided to visit a particular place with their family on seven different days of a week starting from Monday to Sunday. A lives on an odd numbered floor but not on floor numbered 3. Only two persons live between the one who likes yellow color and D. The one who likes green color lives immediately above C. The one who likes Yellow visited the place on one of the days before Thursday. The one who likes blue color lives on an odd numbered floors on one of the floors above D. B lives on one of the floors above E. Only three persons live between C and the one who likes blue color. The person who visited the place on Friday likes red Color. The one who likes red color lives on floor immediately above the one who likes white color. G lives on an odd numbered floor. E does not like green color. D visited the place on Sunday. Only one person lives between the floors of B and E. The one who likes yellow color lives on floor immediately above A. Neither of A or C likes black color. The one who visited the place on Wednesday visited the place immediately after the one who likes blue color. The one who likes green color visited the place immediately before the one who lives on floor no 3. Two persons live between the floor no 4 and the person who visited the place on Wednesday. The one who visited the place on Wednesday is not C. 6. A likes which of the following colors? A) Black B) Yellow C) Red D) Blue E) Orange

7. Which of the following combination of color – person is true with respect to the given arrangement? A) Orange – C B) Blue – F C) White – G D) Red – D E) Orange – B Answer

Option A 8. Who likes white color? A) D B) B C) G D) E E) None of these Option A 9. Which of the following statements is true with respect to the given arrangement? A) Only four persons live between D and G B) F likes red color C) The one who likes yellow color visited the place on Tuesday. D) The one who likes green color visited the place on the day immediately after E. E) G lives immediately below E. Option D

GovernmentAdda.com | IBPS SBI SSC RBI RRB FCI RAILWAYS

8

Daily Visit

[GOVERNMENTADDA.COM]

10. Who among the following lives on the floor numbered 2? A) B B) The one who likes red color C) The one who likes blue color D) D E) F

Option C Arrangement:

Option E Directions (6 – 10): Answer the questions on the basis of the information given below. 10 friends – A, B, C, D, E, P, Q, R, S, and T have birthdays in different months – January, March, April, June and September but not necessarily in the same order. There birthdays in on 2 different dates -22 and 28. So in each month there are 2 birthdays. There are 2 birthdays after the birthdays of B. There are 2 birthdays between the birthdays of B and D. A and T have birthdays in March. There is one birthday between the birthdays of A and P. P’s birthday is not in same month as D. There are same number of birthdays between T and C as between B and Q. C’s birthday is not in April. Q’s birthday is in a month having 30 days. No birthday is there between the birthdays of R and E. Also their birthdays are in different months. E’s birthday is exactly between the birthday of A and S. 6. R’s birthday is on ? A) 22 April B) 28 March C) 22 June D) 28 April E) None of these

7. Who has birthday in April? A) D B) C C) Q D) P E) B Option A 8. How many birthdays are there in between the birthdays of E and Q? A) Four B) Two C) Three D) One E) Six Option C

9. Which of the following combination of MonthPerson-Date is correct as per the given arrangement? A) June – B – 22 B) April – R – 22 C) March – A – 22 D) January – C- 22 E) January – P – 28 Option C GovernmentAdda.com | IBPS SBI SSC RBI RRB FCI RAILWAYS

9

Daily Visit

[GOVERNMENTADDA.COM]

10. Which of the following pair has birthday on 22nd of a month? A) P, B B) P, D C) C, E D) R, T E) S, E

Option B Arrangement:

Option B Directions (1 – 5): Answer the questions on the basis of the information given below. There are 8 children – A, B, C, D, E, F, G and H who live on different floors of a 8-floor building numbered 1 to 8 not necessarily in the same order. They have different chocolates – 11, 16, 19, 25, 34, 41, 46 and 50 again not necessarily in the same order. The one who lives on 6th floor has 25 chocolates. One children lives between F and the one having 25 chocolates. G lives below F on an even numbered floor. G does not have 25 chocolates. The one having 46 chocolates lives just above G. Two children Live between F and H. H lives below F. The total of number of chocolates with D and H is a multiple of 4. Two children live between A and the one having 41 chocolates. A lives above G. The one having 41 chocolates live above A. B has 34 chocolates. The one having 11 chocolates lives just above the one having 16 chocolates. 1 child lives between C and E. The difference between the number of chocolates with E and G is 6 1. How many children live above E? A) 1 B) 2 C) 4 D) None E) 5

2. Who has 41 chocolates? A)C B) E C) F D) H E) B Option A 3. Who lives on third floor? A)A B) B C) C D) D E) E Option D 4. Which of the following is correct with respect to the arrangement? A) 2 children live between B and one having 25 chocolates B) F lives on floor above A. C) A lives just below D D) H has maximum chocolates

GovernmentAdda.com | IBPS SBI SSC RBI RRB FCI RAILWAYS

10

Daily Visit

[GOVERNMENTADDA.COM]

E) None of these is correct

Option B Arrangement:

Option D 5. What is the sum of chocolates with B and D? A) 80 B) 83 C) 76 D) 96 E) 84 Option A Directions (6 – 10): Answer the questions on the basis of the information given below. There are 8 children – A, B, C, D, E, F, G and H who live on different floors of a 8-floor building numbered 1 to 8 not necessarily in the same order. They are in different class – 3, 5, 6 and 10 such that 2 children in same class. Children who are in same class live on even-odd floors. Example: If B is in class 10 with H, then if B lives on 6th floor then H lives on any odd floors – 1/3/5/7 and not 2/4/8. B and G are in same class. One of the children in class 3 lives on 5th floor. The one who lives on 3rd floor is in even numbered class. The one who lives on 7th floor and C are in same class. A is in class 6 and lives on 4th floor. 2 children live between E and A. D is in 10th class. One child lives between E and G, both of which are in odd numbered classes. 2 children live between one of the children in class 10 and F. F lives below this child. Both children in class 10 live above C. 6. Which of the following pair is in same class? A) B, F B) D, F C) E, H D) A, D E) None of these

7. Who lives on 8th floor? A) E B) One of the children in class 6 C) One of the children in class 10 D) H E) One of the children in class 3 Option E 8. How many children live between D and F? A) 2 B) 1 C) None D) 4 E) Cannot be determined Option A

9. Who lives just above G? A) C B) A GovernmentAdda.com | IBPS SBI SSC RBI RRB FCI RAILWAYS

11

Daily Visit

[GOVERNMENTADDA.COM]

C) D D) H E) B

Option E

Option C 10. Which of the following combination of floor number – child – class is correct? A) 1 – C – 5 B) 6 – B – 10 C) 5 – D – 3 D) 3 – F – 10 E) 2 – C – 6 Option D Directions (1 – 5): Answer the questions on the basis of the information given below. Eight persons Bhavya, Ravi, Kavya, Meena, Amit, Rohit, Sahil, and Medha have their birthdays on either 12th or 14th of four different months i.e March, April, May, June not necessarily in the same order. Bhavya has birthday on 12th of a month having 31 days. Only one person has birthday before Ravi. Number of persons between Bhavya and Ravi is one less than the number of persons between Ravi and Megha. Kavya and Meena have their birthdays in the same month having 30 days, but not in April. Rohit’s birthday is after Sahil’s. Only three persons have birthday between Ravi and Amit. Kavya has the birthday before Meena. 1. How many persons have birthday between Sahil and Megha? A) Three B) Two C) Four D) Five E) No one

2. Which of the following statements is/are true according to the given information? A) Amit has birthday immediately after Bhavya. B) Bhavya has birthday on 14th of June C) Kavya has birthday immediately after Amit. D) Only one person has birthday between Kavya and Meena. E) All the statements are true Option C 3. Who among the following has birthday on 14th May? A) Ravi B) Rohit C) Amit D) Kavya E) No one Option C 4. Four of the following five are alike in a certain way and so form a group. Which one does not belong to that group? A) Kavya B) Rohit C) Bhavya D) Amit E) Sahil Option D

GovernmentAdda.com | IBPS SBI SSC RBI RRB FCI RAILWAYS

12

Daily Visit

[GOVERNMENTADDA.COM]

5. Who among the following has birthday on 12th March? A) Rohit B) Meena C) Ravi D) Amit E) Bhavya Option E Directions (6 – 10): Answer the questions on the basis of the information given below. Seven boxes – A, B, C, D, E, F and G are arranged vertically from top to bottom. They all have different colors – blue, pink, green, orange, red, yellow and white but not necessarily in the same order. Also they contain different items – Rasgulla, Laddu, Peda, Nankhatai, Jalebi, Petha and Burfi but not necessarily in the same order. The box which has rasgulla is at the even numbered place from top to bottom. E has laddu and F doesn’t have nankhatai. G is kept at the bottom. Rasgulla is in either a blue or red box. A has Jalebi but not in the orange-colored box and is not kept vertically adjacent to B. There are 2 boxes between F which is a yellow colored box and the green colored box. D is neither a green nor red colored box. The orange colored box is kept at the third place from the bottom. B is a white colored box containing Burfi but is not kept with F which is at an odd numbered place. Neither petha nor nankhatai is kept in a blue box. B and D boxes are kept vertically adjacent to each other and none of them is an orange colored box. The box which has petha is kept at second place from the top. 6. How many boxes are there between boxes F and the one which is orange in color? A) None B) 3 C) 2 D) 5 E) 1 Option E Arrangement:

7. Which box contains Laddu? A) A B) D C) Yellow box D) Orange box E) None of these Option D 8. White box contains which item? A) Laddu B) Peda C) Other than the given options D) Burfi E) Jalebi Option D 9. Color of box G is A) Red B) Yellow C) Blue D) White E) None of these Option A 10. How many boxes are below blue box? A) 2 B) 3

GovernmentAdda.com | IBPS SBI SSC RBI RRB FCI RAILWAYS

13

Daily Visit

[GOVERNMENTADDA.COM]

C) 1 D) 5 E) None of these

6. Who stays on 8th floor? A) Charu B) The one studying BCom C) The one who likes Purple color D) The one who likes White color E) The one from Haryana

Option B Directions (6 – 10): Answer the questions on the basis of the information given below. Nine people Ankul, Priyal, Seema, Abhi, Rajat, Charu, Reema, Anjali and Gaurav stay in a building (floors numbered 1 to 9). They are studying different courses – BSc and BCom. Four of them are studying BCom and remaining persons are studying BSc. All of them belong to a different state viz – Haryana, Punjab, Assam, Nagaland, Telangana, Kerala, Jharkhand, Maharashtra and Bihar, but not necessarily in the same order.. Each of them also likes a different color – Orange, Grey, Pink, Purple, White, Blue, Green, Red and Yellow, again but not necessarily in the same order. There is one floor between the floors on which Charu, who likes Grey color and Reema stay. Abhi, who likes Blue, stays on a floor immediately above Gaurav’s floor, who likes Green color. Neither Ankul nor Seema belongs to Telangana. Seema does not belong to Haryana and likes Red color. The one who belongs to Jharkhand stays on the fourth floor but is not studying BSc. There are two floors between the floors on which the people who belong to Kerala and Nagaland stay. Rajat stays on the second floor and belongs to Assam. There are three floors between the floor on which Seema, who is studying BCom and Reema stay, who does not belong to Bihar and likes Pink color. The person who belongs to Telangana and Haryana are studying same course. Ankul is studying BSc and lives on an even numbered floor which is below the floor on which Anjali stays, who likes purple color. The one who belongs to Bihar stays on the third floor and is not studying BCom. The one who belongs to Haryana stays on the topmost floor and likes Orange color. The person belongs to Kerala and Nagaland are studying same course. Anjali belongs to Nagaland and is studying BSc and lives on an even numbered floor. The person belongs to Maharashtra does not live below the person who belongs to Punjab. Ankul does not like Yellow color.

Option C Arrangement:

7. How many floors are there between the ones who like green color and who is from Maharashtra? A) None B) Two C) Four D) One E) None of these Option B 8. Who is staying 4 floors above the one from Assam? A) The one from Kerala B) The one who likes purple color C) The one from Telangana D) Ankul E) The one who likes grey color Option D

GovernmentAdda.com | IBPS SBI SSC RBI RRB FCI RAILWAYS

14

Daily Visit

[GOVERNMENTADDA.COM]

9. Four of the following five are alike in a certain way and forms a group. Find the one who does not belong to the group? A) Priyal B) The one from Telangana C) The who likes red color D) The one from Nagaland E) Gaurav Option D All others stay on odd numbered floor

Option C 7. How many persons have seminars before Ananya? A) Two B) Three C) Five D) None E) One Option B

10. Which of the following is correct combination as per given arrangement? A) Anjali – Nagaland – Grey – BSc B) Ankul – Telangana – Grey – BCom C) Reema – Kerala – Pink – BSc D) Seema – Assam – Red – BCom E) Priyal – Haryana – Orange – BSc

8. How many seminars are between seminars of Ankur and Rohit? A) Five B) Three C) Two D) One E) None

Option C Directions (6 – 10): Answer the questions on the basis of the information given below. Eight persons – Adiya, Sahil, Ananya, Anshika, Ankur, Tiya, Rohit, Kavya have seminars in January, April, September and December with two seminars in each month. The seminar is scheduled either on 22 or 28th of the month. Aditya’s seminar is scheduled in a month having 30 days. Seminar of Kavya is somewhere after Tiya’s seminar. Ankur’s seminar is on 22nd April. There are two seminars scheduled between Sahil’s and Ananya’s seminars. One of seminars of Sahil and Ananya is somewhere before Ankur’s seminar. Rohit’s seminar is immediately after Ananya’s seminar. There is only one seminar after Kavya’s seminar. Ananya and Tiya have seminars on same date but not in September. 6. Who has seminar on 28th September? A) Sahil B) Ananya C) Aditya D) Rohit E) Cannot be determined

Option D 9. Who has seminar on 22nd December? A) Tiya B) Aditya C) Anshika D) Kavya E) Sahil Option D 10. Find the odd one out. A) Rohit – 22 B) Ananya – September C) Ankur – April D) Aditya – 28 E) Anshika – 28 Option B Directions (1 – 5): Answer the questions on the basis of the information given below.

GovernmentAdda.com | IBPS SBI SSC RBI RRB FCI RAILWAYS

15

Daily Visit

[GOVERNMENTADDA.COM]

Eight people – Sheetal, Seema, Sakshi, Swati, Saina, Sanya, Sheena, Shrishti are born in different years – 1962, 1965, 1973, 1978, 1982, 1989, 1996, and 2005. There ages are with respect to the current year, same month and same date. There is 9 years difference between Shrishti and Sanya. Sheetal was born before 1973. Sheena is not 12 years old. Sakshi is 28 years old. Saina is 16 years older than Seema. Seema is younger than Shrishti. 1. Who was born in the year 1989? A) Swati B) Sakshi C) Sheetal D) Saina E) Seema

E) Sanya – 1982 Option C 5. What is the sum of ages of Sheetal and Sanya? A) 81 B) 84 C) 79 D) 87 E) 90 Option D Sheetal is 52 years old and Sanya is 35 years old. So total = 52+35

Option B Directions (6 – 10): Answer the questions on the basis of the information given below.

2. Sheetal was born in which year? A) 1962 B) 1965 C) 1973 D) 1978 E) 1989 Option B 3. What is the age difference between Seema and Sheena? A) 14 B) 17 C) 12 D) 18 E) 20

Eight persons – A, B, C, D, E, F, G and H have seminars in March, April, September and December with two seminars in each month. The seminar is scheduled either on 22 or 28th of the month. There is one seminar before the seminar of B. There are at least 2 seminars after D’s seminar. E and F have seminars on same date. A’s seminar is just before E’s. There is only one seminar between H and A and similarly between A and G. A’s seminar is on 22nd April. H’s seminar is before G’s seminar. 6. Who has seminar on 28th September? A) D B) H C) G D) C E) B

Option D 4. Find the odd one out from the following pairs? A) Saina – 1962 B) Seema – 1978 C) Swati – 1996 D) Sakshi – 1989

GovernmentAdda.com | IBPS SBI SSC RBI RRB FCI RAILWAYS

16

Daily Visit

[GOVERNMENTADDA.COM] D) E – April E) G – September

Option A

Option C Directions (1 – 5): Answer the questions on the basis of the information given below. Eight people – P, Q, R, S, T, U, V and W are born in different years – 1942, 1948, 1963, 1969, 1972, 1978, 1991, and 2003. There ages are with respect to the current year (2017), same month and same date. T was born before 1963. Difference between the ages of T and W is 21 years. P’s is older than W but not the oldest. Q’s birth year is an odd-numbered year. Neither Q nor R is 14 years old. Difference between the ages of Q and U is less than 5 years. The sum of ages of R and S is 53 years.

7. How many persons have seminars before E? A) Two B) Three C) Five D) None E) One Option B 8. How many seminars are between seminars of B and D? A) Five B) Three C) Two D) One E) None Option B 9. Who has seminar on 22nd March? A) B B) E C) H D) D E) A Option C 10. Find the odd one out. A) H – 22 B) C – 22 C) D – April

1. Who was born in the year 1991? A) U B) R C) S D) V E) Cannot be determined Option D 2. What is the age difference between P and Q? A) 22 B) 21 C) 23 D) 24 E) 20 Option B 3. U is born in which year? A) 1963 B) 1978 C) 1972 D) 1991 E) None from above is facing same direction

GovernmentAdda.com | IBPS SBI SSC RBI RRB FCI RAILWAYS

17

Daily Visit

[GOVERNMENTADDA.COM] E) B

Option C 4. Find the odd one out from the following pairs? A) T – 1942 B) W – 1963 C) Q – 1972 D) V – 1991 E) R – 1978

Option C

Option C 7. How many persons have seminars before E? A) Two B) Three C) Five D) None E) One

5. What is the sum of ages of P and R? A) 108 B) 114 C) 99 D) 102 E) 110

Option B Option A P is 69 years old and R is 39 years old. So total = 69+39

Directions (6 – 10): Answer the questions on the basis of the information given below. Eight persons – A, B, C, D, E, F, G and H have seminars in January, March June and September with two seminars in each month. The seminar is scheduled either on 22 or 28th of the month. B’s seminar is in March. There is one seminar scheduled between A and B’s. D and C have seminars in same month. G has seminar on 28th of one of the month. A’s seminar is in month having 31 days. Seminar of H is immediately after the seminar of C. The number of seminars between A and E is same as number of seminars between D and G. Seminar of F is somewhere before G. 6. Who has seminar on 28th September? A) D B) H C) G D) C

8. How many seminars are between seminars of B and D? A) Five B) Three C) Two D) One E) None Option D 9. Who has seminar on 22md March? A) B B) E C) F D) D E) A Option A 10. Find the odd one out. A) H – 22 B) C – 28 C) D – June

GovernmentAdda.com | IBPS SBI SSC RBI RRB FCI RAILWAYS

18

Daily Visit

[GOVERNMENTADDA.COM]

D) E – 22 E) G – September Option D

GovernmentAdda.com | IBPS SBI SSC RBI RRB FCI RAILWAYS

19

Daily Visit

[GOVERNMENTADDA.COM]

I. Study the following information carefully to answer the given questions.

Seven different boxes A, B, C, D, E, F and G of different colours viz., Green, Red, Blue, Yellow, Purple, Pink and Orange are arranged one above the other. The box at the bottom of arrangement is numbered 1, the above box is numbered 2 and so on. B is immediately above E. More than two boxes are above the Green box. The Yellow box is immediately below A. Only one box is between the Green box and F. G is immediately above the Pink box. Only one box is between B and the Red box. Only two boxes are between the Red and the Orange box. Only two boxes are between the Yellow box and the Green box. The blue box is neither at the top nor at the bottom of the arrangement.B is above Red box. C is immediately above F. Neither C nor G is a Yellow box. G is not a Green box. Explanation Position 7 6 5 4 3 2 1

Box C F B E G A D

Colour Purple Orange Blue Green Red Pink Yellow

1. As per the given arrangement, G is related to F and A is related to E in a certain way. To which of the following is D related to the same way? A. D B. B C. A D.C E. None of the Above Answer Answer – C. A 2. Which of the following pairs of people occupy the top and botttom positions of the arrangement? A. G, A GovernmentAdda.com | IBPS SBI SSC RBI RRB FCI RAILWAYS

20

Daily Visit

[GOVERNMENTADDA.COM]

B. D, C C. C, B D. B, D E. A, D Answer Answer – B. D, C 3. Which combination represents the position of C and its colour? A. 5 – Purple B. 4 – Pink C. 1 – Yellow D. 4 – Green E. 7 – Purple Answer Answer – E. 7 – Purple 4. Which amongst the following arranged exactly in the middle? A. A B. D C. B D. E E. Other than those given as options Answer Answer – D. E 5. Which of the following is the colour of Box “A”? A. Red B. Purple C. Pink

GovernmentAdda.com | IBPS SBI SSC RBI RRB FCI RAILWAYS

21

Daily Visit

[GOVERNMENTADDA.COM]

D. Green E. None of the Above Answer Answer – C. Pink II. Study the following information carefully to answer the given questions.

Seven students namely A, B, C, D, E, F and G are studying in three different standards- IV, V, VI. At-least one and not more than three among them in any of these standards. Each of them went to tour in a week starting from Monday to Sunday. B is studying only with E in standard V and he went to tour immediately on the next day of A’s tour. D went to tour on Sunday and he is not in the same class with either C or B. C went to tour immediately after B and immediately before E. The boy went to tour on Friday studying in Standard VI. The one who went to tour on Saturday did not study with E. G did not study either in standard V or in Standard VI. A is studying in standard IV with C. F went to tour immediately after E but not on Saturday. Explanation Day Monday Tuesday Wednesday Thursday Friday Saturday Sunday

Person A B C E F G D

Standard IV V IV V VI IV VI

6. Which of the following studing in standard IV ? A. A, C, G B. D, B, G C. B, E, G D. F, G, D E. None of the Above Answer

GovernmentAdda.com | IBPS SBI SSC RBI RRB FCI RAILWAYS

22

Daily Visit

[GOVERNMENTADDA.COM]

Answer – A. A, C, G 7. Which of the following combinations of Day-Person-Standard is correct ? A. Saturday – G -VI B. Tuesday – B – III C. Thursday – F – III D. Monday – A – V E. Sunday – D – VI Answer Answer – E. Sunday – D – VI 8. Which of the following statements is true with respect to the given arrangement? A. B went to tour on Tuesday B. E is studying in IV standard C. One who are studying in V went to tour on neither Tuesday nor Thursday D. G went to tour on Sunday E. All of the given statements are true Answer Answer – A. B went to tour on Tuesday 9. Who among the following went to tour in Thursday? A. B B. E C. G D. F E. A Answer Answer – B. E

GovernmentAdda.com | IBPS SBI SSC RBI RRB FCI RAILWAYS

23

Daily Visit

[GOVERNMENTADDA.COM]

10. Who among the following went to tour immediately after F ? A. B B. E C. G D. F E. A Answer Answer – C. G I. Study the following information carefully to answer the given questions.

Seven persons – A, B, C, D, E, F and G live on seven different floors of a building but not necessarily in same order. Lowermost floor of building is numbered 1, one above that is numbered 2 and so on till top most floor is numbered 7. Each one of them also likes a different food recipe, namely Egg roll, Pizza, Burger, Noodles, Briyani, Fried Rice and Masala Dosa.(but not necessarily in same order). 

A lives an odd numbered floor but not on floor numbered Three. Only two persons live between D and the one who likes Briyani. The One who likes Burger lives immediately above C.



The one who likes Pizza lives on odd numbered floors above D. B lives on the floors above E. Only three persons live between C and the one who likes Pizza.



The one who likes Egg roll lives immediately above the one who likes Masala Dosa. G lives on an odd numbered floor. E does not like Burger.



Only one person lives between B and E. The one who likes Briyani lives immediately above A. Neither C nor A likes Noodles.

Explanation Floor No

Person

Food Recipe

7

G

Noodles

6

B

Briyani

5

A

Pizza

4

E

Egg roll

3

D

Masala dosa

2

F

Burger GovernmentAdda.com | IBPS SBI SSC RBI RRB FCI RAILWAYS

24

Daily Visit

[GOVERNMENTADDA.COM]

1

C

Fried Rice

1. Which of the following food recipe does A like? A. Fried Rice B. Masala Dosa C. Burger D. Pizza E. Egg roll Answer Answer – D. Pizza 2. Which of the following combinations is True with respect to the given arrangement? A. Fried Rice – C B. Pizza – F C. Masala Dosa – G D. Briyani – D E. Noodles – B Answer Answer – A. Fried Rice – C 3. If all the persons are made to sit in alphabetical order from top to bottom, positions of how many persons will remain unchanged? A. Four B. None C. Two D. One E. Three Answer Answer – C. Two

GovernmentAdda.com | IBPS SBI SSC RBI RRB FCI RAILWAYS

25

Daily Visit

[GOVERNMENTADDA.COM]

4. Which of the following statements is True with respect to the given arrangement? A. The one who likes Masala Dosa immediately below A. B. F likes Pizza C. None of the given options is true. D. Only four persons live between D and G E. G lives immediately below E. Answer Answer – C. None of the given options is true. 5. Who among the following lives on the floor numbered 2? A. B B. The one who likes Egg roll C. The one who likes Masala Dosa D. D E. F Answer Answer – E. F II. Study the following information carefully to answer the given questions.

Seven Friends, namely A, B, C, D, E, F and G visit seven different Countries namely US Germany China, UAE, Netherlands, UK and Russia,(but not necessarily in same order). starting from Monday to Sunday(of the same week).

C visits on Thursday. Only two people visit between C and the one who visits Germany. Only four people visit between the one who visits Germany and G. The one who visits Russia visits immediately before G. Only two people visit between the one who visits Russia and A. D visits one of the days after the one who visit Russia. F visits immediately after the one who visit US. F does not visit Russia. Only three people visit between the one who visits US and the one who visits Netherlands. The one who visits UK visits immediately before the one who visits China. B does not visit on Monday. Explanation Day

Person

Country

GovernmentAdda.com | IBPS SBI SSC RBI RRB FCI RAILWAYS

26

Daily Visit

[GOVERNMENTADDA.COM]

Monday

E

Germany

Tuesday

A

US

Wednesday

F

UK

Thursday

C

China

Friday

B

Russia

Saturday

G

Netherlands

Sunday

D

UAE

6. Which of the following countries does B visit? A. China B. Russia C. US D. Netherlands E. UK Answer Answer – B. Russia 7. On which of the following days does F visit a country? A. Friday B. Saturday C. Sunday D. Wednesday E. Tuesday Answer Answer – D. Wednesday 8. Which of the following is true about E? A. All the options are true B. E visits on Friday C. E visits China D. Only three people visit between E and C E. E visits immediately before A. GovernmentAdda.com | IBPS SBI SSC RBI RRB FCI RAILWAYS

27

Daily Visit

[GOVERNMENTADDA.COM]

Answer Answer – E. E visits immediately before A. 9. Who amongst the following visits UAE? A. D B. E C. A D. G E. C Answer Answer – A. D 10. As per the given arrangement, A is related to the one who visits US in a certain way and G is related to the one who visits Netherlands in the same way. To which of the following is C related to in the same way. A. The one who visits UK B. The one who visits China C. The one who visits UAE D. The one who visits Russia E. The one who visits Germany Answer Answer – B. The one who visits China I. Study the following information carefully to answer the given questions.

Seven people, namely P,Q,R,S,T,U and V like seven different e-commerce websites namely Amazon, Flipkart, Snapdeal, E-bay, Jabong, Myntra and Paytm but not necessarily in the same order. Each people also works in the same office but at a different department on the basis of experience namely Administration (ADMIN), Marketing & Sales, (M&S), Accounts (ACC), Production (PO), Quality Management (QM), Human Resources (HR), and Public Relations (PR), but not necessarily in the same order. Each person also like different cars namely viz – Audi, BMW, Ford, Fiat, Hyundai, Chevrolet and Ferrari.

GovernmentAdda.com | IBPS SBI SSC RBI RRB FCI RAILWAYS

28

Daily Visit

[GOVERNMENTADDA.COM]

Note: Each person has been allocated to a department as per increasing order of experience with the one in ADMIN being the least experienced whilst the one in PR Being the most experienced.

T neither has the least experience than the one who likes Snapdeal. T neither has the least experience nor he works in QM. Q does not work in QM. The one who likes Flipkart does not work in PO. The person who likes Myntra has more experience than the one who likes Fiat. The one in Quality Management likes Chevrolet. The person who likes Jabong also likes the Hyundai car. Persons who have the least experience and most experience like BMW and Ferrari car respectively. Only one person has less experience than U. V likes Paytm and has more experience than the one who likes Amazon. S has less experience than the one in PO, but more experience than the one who likes Snapdeal. The one who has less experience than U likes E-bay. Only one person has more experience than P. P does not like Audi. The one in Marketing and Sales like Ford. The one in HR likes Jabong. Only two people have more experience than the one who likes Amazon Explanation Person V P R T S U Q

Job PR HR QM PO ACC M&S ADMIN

E-Commerce Paytm Jabong Amazon Myntra Flipkart Snapdeal E-bay

Car Ferrari Hyundai Chevrolet Audi Fiat Ford BMW

1. As per the given arrangement, ADMIN is related to Ferrari and PR is related to Hyundai in a certain way. To which of the following is ACC related to the same way? A. Ferrari B. BMW C. Ford D. Fiat E. Chevrolet Answer Answer –E. Chevrolet

GovernmentAdda.com | IBPS SBI SSC RBI RRB FCI RAILWAYS

29

Daily Visit

[GOVERNMENTADDA.COM]

2. Which of the following pairs of people who have more experience than P less experience than S? A. V, P B. V, U C. R, V D. T, Q E. R, P Answer Answer – B. V, U 3. Which combination represents the department that T works in and the movie he likes? A. QM – Amazon B. PO – Snapdeal C. PO – Myntra D. ACC – E-bay E. ADMIN – E-bay Answer Answer – C. PO – Myntra 4. Who amongst the following likes Fiat? A. S B. R C. P D. Q E. Other than those given as options Answer Answer – A. S 5. Which of the following e-commerce websites does Q like? A. Snapdeal B. Myntra GovernmentAdda.com | IBPS SBI SSC RBI RRB FCI RAILWAYS

30

Daily Visit

[GOVERNMENTADDA.COM]

C. Amazon D. E-bay E. Flipkart Answer Answer – D. E-bay II. Study the following information carefully to answer the given questions.

Seven persons – A, B, C, D, E, F and G – went to tour in the months of February, March, April May, July, October and December but not necessarily in the same order. Each one of them likes different brand of cycle viz., Firefox, Hercules, Atlas, BSA, Hero, Montra and Kross but not necessarily in the same order. Each person also like seven different brand of bikes namely viz – Honda, Yamaha, Suzuki, Harley Davidson, TVS, Royal Enfield and Vespa.

There are two persons went to tour between the one who likes Honda and the one who likes Vespa. E does not like Atlas. The person who likes Montra went to tour in the month having less than 31 days. The person who likes Honda went to tour on one of the months after March which has less than 31 days. The one who likes Hero went to tour in the month having less than 31 days. There is only one person between A and the person who likes Hero. The person who likes Vespa went to tour immediately before the one who likes Suzuki. G went to tour in that month which has less than 31 days. F went to tour immediately after G. Only one person went to tour between A and the who likes BSA. F does not like Harley Davidson. A does not like Montra. The one who likes Firefox went to tour immediately before the one who likes Kross. The person who likes Yamaha went to tour immediately before the one who likes Royal Enfield and immediately after the one who likes Honda. The one who likes Atlas went to tour immediately before A. C went to tour immediately after A. Only two persons went to tour between C and B. Explanation Month February March April May

Person G F B D

Cycles Montra Hercules Hero Atlas

Bike Harley Davidson Kit-Kat Honda Yamaha

July

A

Firefox

Royal Enfield

October December

C E

Kross BSA

Vespa Suzuki

GovernmentAdda.com | IBPS SBI SSC RBI RRB FCI RAILWAYS

31

Daily Visit

[GOVERNMENTADDA.COM]

6. Which of the following brand of cycles is liked by C? A. Firefox B. BSA C. Montra D. Hercules E. Kross Answer Answer – E. Kross 7. Which of the following combinations of Month-Person-Cycle-Bike is correct? A. March – G – Firefox – Harley Davidson B. July – A – Firefox – Royal Enfield C. October – E – Montra – Yamaha D. May – C – Atlas – TVS E. April – F – Hero – Vespa Answer Answer – B. July – A – Firefox – Royal Enfield 8. Which of the following statements is true with respect to the given arrangement? A. C went to tour in October B. A likes Kross C. D went to tour immediately before E. D. E went to tour in July E. None of the given statements is true Answer Answer – A. C went to tour in October 9. Who among the following went to tour in May? A. F B. A GovernmentAdda.com | IBPS SBI SSC RBI RRB FCI RAILWAYS

32

Daily Visit

[GOVERNMENTADDA.COM]

C. C D. D E. B Answer Answer – D. D 10. Who among the following likes Royal Enfield? A. E B. C C. A D. G E. B Answer Answer – C. A 1. Study the following information carefully to answer the given questions. Seven people namely B, C, D, E, F, G and H have to attend a workshop but not necessarily in the same order, in seven different months of the same year namely January, March, April, July, August, September and December. Each of them also likes a different bank namely CANARA, SBI, ICICI, HDFC, AXIS, PNB and Yes but not necessarily in the same order. The one who likes AXIS will attend a workshop in the month which has less than 31 days. Only one person will attend a workshop between the one who likes AXIS and B. The one who likes CANARA will attend a workshop immediately before B. Only three people will attend a workshop between one who likes AXIS and the one who likes SBI. H will attend a workshop immediately after B. Only three people will attend a workshop between H and G. The one who likes PNB will attend a workshop immediately before G. The one who likes ICICI will attend a Workshop immediately before the one who likes Yes. E will attend a Workshop immediately after the one who likes Yes. C will attend a workshop in a month which has only 30 days. F does not like CANARA. Explanation Month January

Person F

Banks PNB

GovernmentAdda.com | IBPS SBI SSC RBI RRB FCI RAILWAYS

33

Daily Visit March April July August September December

[GOVERNMENTADDA.COM] G C D B H E

HDFC AXIS CANARA ICICI Yes SBI

1. How many people will attend a workshop between the months in which G and D will attend a Workshop? A. Two B. One C. Three D. More than three E. None Answer Answer – B. One 2. As per the given arrangement, G is related to CANARA and B is related to SBI following a certain pattern, which of the following is D related to following the same pattern? A. ICICI B. PNB C. Axis D. CANARA E. Yes Answer Answer – E. Yes 3. Which of the following represents the month in which F will attend a workshop? A. March B. January C. December D. April E. Cannot be determined GovernmentAdda.com | IBPS SBI SSC RBI RRB FCI RAILWAYS

34

Daily Visit

[GOVERNMENTADDA.COM]

Answer Answer – B. January 4. Which of the following represents the people who will attend a workshop in March and December respectively? A. G, H B. F, B C. F, E D. F, H E. G, E Answer Answer – E. G, E 5. Which of the following banks does G like? A. ICICI B. CANARA C. AXIS D. HDFC E. SBI Answer Answer – D. HDFC II. Study the following information carefully to answer the given questions Seven People namely M, N, O, P, Q, R and S have an anniversary but not necessarily in the same order, in seven different months of the same year namely February, March, April, June, September, October and November. Each of them also likes a different book namely One Indian Girl, The God of Small Things, Everyone Has a Story, The Ministry of utmost Happiness, Half Girl Friend, Bahubali: The Battle of Bold and The Small Town Sea but not necessarily in the same order. R has an anniversary in the month which has more than 30 days. Only one person has an anniversary between R and the one who likes One Indian Girl. Both S and O have an anniversary in one of the months after the one who likes One Indian Girl. S has an anniversary immediately before O. The one who likes Everyone Has a Story has an anniversary in the month which has less than 30 days. Only three people have an anniversary between the one who likes Everyone Has a GovernmentAdda.com | IBPS SBI SSC RBI RRB FCI RAILWAYS

35

Daily Visit

[GOVERNMENTADDA.COM]

Story and the one who likes The Small Town Sea. Only two people have an anniversary between S and the one who likes The Ministry of utmost Happiness. P has an anniversary immediately after the one who likes The Ministry of utmost Happiness. Only two people have an anniversary between P and Q. M has an anniversary immediately before the one who likes The God of Small Things. O does not like Bahubali: The Battle of Bold. Explanation Month February March April June September October November

Person Q R N P M S O

Book Everyone Has a Story Bahubali: The Battle of Bold The Ministry of utmost Happiness One Indian Girl The Small Town Sea The God of Small Things Half Girl Friend

6. Which of the following represents the month in which S has an anniversary? A. October B. March C. April D. September E. Can not be determined Answer Answer – A. October 7. Which of the following does O like? A. One Indian Girl B. The God of Small Things C. The Ministry of utmost Happiness D. Half Girl Friend E. The Small Town Sea Answer Answer – D. Half Girl Friend GovernmentAdda.com | IBPS SBI SSC RBI RRB FCI RAILWAYS

36

Daily Visit

[GOVERNMENTADDA.COM]

8. As per the given arrangement, Everyone Has a Story is related to April and The Ministry of utmost Happiness is related to September following a certain pattern, which of the following is The Small Town Sea related to following the same pattern? A. February B. June C. October D. November E. March Answer Answer – D. November 9. Which of the following represents the people who have an anniversary in April and November respectively? A. N,M B. Q,M C. Q,O D. N,O E. N,S Answer Answer – D. N,O 10. How many people have an anniversary between the months in which Q and M have an anniversary? A. None B. One C. Three D. Two E. More than three Answer Answer – C. Three

GovernmentAdda.com | IBPS SBI SSC RBI RRB FCI RAILWAYS

37

Daily Visit

[GOVERNMENTADDA.COM]

I. Study the following information carefully to answer the given questions.

Seven Lecturers – A, B, C, D, E, F and G will go to Seven different Universities namely Indian Institute of Science, University of Delhi, Banaras Hindu University, Osmania university, IIT Madras, Shivaji University and University of Mumbai but not in necessarily same order to give guest lectures. They take lectures on the subjects like Economics, History, Electronics, Computer, Agriculture, Bio Technology and Chemistry but not necessarily in the same order. They delivers lectures on Monday to Sunday(of the same week) not necessarily in same order. 

A gives guest lecture in Indian Institute of Science either in Economics or Bio Technology. The person who give guest lecture on the seventh day of the week went to University of Mumbai and He is expert in Chemistry. C does not give lecture in Bio Technology



B takes the lecture on Wednesday only and He is not expert in computer and Economics The lecturer who takes guest lecture in agriculture went on Thursday and neither in University of Delhi nor in Shivaji University.



D delivers lecture on Electronics in Osmania Univeresity except on Monday and Tuesday. C delivers a guest lecture at Banaras Hindu University and takes lecture one day before D.



G does not went to Thursday.On the second day of the week F gives the guest lecture in Shivaji University.The lecturer who delivers lecture on Friday is not expert in Computer, Bio Technology and Economics.

Explanation Persons

Subjects

Universities

Days

A

Economics

Indian Institute of Science

Monday

F

Computer

Shivaji University

Tuesday

B

Bio Technology

University of Delhi

Wednesday

E

Agriculture

IIT Madras

Thursday

C

History

Banaras Hindu University

Friday

GovernmentAdda.com | IBPS SBI SSC RBI RRB FCI RAILWAYS

38

Daily Visit

[GOVERNMENTADDA.COM]

D

Electronics

Osmania university

Saturday

G

Chemistry

University of Mumbai

Sunday

1. Which of the following subject is handled by A? A. Computer B. Electronics C. Chemistry D. History E. Economics Answer Answer – E. Economics 2. Which of the following combinations of “University – Day” is True with respect to the given arrangement? A. Osmania University – Friday B. University of Mumbai -Saturday C. IIT Madras – Thursday D. University of Mumbai – Friday E. Osmania University – Thursday Answer Answer – C. IIT Madras – Thursday 3. According to the arrangement, B is related to? A. Computer B. Electronics C. Chemistry D. Bio Technology E. Economics Answer Answer – D. Bio Technology GovernmentAdda.com | IBPS SBI SSC RBI RRB FCI RAILWAYS

39

Daily Visit

[GOVERNMENTADDA.COM]

4. In this arrangement, A is related to Monday, F is related to Tuesday then C is related to? A. Thursday B. Friday C. None of the given options is true. D. Saturday E. Sunday Answer Answer – B. Friday 5. Who among the following went to Banaras Hindu University? A. B B. C C. A D. D E. F Answer Answer – B. C II. Study the following information carefully to answer the given questions.

Eight people P, Q, R, S, T, U, V and W were born in three different months(of the same year) but not necessarily in the same order, namely March June and December such that not less than two people and not more than three people were born in a month. Each of them also likes a different fruit namely Guava, Peach, Banana, Cherry, Mango, Orange, Kiwi and apple but not necessarily in the same order. 

Only Q and W were born in March. R likes Apple and was born in the same month as T. R was not born in December. The one who likes Mango was born in the month which has 30 days only.



U was not born in the same month as T. S likes Cherry and born in the same month as U. V does not like Mango.



The one who likes Kiwi and the one who likes Banana were born in the same month, The one who likes Kiwi was not born in the same month as W. GovernmentAdda.com | IBPS SBI SSC RBI RRB FCI RAILWAYS

40

Daily Visit 

[GOVERNMENTADDA.COM]

U does not like Kiwi. The one who likes Guava was born in the same month as P. Q does not like Peach. T does not like Mango.

Explanation Persons

Months

Fruits

Q

March

Orange

W

March

Peach

R

June

Apple

T

June

Guava

P

June

Mango

S

December

Cherry

U

December

Banana

V

December

Kiwi

6. As per the given arrangement which of the following combination represents only the people who were born in December?. A. T, V B. U, P, T C. V, U D. P, T E. U, V, S Answer Answer – E. U, V, S 7. As per the given arrangement which of the following person represent the one who was born in the same month as the one who likes Orange? A. U B. P C. R D. T E. W Answer

GovernmentAdda.com | IBPS SBI SSC RBI RRB FCI RAILWAYS

41

Daily Visit

[GOVERNMENTADDA.COM]

Answer – E. W 8. Which of the following fruits does T like as per the given arrangement? A. Orange B. Peach C. Guava D. Banana E. Other than those given as options Answer Answer – C. Guava 9. Which of the following combinations is correct as per the given arrangement? A. December – Peach B. June – Orange C. June – Banana D. March – Guava E. December – Banana Answer Answer – E. December – Banana 10. Who amongst the following likes Peach as per the given arrangement? A. P B. V C. U D. W E. T Answer Answer – D. W I. Study the following information carefully to answer the given questions. GovernmentAdda.com | IBPS SBI SSC RBI RRB FCI RAILWAYS

42

Daily Visit

[GOVERNMENTADDA.COM]

There are Seven different TV Shops – A, B, C, D, E, F and G sold TVs starting from Monday to Sunday (of the same week) not necessarily in same order. The number of TVs sold by the Seven TV shops in seven different days are 4, 6, 9, 12, 18, 15 and 27 (not necessarily in same order). 

The Shop ‘A’ sold TVs on one of the days after Friday. On Wednesday, the number of TVs sold are 12. The difference between the number of TVs sold on Monday and Friday is the multiple of the number 7



There are two shops between the shop F and the shop which one sold 4TVs. Shop B sold TVs on one of the days immediately before the shop that sold 9 TVs. There are two shops between B and G.



There are three shops between the shop F and D. C not sold the least number of TVs. The sum of TVs sold on Wednesday and Saturday is more than ten and the sum equals to the number of TVs sold on Friday.



The difference between the number of TVs sold by Shop B and G is less than five. The Shop which sold 9 TVs not on Friday.The shop F sold the TVs on one of the days after the shop which sold 4TVs.



The shop which sold more than 12 TVs(not an odd number) is immediately after the one which one sold 12 TVs. Shop F sold more number of TVs than Shop D.

Explanation Days

Shops

Number of TVs

Monday

D

6

Tuesday

E

4

Wednesday

G

12

Thursday

C

18

Friday

F

27

Saturday

B

15

Sunday

A

9

1. Which of the following is sold by Shop A? A. 12 B. 15 C. 27 D. 6 E. 9 Answer Answer – E. 9 GovernmentAdda.com | IBPS SBI SSC RBI RRB FCI RAILWAYS

43

Daily Visit

[GOVERNMENTADDA.COM]

2. Which of the following combinations of “Shop – Day” is True with respect to the given arrangement? A. E – Friday B. A – Saturday C. C – Thursday D. B – Friday E. E – Thursday Answer Answer – C. C – Thursday 3. Which of the following shop sold TV’s immediately after A? A. D B. E C. F D. G E. None Answer Answer – E. None 4. In this arrangement, A is related to Monday, F is related to Wednesday then B is related to? A. Tuesday B. Friday C. None of the given options is true. D. Saturday E. Sunday Answer Answer – A. Tuesday 5. Which among the following shop sold TVs in Monday? A. B B. C GovernmentAdda.com | IBPS SBI SSC RBI RRB FCI RAILWAYS

44

Daily Visit

[GOVERNMENTADDA.COM]

C. A D. D E. F Answer Answer – D. D II. Study the following information carefully to answer the given questions.

Seven cricket players viz. P, Q, R, S, T, V and W played in seven different matches in a week starting from Monday and ending on Sunday. Some of them are bowlers and some of them are batsman. Their Run/Wickets in the seven matches are – 50, 5, 4, 16, 8, 4 and 25. 

V plays on Thursday. Two matches are played between the matches played by player V and player P. On the first day of the week, the batsman scored half century played in the match.



The sum of Runs/Wickets of the player W and Q is equalled to the Runs/Wickets of S. All the bowlers should be preceded and succeeded by batsman and T is the only bowler which can not be succeeded by any one. P is the only batsman which is followed by another batsman.



Only one match is played between the matches played by the player T and S. The score of the batsman who played the match on Thursday is the perfect square of the score of the player who played on Saturday.



The match played by T is not played on the day immediately before or immediately after the day when the match of player V is played.



The match played by R is played on the day immediately before the day when the match of player W is played. Match played by S is not played after the match of player Q.

Explanation Days

Player

Role

Runs/Wickets

Monday

P

Batsman

50

Tuesday

R

Batsman

25

Wednesday

W

Bowler

4

Thursday

V

Batsman

16

Friday

S

Bowler

8

Saturday

Q

Batsman

4

Sunday

T

Bowler

5

GovernmentAdda.com | IBPS SBI SSC RBI RRB FCI RAILWAYS

45

Daily Visit

[GOVERNMENTADDA.COM]

6. As per the given arrangement which of the following combination represents only the people who are bowlers?. A. T, V B. Q, P, T C. V, W, R D. P, T E. W, T, S Answer Answer – E. W, T, S 7. As per the given arrangement which of the following person represent the one who was played in between the W and S? A. U B. P C. R D. V E. W Answer Answer – D. V 8. Who among the following is a bowler? A. P B. R C. S D. All of the Above E. Other than those given as options Answer Answer – C. S

GovernmentAdda.com | IBPS SBI SSC RBI RRB FCI RAILWAYS

46

Daily Visit

[GOVERNMENTADDA.COM]

9. Which of the following combinations is correct as per the given arrangement? A. Monday – P B. Tuesday – W C. Wednesday – Q D. Thursday – S E. Friday – T Answer Answer – A. Monday – P 10. Who amongst the following scored half century? A. P B. V C. U D. W E. T Answer Answer – A. P I. Study the following information carefully to answer the given questions

There are Seven Lecturers – A, B, C, D, E, F and G taught seven subjects,viz., Maths, Zoology, Botany, Chemistry, Physics, English and Statistics on one day in a week starting from Monday and ending on Sunday (of the same week). There will be separate timings for each lecture.

Note: Total hours taken by all the lecturers = 18 hours. The minimum and maximum timing of any lecture will be one hour and five hour respectively. There are two pairs of timings that can be followed by four lecturers. 

Chemistry is taught on Thursday. English is neither taught on Tuesday nor on Saturday. The Botany professor gave lecture immediately after the lecturer A. B is not a Chemistry Professor.

GovernmentAdda.com | IBPS SBI SSC RBI RRB FCI RAILWAYS

47

Daily Visit 

[GOVERNMENTADDA.COM]

Maths is taught for one hour. The Professor B gave his lecture on one of the days before Friday. Either the professor E or the professor F not gave his lecture on Sunday. Professor F gave his lecture immediately after E. Lecturer A spent more time than Lecturer C.



Time taken by lecturer C is the sum of time taken by the lecturers B and F. Subjects Maths & Zoology are taught for same duration. The lecturer who took maximum time is immediately preceded by the person who took less than one hour of maximum time.



The difference between the subjects taught on Friday and Sunday is equalled to the time taken by the lecturer A. Professor who gave maths lecture immediately preceded and followed by C and G respectively.



Professor who gave lecture on Sunday spent less than three hours. Only one lecture is held between Chemistry and Botany. Zoology is taught after two days of maths lecture. Statistics is neither taught on Monday nor Sunday.



Botany is not taught on the immediate next day on which Zoology is taught. Physics is taught on Monday.

Explanation Days

Subjects

Lecturers

Hours

Monday

Physics

A

3

Tuesday

Botany

C

2

Wednesday

Maths

B

1

Thursday

Chemistry

G

4

Friday

Statistics

E

5

Saturday

Zoology

F

1

Sunday

English

D

2

1. Which of the following Subject is taught by A ? A. Statistics B. Chemistry C. Zoology D. Physics E. English Answer Answer – D. Physics

GovernmentAdda.com | IBPS SBI SSC RBI RRB FCI RAILWAYS

48

Daily Visit

[GOVERNMENTADDA.COM]

2. Which of the following combinations is True with respect to the given arrangement? A. Maths – Wednesday B. Chemistry – Friday C. Zoology – Wednesday D. Physics – Friday E. English – Sunday Answer Answer – E. English – Sunday 3. If all the persons are made to arrange in alphabetical order from Monday to Sunday, positions of how many persons will remain unchanged? A. Four B. None C. Two D. One E. Three Answer Answer – E. Three 4. Professor D gave lecture for how many hours? A. One hour B. Two hours C. None of the given options is true. D. Three hours E. Four hours Answer Answer – B. Two hours 5. Who among the following gave lecture immediately after F? A. B GovernmentAdda.com | IBPS SBI SSC RBI RRB FCI RAILWAYS

49

Daily Visit

[GOVERNMENTADDA.COM]

B. E C. A D. D E. F Answer Answer – D. D II. Study the following information carefully to answer the given questions.

Seven persons – A, B, C, D, E, F and G live on seven different floors of a building but not necessarily in same order. Lowermost floor of building is numbered 1, one above that is numbered 2 and so on till top most floor is numbered 7. Each one of them also own different brands of Car, namely Fiat, Hyundai, Honda, Skoda, Toyoto, Ford and Chevrolet.(but not necessarily in same order). All of them went to tour on seven different days starting from Monday to Sunday(of the same week) 

A lives an odd numbered floor but not on floor numbered Three. Only two persons live between D and the one who owns Toyoto. The one who owns Honda lives immediately above C. The one who owns Toyoto went to tour on one of the days before Thursday.



The one who owns Hyundai lives on odd numbered floors above D. B lives on the floors above E. Only three persons live between C and the one who owns Hyundai. The person who went to tour on Friday owns Fiat Car.



The one who owns Fiat lives immediately above the one who owns Chevrolet. G lives on an odd numbered floor. E does not own Honda. D went to tour on Sunday



Only one person lives between B and E. The one who owns Toyoto lives immediately above A. Neither C nor A owns Skoda. The one who went to tour on Wednesday immediately after the one who owns Hyundai.



The one who owns Honda went to tour immediately before the one who lives in floor no 3. There are two persons live between the floor no 4 and the person who went to tour on Wednesday. The one who went to tour on Wednesday is not C.

Explanation Floor No

Person

Car

Days

7

G

Skoda

Wednesday

GovernmentAdda.com | IBPS SBI SSC RBI RRB FCI RAILWAYS

50

Daily Visit

[GOVERNMENTADDA.COM]

6

B

Toyoto

Monday

5

A

Hyundai

Tuesday

4

E

Fiat

Friday

3

D

Chevrolet

Sunday

2

F

Honda

Saturday

1

C

Ford

Thursday

6. Which of the following car does A own? A. Skoda B. Toyoto C. Fiat D. Hyundai E. Ford Answer Answer – D. Hyundai 7. Which of the following combinations is True with respect to the given arrangement? A. Ford – C B. Hyundai – F C. Chevrolet – G D. Fiat – D E. Ford – B Answer Answer – A. Ford – C 8. If all the persons are made to sit in alphabetical order from top to bottom, positions of how many persons will remain unchanged? A. Four B. None C. Two D. One E. Three GovernmentAdda.com | IBPS SBI SSC RBI RRB FCI RAILWAYS

51

Daily Visit

[GOVERNMENTADDA.COM]

Answer Answer – C. Two 9. Which of the following statements is True with respect to the given arrangement? A. The one who owns Honda went to tour on the day immediately after E. B. F likes Fiat C. None of the given options is true. D. Only four persons live between D and G E. G lives immediately below E. Answer Answer – A. The one who owns Honda went to tour on the day immediately after E. 10. Who among the following lives on the floor numbered 2? A. B B. The one who owns Fiat C. The one who owns Hyundai D. D E. F Answer Answer – E. F I. Study the following information carefully to answer the given questions.

Eight friends namely P, Q, R, S, T, U, V and W live on eight different floors of a building but not necessarily in the same order. The ground floor is numbered 1 and the one above that is numbered 2 and so on till the topmost floor is numbered 8. Each one of them owns a different car, namely Santro, Brio, Amaze, Civic, Etios, Celerio, Micra and Fabia (but not necessarily in the same order). They went to tour on eight different months viz, February, April, June, July, August, September, October, December. 

S lives on an even numbered floor. Only three people live between S and T. Only one person lives between T and V. V lives on one of the floors below T. Only two persons live between V and U. GovernmentAdda.com | IBPS SBI SSC RBI RRB FCI RAILWAYS

52

Daily Visit 

[GOVERNMENTADDA.COM]

W lives on a floor that is immediately below U. Only two persons live between T and Q. P does not live on the lowermost floor. The person who owns civic car lives on the floor numbered 7. W owns Amaze. The person who owns Brio lived on the topmost floor went to tour after August.



The persons who lived on odd numbered floors went to tour on the months which has less than 31 days. The person who went to tour on February lived on one of the floors above the 4th floor.



The person who lived on the 5th floor went to tour on one of the months after April. The person who lived on the lowermost floor went to tour on one of the months after June and he owns Santro.



The person who owns Celerio car lived immediately above the 4th floor. The person who went to tour on July owns Amaze. U owns Fabia went to tour on the month before June. The person who owns Micra went to tour on the month after July. V owns Etios went to tour after October

Explanation Floor No

Person

Car

Months

8

T

Brio

October

7

P

Civic

February

6

V

Etios

December

5

Q

Celerio

June

4

S

Micra

August

3

U

Fabia

April

2

W

Amaze

July

1

R

Santro

September

1. Which of the following Statements is true with respect to the given information? 1. T lives immediately above the one who owns Civic 2. U lives immediately above R 3. Only three people live between U and the one who owns Brio. 4. W owns Etios 5. All the given statements are true. Answer Answer – 1. T lives immediately above the one who owns Civic 2. Who amongst the following lives exactly between V and the one who owns Brio? 1. T GovernmentAdda.com | IBPS SBI SSC RBI RRB FCI RAILWAYS

53

Daily Visit

[GOVERNMENTADDA.COM]

2. P 3. W 4. U 5. R Answer Answer – 2. P 3. Four of the following cars does W own? 1. Civic 2. Celerio 3. Brio 4. Amaze 5. Fabia Answer Answer – 4. Amaze 4. Four of the following five are alike in a certain way and so form a group. Which one of the following does not belong to the group? 1. T – Civic 2. V – Celerio 3. V – Etios 4. S – Fabia 5. U – Amaze Answer Answer – 3. V – Etios 5. How many people live between P and the one who owns Amaze? 1. Four 2. Three 3. Two GovernmentAdda.com | IBPS SBI SSC RBI RRB FCI RAILWAYS

54

Daily Visit

[GOVERNMENTADDA.COM]

4. Five 5. None Answer Answer – 1. Four II. Study the following information carefully to answer the given questions.

Seven different food boxes P, Q, R, S, T, U and V contain seven different dishes namely viz., Mutton Briyani, Sandwich, Veg Fried Rice, Pizza, Burger, Egg Noodles and Chicken Fried Rice are arranged one above the other. The box at the bottom of arrangement is numbered 1, the above box is numbered 2 and so on. Box V does not contain Mutton Briyani. Q is immediately above T. The box which contains the Pizza is immediately below P. Only one box is between the box that contain Mutton Briyani and U. V is immediately above the box that contain Egg Noodles. More than two boxes are above the box that contains Mutton Briyani. Only two boxes are between the box that contain sandwich and the box that contains Chicken Fried Rice. Only two boxes are between the box which contains Pizza and the box which contains Mutton Briyani. The box filled with Veg Fried Rice is neither at the top nor at the bottom of the arrangement. Only one box is between Q and the box which contain Sandwich. Q is placed above the Sandwich box. R is placed immediately above U. Neither R nor V contains Pizza. Explanation Position 7 6 5 4 3 2 1

Box R U Q T V P S

Fruit Burger Chicken Fried Rice Veg Fried Rice Mutton Briyani Sandwich Egg Noodles Pizza

6. As per the given arrangement, S is related to Burger and U is related to Egg Noodles in a certain way. To which of the following is V related to the same way? A. Sandwich B. Egg Noodles C. Pizza GovernmentAdda.com | IBPS SBI SSC RBI RRB FCI RAILWAYS

55

Daily Visit

[GOVERNMENTADDA.COM]

D. Veg Fried Rice E. None of the Above Answer Answer – D. Veg Fried Rice 7. Which of the following pairs of people occupy the top, middle and bottom positions of the arrangement? A. R, P, T B. R, T, U C. P, T, V D.Q, U, S E. R, T, S Answer Answer – E. R, T, S 8. Which combination represents the position of R and the receipe? A. 5 – Sandwich B. 4 – Egg Noodles C. 1 – Mutton Briyani D. 4 – Pizza E. 7 – Burger Answer Answer – E. 7 – Burger 9. Which amongst the following arranged exactly in between U and T? A. P B. S C. Q D. T E. Other than those given as options Answer GovernmentAdda.com | IBPS SBI SSC RBI RRB FCI RAILWAYS

56

Daily Visit

[GOVERNMENTADDA.COM]

Answer – C. Q 10. Which of the following receipe is contained in Box “P”? A. Sandwich B. Egg Noodles C. Pizza D. Veg Fried Rice E. None of the Above Answer Answer – B. Egg Noodles Seven People namely A, B, C, D, E, F and G went to tour on seven different months of the same year namely February, March, April, June, September, October and November. Each of them also likes a different Chocolate namely 5Star, Dairy Milk, Munch, Kit-kat, Milky Bar, Luv it and Perk but not necessarily in the same order. F went to tour on the month which has more than 30 days. A went to tour immediately before the one who likes Dairy Milk. Only one person went to tour between F and the one who likes 5Star. Both G and C went to tour on one of the months after the one who likes 5Star. G went to tour immediately before C. The one who likes Munch went to tour on the month which has less than 30 days. Only three people went to tour between the one who likes Munch and the one who likes Perk. Only two people went to tour between G and the one who likes Kit-kat. D went to tour immediately after the one who likes Kit-kat. Only two people went to tour between D and E. C does not like Luv it. Explanation Month February March April June September October November

Person E F B D A G C

Chocolate Munch Luv it Kit-kat 5Star Perk Dairy Milk Milky Bar

1. Which of the following represents the month in which G went to tour? A. October GovernmentAdda.com | IBPS SBI SSC RBI RRB FCI RAILWAYS

57

Daily Visit

[GOVERNMENTADDA.COM]

B. March C. April D. September E. Can not be determined Answer Answer – A. October 2. Which of the following does C like? A. 5Star B. Kit-kat C. Dairy Milk D. Munch E. Milky bar Answer Answer – E. Milky bar 3. As per the given arrangement, Munch is related to April and Kit-kat is related to September following a certain pattern, which of the following is Perk related to following the same pattern? A. February B. June C. October D. November E. March Answer Answer – D. November 4. Which of the following represents the people who went to tour April and November respectively? A. A,B B. C,D C. E,F GovernmentAdda.com | IBPS SBI SSC RBI RRB FCI RAILWAYS

58

Daily Visit

[GOVERNMENTADDA.COM]

D. B,C E. B,G Answer Answer – D. B,C 5. How many people went to tour between E and A ? A. None B. One C. Three D. Two E. More than three Answer Answer – C. Three II. Study the following information carefully to answer the given questions.

I. Study the following information carefully to answer the given questions.

Seven people, namely A,B,C,D,E,F and G like seven different flowers namely Rose, Jasmine, Lily, Sun flower, Orchid, Marigold and Daffodil but not necessarily in the same order. Each people also works in the same office but at a different department on the basis of experience namely Administration (ADMIN), Marketing & Sales, (M&S), Accounts (ACC), Production (PO), Quality Management (QM), Human Resources (HR), and Public Relations (PR), but not necessarily in the same order.

Note : Each person has been allocated to a department as per increasing order of experience with the one in ADMIN being the least experienced whilst the one in PR Being the most experienced.

G likes Daffodil and has more experience than the one who likes Rose. Only one person has more experience than A. Only one person has less experience than F. B does not work in QM. The one who has less experience than F likes Sun flower. The one in HR likes Orchid. D has less experience than the one in PO, but more experience than the one who

GovernmentAdda.com | IBPS SBI SSC RBI RRB FCI RAILWAYS

59

Daily Visit

[GOVERNMENTADDA.COM]

likes Lily. E neither has the least experience than the one who likes Lily nor he works in QM. The one who likes Jasmine does not work in PO. Only two people have more experience than the one who likes Rose. Explanation Person G A C E D F B

Job PR HR QM PO ACC M&S ADMIN

Flower Daffodil Orchid Rose Mari gold Jasmine Lily Sun flower

6. As per the given arrangement, ADMIN is related to Lily and PO is related to Rose in a certain way. To which of the following is ACC related to the same way? A. Jasmine B. Lily C. Sun flower D. Orchid E. Marigold Answer Answer – E. Marigold 7. Which of the following pairs of people who have more experience than C less experience than E? A. F, G B. F, B C. G, A D. A, D E. None of the Above Answer Answer – D. A, D

GovernmentAdda.com | IBPS SBI SSC RBI RRB FCI RAILWAYS

60

Daily Visit

[GOVERNMENTADDA.COM]

8. Which combination represents the department that C works in and the flower he likes? A. QM – Rose B. PO – Lily C. PO – Marigold D. ACC – Sunflower E. ADMIN – Sunflower Answer Answer – A. QM – Rose 9. Who amongst the following works in ADMIN? A. A B. E C. G D. B E. Other than those given as options Answer Answer – D. B 10. Which of the following flowers does D like? A. Lily B. Marigold C. Rose D. Sunflower E. Jasmine Answer Answer – E. Jasmine I. Study the following information carefully to answer the given questions.

GovernmentAdda.com | IBPS SBI SSC RBI RRB FCI RAILWAYS

61

Daily Visit

[GOVERNMENTADDA.COM]

Seven different Mobile shops – A, B, C, D, E, F and G sold Mobiles starting from Monday to Sunday (of the same week) not necessarily in same order. The number of “Apple Phones” sold by the Seven shops in seven different days are 6, 13, 10, 12, 20, 15 and 27 (not necessarily in same order).

The shop F sold Mobiles on one of the days after the shop which sold 6 Mobiles. The Shop ‘A’ sold on one of the days after Friday. On Wednesday, the number of Mobiles sold are 12. The difference between the number of Mobiles sold on Monday and Friday is the multiple of the number 7. There are two shops sold mobiles between the shop F and the shop which one sold 6 Mobiles. Shop B sold Mobiles on one of the days immediately before the shop that sold 10 Mobiles. There are two shops sold mobiles between B and G. There are three shops sold mobiles between the shop F and D. C not sold the least number of Mobiles. The sum of Mobiles sold on Wednesday and Saturday is more than ten and the sum equals to the number of Mobiles sold on Friday. The difference between the number of Mobiles sold by Shop B and G is less than five. The Shop which sold 10 Mobiles not on Friday. The shop which sold more than 12 Mobiles(not an odd number) is immediately after the one which sold 12 Mobiles. Shop F sold more number of Mobiles than Shop D. Explanation Days

Shops

Number of Mobiles

Monday

D

13

Tuesday

E

6

Wednesday

G

12

Thursday

C

20

Friday

F

27

Saturday

B

15

Sunday

A

10

1. Which of the following is sold by Shop A? A. 20 B. 15 C. 27 D. 10 E. 13 Answer Answer – D. 10 GovernmentAdda.com | IBPS SBI SSC RBI RRB FCI RAILWAYS

62

Daily Visit

[GOVERNMENTADDA.COM]

2. Which of the following combinations of “Shop – Day – Number of Mobiles” is True with respect to the given arrangement? A. E – Friday – 15 B. A – Saturday – 6 C. C – Thursday – 20 D. B – Friday – 13 E. E – Thursday – 10 Answer Answer – C. C – Thursday – 20 3. Which of the following shop sold Mobiles immediately after B? A. D B. E C. F D. A E. None Answer Answer – D. A 4. In this arrangement, A is related to Monday, B is related to Tuesday then F is related to? A. Tuesday B. Wednesday C. None of the given options is true. D. Saturday E. Sunday Answer Answer – B. Wednesday 5. Which among the following shop sold “Apple Phones” on Tuesday? A. B GovernmentAdda.com | IBPS SBI SSC RBI RRB FCI RAILWAYS

63

Daily Visit

[GOVERNMENTADDA.COM]

B. C C. A D. E E. F Answer Answer – D. E II. Study the following information carefully to answer the given questions

Seven Persons – A, B, C, D, E, F, and G – live on separate floors of a seven storey-ed building, but not in the same order. The ground floor of the building is numbered 1, the floor above it 2 and so on until the topmost floor is numbered 7. Each person likes different fruits – Apple, Banana, Orange, Guava, Pomegranate, Grapes and Pineapple, but not necessarily in the same order.

The person who likes Guava lives on floor numbered four. A does not live on the lowermost floor. A lives on any odd numbered floor below the one who likes Guava. Only two persons live between A and the person who likes Pineapple. Only one person lives between B and F. F lives on an even numbered floor and does not like Guava. Only three persons live between the persons who like Apple and Orange respectively. The person who likes Apple live on any floor above the B’s floor. The person who likes Apple does not live on the topmost floor. G lives on an even numbered floor but neither immediately above nor immediately below the floor of A. C does not like Apple or Orange. Only two persons live between D and the one who likes Guava. The person who likes Pomegranate lives on the floor immediately above the floor of the person who likes Grapes. Explanation Floor No 7 6 5 4 3 2 1

Person C G E B A F D

Fruits Banana Pineapple Apple Guava Pomegranate Grapes Orange

GovernmentAdda.com | IBPS SBI SSC RBI RRB FCI RAILWAYS

64

Daily Visit

[GOVERNMENTADDA.COM]

6. How many persons live between the floors on which G and D live? A. Three B. Two C. Four D. Five E. No one Answer Answer – C. Four 7. Which of the following statements is/are true according to the given information? A. E lives on floor numbered 5 and he does not like Guava B. A likes Pomegranate and he does not live on floor numbered 4 C. C likes Banana and he lives on the topmost floor D. Only two persons live between the floors of E and F E. All the statements are true. Answer Answer – E. All the statements are true. 8. Who among the following lives on the floor immediately above the floor of A? A. B B. F C. G D. C E. No one Answer Answer – A. B 9. Who among the following lives exactly between the floors on which G and B live? A. F B. E GovernmentAdda.com | IBPS SBI SSC RBI RRB FCI RAILWAYS

65

Daily Visit

[GOVERNMENTADDA.COM]

C. D D. C E. No one Answer Answer – B. E 10. Who among the following does like Orange? A. A B. D C. B D. C E. No one Answer Answer – B. D I. Study the following information carefully to answer the given questions Eight People – L, M, N, O, P, Q, R and S live in eight different floors of building (but not necessarily in the same order). The lowermost floor of the building is numbered one, the one above that is numbered two, and so on till the topmost floor is numbered eight. Each one of them also owns a different brands of bikes, namely Hero, Honda, TVS, Yamaha, Royal Enfield, Bajaj, Harley Davidson and KTM (but not necessarily in the same order).

Only one person lives between M and the one who owns Royal Enfield. Q lives an odd numbered floor above the floor numbered four. Only three people live between O and the one who owns Bajaj. N lives an one of the odd numbered floors above the one who owns Bajaj. The one who owns Harley Davidson lives immediately above R, R owns neither Bajaj nor Hero. P does not own Harley Davidson. Only three people live between R and L. The one who owns KTM lives immediately above the one who owns Honda, but not on the topmost floor. Only one person lives between Q and the one who owns TVS. The number of people leaving above Q is same as the number of people living between Q and O. Only one person lives between the one who owns KTM and S. Only two people live between N and the one who owns Hero. Explanation GovernmentAdda.com | IBPS SBI SSC RBI RRB FCI RAILWAYS

66

Daily Visit

[GOVERNMENTADDA.COM]

Floor No 8 7 6 5 4 3 2 1

Person M N R Q S P L O

Bike Yamaha Harley Davidson Royal Enfield Bajaj Hero TVS KTM Honda

1. Which of the following Statements is true with respect to the given information? A. R lives immediately above the one who owns Bajaj B. P lives immediately above N C. Only three people live between Q and the one who owns KTM. D. O owns Royal Enfield bike. E. All the given statements are true. Answer Answer – 1. R lives immediately above the one who owns Bajaj 2. Who amongst the following lives exactly between S and the one who owns Honda bike? A. M, N B. R, O C. Q, R D. L, M E. P, L Answer Answer – E. P, L 3. Which of the following bikes does P own? A. KTM B. Honda C. TVS

GovernmentAdda.com | IBPS SBI SSC RBI RRB FCI RAILWAYS

67

Daily Visit

[GOVERNMENTADDA.COM]

D. Royal Enfield E. Harley Davidson Answer Answer – C. TVS 4. Four of the following five are alike in a certain way and so form a group. Which one of the following does not belong to the group? A. R – Yamaha B. N – Bajaj C. Q – Royal Enfield D. S – KTM E. O – TVS Answer Answer – C. Q – Royal Enfield 5. How many people live between N and the one who owns TVS? A. Four B. Three C. Two D. Five E. None Answer Answer – B. Three II. Study the following information carefully to answer the given questions

P, Q, R, S, T, U, V and W live on eight different floors of a building but not necessarily in the same order. The ground floor is numbered one and the floor above it is numbered two and so on. The top most floor is numbered eight. Each of them likes a different colour viz Green, Yellow, Black, Blue, Pink, White, Brown and Orange but not necessarily in the same order. GovernmentAdda.com | IBPS SBI SSC RBI RRB FCI RAILWAYS

68

Daily Visit

[GOVERNMENTADDA.COM]

The one who likes pink does not live on first floor. The one who likes Yellow lives on an even numbered floor but not on floor number 8. Only two persons live between U and the one who likes Yellow. Only one person live between U and the one who likes Pink. Neither T nor R lives on first floor. Only one person lives between R and S, who likes Black. P lives immediately above U, who lives on an odd numbered floor. Only two persons live between T and P. Q lives on an even numbered floor and immediately above R. The one who likes blue lives on an even numbered floor and live immediately above the person who likes orange. R does not like Pink or Orange. Only two persons live between W and the one who likes Green. Q lives on floor number four. The one who likes White does not live on odd numbered floor. W does not like Brown. Explanation Floor

Person

Colour

8

P

Blue

7

U

Orange

6

W

White

5

T

Pink

4

Q

Yellow

3

R

Green

2

V

Brown

1

S

Black

6. Who among the following live on floor 5 ? A. S B. R C. V D. T E. None of these Answer Answer – D. T 7. Which of the following colours does P like ? A. Blue B. Orange C. Yellow GovernmentAdda.com | IBPS SBI SSC RBI RRB FCI RAILWAYS

69

Daily Visit

[GOVERNMENTADDA.COM]

D. Brown E. None of these Answer Answer – A. Blue 8. Who among the following likes White? A. Q B. R C. W D. S E.None of these Answer Answer – C. W 9. Who among the following lives between P and W ? A. P B. R C. S D. U E.None of these Answer Answer – D. U 10. Which of the following is correct ? A. 4-R-Green B. 7-W-Blue C. 5-T-Orange D. 3-Q-Yellow E. None of these Answer GovernmentAdda.com | IBPS SBI SSC RBI RRB FCI RAILWAYS

70

Daily Visit

[GOVERNMENTADDA.COM]

Answer – E. None of these I. Study the following information carefully to answer the given questions.

Seven Persons – A, B, C, D, E, F, and G – live on separate floors of a seven storey-ed building, but not in the same order. The ground floor of the building is numbered 1, the floor above it 2 and so on until the topmost floor is numbered 7. Each person likes different vegetables – Cabbage, Potato, Tomato, Onion, Carrot, Radish and Bean, but not necessarily in the same order. Each person has 7 different weight of their favorite vegetables starting from 1kg to 10 kg. The weight of Onion is more than 2 kg. The total weight of Carrot and Radish is 10 kg. The person who likes Onion lives on floor numbered four. A does not live on the lowermost floor. A lives on any odd numbered floor below the one who likes Onion. Only two persons live between A and the person who likes Bean. Only one person lives between B and F. The total weight of Cabbage is square of the total weight of Carrot while The total weight of Bean is square of the total weight of Onion. F lives on an even numbered floor and does not like Onion. Only three persons live between the persons who like Cabbage and Tomato respectively. The person who likes Cabbage live on any floor above the B’s floor. The person who likes Cabbage does not live on the topmost floor. G lives on an even numbered floor but neither immediately above nor immediately below the floor of A. C does not like Cabbage or Tomato. Only two persons live between D and the one who likes Onion. The person who likes Carrot lives on the floor immediately above the floor of the person who likes Raddish. The difference between the weight of the Tomato and Radish is 2 kg. The floor number and the weight of favorite vegetable is same for the person C. Explanation Floor No 7 6 5 4 3 2 1

Person C G E B A F D

Vegetables Potato Bean Cabbage Onion Carrot Raddish Tomato

weight 7 kg 9 kg 4 kg 3 kg 2 kg 8 kg 6 kg

1. D has how many kg of favourite vegetable? A. 2 kg B. 8 kg C. 6 kg GovernmentAdda.com | IBPS SBI SSC RBI RRB FCI RAILWAYS

71

Daily Visit

[GOVERNMENTADDA.COM]

D. 4 kg E. No one Answer Answer – C. 6 kg 2. Which of the following statements is/are true according to the given information? A. E lives on floor numbered 5 and he does not like Onion B. A likes Carrot and he does not live on floor numbered 4 C. C likes Potato and he does not have 6 kg. D. Only two persons live between the floors of E and F E. All the statements are true. Answer Answer – E. All the statements are true. 3. Who among the following lives on the floor immediately above the floor of A? A. B B. F C. G D. C E. No one Answer Answer – A. B 4. Who among the following lives exactly between the floors on which B and F live? A. F B. A C. D D. C E. No one Answer GovernmentAdda.com | IBPS SBI SSC RBI RRB FCI RAILWAYS

72

Daily Visit

[GOVERNMENTADDA.COM]

Answer – B. A 5. Who among the following does like Carrot? A. A B. D C. B D. C E. No one Answer Answer – A. A II. Study the following information carefully to answer the given questions

P, Q, R, S, T, U, V and W live on eight different floors of a building but not necessarily in the same order. The ground floor is numbered one and the floor above it is numbered two and so on. The top most floor is numbered eight. Each of them likes a different news paper viz The Times of India, The Economic Times, Business Standard, THE HINDU, The Indian Express, The Financial Express, Deccan Chronicle and THE TRIBUNE but not necessarily in the same order.

P lives immediately above U, who lives on an odd numbered floor. The one who likes The Indian Express does not live on first floor. The one who likes The Economic Times lives on an even numbered floor but not on floor number 8. Only two persons live between U and the one who likes The Economic Times. Only one person lives between U and the one who likes The Indian Express. Neither T nor R lives on first floor. Only one person lives between R and S, who likes Business Standard. Only two persons live between T and P. Q lives on an even numbered floor and immediately above R. The one who likes THE HINDU lives on an even numbered floor and live immediately above the person who likes THE TRIBUNE. R does not like The Indian Express or THE TRIBUNE. Only two persons live between W and the one who likes The Times of India. Q lives on floor number four. The one who likes The Financial Express does not live on odd numbered floor. W does not like Deccan Chronicle. Explanation Floor

Person

News paper

8

P

THE HINDU

7

U

THE TRIBUNE GovernmentAdda.com | IBPS SBI SSC RBI RRB FCI RAILWAYS

73

Daily Visit

[GOVERNMENTADDA.COM]

6

W

The Financial Express

5

T

The Indian Express

4

Q

The Economic Times

3

R

The Times of India

2

V

Deccan Chronicle

1

S

Business Standard

6. Who among the following lives on floor 5? A. S B. R C. V D. T E. None of these Answer Answer –D. T 7. Which of the following News papers does P like? A. THE HINDU B. THE TRIBUNE C. The Economic Times D. Deccan Chronicle E. None of these Answer Answer – A. THE HINDU 8. Who among the following likes The Financial Express News paper? A. Q B. R C. W D. S E.None of these

GovernmentAdda.com | IBPS SBI SSC RBI RRB FCI RAILWAYS

74

Daily Visit

[GOVERNMENTADDA.COM]

Answer Answer –C. W 9. Who among the following lives between P and W? A. P B. R C. S D. U E.None of these Answer Answer –D. U 10. Which of the following is true as per the arrangement? A. 4-R-The Times of India B. 7-W-THE HINDU C. 5-T-THE TRIBUNE D. 3-Q-The Economic Times E. None of these Answer Answer – E. None of these I. Study the following information carefully to answer the given questions

Ten students namely viz A, B, C, D, E, F, G, H, I and J of ten different colleges but not necessarily in the same order have exam on five different days starting from Monday to Friday of the same week. Each student have exam at two different time slots, i.e 08.00 AM or 11.00 A.M Only two people have exam between F and J. Neither E nor G does not have exam on Friday. I has exam on Tuesday at 08.00 A.M. H does not have exam at 11.00 AM. The number of people who have exam between G and D is same as the number of people who have exam between C and H. D does not have exam on any one of the days after E. F does not have exam on any of the days after H. B has exam immediately before I. I does not have exam on any of the days

GovernmentAdda.com | IBPS SBI SSC RBI RRB FCI RAILWAYS

75

Daily Visit

[GOVERNMENTADDA.COM]

before G. The one who has exam at 08.00 A.M. immediately before J. D has exam immediately after the day of one who has exam on Monday. F does not have exam at 11.00 A.M. Only three people have exam between G and E. Explanation Person G B I D E C F A H J

Day Mon Mon Tue Tue Wed Wed Thu Thu Fri Fri

Time Slot 08.00 AM 11.00 AM 08.00 AM 11.00 AM 08.00 AM 11.00 AM 08.00 AM 11.00 AM 08.00 AM 11.00 AM

1. How many persons have exam at 11’0 clock between E and H? A. 5 B. 6 C. 2 D. 4 E. None of these. Answer Answer – C. 2 2. Who among the following person has exam at 8 A.M? A. J B. H C. A D. C E. D Answer

GovernmentAdda.com | IBPS SBI SSC RBI RRB FCI RAILWAYS

76

Daily Visit

[GOVERNMENTADDA.COM]

Answer – B. H 3. Four among the following form a group in a certain way. Which of the following does not belong to Group? A. B – Tuesday B. D – Wednesday C. G – Tuesday D. A – Friday E. H – Friday Answer Answer – E. H – Friday 4. Which of the following is correctly matched? A. I – Monday B. D – Tuesday C. B – Friday D. G – Tuesday E. I – Wednesday Answer Answer – B. D – Tuesday 5. Who among the following have exam on Friday? A. A, B B. C, D C. E, G D. H, J E. G, I Answer Answer – D. H, J II. Study the following information carefully to answer the given questions. GovernmentAdda.com | IBPS SBI SSC RBI RRB FCI RAILWAYS

77

Daily Visit

[GOVERNMENTADDA.COM]

Eight boxes namely A, B, C, D, E, F, G and H are placed from top to bottom not in the same order. They contain different types of flowers namely viz Jasmine, Rose, Lily, Lotus, Sunflower, Tulip, Orchid and Chrysanthemum. Boxes are made up of different materials among steel, plastic, wood and Aluminum box. Exactly two boxes are made of same material. Consider the box kept at top as 1st position.

Box D which is made up of plastic kept two places above B and both are in the top 4 positions when boxes are arranged from top to bottom. The box containing Lotus is kept immediately below wooden box and made up of same material as D. Box C is kept somewhere between G and H and H being below C. The two Aluminum boxes are kept vertically adjacent to each other. Lily is kept exactly middle between F and the box containing Tulip. H doesn’t contain Jasmine. Box E kept two places below box G which is kept immediately below the box containing Lotus. The box containing Chrysanthemum is placed at even numbered place but is not placed at the bottom. The steel box which kept at the top either contains Jasmine or Orchid. Box E, not made up of Aluminum. F which contains Sunflower is made up of wood. The box containing Rose is made up of wood. C doesn’t contain Jasmine or Chrysanthemum. Explanation Position

Box

Flower

Box Type

1

A

Jasmine

Steel

2

D

Chrysanthemum

Plastic

3

F

Sunflower

Wood

4

B

Lotus

Plastic

5

G

Lily

Aluminium

6

C

Orchid

Aluminium

7

E

Tulip

Steel

8

H

Rose

Wood

6. Which of the following box contains Rose? A. A B. D C. F D. H E. None of these. Answer D. H GovernmentAdda.com | IBPS SBI SSC RBI RRB FCI RAILWAYS

78

Daily Visit

[GOVERNMENTADDA.COM]

7. Which of the following box is made up of wood? A. A, B B. E, F C. B, E D. D, G E. H, F Answer E. H, F 8. Which of the following statements is true? A. B which contains Lotus is made up of wood B. G which is placed at 5th position is made up of plastic material C. E which contains Tulip is not made up of steel D. H is placed at bottom and is made up of steel. E. None of these Answer E. None of these 9. Which of the following is correctly matched? A. G – Lily – Plastic B. F – Sunflower – Wood C. H – Rose – Steel D. E – Tulip – Plastic E. D – Chrysanthemum – Wood Answer B. F – Sunflower – Wood 10. What does box G contain? A.Orchid B.Tulip GovernmentAdda.com | IBPS SBI SSC RBI RRB FCI RAILWAYS

79

Daily Visit

[GOVERNMENTADDA.COM]

C.Lily D.Sand Wich E.Chrysanthemum Answer C.Lily I. Study the following information carefully to answer the given questions

A, B, C, D, E, F and G are seven people live on seven different floors of a building but not necessarily in the same order. The lowermost floor of the building is numbered 1, the one above that is numbered 2 and so on till the topmost floor is numbered 7. Each person likes different vegetables namely Potato, Brinjal, Carrot, Tomato, Drumstick, Cabbage, and Cauli flower.

Three persons live between the one who likes Potato and the one who likes Brinjal. The one who likes Potato lives one of the floors above the floor numbered 4. Three persons live between the one who likes Drumstick and the one who likes Cauli flower. The person who lives on the ground floor like neither Carrot nor Brinjal. The one who likes Carrot lives on one of the floors below the floor numbered 4. The person who likes Cabbage does not live on one of the floors below the floor numbered 4. The person who likes Potato and Drumstick live on one of the floors above the floor numbered 4. The person who lives on the top most floor does not like Potato. Three persons live between the one who likes Cabbage and the one who likes Carrot. A lives between B and E. B lives on one of the floors above E. A lives on an odd numbered floor but not on the floor numbered three. Only one person lives between A and G. F lives immediately below the one who likes Carrot. Only two persons live between E and C. C does not live on the top most floor. G does not like Carrot. Explanation Floor

Persons

Vegetables

7

G

Cabbage

6

B

Potato

5

A

Drumstick

4

E

Tomato

3

D

Carrot

2

F

Brinjal

1

C

Cauli flower

GovernmentAdda.com | IBPS SBI SSC RBI RRB FCI RAILWAYS

80

Daily Visit

[GOVERNMENTADDA.COM]

1. Which among the following is liked by B? A. Tomato B. Carrot C. Potato D. Cauli Flower E. None of these Answer C. Potato 2.

Which of the following combination is true as per the given arrangement? A. A – 5 – Drumstick B. C – 3 – Tomato C. B – 5 – Carrot D. F – 2 – Cauli flower E. None of these. Answer A. A – 5 – Drumstick

3.

Who among the following lives in floor no 3? A. A B. C C. D D. F E. None of these. Answer C. D

4. Four among the following form a group in a certain way. Which of the following does not belong to Group? A. G – Cauli Flower B. A – Carrot GovernmentAdda.com | IBPS SBI SSC RBI RRB FCI RAILWAYS

81

Daily Visit

[GOVERNMENTADDA.COM]

C. B – Brinjal D. F – Potato E. D – Tomato Answer E. D – Tomato 5.

Who among the following likes Cauli flower? A. A B. C C. D D. F E. None of these. Answer B. C

II. Study the following information carefully to answer the given questions

Eight people P, Q, R, S, T, U, V, and W were born in three different months(of the same year) but not necessarily in the same order, namely March, June and December such that not less than two people and not more than three people were born in a month. Each of them also likes different watches namely Fastrack, Titan, Fossil, Giordano, Timex, Casio, Maxima and Steve Madden but not necessarily in the same order. Each goes to eight different places namely Agra, Delhi, Mysuru, Hyderabad, Chennai, Bengaluru, Kochi and Mumbai but not in necessarily same order

R likes Steve Madden and was born in the same month as T. Only Q and W were born in March. R was not born in December. The one who likes Timex was born in the month which has 30 days only. U was not born in the same month as T. S likes Giordano and born in the same month as U. V does not like Timex. The one who likes Maxima and the one who likes Fossil were born in the same month, The one who likes Maxima was not born in the same month as W. U does not like Maxima. The one who likes Fastrack was born in the same month as P. Q does not like Titan. T does not like Timex. Three people are there between the one who goes to Chennai on one of the months which has more than 30 days and the one who goes to Bengaluru on one of the months which has less than 31 days. Three people are there between the one who goes to Kochi on one of the months which has more than 30 days and the one who goes to Delhi GovernmentAdda.com | IBPS SBI SSC RBI RRB FCI RAILWAYS

82

Daily Visit

[GOVERNMENTADDA.COM]

on one of the months which has more than 30 days. Three people are there between the one who goes to Hyderabad on one of the months which has less than 31 days and the one who goes to Mysuru on one of the months which has more than 30 days. V goes to neither Mumbai nor Chennai. W does not go to Delhi. Explanation Persons

Months

Watches

University

Q

March

Casio

Chennai

W

March

Titan

Kochi

R

June

Steve Madden

Hyderabad

T

June

Fastrack

Mumbai

P

June

Timex

Bengaluru

S

December

Giordano

Delhi

U

December

Fossil

Mysuru

V

December

Maxima

Agra

6. As per the given arrangement which of the following combination represents only the people who were born in December?. A. T, V B. U, P, T C. V, U D. P, T E. U, V, S Answer Answer – E. U, V, S 7. As per the given arrangement which of the following person represent the one who was born in the same month as the one who likes Casio? A. U B. P C. R D. T E. W

GovernmentAdda.com | IBPS SBI SSC RBI RRB FCI RAILWAYS

83

Daily Visit

[GOVERNMENTADDA.COM]

Answer Answer – E. W 8. Which of the following Watches does T like as per the given arrangement? A. Casio B. Titan C. Fastrack D. Fossil E. Other than those given as options Answer Answer – C. Fastrack 9. Which of the following combinations is correct as per the given arrangement? A. December – Titan – Bengaluru B. June – Casio – Chennai C. June – Fossil – Agra D. March – Fastrack – Hyderabad E. December – Fossil – Mysuru Answer Answer – E. December – Fossil – Mysuru 10. Who amongst the following goes to Kochi as per the given arrangement? A. P B. V C. U D. W E. T Answer Answer – D. W GovernmentAdda.com | IBPS SBI SSC RBI RRB FCI RAILWAYS

84

Daily Visit

[GOVERNMENTADDA.COM]

I. Study the following information carefully to answer the given questions

A, B, C, D, E, F and G are seven people live on seven different floors of a building but not necessarily in the same order. The lowermost floor of the building is numbered 1, the one above that is numbered 2 and so on till the topmost floor is numbered 7. Each one of them earns different amount per month. i.e. 20000, 15000, 10000, 25000,30000, 35000 and 40000. (But not necessarily in the same order.). Each person likes different e-commerce websites namely Myntra, Shopclues, Jabong, Snapdeal, Flipkart, Amazon, and E-bay.

Three persons live between the one who likes Myntra and the one who likes Shopclues. The one who likes Myntra lives one of the floors above the floor numbered 4. Three persons live between the one who likes Flipkart and the one who likes Ebay. The person who lives on the ground floor does not like neither Jabong nor Shopclues. The one who likes Jabong lives on one of the floors below the floor numbered 4. The person who likes Amazon does not live on one of the floors below the floor numbered 4. The person who likes Myntra and Flipkart live on one of the floors above the floor numbered 4. The person who lives on the top most floor does not like Myntra. The one who earns Rs.20000 lives immediately above the one who earns Rs.40000. The one who earns Rs.15000 lives on one of the odd numbered floors above D. Only three people live between C and the one who earns 15000. The one who earns Rs.10000 lives immediately above C. F earns 10000. Three persons live between the one who likes Amazon and the one who likes Jabong. Only one person lives between B and E. B lives on one of the floors above E. Neither C nor A earns Rs.25000. E does not earn Rs. 10000. A lives on an odd numbered floor but not on the floor numbered three. The one who earns Rs.30000 lives immediately above A. Only two people live between A and the one who earns Rs.10000. Explanation Floor

Persons

Salary

E-Commerce

7

G

25000

Amazon

6

B

30000

Myntra

5

A

15000

Flipkart

4

E

20000

Snapdeal

3

D

40000

Jabong

2

F

10000

Shopclues

1

C

35000

Ebay

1. Which among the following is the salary of B? A. Rs. 20000 B. Rs. 40000 GovernmentAdda.com | IBPS SBI SSC RBI RRB FCI RAILWAYS

85

Daily Visit

[GOVERNMENTADDA.COM]

C. Rs. 35000 D. Rs. 30000 E. None of these Answer D. Rs. 30000 2.

Which of the following combination is true as per the given arrangement? A. A – 15000 – Flipkart B. C – 30000 – Snapdeal C. B – 35000 – Jabong D. F – 20000 – Ebay E. None of these. Answer A. A – 15000 – Flipkart

3.

Who among the following lives in floor no 3? A. A B. C C. D D. F E. None of these. Answer C. D

4. Four among the following form a group in a certain way.Which of the following does not belong to Group? A. G – 15000 B. A – 40000 C. B – 20000 D. F – 40000 E. E – 10000 GovernmentAdda.com | IBPS SBI SSC RBI RRB FCI RAILWAYS

86

Daily Visit

[GOVERNMENTADDA.COM]

Answer D. F – 40000 5.

Who among the following likes E-bay? A. A B. C C. D D. F E. None of these. Answer B. C

II. Study the following information carefully to answer the given questions

Eight people P, Q, R, S, T, U, V, and W were born in three different months(of the same year) but not necessarily in the same order, namely March, June and December such that not less than two people and not more than three people were born in a month. Each of them also likes a different fruit namely Guava, Peach, Banana, Cherry, Mango, Orange, Kiwi and apple but not necessarily in the same order. Each goes to Eight different Universities namely Indian Institute of Science, University of Delhi, Banaras Hindu University, Osmania University, IIT Madras, Shivaji University, Dr. APJ Abdul Kalam Technical University and University of Mumbai but not in necessarily same order

Only Q and W were born in March. R likes Apple and was born in the same month as T. R was not born in December. The one who likes Mango was born in the month which has 30 days only. U was not born in the same month as T. S likes Cherry and born in the same month as U. V does not like Mango. The one who likes Kiwi and the one who likes Banana were born in the same month, The one who likes Kiwi was not born in the same month as W. U does not like Kiwi. The one who likes Guava was born in the same month as P. Q does not like Peach. T does not like Mango. Three people are there between the one who goes to IIT Madras on one of the months which has more than 30 days and the one who goes to Shivaji University on one of the months which has less than 31 days. Three people are there between the one who goes to Dr. APJ Technical University on one of the months which has more than 30 days and the one who goes to University of Delhi on one of the months which has more than 30 days. Three people are there between the one who goes to Osmania University on one of the months which has less than 30 days and the one who

GovernmentAdda.com | IBPS SBI SSC RBI RRB FCI RAILWAYS

87

Daily Visit

[GOVERNMENTADDA.COM]

goes to Banaras Hindu University on one of the months which has more than 30 days. V does not go to Neither University of Mumbai nor IIT Madras. W does not go to University of Delhi. Explanation Persons

Months

Fruits

University

Q

March

Orange

IIT Madras

W

March

Peach

Dr.APJ Technical

R

June

Apple

Osmania

T

June

Guava

University of Mumbai

P

June

Mango

Shivaji

S

December

Cherry

University of Delhi

U

December

Banana

Banaras Hindu Univ

V

December

Kiwi

Indian Institute of Science

6. As per the given arrangement which of the following combination represents only the people who were born in December?. A. T, V B. U, P, T C. V, U D. P, T E. U, V, S Answer Answer – E. U, V, S 7. As per the given arrangement which of the following person represent the one who was born in the same month as the one who likes Orange? A. U B. P C. R D. T E. W Answer GovernmentAdda.com | IBPS SBI SSC RBI RRB FCI RAILWAYS

88

Daily Visit

[GOVERNMENTADDA.COM]

Answer – E. W 8. Which of the following fruits does T like as per the given arrangement? A. Orange B. Peach C. Guava D. Banana E. Other than those given as options Answer Answer – C. Guava 9. Which of the following combinations is correct as per the given arrangement? A. December – Peach – Shivaji B. June – Orange – IIT Madras C. June – Banana – Indian Institute of Science D. March – Guava – Osmania E. December – Banana – Banaras Hindu University Answer Answer – E. December – Banana – Banaras Hindu University 10. Who amongst the following goes to APJ Technical University as per the given arrangement? A. P B. V C. U D. W E. T Answer Answer – D. W I. Study the following information carefully to answer the given questions GovernmentAdda.com | IBPS SBI SSC RBI RRB FCI RAILWAYS

89

Daily Visit

[GOVERNMENTADDA.COM]

Seven People – A, B, C, D, E, F, and G live in eight different floors of the building (but not necessarily in the same order). One of the floors in the building is vacant. The lowermost floor of the building is numbered one, the one above that is numbered two, and so on till the topmost floor is numbered eight. Each one of them also owns different brands of laptops, namely Lenovo, Dell, HP, Samsung, Apple, Asus and TOSHIBA (but not necessarily in the same order).

The number of people leaving above F is same as the number of floors between F and D. F lives an odd numbered floor above the floor numbered four. Only one person lives between B and the one who owns Samsung. Only three floors between D and the one who owns Apple. The one who owns Asus lives immediately above G, G owns neither Apple nor HP. E does not own Asus. Only three floors between G and A. The one who owns TOSHIBA lives immediately above the one who owns Lenovo, but not on the topmost floor. C lives one of the odd numbered floors above the one who owns Apple. The number of floors between F and the one who owns DELL is only one. Only one person lives between the one who owns TOSHIBA and the vacant floor. Only two people live between C and vacant floor. Two persons live between the one who likes Black colour and the one who likes Yellow Colour. Only one person lives between the one who likes Yellow and the one who likes White. There are two floors between the one who likes Orange and Brown. Four persons live between the one who likes Red and Blue. The one who likes Yellow colour lives on one of the floors above the floor numbered four. The person who lives on the sixth floor does not like white. The one who likes Red colour lives immediately above the one who likes Orange. Explanation Floor No 8 7 6 5 4 3 2 1

Person B C G F _ E A D

Laptop HP Asus Samsung Apple _ DELL TOSHIBA Lenovo

Colour Black Red Orange Yellow _ Brown White Blue

1. Which of the following Statements is true with respect to the given information? A. G lives immediately above the one who likes Yellow. B. E lives immediately above C C. Only three people live between F and the one who owns TOSHIBA.

GovernmentAdda.com | IBPS SBI SSC RBI RRB FCI RAILWAYS

90

Daily Visit

[GOVERNMENTADDA.COM]

D. D owns Samsung. E. All the given statements are true. Answer Answer – A. G lives immediately above the one who likes Yellow. 2. Who amongst the following lives exactly between the vacant floor and the one who owns Lenovo Laptop? A. B, C B. G, D C. F, G D. A, B E. E, A Answer Answer – E. E, A 3. Which of the following floor is vacant? A. 5 B. 7 C. 4 D. 3 E. 6 Answer Answer – C. 4 4. Four of the following five are alike in a certain way and so form a group. Which one of the following does not belong to the group? A. G – HP B. C – Apple C. F – Samsung D. E – Lenovo E. D – DELL GovernmentAdda.com | IBPS SBI SSC RBI RRB FCI RAILWAYS

91

Daily Visit

[GOVERNMENTADDA.COM]

Answer Answer – C. F – Samsung 5. How many people live between C and the one who likes Brown colour? A. Four B. Three C. None D. Five E. Two Answer Answer – E. Two II. Study the following information carefully to answer the given questions

Seven students namely viz A, B, C, D, E, F and G of seven different colleges have the seminar on seven different days, namely viz Monday, Tuesday, Wednesday, Thursday, Friday, Saturday and Sunday of the same week but necessarily in the same order. Each student stays in a hostel in different floor. The lowermost floor of the building is numbered 1, the one above that is numbered 2 and so on till the topmost floor is numbered 7. G stayed on the second floor and has a seminar on Wednesday. The one who stayed on the first floor has a seminar on Saturday. B has a seminar immediately before E. B does not have the seminar on any of the days after G. The one who stayed on the seventh floor does not have a seminar on any of the days on or before Friday. The one who stayed on the third floor has a seminar immediately after C. E does not stay on the fifth floor. The one who stays on the sixth floor does not have a seminar immediately before or after G. F does not have any seminar on Sunday and does not live on the third floor. D does not have the seminar on any one of the days before A. Two persons live between the one who likes 5Star and the one who likes Kitkat. Only one person lives between the one who likes Kitkat and the one who likes Munch. Two persons live between the one who likes Munch and Snickers. Four persons live between the one who likes Dairy Milk and Bournville. The one who likes 5star lives on one of the floors above the floor numbered three. The persons who live on the fourth and fifth floor don’t like Snickers. The one who likes Munch lives immediately below the one who likes Dairy Milk. Explanation

GovernmentAdda.com | IBPS SBI SSC RBI RRB FCI RAILWAYS

92

Daily Visit

[GOVERNMENTADDA.COM]

Floor

Person

Day

Chocolate

7

D

Sunday

Kitkat

6

B

Monday

Dairy Milk

5

C

Thursday

Munch

4

E

Tuesday

5star

3

A

Friday

Perk

2

G

Wednesday

Snickers

1

F

Saturday

Bournville

6. F stays on which of the following floor? A. 1 B. 2 C. 3 D. 4 E. None of these. Answer A. 1 7. Who among the following person likes Munch? A. A B. C C. B D. F E. E Answer B. C 8. Four among the following form a group in a certain way. Which of the following does not belong to Group? A. D – Monday B. C – Tuesday C. A – Wednesday

GovernmentAdda.com | IBPS SBI SSC RBI RRB FCI RAILWAYS

93

Daily Visit

[GOVERNMENTADDA.COM]

D. G – Saturday E. E – Sunday Answer E. E – Sunday 9. Which of the following is correctly matched? A. D – Monday – Kitkat B. C – Tuesday – Munch C. A – Friday – Perk D. G – Saturday – Snickers E. E – Sunday – Kitkat Answer C. A – Friday – Perk 10. Who among the following has the seminar on Sunday? A. A B. C C. B D. D E. E Answer D. D I. Study the following information carefully to answer the given questions

Eight People – A, B, C, D, E, F, G and H live in ten different floors of a building (but not necessarily in the same order). Two of the floors in the building is vacant. The lowermost floor of the building is numbered one, the one above that is numbered two, and so on till the topmost floor is numbered ten. Each one of them also likes different mobiles, namely Lenovo, Apple, ONE PLUS, HTC, Samsung, Oppo, ASUS and Sony(but not necessarily in the same order). Each one of them also participates in different number of events starting from 1 to 10. GovernmentAdda.com | IBPS SBI SSC RBI RRB FCI RAILWAYS

94

Daily Visit

[GOVERNMENTADDA.COM]

The one who likes SONY and APPLE not live on the floors numbered 8 and 7 respectively. The one who likes HTC lives immediately below the floor on which C lives. The number of people living below F is same as the number of people living between F and H. Only three floors between D and the one who likes Samsung. The number of floors between the one who likes ASUS and the one who likes HTC is two. The one who likes Oppo lives immediately above G. F lives an odd numbered floor above the floor numbered four. The top most floor is vacant. Only one person lives between B and the one who likes HTC. Only three floors between G and A. The one who likes SONY lives immediately above the one who likes Lenovo. C lives one of the odd numbered floors above the one who likes Samsung. The number of floors between F and the one who likes APPLE is only one. Only one person lives between the one who likes SONY and the vacant floor. Only two people live between C and vacant floor. The number of floors between two vacant floors is five. Total number of events participated by B is one less than that of the total number of events participated by H. Total number of events participated by the person who lives on ground floor is the square of the total number of events participated by B. Total number of events participated by A is the difference of number of events participated by D and H. Total number of events participated by C is one less than that of B. Total number of events participated by F is one more than that of G. Total number of events participated by E is one more than that of F. Total number of events participated by G is the multiple of number of events participated by B and C. Total number of events participated by the one who likes Asus is four. Explanation Floor No 10 9 8 7 6 5 4 3 2 1

Person _ H B C G F _ E A D

Mobile _ ASUS ONE PLUS Oppo HTC Samsung _ Apple Sony Lenovo

Number of events _ 4 3 2 6 7 _ 8 5 9

1. Which of the following Statements is true with respect to the given information? A. G lives immediately above the one who likes Samsung B. E lives immediately above C C. Only three people live between F and the one who likes SONY. GovernmentAdda.com | IBPS SBI SSC RBI RRB FCI RAILWAYS

95

Daily Visit

[GOVERNMENTADDA.COM]

D. D likes HTC. E. All the given statements are true Answer Answer – A. G lives immediately above the one who likes Samsung 2. Who amongst the following participates in 5 events? A. B B. G C. F D. A E. C Answer Answer – D. A 3. Which of the following floor is immediately above the vacant floor? A. 5 B. 7 C. 4 D. 3 E. 6 Answer Answer -A. 5 4. Four of the following five are alike in a certain way and so form a group. Which one of the following does not belong to the group? A. G – ONE PLUS B. C – Samsung C. F – HTC D. E – Lenovo E. D – APPLE GovernmentAdda.com | IBPS SBI SSC RBI RRB FCI RAILWAYS

96

Daily Visit

[GOVERNMENTADDA.COM]

Answer Answer – C. F – HTC 5. How many people live between C and D? A. Four B. Three C. None D. Five E. Two Answer Answer –A. Four II. Study the following information carefully to answer the given questions

Twelve students namely viz A, B, C, D, E, F, G, H, I, J and K of twelve different colleges have Conference in six different months namely viz January, February, June, July, September and October on twelve different days i.e., (all persons have different dates) from 9th to 20th of any month but not necessarily in the same order. Two students must have Conference in the same month. Each student stays in a hostel in different floor. The ground floor of the building is numbered 1, the one above that is numbered 2 and so on till the topmost floor is numbered 12. There are five floors between the person who have Conference in June. The persons who have Conference in June does not occupy the floor above the floor numbered 10 and also not occupy the floor below the floor numbered 4. G stayed on the second floor and has a Conference on 19th. The one who stayed on the first floor has a Conference in September. E has a Conference on the date immediately after the date on which K has Conference. There are two persons live between G and H. The floor number and the Conference date is same for the person B. The person who has Conference on 15th lives immediately below E. K has Conference on one of the dates after 12 and before 16. There are four persons live between K and E. K has Conference in the month of June. B and I have Conference in the same month which has 31 days. D who lived in floor numbered 11, has the Conference in the month which has less than 30 days. Only one person lives between two persons who have Conference in the month of October. The persons who have Conference in the month of October live two of the floors above the floor numbered 6. There are five floors between B and I. There are six floors between the person J and the person who has Conference in the month of September. The person F stays in a middle between the person E and C. L does not have Conference in the month of June. The person who has Conference on 17th lives immediately above the person who has Conference on 20. H does not have GovernmentAdda.com | IBPS SBI SSC RBI RRB FCI RAILWAYS

97

Daily Visit

[GOVERNMENTADDA.COM]

Conference on 20th. L does not have Conference on 16th. There is only one person lives between the persons who have Conference in the month of July. There are four floors between the persons who have Conference on 19th and 10th. The person who has Conference on 17th lives immediately above the person who has Conference on 20th. H does not have Conference on 20th. A does not have Conference on 11th. The person who has Conference on 18th lives immediately above the person who has Conference on 11. Explanation Floor

Person

Month

Date

12

B

January

12

11

D

February

17

10

A

June

20

9

E

October

14

8

F

September

15

7

C

October

10

6

I

January

18

5

H

July

11

4

K

June

13

3

L

July

9

2

G

February

19

1

J

September

16

6. F stays on which of the following floor? A. 12 B. 8 C. 7 D. 4 E. None of these. Answer B. 8 7. Who among the following has Conference in September? A. A, B B. J, F GovernmentAdda.com | IBPS SBI SSC RBI RRB FCI RAILWAYS

98

Daily Visit

[GOVERNMENTADDA.COM]

C. B, E D. D, G E. E, G Answer B. J, F 8. Four among the following form a group in a certain way. Which of the following does not belong to Group? A. C – October B. J – September C. F – September D. B – January E. E – October Answer C. F – September. Except F, others have Conference on even numbered dates. 9. Which of the following is correctly matched? A. 12 – B – June – 17 B. 12 – F – June – 17 C. 2 – G – February – 19 D. 2 – G – September – 19 E. E – Sunday – July Answer C. 2 – G – February – 19 10. Who among the following has Conference on 20th? A. A B. C C. B D. D E. J GovernmentAdda.com | IBPS SBI SSC RBI RRB FCI RAILWAYS

99

Daily Visit

[GOVERNMENTADDA.COM]

Answer A. A I.Study the following information carefully to answer the given questions

Seven Members Sania, Kamal, Pankaj, Anand, Arjun, Shewag and Sreejesh represents seven different states Madhya Pradesh, Uttar Pradesh, Bihar, Kerala, Haryana, Odisha and Maharashtra in seven different games Hockey, Chess, Cricket, Badminton, Table Tennis, Golf and Billiards. The order of persons, states and games is not nescessarily in the same order. 

Anand represents Kerala in chess. Arjun represents golf team but not from Maharashtra or Uttar Pradesh.



Sania represents Madhya Pradesh for either badminton or table tennis. Pankaj represents Odisha but not for cricket or Table tennis.



The one who represents Bihar represent in Table tennis. The one who represents Hockey represents Uttar Pradesh.



Shewag represents Cricket and from Neither Bihar nor Uttar Pradesh.Sreejesh does not represent Bihar.

Explanation Members

Sates

Games

Sania

Madhya Pradesh

Badminton

Kamal

Bihar

Table tennis

Pankaj

Odisha

Billiards

Anand

Kerala

Chess

Arjun

Haryana

Golf

Shewag

Maharashtra

Cricket

Sreejesh

Uttar Pradesh

Hockey

1. Who among the following represent Bihar ? 1.Kamal 2.Anand 3.Sreejesh 4.Sania 5.None of these

GovernmentAdda.com | IBPS SBI SSC RBI RRB FCI RAILWAYS

100

Daily Visit

[GOVERNMENTADDA.COM]

Answer 1.Kamal 2.

Which of the following combination is correct ? 1.Pankaj – Kerala 2.Kamal – Cricket 3.Shewag – Maharashtra 4.Anand – Golf 5.None of these Answer 3.Shewag – Maharashtra

3.

Who represents Badminton ? 1.Arjun 2.Kamal 3.Sreejesh 4.Sania 5.None of these Answer 4.Sania

4.

Four among the following form a group in a certain way.Which of the following does not belong to Group ? 1.Sania-Table Tennis – Bihar 2.Anand –Kerala – Billiards 3.Arjun –Haryana – Golf 4.Pankaj-Uttar Pradesh – Hockey 5.Sreejesh – Chess – Madhya Pradesh Answer 3.Arjun –Haryana – Golf GovernmentAdda.com | IBPS SBI SSC RBI RRB FCI RAILWAYS

101

Daily Visit 5.

[GOVERNMENTADDA.COM]

Who among the following represents Uttar Pradesh ? 1.Kamal 2.Arjun 3.Pankaj 4.Sreejesh 5.None of these Answer 4.Sreejesh

II.Study the following information carefully to answer the given questions

Eight people R, Q, P , O, N and M work in three different companies TCS, CTS and WIPRO. Out of these, two are female who work in different companies and have different specialisations. 

Two of them are specialist in Finance, two in HR, two in Marketing and One in Research Department and analysing department.



Q working in a company TCS has specialized in HR and her friend N, a finance specialist is working in Wipro.



M is Specialized in HR, and working with S, a marketing specialist. No two people with the same specialisation work together.



O is a specialist in marketing , working in CTS and his friend T has specialized in finance and working in TCS with only one another specialist.



No more than three of them work in Wipro. No female is a researcher or an analyser.

Explanation Person

Company

Specialization

T(M)

TCS

Finance

S(M/F)

Wipro

Marketing

R(M)

CTS

Research Analysing

Q(F)

TCS

HR

P(M)

CTS

Analysing/ Research

O(M)

CTS

Marketing

N(M/F)

Wipro

Finance

M(M)

Wipro

HR GovernmentAdda.com | IBPS SBI SSC RBI RRB FCI RAILWAYS

102

Daily Visit

[GOVERNMENTADDA.COM]

6. R is specialized in which of the following ? 1.Finance 2.Analysing 3.Research 4.Can’t be determined 5.None of these Answer 4.Can’t be determined 7. Which of the following pair represents female ? 1.QS 2.NQ 3.NS 4.Either 1 or 2 5.Either 1 or 3 Answer 4.Either 1 or 2 8. Who among the following is working in TCS ? 1.O 2.M 3.S 4.Q 5.None of these Answer 4.Q 9. Which of the following is correctly matched ? 1.P – CTS – Marketing 2.Q – Wipro – HR GovernmentAdda.com | IBPS SBI SSC RBI RRB FCI RAILWAYS

103

Daily Visit

[GOVERNMENTADDA.COM]

3.S – TCS – Fiance 4.M – Wipro – HR 5.None of these Answer 4.M – Wipro – HR 10. Who among the following specialised in HR ? 1.M 2.Q 3.S 4.Both 1 and 2 5.All of these Answer 4.Both 1 and 2 I. Study the following information carefully to answer the given questions

A, B, C, D, E, F and G are seven people live on seven different floors of a building but not necessarily in the same order. The lower most floor of the building is numbered 1, the one above that is numbered 2 and so on till the topmost floor is numbered 7. Each one of them earn different amount per month. i.e. 20000, 15000, 10000, 25000,30000, 35000 and 40000. (But not necessarily in the same order.)

The one who earn Rs.20000 lives immediately above the one who earns Rs.40000. Only one person lives between B and E. B lives on one of the floors above E. Neither C nor A earns Rs.25000. E does not earn Rs. 10000. A lives on an odd numbered floor but not on the floor numbered three. The one who earns Rs.30000 lives immediately above A. Only two people live between A and the one who earns Rs.10000. The one who earns Rs.15000 lives on one of the odd numbered floors above D. Only three people live between C and the one who earns 15000. The one who earn Rs.10000 lives immediately above C. F earns 10000. Explanation Floor

Persons

Salary

GovernmentAdda.com | IBPS SBI SSC RBI RRB FCI RAILWAYS

104

Daily Visit

[GOVERNMENTADDA.COM]

7

G

25000

6

B

30000

5

A

15000

4

E

20000

3

D

40000

2

F

10000

1

C

35000

1. Which among the following is the salary of B? A. Rs. 20000 B. Rs. 40000 C. Rs. 35000 D. Rs. 30000 E. None of these Answer D. Rs. 30000 2.

Which of the following combination is true as per the given arrangement? A. A – 15000 B. C – 30000 C. B – 35000 D. F – 20000 E. None of these. Answer A. A – 15000

3.

Who among the following lives in floor no 3 ? A. A B. C C. D D. F E. None of these. GovernmentAdda.com | IBPS SBI SSC RBI RRB FCI RAILWAYS

105

Daily Visit

[GOVERNMENTADDA.COM]

Answer C. D 4. Four among the following form a group in a certain way.Which of the following does not belong to Group ? A. G – 15000 B. A – 40000 C. B – 20000 D. F – 40000 E. E – 10000 Answer D. F – 40000 5.

Who among the following earns Rs. 35000 ? A. A B. C C. D D. F E. None of these. Answer B. C

II. Study the following information carefully to answer the given questions

Seven students namely viz A, B, C, D, E, F and G of seven different colleges have seminar on seven different days, namely viz Monday, Tuesday, Wednesday, Thursday, Friday, Saturday and Sunday of the same week but necessarily in the same order. Each student stays in a hostel in different floor. The lower most floor of the building is numbered 1, the one above that is numbered 2 and so on till the topmost floor is numbered 7. G stayed in the second floor and has a seminar on Wednesday. The one who stayed in the first floor has a seminar on Saturday. B has a seminar immediately before E. B does not have seminar on any of the days after G. The one who stayed in the seventh floor does not have a seminar on any of the days on or before Friday. The one who stayed in the third floor has a seminar immediately after C. E does not stay in the fifth floor. The one who stays in sixth floor does not GovernmentAdda.com | IBPS SBI SSC RBI RRB FCI RAILWAYS

106

Daily Visit

[GOVERNMENTADDA.COM]

have a seminar immediately before or after G. F does not have any seminar on Sunday and does not live in third floor. D does not have seminar on any one of the days before A. Explanation Floor

Person

Day

7

D

Sunday

6

B

Monday

5

C

Thursday

4

E

Tuesday

3

A

Friday

2

G

Wednesday

1

F

Saturday

6. F stays in which of the following floor? A. 1 B. 2 C. 3 D. 4 E. None of these. Answer A. 1 7. Who among the following person stays in 5th floor ? A. A B. C C. B D. F E. E Answer B. C

GovernmentAdda.com | IBPS SBI SSC RBI RRB FCI RAILWAYS

107

Daily Visit

[GOVERNMENTADDA.COM]

8. Four among the following form a group in a certain way. Which of the following does not belong to Group ? A. D – Monday B. C – Tuesday C. A – Wednesday D. G – Saturday E. E – Sunday Answer E. E – Sunday 9. Which of the following is correctly matched? A. D – Monday B. C – Tuesday C. A – Friday D. G – Saturday E. E – Sunday Answer C. A – Friday 10. Who among the following have seminar on Sunday? A. A B. C C. B D. D E. E Answer D. D

GovernmentAdda.com | IBPS SBI SSC RBI RRB FCI RAILWAYS

108

Daily Visit

[GOVERNMENTADDA.COM]

200+ New Pattern Seating Arrangement Questions With Solution

GovernmentAdda.com

GovernmentAdda.com | IBPS SBI RBI SSC FCI RRB RAILWAYS

1

Daily Visit

[GOVERNMENTADDA.COM]

Directions (1 – 5): Answer the questions on the basis of the information given below. Ten friends are sitting on twelve seats in two parallel rows containing five people each, in such a way that there is an equal distance between adjacent persons. In Row 1: A, B, C, D and E are seated and all of them are facing south, and in Row 2: P, Q, R, S and T are sitting and all of them are facing north. One seat is vacant in each row. Therefore, in the given seating arrangement each member seated in a row faces another member of the other row. All of them like different colors – Red, Green, Black, Yellow, White, Blue, Brown, Purple, Pink and Grey, but not necessarily in the same order. There are two seats between Q and the vacant seat. Q does not like White, Red and Purple. E is not an immediate neighbor of C. B likes Grey. Vacant seat of row 1 is not opposite to S and is also not at any of the extreme ends of Row-1.The one who likes Black sits opposite to the one, who sits third to the right of the seat, which is opposite to S. C is not an immediate neighbor of D. T, who likes neither White nor Blue, does not face vacant seat. D faces R. The vacant seats are not opposite to each other. Two seats are there between C and B, who sits third right of the seat, on which the person who likes Brown is sitting. S sits third to the right of seat on which R sits and likes Yellow. The one who likes Pink faces the one who likes Yellow. The persons who like Red and Purple are adjacent to each other. The vacant seat in row 1 is not adjacent to D.Q sits at one of the extreme ends. E neither likes Pink nor faces the seat which is adjacent to the one who likes Blue. The one who likes White is not to the immediate right of the one who likes Yellow. The person who likes Green doesn’t face the person who likes Purple. 1. How many persons are sitting between T and the one who likes yellow color? A) None B) One C) Two D) Three E) None of these Answer

Option C Arrangement: E(red)……..….D(purple)………B(grey)…………____…………..A(pink)…………C(brown) T(green)……..R(black)…………..____……………P(white)………S(yellow)……….Q(blue) 2. Which of the following faces the vacant seat of Row – 2? A) The one who like white color B) A C) D D) The one who likes grey color E) Cannot be determined

GovernmentAdda.com | IBPS SBI RBI SSC FCI RRB RAILWAYS

2

Daily Visit

[GOVERNMENTADDA.COM]

Answer

Option D 3. Who is sitting at the immediate left of person who likes purple color? A) E B) D C) The one who likes black color D) The one who likes green color E) The one who likes grey color Answer

Option E 4. Who amongst the following sits at the extreme end of the row? A) R, Q B) E, S C) T, C D) C, D E) None of these Answer

Option C 5. If Q is made to sit on vacant seat of his row, then how many persons are there between the persons who sit opposite to Q now and who sat opposite to Q previously? A) Two B) Three C) Four D) None E) One Answer

Option E Directions (1 – 5): Answer the questions on the basis of the information given below. Eight people – Adiya, Sahil, Anaya, Anshika, Ankur, Tiya, Rohit and Kavya are sitting in a straight line facing North. They have different ages – 13, 18, 20, 23, 29, 32, 39 and 49 but not necessarily in the same order. Aditya is sitting third to right the one who is 23 years old. Anshika and the one who is 23 years old are sitting together. One person is sitting between Kavya and one whose age is 49 years. Difference between GovernmentAdda.com | IBPS SBI RBI SSC FCI RRB RAILWAYS

3

Daily Visit

[GOVERNMENTADDA.COM]

the ages of Ankur and Kavya is 5 years. The one who is 10 years older than Anshika is sitting third to right of Anshika. Rohit and Ananya are sitting together. One of them is sitting at an extreme end of line. Ananya is 9 years older than Sahil. Aditya is sitting in the middle of Anshika and Rohit. Aditya is 12 years younger than Ananya. Kavya is not 13 years old. 1. Who is sitting third to right of Anshika? A) Ananya B) Ankur C) Kavya D) Tiya E) Aditya Answer

Option D Solution: Arrangement from left to right in north direction is Ankur…..Sahil….Anshika……Kavya…….Aditya…..Tiya……Rohit… Ananya 13……….23…….…..39……………18……….20……..…49……….29………….32 2. Who is 29 years old? A) Anshika B) Rohit C) Tiya D) Ankur E) Ananya Answer

Option B 3. What is the age difference between Kavya and Aditya? A) 10 years B) 3 years C) 2 years D) 5 years E) 8 years Answer

Option C 4. Find the odd one out from the following pairs? A) Sahil – 23 B) Tiya – 49 C) Aditya – 20 GovernmentAdda.com | IBPS SBI RBI SSC FCI RRB RAILWAYS

4

Daily Visit

[GOVERNMENTADDA.COM]

D) Kavya – 13 E) Ananya – 2 Answer

Option D 5. Who is sitting to 2nd left of the one whose age is 39 years? A) Sahil B) Aditya C) Kavya D) Ankur E) Anshika Answer

Option D Directions (1 – 5): Answer the questions on the basis of the information given below. Eight people – Priya, Isha, Megha, Ruchi, Bhavya, Priti and Trisha are sitting in a straight line facing North. They have different ages – 12, 16, 18, 21, 26, 33, 45 and 50 but not necessarily in the same order. Isha is sitting second to left the one who is 33 years old. 2 persons are sitting between Isha and Priti. Three people are sitting between Priya and the one who is 26 years old. Trisha is immediate neighbor of Isha. One who is 12 years younger than the one having age 33 years is sitting third to right of her. Bhavya and Priti have 2 years difference in their respective ages. One person is sitting between Bhavya and Priti. Ruchi is sitting second to the right of one having age 45 years. Priya is sitting immediate right of the one who is 18 years old. Age difference between Priya and Trisha is more than 16 years. Shikha is nine years older than Isha. 1. Who is sitting third to right of Bhavya? A) Priya B) Priti C) Megha D) Trisha E) Isha Answer

Option C Solution: Arrangement from left to right in north direction is Trisha…..Isha….Bhavya….Priya……Priti…….Megha…..Shikha……Ruchi …50……….12……..18……….33……….16…….…45…..…….21…………26 2. Who is 16 years old? A) Trisha GovernmentAdda.com | IBPS SBI RBI SSC FCI RRB RAILWAYS

5

Daily Visit

[GOVERNMENTADDA.COM]

B) Priti C) Megha D) Ruchi E) Isha Answer

Option B 3. What is the age difference between Bhavya and Priya? A) 15 B) 17 C) 16 D) 18 E) 20 Answer

Option A 4. Find the odd one out from the following pairs? A) Megha – 45 B) Ruchi – 26 C) Isha – 21 D) Trisha – 50 E) Bhavya – 18 Answer

Option C 5. What is the sum of ages of Priya and Priti? A) 41 B) 55 C) 52 D) 49 E) 40 Answer

Option D 33+16 = 49 Directions (6 – 10): Answer the questions on the basis of the information given below. GovernmentAdda.com | IBPS SBI RBI SSC FCI RRB RAILWAYS

6

Daily Visit

[GOVERNMENTADDA.COM]

Eight people – Swati, Prachi, Richa, Kavya, Sheena, Charu, Malika and Naira are sitting in a straight line facing North. They have different ages – 10, 13, 15, 18, 25, 26, 31 and 40 but not necessarily in the same order. There are 2 persons sitting between Swati and the one whose age is 18 years. Neither of them is sitting at an extreme end. The difference between the ages of Kavya and Malika is 3 years. Richa is sitting second to right of one having age 18 years. Prachi is sitting third to left of one having age 31 years. There are three girls sitting between Kavya and whose age is 18 years old. Prachi and Malika are immediate neighbors. Richa is not 31 years old. There are at least 4 persons sitting to the right of Kavya. Naira and the one having age 18 years are immediate neighbors. The one who is 31 years old is not sitting at second position from any end. Sheena is 3 years older than Swati. Charu is one year older than Richa. 6. Who is 13 years old? A) Kavya B) Prachi C) Malika D) Sheena E) Cannot be determined Answer

Option E Solution: Arrangement from left to right in north direction is Kavya…..Swati…….Prachi….Malika….Sheena…Naira….Richa….Charu 10/13……15………….40…..…13/10….….18………31………25…..…26 7. How many persons are sitting to the left of Sheena? A) Two B) Three C) Five D) None E) Four Answer

Option E 8. What is the difference in ages of Prachi and Richa? A) 7 years B) 27 years C) 3 years D) 15 years E) 14 years Answer

GovernmentAdda.com | IBPS SBI RBI SSC FCI RRB RAILWAYS

7

Daily Visit

[GOVERNMENTADDA.COM]

Option D 9. Who is sitting second to left of Naira? A) Prachi B) Charu C) Malika D) Sheena E) Kavya Answer

Option C 10. How many girls are sitting between Kavya and Swati? A) One B) Four C) Two D) None E) Cannot be determined Answer

Option D Directions (6 – 10): Answer the questions on the basis of the information given below. Eight persons – A, B, C, D, E, F, F, G and H are sitting in a straight line facing North (not necessarily in the same order). They have different ages – 12, 18, 27, 32, 34, 49, 55 and 63 (not necessarily in the same order). B is sitting second to left of one having age 49 years. Two persons are sitting between B and D. One who is 32 years old is sitting second to right of D. One person is sitting between the persons having ages 32 and 18 years. A is sitting second to left of E. A is sitting somewhere to the left of D. The one who is 63 years old is sitting to immediate left of B. Difference between the ages of B and G is 7 years. Both are not sitting together. One who is 27 years old is sitting somewhere left of A. C is 6 years younger to D. The one who is 55 years old and H are immediate neighbors. Same number of persons are sitting between H and one having age 34 years and between F and one having age 55 years. 6. What is the age difference between E and H? A) Other than those given in options B) 31 years C) 22 years D) 23 years E) 19 years Answer

GovernmentAdda.com | IBPS SBI RBI SSC FCI RRB RAILWAYS

8

Daily Visit

[GOVERNMENTADDA.COM]

Option D Arrangement:

7. How many person/s are sitting between F and A? A) 1 B) 4 C) 2 D) None E) Cannot be determined Answer

Option C 8. Who is sitting to the immediate left of D? A) E B) G C) B D) H E) A Answer

Option E 9. Who is 32 years old? A) F B) H C) E D) A E) F Answer

Option B

GovernmentAdda.com | IBPS SBI RBI SSC FCI RRB RAILWAYS

9

Daily Visit

[GOVERNMENTADDA.COM]

10. How many person/s are sitting between G and B? A) 2 B) 3 C) 1 D) 5 E) None of these Answer

Option C Directions (1 – 5): Answer the questions on the basis of the information given below. There are eight persons A, B, C, D, E, F, G, and H are sitting around a circle. Some are facing the center while some are not. They also like different colors – Green, Red, Yellow, Black, White, Blue, Grey and Pink, but not necessarily in the same order. D is sitting fourth to right of B. The person who likes Yellow sits second to right of D. A is sitting third to right of C. There is one person sitting between A and B. A does not like Yellow color. Three persons sit between the person who like Yellow and Pink color. The person who likes Green color sits second to right of person who like Pink color. D does not like Green color. The person who likes Blue sits third to right of person who like Green color. C likes Blue color. There are two persons sitting between D and E. There are three persons between the person who like White and Black color. E does not like White and Black color. There is one person sitting between the person who like Black and Red color. The person who likes Yellow color sits third to right of person who like Red color. The person who likes Grey color sits third to right of person who like Black color. G sits fourth to right of person who like White color. F is not immediate neighbor of G. The person who like White color sits third to left of person who like Yellow color and both faces the same direction.(Same direction means if one faces center then other also faces the center and vice-versa).E faces opposite the center. A likes the Pink color. 1. Who among the following like Yellow color? A) A B) B C) C D) E E) H Answer

Option E Arrangement:

GovernmentAdda.com | IBPS SBI RBI SSC FCI RRB RAILWAYS

10

Daily Visit

[GOVERNMENTADDA.COM]

2. If E is related to C and B is related to D, in the same way A is related to? A) H B) B C) C D) E E) F Answer

Option A 3. Who among the following sits third to right of person who like Pink color? A) F B) D C) G D) H E) E Answer

Option C 4. Which of the following is correctly matched? A) E-Green B) C-Yellow C) D-Grey D) F-Black E) H-Blue GovernmentAdda.com | IBPS SBI RBI SSC FCI RRB RAILWAYS

11

Daily Visit

[GOVERNMENTADDA.COM]

Answer

Option C 5. Who sits opposite to person who likes Green color? A) F B) The person who likes Black color. C) C D) The person who likes Pink color. E) The person who likes Grey color. Answer

Option E Directions (6 – 10): Answer the questions on the basis of the information given below. Eight people – A, B, C, D, E, F, G and H are sitting in a straight line facing North. They have different ages – 14, 19, 21, 24, 30, 33, 40 and 50 but not necessarily in the same order. A is sitting third to right the one who is 24 years old. D and the one who is 24 years old are sitting together. One person is sitting between H and one whose age is 50 years. Difference between the ages of E and H is 5 years. The one who is 10 years older than D is sitting third to right of D. G and C are sitting together. One of them is sitting at an extreme end of line. C is 9 years older than B. A is sitting in the middle of D and G. A is 12 years younger than C. H is not 14 years old. 6. Who is 19 years old? A) D B) A C) H D) B E) G Answer

Option C E(14)…..B(24)….D(40)……H(19)….A(21)…..F(50)……G(30)… C(33) 7. Who is sitting to the immediate left of the one whose age is 50 years old? A) A B) G C) H D) C E) D Answer

GovernmentAdda.com | IBPS SBI RBI SSC FCI RRB RAILWAYS

12

Daily Visit

[GOVERNMENTADDA.COM]

Option A 8. What is the age difference between E and G? A) 18 years B) 26 years C) 24 years D) 16 years E) 20 years Answer

Option D 9. How many people are sitting between B and the one who is 20 years younger to F? A) None B) Two C) Five D) Four E) None of these Answer

Option D 10. What is the age of C? A) 14 years B) 33 years C) 40 years D) 21 years E) None of these Answer

Option B Directions (1 – 5): Answer the questions on the basis of the information given below. Eight persons – A, B, C, D, P, Q, R and S are sitting around a circle not in same order. Three of them are facing towards the centre of circle while others are facing outside. D is sitting third to right of A. There are 2 persons between D and Q. C and P are immediate neighbors and both are facing opposite direction. C and P both are not immediate neighbors of Q. S is sitting third to right of C. There is one person sitting between S and R. Q and D are facing opposite directions (like if Q is facing inside, then D is facing outside and vice versa). B is sitting immediate left of D. S sits second to right of R. D and S faces same direction. D and C faces opposite direction. GovernmentAdda.com | IBPS SBI RBI SSC FCI RRB RAILWAYS

13

Daily Visit

[GOVERNMENTADDA.COM]

1. Who is sitting second to left of Q? A) A B) D C) S D) B E) P Answer

Option D Arrangement: D is sitting third to right of A. There are 2 persons between D and Q. C and P are immediate neighbors and both are facing opposite direction. C and P both are not immediate neighbors of Q.

Next – S is sitting third to right of C. There is one person sitting between S and R. So

Next — Q and D are facing opposite directions (like if Q is facing inside, then D is facing outside and vice versa). B is sitting immediate left of D. In 1st figure – D faces outside, so Q inside. In 2nd figure – D faces inside, so Q outside S sits second to right of R. Last statement makes 5 people facing inside in figure 2, so figure 2 gets discarded.

GovernmentAdda.com | IBPS SBI RBI SSC FCI RRB RAILWAYS

14

Daily Visit

[GOVERNMENTADDA.COM]

2. Who is sitting opposite S? A) C B) P C) D D) A E) Q Answer

Option B 3. Who is sitting third to right of P? A) Q B) D C) R D) B E) A Answer

Option A 4. Which of the following persons faces towards the centre? A) B, D, S B) A, D, P C) A, C, Q D) C, Q, S E) None of these Answer

GovernmentAdda.com | IBPS SBI RBI SSC FCI RRB RAILWAYS

15

Daily Visit

[GOVERNMENTADDA.COM]

Option C 5. Four of the following five are alike in a certain way based on the given arrangement and so form a group. Which is the one that does not belong to that group? A) Q – C B) B – Q C) P – S D) A – B E) D – R Answer

Option B Other 4 pairs are sitting opposite to each other. Directions (6 – 10): Answer the questions on the basis of the information given below. Ten people are sitting in two parallel rows containing six people each, in such a way that there is an equal distance between adjacent persons. In row-1 A, B, C, D, and E are seated and all of them are facing South. In row-2 P, Q, R, S, and T are seated and all of them are facing North. Therefore, in the given seating arrangement each member seated in a row faces another member of the other row. B is sitting at second position from right end of row. The person who is sitting second to left of Q is facing B. T is immediate neighbor of P. Either of T or P is sitting at one of the extreme ends of line. D is sitting opposite S. E is sitting second to the right of D. The person who is facing P is sitting second to right of A. R faces one who is sitting second to left of D. 6. Which of the following is true as per the above arrangement? A) Q is one place away from P B) E is sitting second to left of D C) Two persons are sitting between B and C D) B is facing S E) C is not sitting at an extreme end. Answer

Option C Arrangement: E…….B…….D……..A………C —– facing SouthT…….P…….S………Q……..R —– facing North 7. Who is facing C? A) Q B) R C) T D) P E) S Answer

GovernmentAdda.com | IBPS SBI RBI SSC FCI RRB RAILWAYS

16

Daily Visit

[GOVERNMENTADDA.COM]

Option B 8. How many persons are sitting between P and Q? A) None B) Three C) One D) Two E) cannot be determined Answer

Option C 9. Four of the following five are alike in a certain way based on the given arrangement and so form a group. Which is the one that does not belong to that group? A) A – S B) D – Q C) E – P D) B – Q E) C – Q Answer

Option D

10. Which of the following pairs is sitting at extreme end of lines? A) C, P B) C, T C) R, P D) E, A E) None of these Answer

Option B Directions (1 – 5): Answer the questions on the basis of the information given below. There are 8 members in a family – A, B, C, D, P, Q, R, and S. Each of them has a relationship with A – father, mother, sister, brother, wife, son, and daughter but not necessarily in the same order. They are sitting in a straight line facing North. P is sitting second to right of A’s son. Two people are sitting between P and B’s brother. Two people are sitting between A’s son and A’s father. Q is immediate neighbor of A’s father. Q is sitting at one of the GovernmentAdda.com | IBPS SBI RBI SSC FCI RRB RAILWAYS

17

Daily Visit

[GOVERNMENTADDA.COM]

extreme ends. D and A’s brother are sitting together. A’s brother is sitting second to left of A. A is not immediate neighbor of P. Two people are sitting between D and A’s sister. S’s father is sitting to the immediate right of A’s daughter. A’s wife is immediate neighbor of S. R is younger than C. 1. How many people are sitting between P’s mother and A? A) 1 B) 2 C) 4 D) None E) 5 Answer

Option D Arrangement:

2. Who is A’s brother’s sister? A) C B) Q C) S D) P E) B Answer

Option E 3. Who is A’s wife? A) D B) B C) Q D) P E) S Answer

Option A

GovernmentAdda.com | IBPS SBI RBI SSC FCI RRB RAILWAYS

18

Daily Visit

[GOVERNMENTADDA.COM]

4. Who is sitting second to right of A’s sister? A) A B) Q C) P D) C E) None of these Answer

Option D 5. How many people are sitting between D and A’s daughter? A) 2 B) 3 C) 5 D) 6 E) None Answer

Option B Directions (6 – 10): Answer the questions on the basis of the information given below. There are 8 members in a family – A, B, C, D, P, Q, R, and S. Each of them has a relationship with A – father, mother, sister, brother, wife, son, and daughter but not necessarily in the same order. They are sitting in a circle facing centre. One person is sitting between A and B. A’s daughter is sitting third to right of B. One person is sitting between A and his father. A’s daughter and C are sitting together. A’s father is sitting second to right of A’s sister. B’s sister and D’s sister are immediate neighbors. S is sitting opposite B’s mother. P who is a female is sitting third to right of Q. A’s mother and C’s brother are sitting together. D is A’s son who is sitting opposite to the wife of A. No couple is sitting together. 6. Who is A’s father? A) C B) Q C) B D) S E) None of these Answer

Option B Arrangement: GovernmentAdda.com | IBPS SBI RBI SSC FCI RRB RAILWAYS

19

Daily Visit

[GOVERNMENTADDA.COM]

7. Who is sitting opposite B’s mother? A) R B) Q C) P’s daughter D) R’s daughter E) D’s aunt Answer

Option D 8. How many people are sitting between C and S’s brother when counted clockwise from C? A) 2 B) 1 C) None D) 4 E) Cannot be determined Answer

Option D

GovernmentAdda.com | IBPS SBI RBI SSC FCI RRB RAILWAYS

20

Daily Visit

[GOVERNMENTADDA.COM]

9. Who is sitting 3rd to right of A? A) C B) Q C) D’s mother D) S E) B’s sister Answer

Option C 10. P is A’s ______ A) father B) sister C) brother D) wife E) mother Answer

Option E Directions (1 – 5): Answer the questions on the basis of the information given below. 8 friends – A, B, C, D, E, F, G, and H are sitting in a line facing north in any order. They have different ages – 14, 23, 25, 33, 41, 50, 54, 68 in any order. There is 1 person sitting between A and one having age 25 years. F is sitting third to right of the one having age 25 years. Either F or A is sitting at one of the extreme positions. There are 2 persons sitting between A and E. There are 2 persons sitting between C and one having age 14 years. Both C and one having age 14 years are somewhere left of F and right of A. D is sitting third to left of C. The one who is 23 years old is sitting somewhere to the left of D. There is 8 years age difference between A and E. H and G are immediate neighbors. H is oldest. Ages of 2 of the immediate neighbors are a multiple of 3. G is not sitting at extreme end. 1. What is the age of H? A) 68 B) 23 C) 14 D) 54 E) 50 Answer

Option A Arrangement: H(68)…………G(23)……….A(41)…………D(14)…………B(25)………E(33)………C(54)……..F(50) GovernmentAdda.com | IBPS SBI RBI SSC FCI RRB RAILWAYS

21

Daily Visit

[GOVERNMENTADDA.COM]

2. How many persons sit between G and one having age 25 years? A) 5 B) 2 C) 2 D) None E) Cannot be determined Answer

Option C 3. What is the age difference between A and C? A) 12 years B) 13 years C) 15 years D) 14 years E) 19 years Answer

Option B 4. Who is sitting second to the left of F? A) D B) C C) A D) E E) None of these Answer

Option D 5. Who is sitting on extreme end? A) E B) B C) D D) A E) F Answer

Option E Directions (1 – 5): Answer the questions on the basis of the information given below. GovernmentAdda.com | IBPS SBI RBI SSC FCI RRB RAILWAYS

22

Daily Visit

[GOVERNMENTADDA.COM]

There are eight persons A, B, C, D, P, Q, R and S. They are sitting in a straight line facing North. They all have different age – 14, 22, 24, 31, 36, 43, 46 and 53. The one who is 36 years old is sitting second to left of D. There are 2 people between A and D. The one who is 53 years old is sitting to immediate left of A. Those sitting at extreme ends have age difference of 22 years. One person is sitting between the ones who are 36 and 46 years old. S is sitting at one of the extreme ends of line. S is not 46 years old. R is sitting second to the left of S. C is sitting second to the left of B. The one who is 14 years old is sitting second to left of R. A is younger than P by 19 years. 1. Who is 46 years old? A) D B) C C) B D) P E) R Answer

Option A Arrangement: Note that that there are 3 pairs in which difference is 22 years. These are 53-31, 46-24, 36-14. Both of these have to be at extreme end. 46 and 36 ages are getting arranged in between in possibility arrangements. So pair 53-31 will be placed at extreme ends. C(53)…….A(24)……..B(36)……..Q(14)………D(46)……….R(22)……..P(43)……..S(31) 2. How many people are sitting between B and one aged 22 years? A) Three B) Four C) One D) Two E) Five Answer

Option D 3. Who is sitting third to right of A? A) The one aged 14 years old B) C C) The one aged 46 years old D) R E) None of these Answer

GovernmentAdda.com | IBPS SBI RBI SSC FCI RRB RAILWAYS

23

Daily Visit

[GOVERNMENTADDA.COM]

Option C 4. Q is younger than R by how many years? A) 10 years B) 8 years C) 21 years D) 22 years E) 29 years Answer

Option B 5. What is the age of C? A) 36 years B) 53 years C) 22 years D) 43 years E) 14 years Answer

Option B Directions (6 – 10): Answer the questions on the basis of the information given below. There are eight persons A, B, C, D, E, F, G and H. They are sitting in a straight line facing North. They all have different age – 13, 24, 32, 35, 41, 54, 58 and 65. Two people are sitting between E and C. The one who is 54 years old is sitting second to right of E. B is sitting third to left of one aged 54 years. B is sitting to the immediate right of the one aged 41 years. D is sitting second to the right of one aged 35 years. Neither of them is sitting at an extreme end of line. H is sitting second to left of the one aged 35 years. There is one person sitting between G and one aged 13 years. G is sitting at one of the extreme ends of line. B’s age is not 13 years. 2 people are sitting between A and F. A is sitting second to right of the one who is oldest among all. G is older than A by 8 years. 6. Who is 35 years old? A) A B) E C) B D) H E) D Option B Arrangement: GovernmentAdda.com | IBPS SBI RBI SSC FCI RRB RAILWAYS

24

Daily Visit

[GOVERNMENTADDA.COM]

In the possibility arrangements, note that G is at extreme end and D is not at extreme end. H(41)………….B(65)…………E(35)………A(24)……..D(54)………C(13)……….F(58)………G(32) 7. How many people are sitting between A and D? A) One B) Four C) None D) Two E) Three Option C 8. What is the age of C? A) 65 years B) 32 years C) 58 years D) 13 years E) None of these Answer

Option D 9. Which of the following statements is true with respect to the given arrangement? A) Difference between the ages of G and E is 4 years B) B’s age is 41 years C) D is sitting second to right of one aged 35 years D) C is sitting second to left of E E) None is true Option C 10. Who is sitting second to left of A? A) D B) B C) C D) H E) F Answer

Option B Directions (1 – 5): Answer the questions on the basis of the information given below. GovernmentAdda.com | IBPS SBI RBI SSC FCI RRB RAILWAYS

25

Daily Visit

[GOVERNMENTADDA.COM]

There are eight persons A, B, C, D, E, F, G and H. They are sitting in a straight line facing North. They all have different age – 10, 14, 16, 23, 30, 38, 46 and 58 2 people are sitting between C and one aged 58 years. The one whose age is 16 years is sitting second to the left of C. B is sitting immediate to the left of the one whose age is 58 years. One person is sitting between H and one whose age is 58 years. Either of H or one aged 58 years is sitting at any extreme end. There is a difference of 6 years between the ages of F and G. and 2 persons are sitting between them. The one whose age is 14 years is sitting second to left of D. D is not sitting at any extreme end. There is one person between sitting between F and one aged 46 years. A is immediate neighbor of the one whose age is 46 years. E and one aged 23 are immediate neighbors. H is older than A 1. Who is 30 years old? A) G B) C C) B D) A E) D Answer

Option D Arrangement: F (16)…….A(30)………C(46)……..G(10)……..B(14)………E(58)………..D(23)………H(38) 2. G is sitting immediate right to A) A B) one aged 14 years C) E D) D E) one aged 46 years Answer

Option E 3. How many persons are sitting between the ones having age difference of 4 years? A) One B) Two C) Other than given in options D) None E) Four Answer

Option D G = 10 years, B = 14 years. Age difference = 4 years. None between them GovernmentAdda.com | IBPS SBI RBI SSC FCI RRB RAILWAYS

26

Daily Visit

[GOVERNMENTADDA.COM]

4. What is the age difference between F and D? A) 8 years B) 7 years C) 5 years D) 6 years E) None of these Answer

Option B 5. What is the age of H? A) 46 years B) 23 years C) 38 years D) 58 years E) 16 years Answer

Option C

GovernmentAdda.com | IBPS SBI RBI SSC FCI RRB RAILWAYS

27

Daily Visit

[GOVERNMENTADDA.COM]

I. Study the following information carefully to answer the given questions.

Eight members P, Q, R, S, T, U, V and W of a family are sitting around a circular table with all of them facing outwards. Each one of them like different brand of bikes viz. TVS, Yamaha, Hero, Honda, SUZUKI, KTM, Bajaj and Royal Enfield. Three married couples are there in the family.

P who is the father of U and uncle of V, sits to the left of the person who likes TVS. U is an immediate neighbor of her aunty W who does not sit next to S. R does not like Royal Enfield or Bajaj. W is the only sister-in-law of P whereas Q likes KTM and daughterin-law of R. The two youngest members sit next to each other. The one likes the Yamaha sits between V and the one who likes SUZUKI. V is third to the left of S. The one who likes KTM sits between the persons who like Bajaj and Royal Enfield Respectively. S’s husband and son sit next to her. Honda is not liked by V’s father. V does not like Hero or Bajaj. S is the mother of P and T, and sits second to the left of T. Explanation Explanation-

1. Which of the following statements is true regarding the family? 1. P is the brother of W 2. R is the father-in-law of P 3. Q is the aunty of V 4. U and V are married couple 5. None of the Above Answer Answer – 3. Q is the aunty of V

GovernmentAdda.com | IBPS SBI RBI SSC FCI RRB RAILWAYS

28

Daily Visit

[GOVERNMENTADDA.COM]

2. Who among the following sits between Q

5. Which of the following options represent a

and the one who likes Yamaha?

pair?

1. P

1. Y, X

2. T

2. W, T

3. S

3. W, R

4. V

4. S, U

5. W

5. None of the above

Answer

Answer

Answer – 4. V

Answer – 2. W, T

3. What is the position of the person who

II. Study the following information carefully to answer

likes Honda with respect to the one who

the given questions.

likes KTM ?

Ten persons from ten different cities viz. Delhi, Jaipur,

1. Third to the right

Patna, Indore, Mangaluru, Chennai, Hyderabad,

2. Second to the left

Bengaluru, Raipur and Sri Nagar are sitting in two

3. Immediate left

parallel rows containing five people each, in such a

4. Third to the left

way that there is an equal distance between adjacent

5. Fourth to the left

persons. In row 1- A, B, C, D and E are seated and all of them are facing south. In row 2 – P, Q, R, S and T are

Answer

seated and all of them are facing north. Therefore in the given seating arrangement, each member seated

Answer – 1. Third to the right

in a row faces another member of the other row.(All 4. Who among the following likes Hero?

the information given above does not necessarily

1. W

represent the order of seating in the final

2. U

arrangement.)

3. V 4. X

The person from Indore sits to the immediate right of

5. T

Q. P faces one of the immediate neighbors of the

Answer

person from Jaipur. D faces one of the immediate neighbors of the person from Patna. S is not from

Answer – 5. T

Patna. D is not from Mangaluru. R sits second to the left of the persons from Sri Nagar. A sits third to the GovernmentAdda.com | IBPS SBI RBI SSC FCI RRB RAILWAYS

29

Daily Visit

[GOVERNMENTADDA.COM]

right of person from Chennai. Only One person sits

Answer – 5. B or E

between the person from Raipur and Q. C sits to the immediate left of the person who faces Q. Only two

7. T is from which of the following Cities?

people sit between B and E. The person from

1. Patna

Mangaluru sits second to the right of the one who

2. Indore

faces S. S does not sit at an extreme end of the line.

3. Hyderabad

One of the immediate neighbors of the person from

4. Raipur

Mangaluru faces Raipur. P does not face A. The person

5. Mangaluru

from Delhi sits second to the right of the person from

Answer

Bengaluru. Answer – 3. Hyderabad Explanation Explanation-

8. Which of the following is true regarding C? 1. C sits an extreme end of the line 2. None of the given options is true 3. C is from Bengaluru 4. The person from Indore faces C 5. The person from Hyderabad is an immediate neighbor of C Answer Answer – 4. The person from Indore faces C 9. R is related to Indore in the same way as C

6. Who amongst the following faces the person from Hyderabad? 1. The person from Delhi 2. D 3. The person from Chennai 4. The person from Raipur 5. B or E Answer

is related to Jaipur based on the given arrangement, To who amongst the following is T related to the following same pattern? 1. Delhi 2. Sri Nagar 3. Patna 4. Hyderabad 5. Raipur

GovernmentAdda.com | IBPS SBI RBI SSC FCI RRB RAILWAYS

30

Daily Visit

[GOVERNMENTADDA.COM]

Answer

the left of S. The one who likes WRESTLING sits between the persons who like Badminton and Tennis

Answer – 3. Patna

Respectively. S’s husband and son sit next to her. Foot

10. Who amongst the following sit at extreme

Ball is not liked by V’s father. V does not like Basket

end of the row?

Ball or Badminton. S is the mother of P and T, and sits

1. The person from Delhi and R

second to the left of T.

2. The persons from Bengaluru and A Explanation

3. A and the person from Patna

Explanation-

4. The persons from Chennai and Patna 5. A, E Answer Answer – 2. The persons from Bengaluru and A I. Study the following information carefully to answer the given questions.

Eight members P, Q, R, S, T, U, V and W of a family are sitting around a rectangular table with all of them facing outwards. Each one of them like different type of sports viz. HOCKEY, Cricket, Basket Ball, Foot Ball, BOXING, WRESTLING, Badminton and Tennis. Three married couples are there in the family.

W is the only sister-in-law of P whereas Q likes WRESTLING and daughter-in-law of RP who is the

1. Which of the following statements is true

father of U and uncle of V, sits to the left of the person

regarding the family?

who likes HOCKEY. U is an immediate neighbor of her

1. P is the brother of W

aunty W who does not sit next to S. R does not like

2. R is the father-in-law of P

Tennis or Badminton. The two youngest members sit

3. Q is the aunty of V

next to each other. The one likes the Cricket sits between V and the one who likes BOXING. V is third to GovernmentAdda.com | IBPS SBI RBI SSC FCI RRB RAILWAYS

31

Daily Visit

[GOVERNMENTADDA.COM]

4. U and V are married couple

4. X

5. None of the Above

5. T

Answer

Answer

Answer – 3. Q is the aunty of V

Answer – 5. T

2. Who among the following sits between Q

5. Which of the following options represent a

and the one who likes Cricket?

pair?

1. P

1. Y, X

2. T

2. W, T

3. S

3. W, R

4. V

4. S, U

5. W

5. None of the above

Answer

Answer

Answer – 4. V

Answer – 2. W, T

3. What is the position of the person who

II. Study the following information carefully to answer

likes Foot Ball with respect to the one who

the given questions.

likes WRESTLING ?

Ten persons from ten different cities viz. Delhi, Jaipur,

1. Third to the right

Patna, Indore, Mangaluru, Chennai, Hyderabad,

2. Second to the left

Bengaluru, Raipur and Sri Nagar are sitting in two

3. Immediate left

parallel rows containing five people each, in such a

4. Third to the left

way that there is an equal distance between adjacent

5. Fourth to the left

persons. In row 1- A, B, C, D and E are seated and some of them are facing South and some of them are facing

Answer

North. In row 2 – P, Q, R, S and T are seated and some of them are facing South and some of them are facing

Answer – 1. Third to the right

North. Therefore in the given seating arrangement, 4. Who among the following likes Basket Ball?

each member seated in a row either faces another

1. W

member of the other row or seated behind each

2. U

other.(All the information given above does not

3. V GovernmentAdda.com | IBPS SBI RBI SSC FCI RRB RAILWAYS

32

Daily Visit

[GOVERNMENTADDA.COM]

necessarily represent the order of seating in the final arrangement.)

The person from Indore sits to the immediate right of Q, who seated exactly in the middle of the row. P faces one of the immediate neighbors of the person from Jaipur. D faces one of the immediate neighbors of the person from Patna. S is not from Patna. D is neither from Mangaluru nor from Chennai. P sits immediate right of the person from Raipur. R sits one of the extreme ends of the line and from Raipur. C sits third to the right of person from Jaipur. Only One person sits between the person from Raipur and Q. C sits to

6. Who amongst the following faces the

the immediate right of the person who faces S. Only

person from Chennai?

two people sit between C and E. S is neither from

1. The person from Delhi

Delhi nor from Bengaluru. The person from Mangaluru

2. D

sits second to the right of the one who faces North

3. The person from Mangaluru

Direction. One of the immediate neighbors of the

4. The person from Raipur

person from Mangaluru behind the person from Patna.

5. B or E

P does not face A and faces south direction. The person from Delhi sits exactly between the persons from Sri Nagar and Mangaluru. The person from

Answer Answer – 2. D

Chennai faces the person from Sri Nagar. T faces North Direction and sits immediate left of Q. Only one

7. T is from which of the following Cities?

person sit between the persons from Patna and

1. Patna

Indore. A faces the opposite direction to the person

2. Indore

from Hyderabad.

3. Hyderabad 4. Raipur

Explanation Explanation-

5. Mangaluru Answer Answer – 1. Patna GovernmentAdda.com | IBPS SBI RBI SSC FCI RRB RAILWAYS

33

Daily Visit

[GOVERNMENTADDA.COM]

8. Which of the following is true regarding C? 1. C faces south direction

Answer Answer – 3. D and the person from Jaipur

2. None of the given options is true 3. C is from Bengaluru

I. Study the following information carefully to answer

4. The person from Indore faces C

the given questions.

5. The person from Hyderabad is an immediate neighbor of C

Eight friends A, B, C, D, E, F, G and H are sitting around a

Answer

circular table but not necessarily in the same order. Some of them are facing outward. They are working in

Answer – 1. C faces south direction

four different companies Apple, IBM, Google and Intel. Two persons are working at each company.

9. R is related to Indore in the same way as C is related to Mangaluru based on the given arrangement, To who amongst the

G sits on the immediate right of B, who works at the Google. C sits third to the left of H, who works at the

following is T related to the following same pattern?

Apple and both are facing the same direction. C and B are not facing the same direction but C is an immediate

1. Delhi

neighbor of E, who is fourth to the left of G. E and G

2. Sri Nagar

both are facing opposite directions but both work at the

3. Patna

same company. Those who work at the Google sit

4. Raipur

adjacent to each other but face opposite direction.

5. Chennai

Those who work at the IBM sit opposite each other. The

Answer

immediate neighbours of E are not facing outward. A person who works at the Apple is an immediate

Answer – 5. Chennai

neighbor of the persons who work at the Intel. D and F are immediate neighbours of H. D is not facing the

10. Who amongst the following sit at extreme end of the row?

centre and works at the Intel. The one who is on the immediate left of F is not facing the centre. F sits

1. The person from Delhi and R

second to the right of C.

2. The persons from Bengaluru and A 3. D and the person from Jaipur

Explanation

4. The persons from Chennai and Patna 5. A, E

GovernmentAdda.com | IBPS SBI RBI SSC FCI RRB RAILWAYS

34

Daily Visit

[GOVERNMENTADDA.COM] 3. How many persons are facing outward? 1.Two 2.Three 3.Four 4.Can’t be determined 5.None of these Answer Answer – 2.Three

1. Who among the following works at the

4. A works at which of the following?

Apple ?

1.Either Google or Apple

1.D and F

2.Either Intel or IBM

2.H and F

3.Google

3.G and C

4.Intel

4.C and H

5.Can’t be determined

5.None of these Answer Answer Answer – 3.Google Answer – 4.C and H 5. If D and F interchange their places then 2. Who among the following sits on the

who among the following is on the

immediate right of the person who works

immediate left of G?

at the IBM?

1.B

1.B

2.D

2.D

3.H

3.A

4.F

4.F

5.None of these

5.None of these Answer Answer Answer – 4.F Answer – 1.B

GovernmentAdda.com | IBPS SBI RBI SSC FCI RRB RAILWAYS

35

Daily Visit

[GOVERNMENTADDA.COM]

II. Study the following information carefully and answerthe questions given below:

6. Who among the following is on the immediate right of Naveen? 1.Pradeep

Eight friends Karthick, Naveen, Malik, Kumar, Mohan,

2.Prathap

Pradeep, Raja and Prathap are sitting around a circle.

3.Karthick

Some of them are facing the centre.

4.Mohan 5.Malik

Naveen sits second to the right of Pradeep, who sits on the immediate left of Mohan. The immediate neighbours of Raja faces opposite directions. Prathap is

Answer Answer – 2.Prathap

not facing the centre. The immediate neighbours of Karthick faces the same direction. Mohan is not facing

7. Four of the following are alike in a certain

outside but sits third to the left of Malik. Only one

way and hence form a group. Which one

person sits between Raja and Malik. Neither Prathap

does not belong to that group?

nor Karthick is an immediate left of Raja. Naveen and

1.Raja, Karthick

Karthick both are facing opposite direction. Kumar faces

2.Kumar, Malik

the same direction as Pradeep. Kumar sits second to the

3.Naveen, Mohan

right of Karthick, who is on the immediate left of

4.Pradeep, Malik

Pradeep. Malik sits second to the left of Raja. Naveen is

5.Naveen, Raja

not the immediate neighbour of Karthick. Explanation

Answer Answer – 1.Raja, Karthick 8. Who among the following is an immediate neighbour of Mohan and Prathap? 1.Karthick 2.Naveen 3.Raja 4.Kumar 5.Malik Answer Answer – 2.Naveen GovernmentAdda.com | IBPS SBI RBI SSC FCI RRB RAILWAYS

36

Daily Visit

[GOVERNMENTADDA.COM]

9. Who among the following is opposite to

and 5th. The person whose rank is 5th sits

Karthick?

second to the right of Bhavani. 

1.Malik

Bhagya does not rank 2nd or 1st and she is

2.Mohan

also immediate neighbour of the one whose

3.Prathap

rank is 5th. The person whose rank is 1st

4.Kumar

cannot sit with the person whose rank is

5.None of these

4th. 

Answer

There are two persons sitting between Bindu and the person whose rank is 6th. Bala and Balbir are immediate neighbours of

Answer – 3.Prathap

each other. 10. How many persons are facing the centre?



The person whose rank is 1st sits on the

1.Two

immediate left of the person whose rank is

2.Three

8th. Bharat is second to the right of the

3.Four

person whose rank is 7th. 

4.Five 5.None of these

Bipin’s rank is neither 6th nor 7th. Balbir and Baskar are not immediate neighbours. Baskar, who does not rank 5th, sits on the

Answer

immediate right of Bhagya. There is only one person sitting between those two persons

Answer – 5.None of these

whose ranks are 8th and 6th. I. Study the following information carefully to answer the given questions.



There is only one person sitting between Baskar and the person whose rank is 6th and the person can never be Bhavani.

Eight persons Bharat, Bala, Bindu, Bipin, Balbir, Bhavani, Baskar and Bhagya are sitting around a circular table

Explanation

facing away from the centre. Each of them has a different rank in the test, viz 1st, 2nd, 3rd, 4th, 5th, 6th, 7th and 8th but not necessarily in the same order. 

Bharat and Binduwhose rank is 3rd, can never sit together. There are two persons between the persons whose ranks are 7th GovernmentAdda.com | IBPS SBI RBI SSC FCI RRB RAILWAYS

37

Daily Visit

[GOVERNMENTADDA.COM] 3. Which of the following is the position of Bhavani with respect to the person whose rank is 4th? 1.Second to the left 2.Third to the right 3.Third to the left 4.Fourth to the right 5.Can’t be determined Answer Answer –3.Third to the left

1. Which of the following pairs of persons and 4. If Bharat interchanges his place with the

ranks is definitely true? 1.Bindu – 8th

person whose rank is 2nd, then which of

2.Bipin– 1st

the following is the position of the person

3.Baskar – 2nd

whose rank is 3rd with respect to the

4.Bala – 4th

person whose rank is 6th?

5.All are false

1.Second to the left 2.Second to the right

Answer

3.Third to the right 4.Third to the left

Answer –3.Baskar – 2nd

5.None of these 2. How many persons are sitting between

Answer

Balbir and Bala when counted in anticlockwise direction from Bala?

Answer –3.Third to the right

1.None 2.Two

5. If Bharat is related to Binduand Baskaris

3.Three

related to Balbir in the same way Bhagya is

4.Five

related to?

5.Six

1.Bhavani 2.Bala

Answer

3.Bipin

Answer –5.Six GovernmentAdda.com | IBPS SBI RBI SSC FCI RRB RAILWAYS

38

Daily Visit

[GOVERNMENTADDA.COM]

4.Baskar

Deepa. Deepa’s husband sits second to the

5.None of these

left of Durga. Darshan’s father sits to the immediate right of Divya.Darshan sits

Answer

second to the right of Father of Deepak.

Answer –2.Bala

Explanation

II. Study the following information carefully to answer the given questions.

Eight family members Deepak, Darshan, Deepa, Dinesh, Divya, Dhruv, Durga and Deepthi are sitting around a circular table but not necessarily in the same order. Some of them are females and some of them are males. All of them are related to each other in the same way or the other. Some of them are facing the centre while some are facing outside. 

Only two people sit between Darshan and Divya, Darshan faces the centre. Dhruv sits second to the right of Darshan. Divya is the wife of Deepak. No female is an

centre ? 1.Deepak and Husband of deepa 2.Dhruv and brother of Deepa

intermediate neighbour of Divya. 

6. Who among the following faces outside the

3.Durga and and the father of deepak

Deepa is not an immediate neighbour of Darshan. Deepa is the daughter of Divya.

4.Deepa and the father of deepak 5.None of these

Both the immediate neighbours of Deepa face the centre. Only three people sit between Deepak and Deepa’s brother. Dhruv is not the brother of Deepa. Neither Deepak nor Deepa’s brother is an 

Answer Answer – 4.Deepa and the father of deepak 7. Who among the following sits exactly

immediate neighbour of Dhruv.

between Deepa and Dinesh ?

Deepthi, wife of Darshan sits to the

1.Deepa

immediate left of Dinesh. Both Durga and

2.Father of Darshan

Deepak face a direction opposite to that of

3.Divya

GovernmentAdda.com | IBPS SBI RBI SSC FCI RRB RAILWAYS

39

Daily Visit

[GOVERNMENTADDA.COM]

4.Wife of Darshan

3.Immediate right of Deepak

5.None of these

4.Third to the left of Deepak 5.None of these

Answer

Answer Answer – 4.Wife of Darshan Answer – 4.Third to the left of Deepak 8. Who among the following is brother of Deepa ?

I. Study the following information carefully to answer

1.Deepak

the given questions.

2.Dinesh 3.Darshan

Directions(1-5):Study the following information

4.Deepthi

carefully and answer the given questions.

5.None of these Ten Sports personalities are sitting in two parallel rows

Answer

of six seats each. One seat is vacant in each row. Geeta Phogat,Sushil Kumar, Ravichandran Ashwin, Vijender

Answer – 3.Darshan

Singh and Sania Mirza are sitting in row-1 facing South. 9. How many people sits between Darshan

Sachin, M.S.Dhoni, P.V.Sindhu, Deepa Karmakar and

and Deepa, when counted from the left of

Sakshi Malik are facing North. Each likes a different

Darshan ?

brand of laptops i.e. Lenovo, HP, Dell, Acer, Samsung,

1.Four

Apple, Sony, Asus, Toshiba and HCL.

2.Three 3.Two

DeepaKarmakar sits third to the right of P.V.Sindhu and

4.One

likes Acer. Only two people sit between M.S.Dhoni and

5.None of these

the vacant seat. M.S.Dhoni does not like Samsung or Dell laptop. Sania Mirza is not an immediate neighbour

Answer

of Ravichandran Ashwin. Sushil Kumar likes HCL. The one who likes Dell laptop faces the one who likes Asus.

Answer – 3.Two

The one who likes Dell sits opposite to the one who sits 10. What is the position of Durga with respect

third right of the person who sits opposite to Deepa

to Deepak?

Karmakar. Ravichandran Ashwin is not an immediate

1.Third to the right of Deepak

neighbour of Vijender Singh. Sakshi Malik, who likes

2.Second to the left of Deepak

neither Samsung nor Apple, does not face the vacant GovernmentAdda.com | IBPS SBI RBI SSC FCI RRB RAILWAYS

40

Daily Visit

[GOVERNMENTADDA.COM]

seat. Neither Deepa Karmakar nor P.V.Sindhu sits at any

1.Saniya Mirza, HCL

of the extreme ends of the row. Vijender Singh faces

2.Dell, Sachin

P.V.Sindhu. Vacant seats are not opposite to each other.

3.Samsung, M.S.Dhoni

Two seats are there between Ravichandran Ashwin and

4.Sushilkumar, Toshiba

Sushil Kumar, who sits third right of the one who likes

5.Sakshi, Acer

Sony. The one who likes Toshiba laptop faces the one who likes Acer. The persons who like the Lenovo and Asus are adjacent to each other. Vacant seat of row – 1

Answer Answer – 5.Sakshi, Acer

is not an immediate neighbour of VijenderSingh .M.S.Dhoni sits at one of the extreme ends of the row.

3. How many people will sit between Sachin

P.V.Sindhu does not like Lenovo and Asus. Vacant seat

and M.S.Dhoni?

of row-1 does not face Deepa Karmakar who doesn’t sit

1.One

at any of the extreme ends of the row.

2.Two 3.Three

Explanation

4.Four 5.More than four Answer Answer – 1.One

1. Who among the following likes Apple?

4. Which of the following is correctly

1.Sakshi

matched?

2.Deepa Karmakar

1.Vijender – HCL

3.M.S.Dhoni

2.P.V.Sindhu – Acer

4.Vijender Singh

3.Sakshi – Sony

5.None of these

4.Sachin – Samsung 5.M.S.Dhoni – Toshiba

Answer Answer Answer – 3.M.S.Dhoni Answer – 4.Sachin – Samsung 2. Four among the following form a group in a certain way. Which of the following does not belong to Group ?

5. Which of the following brand Sakshi Malik likes?

GovernmentAdda.com | IBPS SBI RBI SSC FCI RRB RAILWAYS

41

Daily Visit

[GOVERNMENTADDA.COM]

1.HCL

who like Orange and White sit next to each other. Devi

2.Acer

does not like Pink. Daniel sits second to the right of

3.Sony

Dinesh. Only three persons sit between Dinesh and

4.HP

Damodar.

5.Toshiba

Divya sits to the immediate right of Damodar. The persons who like Yellow and Pink sit on the corners and

Answer

opposite to each other. The person who like Orange sit immediate right of the person who likes Red.

Answer – 4.HP Directions (6-10): Study the following information

Explanation

carefully and answer the given questions.

Eight friends Damodar, Dinesh, Daniel, Deepak, Devi, Deepti, Dilip and Divya are sitting around a square table in such a way that four of them sit at four corners while four sit in the middle of each of the four sides, but not necessarily in the same order. Each one of them like different colours viz. Green, Yellow, Black, Purple, Pink, White, Orange and Red. The one who sit in the middle of the sides face the centre while those who sit at the four corners face outside(i.e opposite to the centre).

6. Who among the following likes Black? 1.Dinesh 2.Daniel 3.Deepak

Dilip sits third to the right of Devi. The one who faces the centre likes Green Colour. Devi sits on one of the

4.Devi 5.Damodar

corners of the table. Dinesh does not like yellow. The one who likes black colour is one of the immediate

Answer

neighbours of Deepti. The one who likes green colour sits immediate left of the person one who likes black colour. Only one person sits between Deepak and Dilip. Dinesh is one of the immediate neighbours of Deepak. The one who faces the outside of the centre likes Pink Colour.

Answer – 5.Damodar 7. Four of the following five are alike in a certain way based on the given arrangement and so form a group. Which is the one that does not belong to that

The one who likes Purple faces the Dilip. The persons

group?

GovernmentAdda.com | IBPS SBI RBI SSC FCI RRB RAILWAYS

42

Daily Visit

[GOVERNMENTADDA.COM]

1.Daniel,Damodar

10. Which of the following statements is true

2.Damodar, Devi

regarding Dilip?

3.Daniel, Dinesh

1.Dilip face outside

4.Dinesh, Deepak

2.Dilip likes Purple

5.None of the above

3.Dilip sits immediate left of the person who likes Orange

Answer

4.There are two people sit betwenDilip and Deepti.

Answer – 4.Dinesh, Deepak

5.None of the Above Answer

8. What is the position of Damodar with respect to the one who likes pink colour?

Answer – 5.None of the Above

1.Third to the right 2.Second to the right

Direction (Q1 – Q5): Read the following information

3.Immediate left

and answer the questions below:

4.Third to the left 5.Fourth to the left

Eight family members Prabhu, Priya, Pradeep, Praveen, Preeti, Puja, Poorna and Pragati are sitting around a

Answer

circular table facing the centre. Each has different professions – CA, CS, ICWA, FCA, Lawyer, IAS, Engineer

Answer – 2.Second to the right

and Pilot – but not necessary in the same order. 9. Who among the following sits between Daniel and the one who likes



Priya sits second to the left of Pragati’s

Orange colour?

husband, who is neither an FCA nor a

1.Deepak

Engineer. No female is an immediate

2.The one who likes purple

neighbor of Priya. Praveen’s daughter sits

3.Devi

second to the right of Puja and on the

4.The one who likes red

immediate left of ICWA.Puja, who is sister of

5.Deepti

Poorna, is a Engineer. Puja is not an

Answer Answer – 4.The one who likes red

immediate neighbor of Pragati’s husband. 

Praveen’s daughter is a CA.Only one person is sitting between Prabhu and Puja.Pragati’s brother Praveen sits on the immediate left

GovernmentAdda.com | IBPS SBI RBI SSC FCI RRB RAILWAYS

43

Daily Visit

[GOVERNMENTADDA.COM]

of his mother, who is an IAS. Prabhu is the father of Poorna. Only one person sits between Pragati’s mother and Preeti.Preeti sits on the immediate right of the person who is a CS.Only one person sits between Pragati and Poorna. Poorna sits second to the right of the person who is a pilot. Poorna is mother of Pradeep and not an immediate neighbor of Preeti. Explanation

1. Who amongst the following is Praveen’s

1.Puja, Preeti

daughter ?

2.Pradeep, Puja

1.Priya

3.Preeti, Priya

2.Pradeep

4.Pragati, Praveen

3.Preeti

5.Poorna, Pragati

4.Poorna

Answer

5.Pragati

Answer – 2.Pradeep, Puja

Answer

3. The person who is a Pilot is sitting between

Answer – 3.Preeti

which of the following persons ? 2. Four of the following five are alike in a

1.CA and FCA

certain way based on the given information

2.IAS and CA

and so form a group. Which is the one that

3.IAS and Lawyer

does not belong to that group ? GovernmentAdda.com | IBPS SBI RBI SSC FCI RRB RAILWAYS

44

Daily Visit

[GOVERNMENTADDA.COM]

4.FCA and Engineer

Krishna, Kabir, Kajal, Kalyan, Kalpana and Kamal are

5.None of these

seated(but not necessarily in the same order) and all of them are facing South. In Row -2, Kripa, Kailash, Kavya,

Answer

Kannan, Kasturi and Kiran are seated (but not necessarily in the same order) and all of them are facing

Answer – 3.IAS and Lawyer

North. Therefore, in the given seating arrangement, 4. Who among the following is anIAS?

each member seated in a row faces another member of

1.Can’t be determined

the other row. All of them have a car of different

2.Prabhu

companies, viz Honda, Hyundai, Ford, Fiat, Chevrolet,

3.Pradeep

Audi, BMW, Toyota, Renault, Skoda, Volkswagen and

4.Mother of Praveen

Maruti but not necessarily in the same order.

5.None of these 

Answer

Kailash faces one who sits third to the right of Kalpana. Kannan neither owns Fiat nor Ford. The person who owns Maruti car

Answer – 4.Mother of Praveen

facing South Direction.Kabir owns Skoda and 5. What is the position of Prabhu with respect

faces the one who owns Audi car.Only three

to his grandson ?

persons sit between Kalpana and Kabir.

1.Immediate left



The person who owns Fiat car facing North

2.Third to the left

Direction.Kamal does not own Maruti. The

3.Immediate right

persons who own Audi and BMW sit next to

4.Second to the right

each other. The one who owns Toyota sits

5.Fourth to the left

second to the right of Krishna. Kamal is neither an immediate neighbour of Kabir

Answer

nor Kalpana. 

Answer – 3.Immediate right

One of the immediate neighbours of Kamal faces Kiran. The one who owns Renault car

Directions (6-10)Study the following information

sits at one of the extreme ends of the line.

carefully to answer the given questions.

Only three persons sit between Kiran and the one who faces Kajal. The one who owns

Twelve persons are sitting in two parallel rows

Honda car sits at one of the extreme ends of

containing six people in such a way that there is an

the line.

equal distance between adjacent persons. In Row – 1, GovernmentAdda.com | IBPS SBI RBI SSC FCI RRB RAILWAYS

45

Daily Visit 



[GOVERNMENTADDA.COM]

There are two people sit between one who

7. Who amongst the following sit at the

owns Maruti car and the one who owns

extreme ends of the rows?

Renault.Kripa sits second to the left of the

1.Kalpana, Kabir

one facing Krishna. The person who owns

2.Kripa, Krishna

Chevrolet faces North Direction.Kavya is not

3.Kalpana, Kasturi

an immediate neighbour of Kailash.

4.Kalpana, Kannan

Kabir does not face Kasturi. There are three

5.Kripa, Kalpana

people sit between one who owns Honda car and the one who owns Fiat. The person who owns Ford faces one of the immediate

Answer Answer – 5.Kripa, Kalpana

neighbours of Maruti. There are two people sit between one who owns Toyota and 

8. Who amongst the following faces Kripa? 1.Kalyan

Volkswagen. Explanation

2.Kalpana 3.Kamal 4.Kajal 5.None of the Above Answer Answer – 4.Kajal 9. Krishna is related to Ford in the same way

6. How many persons are seated between

as Kannan is related to Toyota based on the

Kannan and Kiran?

given arrangement. Then Kamal is related

1.One

to?

2.Two

1.Ford

3.Three

2.BMW

4.Four

3.Fiat

5.None of these

4.Hyndai

Answer

5.None of the Above

Answer – 2.Two

Answer Answer – 3.Fiat GovernmentAdda.com | IBPS SBI RBI SSC FCI RRB RAILWAYS

46

Daily Visit

[GOVERNMENTADDA.COM]

10. Four of the following five are alike in a



Two males of the group are from India and

certain way based on the given

UK, and they are sitting opposite to each

arrangement and so form a group. Which is

other.

the one that does not belong to that



E sits fifth to the left of the one from Brazil,

group?

who is one of the six females of the

1.Kripa, Kabir

group.The one from Kenya is sitting

2.Kavya, Kalyan

opposite to the one from USA 

3.Kiran, Kamal

G is not an immediate neighbor of USA or

4.Kannan, Krishna

China, and is sitting fourth to the left of C,

5.Kripa, Kajal

the one from UK.

Answer

Explanation

Answer – 5.Kripa, Kajal Direction (Q1 – Q5): Read the following information and answer the questions below:

Eight friends A,B,C,D,E,F,G & H from different countries are sitting around a circular table facing the center. Some of these are females and others are males. 

H, who is from USA, sits third to the right of the one from China.Three people sit between the ones from Japan and Peru.



B is not an immediate neighbor of the one from Japan.D sits sixth to the left of E.Only one person sits between H & B.



The one from Kenya is the immediate neighbor of the ones from India and China.F is sitting fifth to the left of the one from

1. Who is sitting seventh to the right of A? 1.The one from Peru 2.H 3.The one from Kenya 4.G 5.F Answer

Peru, but is not an immediate neighbor of the ones from India or Kenya.

Answer – 3.The one from Kenya

GovernmentAdda.com | IBPS SBI RBI SSC FCI RRB RAILWAYS

47

Daily Visit

[GOVERNMENTADDA.COM]

2. Which country does E belong to?

5. Who is sitting third to the right of the

1.Peru

person who is fifth to the right of the one

2.Kenya

from India?

3.India

1.The one from Peru

4.USA

2.H

5.Japan

3.B 4.The one from India

Answer

5.None of these

Answer – 2.Kenya

Answer

3. If these people made to sit in the

Answer – 4.The one from India

alphabetical order, vowels first and then consonants , starting from A, who will be

Direction (Q6 – Q10): Read the following information

sitting third to the left of D?

and answer the questions below:

1.A 2.B

Eight teachers A,B,C,D,E,F,G & H are sitting on a cement

3.C

bench. Some are facing north and some are facing

4.G

south. They all like different colors ; Green, yellow, blue,

5.H

white, brown,pink , black and red.

Answer



fifth to the right of G

Answer – 5.H 

4. If A is related to UK in the same way E is

yellow. 

2.D 3.A

Answer

E doesn’t like white or black or green , and faces the direction opposite to that of H.



4.C 5.B

A & D sitting at the extreme ends and no one sitting at the extreme ends like black or

related to Peru , what is related to Japan? 1.E

G is not an immediate neighbor of A.E sits

B faces south. And none of the people who sit at the extreme ends face south.



The one who likes pink is the immediate neighbor of two north facing teachers.There are four people between A & C

Answer – 1.E GovernmentAdda.com | IBPS SBI RBI SSC FCI RRB RAILWAYS

48

Daily Visit 



[GOVERNMENTADDA.COM]

None of the people who face north likes

8. Who likes green?

pink, or white or blue.H likes red.

1.A

G sits third to the right of F and does not sit

2.B

on any ends. There are as many people

3.C

between E & H as there is to the right of C.

4.G

Explanation

5.A or D Answer Answer – 5.A or D 9. Find the odd one out 1.A

6. What color does F like?

2.F

1.Black

3.H

2.Green

4.C

3.Red

5.D

4.Yellow

Answer & Explanation

5.Cannot be determined Answer

Answer – 3.H Explanation:

Answer – 5.Cannot be determined

Because H is the only one whose colour is fixed

7. Who is sitting third to the left of E? 1.G

10. If A likes green, what color does C like?

2.B

1.Pink

3.The one who likes green

2.Brown

4.The one who liked red

3.Yellow

5.Cannot be answered

4.Black

Answer

5.Cannot be determined Answer

Answer – 4.The one who liked red Answer – 5.Cannot be determined

GovernmentAdda.com | IBPS SBI RBI SSC FCI RRB RAILWAYS

49

Daily Visit

[GOVERNMENTADDA.COM]

I. Study the following information carefully to answer the given questions

A group of eight friends – A, C, F, S, X, M, I and W – are sitting in a straight line facing north. 1. The person who likes yellow has which Each of them has different company cars– Datsun,

company car ?

Renault, Hyundai, Tata, Maruti, Toyota, BMW and

1.Datsun

Volkswagen. Each of them likes different colours – Pink,

2.BMW

Yellow, Red, Black, Blue, Orange, White and Green, but

3.Toyota

not necessarily in the same order.

4.Tata 5.None of these



C, who has a Maruti car, sits third to the left

Answer

of F. Neither C nor F sits at the extreme ends of the line. A, who likes Pink colour, has

Answer – 3.Toyota

Hyundai car. A is not an immediate 2. What is the position of F with respect to X ?

neighbour of either C or F. 

S is two places away from M and likes Orange colour. M, who has BMW car, likes Red colour. I, who has Datsun car, sits at an extreme end of the line and likes White

2.Second to the left 3.Immediate right 4.Third to the left 5.None of these

colour. 

1.Fourth to the right

X, who has Tata car likes Green colour and

Answer

sits on the immediate left of C. C does not Answer – 1.Fourth to the right

like either Blue or Yellow colour. 

One who has Volkswagen sits on the immediate left of who has Datsun car. F, who has Renault car, does not like Yellow colour.

3. If A is related to black, W is related to Blue then M is related with which color ? 1.Green 2.Orange

Explanation

3.White 4.Pink 5.None of these GovernmentAdda.com | IBPS SBI RBI SSC FCI RRB RAILWAYS

50

Daily Visit

[GOVERNMENTADDA.COM]

Answer

Ten Stationery items Pencil, Books, Notes, Pens, Sharpners, Chalk, Stickers, Gum, Scale, Covers are

Answer – 2.Orange

palced in 10 boxes numbered 1, 2, 3, 4, 5, 6, 7, 8, 9 and

4. Four of the following five are alike in a certain way based on their position, which

10 are placed adjacent to one another in two different rows of 5 boxes each.

one does not belong to that group ? 

1.WFI

Boxes with odd numbers are situated

2.ACM

opposite to the boxes with even numbers

3.SMC

and no two even number boxes with even

4.XCW

number and no two boxes with odd

5.None of these

numbers are adjacent to each other. No two boxes have consecutive numbers.

Answer



Box no 8 is occupied by books and it is to the extreme left of one end of the row.

Answer – 4.XCW



Box 4 and 6 are not in the row of box no 8 and any of the boxes numbered 4, 6, and 3

5. Which of the following is correct according

are not in the middle of the row.

to the given information ? 1.Two persons sits between W and who



Gums are placed in Box no 5.and its box is

likes pink colour

not situated in the row where box no 6 is

2.X and I are sits at the extreme end of the

situated. 

line

Notes are placed in an odd numbered box

3.C likes Orange colour

and placed in a row where Books are

4.X is sit between who has Maruti and

situated but both are not adjacent to each

Hyundai cars

other. 

5.Both 1 and 4

Sharpner’s and Chalk’s box are adjacent to box no 6 and Chalk’s box is not adjacent to

Answer

box no 4. 

Answer – 5.Both 1 and 4

Pencil’s & Cover’s box are placed in the same row. Pen’s box is even numbered but

II. Study the following information carefully to answer

not 10 and diagonally opposite to box no 1

the given questions

which contains stickers

GovernmentAdda.com | IBPS SBI RBI SSC FCI RRB RAILWAYS

51

Daily Visit 

[GOVERNMENTADDA.COM]

Box no 6 is second from the one of the end

1.5, Chalk

of the row. Stickers’s box is neither opposite

2.6, Covers

to Sharpners’s box nor adjacent to it.

3.8, Pens 4.10, scale

Explanation

5.None of these Answer Answer – 4.10, scale 9. If pencil is placed in the box numbered 4

6. The pack of notes are placed in which number box ?

then which box is placed opposite to the box which contains pencils ?

1.7

1.Pencils

2.9

2.Covers

3.8

3.Stickers

4.3

4.Gum

5.None of these

5.None of these

Answer

Answer

Answer – 1.7

Answer – 4.Gum

7. Covers are kept in which number box ? 1.4

10. If the box 1 is related to Sharpners, box 10 is related to Pens then Box 8 is related with

2.6

?

3.2

1.Scale

4.3

2.Books

5.Can’t be determined

3.Chalk 4.Notes

Answer

5.None of these Answer – 5.Can’t be determined Answer 8. The number 9 box is placed opposite to which box and contains which stationery ?

Answer – 1.Scale

GovernmentAdda.com | IBPS SBI RBI SSC FCI RRB RAILWAYS

52

Daily Visit

[GOVERNMENTADDA.COM]

I. Study the following information carefully to answer

neighbors of the person from Pakistan. Only One

the given questions.

person sits between the person from Russia and Q. C sits to the immediate right of the person who faces S.

Ten persons from ten different countries viz. Denmark,

Only two people sit between C and E. S is neither from

Japan, Pakistan, India, Mauritius, Canada, Hungary,

Denmark nor from Bangladesh. The person from

Bangladesh, Russia and Singapore are sitting in two

Mauritius sits second to the right of the one who faces

parallel rows containing five people each, in such a

North Direction. One of the immediate neighbors of

way that there is an equal distance between adjacent

the person from Mauritius behind the person from

persons. In row 1- A, B, C, D and E are seated and some

Pakistan. A faces the opposite direction to the person

of them are facing South and some of them are facing

from Hungary.

North. In row 2 – P, Q, R, S and T are seated and some of them are facing South and some of them are facing North. Therefore in the given seating arrangement,

Explanation Explanation-

each member seated in a row either faces another member of the other row or seated behind each other.(All the information given above does not necessarily represent the order of seating in the final arrangement.)

S is not from Pakistan. D is neither from Mauritius nor from Canada. P sits immediate right of the person from Russia. R sits one of the extreme ends of the line and from Russia. C sits third to the right of person from Japan. P does not face A and faces south direction. The person from Denmark sits exactly between the

1. Who amongst the following faces the

persons from Singapore and Mauritius. The person from Canada faces the person from Singapore. T faces North Direction and sits immediate left of Q. Only one person sit between the persons from Pakistan and India. The person from India sits to the immediate right of Q, who seated exactly in the middle of the

person from Canada? A. The person from Denmark B. D C. The person from Mauritius D. The person from Russia E. B or E

row. P faces one of the immediate neighbors of the person from Japan. D faces one of the immediate

Answer

GovernmentAdda.com | IBPS SBI RBI SSC FCI RRB RAILWAYS

53

Daily Visit

[GOVERNMENTADDA.COM] Answer

Answer – B. D 2. T is from which of the following Countries? A. Pakistan

Answer – E. Canada 5. Who amongst the following sit at extreme

B. India

end of the row?

C. Hungary

A. The person from Denmark and R

D. Russia

B. The persons from Bangladesh and A

E. Mauritius

C. D and the person from Japan

Answer

D. The persons from Canada and Pakistan E. A, E

Answer – A. Pakistan Answer 3. Which of the following is true regarding C? Answer – 3. D and the person from Japan

A. C faces south direction. B. None of the given options is true

II. Study the following information carefully to answer

C. C is from Bangladesh

the given questions.

D. The person from India faces C E. The person from Hungary is an immediate neighbor of C

Ten persons – P, Q, R, S, T, U, V, W, X and Y are sitting in two rows with five persons in each row. In Row – 1,

Answer

the persons are facing south and in Row – 2, the persons are facing north. Each person in row one faces

A. C faces south direction.

a person from the other row. All of them have a laptop

4. R is related to India in the same way as C is related to Mauritius based on the given arrangement, To who amongst the

of different companies, viz HP, Lenovo, Acer, Dell, Sony, Samsung, TOSHIBA, Apple, Asus and HCL but not necessarily in the same order.

following is T related to the following same pattern? A. Denmark B. Sri Nagar C. Pakistan D. Russia E. Canada

R owns TOSHIBA. The one who likes Dell sits opposite to the one who is second to the right of Q. X does not like HCL. T sits opposite to the one who sits second to the left of the person who owns Acer. There is only one person stands between Q and R. The persons who like Sony and Samsung sit opposite each other. U sits opposite to P, who owns HP. The one who owns GovernmentAdda.com | IBPS SBI RBI SSC FCI RRB RAILWAYS

54

Daily Visit

[GOVERNMENTADDA.COM]

Lenovo sits opposite the one who owns Apple. Y is not

B. Two

facing the north but sits third to the left of V, who

C. Three

owns Lenovo. T sits at one of the ends of the row and

D. Can’t be determined

owns Samsung. The one who owns Apple is on the

E. None of the above

immediate right of S, who does not own HCL. The persons who own Acer and Dell are not facing the north direction.

Answer Answer – B. Two

Explanation Explanation-

8. Which of the following statements is/are true? A. W owns HCL and sits at one of the ends of the row. B. U sits on the immediate left of the one who owns Lenovo. C. The one who owns Asus sits on the immediate left of the one who owns HCL. D. Only (A) and (B) E. Only (B) and (C) Answer

6. S owns which of the following brand of

Answer – D. Only (A) and (B)

laptop? 9. Four of the following five are alike in a

A. Asus B. Apple

certain way and hence form a group. Which

C. Sony

one of the following does not belong to

D. TOSHIBA

that group?

E. None of the above

A. W, S B. U, P

Answer

C. V, P D. X, R

Answer – A. Asus

E. V, Q 7. How many persons sit between S and R?

Answer

A. One GovernmentAdda.com | IBPS SBI RBI SSC FCI RRB RAILWAYS

55

Daily Visit

[GOVERNMENTADDA.COM]

Answer – C. V, P

S’s husband and son sit next to her. Piano is not liked by V’s father. V does not like Guitar or Accodion. S is

10. Who among the following sit at the

the mother of P and T, and sits second to the left of T.

extreme ends of the row? A. Y, X

Explanation Explanation-

B. W, T C. W, R D. S, U E. None of the above Answer Answer – B. W, T I. Study the following information carefully to answer the given questions.

Eight members P, Q, R, S, T, U, V and W of a family are sitting around a rectangular table with all of them facing outwards. Each one of them like different type of music instruments viz. XYLOPHONE, Balafon, Guitar, Piano, VIOLIN, TRUMPET, Accodion and Flute. Three married couples are there in the family.

1. Which of the following statements is true regarding the family? 1. P is the brother of W

W is the only sister-in-law of P whereas Q likes TRUMPET and daughter-in-law of RP who is the father of U and uncle of V, sits to the left of the person who likes XYLOPHONE. U is an immediate neighbor of her aunty W who does not sit next to S. R does not like Flute or Accodion. The two youngest members sit next

2. R is the father-in-law of P 3. Q is the aunty of V 4. U and V are married couple 5. None of the Above Answer

to each other. The one who likes the Balafon sits between V and the one who likes VIOLIN. V is third to the left of S. The one who likes TRUMPET sits between the persons who like Accodion and Flute Respectively.

Answer – 3. Q is the aunty of V 2. Who among the following sits between Q and the one who likes Balafon?

GovernmentAdda.com | IBPS SBI RBI SSC FCI RRB RAILWAYS

56

Daily Visit

[GOVERNMENTADDA.COM]

1. P

1. Y, X

2. T

2. W, T

3. S

3. W, R

4. V

4. S, U

5. W

5. None of the above

Answer

Answer

Answer – 4. V

Answer – 2. W, T

3. What is the position of the person who likes Piano with respect to the one who

II. Study the following information carefully to answer the given questions.

likes TRUMPET ? 1. Third to the right

Eight players – P, Q, R, S, T, U, V and W sit around a

2. Second to the left

square table in such a way that four of them sit on the

3. Immediate left

four sides while the rest at corners. They play different

4. Third to the left

instruments namely Xylophone, Balafon, Guitar, Piano,

5. Fourth to the left

Violin, Trumpet, Accodion and Flute. Some of them are facing the centre while some are facing outside.(i.e

Answer

away from the centre)

Answer – 1. Third to the right Note: Same directions means that if one person facing 4. Who among the following likes Guitar?

the centre then the other person also faces the centre

1. W

and vice versa. Opposite direction means if one person

2. U

is facing the centre then the other person faces outside

3. V

and vice versa.

4. X 

5. T

Q faces the centre of the table and does not sit on any corner. V sits on one of the corner

Answer

between the Flute player and Trumpet player. W sits second to the right of Balafon

Answer – 5. T

player who faces the centre. 5. Which of the following options represent a pair?



The Violin player sits third to the left of Q. S sits opposite to W. P sits on the corner

GovernmentAdda.com | IBPS SBI RBI SSC FCI RRB RAILWAYS

57

Daily Visit

[GOVERNMENTADDA.COM]

exactly opposite to T. The Balafon player sits

D. Can’t be determined

third to the right of Accodion player. The

E. None of these

Xylophone player does not facing the centre. 

The Trumpet sits opposite to Q, also faces in

Answer Answer – B. U

opposite direction of Q and sits between Accodion player and Violin player. T who is





7. R is related to which of the following

the Violin player sits immediate right to the

Instruments?

Piano player.

A. Guitar

The Piano player faces the same direction of

B. Xylophone

the U. The immediate neighbours of Q are

C. Accodion

facing opposite directions. The Accodion

D. Can’t be determined

player sits exactly opposite to Guitar player.

E. None of these

The one who is on the immediate left of U is

Answer

facing the same direction as W. R sits third to the left of W.

Answer – A. Guitar

Explanation Explanation-

8. Who among the following sits exactly between R and the Xylophone Player? A. The person who plays Flute B. The person who plays Accodion C. The person who plays Balafon D. Trumpet E. None of these Answer Answer – C. The person who plays Balafon

6. Who among the following is a Trumpet player?

9. How many persons sit facing the centre?

A. P

A. None

B. U

B. One

C. T

C. Two

GovernmentAdda.com | IBPS SBI RBI SSC FCI RRB RAILWAYS

58

Daily Visit

[GOVERNMENTADDA.COM]

D. Three

likes USPA sits opposite to the one who sits third right

E. Four

of the person who sits opposite to G. O is not an immediate neighbour of P. H, who likes neither Van

Answer

Heusan nor John Players, does not face the vacant seat. Neither G nor F sit at any of the extreme ends of the

Answer – E. Four

row. P faces F. Vacant seats are not opposite to each 10. Which of the following pairs are the

other. Two seats are there between O and N, who sits

immediate neighbors of the Flute player?

third right of the one who likes Dennis Lingo. The one

A. P, V

who likes Peter England faces the one who likes Lee.

B. P, R

The persons who like the Arrow and Highlander are

C. V, R

adjacent to each other. Vacant seat of row – 1 is not an

D. Q, R

immediate neighbour of P. E sits at one of the extreme

E. V, Q

ends of the row. F does not like Arrow and Highlander. Vacant seat of row-1 does not face G who doesn’t sit at

Answer

any of the extreme ends of the row. Answer – A. P, V I. Study the following information carefully to answer

Explanation Explanation-

the given questions.

Ten friends are sitting in two parallel rows of six seats each. One seat is vacant in each row. M, N, O, P and Q are sitting in row-1 facing South. D, E, F, G and H are facing North. Each likes the different brand of Shirts i.e. Arrow, Wrangler, USPA, Lee, Van Heusan, John Players, Dennis Lingo, Highlander, Peter England and John Miller. 1. In the given arrangement, if two people G sits third to the right of F and likes Lee. Only two

come and sit to the immediate left of E,

people sit between E and the vacant seat. E does not

how many people will sit between D and E?

like Van Heusan or USPA. Q is not an immediate

A. Two

neighbour of O. N likes John Miller. The one who likes

B. Three

USPA faces the one who likes Highlander. The one who

C. Four

GovernmentAdda.com | IBPS SBI RBI SSC FCI RRB RAILWAYS

59

Daily Visit

[GOVERNMENTADDA.COM]

D. More than four

A. QE

E. One

B. ND C. HO

Answer

D. PG E. FP

Answer – B. Three 2. Who amongst the following sits third to the right of F?

Answer Answer – E. FP

A. The one who likes Lee B. E

5. Who amongst the following sits at the

C. Other than those given as options

extreme end of the row?

D. D

A. Q, E

E. The one who likes Van Heusan

B. N, D C. P, G

Answer

D. F, P E. Other than those given as options

Answer – A. The one who likes Lee 3. Which of the following faces the vacant seat of Row – 1?

Answer Answer – A. Q, E

A. The one who likes Lee B. E

II. Study the following information carefully to answer

C. Other than those given as options

the given questions.

D. F

Eight friends P, Q, R, S, T, U, V and W are sitting around

E. The one who likes Van Heusan

a circular table facing the centre. Each one of them likes different brands of watch namely viz, Sonata, Titan,

Answer

Fastrack, LAURELS, Casio, Timex, Tissot and Matrix but

Answer – E. The one who likes Van Heusan 4. Four of the following five are alike in a

not necessarily in the same order.

P sits third to the right of the person who likes Matrix. P

certain way based upon their seating

does not like Fastrack. T is not an immediate neighbour

arrangement and so form a group. Which

of W. R sits on the immediate right of the person who

of the following does not belong to the

likes Fastrack. Neither P nor W likes Casio or a Titan.

group?

The person who likes Casio is not an immediate GovernmentAdda.com | IBPS SBI RBI SSC FCI RRB RAILWAYS

60

Daily Visit

[GOVERNMENTADDA.COM]

neighbour of the person who likes Matrix. The person

arrangement and thus form a group. Which

who likes Sonata sits second to the left of T. The person

is the one that does not belong to that

who likes Sonata is an immediate neighbour of the both

group?

persons who like Timex and Tissot Mobile. Person who

A. U – Fastrack

likes Tissot sits third to the right of Q. Q does not like

B. V – LAURELS

Casio. U is not an immediate neighbour of P. V is not an

C. S – Sonata

immediate neighbour of the person who likes Sonata.

D. P – Tissot

Only two people sit between the person who likes

E. Q – Matrix

Matrix and W. The persons who like Casio and the Titan are immediate neighbours of each other.

Answer Answer – E. Q –Matrix

Explanation 8. Who amongst the following likes LAURELS? A. P B. Q C. R D. S E. T Answer 6. Who amongst the following sits third to the Answer – A. P

left of T? A. P

9. Which of the following is true with respect

B. Q

to the given seating arrangement?

C. R

A. U and S are immediate neighbours of

D. S

each other

E. U

B. V sits second to right of S C. The persons who like Timex and Titan are

Answer

immediate neighbours of each other Answer – D. S

D. T sits to the immediate left of the person

7. Four of the following five are alike in a certain way based on the given

who likes Matrix E. None of the given statements is true

GovernmentAdda.com | IBPS SBI RBI SSC FCI RRB RAILWAYS

61

Daily Visit

[GOVERNMENTADDA.COM]

Answer

The person from Delhi is second to the right of the

Answer – D. T sits to the immediate left of the person who likes Matrix.

person who faces C. The immediate neighbour of person from Delhi faces the person from Patna. Only one person sits between person from Patna and B. One

10. Which of the following brands is liked by

who faces B is second to the left of P. One who is

Q?

immediate neighbour of B faces person from Jaipur. A

A. Matrix

person facing the person from Jaipur is second to the

B. Timex

right of the person from Pune. There is only one person

C. Titan

between the person from Jaipur and S. R faces the

D. Tissot

person from Surat. R is neither from Jaipur nor Mumbai.

E. Sonata

The person who is immediate neighbour of D is from Shillong. D is not an immediate neighbour of B. The one

Answer

who likes Economics faces the one who likes Electronics. only one sits between the one who likes

Answer –C. Titan

Commerce and the one who likes Zoology. The one who I. Study the following information carefully to answer

likes Chemistry is an immediate neighbour of one who

the given questions.

like Physics and the one who likes Botany. A likes Electronics. P does not like Neither Commerce nor

Eight persons are sitting in two parallel rows containing

Zoology. The one who likes Commerce is an immediate

four persons each in such a way that there is an equal

neighbor of the person from Pune. The one who likes

distance between adjacent persons. In Row-1, P, Q, R

Zoology faces the one who likes Botany.

and S are seated and all of them are facing south. In Row – 2, A, B, C, and D are seated and all of them are

Explanation

facing north. Therefore in the given seating arrangement each person seated in a row faces another person of the other row. Each of them belongs to different places viz., Delhi, Jaipur, Patna, Pune, Mumbai, Chennai, Shillong and Surat, but not necessarily in the same order. Each of them likes different subjects like Electronics, Commerce, Zoology, Chemistry, Botany, Physics, Biotechnology and Economics.

GovernmentAdda.com | IBPS SBI RBI SSC FCI RRB RAILWAYS

62

Daily Visit

[GOVERNMENTADDA.COM]

Explanation

Answer Answer – A. R is at the extreme right end of the row 3. Four of the five are alike in a certain way based on the above seating arrangement and hence they form the group. Which one of the following does not belong to that group? A. R B. S C. C D. A

1. Which of the following pairs of persons is seated at the extreme ends in any of the

E. B Answer

two rows? Answer – C. C

A. SQ B. PR

4. Which of the following combinations of a

C. AC

person and place is true?

D. BD

A. P – Mumbai – Chemistry

E. RS

B. R – Delhi – Economics C. D – Patna – Commerce

Answer

D. B – Surat – Electronics Answer – E. RS

E. C – Shillong – Physics

2. Which of the following statements is not

Answer

true regarding R? A. R is at the extreme right end of the row B. R faces the person from Surat

5. Who amongst the following likes

C. R is second to the left of P

Economics?

D. There are two persons between R and S E. R is an immediate neighbour of Q.

Answer – C. D – Patna – Commerce

A. S B. Q

GovernmentAdda.com | IBPS SBI RBI SSC FCI RRB RAILWAYS

63

Daily Visit

[GOVERNMENTADDA.COM]

C. R

between the one who likes White and the one who likes

D. A

Purple. Only one person sits between the one who likes

E. B

Purple and Pink. N does not like Violite and K does not like Purple.

Answer

Explanation Explanation

Answer – A. S II. Study the following information carefully to answer the given questions.

Eight persons – J, K, L, M, N, O, P, Q are sitting around a circular table facing the centre with equal distances between each other(but not necessarily in the same order). Each of them is also related to N in some way or the other. Each of them also likes eight different colours namely viz – White, Black, Blue, Brown, Pink, Violite, 6. Who amongst the following is the son of N?

Purple and Green. K sits third to the left of N. Only one person sits

A. Q

between N and Q. N’s sister sits to the immediate right

B. P

of Q. Only two people sit between N’s sister and N’s

C. K

mother. J sits to the immediate right of N’s mother. P

D. J

sits to the immediate right of M. N’s brother sits third to

E. M

the right of P. N’s wife sits second to the left of N’s

Answer

brother. Only three people sit between N’s wife and L. N’s son sits second to the right of N’s father. Only two

Answer – A. Q

people sit between N’s father and N’s daughter. Four people sit between the one who likes Violite and the

7. How many people sit between N and K,

one who likes Brown. The person who likes Violite sits

when counted from the right of K?

immediate right/left of the N’s Sister. The person who

A. Five

likes Brown sits exactly between the one who likes

B. Two

Green and the one who likes Black. Three persons are

C. Four

there between the one who likes Green and the one

D. Three

who likes Blue. The person who likes Violite sits exactly

E. None

GovernmentAdda.com | IBPS SBI RBI SSC FCI RRB RAILWAYS

64

Daily Visit

[GOVERNMENTADDA.COM]

Answer

I. Study the following information carefully to answer the given questions.

Answer – B. Two 8. Who among the following likes Purple?

Ten persons from ten different countries viz.

A. J

Switzerland, Spain, Italy, USA, UK, Australia, New

B. N’s sister

Zealand, Brazil, Canada and Singapore are sitting in two

C. N

parallel rows containing five people each, in such a way

D. N’s Wife

that there is an equal distance between adjacent

E. K

persons. In row 1- A, B, C, D and E are seated and some of them are facing South and some of them are facing

Answer

North. In row 2 – P, Q, R, S and T are seated and some of them are facing South and some of them are facing

Answer – B. N’s sister

North. Therefore in the given seating arrangement, 9. Which of the following statements is true

each member seated in a row either faces another

with respect to the given information?

member of the other row or seated behind each

A. All the given options are true

other.(All the information given above does not

B. P sits to the immediate left of J

necessarily represent the order of seating in the final

C. N’s mother sits to the immediate left of N

arrangement.). Each person stays in ten different floors

D. M is the mother-in-law of Q

numbered 1 to 12.(From Ground Floor to Top floor)

E. N is an immediate neighbour of his father. There is only one floor between the person from Answer

Switzerland and the person from UK. S is not from Italy.

Answer – B. P sits to the immediate left of J

D is neither from UK nor from Australia. P sits immediate right of the person from Canada. R sits one

10. How is J related to L?

of the extreme ends of the line and from Canada. C sits

A. Sister

third to the right of the person from Spain. P does not

B. Uncle

face A and faces south direction. The person from

C. Sister-in-law

Switzerland sits exactly between the persons from

D. Father

Singapore and UK. The person from Australia faces the

E. Daughter

person from Singapore. The person from Canada stays

Answer Answer – C. Sister-in-law

on the odd numbered floor. T faces North Direction and sits immediate left of Q. Only one person sit between the persons from Italy and USA. The person from USA GovernmentAdda.com | IBPS SBI RBI SSC FCI RRB RAILWAYS

65

Daily Visit

[GOVERNMENTADDA.COM]

sits to the immediate right of Q, who seated exactly in

1. Who amongst the following faces the

the middle of the row. P faces one of the immediate

person from Australia?

neighbors of the person from Spain. D faces one of the

A. The person from Switzerland

immediate neighbors of the person from Italy. The

B. D

person from Singapore stays on the top floor. Only One

C. The person from UK

person sits between the person from Canada and Q. C

D. The person from Canada

sits to the immediate right of the person who faces S.

E. B or E

th

The person from Australia stays on the 4 floor. Only two people sit between C and E. S is neither from Switzerland nor from Brazil. The person from UK sits

Answer Answer – B. D

second to the right of the one who faces North Direction. One of the immediate neighbors of the

2. T stays on which of the following floors?

person from UK behind the person from Italy. A faces

A. 1

the opposite direction to the person from New Zealand.

B. 2

The persons from Italy, New Zealand and USA stay on

C. 4

the consecutive floors. The floor number of the person

D. 6

from Spain is the double of the floor number of the

E. 8

person from Canada. The floor number of the B is the

Answer

square of the floor number of P. Neither E nor A stays on floor numbered 6. Explanation

Answer – A. 1 3. Which of the following is true regarding C? A. C faces south direction. B. None of the given options is true C. C is from Bangladesh D. The person from India faces C E. The person from Hungary is an immediate neighbor of C Spoiler title

Explanation-

A. C faces south direction.

GovernmentAdda.com | IBPS SBI RBI SSC FCI RRB RAILWAYS

66

Daily Visit

[GOVERNMENTADDA.COM]

4. R is related to USA in the same way as C is

Characters i.e. Mickey mouse, Bugs Bunny, Tweety,

related to UK based on the given

Donald Duck, Minnie Mouse, Popeye, Jerry,

arrangement, To who amongst the

Tom, Goofy and Sylvester. Each person has different

following is T related to the following same

number of their favourite chocolates – 2, 3, 4, 6, 7, 8, 9,

pattern?

11, 15 and 16.

A. Switzerland B. Sri Nagar

The difference between the chocolates hold by N and O

C. Italy

is 3. G sits third to the right of F and likes Donald Duck.

D. Canada

Only two people sit between E and the vacant seat.

E. Australia

E does not like Minnie Mouse or Tweety. Q is not an immediate neighbour of O. N likes Sylvester. The

Answer

persons who sit at the extreme end of the line have chocolates in consecutive order. Neither E nor H has 8

Answer – E. Australia

chocolate. One of the neighbors of vacant seat in both 5. Who amongst the following sit

rows have chocolates in odd number. The one who likes

at extreme ends of the row?

Tweety faces the one who likes Tom. The one who likes

th

A. The person stays on 8 floor and R

Tweety sits opposite to the one who sits third right of

B. The persons from Brazil and A

the person who sits opposite to G. O is not an

C. D and the person stays on 10th floor

immediate neighbour of P. H, who likes neither Minnie

D. The persons from Australia and Italy

Mouse nor Popeye, does not face the vacant seat.

E. A, E

Neither G nor F sits at any of the extreme ends of the row. P faces F. Vacant seats are not opposite to each

Answer

other. Two seats are there between O and N, who sits Answer – C. D and the person stays on

third right of the one who likes Jerry. The one who likes

10th floor

Goofy faces the one who likes Donald Duck. The persons who like the Mickey mouse and Tom are

II. Study the following information carefully to answer

adjacent to each other. Vacant seat of row – 1 is not an

the given questions.

immediate neighbour of P. E sits at one of the extreme ends of the row. F does not like Mickey mouse and Tom.

Ten friends are sitting in two parallel rows of six seats

Vacant seat of row-1 does not face G who doesn’t sit at

each. One seat is vacant in each row. M, N, O, P and

any of the extreme ends of the row. The person who

Q are sitting in row-1 facing South. D, E, F, G and H are

likes Mickey mouse has 3 chocolates. The total number

facing North. Each likes a different Cartoon

of chocolates hold by Q is the half of the total number GovernmentAdda.com | IBPS SBI RBI SSC FCI RRB RAILWAYS

67

Daily Visit

[GOVERNMENTADDA.COM]

of chocolates hold by H. The total numbers of

7. Who amongst the following sits third to the

chocolates hold by M, F and G is the Square of the total

right of F?

number of chocolates hold by P, Q and M respectively.

A. The one who likes Donald Duck

Neither P nor G has 4 chocolate.

B. E C. Other than those given as options

Explanation Explanation-

D. D E. The one who likes Minnie Mouse Answer Answer – A. The one who likes Donald Duck 8. Which of the following faces the vacant seat of Row – 1? A. The one who likes Donald Duck B. E C. Other than those given as options D. F E. The one who has 15 chocolate Answer

6. In the given arrangement, if two people come and sit to the immediate left of E, how many people will sit between D and E?

Answer – E. The one who has 15 chocolate 9. Four of the following five are alike in a

A. Two

certain way based upon their seating

B. Three

arrangement and so form a group. Which

C. Four

of the following does not belong to the

D. More than four

group?

E. One

A. QE

Answer

B. ND C. HO

Answer – B. Three

D. PG E. FP

GovernmentAdda.com | IBPS SBI RBI SSC FCI RRB RAILWAYS

68

Daily Visit

[GOVERNMENTADDA.COM]

Answer

Explanation Explanation-

Answer – E. FP 10. Who among the following has 11 chocolate? A. Q B. N C. D 1. Which of the following is true, based on the

D. E

given arrangement?

E. None of the above

A. D faces North Answer

B. Only three people face South C. L sits at one of the extreme ends of the

Answer – B. N

line

I. Study the following information carefully to answer

D. O and E face the same directions

the given questions.

E. None of the given options is true Answer

Eight friends C, D, E, F, L, M, N and O are seated in a straight line, but not necessarily in the same order.

Answer – D. O and E face the same

Some of them are facing north while some face South.

directions

Only three people sit to the right of M. E sits second to the left of M. F sits third to the right of O. O is not an

2. How many people sit to the left of O?

immediate neighbour of M. O does not sit at any of the

A. Three

extreme ends of the line. Both the immediate

B. More than four

neighbours of O face south.

C. One

D sits second to the right of N. As many people sit

D. Four

between M and D as between M and L. Immediate

E. Two

neighbours of F face opposite directions(i.e., If one

Answer

person faces north then the other person faces south and vice-versa). C faces south. L and F face direction

Answer – C. One

opposite to C.(i.e If C faces north then both L and F face south and vice-versa)

3. Who amongst the following faces South? A. E GovernmentAdda.com | IBPS SBI RBI SSC FCI RRB RAILWAYS

69

Daily Visit

[GOVERNMENTADDA.COM]

B. M

Eight people B, C, D, E, F, G, H and I are sitting in a

C. F

straight line with equal distances between each other,

D. L

but not necessarily in the same order. Some of them are

E. O

facing North and some of them are facing south.

Answer



E sits immediate right of the person who sits at one of the extreme ends of the line. Only

B. M

three people sit between E and G. B sits 4. Who amongst the following sits second to

exactly between D and H. 

the left of L?

C sits third to the right of H. F is an

A. O

immediate neighbour of G and faces south.

B. F

G sits second to the right of C. D sits third to

C. D

the left of G. B and E face the same direction

D. No one as less than two people sit to the

as C(i.e if C faces north then B and E also

left of L

face North and Vice-Versa). 

E. N

Immediate neighbours of G face opposite directions(i.e. if one neighbour faces North

Answer

then the other neighbour faces south and Vice-Versa)

Answer –B. F 

5. Who amongst the following represent the

Person who sit at the extreme ends of the line face opposite directions(i.e. if one

persons sitting at extreme ends of the line?

neighbour faces North then the other

A. D, N

neighbour faces south and Vice-Versa)

B. C, D



D and H face the same direction as I(i.e if

C. L, N

I faces north then D and H also face North

D. D, L

and Vice-Versa).

E. C, N Explanation Explanation-

Answer Answer – B. C, D II. Study the following information carefully to answer the given questions.

GovernmentAdda.com | IBPS SBI RBI SSC FCI RRB RAILWAYS

70

Daily Visit

[GOVERNMENTADDA.COM]

6. In the given arrangement, how many

9. Four of the following five are alike in a

people will sit between D and G?

certain way based upon their seating

A. Two

arrangement and so form a group. Which

B. Three

of the following does not belong to the

C. Four

group?

D. More than four

A. IH

E. One

B. EG C. DF

Answer

D. EB E. BC

Answer – A. Two 7. Who amongst the following sits third to the right of B?

Answer Answer – D. EB

A. E B. I

10. Who amongst the following sits at

C. Other than those given as options

the extreme right end of the row?

D. F

A. G

E. G

B. C C. I

Answer

D. H E. E

Answer – B. I 8. How many people face South as per the given arrangement?

Answer Answer – B. C

A. Two B. Three

I. Study the following information carefully to answer

C. Four

the given questions.

D. More than four E. One Answer Answer – C. Four

Eight members P, Q, R, S, T, U, V and W of a family are sitting around a rectangular table with all of them facing outwards. Each one of them like different type of sports viz. HOCKEY, Cricket, Basket Ball, Foot Ball, BOXING, WRESTLING, Badminton and Tennis. Three GovernmentAdda.com | IBPS SBI RBI SSC FCI RRB RAILWAYS

71

Daily Visit

[GOVERNMENTADDA.COM]

married couples are there in the family. Each one of

Explanation-

them also like different colors viz, Purple, Brown, Blue, White, Yellow, Red, Orange and Green but not necessarily in the same order.

The person who likes Orange sit between Brown and Yellow. W is the only sister-in-law of P whereas Q likes WRESTLING and daughter-in-law of R. The person who likes brown sit between Purple and Orange. P who is the father of U and uncle of V, sits to the left of the person who likes HOCKEY. U is an immediate neighbor of her aunty W who does not sit next to S. R does not like Tennis or Badminton. The two youngest members sit next to each other. The one likes the Cricket sits between V and the one who likes BOXING. The person who likes white sit between Yellow and Blue. V is third to the left of S. V does not like Brown and Red. The person who likes brown sit between Purple and Orange. The one who likes WRESTLING sits between 1. Who among the following likes White?

the persons who like Badminton and Tennis Respectively. The person who likes white does not sit

A. Q

at any corner in the rectangle. S’s husband and son sit

B. U

next to her. Three persons sit between the one who

C. V

likes white and the one who likes Wrestling. Neither S

D. X

nor U like White. Foot Ball is not liked by V’s father. V

E. T

does not like Basket Ball or Badminton. S is the mother

Answer

of P and T, and sits second to the left of T. Two persons sit between the one who likes Brown and the one who

Answer – E. T

likes White. 2. Who among the following sits between Q Explanation

and the one who likes Hockey? A. P B. T GovernmentAdda.com | IBPS SBI RBI SSC FCI RRB RAILWAYS

72

Daily Visit

[GOVERNMENTADDA.COM]

C. S

3. W, R

D. V

4. S, U

E. W

5. None of the above

Answer

Answer

Answer – A. P

Answer – 2. W, T

3. What is the position of the person who

II. Study the following information carefully to answer

likes Foot Ball with respect to the one who

the given questions.

likes Tennis?

Ten persons from ten different cities viz. Delhi, Jaipur,

A. Third to the right

Patna, Indore, Mangaluru, Chennai, Hyderabad,

B. Second to the left

Bengaluru, Raipur and Sri Nagar are sitting in two

C. Immediate left

parallel rows containing five people each, in such a

D. Third to the left

way that there is an equal distance between adjacent

E. Fourth to the right

persons. In row 1- A, B, C, D and E are seated and some of them are facing South and some of them are facing

Answer

North. In row 2 – P, Q, R, S and T are seated and some of them are facing South and some of them are facing

Answer – E. Fourth to the right

North. Therefore in the given seating arrangement, 4. Who among the following likes Wrestling?

each member seated in a row either faces another

A. Q

member of the other row or seated behind each

B. U

other.(All the information given above does not

C. V

necessarily represent the order of seating in the final

D. X

arrangement.) Each one of them like different

E. T

colors viz, Grey, Brown, Black, Blue, White, Yellow, Red, Pink, Orange and Green but not necessarily in the

Answer

same order. Answer – A. Q The person from Indore sits to the immediate right of 5. Which of the following options represent a

Q, who seated exactly in the middle of the row. P faces

pair?

one of the immediate neighbors of the person from

1. Y, X

Jaipur. D faces one of the immediate neighbors of the

2. W, T

person from Patna. S is not from Patna. D is neither GovernmentAdda.com | IBPS SBI RBI SSC FCI RRB RAILWAYS

73

Daily Visit

[GOVERNMENTADDA.COM]

from Mangaluru nor from Chennai. P sits immediate

Explanation-

right of the person from Raipur. R sits one of the extreme ends of the line and from Raipur. C sits third to the right of person from Jaipur. Only One person sits between the person from Raipur and Q. C sits to the immediate right of the person who faces S. Only two people sit between C and E. S is neither from Delhi nor from Bengaluru. The person from Mangaluru sits second to the right of the one who faces North Direction. One of the immediate neighbors of the person from Mangaluru behind the person from Patna. P does not face A and faces south direction. The person from Delhi sits exactly between the persons from Sri Nagar and Mangaluru. The person from Chennai faces the person from Sri Nagar. T faces North Direction and sits immediate left of Q. Only one person sit between the persons from Patna and Indore. A faces the opposite direction to the person from Hyderabad. Two persons sit between the one who likes Brown and the one who likes Grey. Three persons sit between the one who likes Red and the one who likes Green. Neither s nor R like Red. D does

6. Who amongst the following faces the person from Indore? A. The person from Delhi B. D C. The person from Mangaluru D. The person from Raipur E. B Answer

not like Green. R and S does not like Black. The person who likes Brown sits between the person who likes White and Black. The person who likes Pink sit

7. Q is from which of the following Cities?

between Blue and Orange. C does not like orange. Explanation

Answer – E. B

A. Patna B. Indore C. Hyderabad D. Raipur E. Mangaluru Answer

GovernmentAdda.com | IBPS SBI RBI SSC FCI RRB RAILWAYS

74

Daily Visit

[GOVERNMENTADDA.COM] Answer

Answer – C. Hyderabad 8. Which of the following is true regarding C? A. C faces south direction

Answer – D. S I. Study the following information carefully to answer

B. None of the given options is true

the given questions.

C. C is from Bengaluru D. The person from Indore faces C

Ten persons – P, Q, R, S, T, U, V, W, X and Y are sitting in

E. The person from Hyderabad is an

two rows with five persons in each row. In Row – 1,

immediate neighbor of C

the persons are facing south and in Row – 2, the

Answer

persons are facing north. Each person in row one faces a person from the other row. All of them have a car of

Answer – A. C faces south direction

different companies, viz Honda, Hyundai, Ford, Fiat,

9. R is related to Indore in the same way as C is related to Mangaluru based on the given arrangement, To who amongst the following is T related to the following same

Chevrolet, Audi, BMW, Toyota, Renault, and Maruti but not necessarily in the same order. Each person likes different colors namely viz – White, Orange, Pink, Purple, Black, Blue, Yellow, Brown, Red and Green.

pattern? The person who likes Red faces the person who owns

A. Delhi

Renault. Y does not like Pink. The person who likes

B. Sri Nagar

White stands exactly in the middle between the persons

C. Patna

who like Yellow and Orange. T does not like Brown.

D. Raipur

There is only one person stands between Q and R. The

E. Chennai

persons who like Chevrolet and Audi sit opposite each

Answer

other. U sits opposite to P, who owns Honda.The one who owns Hyundai sits opposite the one who owns

Answer – E. Chennai

Toyota. Y is not facing the north but sits third to the left

10. Who among the following likes White? A. Q B. U C. V D. S E. T

of V, who owns Hyundai. The one who likes Fiat sits opposite to the one who is second to the right of Q. X does not like Maruti. The person who likes Green facing south direction and stands opposite to the person who likes Yellow. T sits at one of the ends of the row and owns Audi. The one who owns Toyota is on the immediate right of S, who does not own Maruti. The

GovernmentAdda.com | IBPS SBI RBI SSC FCI RRB RAILWAYS

75

Daily Visit

[GOVERNMENTADDA.COM]

person who likes Blue stands between the persons who

C. Three

like pink and Black respectively. The persons who own

D. Can’t be determined

Ford and Fiat are not facing the north direction. R owns

E. None of the above

BMW car. T sits opposite to the one who sits second to the left of the person who owns Ford. There are two persons stand between the persons who like Red and

Answer Answer – B. Two

Blue. 3. Which of the following statements is/are Explanation

true? A. W owns Maruti and sits at one of the ends of the row. B. U sits on the immediate left of the one who likes Green. C. The one who owns Renault sits on the immediate left of the one who likes Red. D. Only (A) and (B) E. Only (B) and (C)

Explanation-

Answer

1. Q owns which of the following brand of car? A. Renault

Answer – D. Only (A) and (B) 4. Four of the following five are alike in a

B. Toyota

certain way and hence form a group. Which

C. Chevrolet

one of the following does not belong to

D. BMW

that group?

E. None of the above

A. W, S

Answer

B. U, P C. V, P

Answer – B. Toyota

D. X, R E. V, Q

2. How many persons sit between W and X? A. One B. Two

Answer Answer – C. V, P GovernmentAdda.com | IBPS SBI RBI SSC FCI RRB RAILWAYS

76

Daily Visit

[GOVERNMENTADDA.COM]

5. Who among the following likes Orange

exactly in the middle between the persons who like

Colour?

Batman and Avatar. The Manager sits second to the left

A. X

of E. The Manager is an immediate neighbor of the both

B. R

Group Leader and the Financial Advisor. The Financial

C. W

Advisor sits third to the right of B. B is not the Vice –

D. S

President. F is not an immediate neighbor of A. G is not

E. None of the above

an immediate neighbor of the Manager. Only two people sit between the Managing Director and H. The

Answer

Vice – President and the Company Secretary are immediate neighbors of each other. Financial Advisor

Answer – B. R

faces the person who likes Finding Nemo. II. Study the following information carefully to answer Explanation

the given questions.

Eight colleagues A, B, C, D, E, F, G and H are sitting around a circular table facing the centre. Each one of them holds a different post viz. Manager, Company Secretary, Chairman, President, Vice – President, Group Leader, Financial Advisor and Managing Director but not necessarily in the same order. Each person likes different movies namely viz – Spider Man, Titanic, Batman, Super Man, Finding Nemo, Jurrasic Park, Avengers and Avatar. 6. Who amongst the following sits Second to President faces the person who likes Spider-Man. B does not like Titanic. A sits third to the right of the Managing Director. A is not the Chairman. The person who likes Avengers sits immediately right to the person who likes Avatar. E is not an immediate neighbor of H. C sits on the immediate right of the Chairman. Neither A nor H is a Vice – President or a Company Secretary. Vice

the left of E? A. The Manager B. G C. A D. The Financial Advisor E. B Answer

–President is not an immediate neighbor of the Managing Director. The person who likes Superman sits

Answer – A. The Manager

GovernmentAdda.com | IBPS SBI RBI SSC FCI RRB RAILWAYS

77

Daily Visit

[GOVERNMENTADDA.COM]

7. Four of the following five are alike in a

D. E sits to the immediate left of the

certain way based on the given

Managing Director

arrangement and thus form a group. Which

E. The Group Leader sits second to the left

is the one that does not belong to that

of the person who likes Super Man

group?

Answer

A. F – Chairman B. G – President

Answer – D. E sits to the immediate left of

C. D – Manager

the Managing Director

D. A – Financial Advisor 10. Who among the following likes Titanic?

E. B – Managing Director

A. Chairman

Answer

B. Manager C. Company Secretary

Answer – E. B – Managing Director

D. Vice – President 8. Who amongst the following is the Vice – President of the company?

E. Financial Advisor Answer

A. A B. C

Answer – D. Vice – President

C. H D. G

I. Study the following information carefully to answer

E. D

the given questions. There are 16 persons – B,C,D,E,F,G,H,I,P,Q,R,S,T,U,V and

Answer

W standing in a square plot. Inside a Square plot, a square shaped garden is developed. The persons who

Answer – D. G

are standing inside the garden facing outside. The 9. Which of the following is true with respect

persons who are standing outside the garden facing

to the given seating arrangement?

inside the centre and likes colours namely viz., – Red,

A. The Group Leader of the company is an

Blue, Black, Brown, Yellow, Green, Violate and Pink. So

immediate neighbour of the Vice – President

all the persons standing in the inner square faces the

B. G sits second to right of D

persons standing in the outer Square and likes fruits

C. The Group Leader and the Company

namely viz., – Apple, Orange, Mango, Grapes, Papaya,

Secretary are immediate neighbours of each

Pomegranate, Guava and Banana.

other GovernmentAdda.com | IBPS SBI RBI SSC FCI RRB RAILWAYS

78

Daily Visit

[GOVERNMENTADDA.COM]

G faces the centre and W faces G. D sits second to the

Explanation-

right of G. There are four persons sits between G and E. D is not an immediate neighbour of E. There are three persons standing between I and E. There are two persons standing between I and B. B stands exactly between the E and F. F stands to the immediate left of G. There are two persons standing between W and U. U faces H. T faces outside. There are two persons standing between T and Q. T faces C. Q stands to the immediate left of W. R, the one who faces B stands exactly between the persons P and V. P faces E. The one who sits in the corner of the square likes Red. The one who likes Red sits between the persons who like Black and Blue. The one who likes Blue sits second to the right of the person who likes Green. Three persons sit between

1. In the given arrangement, if three people

one who likes Black and one who likes Green. Two

come and stand to the immediate left of E,

persons sit between one who likes Black and one who

how many people will sit between F (From

likes Yellow. Two persons sit between one who likes

the left of F) and C?

Yellow and one who likes Pink. G and F do not like

A. Two

Violate and Yellow respectively. The one who likes Red

B. Three

faces P. The immediate neighbours of P are the one

C. Five

who likes apple and the one who likes Grapes. The one

D. More than four

who likes Apple faces the one who likes Black. Three

E. One

persons sit between the one who likes apple and the

Answer

one who likes Guava. The immediate neighbours of the person who likes Orange are the one who likes apple

Answer – C. Five

and the one who likes Pomegranate. The one who likes 2. Who amongst the following likes Green?

Papaya sits exactly behind to the one who likes Orange. The one who likes Banana sits exactly behind to

A. C

the one who likes Mango. The one who likes Banana

B. B

faces E.

C. Other than those given as options D. D

Explanation

E. E GovernmentAdda.com | IBPS SBI RBI SSC FCI RRB RAILWAYS

79

Daily Visit

[GOVERNMENTADDA.COM]

Answer

Answer

Answer – D. D

Answer – B. U

3. How many people stand between V and U? A. Two

II. Study the following information carefully to answer the given questions.

B. Three C. Four

There are 16 persons – B,C,D,E,F,G,H,I,P,Q,R,S,T,U,V and

D. More than four

W standing in a Circular plot. Inside a circular plot, a

E. One

circularly shaped garden is developed. The persons who are standing inside the garden facing outside. The

Answer

persons who are standing outside the garden facing inside the centre and lives in a different number of

Answer – B. Three

floors. So all the persons standing in the inner circle 4. Four of the following five are alike in a certain way based upon their arrangement

faces the persons standing in the outer circle and hold a different number of chocolates.

and so form a group. Which of the following does not belong to the group?

G faces outside and S faces G. D sits immediate right of

A. EP

G. There are four persons sits between G and E. H is not

B. FV

an immediate neighbour of E. There are two persons

C. UH

standing between D and H. H faces R. There are three

D. GW

persons standing between R and U. U stands exactly

E. IQ

between the B and F. B faces D. There are two persons standing between P and C. Neither S nor R is an

Answer

immediate neighbour of P. I stands to the immediate left of H. I faces T. The one who faces F stands exactly

Answer – D. GW

between the persons Q and W. W faces P. H stands 5. Who amongst the following likes Papaya?

second to the left of G. B lives on the second floor and

A. P

sits exactly opposite to the person who lives on the

B. U

floor which is the square number of the floor of B. F

C. Q

lives on the third floor and stands exactly opposite to

D. E

the person who lives on the floor which is the square

E. Other than those given as options

number of the floor of F. P lives on 6th floor and S lives immediately above P. U lives immediately below B. R GovernmentAdda.com | IBPS SBI RBI SSC FCI RRB RAILWAYS

80

Daily Visit

[GOVERNMENTADDA.COM]

lives immediately above T. The one who faces P holds chocolates two less than the number of the floor occupied by P. The one who faces U holds chocolates six more than the number of the floor occupied by U.

Answer Answer – B. Three 7. Who amongst the following lives on the

Number of chocolates hold by E is the difference

seventh floor?

between the number of chocolates hold by D and W.

A. S

Number of chocolates hold by G is the sum of the

B. Q

number of chocolates hold by D and E also equals to

C. Other than those given as options

number of chocolates hold by V and H. Number of

D. U

chocolates hold by I is the square of the number of

E. C

chocolates hold by H. Answer Explanation Explanation-

Answer – A. S 8. If persons counted from the right of G, then how many people stand between G and E as per the given arrangement? A. Five B. Three C. Four D. More than four E. Two Answer Answer – E. Two

6. In the given arrangement, how many

9. Four of the following five are alike in a

people will sit between B and T?

certain way based upon their seating

A. Two

arrangement and so form a group. Which

B. Three

of the following does not belong to the

C. Four

group?

D. More than four

A. F

E. One

B. E GovernmentAdda.com | IBPS SBI RBI SSC FCI RRB RAILWAYS

81

Daily Visit

[GOVERNMENTADDA.COM]

C. C

persons standing between I and B. B stands exactly

D. G

between the E and F. F stands to the immediate left of

E. V

G. There are two persons standing between W and U. U faces H. T faces outside. There are two persons standing

Answer

between T and Q. T faces C. Q stands to the immediate left of W. R, the one who faces B stands exactly

Answer – D. G

between the persons P and V. P faces E. 10. Who amongst the following have seven Explanation Explanation-

chocolates? A. T B. F C. Q D. P E. S Answer Answer – C. Q I. Study the following information carefully to answer the given questions. There are 16 persons – B,C,D,E,F,G,H,I,P,Q,R,S,T,U,V and W standing in a square plot. Inside a Square plot, a square shaped garden is developed. The persons who

1. In the given arrangement, if three people

are standing inside the garden facing outside. The

come and stand to the immediate left of E,

persons who are standing outside the garden facing

how many people will sit between F and C?

inside the centre. So all the persons standing in the

A. Two

inner square faces the persons standing in the outer

B. Three

Square.

C. Four D. More than four

G faces the centre and W faces G. D sits second to the

E. One

right of G. There are four persons sits between G and E.

Answer

D is not an immediate neighbour of E. There are three persons standing between I and E. There are two

Answer – C. Four

GovernmentAdda.com | IBPS SBI RBI SSC FCI RRB RAILWAYS

82

Daily Visit

[GOVERNMENTADDA.COM]

2. Who amongst the following faces S?

5. Who amongst the following sits at the

A. C

corner of the outer square?

B. B

A. P

C. Other than those given as options

B. U

D. D

C. Q

E. E

D. E E. Other than those given as options

Answer

Answer Answer – D. D Answer – D. E 3. How many people stand between V and U? A. Two

II. Study the following information carefully to answer

B. Three

the given questions.

C. Four D. More than four

There are 16 persons – B,C,D,E,F,G,H,I,P,Q,R,S,T,U,V and

E. One

W standing in a Circular plot. Inside a circular plot, a circularly shaped garden is developed. The persons who

Answer

are standing inside the garden facing outside. The persons who are standing outside the garden facing

Answer – B. Three

inside the centre. So all the persons standing in the 4. Four of the following five are alike in a certain way based upon their arrangement

inner circle faces the persons standing in the outer circle.

and so form a group. Which of the following does not belong to the group?

G faces outside and S faces G. D sits immediate right of

A. EP

G. There are four persons sits between G and E. H is not

B. FV

an immediate neighbour of E. There are two persons

C. UH

standing between D and H. H faces R. There are three

D. GW

persons standing between R and U. U stands exactly

E. IQ

between the B and F. B faces D. There are two persons

Answer

standing between P and C. Neither S nor R is an immediate neighbour of P. I stands to the immediate

Answer – D. GW

left of H. I faces T. The one who faces F stands exactly

GovernmentAdda.com | IBPS SBI RBI SSC FCI RRB RAILWAYS

83

Daily Visit

[GOVERNMENTADDA.COM]

between the persons Q and W. W faces P. H stands

D. U

second to the left of G.

E. C Answer

Explanation Explanation-

Answer – A. F 8. If persons counted from the right of G, then how many people stand between G and E as per the given arrangement? A. Five B. Three C. Four D. More than four E. Two Answer

6. In the given arrangement, how many people will sit between B and T?

Answer – E. Two 9. Four of the following five are alike in a

A. Two

certain way based upon their seating

B. Three

arrangement and so form a group. Which

C. Four

of the following does not belong to the

D. More than four

group?

E. One

A. F B. E

Answer

C. C Answer – B. Three

D. G E. V

7. Who amongst the following sits second to the right of T? A. F B. Q

Answer Answer – D. G

C. Other than those given as options GovernmentAdda.com | IBPS SBI RBI SSC FCI RRB RAILWAYS

84

Daily Visit

[GOVERNMENTADDA.COM]

10. Who amongst the following stands in the

number of Q is greater than 7 and is the sum of the

Inner Circle?

lucky numbers of O and N. Both the immediate

A. T

neighbors of Q face south. The lucky number of M is the

B. F

square of the lucky number of S.

C. Q Explanation Explanation-

D. P E. S Answer Answer – C. Q I. Study the following information carefully to answer the given questions.

1. In the given arrangement, if two people

Eight people M, N, O, P, Q, R, S and T are sitting in a straight line with equal distances between each other, but not necessarily in the same order. Some of them are facing North and some of them are facing south. Each person has different lucky numbers starting from 1 to 9.

come and sit to the immediate left of Q, how many people will sit between R and M? A. Two B. Three C. Four

T and R have minimum and maximum lucky numbers respectively. M sits at one of the extreme ends of the

D. More than four E. One

line. The lucky numbers of P and Q are multiple of 3 and 2 respectively. Only three people sit between M and S.

Answer

Q sits exactly between M and S. T sits third to the right of Q. The lucky number of N is greater than 3 and is the difference between the lucky numbers of P and T. N is an immediate neighbor of T and faces south. O sits second to the right of R. O is not an immediate neighbor of S. Immediate neighbor of S face opposite

Answer – C. Four 2. Who amongst the following sits third to the left of R? A. M B. Q

directions(i.e. if one neighbor faces North then the other neighbor faces south and Vice-Versa). M and P face the same direction as Q(i.e if Q faces north then M

C. Other than those given as options D. N E. S

and P also face North and Vice-Versa). The lucky GovernmentAdda.com | IBPS SBI RBI SSC FCI RRB RAILWAYS

85

Daily Visit

[GOVERNMENTADDA.COM]

Answer

D. 6 E. 9

Answer – D. N

Answer 3. How many people face South as per the given arrangement?

Answer – D. 6

A. Two II. Study the following information carefully to answer

B. Three

the given questions.

C. Four D. More than four

Ten persons are sitting in two parallel rows containing

E. One

five persons each in such a way that there is equal Answer

distance between adjacent persons. In row – 1, D, E, F, G and H are seated and all of them are facing south but

Answer – B. Three

not necessarily in the same order. In row – 2, T, U, V,

4. Four of the following five are alike in a

W and X are seated and all of them are facing north but

certain way based on their seating

not necessarily in the same order. Therefore in the

arrangement and so form a group. Which

given seating arrangement, each member seated in a

of the following does not belong to the

row faces another member of the other row. Each

group?

person likes different colours namely viz – Red, Yellow,

A. QO

Green, Purple, Pink, Orange, White, Brown, Blue and

B. MR

Black.

C. NR D. OS

W faces the one of the immediate neighbours of the

E. PS

person who likes Orange. V sits exactly in the middle of the row. The one who likes Blue faces the one who likes

Answer

Red. The one who faces V sits to the immediate left of F. U faces the one of the immediate neighbours of the

Answer – A. QO

person who likes Brown. H is an immediate neighbour 5. Which of the following is the lucky number

of F but does not face V. There are two people sit

of P?

between the one who likes Green and the one who

A. 3

likes Purple. W sits second to the left of U. U faces the

B. 2

one who is an immediate neighbour of D. There are two

C. 4

people sit between the one who likes Pink and the one GovernmentAdda.com | IBPS SBI RBI SSC FCI RRB RAILWAYS

86

Daily Visit

[GOVERNMENTADDA.COM]

who likes Brown. G is not an immediate neighbour of D.

B. Only two persons sit to the left of T

G does not sit at the extreme end of the row. The one

C. T sits to the immediate left of W

who likes orange sits in the middle between the one

D. U sits fourth to the right of T

who likes Pink and the one who likes White. X does not

E. None of the given statements is true.

face H. The one who likes Pink faces the one who likes Green. There are two people sit between the one who likes Red and the one who likes Black.

Answer Answer – C. T sits to the immediate left of W

Explanation

8. What is the position of D with respect to the one who likes Pink? A. Third to the Right B. Second to the Right C. Immediate left D. Third to the left E. Second to the left Answer Explanation

Answer – D.Third to the left

6. Who amongst the following is facing V?

9. Four of the following five are alike in a

A. The one who likes Orange

certain way based on the given seating

B. E

arrangement and hence they form the

C. The one who likes Pink

group. Which of the following does not

D. D

belong to that group?

E. Can not be determined

A. H B. T

Answer

C. X Answer – A. The one who likes Orange

D. D E. U

7. Which of the following statements is true regarding T? A. T faces the one who likes Pink

Answer Answer – E. U GovernmentAdda.com | IBPS SBI RBI SSC FCI RRB RAILWAYS

87

Daily Visit

[GOVERNMENTADDA.COM]

10. Who amongst the following likes Green? A. T B. W

Answer Answer – B. W

C. V D. X E. U

GovernmentAdda.com | IBPS SBI RBI SSC FCI RRB RAILWAYS

88

Daily Visit :

[GOVERNMENTADDA.COM]

100+ Logical Reasoning Questions With Solution GovernmentAdda.com Directions (1-4): Read the following information carefully and answer the questions that follow:

GovernmentAdda.com | IBPS SBI RBI RRB SSC FCI RAILWAYS

1

Daily Visit :

[GOVERNMENTADDA.COM]

Fortunately, more and more countries are shifting their focus away from industrial development to control of climate change these days. A. The countries which focus more on controlling climate change than industrial development are only the richer ones which can afford to concentrate on areas other than industrial development. B. Many countries had once prioritized industrial development which proved to be harmful to the environment in the longer run. C. Some experts are of the view that climate change is not as alarming an issue as it is made to be because it is a natural phenomenon and has been occurring regularly throughout the history of earth D. If climate change continues at the present rate, it would bring in large scale destruction to human habitation in a very short time E. Industrial development is one of the biggest but definitely not the only reason behind global warming. 1. Which of the statements numbered (A), (B), (C), (D) and (E) can be an assumed/inferred from the facts/information given in the statement? (An assumption is something supposed or taken for granted and an inference is something which is not directly stated but can be inferred from the given facts.) A) E B) C C) A D) B E) Either D or A View Answer

Option D Explanation: Statement says: more and more countries are shifting their focus away from industrial development So it can be assumed that Many countries once prioritized industrial development and now it is proving to be harmful. 2. Which of the statements numbered A, B, C and D mentioned above would weaken the argument for the need of a shift away from industrial development to that of controlling climate changes? A) A B) B C) C D) D E) Both B and D View Answer

Option C Explanation: Countries are shifting from industrial development to control of climate, means to save environment they are shifting. SO option C weakens it which says that climate change is not as alarming an issue and it is a natural phenomena which will keep on continuing. GovernmentAdda.com | IBPS SBI RBI RRB SSC FCI RAILWAYS

2

Daily Visit :

[GOVERNMENTADDA.COM]

3. Which of the following can be inferred from statement (E) if it is considered to be true with regard to the given information? A) Nations also need to focus on sources other than those generated due to industrial development. B) Other sources of pollution have more adverse effects as compared to those generated due to industrial development. C) Unlike older times, industrial development has ceased to be a reason behind global warming these days. D) If Industrial Development stops, global warming would automatically come to an end. E) If sources other than Industrial Development are identified and controlled global warming will end completely. View Answer

Option D Explanation: Statement E. Industrial development is one of the biggest but definitely not the only reason behind global warming. Option A – seems to be true. Option B is not talked about in statement E. Option C is true but it cannot be inferred from E. Option D – Since Industrial development is one of the biggest reason – so if it stops, positive results can be seen definitely. D is more apt as an interference. Option E – Industrial Development is one of the biggest reasons, so if other sources are controlled then also, the biggest reason will cause negative impacts. 4. Which of the statements numbered A, C, D and E mentioned above represents the reason behind curtailing industrial development by some of the countries? A) Either C or E B) D C) C D) Either A or C E) A View Answer

Option B Explanation: Statement D says climate change in future would bring in large scale destruction to human habitation in a very short time, so this is a reason that countries are shifting from or are curtailing industrial development. Directions (5-8): Read the following information carefully and answer the questions that follow: A host of foreign companies are in talks with the Indian government for selling B150, a tough, shorthaul plane ideal for connectivity of smaller towns which is lacking in India at present. A. B150 planes have not only low operating costs than competing planes like Cezana but also a much better track record in terms of safety and efficiency. GovernmentAdda.com | IBPS SBI RBI RRB SSC FCI RAILWAYS

3

Daily Visit :

[GOVERNMENTADDA.COM]

B. The profit margin of road transport operators in the smaller towns connected by B150 planes has been reduced substantially as a majority of people prefer air transport over other means of transport. C. Smaller towns, at present, are better connected by roads and railways as compared to flight services. D. B150 planes are capable of operating in sectors where large airlines cannot fly due to challenging conditions such as mist, short runways, etc. Such planes can also double up as cargo planes and charter flights for the rich and the elite. E. B150 planes need to operate in the existing airports which are situated in bigger cities only and are poorly connected to the smaller cities. 5. Which of the statements (A), (B), (C), (D) and (E) can be inferred from the facts/information given in the statement? (An inference is something which is not directly stated but can be inferred from the given facts.) A) Only A B) Only B C) Only C D) Both B and D E) Only E View Answer

Option D 6. Which of the statements (A), (B), (C), (D) and (E) mentioned above would weaken the offer made by the foreign companies for selling B150 planes to Indian government? A) A B) B C) C D) D E) E View Answer

Option C 7. Which of the statements (A), (B), (C), (D) and (E) mentioned above represents a possible consequence of the success of B150 planes in smaller cities? A) A B) B C) C D) D E) E View Answer

GovernmentAdda.com | IBPS SBI RBI RRB SSC FCI RAILWAYS

4

Daily Visit :

[GOVERNMENTADDA.COM]

Option D 8. Which of the statements (A), (B), (C), (D) and (E) would favour the foreign companies‘ bid to sell B150 planes in India? A) Only A B) Only B C) Both B and C D) Both A and D E) Both E and C View Answer

Option D Directions (9-10): Read the following information carefully and answer the questions that follow: During the past decade of economic reforms, the Indian economy has undergone far-reaching changes like the emergence of India as an IT superpower, a phenomenal growth in global careers for Indian managers upwardly spiralling salaries and so on. But there has also been corporate bloodshed, closure of firms and drastic organisational downsizing. A culture of ‗hire and fire‘ has seeged into the corporate world. The overall job market has shrunk substantially. This could be a boon in disguise as the employed youth would be on their toes to face the challenge and complete successfully, as the adversities will make people more alert and wiser. 9. Which of the following is an assumption which is implicit in the facts stated in the above paragraph? A) The Economic reforms are beneficial to selective ones. B) The devastated job market scenario is not likely to become lively in the near future C) Indian economy has emerged as the most powerful economy in the world D) India needs to review its economic policy E) None of these View Answer

Option E 10. Which of the following is a positive outcome on the basis of the facts stated in the paragraph A) The Economic reforms will continue for some more time, say a decade or so. B) India‘s emergence as an IT superpower is attributable to economic reforms. C) The silver lining of economic reforms is not free from the darkness of the cloud. D) A culture of ‗hire and fire‘ has seeped into the corporate world due to economic reforms. E) None of these. View Answer

GovernmentAdda.com | IBPS SBI RBI RRB SSC FCI RAILWAYS

5

Daily Visit :

[GOVERNMENTADDA.COM]

Option B Directions: Below in each question are given two statements I and II. These statements may be either independent causes or may be effects of independent causes or a common cause. One of these statements may be the effect of the other statement. Read both the statements and decide which of the following answer choices correctly depicts the relationship between these two statements. 1. Statements: I. Railways has increased the number of trains between Bhopal and Chennai II. Railways has increased the frequency of popular trains between Bhopal and Chennai A) If statement I is the cause and statement II is its effect. B) If statement II is the cause and statement I is its effect. C) If both the statements I and II are independent causes. D) If both the statements I and II are effects of independent causes. E) If both the statements I and II are effects of some common causes. View Answer

Option E 2. Statements: I. The bank Association conducted a combined entrance exam for all banks this year. II. As the dates of exam for many banks clashed last year, many candidates complained that they could not appear for a number of exams A) If statement I is the cause and statement II is its effect. B) If statement II is the cause and statement I is its effect. C) If both the statements I and II are independent causes. D) If both the statements I and II are effects of independent causes. E) If both the statements I and II are effects of some common causes. View Answer

Option B 3. Statements: I. The shop owner put cameras for surveillance of the store II. Many items were reported missing by the staff working in the departmental store A) If statement I is the cause and statement II is its effect. B) If statement II is the cause and statement I is its effect. C) If both the statements I and II are independent causes. D) If both the statements I and II are effects of independent causes. E) If both the statements I and II are effects of some common causes. View Answer

GovernmentAdda.com | IBPS SBI RBI RRB SSC FCI RAILWAYS

6

Daily Visit :

[GOVERNMENTADDA.COM]

Option B 4. Statements: I. The university Authority has instructed all the colleges under its jurisdiction to ban use of all phones inside the college premises II. Majority of teachers of the colleges signed a joint petition to the university complaining the disturbance caused by cell phone ringtones inside the classrooms A) If statement I is the cause and statement II is its effect. B) If statement II is the cause and statement I is its effect. C) If both the statements I and II are independent causes. D) If both the statements I and II are effects of independent causes. E) If both the statements I and II are effects of some common causes. View Answer

Option B 5. Statements: I. The meteorological department has issued a statement mentioning deficient rainfall during monsoon in many parts of the country. II. The government has lowered the revised estimate GDP growth from the level of earlier estimates A) If statement I is the cause and statement II is its effect. B) If statement II is the cause and statement I is its effect. C) If both the statements I and II are independent causes. D) If both the statements I and II are effects of independent causes. E) If both the statements I and II are effects of some common causes. View Answer

Option A 6. Statements: I. The government has imported large quantity of onions as per the trade agreement with other countries II. The price of onion in the domestic market has fallen sharply in the recent months A) If statement I is the cause and statement II is its effect. B) If statement II is the cause and statement I is its effect. C) If both the statements I and II are independent causes. D) If both the statements I and II are effects of independent causes. E) If both the statements I and II are effects of some common causes. View Answer

GovernmentAdda.com | IBPS SBI RBI RRB SSC FCI RAILWAYS

7

Daily Visit :

[GOVERNMENTADDA.COM]

Option A 7. Statements: I. Delhi is very prone to earthquakes II. According to the seismologists, there is a lot of tectonic activity going on in the belt below Delhi. A) If statement I is the cause and statement II is its effect. B) If statement II is the cause and statement I is its effect. C) If both the statements I and II are independent causes. D) If both the statements I and II are effects of independent causes. E) If both the statements I and II are effects of some common causes. View Answer

Option B 8. Statements: I. The government has decided to make all the information related to Malegaon blasts available to the public. II. In the past, the public did not have access to all the information related to Malegaon blasts. A) If statement I is the cause and statement II is its effect. B) If statement II is the cause and statement I is its effect. C) If both the statements I and II are independent causes. D) If both the statements I and II are effects of independent causes. E) If both the statements I and II are effects of some common causes. View Answer

Option D 9. Statements: I. The Railway employees called off the strike for demanding a Diwali bonus. II. The Railway employees went on strike anticipating a threat to their jobs. A) If statement I is the cause and statement II is its effect. B) If statement II is the cause and statement I is its effect. C) If both the statements I and II are independent causes. D) If both the statements I and II are effects of independent causes. E) If both the statements I and II are effects of some common causes. View Answer

Option D

GovernmentAdda.com | IBPS SBI RBI RRB SSC FCI RAILWAYS

8

Daily Visit :

[GOVERNMENTADDA.COM]

10. Statements: I. The mobile phones of company X are very costly as compared to other mobile phones of the same quality. II. The mobile phones of company X have not been able to penetrate much in the market. A) If statement I is the cause and statement II is its effect. B) If statement II is the cause and statement I is its effect. C) If both the statements I and II are independent causes. D) If both the statements I and II are effects of independent causes. E) If both the statements I and II are effects of some common causes. View Answer

Option A

1. A fashion designer‘s fall line for women utilizing new soft fabrics broke all sales records last year. To capitalize on her success, the designer plans to launch a line of clothing for men this year that makes use of the same new soft fabrics. The designer’s plan assumes that A) other designers are not planning to introduce new lines for men utilizing the same soft fabrics(B) men will be as interested in the new soft fabrics as women were the year before C) the designer will have time to develop new lines for both men and women D) the line for men will be considered innovative and daring because of its use of fabrics E) women who bought the new line last year will continue to buy it this year View Answer

Option B Explanation: B) — the fashion designed will be using the same new soft fabrics that he used for women last and expects men will be interested in the same. 2. Given that employees of the XYZ Company could, in theory, do their work at home, the company developed a radical plan to increase efficiency: eliminate office-space expenditures by having employees work at home.To evaluate this plan, XYZ‘s managers asked volunteers from the company‘s staff to try the arrangement for six months. There were several volunteers; significantly, their productivity during this period was as high as or higher than before. Which of the following, if true, would argue most strongly against deciding, on the basis of the trial results, to implement the company’s plan? A) The employees who agreed to participate in the test of the plan were among the company‘s most self-motivated and independent workers. B) The savings that would accrue from reduced office-space expenditures alone would be sufficient to justify implementation of the plan apart from any productivity increases. C) Other companies that have achieved successful results from work-at-home plans have work forces that are substantially larger than that of XYZ. D) The volunteers who worked at home were able to communicate with other employees as necessary for performing the work. GovernmentAdda.com | IBPS SBI RBI RRB SSC FCI RAILWAYS

9

Daily Visit :

[GOVERNMENTADDA.COM]

E) Recent changes in the way work is organized at XYZ‘s company offices have not brought about any productivity increases. View Answer

Option A Explanation: A) – attacks the group of people who were part of the trail. 3. After its customers complained about being pressured to buy unneeded insurance, an insurance agency stopped rewarding its agents for high sales volume and instead gave them bonuses for high levels of customer satisfaction. Under this new plan, both customer satisfaction and the insurance agency‘s sales increased. Each of the following, if true, helps to explain how the change in incentives for agents could have resulted in increased sales EXCEPT: A) Customers were so pleased that the insurance agency had responded to their complaints that they recommended the agency to their friends. B) Agents listened more closely to customers of long standing and were able to sell them additional insurance policies that met new needs. C) Agents more frequently postponed completing the attendant paperwork even after the terms for an insurance policy were settled to the satisfaction of the client. D) Dissatisfied customers of other agencies, attracted by the reports of the change in agency policy, became customers of the agency. E) Having come to trust the increased judiciousness of the agents‘ recommendations, customers approached the agency to discuss and ultimately to buy more supplementary insurance than they previously had bought under pressure. View Answer

Option C Explanation: Agents more frequently postponed completing the attendant paperwork even after the terms for an insurance policy were settled to the satisfaction of the client. Nothing related to increase in sales. So C 4. Consumers planning to buy recreational equipment tend to buy higher quality, more expensive equipment when the economy is strong than when it is weak. Hill and Dale is a business that sells high-quality, expensive camping and hiking equipment in Boravia. Although all the signs are that Boravia‘s economy is now entering a period of sustained strength, the managers of the business do not expect a substantial increase in sales. Which of the following, if true, would provide the strongest justification for the managers’ judgment? A) A significant proportion of Hill and Dale‘s sales are made to customers who enter the store in order to buy one particular item but, once there, find other items to buy as well. B) In Boravia when the economy is strong, those who might otherwise go camping tend to take vacations overseas. C) The economic upturn is likely to allow Boravia‘s national parks, where most of the camping and hiking is done, to receive extra funding to improve their visitor facilities. D) Advances in materials technology have led to the development of hiking and camping equipment that is more comfortable and lightweight than before. GovernmentAdda.com | IBPS SBI RBI RRB SSC FCI RAILWAYS

10

Daily Visit :

[GOVERNMENTADDA.COM]

E) Many people in Boravia not only are committed to preserving the country‘s wilderness areas but also are interested in spending some time in them. View Answer

Option B Explanation: B suggests a reason that sales of equipment won’t grow. If the economy is strong, people are leaving the country, not camping and thus equipment won’t sell. 5. Highway Official: When resurfacing our concrete bridges, we should use electrically conductive concrete (ECC) rather than standard concrete. In the winter, ECC can be heated by passing an electric current through it, thereby preventing ice buildup. The cost of the electricity needed is substantially lower than the cost of the de-icing salt we currently use. Taxpayer: But construction costs for ECC are much higher than for standard concrete, so your proposal is probably not justifiable on economic grounds. Which of the following, if true, could best be used to support the highway official’s proposal in the face of taxpayer’s objection? A) The use of de-icing salt causes corrosion of the reinforcing steel in concrete bridge decks and damage to the concrete itself, thereby considerably shortening the useful life of concrete bridges. B) Severe icing conditions can cause power outages and slow down the work of emergency crews trying to get power restored. C) In weather conditions conducive to icing, ice generally forms on the concrete surfaces of bridges well before it forms on parts of the roadway that go over solid ground. D) Aside from its potential use for de-icing bridges, ECC might also be an effective means of keeping other concrete structures such as parking garages and airport runways ice free. E) If ECC were to be used for a bridge surface, the electric current would be turned on only at times at which ice was likely to form. View Answer

Option A Explanation: A) -corrosion = short life = more money spent on repairs and maintenance 6. Country Casa‘s lakes have been experiencing a constant deterioration in water quality due to too many visitors. Therefore, Casa has decided that the best way to save its lakes is by charging an extra fee to users of the lakes. Which of the following is an assumption underlying the conclusion? A) Changing global temperatures has been shown to be mostly responsible for the lakes‘ deterioration. B) It is clear that charging higher fees will not be enough to stop the destruction of the lakes. C) The level of the lakes in Country Casa has fallen by 20 centimeters per year over the last decade. D) Camping at the lakes has never been prohibited. E) Higher fees would actually deter a significant amount of people from visiting the lakes.

GovernmentAdda.com | IBPS SBI RBI RRB SSC FCI RAILWAYS

11

Daily Visit :

[GOVERNMENTADDA.COM]

View Answer

Option E Explanation: E) -If higher fees would deter most number of people from visiting , water deterioration can be reduced as the water is getting polluted due to too many visitors 7. Installing scrubbers in smokestacks and switching to cleaner-burning fuel are the two methods available to Northern Power for reducing harmful emissions from its plants. Scrubbers will reduce harmful emissions more than cleaner-burning fuels will. Therefore, by installing scrubbers, Northern Power will be doing the most that can be done to reduce harmful emissions from its plants. Which of the following is an assumption on which the argument depends? A) Switching to cleaner-burning fuel will not be more expensive than installing scrubbers. B) Northern Power can choose from among various Kinds of scrubbers, some of which are more effective than others. C) Northern Power is not necessarily committed to reducing harmful emissions from its plants. D) Harmful emissions from Northern Power‘s plants cannot be reduced more by using both methods together than by the installation of scrubbers alone. E) Aside from harmful emissions from the smokestacks of its plants, the activities of Northern Power do not cause significant air pollution View Answer

Option D Explanation: D) – The argument assumes that both cannot be used at the same time. If this were true, then Northern Power would not be doing everything it can, thus the argument falls apart. 8. To reduce the danger to life and property posed by major earthquakes, scientists have been investigating several techniques for giving advance warning of dangerous earthquakes. Since catfish swim erratically before earthquakes, some investigators have proposed monitoring catfish to predict dangerous earthquakes. Which of the following, if true, most seriously undermines the usefulness of the proposal? A) In Japan, which is subject to frequent earthquakes, the behavior of catfish has long been associated with earthquakes. B) Mechanical methods for detecting earthquakes have not proved effective. C) Tremors lead to the release of hydrogen sulfide gas into water, thereby causing various fish and shellfish to behave erratically. D) Careful construction can reduce the dangers posed by earthquakes. E) Even very slight, fleeting tremors cause catfish to swim erratically View Answer

Option C Explanation: C) – The text talks about catfish and not about ―various fish‖, we cannot say anything about catfish. GovernmentAdda.com | IBPS SBI RBI RRB SSC FCI RAILWAYS

12

Daily Visit :

[GOVERNMENTADDA.COM]

9. Children of alcoholics are much more likely to become alcoholics than are children of nonalcoholics. The likelihood varies depending on which parent is alcoholic and is much greater if both are alcoholic. This pattern holds true even if children are separated from their alcoholic parent[s] at birth. Which of the following conclusions can most reliably be drawn from the statements above? A) Children of nonalcoholics do not become alcoholics. B) Fathers influence their children‘s alcoholic tendencies more than mothers do. C) A tendency toward alcoholism can be at least partially determined by genetics. D) Environment has little influence on alcoholism. E) Because it is inherited, alcoholism is difficult to treat. View Answer

Option C Explanation: A) – nowhere it talks about NON ALCOHOLICS ! B) – cannot say for sure if mother/father influence alcoholism more ! eliminate b C) – Means it has to do something with genes..plus the tone of the answer choice is very mild..‖ partially determined‖ 10. Half of the subjects in an experiment—the experimental group—consumed large quantities of a popular artificial sweetener. Afterward, this group showed lower cognitive abilities than did the other half of the subjects—the control group—who did not consume the sweetener. The detrimental effects were attributed to an amino acid that is one of the sweetener‘s principal constituents. Which of the following, if true, would best support the conclusion that some ingredient of the sweetener was responsible for the experimental results? A) Most consumers of the sweetener do not consume as much of it as the experimental group members did. B) The amino acid referred to in the conclusion is a component of all proteins, some of which must be consumed for adequate nutrition. C) The quantity of the sweetener consumed by individuals in the experimental group is considered safe by federal food regulators. D) The two groups of subjects were evenly matched with regard to cognitive abilities prior to the experiment. E) A second experiment in which subjects consumed large quantities of the sweetener lacked a control group of subjects who were not given the sweetener. View Answer

Option D Explanation: D) – Supports the claim mentioned in the argument by stating that there was no disparity between the subjects chosen.

Directions (1-6): Each question below is followed by two arguments numbered I and II. You have to decide which of the arguments is a “strong” argument and GovernmentAdda.com | IBPS SBI RBI RRB SSC FCI RAILWAYS

13

Daily Visit :

[GOVERNMENTADDA.COM]

which is a “weak” argument. “Strong” arguments are those which are both important and directly related to the question. “Weak” arguments are those which are of minor importance and also may not be directly related to the question or may be related to a trivial aspect of the question. 1. Statement: Should public holidays be declared on demise of important national leaders? Arguments: I. No, such unscheduled holidays will hamper the Nations progress II. Yes, people would like to pay homage to the departed soul. A) If only argument I is ―strong‖. B) If only argument II is ―strong‖. C) If either I or II is ―strong‖. D) If neither I nor II is ―strong‖. E) If both I and II are ―strong‖. View Answer

Option D

2. Statement: Should we impart sex education in schools. Arguments: I. Yes, all the developing nations are doing so II. No, we cannot impart it in coeducational schools. A) If only argument I is ―strong‖. B) If only argument II is ―strong‖. C) If either I or II is ―strong‖. D) If neither I nor II is ―strong‖. E) If both I and II are ―strong‖. View Answer

Option D 3. Statement: Should new universities be established in India? Arguments: I. Yes, we have not yet achieved the target of literacy. II. No, fresh graduates will only add to the already grave situation of employment in India. A) If only argument I is ―strong‖. B) If only argument II is ―strong‖. C) If either I or II is ―strong‖. D) If neither I nor II is ―strong‖. E) If both I and II are ―strong‖.

GovernmentAdda.com | IBPS SBI RBI RRB SSC FCI RAILWAYS

14

Daily Visit :

[GOVERNMENTADDA.COM]

View Answer

Option B 4. Statement: Keeping in consideration the longevity of life in India, should the age limit for retirement in government jobs be increased? Arguments: I. Yes, other countries have decided on this long before. II. Yes, it is the demand of lakhs of employees. A) If only argument I is ―strong‖. B) If only argument II is ―strong‖. C) If either I or II is ―strong‖. D) If neither I nor II is ―strong‖. E) If both I and II are ―strong‖. View Answer

Option D 5. Statement: Show that the government levi tax on agricultural income also? Arguments: I. Yes, that is the only way to increase governments income. II. No, 80% of our population lives in rural areas A) If only argument I is ―strong‖. B) If only argument II is ―strong‖. C) If either I or II is ―strong‖. D) If neither I nor II is ―strong‖. E) If both I and II are ―strong‖. View Answer

Option A 6. Statement: Should a close relative of a retired government employee be given a government job in India? Arguments: I. Yes, where else will the relative get a job in like this? II. No, it will closed doors of government service to complaint and needy youth. A) If only argument I is ―strong‖. B) If only argument II is ―strong‖. C) If either I or II is ―strong‖. D) If neither I nor II is ―strong‖. E) If both I and II are ―strong‖. View Answer

GovernmentAdda.com | IBPS SBI RBI RRB SSC FCI RAILWAYS

15

Daily Visit :

[GOVERNMENTADDA.COM]

Option B 7. Statement: Should purchase of gold by individuals be restricted in India to improve its foreign exchange position? Arguments: I. Yes, interference on customers right and freedom is desirable. II. No, business interest has to be guarded first. A) If only argument I is ―strong‖. B) If only argument II is ―strong‖. C) If either I or II is ―strong‖. D) If neither I nor II is ―strong‖. E) If both I and II are ―strong‖. View Answer

Option D

8. Statement: Should teaching of Sanskrit be made compulsory at school level in India? Arguments: I. No, where are the trained teachers to teach this language? II. Yes, we should be proud of our ancient language. A) If only argument I is ―strong‖. B) If only argument II is ―strong‖. C) If either I or II is ―strong‖. D) If neither I nor II is ―strong‖. E) If both I and II are ―strong‖. View Answer

Option B 9. Statement: Should all education be made free for girls and women of all ages in India? Arguments: I. No, this will weaken our present social structure. II. Yes, this is the only way to bring back glory to Indian womanhood. A) If only argument I is ―strong‖. B) If only argument II is ―strong‖. C) If either I or II is ―strong‖. D) If neither I nor II is ―strong‖. E) If both I and II are ―strong‖. View Answer

Option D

GovernmentAdda.com | IBPS SBI RBI RRB SSC FCI RAILWAYS

16

Daily Visit :

[GOVERNMENTADDA.COM]

10. Statement: Should private colleges offering professional courses like Engineering, Medical and Management be banned in India? Arguments: I. Yes, such courses should be run by government colleges only. II. Yes, no other country allows private colleges. A) If only argument I is ―strong‖. B) If only argument II is ―strong‖. C) If either I or II is ―strong‖. D) If neither I nor II is ―strong‖. E) If both I and II are ―strong‖. View Answer

Option D Directions (1-6): In each question below is given a statement followed by two courses of actions numbered I and II. A course of action is taken for improvement follow up at etc. Read the statement carefully and pick the correct answer choice. 1. Statement: Majority of students have failed in the final examination in South Asian University. Courses of Action: I. The faculty should be immediately retrenched. II. The university should be closed down temporarily. A) If only course of action I follows B) If only course of action II follows C) If either course of action I or II follows D) If neither course of action I nor II follows E) If both courses of action I and II follow View Answer

Option D 2. Statement: If faculty members also join the strike, there is going to be a serious problem. Courses of Action: I. The faculty members should be persuaded not to go on strike. II. Those faculty members who joined the strike should be suspended. A) If only course of action I follows B) If only course of action II follows C) If either course of action I or II follows D) If neither course of action I nor II follows E) If both courses of action I and II follow

GovernmentAdda.com | IBPS SBI RBI RRB SSC FCI RAILWAYS

17

Daily Visit :

[GOVERNMENTADDA.COM]

View Answer

Option A 3. Statement: Higher disposal costs encourage those who produce waste to look for cheaper ways to get rid of it. Courses of Action: I. The disposal costs should be made highest. II. The disposal costs should be brought down. A) If only course of action I follows B) If only course of action II follows C) If either course of action I or II follows D) If neither course of action I nor II follows E) If both courses of action I and II follow View Answer

Option B 4. Statement: The Indian electronic component venture in the West European markets is facing tough competition from the Japanese. Courses of Action: I. India should search for other international markets for its product. II. India should improve the quality of the electronic components to compete with the Japanese in capturing these markets. A) If only course of action I follows B) If only course of action II follows C) If either course of action I or II follows D) If neither course of action I nor II follows E) If both courses of action I and II follow View Answer

Option E 5. Statement: Financial stringency has prevented The Assam government from paying salaries to its employees since April last year. Courses of Action: I. The Assam government should immediately curtail its staff strength by at least 30%. II. The Assam Government show the reduced wasteful expenditure and arrange to pay the salaries of its employees. A) If only course of action I follows B) If only course of action II follows C) If either course of action I or II follows D) If neither course of action I nor II follows GovernmentAdda.com | IBPS SBI RBI RRB SSC FCI RAILWAYS

18

Daily Visit :

[GOVERNMENTADDA.COM]

E) If both courses of action I and II follow View Answer

Option B 6. Statement: About 25 to 45% of the children who are enrolled, do not attend school on any given day. Courses of Action: I. More schools should be started. II. reasons for this absenteeism should be find out. A) If only course of action I follows B) If only course of action II follows C) If either course of action I or II follows D) If neither course of action I nor II follows E) If both courses of action I and II follow View Answer

Option B Directions (7-10): A statement is given followed by 3 courses of action. A course of action is taken for improvement follow up at etc. Read the statement carefully and pick the correct answer choice. 7. Statement: The central government has been planning to levy tax on agricultural income. Courses of Action: I. Government should seek suggestion from the farmers of the country. II. Government should gather relevant information on cost and Returns of various crops as well as total income from and expenditure on various types of holdings, regionwise and farmwise. III. Services of experts and their suggestions should be taken into consideration while formulating the planning. A) None follows B) Only I and II follow C) All follow D) Only II and III follow E) None of these View Answer

Option D 8. Statement: X state government stop the supply of free power to farmers. The decision came as a major setback to the farmers and gave rise to resentment among them. Courses of Action: I. The government of state X should withdraw its decision in order to placate the farmers. GovernmentAdda.com | IBPS SBI RBI RRB SSC FCI RAILWAYS

19

Daily Visit :

[GOVERNMENTADDA.COM]

II. The government of state X should assure the farmers that if there is an improvement in the fiscal situation of the electricity supply board, free power supply to farmers will be resumed. III. Opposition parties of State X should start demonstrations as well as persuade people that the steps taken by the government is a total betrayal for the farming community of the state. A) All follow B) Only I and III follow C) Only II and II follow D) Only II follows E) None of these View Answer

Option D 9. Statement: India is facing the tax on religious places by suicide squads of muslim terrorists. Courses of Action: I. Other religions should also develop suicide squad who need you to be ready to do anything and everything. II. The activities of every Muslim should be watched by intelligence authorities. III. People should make extremist groups at village level to counter search strikes. A) None follows B) All follow C) Only I and III follow D) Only II and III follow E) None of these View Answer

Option A 10. Statement: The Adamant attitude of State X has created much Hue and cry. The state X is still Adamant and has not released water to state Z thus violating Central rule. Courses of Action: I. The central government should invoke article 256, which empowers the centre to issue a directive to the states, if an existing Central law is violated, and to implement the same properly. II. President rule should be imposed in state X. III. The opposition and ruling parties of state Z should start demonstrations against the failure of Central rule. A) None follows B) All follow C) Only I and II follow D) Only II and III follow E) None of these View Answer

GovernmentAdda.com | IBPS SBI RBI RRB SSC FCI RAILWAYS

20

Daily Visit :

[GOVERNMENTADDA.COM]

Option E Only I follows. Directions (1-6): Below in each question are given two statements I and II. These statements may be independent causes or may be effects of independent causes or a common cause. One of these statements may be the effect of the other statement. Read both statements and decide which of the following answer choice correctly depicts the relationship between these two statements and mark answer. 1. I. The government has increased number of buses plying between Mumbai and Surat. II. The government has allowed private bus owners to operate between Mumbai and Surat. A) If Statement I is the cause and statement II is its effect. B) If Statement II is the cause and statement I is its effect. C) If both the statements I and II are independent causes. D) If both the statements I and II are effects of independent causes. E) If both the statements I and II are effects of some common causes. View Answer

Option E 2. I. The association of management colleges conducted a combined admission exam for all the institutes this year. II. As the dates for entrance exam for many management colleges closed last year, many candidates complained that they could not appear for a number of entrance exams. A) If Statement I is the cause and statement II is its effect. B) If Statement II is the cause and statement I is its effect. C) If both the statements I and II are independent causes. D) If both the statements I and II are effects of independent causes. E) If both the statements I and II are effects of some common causes. View Answer

Option B 3. I. The Departmental Store owner put cameras for surveillance of the store. II. Main products were reported missing by the staff working in the Departmental Store. A) If Statement I is the cause and statement II is its effect. B) If Statement II is the cause and statement I is its effect. C) If both the statements I and II are independent causes. D) If both the statements I and II are effects of independent causes. E) If both the statements I and II are effects of some common causes. View Answer

GovernmentAdda.com | IBPS SBI RBI RRB SSC FCI RAILWAYS

21

Daily Visit :

[GOVERNMENTADDA.COM]

Option B 4. I. Asthma cases in the city or on a continuous rise since last year. II. The number of commonly found animals such as sparrows, squirrels and pigeons in the city has continuously dwindled that since last year A) If Statement I is the cause and statement II is its effect. B) If Statement II is the cause and statement I is its effect. C) If both the statements I and II are independent causes. D) If both the statements I and II are effects of independent causes. E) If both the statements I and II are effects of some common causes. View Answer

Option D

5. I. Prices of food grains in India has increased substantially. II. Lakhs of farmers have quit farming owing to the lack of governmental support in the form of subsidies. A) If Statement I is the cause and statement II is its effect. B) If Statement II is the cause and statement I is its effect. C) If both the statements I and II are independent causes. D) If both the statements I and II are effects of independent causes. E) If both the statements I and II are effects of some common causes. View Answer

Option B 6. I. The university decided to postpone the final exams for course X by a month. II. Nearly 25 students out of 60 students in course X failed to pass the final exams last year. A) If Statement I is the cause and statement II is its effect. B) If Statement II is the cause and statement I is its effect. C) If both the statements I and II are independent causes. D) If both the statements I and II are effects of independent causes. E) If both the statements I and II are effects of some common causes. View Answer

Option D 7. Effect: The temple at the religious cite wears a deserted look with the number of devotees trickling down. Which of the following can be a possible cause of the above effect A) A structural engineer had visited the temple a month back and had declared the structure GovernmentAdda.com | IBPS SBI RBI RRB SSC FCI RAILWAYS

22

Daily Visit :

[GOVERNMENTADDA.COM]

unsafe. B) The temple is facing a drastic depletion of its funds which had accumulated over the years due to offerings made by devotees. C) The local Corporation decided to donate a huge amount of money to the temple for its renovations. D) The village housing the religious site has qualified priests to perform religious ceremonies. E) A famous actor recently visited the temple and paid his respect to the diety. View Answer

Option A 8. Statement: The income tax authorities carried out raids at three different business houses in the City last week. Which of the following can be a possible effect of the above statement? A) The three business houses are regular defaulters in paying of their income tax B) The income tax department had received the tip off about the illegal activities going on in the three business houses. C) The government decided to look into the matter and has appointed an enquiry committee. D) Other business houses took immediate action to clear off all their Income Tax dues in order to avoid a raid on their establishments. E) The authorities intend to conduct raids in several other business houses in the vicinity. View Answer

Option D 9. Effect: As a step to regulate private hospitals, the State Health Department is framing rules to ensure all such hospitals are registered with it. Which of the following can be a possible cause of the above statement? A) The department realised that private hospitals charge much less for treatment as compared to government hospitals B) Government Run hospitals do not maintain the same standards as private hospitals. C) The department realised that several hospitals were rejecting cases stating lack of infrastructure. D) Apart from the number of doctors, nurses and beds, the kind of procedure a hospital can carry out based on its infrastructure will also be registered and detailed. E) Private hospitals not registering with the department shall be forced to do so and will have to pay hefty penalties. View Answer

Option E 10. Statement: The college has finally received accreditation and has gained the status of a deemed university. Which of the following can be a possible effect of the above statement? GovernmentAdda.com | IBPS SBI RBI RRB SSC FCI RAILWAYS

23

Daily Visit :

[GOVERNMENTADDA.COM]

A) The principal of the college will now have to be a retired government official. B) Number of students seeking admission to this college in the next academic year would drop significantly C) The college will charge lesser fees from all its students despite not getting a grant from the government D) The college will reduce the number of course that it runs by a significant margin. E) the reputation of the college amongst the student population in general has improved. View Answer

Option E Directions: In each question below is given a statement followed by two courses of actions numbered I and II. A course of action is taken for improvement follow up at etc. Read the statement carefully and pick the correct answer choice. 1. Statements: Statement according to latest statistics the number of rhinos killed by Tiger was much more than the number of rhinos killed by poachers in a wildlife sanctuary Courses of Action: I.Stricter regulations should be framed in order to punish the poachers. II. As done in some sanctuaries, the areas where tigers hunt and the areas with dominant rhino population should be artificially separated A) If only course of action I follows B) If only course of action II follows C) If either course of action I or II follows D) If neither course of action I nor II follows E) If both courses of action I and II follow View Answer

Option B 2. Statements: A mid air collision was narrowly avoided when the pilot of one of the aircrafts neglected the air traffic controller‘s instructions. Courses of Action: I. Pilots of both of the air crafts should be immediately reprimanded by revoking there licences II. The training of Air traffic controllers should be improved and made more comprehensive in order to avoid such incidents in future. A) If only course of action I follows B) If only course of action II follows C) If either course of action I or II follows D) If neither course of action I nor II follows E) If both courses of action I and II follow

GovernmentAdda.com | IBPS SBI RBI RRB SSC FCI RAILWAYS

24

Daily Visit :

[GOVERNMENTADDA.COM]

View Answer

Option D 3. Statements: Oil spill from the oil carrier of one of the biggest Oil Companies has really affected the marine life in a large area near the Gulf region. Courses of Action: I. The oil company should be penalized for the negligence and the harm caused to the environment. II. Efforts should be made to shift as many Marine animals in the area as possible to safer habitats. A) If only course of action I follows B) If only course of action II follows C) If either course of action I or II follows D) If neither course of action I nor II follows E) If both courses of action I and II follow View Answer

Option E 4. Statements: Many private schools have been reportedly denying admission to students from poor families while preferring those with sound financial background regardless of their merit. Courses of Action: I. Poor students should be encouraged to take admissions only in government Run schools where such discrimination does not exist. II. The government should in enforce a certain percentage of seats for students from poor families in every schools. A) If only course of action I follows B) If only course of action II follows C) If either course of action I or II follows D) If neither course of action I nor II follows E) If both courses of action I and II follow View Answer

Option B 5. Statements: GovernmentAdda.com | IBPS SBI RBI RRB SSC FCI RAILWAYS

25

Daily Visit :

[GOVERNMENTADDA.COM]

The number of Malaria cases has been rising significantly in the city for the past few months. Courses of Action: I. The municipal authorities should take immediate steps to destroy the breeding places of mosquitoes and improve hygiene in the area. II. A campaign to educate people about the ways to prevent the disease should be started using local media and social workers. A) If only course of action I follows B) If only course of action II follows C) If either course of action I or II follows D) If neither course of action I nor II follows E) If both courses of action I and II follow View Answer

Option E Directions (6-10): A statement is given followed by 3 courses of action. A course of action is taken for improvement follow up at etc. Read the statement carefully and pick the correct answer choice. 6. Statements: Tomorrow will be the first day of operation of metro Railways for general public. No doubt a large number of people will turn up tomorrow to enjoy the facility of metro Railways. Courses of Action: I. Metro authorities should seek additional Police Force. II. Sightseers should be appealed to come only during non peak hours. III. All the windows for issuing tickets should be ready to issue tickets in entry of the people should be regulated to manageable limits. A) Only I and II follow B) Only II and III follow C) Only I and III follow D) All follow E) None of these View Answer

Option D 7. Statements: About 125 dalits including 15 women and 5 children of a village converted to Christianity during a special Christmas prayer. Courses of Action: GovernmentAdda.com | IBPS SBI RBI RRB SSC FCI RAILWAYS

26

Daily Visit :

[GOVERNMENTADDA.COM]

I. All the 125 persons should be issued summons by the court. II. An FIR should be lodged against the concerned person who converted these dalits into Christians. III. A ritual bath ceremony should be organised to read convert these Christians to dalits. A) None follows B) Only I and II follow C) All follow D) Only II and II follow E) None of these View Answer

Option A 8. Statements: Three persons of an upper caste family died of Hunger in State X recently. Courses of Action: I. High level enquiry should be made into irregularities in the listing of people under below poverty line scheme. II. The remaining members of the family been listed under BPL schemes. III. A Committee should be constituted to find out ways to restrict such deaths further as well as the circumstances which caused them hunger. A) All follow B) Only I and III follow C) Only II and II follow D) Only III follows E) None of these View Answer

Option D 9. Statements: Despite the government‘s continued efforts for the development of tribal groups, the progress made in relation to development of these groups is negligible – a leader of tribal group of country X. Courses of Action: I. Ministry of Tribal Affairs should collect authentic data of the tribal group population in the country and conductor tried by benchmark of tribal groups within a time frame. II. Information including tribal groups social status should be gathered in a time bound manner.

GovernmentAdda.com | IBPS SBI RBI RRB SSC FCI RAILWAYS

27

Daily Visit :

[GOVERNMENTADDA.COM]

III. Rehabilitation program should be launched to rehabilitate tribal groups in their own areas and a bill should be moved in the parliament in this regard. A) None follows B) All follow C) Only I and III follow D) Only II and III follow E) None of these View Answer

Option B 10. Statements: Tourism in India is an industry with huge potential but India‘s share is very less in world‘s total tourism sector Courses of Action: I. Tourism should be declared as an infrastructure industry. II. A committee should be constituted under the ministry of tourism to address the problems of tourism industry. III. Heritage monuments should be sold to a country which has better experience than India in this regard. A) None follows B) All follow C) Only I and II follow D) Only II and III follow E) None of these View Answer

Option C Directions: In each question below is given a statement followed by two conclusions numbered I and II. You have to assume everything in the statement to be true, then consider the two conclusions together and decide which of them logically follows beyond a reasonable doubt from the information given in the statement. 1. Statements: For over three decades the Government of country ‗X‘ has been making efforts to raise the per capita income of its citizens despite and intelligent team of ministry and experts bureaucrats it has failed and its objective. Conclusions: I. It is strong will power and dedication that is more important than knowledge and expertise. II. Without restricting the growth of population growth improvement in per capita income is impossible. A) If only conclusion I follows GovernmentAdda.com | IBPS SBI RBI RRB SSC FCI RAILWAYS

28

Daily Visit :

[GOVERNMENTADDA.COM]

B) If only conclusion II follows C) If either I or II follows D) If neither I nor II follows E) If both I and II follow View Answer

Option D 2. Statements: ―Because of the financial crunch and less number of Cops, the government of state X is unable to arrange security for all temples but continues arrangement of security for those temples which are famous and where there is a regular visit of devotees in large numbers‖ – CM of state X. Conclusions: I. This will encourage the devotees who want to visit famous temples to perform Pooja. II. All state governments lakh fund to facilitate security arrangement for all temples. A) If only conclusion I follows B) If only conclusion II follows C) If either I or II follows D) If neither I nor II follows E) If both I and II follow View Answer

Option A 3. Statements: Even after 15 years of establishing publishing company ‗Z‘ the books published through it still far from the reach of the common people of rural area despite the prices of books being too low. Conclusions: I. To make the books of publishing company Z accessible to each individual of rural areas, is the only objective of the company. II. Rural market has more potential than urban for the publishing company Z. A) If only conclusion I follows B) If only conclusion II follows C) If either I or II follows D) If neither I nor II follows E) If both I and II follow View Answer

Option D 4. Statements: The book X is the only one which focuses on cross border terrorism in India during the last two GovernmentAdda.com | IBPS SBI RBI RRB SSC FCI RAILWAYS

29

Daily Visit :

[GOVERNMENTADDA.COM]

decades. Conclusions: I. No other book deals with cross border terrorism in India. II. There were not any kind of threat from cross border terrorism two decades ago. A) If only conclusion I follows B) If only conclusion II follows C) If either I or II follows D) If neither I nor II follows E) If both I and II follow View Answer

Option D 5. Statements: We are not going to recruit those persons as our research team members who are below 30 years of age and do not have versatile qualities – Statement of an organisation XYZ. Conclusions: I. Those who will be selected will possess many different skills. II. Those who will be selected for the post will be more than 30 years of age. A) If only conclusion I follows B) If only conclusion II follows C) If either I or II follows D) If neither I nor II follows E) If both I and II follow View Answer

Option E 6. Statements: ―The militants killed in recent Commando operation in country X road new shows‖ – A leader of country X. Conclusions: I. The militants were not citizens of country X. II. The militants were not foreigners. A) If only conclusion I follows B) If only conclusion II follows C) If either I or II follows D) If neither I nor II follows E) If both I and II follow View Answer

Option C GovernmentAdda.com | IBPS SBI RBI RRB SSC FCI RAILWAYS

30

Daily Visit :

[GOVERNMENTADDA.COM]

7. Statements: ―Today instead of saving the governments overspending amounts to a figure equal to 4.5% of GDP. This has to be brought down to 0.5% of GDP‖ – PM of country X. Conclusions: I. Government will have to make big cuts in subsidies and other wasteful expenditure to get the desired objective. II. GDP of country X is very low. A) If only conclusion I follows B) If only conclusion II follows C) If either I or II follows D) If neither I nor II follows E) If both I and II follow View Answer

Option A 8. Statements: The Representation of the People Act will be amended to do away with the clause that disqualified from contesting the elections only for those convert who have been handed down prison terms of 6 months or more. – Information and broadcasting minister of country X Conclusions: I. Convicted persons are not suitable to serve as a true representative of the common people. II. After the amendment maja conviction would lead to disqualification irrespective of what punishment is meted out. A) If only conclusion I follows B) If only conclusion II follows C) If either I or II follows D) If neither I nor II follows E) If both I and II follow View Answer

Option A 9. Statements: Untrained drivers are found to be largely responsible for 95% of the accidents in even advanced countries where infrastructure is very rarely the cause of mishaps. – A report of a survey Conclusions: I. In developing countries Road agriculture has not developed and kept pace with the advancement of Technologies. II. Driver development is directly related to road environment. A) If only conclusion I follows B) If only conclusion II follows C) If either I or II follows D) If neither I nor II follows GovernmentAdda.com | IBPS SBI RBI RRB SSC FCI RAILWAYS

31

Daily Visit :

[GOVERNMENTADDA.COM]

E) If both I and II follow View Answer

Option B 10. Statements: The idea floated by the University Grants Commission to higher teachers on contract basis, though bold, is not acceptable to the teaching community. Conclusions: I. Only when a teacher us sure of the job will he put in sincere effort. II. There is a fear among the teaching community that teachers on contract may not be dedicated. A) If only conclusion I follows B) If only conclusion II follows C) If either I or II follows D) If neither I nor II follows E) If both I and II follow View Answer

Option D Directions: Below in each question are given two statements I and II. These statements may be independent causes or may be effects of independent causes or a common cause. One of these statements may be the effect of the other statement. Read both statements and decide which of the following answer choice correctly depicts the relationship between these two statements and mark answer. 1. I. A majority of the first year students of the engineering colleges failed in Mathematics in the semester examination. II. The college authority terminated the contract of the professor who taught Mathematics to the first year students. A) If Statement I is the cause and statement II is its effect. B) If Statement II is the cause and statement I is its effect. C) If both the statements I and II are independent causes. D) If both the statements I and II are effects of independent causes. E) If both the statements I and II are effects of some common causes. View Answer

Option A 2. I. The Government of India has allowed the private airline companies to operate on specified International routes. II. There has been a considerable increase in the flow of foreign tourists to India. GovernmentAdda.com | IBPS SBI RBI RRB SSC FCI RAILWAYS

32

Daily Visit :

[GOVERNMENTADDA.COM]

A) If Statement I is the cause and statement II is its effect. B) If Statement II is the cause and statement I is its effect. C) If both the statements I and II are independent causes. D) If both the statements I and II are effects of independent causes. E) If both the statements I and II are effects of some common causes. View Answer

Option B 3. I.Many seats in the private Engineering Colleges in the state have remained vacant this year. II. The Government Engineering Colleges in the state could not accommodate all the students who sought admission this year. A) If Statement I is the cause and statement II is its effect. B) If Statement II is the cause and statement I is its effect. C) If both the statements I and II are independent causes. D) If both the statements I and II are effects of independent causes. E) If both the statements I and II are effects of some common causes. View Answer

Option D 4. I. Majority of the students of the school passed in mathematics in the annual examination. II. Majority of the students of the school field in English in the annual examination A) If Statement I is the cause and statement II is its effect. B) If Statement II is the cause and statement I is its effect. C) If both the statements I and II are independent causes. D) If both the statements I and II are effects of independent causes. E) If both the statements I and II are effects of some common causes. View Answer

Option D 5. I. The banks have decided to give advances to the priority sector at the rate of interest at par with the corporate sector. II. The percentage of bad loans given by the banks to the priority sector is very low as compared to the corporate sector. A) If Statement I is the cause and statement II is its effect. B) If Statement II is the cause and statement I is its effect. C) If both the statements I and II are independent causes. D) If both the statements I and II are effects of independent causes. E) If both the statements I and II are effects of some common causes. GovernmentAdda.com | IBPS SBI RBI RRB SSC FCI RAILWAYS

33

Daily Visit :

[GOVERNMENTADDA.COM]

View Answer

Option B 6. I. The state government has now decided to increase the stamp duty on house purchase is with immediate effect. II. The real estate prices have increased considerably during the last few months. A) If Statement I is the cause and statement II is its effect. B) If Statement II is the cause and statement I is its effect. C) If both the statements I and II are independent causes. D) If both the statements I and II are effects of independent causes. E) If both the statements I and II are effects of some common causes. View Answer

Option B 7. I. Most of the Steel manufacturing companies in the country has made considerable profit during the last fiscal year. II. Many Asian countries have been importing huge quantities of Steel from India. A) If Statement I is the cause and statement II is its effect. B) If Statement II is the cause and statement I is its effect. C) If both the statements I and II are independent causes. D) If both the statements I and II are effects of independent causes. E) If both the statements I and II are effects of some common causes. View Answer

Option B 8. I. 3 top players of the Indian Cricket missed all the matches of the five match tournament due to injury. II. India lost the Cricket tournament to the visitors with a 2 : 3 margin A) If Statement I is the cause and statement II is its effect. B) If Statement II is the cause and statement I is its effect. C) If both the statements I and II are independent causes. D) If both the statements I and II are effects of independent causes. E) If both the statements I and II are effects of some common causes. View Answer

Option A

GovernmentAdda.com | IBPS SBI RBI RRB SSC FCI RAILWAYS

34

Daily Visit :

[GOVERNMENTADDA.COM]

9. I. The prices of food grains have increased by about 20% over the same period last year. II. The prices of vegetables have substantially decreased during the past few weeks. A) If Statement I is the cause and statement II is its effect. B) If Statement II is the cause and statement I is its effect. C) If both the statements I and II are independent causes. D) If both the statements I and II are effects of independent causes. E) If both the statements I and II are effects of some common causes. View Answer

Option D 10. I. A huge explosion rocked the Housing Complex as a cooking gas cylinder exploded in one of the flats in the complex. II. Residence of the Housing Complex came running out of their houses and assembled in the open courtyard. A) If Statement I is the cause and statement II is its effect. B) If Statement II is the cause and statement I is its effect. C) If both the statements I and II are independent causes. D) If both the statements I and II are effects of independent causes. E) If both the statements I and II are effects of some common causes. View Answer

Option A

1. Jay: Of course there are many good reasons to support the expansion of preventive medical care, but arguments claiming that it will lead to greater societal economic gains are misguided. Some of the greatest societal expenses arise from frequent urgent-care needs for people who have attained a long life due to preventive care. Sunil: Your argument fails because you neglect economic gains outside the health care system: society suffers an economic loss when any of its productive members suffer from preventable illness. Sunil‘s response to Jay makes which of the following assumptions? A) Those who receive preventive care are not more likely to need urgent care than are those who do not receive preventive care B) Jay intends the phrase ―economic gains‖ to refer only to gains accruing to institutions within the health care system. C) Productive members of society are more likely than others to suffer preventable illnesses. D) The economic contributions of those who receive preventive medical care may outweigh the economic losses caused by preventive care. E) Jay is incorrect in stating that patients who receive preventive medical care are long-lived.

GovernmentAdda.com | IBPS SBI RBI RRB SSC FCI RAILWAYS

35

Daily Visit :

[GOVERNMENTADDA.COM]

View Answer

Option D Explanation: All other options do not deal with people and economy.. 2. Hatfield Airport, the closest airport to the city of Hatfield, has recently opened a new terminal for AirJet International, a popular low-cost airline. Since the terminal has opened, Hatfield Airport has welcomed significantly more customers, to the extent that airline staff members have complained that they can no longer serve their customers efficiently. If AirJet International were to move their major terminal to the more remote Chesterfield Airport, the staff argues, the customers of both AirJet and the airlines based at Hatfield Airport would be happier. Which of the following is an assumption? A) The increased traffic at the airport is due to the popularity of AirJet‘s flights. B) Other airlines based at Hatfield Airport are not as widely used as AirJet. C) Customers are unhappy with the slowness of service at Hatfield Airport. D) Chesterfield Airport has been losing customers since the AirJet terminal at Hatfield opened. E) AirJet customers would not mind traveling further from Hatfield, since AirJet‘s low fares save them money. View Answer

Option E Explanation: Last line of paragraph says, If AirJet International were to move their major terminal to the more remote Chesterfield Airport, the staff argues, the customers of both AirJet and the airlines based at Hatfield Airport would be happier. Being a low fare air service customers would be happy even if AirJet moves to remote Chesterfield Airport.Also it will result in efficient service at Hatfield Airport 3. At many top restaurants, professional chefs use cast iron pans, which retain their heat better than other pans and can be transferred directly from the stovetop to the oven. They even work well for foods such as eggs and fish, which require nonstick surfaces. However, to retain their nonstick quality, cast iron pans need to be regularly seasoned, a process that involves coating the pan with oil or butter and heating it in the oven. Coated aluminum pans use synthetic fluoropolymers to create a nonstick surface that doesn‘t require as much maintenance. Thus, coated aluminum pans are ultimately more convenient than cast iron pans for cooking eggs and fish at home. The argument above relies on which of the following assumptions? A) Cast iron pans are difficult to maintain. B) Cookware that requires more maintenance is not as convenient for home cooking. C) There are no pans that are more convenient for nonstick purposes than coated aluminum pans. D) Coated aluminum pans can be used to cook foods that do not require a nonstick coating. E) There is no need to use butter or oil to cook in a coated aluminum pan. View Answer

GovernmentAdda.com | IBPS SBI RBI RRB SSC FCI RAILWAYS

36

Daily Visit :

[GOVERNMENTADDA.COM]

Option B 4. An increasing number of online retailers now allow customers to create ―wish lists‖ of items they would like to receive as gifts. Such lists are certainly useful, but these retailers should also explore other ways to suggest gift purchases for these customers. In particular, without revealing the specifics of a customer‘s purchase history, a website could quickly analyze a retailer‘s entire inventory, select a list of items similar to those the customer has already purchased, and then email that list to a group of contacts specified by the customer. Such a system would suggest gifts that, because of their similarity to the customer‘s prior purchases, would be extremely likely to appeal to the customer. In the argument, the assumption is that the hypothetical customers A) are familiar with most or all of the items in stock on the websites where they shop B) would prefer novel gifts that are unlike the items they currently own C) do not use retail websites primarily to purchase gifts D) would be relatively unconcerned if their retail purchase histories were available to others E) prefer online shopping to shopping in physical retail stores View Answer

Option C Explanation: C) – If customers do not use online websites to purchase gifts their plan will most likely not succeed. 5. A recent report determined that although only 3 percent of drivers on Maryland highways equipped their vehicles with radar detectors, 33 percent of all vehicles ticketed for exceeding the speed limit were equipped with them. Clearly, drivers who equip their vehicles with radar detectors are more likely to exceed the speed limit regularly than are drivers who do not. The conclusion drawn above depends on which of the following assumptions? A) Drivers who equip their vehicles with radar detectors are less likely to be ticketed for exceeding the speed limit than are drivers who do not. B) Drivers who are ticketed for exceeding speed limit are more likely to exceed the speed limit regularly than are drivers who are not ticketed. C) The number of vehicles that were ticketed for exceeding speed limit was greater than the number of vehicles that were equipped with the radar detectors. D) Many of the vehicles that were ticketed for exceeding the speed limit were ticketed more than once in the time period covered by the report. E) Drivers on Maryland highways exceed the speed limit more often than drivers on other state highways not covered in the report. View Answer

Option B Explanation: Use negation: A – It is the Conclusion C – Restatement of the Premise GovernmentAdda.com | IBPS SBI RBI RRB SSC FCI RAILWAYS

37

Daily Visit :

[GOVERNMENTADDA.COM]

D – talks about the number of vehicles and not the number of tickets. E – Out of Scope 6. The newest trend in home buying is interest-only mortgages. These mortgages require a borrower to pay only the interest on the loan. This means that the principle (which is the amount borrowed) never gets any smaller. Buyers never accumulate any equity in their homes and often have to default. Therefore, these loans are bad for Americans and should be made illegal. The argument in the above passage depends on which of the following assumptions? A) Homeowners can‘t afford to pay more than the interest on the loan. B) Some things that are bad for Americans should be made illegal. C) Interest-only mortgages don‘t require the buyer to pay more than the interest. D) Buyers with no equity in their homes often have to default on their loans. E) Owners won‘t accumulate equity based on the increasing value of their house. View Answer

Option E Explanation: It depends on the assumption in E that buyers who aren‘t paying down the principle on their loan won‘t accumulate value as their home value increases. 7. Studies have shown that children who watch a lot of television do not typically read above grade level. In the United States, more than three out of ten children watch television at least 35 hours per week. In other western countries, less than one child out of ten watches that much television. In contrast, these other children perform much better than American children on reading tests and more often read above grade level. A majority of these non American children read above their school grade level; a smaller percentage of American children read above grade level. In order to read as well as children in other western countries, American children will have to reduce the amount of time they spend watching television. The passage depends upon which one of the following assumptions? A) American children are not as interested in reading well as children in other western countries. B) Other children are more structured in their studies than children in the United States. C) If a child wants to become a better reader, he or she will not watch as much television. D) If a child watches less television, his or her reading level will increase. E) The methods American teachers and parents use to teach reading are not significantly worse than the methods used by teachers and parents in other western countries. View Answer

Option E Explanation: The argument is saying watching more television leads to decrease in grade level. It is based on the study done on American and western children. So, the assumption should be other conditions were same on those children. It was just the TV viewing duration. This is what option E is doing. It is saying there was no other change in teaching methods. Hence, an assumption.

GovernmentAdda.com | IBPS SBI RBI RRB SSC FCI RAILWAYS

38

Daily Visit :

[GOVERNMENTADDA.COM]

8. Researchers have found that when very overweight people, who tend to have relatively low metabolic rates, lose weight primarily through dieting, their metabolism generally remain unchanged. They will thus burn significantly fewer calories at the new weight than do people whose weight is normally at that level. Such newly thin persons will, therefore, ultimately regain weight until their body size again matches their metabolic rate. The conclusion of the argument above depends on which of the following assumptions? A) Relatively few very overweight people who have dieted down to a new weight tend to continue to consume substantially fewer calories than do people whose normal weight is at that level. B) The metabolisms of people who are usually not overweight are much more able to vary than the metabolisms of people who have been very overweight. C) The amount of calories that a person usually burns in a day is determined more by the amount that is consumed that day than by the current weight of the individual. D) Reseachers have not yet determined whether the metabolic rates of formerly very overweight individuals can be accelerated by means of chemical agents. E) Because of the constancy of their metabolic rates, people who are at their usual weight normally have as much difficulty gaining weight as they do losing it. View Answer

Option A Explanation: A – the argument assumes overweight people after losing weight by consuming less food to cater to their low maetabolic rate, will start consuming more food after they achieve their goal that is why their weight will increase. 9. Graduates of medical schools are interested in practical work as practicing physicians. However, research laboratories mainly deal with theoretical work, and therefore, they are reluctant to hire personnel who are not interested in research. Consequently, research laboratories rarely hire graduates of medical schools. The conclusion drawn above depends on which of the following assumptions? A) Graduates of medical schools are interested in working for research laboratories. B) The only people not interested in research work are those who are interested in practical work. C) Most employees of research laboratories are not graduates of medical schools. D) Research laboratories would hire graduates of medical schools if such graduates were interested in research work. E) Few medical school graduates who are interested in practical work are also interested in research. View Answer

Option E Explanation: E — If many of those graduates were interested in research work as well as in practical work, then laboratories would be willing to hire them and more graduates would work for research labs. Therefore, it must be assumed that medical school graduates who are interested in practical work are rarely interested in research work. GovernmentAdda.com | IBPS SBI RBI RRB SSC FCI RAILWAYS

39

Daily Visit :

[GOVERNMENTADDA.COM]

10. The local university recently hired a new soccer coach. Although she has several years‘ worth of coaching experience and is a diligent student of the game, she was never a member of a collegiate soccer team. For this reason, the new coach will be unable to build a successful program. The argument above is based on which of the following assumptions? A) The local university should have hired a former collegiate soccer player as its new coach. B) Coaching experience is one of the most crucial factors for coaching success. C) The,previous coach at the university was dismissed due to her lack of success, D) To build a successful soccer program as a coach,one must be a former collegiate soccer player. E) The university does not plan to provide the new coach with the resources necessary to build a successful program. View Answer

Option D

GovernmentAdda.com | IBPS SBI RBI RRB SSC FCI RAILWAYS

40

Daily Visit:

[GOVERNMENTADDA.COM]

100+ Decision Making Questions With Solution Governmentadda.com

Governmentadda.com | IBPS SBI RBI SSC FCI RRB RAILWAYS

1

Daily Visit:

[GOVERNMENTADDA.COM]

Decision Making Questions are re-introduced in banking examinations with SBI PO 2016, IBPS PO 2016 and IPPB 2016. But there is a little change in these questions as compared to they were asked previously. So these are Reasoning New Pattern Questions. Directions (1 – 5): Study the following information carefully and answer the questions given below: Following are the conditions for selecting a Manager in a company. The candidate must – (i) be a graduate in any discipline with at least 55% marks. (ii) be a post graduate in Management with specialization in Finance with at least 65% marks. (iii) have work experience of at least 4 years in finance department. (iv) be at least 26 years and not more than 36 years as on 01-12-2016. In the case of candidate who fulfills all conditions except – (a) at (ii) above, but has secured at least 60% marks in post graduate in Management with specialization in Finance and at least 70% marks in graduation, his case to be referred as DGM. (b) at (iii) above, but has work experience of at least 2 years as Assistant Finance Manager, his case to be referred as GM. In each question below, details of a candidate are provided followed by a question and 5 courses of action. Select the course of action that applies to the person’s candidature. 1. Karan was born on 19th March 1986. He has been working in finance department of an organization for the past 5 years. He has secured 67% marks in B.Com and 75% marks in his post-graduation. Which of the following will be a course of action if Karan applies? A) Karan will be selected. B) Data provided are inadequate to take the decision. C) Karan will not be selected. D) Karan will be offered as DGM post. E) Karan will be offered as GM post. View Answer Option B Explanation: All condition specify for Karan can be selected as manager. But it is not known that he has done his post-graduation in which stream. So data inadequate. 2. Bhuvan was born on 7th November 1987. He has been working for the past four years as Assistant Finance Manager in an organization. He has secured 70% marks in B.Sc and 70% marks in his post-graduation in Management with specialization in Finance. Which of the following will be a course of action if Bhuvan applies? A) He will be selected. B) Data provided are inadequate to take the decision. C) He will not be selected. D) He will be offered as DGM post. E) He will be offered as GM post. View Answer Governmentadda.com | IBPS SBI RBI SSC FCI RRB RAILWAYS

2

Daily Visit:

[GOVERNMENTADDA.COM]

Option A Explanation: First we have to check all the 4 conditions, these all follow so he will be selected. He is working as Assistant Manager will be taken into case only if point (ii) does not get satify. 3. Karuna has been working in the finance department of an organization for the past 7 years after completing her post-graduation with 68% marks. She secured 52% in her graduation and was born on 12th January 1988. Which of the following will be a course of action if Karuna applies? A) She will be selected. B) Data provided are inadequate to take the decision. C) She will not be selected. D) She will be offered as DGM post. E) She will be offered as GM post. View Answer Option C Explanation: She secured 52% in her graduation, but 55% is required. 4. Meeta was born on 26th July 1989. She has been working in the finance department of an organization. Which condition/s below will lead to her selecting as the DGM. (i) She secured 66% marks in post-graduation and 57% marks in graduation. (ii) She worked in finance department for 5 years. (iii) She secured 63% marks in post-graduation and 74% marks in graduation. A) Both (i) and (ii) B) Only (iii) C) Only (i) D) Both (ii) and (iii) E) None of these View Answer Option D Explanation: Meeta was born on 26th July 1989- Satisfies (iv) point She worked in finance department for 5 years- Satisfies (iii) point She secured 63% marks in post-graduation and 74% marks in graduation. – Satisfies DGM point. 5. Medha has been working as Assistant Finance Manager in an organization for the past 3 years. Which condition/s below will lead to her selecting as the GM. (i) She was born on 5th April 1992 (ii) She was born on 4th November 1990 (iii) She secured 69% marks in post-graduation and 74% marks in graduation. A) Either (i) or (ii) and (iii) B) Only (iii) C) Both (ii) and (iii) Governmentadda.com | IBPS SBI RBI SSC FCI RRB RAILWAYS

3

Daily Visit:

[GOVERNMENTADDA.COM]

D) Both (i) and (iii) E) None of these View Answer Option C Explanation: She was born on 5th April 1992 – makes her 24 in age Directions (6 – 10): Study the following information carefully and answer the questions given below: Following are the conditions for selecting an IT officer in an organization. The candidate must – (i) be a BE/BTech in IT or Computer Science with at least 60% marks. (ii) be at least 20 years and not more than 26 years as on 01-11-2016. (iii) have secured at least 60% marks in written test (iv) have secured at least 40% marks in interview In the case of candidate who fulfills all conditions except – (a) At (ii) above, but is a post graduate, his case to be referred as GM. (b) at (iv) above, but has secured at least 35% marks in interview and at least 70% marks in written test, his case to be referred as VP. In each question below, details of a candidate are provided followed by a question and 5 courses of action. Select the course of action that applies to the person’s candidature. 6. Karan was born on 19th March 1995. He has secured 65% marks in BE-IT. He has secured 62% marks in both written test and interview. Which of the following will be a course of action if Karan applies? A) Karan will be selected. B) Data provided are inadequate to take the decision. C) Karan will not be selected. D) Karan will be offered as GM post. E) Karan will be offered as VP post. View Answer Option A Explanation: All 4 conditions are met. 7. Bhuvan was born on 7th November 1989. He has secured 70% marks in B.Tech and 70% marks in his M.Tech. He has secured 65% marks in both written test and interview. Which of the following will be a course of action if Bhuvan applies? A) He will be selected. B) Data provided are inadequate to take the decision. C) He will not be selected. D) He will be offered as GM post.

Governmentadda.com | IBPS SBI RBI SSC FCI RRB RAILWAYS

4

Daily Visit:

[GOVERNMENTADDA.COM]

E) He will be offered as VP post. View Answer Option B Explanation: It is not given that he has done BTech and MTech in IT or Computers or any other other stream. 8. Karuna has secured 78% marks in written test and 38% marked in interview. She secured 62% in B.tech-IT and was born on 12th January 1991. Which of the following will be a course of action if Karuna applies? A) She will be selected. B) Data provided are inadequate to take the decision. C) She will not be selected. D) She will be offered as GM post. E) She will be offered as VP post. View Answer Option E Explanation: She has secured 38% in interview which is leas then required 40%, but is greater than 35% and in written test, she secured greater than 70%, so VP 9. Meeta was born on 26th July 1991. She did her graduation and post-graduation in Computer department. Which condition/s below will lead to her selecting as the GM. (i) She secured 68% in graduation and 65% both in written test and interview. (ii) She secured 65% in BE. (iii) She secured 63% marks in written test and 74% marks in interview. A) Both (i) and (ii) B) Only (iii) C) Only (i) D) Both (ii) and (iii) E) None of these View Answer Option D Explanation: In question it is not given that she did her graduation in which field. So (ii) is important and then (iii) In (i), it is not given then she did graduation in BE/Btech. 10. Medha completed her BTech in IT in the year 2012. Which condition/s below will lead to her selecting as the IT officer? (i) She secured 65% marks in written test. (ii) She was born on 4th November 1990. (iii) She secured 50% marks in interview. A) Both (i) and (iii) B) Both (ii) and (iii) Governmentadda.com | IBPS SBI RBI SSC FCI RRB RAILWAYS

5

Daily Visit:

[GOVERNMENTADDA.COM]

C) Both (i) and (ii) D) All (i), (ii) and (iii) E) Data provided are inadequate to take the decision. View Answer Option E Explanation: None condition gives her marks in BTech.

Directions (1 – 5): Study the following information carefully and answer the questions given below: Following are the conditions for selecting a Senior Manager in a bank. The candidate must – (i) have passed class 12th with at least 60% marks. (ii) be a graduate in any discipline with at least 55% marks. (iii) be a post graduate in Management with specialization in Economic/Statistics with at least 60% marks. (iv) have work experience of at least 2 years in a bank. (v) be at least 25 years and not more than 35 years as on 01-03-2016. (vi) cleared written examination with at least 50% marks (vii) cleared personal interview with at least 40% marks In the case of candidate who fulfills all conditions except – (a) at (iii) above, but has secured at least 60% marks in CA, the case is to be referred as VP. (b) at (vii) above, but has cleared written examination with at least 60% marks and personal interview with at least 35% marks, his case to be referred as President. In each question below, details of a candidate are provided followed by a question and 5 courses of action. Select the course of action that applies to the person’s candidature. 1. Karan has secured 65% in B.Com and 70% marks in M.Com Economics. He has been working in a bank as officer for the past 3 years after completing his post-graduation. He secured 55% marks in the written examination and 50% marks in the personal interview. He was born on July 8, 1988. A) if the data provided are inadequate to take a decision. B) if the case is to be referred to VP. C) if the case is to be referred to President. D) if the candidate is to be selected. E) if the candidate is not to be selected. View Answer Option A His class 12 marks are not given, so data inadequate 2. Ankita was born on June 2, 1986. She has been working as officer in a bank for the past 3 years after completing her post graduate degree in Statistics with 60% marks. She has secured 68% marks in class 12 and 58% marks in B.Sc. She has also secured 50% marks in both the written examination and personal interview. A) if the data provided are inadequate to take a decision. B) if the case is to be referred to VP. Governmentadda.com | IBPS SBI RBI SSC FCI RRB RAILWAYS

6

Daily Visit:

[GOVERNMENTADDA.COM]

C) if the case is to be referred to President. D) if the candidate is to be selected. E) if the candidate is not to be selected. View Answer Option D 3. Keshav has been working in a bank as officer for the past 4 years after completing his post graduate diploma in management with 60% marks. He has secured 50% and 40% marks in the written examination and personal interview respectively. He also secured 70% marks in class 12. He was born on February 25, 1981. A) if the data provided are inadequate to take a decision. B) if the case is to be referred to VP. C) if the case is to be referred to President. D) if the candidate is to be selected. E) if the candidate is not to be selected. View Answer Option A All conditions satisfy but graduation marks are not given 4. Neha was born on November 8, 1984. She secured 65% marks in class 12 and 60% marks in graduation degree. She secured 58% marks in M.Com Statistics and 60% marks in CA. She has been working in a bank as officer for the past 2 years. She also secured 50% marks in the written examination and 45% marks in personal interview. A) if the data provided are inadequate to take a decision. B) if the case is to be referred to VP. C) if the case is to be referred to President. D) if the candidate is to be selected. E) if the candidate is not to be selected. View Answer Option B For VP, instead of at least 60% marks in post-grad, if secures at least 60% marks in CA, the case is to be referred as VP. This case is satisfied here. 5. Yukta has been working in a bank as officer for the past 4 years after competing her post graduate degree in Economics with 60% marks. She also secured 60% marks both in graduation and class 12. She was born on August 24, 1985. She has secured 70% marks in the written examination and 38% marks in the personal interview. A) if the data provided are inadequate to take a decision. B) if the case is to be referred to VP. C) if the case is to be referred to President. D) if the candidate is to be selected. E) if the candidate is not to be selected. View Answer Option C Governmentadda.com | IBPS SBI RBI SSC FCI RRB RAILWAYS

7

Daily Visit:

[GOVERNMENTADDA.COM]

Directions (6 – 10): Study the following information carefully and answer the questions given below: Following are the conditions for selecting Trainee Officers in an organization. The candidate must – (i) be a graduate in any discipline with at least 55% marks. (ii) has secured at least 60% marks in class 12. (iii) be at least 22 years and not more than 29 years as on 1.1.2016 (iv) have secured at least 40% marks in the selection examination (v) have secured at least 50% marks in personal interview (v) be ready to sign a bond for 2 years. In the case of candidate who fulfills all conditions except – (a) At (i) above, but is a qualified CA, the case is to be referred to ED. (b) at (v) above, but has secured at least 60% marks in the selection examination, the case is to be referred to Chairman. In each question below, details of a candidate are provided followed by a question and 5 courses of action. Select the course of action that applies to the person’s candidature. 6. Yuvan has secured 62% marks in class 12 and 59% marks in BSc. He was born on July 15, 1990. He secured 50% marks in both the selection examination and personal interview. He is ready to sign a bond for 2 years. A) if the candidate is to be selected B) if the candidate is not to be selected C) if the case is to be referred as ED. D) if the case is to be referred as Chairman. E) if the data provided are inadequate to take a decision. View Answer Option A 7. Archana was born on May 8, 1992. She has secured 60% marks in class 12 and BA. She is ready to sign a bond for 2 years. She has secured 50% marks in selection examination and 40% marks in personal interview. A) if the candidate is to be selected B) if the candidate is not to be selected C) if the case is to be referred as ED. D) if the case is to be referred as Chairman. E) if the data provided are inadequate to take a decision. View Answer Option B Archana secured 40% marks in personal interview, while at least 50% is required. 8. Anuj was born on December 2, 1986. He has secured 56% marks in BTech and 63% marks in class 12. He has also secured 55% marks in both the selection examination and personal interview. He is ready to sign a bond for 2 years. A) if the candidate is to be selected B) if the candidate is not to be selected C) if the case is to be referred as ED. Governmentadda.com | IBPS SBI RBI SSC FCI RRB RAILWAYS

8

Daily Visit:

[GOVERNMENTADDA.COM]

D) if the case is to be referred as Chairman. E) if the data provided are inadequate to take a decision. View Answer Option B Anuj was born on December 2, 1986, so on 1.1.2016 she is greater than 29 years. 9. Lata was born on March 14, 1991. She has secured 62% marks in class 12 and 65% marks in selection examination. She has also secured 58% marks in graduation and 48% marks in personal interview. She is ready to sign a bond for 2 years. A) if the candidate is to be selected B) if the candidate is not to be selected C) if the case is to be referred as ED. D) if the case is to be referred as Chairman. E) if the data provided are inadequate to take a decision. View Answer Option D For Chairman, instead of at least 50% marks in personal interview, if secures at least 60% marks in selection exam, the case is to be referred as Chairman. This case is satisfied here. 10. Sunil was born on November 4, 1988. He has secured 67% marks in class 12 and 50% marks in graduation. He also has qualified CA exam. He has secured 50% marks in both the selection examination and personal interviewer. He is ready to sign a bond for 2 years. A) if the candidate is to be selected B) if the candidate is not to be selected C) if the case is to be referred as ED. D) if the case is to be referred as Chairman. E) if the data provided are inadequate to take a decision. View Answer Option C Directions (1-2): For final selection, anyone can be selected only after fulfilling all of the following criteria. The person should be in the age range 22 to 28 years as on 01.02.2017. The person should have obtained at least 60% marks in class 12 and graduation degree both. The person should have a work experience of at least 1 year. The person should have a certification in ABC course. The person should have cleared at least 3 rounds of interview. 1. Bhuvan fulfills all the following criteria, then which of the following criterion/criteria will not lead to his final selection? (i) He was born on 12.10.1989. (ii) He completed his graduation in 2016 with 62% marks and 67% marks in class 12. (iii) After completing his graduation, he started job and left on 30.11.2016 to complete certification in ABC course. (iv) He cleared 4 rounds of interview. A) Only (i) B) Only (ii) and (iii) Governmentadda.com | IBPS SBI RBI SSC FCI RRB RAILWAYS

9

Daily Visit:

[GOVERNMENTADDA.COM]

C) Only (ii) and (iv) D) Only (iii) E) Only (ii), (iii) and (iv) View Answer Option D He completed graduation in 2016 and left on 30.11.2016. This means he has worked for less than a year. So (iii) condition will not lead to his selection 2. Bhavika fulfills all the following criteria, then which of the following criterion/criteria will not lead to his final selection? (i) She was born on 22.12.1990. (ii) She completed her graduation with 69% marks and 82% marks in class 12. (iii) She has worked for 15 months, where she got certification in ABC course. (iv) She cleared 3 rounds of interview. A) Only (i) B) Only (i) and (iii) C) Only (ii) and (iv) D) Only (iii) E) None of these View Answer Option E The criteria in all 4 points fulfills the condition for her selection. Directions (3-4): For a job in a company following conditions need to be fulfilled. The candidate’s age must be in between 22 and 28 years as on 01.02.2017. The candidate should have got at least 70% marks in written examination and 40% marks in interview. The candidate must be a graduate and have a work experience of at least a year. If the candidate is a female, then age relaxation of 2 years will be given. 3. A candidate has applied for the job and satisfies the following conditions. Give the correct sequence in which it can be found that the candidate gets the job in minimum number of conditions. (i) Got 50% marks in interview. (ii) Got 60% marks in his graduation. (iii) Was born on 07.08.1990. (iv) After graduation, worked for 13 months in ABC company. (v) Got 78% marks in written test. A) (iv), (i), (ii), (iii), (v) B) (v), (i), (iii), (ii), (iv) C) (iv), (v), (iii), (i), (ii) D) (i), (v), (iv), (ii), (iii) E) None of these View Answer Governmentadda.com | IBPS SBI RBI SSC FCI RRB RAILWAYS

10

Daily Visit:

[GOVERNMENTADDA.COM]

Option C (ii) point is extra as marks of graduation are not required to fulfill the criteria for selection. Only graduation is required, which is given in point (iv). So the option in which (ii) is at last is the answer. 4. A candidate who has applied for job satisfies the following conditions: (i) The candidate got 52% marks in interview. (ii) The candidate has work experience of 15 months. (iii) The candidate was born on 22.12.1990. (iv) The candidate is a female. (v) The candidate got 72% marks in written test and 50% in graduation. Give the correct sequence in which it can be found that the candidate gets the job in minimum number of conditions. A) (iv), (i), (ii), (iii), (v) B) (v), (i), (iii), (ii), (iv) C) (iv), (v), (iii), (i), (ii) D) (i), (v), (iv), (ii), (iii) E) None of these View Answer Option B From (iii), the candidate whether female or male will definitely get job with other conditions satisfying. So there is no need to go to condition (iv) to know whether the candidate is male or female. So sequence having (iv) at last will the correct sequence. 5. For a job in a bank as manager following conditions need to be fulfilled. Work experience of at least 3 years in a bank. At least 55% marks in post-graduation in Economics. At least 60% marks in graduation. Must be 23 – 27 years as on 01.02.2017. Must secure at least 70% marks in the written examination and at least 35% marks in the personal interview. Must have qualified at least one promotional exam in the bank. If there is a certification in ABC course, then that will be a plus point for the candidate. Suhani who has applied for the manager post in the bank has previously worked in a bank as officer for 4 years. In the 4 years of service, she gave 3 promotional exams and qualified them. She qualified the written examination with 80% marks and interview with 55%. Which of the following condition/s she must fulfill to get the job? (i) She did ABC course last year. (ii) She secured 60% in M.Sc with Economics as her subject. (iii) She did her graduation in economics stream with 62% marks (iv) She got 62% in B.Sc and 60% in M.Sc with economics. (v) As per her resume, her date of birth is 17.02.1993. A) Only (i) and (iv) B) Only (ii), (iii), and (v) C) Only (ii), (iv) and (v) D) Only (i), (iv) and (v) E) Only (iv) and (v) View Answer Option E (i) is not mandatory, that is just an extra thing to be satisfied. Governmentadda.com | IBPS SBI RBI SSC FCI RRB RAILWAYS

11

Daily Visit:

[GOVERNMENTADDA.COM]

From (v) we know her age. and from (iv) her grad and post grad marks. So these two conditions are sufficient to get her a selection. 6. Following are the conditions for selecting Trainee Officers in an organization. The candidate must – Be a graduate in any discipline with at least 55% marks. Has secured at least 60% marks in class 12. Be at least 22 years and not more than 29 years as on 1.1.2017. Have secured at least 40% marks in the selection examination. Have secured at least 50% marks in personal interview. Be ready to sign a bond for 2 years. Kavya who has applied for the Trainee Officer did B.Com with 65% marks. She was born on 06.05.1985. She is ready to sign a bond of 2 years. Which of the following condition/s she must fulfill to get the job? (i) She secured 82% marks in class 12. (ii) She secured 56% marks in M.Com (iii) She got 55% marks in interview and 45% in selection examination. (iv) She got 65% in B.Com and 56% in M.Com A) Only (i) and (iii) B) Only (ii) and (iv) C) Only (iii) D) Only (i), (ii) and (iii) E) She will not be selected View Answer Option E She does not satisfy the age criteria, so she will not be selected. Directions (7 – 10): Study the following information carefully and answer the questions given below: Following are the conditions for selecting a Deputy personnel Manager in an organization. The candidate must – (i) be a graduate in any discipline with at least 55% marks. (ii) be at least 30 years and not more than 35 years as on 01-02-2017. (iii) be ready to pay Rs 40,000 as security deposit. (iv) have work experience of at least 3 years. (v) cleared written examination with at least 50% marks (vi) cleared personal interview with at least 40% marks In the case of candidate who fulfills all conditions except – (a) at (i) above, but has secured at least 55% marks in post graduate in Management with specialization in Economic/Statistics, the case is to be referred as GM-Personnel. (b) at (iii) above, but is ready to sign a bond of 1 year, the case is to be referred as SVP-Personnel. In each question below, details of a candidate are provided followed by a question and 5 courses of action. Select the course of action that applies to the person’s candidature. 7. Roopa was born on 17.07.1985. She has secured 64% marks in BA and 54% marks in personal interview. She is ready to pay a security deposit of Rs 40,000. She has secured 57% marks in written examination. She has work experience of 4 years. A) if the candidate is to be selected B) if the candidate is not to be selected C) if the case is to be referred as GM-Personnel. Governmentadda.com | IBPS SBI RBI SSC FCI RRB RAILWAYS

12

Daily Visit:

[GOVERNMENTADDA.COM]

D) if the case is to be referred as SVP-Personnel. E) if the data provided are inadequate to take a decision. View Answer Option A Roopa will be selected as she fulfills all the criteria, 8. Kashish was born on 12.12.1986. She has secured 56% marks in graduation. She has also secured 55% marks in both the selection examination and personal interview. She is ready to sign a bond for 1 year. A) if the candidate is to be selected B) if the candidate is not to be selected C) if the case is to be referred as GM-Personnel. D) if the case is to be referred as SVP-Personnel. E) if the data provided are inadequate to take a decision. View Answer Option E Nothing is given about the work experience, so data is inadequate to tell anything about her candidature. Answer would be option B only if some information about her work experience was given that she has work experience of less than 3 years. But since the full information is not given so Data is inadequate to take the decision. 9. Kavita was born on 14.03.1983. She has secured 65% marks in selection examination. She has also secured 60% marks in graduation and 48% marks in personal interview. She has a work experience of 3 years. She did her post-graduation in Management with specialization in Economics with 65% marks. A) if the candidate is to be selected B) if the candidate is not to be selected C) if the case is to be referred as GM-Personnel. D) if the case is to be referred as SVP-Personnel. E) if the data provided are inadequate to take a decision. View Answer Option A Kavita fulfills all the required criteria, so there is no need to check that whether she has done postgraduation or not. (She secured 60% marks in graduation, which was the requirement) 10. Isha was born on 04.11.1986. She has secured 70% marks in graduation. She has secured 50% marks in both the selection examination and personal interview. She has a work experience of 5 years. She can pay a maximum of Rs 30,000 as a security deposit. Additionally, she is ready to sign a bond of 1 year. A) if the candidate is to be selected B) if the candidate is not to be selected C) if the case is to be referred as GM-Personnel. D) if the case is to be referred as SVP-Personnel. Governmentadda.com | IBPS SBI RBI SSC FCI RRB RAILWAYS

13

Daily Visit:

[GOVERNMENTADDA.COM]

E) if the data provided are inadequate to take a decision. View Answer Option D She fulfills all criteria, except that she can pay only 30,000, so this leads – not to her selection. But since she can sign a bond of 1 year, so her case will be referred to as SVP-Personnel.

1. For section in an interview, anyone can be selected only after fulfilling all of the following criteria. The person should have age between 21 and 30 years as on 01.03.2017. The person should have obtained at least 60% marks in class 12 and graduation degree both. The person should have a work experience of at least a year. The person should have got 70% marks in written test and 90% in language proficiency. Sahil fulfills all the following criteria, then which of the following criterion/criteria will not lead to his final selection? (i) He was born on 21.02.1996. (ii) He completed his graduation in 2016 with 62% marks and 67% marks in class 12. (iii) He has worked for 2 years in ABC Company. (iv) He got 72% in written test and 95% in language proficiency. A) Only (iii) B) Only (ii) and (iii) C) Only (ii) and (iv) D) Only (i) E) Only (i), (iii) and (iv) View Answer Option D He was born on 21.02.1996 so on 01.03.2017, his age is 21 years, but the age should be between 21 and 30 means 22 to 29. 2. For a job in a company following conditions need to be fulfilled. The candidate’s age must be in between 22 and 28 years as on 01.02.2017. The candidate should have got at least 70% marks in written examination and 40% marks in interview. The candidate must be a graduate and have a work experience of at least a year. If the candidate is a female, then age relaxation of 2 years will be given. A candidate has applied for the job and satisfies the following conditions. Give the correct sequence in which it can be found that the candidate gets the job in minimum number of conditions. (i) Got 75% marks both in written test and interview. (ii) Has certification in ABC Course. (iii) Was born on 07.08.1988. (iv) Has a work experience of 18 months (v) The candidate is a female A) (iv), (i), (ii), (iii), (v) B) (v), (i), (iii), (ii), (iv) C) (iv), (v), (iii), (i), (ii) D) (i), (v), (iv), (ii), (iii) Governmentadda.com | IBPS SBI RBI SSC FCI RRB RAILWAYS

14

Daily Visit:

[GOVERNMENTADDA.COM]

E) None of these View Answer Option C (ii) point is not required as per given conditions so sequence having (ii) at last is your answer. All other conditions are required for selection. Candidate is female so age 29 will work. Directions (3-6): Read the following information carefully and then answer the questions given below it. For getting a residential accommodation by a company, the employee must fulfill the following criteria: (i) have worked with company for at least 10 years with at least 4 years in ABC department. (ii) have at most 5 members in the family (iii) have at least 5 years of service remaining with retirement age being 58 years. (iv) not be owner or co-owner (if spouse is owner) of a house In the case of an employee who: (v) satisfies all except (i) above and joined the company as a manager, should be referred to Director (vi) satisfies all except (iii) above and working as senior manager in company, to be referred to Managing Director (vii) has been transferred from another city, condition (i) can be waived. Based on above conditions, decide whether or not the accommodation will be provided or the case to be referred to higher authority. All cases are presented on 31st July 2016. 3. Sakshi stays in a house with 2 other members. Neither she nor her husband owns a house. She is a senior manager in company for the last years and has been working in the company for the last 12 years with 4 years in ABC department. She will retire in the year 2020 A) if the employee is to be provided with accommodation B) if the employee is not to be provided with accommodation C) Data inadequate D) if the case to be referred as Director E) if the case to be referred as Managing Director. View Answer Option E 4. Rohit stays in a rented house with wife and 3 children. He was born on July 12 1969. He has been working in the company for the last 12 years out of which 5 years in ABC department. A) if the employee is to be provided with accommodation B) if the employee is not to be provided with accommodation C) Data inadequate D) if the case to be referred as Director E) if the case to be referred as Managing Director. View Answer

Governmentadda.com | IBPS SBI RBI SSC FCI RRB RAILWAYS

15

Daily Visit:

[GOVERNMENTADDA.COM]

Option C Not given about condition (iv) 5. Dhruv joined the company as manager in 2010 and was 30 years old on August 15, 2013. He has four members in his family and both he and his wife do not own any house. A) if the employee is to be provided with accommodation B) if the employee is not to be provided with accommodation C) Data inadequate D) if the case to be referred as Director E) if the case to be referred as Managing Director. View Answer Option D All conditions satisfied for Director 6. Kunal has been transferred from another office and was 53 years old on February 6 2016. He has been working in the company for the last 20 years out of which 6 years in ABC department. There are 4 members in his family and both he and his wife do not own any house. A) if the employee is to be provided with accommodation B) if the employee is not to be provided with accommodation C) Data inadequate D) if the case to be referred as Director E) if the case to be referred as Managing Director. View Answer Option B He has less than 5 years of remaining service. Directions (7-10): Read the following information carefully and then answer the questions given below it. A society has kept the following conditions for the allotment of flats in their society: (i) produce domicile certificate of the State (ii) must be earning for minimum of 5 years (iii) be ready to pay the entire amount in 5 years period (iv) not be owner or co-owner (if spouse is owner) of a residential accommodation in that city (v) not be less than 35 years of age as on 31st Dec 2016. In case of applicant who satisfies all other criteria except: A) at (i) above, be referred to President of Town Council B) at (ii) above, but is ready to provide Aadhar card should be referred to Vice-Chairman of the Committee. C) At (iii) above, but is a freedom fighter or an ex-serviceman or first relation with these should be referred to member of Committee The last date for receipt of application was 31st Dec, 2016. Conditions are to be fulfilled as on 31st Dec, 2016. Based on these criteria decide whether to allot flat or not. The cases are given to you as on 1st Jan, 2017.

Governmentadda.com | IBPS SBI RBI SSC FCI RRB RAILWAYS

16

Daily Visit:

[GOVERNMENTADDA.COM]

7. Sumit is son of an industrialist and from other state who has set his factory in 2000 and has a domicile certificate of the state. He is ready to pay the entire amount in 4 years if required. He does not own a house in that city but his wife owns a flat there. His date of birth is 11th Nov 1970. A) Do not allot flat B) Refer to the Chairman C) Refer to the President D) Allot flat E) Data inadequate View Answer Option A Sumit is co-owner of flat in the city 8. Shilpa is daughter of a freedom fighter from another state. She has domicile certificate of the state and employed in the Town Council of city for last 6 years. She can pay the entire amount in 5 years. She has completed 34 years on 10th Dec, 2004. She does not own a house in the city. A) Refer to the Chairman B) Allot flat C) Do not allot flat D) Refer to the President E) Data inadequate View Answer Option B Satisfies all conditions 9. Madhav is a 38 years old senior clerk in an office in the city. He is employed for the last 13 years but still does not own a house. He has domicile certificate of the state and is ready to pay the entire amount in 8 years. He is nephew of a freedom fighter. A) Do not allot flat B) Data inadequate C) Refer to the Vice-Chairman D) Refer to the Chairman E) Allot flat View Answer Option A nephew is not first relation 10. Garima, a domicile of the state and a married woman of 36 years, has been running an ice-cream parlor in the city since 4th March 2002. Her husband works in a nearby city but both of them do not own a house in the city. She can pay the entire amount in 5 years. A) Allot flat B) Data inadequate C) Do not allot flat D) Refer to the Chairman Governmentadda.com | IBPS SBI RBI SSC FCI RRB RAILWAYS

17

Daily Visit:

[GOVERNMENTADDA.COM]

E) Refer to the Vice-Chairman View Answer Option B Does not satisfy condition (ii)

1. For a job, a candidate must fulfill the following criteria: The age should be from 22 to 28 as on 15th April 2017. Must have secured at least 60% marks in class 12. Must have secured at least 58% marks in graduation. Must have done post-graduate in finance. Must provide an NOC certificate if working somewhere. Must provide Aadhar Card or PAN Card. Must have secured at least 65% in written test and 70% in interview. Sahil fulfills all the following criteria, then which of the following criterion/criteria is not essential for final selection? (i) He was born on 19.02.1990. (ii) He completed his graduation with 66% marks and secured 76% marks in class 12. (iii) He has worked for 2 years in ABC Company. (iv) He got 72% in written test and 75% in interview. (v) He did his post-graduation in finance and provided Aadhar Card. A) Only (ii) B) Only (ii) and (iii) C) Only (iii) D) Only (i) and (iii) E) Even with all given criteria fulfilling, he will not be selected. View Answer Option C Explanation: NOC is to be provided if the person is working somewhere. But (iii) parts says – He has worked for 2 years in ABC Company. He worked sometime before, not working right now. 2. For a job in a company following conditions need to be fulfilled. The candidate’s age must be in between 22 and 28 years as on 01.04.2017. The candidate should have got at least 70% marks in written examination and 40% marks in interview. The candidate must be a graduate and have a work experience of at least a year. If the candidate is a female, then age relaxation of 2 years will be given. A candidate has applied for the job and satisfies the following conditions. Give the correct sequence in which it can be found that the candidate gets the job in minimum number of conditions. (i) Got 75% marks both in written test and interview. (ii) The candidate is a female (iii) Was born on 07.08.1991. (iv) Has a work experience of 18 months A) (iv), (i), (ii), (iii) B) (i), (iii), (ii), (iv) C) (iv), (iii), (i), (ii) D) (i), (iv), (ii), (iii) Governmentadda.com | IBPS SBI RBI SSC FCI RRB RAILWAYS

18

Daily Visit:

[GOVERNMENTADDA.COM]

E) None of these View Answer Option C (ii) point is not required as age relaxation will be taken into account in case of female. But given candidate is already between 22-28 years. So there is no need to know that whether the candidate if female or male. So option in which (ii) is at last will be the answer. Directions (3-4): Read the following information carefully and then answer the questions given below it. For getting a job by a company, the candidate must fulfill the following criteria: Must have worked with a company for at least 4 years. Must have got 60% in graduation. Must have cleared the written test and interview. Must be ready to sign a bond of 2 years with the company. The mean of percentage points in written test and interview must be at least 40%. 3. i) Sakshi got 60% in written test and 30% in interview ii) Sakshi is ready to sign a bond of 2 years. iii) Sakshi is working with ABC company for the last 5 years. iv) Sakshi got 72% in BCom Sakshi fulfills the above given criteria, then from which of the following points it can be concluded that she is not selected for the job? A) Only (i) B) Only (ii) and (iii) C) Only (iv) D) Only (iii) E) From none of (i), (ii), (iii) and (iv) View Answer Option E Sakshi got 60% in written test and 30% in interview so mean of percentage points = (60+30)/2 = 45% So all given criteria are necessary 4. Mohit has worked with a company for 4 years. He got 70% marks in BTech. He got 46% as mean of percentage points in written test and interview. Which of the following points will lead to Mohit’s final selection? A) He provided an No Objection Certificate from his previous employer. B) He signed a bond of 2 years with his previous employer. C) He was born on 02.06.1992. D) He completed Btech degree in Computer Stream. E) None of these View Answer Option E The condition which must be fulfilled to get the job – He is ready to sign a bond of 2 years with this new company. Governmentadda.com | IBPS SBI RBI SSC FCI RRB RAILWAYS

19

Daily Visit:

[GOVERNMENTADDA.COM]

5. For a job, a candidate must fulfill the following criteria: The age should be from 24 to 30 as on 15th April 2017. Must have secured at least 58% marks in BSc – Math. Must have done post-graduate in Math or Statistics with at least 62% marks. Must have an experience of at least 2 years. Must have secured at least 60% in written test and 70% in interview. Sahil fulfills all the following criteria, then which of the following criterion/criteria is not essential for final selection? (i) He was born on 7th July 1988. (ii) He completed his graduation with 66% marks and secured 76% marks in class 12. (iii) He has worked for 3 years in ABC Company. (iv) He got 72% in written test and 75% in interview. (v) He did his graduation and post-graduation in Math. (vi) He secured 64% marks in post-graduation. A) Only (ii) B) Only (ii) and (iii) C) Only (iii) D) Only (i) and (iii) E) Even with all given information, he will not be selected. View Answer Option E Explanation: According to the conditions, graduation must be BSc – Math. But in points (i) to (vi) – this is not specified anywhere. Directions (6 – 10): Study the following information carefully and answer the questions given below: Following are the conditions for selecting a candidate in an organization. The candidate must – (i) be a graduate with at least 60% marks. (ii) be at least 21 years and not more than 28 years as on 01-04-2017. (iii) have secured at least 60% marks in written test (iv) have secured at least 50% marks in interview In the case of candidate who fulfills all conditions except – (a) At (ii) above, but is a post graduate with at least 60% marks, his case to be referred as Manager. (b) at (iv) above, but has secured at least 35% marks in interview and at least 70% marks in written test, his case to be referred as Assistant. 6. In each question below, details of a candidate are provided followed by a question and 5 courses of action. Select the course of action that applies to the person’s candidature. Megha was born on 19 March 1991. He secured 65% marks in BCom. He also secured 62% marks in both written test and interview. Which of the following will be a course of action if Megha applies for the job? A) Megha will be selected. B) Data provided are inadequate to take the decision. C) Megha will not be selected. D) Megha will be offered as Manager post.

Governmentadda.com | IBPS SBI RBI SSC FCI RRB RAILWAYS

20

Daily Visit:

[GOVERNMENTADDA.COM]

E) Megha will be offered as Assistant post. View Answer Option A Explanation: All 4 conditions are met. 7. Pankil was born on 7th November 1989. He secured 68% marks in BSc and 63% marks in his MSc. He has secured 65% marks in both written test and interview. Which of the following will be a course of action if Pankil applies? A) Pankil will be selected. B) Data provided are inadequate to take the decision. C) Pankil will not be selected. D) Pankil will be offered as Manager post. E) Pankil will be offered as Assistant post. View Answer Option A 8. Varun has secured 78% marks in written test and 48% marks in interview. He secured 62% in B.Tech and was born on 12th January 1991. Which of the following will be a course of action if Varun applies? A) He will be selected. B) Data provided are inadequate to take the decision. C) He will not be selected. D) He will be offered as Manager post. E) He will be offered as Assistant post. View Answer Option E Explanation: He has secured 48% in interview which is leas then required 50%, but is greater than 35% and in written test greater than 70%, so Assistant 9. Varun has secured 78% marks in written test and 58% marks in interview. He completed his B.Sc 4 years back and was born on 12th January 1991. Which of the following will be a course of action if Varun applies? A) He will be selected. B) Data provided are inadequate to take the decision. C) He will not be selected. D) He will be offered as Manager post. E) He will be offered as Assistant post.

Governmentadda.com | IBPS SBI RBI SSC FCI RRB RAILWAYS

21

Daily Visit:

[GOVERNMENTADDA.COM]

View Answer Option B Explanation: He has done graduation but marks in graduation is not provided in question. So data is inadequate to answer the question 10. Kavita was born on 8th November 1985. She did her graduation and post-graduation in Computer department with same marks. Which condition/s below will lead to her selecting as the Manager? (i) She secured 68% in graduation and 65% both in written test and interview. (ii) She secured 68% in BE. (iii) She secured 63% marks in written test and 74% marks in interview. A) Both (i) and (ii) B) Only (iii) C) Only (i) D) Both (i) and (iii) E) None of these View Answer Option C Explanation: From (i), she is a graduate with at least 60% marks, have secured at least 60% marks in written test and have secured at least 50% marks in interview

Directions (1 – 5): Study the following information carefully and answer the questions given below: Following are the conditions for selecting a candidate in an organization. The candidate must – (i) have passed class 12 exam with at least 60% marks. (ii) have passed graduation degree in any discipline with at least 55% marks. (iii) be not less than 21 years and not more than 30 years of age as on 1.7.2016. (iv) possess a certificate/diploma/degree course in Computer Science. In the case of candidate who fulfills all conditions except – (a) at (ii) above but is a post-graduate, case may be referred to the Executive Director (ED). (b) at (iv) above but has studied Computer Science as one of subjects in graduation, case may be referred to the Vice President (VP). In each question below, details of a candidate are provided followed by a question and 5 courses of action. Select the course of action that applies to the person’s candidature. All these cases are given to you as on 1.11.2016. 1. Lakshay is a Commerce graduate with 57% marks. He had secured 73% marks class 12 examination. He has studied Computer Science as one of the subjects in class 12. His date of birth is 22.9.1990. A) if the case is to be referred to Executive Head. B) if the case is to be referred to Vice President. C) if the candidate is to be selected. D) if the information is inadequate to take a decision. Governmentadda.com | IBPS SBI RBI SSC FCI RRB RAILWAYS

22

Daily Visit:

[GOVERNMENTADDA.COM]

E) if the candidate is not to be selected. View Answer Option E Explanation: (iv) condition is not specified. So move to (b). But this is also not fulfilled so he will not be selected for any job 2. Ruchi has passed BA degree examination in second class with 56% marks and class 12 in first class with 68% marks. She has completed a diploma in Computer Science. She will be 35 years of age in November 2021. A) if the case is to be referred to Executive Head. B) if the case is to be referred to Vice President. C) if the candidate is to be selected. D) if the information is inadequate to take a decision. E) if the candidate is not to be selected. View Answer Option D Explanation: She will be 35 years of age in Nov 21. If her birthday falls on 1st Nov then she will be selected and if after 1st Nov then she will be not selected. So data inadequate 3. Rahul has passed his class 12 exam with 67% marks after which he did a 6 months certificate course in Computer Science. He completed his graduation with 62% marks. His date of birth is 25.05.1988. A) if the case is to be referred to Executive Head. B) if the case is to be referred to Vice President. C) if the candidate is to be selected. D) if the information is inadequate to take a decision. E) if the candidate is not to be selected. View Answer Option C

4. Karuna is a Science graduate with 47%. She had scored 64% marks in Class 12 examinations. She has also passed M.Sc. with 48% marks. She has done a certificate course in computers. Her date of birth is 22.12.1990 A) if the case is to be referred to Executive Head. B) if the case is to be referred to Vice President. C) if the candidate is to be selected. D) if the information is inadequate to take a decision. E) if the candidate is not to be selected.

Governmentadda.com | IBPS SBI RBI SSC FCI RRB RAILWAYS

23

Daily Visit:

[GOVERNMENTADDA.COM]

View Answer Option A 5. Kavita is a B.A. passed in first class with 61% marks. She had passed her class 12 examinations with 72% marks. She has also studied Computer Science as a subject in B.A. Her date of birth is 23.9.1992. A) if the case is to be referred to Executive Head. B) if the case is to be referred to Vice President. C) if the candidate is to be selected. D) if the information is inadequate to take a decision. E) if the candidate is not to be selected. View Answer Option B Directions (6 – 10): Study the following information carefully and answer the questions given below: Following are the conditions for admitting students for a Programme in Management. The applicant must – (i) not be less than twenty four years as on 1.12.2016. (ii) be a post graduate in any discipline with at least 55% marks. (iii) have passed the written test with at least 60% marks. (iv) have work experience of at least 6 months after completing post-graduation. In the case of an applicant who fulfills all other conditions except: (a) at (ii) above, but have completed M. Tech degree, his/her case is to be referred to Director. (b) at (iii) above, but has work experience of more than 2 years, his/her case is to be referred to Registrar. 6. Megha has secured 72% marks in post-graduation and has been working for five years after postgraduation. She was born on 11.02.1988. A) if the case is to be referred to Registrar. B) if the case is to be referred to Director. C) if the applicant is to be admitted. D) if the information is inadequate to take a decision. E) if the applicant is not to be admitted. View Answer Option D Explanation: If the condition have passed the written test with at least 60% marks – is not fulfilled then point (b) is to be looked at. But there is no information given about condition – have passed the written test with at least 60% marks. So data is inadequate

Governmentadda.com | IBPS SBI RBI SSC FCI RRB RAILWAYS

24

Daily Visit:

[GOVERNMENTADDA.COM]

7. Anil has been working for 3 years after completing M.Tech degree. He was born on 1-11-1990. He has secured 60% marks in the written test and 45% marks in post-graduation. A) if the case is to be referred to Registrar. B) if the case is to be referred to Director. C) if the applicant is to be admitted. D) if the information is inadequate to take a decision. E) if the applicant is not to be admitted. View Answer Option B Explanation: Condition – be a post graduate in any discipline with at least 55% marks. – not fulfilled. BUT he has done MTech 8. Ravina has completed her post-graduation with 67% marks. She has secured 52% marks in the written test. She has been working for 6 years after post-graduation. She was born on 20.08.1981. A) if the case is to be referred to Registrar. B) if the case is to be referred to Director. C) if the applicant is to be admitted. D) if the information is inadequate to take a decision. E) if the applicant is not to be admitted. View Answer Option A 9. Vivek has secured 50% and 60% marks in post-graduation (M.Tech) and written test respectively. He has been working for 4 years after his education and he was born on 11.12.1971. A) if the case is to be referred to Registrar. B) if the case is to be referred to Director. C) if the applicant is to be admitted. D) if the information is inadequate to take a decision. E) if the applicant is not to be admitted. View Answer Option B 10. Vishal was born on 05-03-1986. He has secured 85% marks in the written test and 55% marks in post-graduation. He has been working for last 1 year. A) if the case is to be referred to Registrar. B) if the case is to be referred to Director. C) if the applicant is to be admitted. D) if the information is inadequate to take a decision. E) if the applicant is not to be admitted. View Answer Governmentadda.com | IBPS SBI RBI SSC FCI RRB RAILWAYS

25

Daily Visit:

[GOVERNMENTADDA.COM]

Option C

Directions (1 – 5): Study the following information carefully and answer the questions given below: Following are the criteria to recruit a Finance Consultant in an organisation: The candidate must (a) be a graduate in any discipline with minimum 65% marks. (b) be above 26 years and not above 35 years of age as on 01-01-2017. (c) have a postgraduate degree or diploma (2 years full time in Finance) or must have completed CA/CS/ ICWA with above 50% marks. (d) have an experience of minimum three years in the above areas. (e) be ready to be on probation for one year. If a candidate satisfies all the criteria except (i) at (c) above, but is a Commerce graduate with minimum 70% marks and has been working in financial areas for at least last five years, his/her case is to be referred to the Asst General Manager – Personnel and Administration (PA). (ii) at (d) above, but has scored minimum 70% in post graduation (Finance) and has a minimum of two years of experience in Finance, his/her case is to be referred to the General Manager (PA). In each question below are given details of one candidate. You have to take one of the following courses of action based on the information provided and the conditions and sub conditions given above and mark the number of that course of action as your answer. You are not to assume anything other than the information provided in each question. All these cases are given to you as on 01-01-2017. 1. Rahul is a Science graduate with 65% marks and has completed post-graduation in Finance. He completed 26 years of age in 2016 and has four years’ experience as a Senior Manager. He is ready to join on one-year probation. A) if the candidate is to be selected. B) if the data given are not sufficient to take any decision. C) if the case is to be referred to the Asst General Manager. D) if the case is to be referred to the General Manager. E) if the candidate is not to be selected. View Answer Option A 2. Mukul is a gold medalist CA and Class I graduate with 77% and 82% marks respectively. He has been working as a professor in Finance for the past four years and has also been a freelance Finance Consultant for the past two years. A) if the case is to be referred to Executive Head. B) if the case is to be referred to Vice President. C) if the candidate is to be selected. D) if the information is inadequate to take a decision. E) if the candidate is not to be selected. Governmentadda.com | IBPS SBI RBI SSC FCI RRB RAILWAYS

26

Daily Visit:

[GOVERNMENTADDA.COM]

View Answer Option D 3. Shivani, a 27-year-old Commerce graduate with 62% post-graduated in Finance with 72% marks. She has been working with a Finance organisation for the last two years. She is ready to join on oneyear probation. A) if the case is to be referred to Executive Head. B) if the case is to be referred to Vice President. C) if the candidate is to be selected. D) if the information is inadequate to take a decision. E) if the candidate is not to be selected. View Answer Option E 4. Avneesh is a Commerce graduate with 74% marks and has been working for the past eight years during which he worked as Accounts Manager for two years. He is pursuing ICWA. He was born on 05.10.1985 and is ready to join on one-year probation. A) if the case is to be referred to Executive Head. B) if the case is to be referred to Vice President. C) if the candidate is to be selected. D) if the information is inadequate to take a decision. E) if the candidate is not to be selected. View Answer Option E 5. Aditya has been working as Senior Manager for the past eight years. He has completed MBA Finance and graduation with 70% marks each and is ready to join on probation for a year. He was born on 19th March 1984. A) if the case is to be referred to Executive Head. B) if the case is to be referred to Vice President. C) if the candidate is to be selected. D) if the information is inadequate to take a decision. E) if the candidate is not to be selected. View Answer Option A Directions (6 – 10): Study the following information carefully and answer the questions given below: A software company has decided to recruit engineers. The following criteria is to be applied. A candidate must Governmentadda.com | IBPS SBI RBI SSC FCI RRB RAILWAYS

27

Daily Visit:

[GOVERNMENTADDA.COM]

(i) be an engineering graduate with at least 60% marks in graduation and 80% marks in HSC. (ii) have work experience of minimum 1 year. (iii) be ready to sign a bond of three years. (iv) not be more than 28 years and not less than 21 years as on 1.2.2017. However, if the candidate fulfils all the above criteria except (a) (i), but has obtained 50% in graduation and 70% in HSC and has at least three years, experience of working, the case may be referred to the Director. (b) (iii), but is willing to pay an amount of ?1 lakh if he is required to leave, the case may be referred to the President. (c) (ii), but is a computer engineer, the case is to be referred to the DGM. In each question below are given details of one candidate. You have to take one of the following courses of action based on the information provided and the conditions and sub-conditions given above and mark the number of that course of action as your answer. The reference date is 1.2.2017. You are not to assume anything other than the information provided in each question. 6. Kashish is an Electrical Engineer and has been working as an Assistant Engineer for the past two years. She had scored 82% and 87% marks in graduation and HSC respectively. She has just completed 25 years of age. A) if the candidate is to be selected. B) if the case is to be referred to the President. C) if the case is to be referred to the Director. D) if the case is to be referred to the DGM. E) if the candidate is not to be selected. View Answer Option E 7. Rohit is 26 years old and has been working in a software company for the past three years. He has scored 74% and 81% marks in HSC and graduation respectively. He is willing to sign a bond with the company. A) if the candidate is to be selected. B) if the case is to be referred to the President. C) if the case is to be referred to the Director. D) if the case is to be referred to the DGM. E) if the candidate is not to be selected. View Answer Option C 8. Navneet is a Mechanical Engineer with 65% marks in graduation and 87% marks in HSC. He completed his engineering degree in 2013 at the age of 22 years and immediately started working in an engineering firm. He is interested in going to France and is not ready to sign a bond. However, he does not mind to pay an amount of Rs.1 lakh. A) if the candidate is to be selected. B) if the case is to be referred to the President. Governmentadda.com | IBPS SBI RBI SSC FCI RRB RAILWAYS

28

Daily Visit:

[GOVERNMENTADDA.COM]

C) if the case is to be referred to the Director. D) if the case is to be referred to the DGM. E) if the candidate is not to be selected. View Answer Option B 9. Karishma is an engineering graduate with 64% marks in degree and 92% marks in HSC. She joined an engineering firm two years ago at the age of 24 years. She is ready to sign a bond with the company. A) if the candidate is to be selected. B) if the case is to be referred to the President. C) if the case is to be referred to the Director. D) if the case is to be referred to the DGM. E) if the candidate is not to be selected. View Answer Option A 10. Ravina is a Computer Engineer and has obtained 61% and 83% marks in graduation and HSC respectively. She is 27 years old and can sign a bond. A) if the candidate is to be selected. B) if the case is to be referred to the President. C) if the case is to be referred to the Director. D) if the case is to be referred to the DGM. E) if the candidate is not to be selected. View Answer Option D

Governmentadda.com | IBPS SBI RBI SSC FCI RRB RAILWAYS

29

GovernmentAdda.com

QUANTITATIVE APTITUDE – 250 WORD PROBLEMS

Like My Facebook Page

1. Rahul invested 20% more than Mohit. Mohit invested 10% less than Ragu. If the total sum of their investment is Rs.17880, how much amount did Raghu invest? Answer: Let the investment made by Raghu be 100x, so Mohit’s investment = 90x and rahul’s investment =108x 108x+90x + 100x = 298x = 17880 X=60 So, Raghu investment = 100x = Rs.6000 2. A, B, C and D are four salesman in the first month they received a commission of Rs.3200 from their company and divided it in the ratio of 2 : 3 : 4 : 7 in the second month the commission doubled, the amount was doubled in the ratio 3 : 4 : 5 : 4. In the third month the commission tripled when compared to the first month and they shared in the ratio of 4 : 7 : 3: 2 and in the fourth month the commission became half of the previous month and they shared it in the ratio of 4 : 3 : 5 : 4 . What was the average monthly earning of C over the period? Answer: Total commission in first month = Rs.3200 Total commission in second month = Rs.6400 Total commission in thired month = Rs. 9600 Total commission in fourth month = Rs.4800 “C’s share in the commission =” 4/16 of 3200+ 5/16 of 6400+ 3/16 of 9600+ 5/16 “of 4800” =800 + 2000 + 1800 + 1500 = Rs.6100 “C’s average monthly earnings = ” “6100” /4 “= Rs.1525.” 3. Two shops A and B marked the same brand of jeans for Rs.900. Shop A offers successive discounts of 15% and 15% While shop B offers successive discounts of 20% and 10%. Then the difference in the selling price of jeans is? Answer: Final selling price of jeans in shop A = (900- 900×0.15)- (900-900×0.15)×0.15 = Rs. 650.25 Final selling price of jeans in shop B = (900-900×0.20)- (900-900×0.20)×0.10= Rs.648 Difference = Rs.2.25 4. Let us assume that 20g of sugar dissolves in 100g of water. Even an extra pellet will remain undissolved and sediment at the bottom of the solution. Now, water starts evaporating from 1kg of 7% solution at the rate of 32.5g per hour. After how long will the sediments start to evaporate? Answer: Sedimentation occurs when more than 20g of sugar is present in 100g of water. Amount of sugar in 1kg of water = 70g “Amount of water needed for sedimentation to start = ” “70x×100″ /20 ” = 350g” Amount of water that should evaporate = 1000-350 = 650g “Time required for eveporation =” ” 650″ /”32.5 ” “= 20 hours.” 5. Two liquids A and B are in ratio 4:1 in container X and 3:5 in container Y. In what ratio should the content of both container be mixed so that the resulting mixture has A and B in ratio 2:3? Answer: Let the ratios in which they are mixed is x and y Therefore A=4/5x+3/8y B=1/5x+5/8y Now A/B=2/3 On solving we get x:y=1:16 6. 8 women can complete the work in 10 days and 5 men can complete the work in 8 days where as 25 children can complete in 4 days. 16 women,4 men and 20 children work together for 2 days.If only women were to complete the remaining work in 1 day, how many women would be required? Answer: (M1)(H1)(D1)/(W1)=(M2)(H2)(D2)/(W2) Hence (8*10)W=(5*8)M=(25*4)C 4W=2M=5C Now 16W+4M+20C=16W+8W+16W=40W 8W one day work=1/10 40W one day work=(1/10)*(40/8)=1/2 40W 2 day work=1 Remaining Work=0 Work is already completed. 7. A tank has a leak which would empty it in 6 hrs. A tap pumps water @ 8 litres/ minute into the tank, and it is now emptied in 12 hrs. What is the capacity of tank?

Page 1

GovernmentAdda.com

QUANTITATIVE APTITUDE – 250 WORD PROBLEMS

Like My Facebook Page

Answer: In the absence of leak time taken by tap to fill tank=12*6/12-6=12hour Water filled in 1 hour=8*60=480L Therefore water filled in 12 hour=12*480=5760L 8. A man borrows Rs. 25,000 at 20% compound interest. At the end of every year he pays Rs. 5000 as part of repayment. How much does he still owe after three such installments? Answer: C.I of 20000 in 3 years =25000*(1+20/100)^3=43200Rs But as we are paying 2000Rs at the end of every year hence that should should be subtracted at the end of every year and the CI on remaining amount must be calculated. Therefore CI of 2000Rs that is paid at the end of 1st year=5000*(1+10/100)^2=7200 CI of 2000Rs that is paid at the end of 2nd year=5000*(1+10/100)^1=6000 Hence due amount after 3rd payment=43200-(7200+6000+5000)=25000 9. In a triangle, two sides of right angle triangle are 8 cm and 6 cm. If the triangle is revolved along the 8 cm side, the curved surface area of the cone so formed will be Answer: Radius of cone =8cm Slant height=10cm Curved surface area=πrl=22/7*6*10=188.4 cubic cm 10. Anil and Ruhi started a business by investing Rs 2000 and Rs 2800 respectively. After 8 months, Anil added Rs 600 and Ruhi added Rs 400. At the same time Teena joined them with Rs 4200. Find the share of Teena if they get a profit of Rs 34,300 after a year. Answer: Share of Anil : Share of Ruhi : Share of Teena is 2000×8 + 2600×4 : 2800×8 + 3200×4 : 4200×4 33 : 44 : 21 so share of Teena = 21/(33+44+21) × 34300 = Rs 7350 11. A sum of Rs 7000 is deposited in two schemes. One part is deposited in Scheme A which offers 8% rate of interest. Remaining part is invested in Scheme B which offers 10% rate of interest compounded annually. If interest obtained in scheme A after 4 years is Rs 226 more than the interest obtained in scheme B after 2 years, find the part deposited in scheme B. Answer: (7000-x)*8*4/100 = x [ (1 + 10/100)2 – 1] + 226 70*8*4 – 32x/100 = 21x/100 + 226 2240 – 226 = 53x/100 2014 = 53x/100 So, x = Rs 3800 12. A work which is completed by 20 men in 8 days can be completed by 25 women 12 days. 16 men and 10 women start doing the work. After 3 days, they leave. If the remaining work is to be completed in 6 days by x number of men, find x. Answer: 20 men in 8 days so 16 men in 20 × 8/16 = 10 days and 25 women in 12 days so 10 women in 25 × 12/10 = 30 days So in 3 days, they complete (1/10 + 1/30) × 3 = 2/5 So remaining work = 1 – 2/5 = 3/5 20 m 1 work in 8 days and x men 3/5 work in 6 days So 20 × 8 × 3/5 = x × 6 × 1 So, x = 16 men 13. There are 140 tickets (numbered 1 to 140) in a bowl. Find the probability of choosing a ticket which bears multiple of either 3 or 7. Answer: Number of multiples of 3 in 140 = 140/3 = 46 Number of multiples of 7 in 140 = 140/7 = 20 Number of multiples of 3×7= 21 in 140 = 140/21 = 6 So required probability = (46+20 – 6)/140 = 60/140 = 3/7 14. A 48 litres solution contains liquids water and milk in the ratio 3 : 5. How much amount of milk is to be added so that amount of milk is 70% of the new solution? Answer: Water present in solution = 3/8 * 48 = 18 l Milk present in solution = 5/8 * 48 = 30 l Let x litres of milk to be added

Page 2

GovernmentAdda.com

QUANTITATIVE APTITUDE – 250 WORD PROBLEMS

Like My Facebook Page

Milk is to be 70% of new solution, so water is to be 30% of new solution. So 30/100 of new solution = Water present in new solution 30/100 * (48+x) = 18 So, x = 12 litres OR 70/100 of new solution = Milk present in new solution 70/100 * (48+x) = 30+x So, x = 12 litres 15. In a class, average age of 30 students is 18 years. If the age of 2 more students is taken into consideration, then the average of all students gets increase by 1. Find the average of the ages of those 2 students. Answer: 30 students – 18 32 students – 19 So total age of those 2 students = 30×1 + 19×2 = 68 So average = 68/2 = 34 16. The ratio of A’s age 3 years ago and B’s age 5 years hence is 3 : 4. The average of the ages of A and C is 20 years. Also C’s age after 10 years will be 2 more than twice the age present age of B. Find the age of C. Answer: (A – 3)/(B + 5) = ¾ => 4A - 12 = 3B + 15 => 4A – 3B = 27 --------> (1) (A + C)/2 = 20 => A + C = 40 ------> (2) C + 10 = 2B + 2 => B = (C + 8)/2 ---------> (3) From (1) and (2) (27 + 3B)/4 + C = 40 -------> (4) From (3) and (4) (27 + 3 (C + 8)/2) + 4C = 160 => C = 22 17. The circumference of a circle having radius equal to 35 cm is equal to the perimeter of a rectangle. If the area of rectangle is 2400 cm2, find the length of rectangle. Answer: 2 × 22/7 × 35 = 2 (l + b) so (l + b) = 110 also given, lb = 2400 So (l + 2400/l) = 110 So l2 – 110 l + 2400 = 0 So, l = 80 or 30. 18. The market price of an item is 20% more than its cost price. If after selling the item, the profit percent obtained is 10%, find the discount given. Answer: Use MP = (100+p%)/(100-d%) * CP So 120/100 * CP = (100+10)/(100-d%) * CP Solve, d% is 25/3% Let CP = Rs 100, so MP = Rs 120, and SP = Rs 110 So when discount % = (120-110)/120 * 100 = 25/3%, discount = Rs 10 19. A, B and C divide Rs 3900 among them in the ratio 4 : 4 : 5 respectively. Now if each of them got Rs 300 more, what will be the respective new ratio of dividing the total money among them? Answer: A got = [4/(4+4+5)] * 3900 = 1200, B got = [4/(4+4+5)] * 3900 = 1200, C got = [5/(4+4+5)] * 3900 = 1500 When 300 is added to their shares, A gets=1200+300 = 1500, B = 1500, C =1800 So new ratio is 1500 : 1500 : 1800 = 5 : 5 : 6 20. Mohan distributed his assets to his wife , four sons, three daughters and six grand children in such a way that each grand child got onesixteenth of each son and one-tenth of each daughter. His wife got 60% of the total share of his sons and daughter together. If each daughter receives assets of worth Rs.1.25 lakh, what is the share of his wife? Answer: “Share of 1 grand child = 1 /10×1.25lakh=0.125 lakhs Share of 1 son = 16 × 0.125 lakh = 2 lakhs Share of 4 sons= 4× 2lakhs= 8 lakhs Share of 3 daughters = 3×1.25 lakhs = 3.75 lakhs Total share of sons and daughters = (8+3.75)lakhs=11.75 lakhs 6/10×11.75 lakhs=Rs.705000.

Page 3

GovernmentAdda.com

QUANTITATIVE APTITUDE – 250 WORD PROBLEMS

Like My Facebook Page

21. There are 3 inlet pipes X, Y and Z connected to a tank. If only one pipe is opened at a time, then it takes 50, 40 and 25 minutes for pipes X, Y and Z respectively to fill the tank. Find the time taken to fill 99% of the tank if it is known that in every 5 minutes for the first 2 minutes pipe Y is opened and then closed for 3 minutes. The remaining pipes are always kept open. Answer: Part of the tank filled per minute by pipes X and Z respectively = 2% and 4% Pipe Y fills 5% of the tank for every 2 minutes it operates. In 5 minutes, the tank filled by X and Z = 30% and by pipe Y = 5% So, in 5 minutes , % of tank filled =30+5=35% In 10 minutes, the tank is filled 70% For next 2 minutes part of tank filled = 5+12 = 17% The remaining 12% is filled in time = 2 minutes Total time taken = 10+2+2 =14minutes 22. A certain number of people get together to contribute in the construction of a charity hospital. But every month four people step out of this plan. Due to this the task is completed in half more year instead of one year. Then how many people were originally involved in this plan? Answer: Let the total number of people = x Then, 12x = (x+ (x-4)+(x-8)+(x-12)+(x-16)+(x-20)+… 18 times) 12x= 18x-4(1+2+3+…17) 6x=(4×17×18)/2 x=102 23. Find the percentage by which the volume of the circular cylinder change assuming that the radius and the height of the circular cylinder decreases by 20%? Answer: 2 Volume = π r h Let the radius and height = 10 cm 2 So area = π×10×10×10 =1000 π cm After decrease New radius = 10-20× 10 /100 = 8 cm New height = 10-20× 10 /100 = 8 cm 2 New volume = π×8×8×8 = 512 π cm 2 Decresed volume = 488 π cm Percentage decrease = 488π× 100 /1000π =48.8% 24. A cyclist, cycling on a road, passes a man who was walking at the rate of 4 km/hr in the same direction. The man could see the cycle for 12 min and it was visible to him up to a distance of 1.2 km. What was the speed of the cycle? Answer: Let the speed of cycle be x km/h. Speed of man = 4 km/h Relative speed = (x-4) km/h Therefore, (x-4)×12/60= 1.2 x–4=6 x = 10 km/h 25. A man can walk up a moving “UP” escalator in 20sec and walk down this moving “UP” escalator in 60sec.Walking speed is same in case of both upwards and downwords.How much time will he take to walk up the escalator,when the escalrtor is stationary? Answer: Assume speed of escalator=x Speed of man=y Assume length of escalator=120 Then y+x=120/20=6 y-x=120/60=2 on solving y=4 ,x=2 Time taken by man when escaltor is stationary=120/4=30sec 26. If a 5 digit number is formed with digits 1,2,3,4 and 5.What is the probability that the number is divisible by 10,if repetition is not allowed. Answer: Total numbers=5!=120 For any number to be divisible by 10 the last digit has be zero,which is not in any case

Page 4

GovernmentAdda.com

QUANTITATIVE APTITUDE – 250 WORD PROBLEMS

Like My Facebook Page

27. In an election survey,30% people promised to vote candidate A and remaining promised to vote for candidate B. If on the day of election x% percent of people who promised to vote for A, voted for B and 40% of people who promised to vote B voted against him and in the end B lost by 10 votes.What is value of x,if total 250 votes were casted? Answer: A=75-20%of (75)+40% of (175)=130 B=175+20% of (75)-40% of (175)=120 28. If instead of normal weighing scale a shopkeeper uses forged scale.The shopkeeper uses 1.2Kg scale while buying and 800g scale while selling and in the end he offers 10 percent discount,what is his overall profit percentage(approx.)? Answer: let’s say the price of 1000g of good=1000Rs Now he gets 1200g of good at 1000Rs. Hence CP of shopkeeper for 1g=1000/1200=5/6Rs Cp of shopkeeper for 800gram=5/6*800=666.66Rs Now instead of selling 1000g he sells 800 gram for 900Rs(10% discount) Profit=900-666.66/666.66*100=35%(APPROX) 29. A merchant has 560 kg of wheat which he sells at 10% profit and rest at 18%,overall he gains 15%.The quantity sold at 10% profit is? Answer: 10…………………18 ……….15………… 3…………………..5 3/8*560=210 30. When Anil will become as old as his father is now, he will be four times the present age of his son and then his son will be nine years older than what Anil is now. If the sum of his father’s age and his age is 80 years old, then how old is Anil now? Answer: Let present age of Anil = x years So age of his father = (80-x) years Age of Anil’s son = (80-x)/4 When Anil will be as old as his father i.e. (80-x) years So age of Anil’s son = (80-x)/4 + (80-x-x) Now (80-x)/4 + (80-2x) = x + 9 This gives age of Anil, x = 28 31. After selling an article, it is found that profit is 20% more than the cost price of article. If the cost price is increased by 10% keeping the selling price same, then what percent of selling price is profit? Answer: Let CP = Rs 100, then profit = 20/100 × 100 = Rs 20, and SP = Rs 120 Now New CP = 110/100 × 100 = Rs 110, SP = Rs 120, so profit = 120-110 = Rs 10 Required % = 10/120 × 100 = 25/3 % 32. Two trains which are 150 m long each are moving in opposite directions. They cross each other in 12 seconds. If one train is moving two and a half times as fast the other train, then find the speed of the faster train. Answer: Let speed of slower train = x m/s, then speed of faster train = 2.5x m/s Their relative speed becomes = x+2.5x = 3.5x m/s So (150+150)/12 = 3.5x After solving, speed of slower train, x = 50/7 m/s So speed of faster train = 2.5 × 50/7 = 125/7 m/s = 125/7 × 18/5 km/hr 33. Rohit borrowed Rs. 6000 at 5% p.a. simple interest for 2 years. After that he invests it in a scheme which offers 7 ¼% p.a for 2 years. Find the profit of Rohit in the transaction per year. Answer: Profit in 2 years = [6000 × 29/4 × 2/100 – 6000 × 5 × 2/100] = 870 – 600 = Rs 270 So profit per year = 270/2 = Rs 135 34. A man can row at 15 km/hr in still water. If the velocity of current is 9 km/hr and it takes him 3 hours to row to a place and come back, how far is the place? Answer: 2 2 2 2 Distance = time × [B – R ]/ 2 × B = 3 × [15 – 9 ]/ 2 × 15 = 14.4 km 35. Twenty women can complete a work in 12 days and Twenty-four children can complete the same work in 15 days. How many days will thirty women and eighteen children take to complete the work?

Page 5

GovernmentAdda.com

QUANTITATIVE APTITUDE – 250 WORD PROBLEMS

Like My Facebook Page

Answer: 20 w in 12 days, so 30 w in 20×12/30 = 8 days 24 c in 15 days, so 18 c in 24×15/18 = 20 days So they will complete the work in 20×8/[20+8] = 40/7 days 36. Can A contains 20% water and rest milk. Can B contains 40% water. How much milk should be taken from both the cans and mix in can C to get 15 litres of milk such that the ratio of water to milk in can C is 3 : 7? Answer: Milk in can A is 80% or 80/100 = 4/5 Milk in can B is 60% or 60/100 = 3/5 Milk in final can C = 7/(3+7) = 7/10 So by Alligation method 4/5 3/5 . 7/10 1/10 1/10 which gives 1 : 1 so milk from can A is 1/2 × 15 = 7.5 l 2 3 37. The curved surface area of a cylindrical pillar is 616 m and its volume is 2156 m . Find the ratio of its diameter to its height. Answer: 2 πr h/2πrh = 2156/616 So, radius, r = 7 m 2 × 22/7 × 7 × h = 616 So, height h = 14 Ratio: 2r/h = 14/14 = 1/1 38. The average age of 10 men increases by 1.5 years when a new person comes in place of one of them whose age is 34 years. What is the age of the new person? Answer: Total age increased = 10 × 1.5 = 15 years So age of new person = 34 + 15 = 49 years 39. In a box, there are 6 black, 4 blue and 2 red marbles. One marble is picked up randomly. What is the probability that it is neither black nor red? Answer: Neither black nor red means the ball should be blue 4 12 So probability = C1/ C1 = 4/12 = 1/3 40. The average age of Abhilasha and Aadhira is 35 years. If Aaloka replaces Abhilasha, the average age is 31 years, if Aaloka replaces Aadhira average age is 36 years. If the average age of Aditi and Aashirya is half of average age of Abhilasha, Aadhira and Aaloka. then average age of all the five people is Answer: Abhilasha, Aadhira, Aaloka, Aditi, Aashirya – X, Y, Z, P, Q X + Y = 35 * 2 =70 –(1) Z + Y = 31 * 2 =62 –(2) X + Z = 36 * 2 = 72 –(3) From (1) (2) and (3) X = 40 ; y =30; Z = 32 Average age of P and Q =1/2 * [( X + Y + Z)/3] = 102/6 = 17 Sum of the age of P and Q = 34 Average age of all the five people = (34 + 102)/5 = 27.2 41. A bag contains 6 red balls, 11 yellow balls and 5 pink balls. If two balls are drawn at random from the bag, one after another, what is the probability that the first ball is red and the second ball is yellow? Answer: Total of balls = 6 + 11 + 5 = 22 22 n(S) = C2 = (21x22) / 2 = 231 6 11 Now, n(E) = C1 x C1 = 6 x 11 = 66 P(E) = n(E)/n(S) = 66/231 = 6/21 = 2/7 42. The sum of the radius and the height of a cylinder is 19m. The total surface area bf the cylinder is 1672 m2, what is the volume of the cylinder? (in m3) Answer: Let the radius of the cylinder be r and height be h.

Page 6

GovernmentAdda.com Then, r + h = 19

QUANTITATIVE APTITUDE – 250 WORD PROBLEMS

Like My Facebook Page

…..(i) 2

Again, total surface area of cylinder = (2πrh + 2πr ) Now, 2πr(h + r) = 1672 or, 2πr x 19 = 1672 or, 38πr = 1672 , πr = (1672/38) = 44m, r = (44 × 7) / 22 = 14 Height = 19 - 14 = 5m Volume of cylinder = πr h = (22/7) x 14 x 14 x 5 =14m = 22 × 2 × 14 × 5 = 3080m 2

3

43. The ratio of the speed of the boat upstream to the speed of the boat downstream is 2 : 3. What is the speed of the boat in still water if it covers 42 km downstream in 2 hours 20 minutes? (in km/h) Answer: Let the speed of the boat in still water be x and that of the current be y. Then, downstream speed = x + y and upstream speed = x - y Now, downstream speed = 42 / [2 20/60] = (42 × 3) / 7 =18 km x+y=18 Again, 3 : 18, 2 : 12 (As ratio of downstream to upstream is 2 : 3) x - y = 12 Solving (i) and (ii), we get (x+y=18) + (x - y =12) = 2x =30 x = 15 kmph Hence speed of the boat 15 kmph 44. 35 men complete a piece of work in 16 days and 20 women complete the same piece of work in 30 days. What is the ratio of the amount of work done by 40 men in 1 day to the amount of work done by 15 women in 1 day? Answer: 35 men complete the work in 16 days. 1 man completes the work in 16 x 35 days, 32 men complete the work in (16x35)/40 = 14 days. Again, 20 women complete the same piece of work in 30 days. 1 woman completes the same piece of work in 20 × 30 days. 15 women can complete the work in (20x30)/15 = 40 days. Ratio = 1/14 : 1/40 = 40 : 14 = 20 : 7 45. A man sold an article for Rs. 6800 and incurred a loss. Had he sold the article for Rs.7850, his gain would have been equal to half of the amount of loss that he incurred. At what price should he sell the article to have 20% profit? Answer: Let the cost price be x. Then, loss = (x - 6800) Again, profit = (7850 - x) Now, (7850 - x) = (x - 6800)/2 or, 15700 - 2x = x - 6800 or, 3x = 15700 + 6800 = 22500 => x = 22500/3 = 7500 Selling price = (7500x120)/100 = Rs. 9000 46. A bought a certain quantity of bananas at a total cost of Rs. 1500. He sold 1/3 of these bananas at 25% loss. If he earns an overall profit of 10%, at what percentage profit did A sell the rest of the bananas? Answer: Total CP = 1500 Total SP = 1500 + 10% of 1500 = 1500 + 150 = 1650 CP of 1/3 of bananas = 1500/3 = Rs.500 SP of 1/3 of bananas at 25% loss = 500 – [ (500 x25 / 100)] = 500 - 125 = 375 SP of the rest of bananas = 1650 - 375 = 1275 Now, CP of the test of bananas = 1500 - 500 = 1000 Profit on the rest of bananas = 1275 -1000 = 275 % of profit on the rest of bananas = (275/1000)×100 = 27.5% 47. A tank has two inlets: P and Q. P alone takes 6 hours and Q alone takes 8 hours to fill the empty tank completely when there is no leakage. A leakage was caused which would empty the full tank completely in ‘X’ hours when no inlet is open. Now, when only inlet P was opened, it took 15 hours to fill the empty tank completely. How much time will Q alone take to fill the empty tank completely? (in hours) Answer:

Page 7

GovernmentAdda.com

QUANTITATIVE APTITUDE – 250 WORD PROBLEMS

Like My Facebook Page

(1/P) – (1/X) = (1/15) Or, (1/6) – (1/X) = (1/15) (P = 6 hours) Or, (1/X) = (1/6) – (1/15) = (10-4)/60 = 1/10 x = 10 hours Now, (1/Q) – (1/10) = (1/8) – (1/10) = (5-4)/40 = 1/40 Hence, Q fills the tank in 40 hours. 48. At present, the ratio of the ages of A to B is 3 : 8; and that of A to C is 1 : 4. Three years ago, the sum of the ages of A, B and C was 83 years. What is the present age (in years) of C? Answer: According to the question, A : B = 3 : 8 A:C=1:4 B:A=8:3 A:C=1:4 8 : 3 : 12 Sum = 8x + 3x 12x = 23x Now, 23x = 92 x=4 Hence the present age of C = 12x = 12 x 4 = 48 years 49. The sum invested in Scheme B is thrice the sum invested in Scheme A. The investment in Scheme A is made for 4 years at 8% p.a. simple interest and in Scheme B for 2 years at 13% p.a. simple interest. The total interest earned from both the schemes is Rs.1320. How much amount was invested? Answer: Let the amount invested in scheme A be Rs.x and that in B be Rs. 3x. Then, [(x × 4 × 8)/100] [(3x × 2 × 13) /100] = 1320 Or, (32x/100) + (78x/100) = 1320 110x/100 = 1320 x = (1320 x 100) / 110 = Rs. 1200 50. Kim and Om are travelling from point A to B, Which are 400 km apart. Travelling at a certain speed Kim takes one hour more than Om to reach point B. If Kim doubles her speed she will take 1 hour 30 mins less than Om to reach point B. At what speed was Kim Driving from point A to B? (in kmph) Answer: Let the speed of Kim be x and that of Om be y. Then, (400/x) – (400/y) = 1 Let 1/x = u and 1/y = v 400u — 400v = 1 …(i) Again, (400/y) – (400/2y) = 3/2 400v – 200u = (3/2) Or, 800v – 400u = 3 …(ii) Solving (i) and (ii), we get (400u - 400v =1) + (-400u + 800v) = 400v = 4 v = (4/400) = (1/100) km y = 100 km now, (400/x) – (400/100) = 1 or, (400/x) = 5 x = 80 kmph 51. Find the number of words formed with the letters of the word 'BOOKS' beginning with B and ending with S. Answer: We have to arrange 3 letters (O, O and K) out of which 'O' occurs two times. So, reqd no. = 3! / 2! = 3 ways 52. A box contains 5 Sony, 6 Samsung and 4 Sandisk pen drives. 3 pen drives are drawn at random. What is the probability that they are not of the same company? Answer: The total number of pen drives = 5 + 6 + 4 = 15 n(S)= 15C3 = (15x14x13) / (3×2) = 455 Now, 3 pen drives out of 15 pen drives can be drawn in 455 ways. If all 3 pen drives are of the same company

Page 8

GovernmentAdda.com

QUANTITATIVE APTITUDE – 250 WORD PROBLEMS

Like My Facebook Page

It Can be done in 5C3 + 6C3 + 4C3 = 10 + 20 + 4 = 34 ways Probability that all 3 pen drives are not of the same company = 1 – (34/455) = (421/455) 53. The base of a triangular field is 660 metres and height 440 metres. If the charges for watering the field are at the rate of Rs.26.5 per sq hectometre, find the total cost to water the triangular field. Answer: Area of the field = (Base x Height) / 2 = (660 x 440)/2 sq metre = (660 x 440)/( 2x100x100) sq metre = 14.52 sq hectmetres Cost of watering 1 sq hectometre = Rs.26.5 Cost of watering the field = 26.5 x 14.52 = Rs.384.78 54. In a mixture of milk and water the proportion of milk by weight was 70%. If in a 250-gm mixture 100 gm water was added, what would be the percentage of water? Answer: Proportion of milk in the mixture =250 x (70/100) = 175gm Water = 75 gm After 100 gm water added in mixture the percentage of water = (75 +100) / (250+100) x 100 = (175/350) x 100 = 50% 55. Two pipes can fill a tank in 28 and 24 minutes respectively and a waste pipe can empty 5 gallons per minute. All three pipes working together can fill the tank in 16 minutes. How much time is taken by the waste pipe to empty the full tank? Answer: Let the capacity of the tank be 336 litres LMC of (28, 24 and 16 = 336) Waste pipe empties the tank in (12 + 14 - 21) 5 litres per minute Waste pipe empties the tank in (336/5) = 67.2 minutes 56. The average score of a cricket player after 24 innings is 25 and in the 25th innings the player scores 25 runs. In the 26th innings what minimum number of runs will be required to increase his average score by 2 than it was before the 26th innings? Answer: The average score of a cricket player after 25th Innings = (24 * 25 + 25) / 25 = 25 Required Run = X (625 + X)/26 =27 X = 26 * 27 – 625 = 77 57. There are two vessels P and Q filled with cooking oil with different prices and with volumes 160 and 40 liters respectively. Equal quantities are drawn from both P and Q in such a manner that the cooking oil drawn from P is poured in into Q and oil drawn from Q is poured into P. If the price per liter becomes equal in both vessels. What is the (equal) quantity that was drawn from both P and Q? Answer: Vessel (P) Vessel (Q) Quantity= 160 l Quantity= 40 l rate – p rate – q Let quantity taken out from both = a litres ‘a’ litres removed from p and ‘a’ litres added from q So rate of vessel P after removal and then addition = [(160-a)p + aq]/160 Similarly rate of vessel Q after removal and then addition = [(40-a)q + ap]/40 Now equate these equations [(160-a)p + aq]/160 = [(40-a)q + ap]/40 Solving, we get a = 32 l 58. A book seller sold a book at Rs. 56 in such a way that his percentage profit is same as the cost price of the book. If he sells it at twice the percentage profit of its previous percentage profit then new selling price will be? Answer: CP = x SP = x + (x * x)/100 = 56 x2 + 100x – 5600 = 0 x = 40 SP = 40 + (40 * 80)/100 = Rs. 72 59. A circular road runs round a circular playground. If the difference between the circumferences of the outer circle and the inner circle is 132 metres, then what is the width of the road? Answer: Width of the Road = R – r 2πR – 2πr = 132 R – r = 132 * (7/44) = 21 m

Page 9

GovernmentAdda.com

QUANTITATIVE APTITUDE – 250 WORD PROBLEMS

Like My Facebook Page

60. There are two concentric circles whose areas are in the ratio of 16:25 and the difference between their diameters is 8 m. Find out the area of the inner circle? Answer: 2 2 r /R = 25/16 => r/R = 5/4 5x – 4x = 4 x=4 Inner Radius = 16m 2 Area of Inner Circle = Π (16 * 16) = 256πm 61. Two-thirds of a commodity was sold at a profit of 5% and the remainder at a loss of 2%. If the total profit was Rs.400, what was the cost of the commodity? Answer: Iet the cost of commodity be Rs.x Then , (2x/3) × 1.05 + (1x/3) × 0.98 = x + 400 Or, (1/3) × (3.08) = x + 400 Or, 0.8 x = 400 × 3 Or, (1200 / 0.08) = (120000 / 8) x = Rs.15000 62. A car covers a distance between A and B in 45 minutes. If the speed of the car is reduced by 8 km per hour then the same distance is covered in 49.5 minutes. What is the distance between A and B? Answer: Let the distance between A and B be d km Then. [ d/(45/60)] – [d / (49.5/60)] = 8 Or, 4d/3 – 120d / 99 = 8 Or, (132d – 120d) / 99 = 8 D = (8 × 99) / 12 = 66 km 63. If the difference between the simple interest and the compound interest earned on a certain amount @ 12% at the end of 3 years is Rs.336.96, then what is the amount? Answer: D = Rs. 336.96 T = 3 years R= 12% 3 2 P = Difference × (100) / r (300 + r) = (336 .96 × 1000000) / [144(300+12) = 336960000/44928 = Rs. 7500 64. A race track is in the form of a ring whose inner circumference is 396m and outer circumference is 418m. Find the width of the track. Answer: Circumference of outer track 418 2πR = 418 R = (418 × 7) / 44 = 66.5 Circumference of inner track = 396 2πr = 396 r = (396 × 7) / 44 = 63 width of the track = 66.5 – 63 = 3.5m Note: instead of calculating in two parts perform a single calculation like Width = R – r = 7/44 (418 - 396) = (7/44) × 22 = 3.5m 65. The average age of Ram, Shyam and Ghanshyam is 26years. 3 year ago, average age of Ram and Shyam was 21yrs. 4 years hence the average age of Shyam and Ghanshyam will be 28 years. Find the present age of Shyam? Answer: average age of all three = 26 ∴ total age of all three= 26×3 = 78 3yrs ago, average age of Ram and Shyam = 21 3yrs ago, total age of Ram and Shyam =21*2= 42 ∴ present total age of Ram and Shyam = 42+6=48 ∴ present age of Ghanshyam=78-48=30 Similarly present age of Ram=30 hence,Present age of shyam is =78-30-30=18 years 66. If another guy Danpat joins in who is 2 year younger than Esha and the average of Esha and Raman was 27,two years ago.Also it is given that average of Raman and Ram 4 year hence will be 25.So what is average of present age of Danpat,Raman and Esha? Answer:

Page 10

GovernmentAdda.com

QUANTITATIVE APTITUDE – 250 WORD PROBLEMS

Like My Facebook Page

Ram age 4 year hence will be 34 Average of Ram and Raman 4 year hence=25 Total age of Ram and Raman 4 year hence=50 Therefore Raman age 4 year hence will be=16 Raman present age=12 Raman age 2 year ago=10 Therefore esha age 2 year ago was=54-10=44 Esha current age=46 Danpat current age=44 Average of all three=(46+44+12)/3=34 67. 36 women can do a work in X days and 30 men can do the same work in (X -4 ) days. The ratio of work done by 10 men and 12 women in the same time is 2:1. What is the value of X? Answer: M1D1T1/W1=M2D2T2/W2 [men1*day1*time1/work1= men2*day2*time2/work2] 10M/2=12W/1 5M=12W…………………(i) Also 36W*X=30M*(X-4) From (i) 15M*X=30M*(X-4) X=8 68. A boat travelling at a speed of 60 kmph started at 3 p.m. when there was no current from point X for point Y which is 240 km apart. After some amount of time current started which delayed the entire journey by 15 minutes.Find the time at which current started if the speed of boat is 6 times to the speed of current? Answer: As time is increased boat will be travelling against the current t1+t2=17/4 [t1=time till no current,t2=time after current] 60t1+50t2=240 On solving t1=11/4 hour Hence current started at 3+11/4=5:45 p.m 69. A boat takes 58 hours for travelling downstream from Point X to point Y and coming back to point Z midway between X and Y. If the speed of the stream is 4 kmph and speed of the boat in still water is 11 kmph, then what is the distance between X and Y? Answer: Speed downstream = 11 + 4 = 15 kmph. Speed upstream = 11 – 4 = 7 kmph. Let distance between P and Q be ‘x’ km, then, x/15 + (x/2)/7 = 58. i.e., x/15 + x/14 = 58. Solving we get, x = 420 km. 70. A boat takes 4 hours more while going back in upstream than in downstream when the distance between two places is 32 km and the speed of boat in still water is 6kmph. What must be the speed of boat in still water so that it can row downstream, 32km in 4 hours? Answer: 32/(6-R) – 32/(6+R) = 4 R = 2kmph (B + 2) = 32/4 Speed of boat in still water = 6kmph 71. A milkman mixes 20 lites of water with 80 litres of milk. After selling one-fourth of this mixture, he adds water to replenish the quantity that he has sold. What is the current proportion of water to milk ? Answer: 1/4th of the mixture is sold 1/4th of milk and 1/4th of water is sold. = 3/4th of milk = (3/4)x80 = 60 litres of milk is remaining and rest part 100 - 60 = 40 litres is water (as water is a added in place of milk) Reqd ratio = 40 : 60 2 : 3 72. Ajay and Bala invest Rs. 4000 and Rs. 5000 in a business. Ajay receives Rs. 20 per month out of the profit as remuneration for running the business and the rest of profit is divided in proportion to the investment. In a year Ajay totally receives Rs. 672. What does Bala receives? Answer: Annual profit = x Ratio of profit share between Ajay and Bala = 4 : 5

Page 11

GovernmentAdda.com

QUANTITATIVE APTITUDE – 250 WORD PROBLEMS

Like My Facebook Page

Ajay gets: 20 * 12 + 4/9 * x = 672 Solving, we get, x = 108*9 So Bala gets = 5/9 * x = 5/9 * 108*9 = Rs 540 73. Angel, Beaula and Catherine entered into a partnership in a business. Angel got 5/7 of the profit. Beaula and Catherine distributed the remaining profit equally. If Catherine got Rs.500 less than Angel, then the total profit was? Answer: Total Profit = x Angel’s Share = (5x/7) Remaining Profit = x – (5x/7) = (2x/7) Beaula and Catherine distributed the remaining profit equally- x/7 , x/7 (5x/7) -(1x/7)= 500 (4x/7) = 500 x = 500 * (7/4) = 875 74. A shopkeeper buys an article at a discount of 20% on the listed price from a wholesaler. The shopkeeper marks up the price by 15% on the listed price. A buyer pays Rs.3795 to get it after paying sales tax at the rate of 10% on the price asked for. Find the profit percentage of the shopkeeper. Answer: Let the listed price = Rs. 100 CP of shopkeeper = 100 – 20 = Rs. 80 Marked price by shop keeper = 100 + 15 = Rs. 115 Now, 115 = 3795 x (100/110) = 3450 80 = (3450/115) × 80 = Rs. 2400 CP of shopkeeper = Rs. 2400 Profit = 3450 - 3400 = 1050 Profit % = (1050 / 2400) x100 = 43.75% 75. A sum amounts to Rs.10580 in 2 years and to Rs.12167 in 3 years compounded annually. Find the sum and the rate of interest per annum. Answer: 1 12167 = 10580 [1+ (r/100)] Or, (12167/10580) = 1+(r/100) Or, (1587/10580) = r/100 r = (1587×100) / 10580 = 15% sum = (10580x100x100) / (115 × 115) = Rs. 8000 Sum = Rs. 8000, Rate = 15% 76. Mohit travels 972 km in 10.5 hours in two stages. In the first part of the journey, he travels by bus at the speed of 78 km, per hour. In the second part of the journey, he travels by train at the speed of112 km/hr. How much distance does the travel by train? Answer: We use only alligation on speed (km/hr) to get ratio of time spent in bus and train. overall speed = (972/10.5) = (1944/21) = 648/7

136/7 : 102/7 à 136 : 102 Or 4 : 3 Time spent in train = 10.5 (3/7) = 4.5 hours Distance travelled by train = 112 x 4.5 hours = 504 km 77. A contractor undertook to do a certain work in 75 days and employed 48 men to do it. After 55 days he found that only (2/3) of the work was done. How many more men must be employed so that the work may finished in time? Answer: Apply M1D2 / W2 = M2D2 / W2 Or, (48x55) / (2/3) = (Mx20) / (1/3) M = 66 men Reqd more men = 66 - 48 = 18 men

Page 12

GovernmentAdda.com

QUANTITATIVE APTITUDE – 250 WORD PROBLEMS

Like My Facebook Page

78. 49 pumps can empty a reservoir in 17/2 days working 6 hours a day. If 119 pumps are used for 7 hours a day then in how many days will the same work be completed? Answer: Let the required number of days be x. 49 x (17/2) x 6 = 119 x 7 x x x = 3 days 79. 6 kg of an alloy A is mixed with 8 kg of alloy B. If alloy A has lead and tin the ratio 1 : 3 and B has tin and copper in the ratio 2 : 3, then what is the amount of tin in the new alloy? Answer: Quantity of tin in alloy A = 6 × (3/4) = 4:5 kg Quantity of tin in alloy B = 8 × (2/5) = 3.2kg Quantity of tin in the new alloy = 4 . 5 + 3.2 =7.7 kg 80. There are 5 boys and 4 girls. In how many ways can they be seated in a row so that all the girls do not sit together? Answer: Total number of persons 5 +.4 = 9 When there is no restriction they can be seated in a row in 9!.Ways. But if all the 4 girls sit together, we can consider the group of 4 girls as one person. Therefore, we have only 5 + 1 = 6 persons Number of ways = 6! Ways But 4 girls can be arranged among themselves in 4P4 = 4! Ways Reqd no.of ways in which all the 4 girls do not sit together = 9! – 6! × 4! = 9 × 8 × 7 × 6! – 6! × 24 = 6! (504 - 24) = 720 × 480 = 720 × 480 = 345600 Directions (81 – 85) Study the following passage and answer the questions accordingly. Five members of a family live in Mumbai namely A, B, C, D and E. A and B together can do a piece of work in 80 days. B and C together can do a piece of work in 60 days. C and D together can do a piece of work in 40 days. D and E together can do a piece of work in 20 days. A alone can do a piece of work in 120 days. 81. If A, B and C together can do a piece of work in ‘x’ days then how much work could be done in the same days when E do the same work? Answer: According to question, (A+B+C)’s one day work=1/120+1/240+1/80=(2+1+3)/240=6/240=1/40 Required Answer, E alone works to finish, E=(1/40)/(3/80)=1/40×(80/3)=2/3 of the work 82. B, C and D can complete a piece of work in ‘x’ days. If all of them work together and after three days B left and the remaining work be completed by C and D with help of E. In how many days can C, D and E do the remaining work? Answer: (B+C+D)’s one days work=1/240+1/80+1/80=7/240 According to quesiton, (B+C+D)’s three days work=7/240×3=7/80 Then, remaining work, =1-7/80=73/80 Required answer is, (73/80)/(1/80+1/80+3/80)=73/80×80/5=73/5=14 3/5 days 83. A, C and D can do a piece of work in x, y and z days, respectively. They work alternately in a way that first day A , second day C and third day D, fourth day A and so on. How many days will be needed to complete the work in this way? Answer: A’s one day work=1/120; C’s one day work=1/80; D’s one day work=1/80 According to question, work done in first 3 days =1/120+1/80+1/80=(2+3+3)/240=8/240=1/30 Time taken to complete 1/30 part of work=30 days Required Answer, (Time taken to complete the whole work)=3×30=90 days 84. A, B and C can do a piece of work in ‘x’ days, ‘y’ days and ‘z’ days respectively. As they were ill, they could do 90% , 75% and 80% of their efficiency, respectively. How many days will they take to do the work together? Answer: According to question, A’s one day work=90% of 1/120=90/100×1/120=3/400 B’s one day work=75% of 1/240=75/100×1/240=1/320 C’s one day work=80% of 1/80=80/100×1/80=1/100

Page 13

GovernmentAdda.com

QUANTITATIVE APTITUDE – 250 WORD PROBLEMS

Like My Facebook Page

(A+B+C)’s one day’s work=3/400+1/320+1/100=(12+5+16)/1600=33/1600 Hence, time taken by them to complete the work=1600/33=48 16/33 85. C can do 1/4 of a work in 80 days, D can do 40% of the same work in 80 days and E can do 1/3 of a work in 800/3 days. Who will complete the work first? Answer: Time taken to complete the work by C=80×4=320 days Time taken to complete the work by D=80×100/40=200 days Time taken to complete the work by E=800/3×3=800 days 86. A box contains 6 black and 14 white balls, out of which 3 black and 5 white balls are defective. If we choose two balls at random, what is the probability that either both are white or both are non-defective? Answer: 14 20 12 20 9 20 Required probability- C2/ C2 + C2/ C22 – C2/ C2 = 121/190 87. In a class, the average age of some boys is 16 years, and average age of 16 teachers is 56 years. If the average age of the combined group of all the teachers and boys is 20, then the number of students is Answer: Use allegation method number of boys ‘x’ : number of teacher’16’ . 16 56 . 20 . (56-20)=36 (20-16)=4 . So 36/4 = 9/1 Now, x/16= 9/1, x=144 88. If the CI on a certain sum for 2 yrs at 10% per annum is Rs. 3150, what would be the SI on same rate for same time? Answer: Let principal = Rs x CI for 2years in % = 21% (using successive method) 21% of x = 3150 x = 15000 S.I = 15000*2*10/100 = Rs.3000 89. A started a business with initial investment of rs.12000, after 3 month B invest rs.15000 in this business. After 8 month from starting A withdrew one-fourth of his investment and B further invest 1/15 part of his investment. If at the end of one year the difference between the shares of profit of both is 700, what is the B’s profit share? Answer: (12000*8 + 9000*4) : (15000*5 + 16000*4) 132 : 139 Difference in profit sharing ration is= 139x – 132x = 7x Given 7x = 700, So x = 100 B’s profit share = 139x = 13900 90. There are three taps A, B, and C. A takes thrice as much time as B and C together to fill the tank. B takes twice as much time as A and C to fill the tank. In how much time can the Tap C fill the tank individually, if they would require 10 hours to fill the tank, when opened simultaneously? Answer: Let A, B, C fills a, b, c units per hour. Total units = 10*(a+b+C) Now, 3a = b+c and 2b = a+c Solving both, b = 4c/5 and a = 3c/5 total units of work = 10c*(4/5 + 3/5 + 1) = 24c done by C in 24c/c = 24 hours 91. Mano prepares a budget to visit London. However, he spends 12% of his budget on the first 10% days of his travel when he stays in the city. He knows that he has to spend another 35% of days in city itself, after which he would travel to the country side. What should be the minimum decrease in spending in country side as a percentage of his spending in city so as to complete his travel on the initial budget itself? Answer:

Page 14

GovernmentAdda.com

QUANTITATIVE APTITUDE – 250 WORD PROBLEMS

Like My Facebook Page

92. A merchant can buy goods at the rate of Rs.20 per good. The particular good is part of an overall collection and the value is linked to the number of items that are already on the market. So, the merchant sells the first good for Rs.2, second one for Rs.4, third for Rs.6… and so on. If he wants to make an overall profit of at least 40%, what is the minimum number of goods he should sell? Answer:

93. A train meets with a minor accident after travelling 100 km from starting point A and then proceeding at a reduced speed of three-fourth of original speed arrives at its destination B 90 minutes late. Had the accident occurred 60 kms further on, the train would have reached the destination 15 minutes earlier. The original speed of the train and distance AB are. Answer:

94. Pratap borrowed a sum of money from Arun at simple interest, at the rate of 12% per annum for the first three years, 16% per annum for the next five years and 20% per annum for the period beyond eight years. If at the end of 11 years, the total interest is Rs.6080 more than the sum, the sum borrowed was: Answer:

Page 15

GovernmentAdda.com

QUANTITATIVE APTITUDE – 250 WORD PROBLEMS

Like My Facebook Page

95. A man buys a scooter on making cash down payment of Rs.16224 and promises to pay two more yearly installments of equivalent amounts at the end of first year and second year. If the rate of interest is 4% per annum, compounded annually, the cash value of the scooter is: Answer:

96. A student scored 23% of maximum marks and failed by 23 marks. But if he scores 43% of the marks in the same exam, he passes by 17 marks. What is the maximum marks of the exam? Answer: Let, maximum marks = x (43-23)% of x = (23+17) 20% of x = 40 Solving we get, x= 200 97. If the price of sugar is increased by 20%, its expenditure gets decreased by 25%. What is the net effect on the total sale? Answer: Use successive method 20 + (-25) + (20)(-25)/100 = -10 98. A invested Rs.50000 for starting a venture and B joined his business with a capital of 65000 after 4 months. A get Rs.350 in every 2 monthS for his extra work. Find B’s profit if A receives a total of Rs.62100 as his share. Answer: (50000*12) : (65000*8) 15 : 13 Now, (62100 – 350*6) = 60000 B’s profit = 60000*13/15 = 52000 OR Let after cutting 350*6 from A’s profit, remaining amount is x. So 15/28 * x + 350*6 = 62100 So total x = 112000 So B’s profit = 13/28 * 112000 = 52000 99. Two trains having equal speed take 10 seconds and 15 seconds respectively to cross a 250 meter long bridge. If the length of second train is 150 meters more than the first train, then find the speed of the trains? Answer: Let, length of first train = x (x + 250)/10 = (x + 150 + 250)/15 Solving, we get x=50 Speed = 300/10 or 450/15 = 30m/s Convert this speed into km/h, 30*18/5 = 108km/h

Page 16

GovernmentAdda.com

QUANTITATIVE APTITUDE – 250 WORD PROBLEMS

Like My Facebook Page

100. Thirteen litres are drawn from a cask full of water and then it is filled with milk. Now thirteen litres of mixture are drawn and the cask is again filled with milk. The ratio of quantity of water now left in the cask to that of the milk in it is 16 : 9. How much does the cask hold? Answer: Water = 16 Total mixture = 16+9 = 25 √(16/25) = 4/5 Difference in ratio= 1 And this 1 is equal to 13. So total mixture is 13*5= 65 OR Let x litres is total capacity of cask Using formula, amount of water left in cask = x [1 – 13/x]2 [1 – 13/x]2/x = 16/(16+9) Solving we get, x = 65 l 101. Raman Publishers buys a machine for Rs.50000. The rate of depreciation is 10%. Find the depreciated value of the machine after 4 years. What is the average rate of depreciation? Answer:

102. A bank offers 10% interest rate compounded annually. A person deposits Rs.20,000 every year in his account. If he does not withdraw any amount, then how much balance will his account show after four years? Answer:

103. David invests Rs.7956 in the bank X and Y, so that X’s amount at the end of 5 years is equal to Y’s amount at the end of 7 years at 10 percent compounded annually. Find the amount invested by David in bank Y. Answer:

Page 17

GovernmentAdda.com

QUANTITATIVE APTITUDE – 250 WORD PROBLEMS

Like My Facebook Page

104. Messy borrows a certain sum from David at a certain rate of SI for 3 years. She lends this sum to Malar at the same rate of interest but compounded annually for the same period, that is 3 years. At the end of 3 years, she receives Rs.3300 as compound interest, but paid Rs.3000 as simple interest. What is the rate of interest? (Approximately) Answer:

105. The simple interest on a certain sum of money for 4 years at 10% per annum is half the compound interest on Rs.24000 for 2 years at 25% per annum. The sum placed on simple interest is Answer:

106. If compound interest on a sum for 4 years at 12% per annum is Rs.448062, then simple interest on the same sum at the same rate of interest and for the same period of time is? Answer:

Page 18

GovernmentAdda.com

QUANTITATIVE APTITUDE – 250 WORD PROBLEMS

Like My Facebook Page

107. What is the difference between compound interests on Rs.20,000 for 1.5 years at 10% per annum compounded yearly and half-yearly? Answer:

108. What annual payment will discharge a debt of Rs.7308 due in 3 years at the rate of 40% compound interest? Answer:

109. A man borrowed Rs.30000 at 15% per annum simple interest and immediately lent the whole sum at 15% per annum compound interest. What does he gain at the end of 3 years? Answer:

110. The principal amounts to Rs.27000 in 4 years at 20% per annum in compound interest. Then find the principle? Answer: T 4 According to the formula P = [A/(1 + r/100) ] = 27000/(1 + 20/100) = 27000 × 5/6 × 5/6 × 5/6 × 5/6 = 13020.83 Therefore, Principal = 13020.83 111. A box contains 5 Sony, 6 Samsung and 4 Sandisk pen drives. 3 pen drives are drawn at random. What is the probability that they are not of the same company? Answer: The total number of pen drives = 5 + 6 + 4 = 15 15 n(S)= C3 = (15x14x13) / (3×2) = 455 Now, 3 pen drives out of 15 pen drives can be drawn in 455 ways.

Page 19

GovernmentAdda.com

QUANTITATIVE APTITUDE – 250 WORD PROBLEMS

Like My Facebook Page

If all 3 pen drives are of the same company 5 6 4 It Can be done in C3 + C3 + C3 = 10 + 20 + 4 = 34 ways Probability that all 3 pen drives are not of the same company = 1 – (34/455) = (421/455) 112. The base of a triangular field is 660 metres and height 440 metres. If the charges for watering the field are at the rate of Rs.26.5 per sq hectometre, find the total cost to water the triangular field. Answer: Area of the field = (Base x Height) / 2 = (660 x 440)/2 sq metre = (660 x 440)/( 2x100x100) sq metre = 14.52 sq hectmetres Cost of watering 1 sq hectometre = Rs.26.5 Cost of watering the field = 26.5 x 14.52 = Rs.384.78 113. In a mixture of milk and water the proportion of milk by weight was 70%. If in a 250-gm mixture 100 gm water was added, what would be the percentage of water? Answer: Proportion of milk in the mixture =250 x (70/100) = 175gm Water = 75 gm After 100 gm water added in mixture the percentage of water = (75 +100) / (250+100) x 100 = (175/350) x 100 = 50% 114. Two pipes can fill a tank in 28 and 24 minutes respectively and a waste pipe can empty 5 gallons per minute. All three pipes working together can fill the tank in 16 minutes. How much time is taken by the waste pipe to empty the full tank? Answer: Let the capacity of the tank be 336 litres LMC of (28, 24 and 16 = 336) Waste pipe empties the tank in (12 + 14 - 21) 5 litres per minute Waste pipe empties the tank in (336/5) = 67.2 minutes 115. A box contains 6 black and 14 white balls, out of which 3 black and 5 white balls are defective. If we choose two balls at random, what is the probability that either both are white or both are non-defective? Answer: 14 20 12 20 9 20 Required probability- C2/ C2 + C2/ C22 – C2/ C2 = 121/190 116. In a class, the average age of some boys is 16 years, and average age of 16 teachers is 56 years. If the average age of the combined group of all the teachers and boys is 20, then the number of students is Answer: Use allegation method number of boys ‘x’ : number of teacher’16’ . 16 56 . 20 . (56-20)=36 (20-16)=4 . So 36/4 = 9/1 Now, x/16= 9/1, x=144 117. A started a business with initial investment of rs.12000, after 3 month B invest rs.15000 in this business. After 8 month from starting A withdrew one-fourth of his investment and B further invest 1/15 part of his investment. If at the end of one year the difference between the shares of profit of both is 700, what is the B’s profit share? Answer: (12000*8 + 9000*4) : (15000*5 + 16000*4) 132 : 139 Difference in profit sharing ration is= 139x – 132x = 7x Given 7x = 700, So x = 100 B’s profit share = 139x = 13900 118. There are three taps A, B, and C. A takes thrice as much time as B and C together to fill the tank. B takes twice as much time as A and C to fill the tank. In how much time can the Tap C fill the tank individually, if they would require 10 hours to fill the tank, when opened simultaneously? Answer: Let A, B, C fills a, b, c units per hour. Total units = 10*(a+b+C) Now, 3a = b+c and 2b = a+c Solving both, b = 4c/5 and a = 3c/5 total units of work = 10c*(4/5 + 3/5 + 1) = 24c done by C in 24c/c = 24 hours 119. The probability that a number selected at random from the first 52 natural numbers is a composite number is

Page 20

GovernmentAdda.com

QUANTITATIVE APTITUDE – 250 WORD PROBLEMS

Like My Facebook Page

Answer: 15 prime,36 composite and ‘1’is neither prime nor composite 120. A and B together can do a piece of work in 15 days, while B and C together can do in 24days. After A worked alone for 5 days and B alone for 11 days, C finished it in 21 days. In how many days can C alone finish the entire work? Answer: a+b=1/15 b+c=1/24 5a+11b+21c=1 5a+5b+6b+6c+15c=1 5(a+b)+6(b+c)+15c=1 C=1/36 121. Karan decided to donate x% OF HIS SALARY TO charity but on the day of donation he changed his mind and instead donated Rs 3500 which was 58% of what he decided earlier. If Karan’s salary is 38000 then find x. Answer: (x/100)*(58/100)*38000=3500 122. X speaks the truth in 40 percent of the cases and Y in 60 percent of the cases. Each of them is asked a series of questions, for which the answer can be only yes or no. What is the probability that they will contradict each other in answering a particular question? Answer: P(X)=40/100=2/5 P(Y)=60/100=3/5 P(X∩ȳ)+ P(x∩ y)=P(x)*P(ȳ)+P(y)*P(x) =2/5*2/5+3/5*3/5=13/25 123. I bought 3pen,4pencil and 7 eraser that costed me Rs 83,then I bought 2pen,1 pencil and 3 eraser nd that costed me Rs 17.If I have to buy 2 pen,2 pencil and 4 eraser how much do I need to pay? Answer: 3pen+4Pencil+7eraser=83 2pen+1pencil+3eraser=17 On adding 5pen+5pencil+10 eraser=100 Pen+pencil+2eraser=20 124. The sum of age A and B is 53.After 5 years the ratio of their age will be 2:1.What is the difference between their age? Answer: A+b=53 A+5/b+5=2:1 On solving a=37,b=16 125. If a five digit number is formed with digits 1,2,3,4 and 5.What is the probability that it will be divisible by 25,if repetition is not allowed? Answer: Last two digits must be 25 Therefore first three digits can be arranged in 3! Ways and total number =5! Hence=3!/5!=1/20 126. A boat travelling at a speed of 50 kmph started at 1 p.m. when there was no current from point A for point B which is X km apart. At 3:15p.m current started which fastened the entire journey by certain minutes.The speed of boat is 5 times to the speed of current and If the total time taken is 5hour,find X? Answer: Speed of boat=50 Speed of current=10 Speed of boat in current=60 Time taken=5hour 50*9/4+60*11/4=112.5+165=277.5 127. A,B and C started business with a total investment of 63000.A invested 6000 more than B and C invested 15000 less than B.If A’s profit at the end of year is 30000.What is total profit made by B and C? Answer: A+B+C=63000 A-b=6000 B-C=15000 On solving A=30000,B=24000,C=9000 128. Neeru and Siva invested Rs. 1600 and Rs. 1200 respectively. After 3 months, Neeru withdrew Rs. 500 while Siva invested Rs. 500 more. After 3 more months Shivani joins the business with a capital of Rs. 2100. The share of Siva exceeds that of Shivani, out of a total profit of Rs. 2640 after one year by

Page 21

GovernmentAdda.com

QUANTITATIVE APTITUDE – 250 WORD PROBLEMS

Like My Facebook Page

Answer: Neeru:Siva: Shivani= (1600*3 + 1100*9):(1200*3 + 1700*9):(2100*6 ) = 147:189:126 = 7:9:6 Difference of Siva and Shivani shares = Rs. [2640 * (9/22) — 2640 * (6/22)) = Rs. 360 129. Find the area of trapezium whose parallel sides are 12 cm and 17 cm long, and the distance between them is 13 Answer: Area=1/2*(12+17)*13 130. Three vessels containing sugar solutions the concentrations of sugar as 0.5, 0.25 and 0.75 respectively. Six litres from the first, fourlitres from the second and 12litres from the third are mixed. What is the ratio of water and sugar in the resultant mixture? Answer: According to the question, 6*0.5+4*0.75+12*0.25/6*0.5+4*0.25+12*0.75=9/13 131. A and B together can complete a task in 9 days. B and C together can complete the same task in 16 days. A and C together can complete the same task in 6 days. If A worked alone for 2 days, then B worked alone for 14 days, and then C worked alone for 4 days, what percentage of the task remains to be completed? Answer:

132. If 15:13 is the ratio of present age of Riya and Siva respectively and 17:11 is the ratio between Riya’s age 4 years hence and Siva’s age 4 years ago. Then what will be the ratio of Riya’s age 4 years ago and Siva’s age 4 years hence ? Answer: Let the present age of Riya and Siva be 15X and 13X respectively. Given, Riya’s age 4 years hence and Siva’s age 4 years ago in the ratio 17:11 That is, 15X + 4 / 13X – 4 = 17/11 11(15X + 4) = 17(13X – 4) 165X + 44 = 221X – 68 56X = 112 X=2 Therefore Riya=30 Siva=26 Ratio=30-4/26+4=13/15 133. A man rows 4 km upstream in 2 hours and 8 km downstream in 3 hours then how long(approx) will he take to cover 16 km in still water? Answer: Distance covered in downstream = 8 km Time taken in downstream = 3 hours. Rate of downstream = distance / time = 8km / 3 hours = 8/3 km/hr. Distance covered in upstream =4 km Time taken in upstream = 2 hours. Rate of upstream = distance / time = 4 km / 2 hours = 2 km/hr. Speed in still water = (upstream +downstream)/2 = (1/2)(8/3 + 2) = (1/2)(14/3)= 7/3 km/hr. Time Taken to cover 16 km in still water = distance / speed = 16 x 3/7 = 48 / 7 = 7 hours (approximately.

Page 22

GovernmentAdda.com

QUANTITATIVE APTITUDE – 250 WORD PROBLEMS

Like My Facebook Page

134. Harish bought a book for Rs.485 and sold it at 20% loss. By using that amount he bought another book and sold it at 15% profit. Then overall profit/loss amount is: Answer: 485*.8*1.15=485*.92=446.20 Therefore loss=485-446,20=38.8Rs 135. Two friends A and B simultaneously start running around a circular track . They run in the same direction. A travels at 8 m/s and B runs at b m/s. If they cross each other at exactly three points on the circular track and b is a natural number less than 20, how many values can b take? Answer: Let track length be equal to T. Time taken to meet for the first time = T/relative speed=T/8−b or T/b−8 Time taken for a lap for A = T/8 Time taken for a lap for B = T/b So, time taken to meet for the first time at the starting point = LCM (T/8,T/b)=T/HCF(8,b) Number of meeting points on the track = Time taken to meet at starting point/Time taken for first meeting = Relative speed / HCF (8,b. (8−b)/HCF(8,b) = 3 or (b−8)/HCF(8,b) = 3 b = 2, 5, 11,14 satisfy this equation. So, there are four different values that b can take. 136. A student scored 23% of maximum marks and failed by 23 marks. But if he scores 43% of the marks in the same exam, he passes by 17 marks. What is the maximum marks of the exam? Answer: Let, maximum marks = x (43-23)% of x = (23+17) 20% of x = 40 Solving we get, x= 200 137. If the price of sugar is increased by 20%, its expenditure gets decreased by 25%. What is the net effect on the total sale? Answer: Use successive method 20 + (-25) + (20)(-25)/100 = -10 138. A invested Rs.50000 for starting a venture and B joined his business with a capital of 65000 after 4 months. A get Rs.350 in every 2 monthS for his extra work. Find B’s profit if A receives a total of Rs.62100 as his share. Answer: (50000*12) : (65000*8) = 15 : 13 Now, (62100 – 350*6) = 60000 B’s profit = 60000*13/15 = 52000 139. Two trains having equal speed take 10 seconds and 15 seconds respectively to cross a 250 meter long bridge. If the length of second train is 150 meters more than the first train, then find the speed of the trains? Answer: Let, length of first train = x (x + 250)/10 = (x + 150 + 250)/15 Solving, we get x=50 Speed = 300/10 or 450/15 = 30m/s Convert this speed into km/h, 30*18/5 = 108km/h 140. A train 75 m long overtook a person who was walking at the rate of 6 km/hr, passed him in 7 1/2 seconds. Also it overtook a second person in 6 3/4 seconds. What was the speed of the second person? Answer: Let, speed of train in km/h= x (x-6) * 5/18 = 75 * 2/15, Solving, we get = 42 km/h Now, assume speed of second person is y km/hr, So, (42-y) * 5/18 = 75 * 4/27 Solving, we get y = 2km/h 141. Thirteen litres are drawn from a cask full of water and then it is filled with milk. Now thirteen litres of mixture are drawn and the cask is again filled with milk. The ratio of quantity of water now left in the cask to that of the milk in it is 16 : 9. How much does the cask hold? Answer: Let x litres is total capacity of cask Using formula, amount of water left in cask = x [1 – 13/x]2 [1 – 13/x]2/x = 16/(16+9) Solving we get, x = 65 l

Page 23

GovernmentAdda.com

QUANTITATIVE APTITUDE – 250 WORD PROBLEMS

Like My Facebook Page

142. A reduction of 40% in the price of wheat would enable a purchaser to obtain 36 kg more for Rs. 45. What is the reduced price per kg? Answer: Assume, purchaser buy 100 kg in Rs.45 Now the new price is 60/100 * 45 = 27, It means in Rs. (45-27) = 18, 36kg more wheat is purchased. Rs 18= 1800p Now, 1800/36= 50 p 143. A dishonest rice seller sells rice at 15% profit of rice CP, and he also uses 800gm weight in place of 1kg.Find his total profit percent. Answer: Initial profit on CP = 15% Again profit , (1000-800)/800 *100 = 25% Use successive method, 15 + 25 + (15)(25)/100= 43.75% or 43 ¾ % 144. Sides of the parallelogram are in the ratio of 4:3, and its area is 1500 sq. units. Altitude on the greater side is 15 units. Find out the Altitude on the smaller side is? Answer: Let the side of parallelogram be = 4x and 3x Area of parallelogram = basic * height Given area= 1500 units, so, 4x*15= 1500 X = 25 units Sides = 4*25 and 3* 25= 100 and 75 units, Now, height = 1500/75= 20 units 145. A bag contains 6 red balls, 11 yellow balls and 5 pink balls. If two balls are drawn at random from the bag, one after another, what is the probability that the first ball is red and the second ball is yellow? Answer: Total of balls = 6 + 11 + 5 = 22 22 n(S) = C2 = (21x22) / 2 = 231 6 11 Now, n(E) = C1 x C1 = 6 x 11 = 66 P(E) = n(E)/n(S) = 66/231 = 6/21 = 2/7 146. The sum of the radius and the height of a cylinder is 19m. The total surface area bf the cylinder is 1672 m2, what is the volume of the 3 cylinder? (in m ) Answer: Let the radius of the cylinder be r and height be h. Then, r + h = 19 …..(i) 2

Again, total surface area of cylinder = (2πrh + 2πr ) Now, 2πr(h + r) = 1672 or, 2πr x 19 = 1672 or, 38πr = 1672 , πr = (1672/38) = 44m, r = (44 × 7) / 22 = 14 Height = 19 - 14 = 5m Volume of cylinder = πr h = (22/7) x 14 x 14 x 5 =14m = 22 × 2 × 14 × 5 = 3080m 2

3

147. The ratio of the speed of the boat upstream to the speed of the boat downstream is 2 : 3. What is the speed of the boat in still water if it covers 42 km downstream in 2 hours 20 minutes? (in km/h) Answer: Let the speed of the boat in still water be x and that of the current be y. Then, downstream speed = x + y and upstream speed = x - y Now, downstream speed = 42 / [2 20/60] = (42 × 3) / 7 =18 km x+y=18 Again, 3 : 18, 2 : 12 (As ratio of downstream to upstream is 2 : 3) x - y = 12 Solving (i) and (ii), we get (x+y=18) + (x - y =12) = 2x =30 x = 15 kmph Hence speed of the boat 15 kmph

Page 24

GovernmentAdda.com

QUANTITATIVE APTITUDE – 250 WORD PROBLEMS

Like My Facebook Page

148. 35 men complete a piece of work in 16 days and 20 women complete the same piece of work in 30 days. What is the ratio of the amount of work done by 40 men in 1 day to the amount of work done by 15 women in 1 day? Answer: 35 men complete the work in 16 days. 1 man completes the work in 16 x 35 days, 32 men complete the work in (16x35)/40 = 14 days. Again, 20 women complete the same piece of work in 30 days. 1 woman completes the same piece of work in 20 × 30 days. 15 women can complete the work in (20x30)/15 = 40 days. Ratio = 1/14 : 1/40 = 40 : 14 = 20 : 7 149. A man sold an article for Rs. 6800 and incurred a loss. Had he sold the article for Rs.7850, his gain would have been equal to half of the amount of loss that he incurred. At what price should he sell the article to have 20% profit? Answer: Let the cost price be x. Then, loss = (x - 6800) Again, profit = (7850 - x) Now, (7850 - x) = (x - 6800)/2 or, 15700 - 2x = x - 6800 or, 3x = 15700 + 6800 = 22500 x = 22500/3 = 7500 Selling price = (7500x120)/100 = Rs. 9000 150. A bought a certain quantity of bananas at a total cost of Rs. 1500. He sold 1/3 of these bananas at 25% loss. If he earns an overall profit of 10%, at what percentage profit did A sell the rest of the bananas? Answer: Total CP = 1500 Total SP = 1500 + 10% of 1500 = 1500 + 150 = 1650 CP of 1/3 of bananas = 1500/3 = Rs.500 SP of 1/3 of bananas at 25% loss = 500 – [ (500 x25 / 100)] = 500 - 125 = 375 SP of the rest of bananas = 1650 - 375 = 1275 Now, CP of the test of bananas = 1500 - 500 = 1000 Profit on the rest of bananas = 1275 -1000 = 275 % of profit on the rest of bananas = (275/1000)×100 = 27.5% 151. A tank has two inlets: P and Q. P alone takes 6 hours and Q alone takes 8 hours to fill the empty tank completely when there is no leakage. A leakage was caused which would empty the full tank completely in ‘X’ hours when no inlet is open. Now, when only inlet P was opened, it took 15 hours to fill the empty tank completely. How much time will Q alone take to fill the empty tank completely? (in hours) Answer: (1/P) – (1/X) = (1/15) Or, (1/6) – (1/X) = (1/15) (P = 6 hours) Or, (1/X) = (1/6) – (1/15) = (10-4)/60 = 1/10 x = 10 hours Now, (1/Q) – (1/10) = (1/8) – (1/10) = (5-4)/40 = 1/40 Hence, Q fills the tank in 40 hours. 152. At present, the ratio of the ages of A to B is 3 : 8; and that of A to C is 1 : 4. Three years ago, the sum of the ages of A, B and C was 83 years. What is the present age (in years) of C? Answer: According to the question, A : B = 3 : 8 A:C=1:4 B:A=8:3 A:C=1:4 8 : 3 : 12 Sum = 8x + 3x 12x = 23x Now, 23x = 92 x=4 Hence the present age of C = 12x = 12 x 4 = 48 years

Page 25

GovernmentAdda.com

QUANTITATIVE APTITUDE – 250 WORD PROBLEMS

Like My Facebook Page

153. The sum invested in Scheme B is thrice the sum invested in Scheme A. The investment in Scheme A is made for 4 years at 8% p.a. simple interest and in Scheme B for 2 years at 13% p.a. simple interest. The total interest earned from both the schemes is Rs.1320. How much amount was invested? Answer: Let the amount invested in scheme A be Rs.x and that in B be Rs. 3x. Then, [(x × 4 × 8)/100] [(3x × 2 × 13) /100] = 1320 Or, (32x/100) + (78x/100) = 1320 110x/100 = 1320 x = (1320 x 100) / 110 = Rs. 1200 154. Kim and Om are travelling from point A to B, Which are 400 km apart. Travelling at a certain speed Kim takes one hour more than Om to reach point B. If Kim doubles her speed she will take 1 hour 30 mins less than Om to reach point B. At what speed was Kim Driving from point A to B? (in kmph) Answer: Let the speed of Kim be x and that of Om be y. Then, (400/x) – (400/y) = 1 Let 1/x = u and 1/y = v 400u — 400v = 1 …(i) Again, (400/y) – (400/2y) = 3/2 400v – 200u = (3/2) Or, 800v – 400u = 3 …(ii) Solving (i) and (ii), we get (400u - 400v =1) + (-400u + 800v) = 400v = 4 v = (4/400) = (1/100) km y = 100 km now, (400/x) – (400/100) = 1 or, (400/x) = 5 => x = 80 kmph 155. The ratio of a two-digit natural number to a number formed by reversing its digits is 13 : 31. Which of the following is the sum of all the numbers of all such pairs? Answer: Number=10a+b Reverse=a+10b 10a+b/a+10b=13/31 a/b=1/3 Therefore numbers=13,26,39 156. 1 unit of x% alcohol is mixed with 4 units of y% alcohol to give 50% alcohol. If x > y, how many integer values can x take? Answer: x>y => x > 50 > y. (x – 50) = 4(50 – y. 50 – y is an integer => x – 50 has to be a multiple of 4 x can take values {54, 58, 62 ….. 98}– x can take total of 12 values. 157. 42 men can complete a piece of work in 15 days and 52 women can complete the same work in 21 days. What is the ratio of the amount of work done by 7 men to that done by 13 women, in 1 day? Answer: 42 men one day work=1/15 7 men one day work =7/(15*42)=1/90 52 women one day work=1/21 13 women one day work=13/(21*52)=1/84 Ratio=84/90=14/15 158. The present average age of a family of six members is 28 years. If the present age of the youngest member in the family is EIGHT years, then what was the average age of the family at the time of the birth of the youngest member? Answer: Sum of present age of all 6=6*28=168 sum of Present age of rest 5=168-8=160 Sum of present age of rest 5,8 years ago=160-40=120 Hence average =120/6=20

Page 26

GovernmentAdda.com

QUANTITATIVE APTITUDE – 250 WORD PROBLEMS

Like My Facebook Page

159. Sohan and Mohan enters into a partnership with their capitals in the ratio 7: 4. At the end of 7months, Mohan withdraws his capital. If they receive their shares profits in the ratio 7 : 8, find out how long Sohan’s capital was invested in the business? Answer: 7*x/4*7=7/8 X=3.5 160. A milkman mixes 20 lites of water with 80 litres of milk. After selling one-fourth of this mixture, he adds water to replenish the quantity that he has sold. What is the current proportion of water to milk ? Answer: 1/4th of the mixture is sold 1/4th of milk and 1/4th of water is sold. = 3/4th of milk = (3/4)x80 = 60 litres of milk is remaining and rest part 100 - 60 = 40 litres is water (as water is a added in place of milk) Reqd ratio = 40 : 60 2 : 3 161. A shopkeeper buys an article at a discount of 20% on the listed price from a wholesaler. The shopkeeper marks up the price by 15% on the listed price. A buyer pays Rs.3795 to get it after paying sales tax at the rate of 10% on the price asked for. Find the profit percentage of the shopkeeper. Answer: Let the listed price = Rs. 100 CP of shopkeeper = 100 – 20 = Rs. 80 Marked price by shop keeper = 100 + 15 = Rs. 115 Now, 115 = 3795 x (100/110) = 3450 80 = (3450/115) × 80 = Rs. 2400 CP of shopkeeper = Rs. 2400 Profit = 3450 - 3400 = 1050 Profit % = (1050 / 2400) x100 = 43.75% 162. A sum amounts to Rs.10580 in 2 years and to Rs.12167 in 3 years compounded annually. Find the sum and the rate of interest per annum. Answer: 1 12167 = 10580 [1+ (r/100)] Or, (12167/10580) = 1+(r/100) Or, (1587/10580) = r/100 r = (1587×100) / 10580 = 15% sum = (10580x100x100) / (115 × 115) = Rs. 8000 Sum = Rs. 8000, Rate = 15% 163. Mohit travels 972 km in 10.5 hours in two stages. In the first part of the journey, he travels by bus at the speed of 78 km, per hour. In the second part of the journey, he travels by train at the speed of112 km/hr. How much distance does the travel by train? Answer: We use only alligation on speed (km/hr) to get ratio of time spent in bus and train. overall speed = (972/10.5) = (1944/21) = 648/7

136/7 : 102/7 à 136 : 102 Or 4 : 3 Time spent in train = 10.5 (3/7) = 4.5 hours Distance travelled by train = 112 x 4.5 hours = 504 km 164. A contractor undertook to do a certain work in 75 days and employed 48 men to do it. After 55 days he found that only (2/3) of the work was done. How many more men must be employed so that the work may finished in time? Answer: Apply M1D2 / W2 = M2D2 / W2 Or, (48x55) / (2/3) = (Mx20) / (1/3) M = 66 men Reqd more men = 66 - 48 = 18 men 165. 49 pumps can empty a reservoir in 17/2 days working 6 hours a day. If 119 pumps are used for 7 hours a day then in how many days will the same work be completed? Answer: Let the required number of days be x. 49 x (17/2) x 6 = 119 x 7 x x

Page 27

GovernmentAdda.com

QUANTITATIVE APTITUDE – 250 WORD PROBLEMS

Like My Facebook Page

x = 3 days 166. 6 kg of an alloy A is mixed with 8 kg of alloy B. If alloy A has lead and tin the ratio 1 : 3 and B has tin and copper in the ratio 2 : 3, then what is the amount of tin in the new alloy? Answer: Quantity of tin in alloy A = 6 × (3/4) = 4:5 kg Quantity of tin in alloy B = 8 × (2/5) = 3.2kg Quantity of tin in the new alloy = 4 . 5 + 3.2 =7.7 kg 167. There are 5 boys and 4 girls. In how many ways can they be seated in a row so that all the girls do not sit together? Answer: Total number of persons 5 +.4 = 9 When there is no restriction they can be seated in a row in 9!.Ways. But if all the 4 girls sit together, we can consider the group of 4 girls as one person. Therefore, we have only 5 + 1 = 6 persons Number of ways = 6! Ways 4 But 4 girls can be arranged among themselves in P4 = 4! Ways Reqd no.of ways in which all the 4 girls do not sit together = 9! – 6! × 4! = 9 × 8 × 7 × 6! – 6! × 24 = 6! (504 - 24) = 720 × 480 = 720 × 480 = 345600 168. The difference between 20% of a number and 4/5th of same number is 2499. What is 2/7th of that number? Answer: Let the number be N. Then, 4/5 N-20/100 N=2499 4/5 N-N/5=2499 ⇒ N(3/5)=2499 ⇒ N=(2499 × 5)/3 = 833 × 5 = 4165 Again, 2/7 of N=2/7×4165=2×595=1190 169. Prithvi spent Rs. 89745 on his college fees, Rs. 51291 on personality development classes and the remaining 27% of the total amount he had as cash with him. What was the total amount? Answer: Here, money spent on college fees = Rs. 89745 Money spent on personality development classes = Rs. 51291 Total amount = 89745 + 51291 = Rs. 141036 Now, remaining amount = (100 - 27)% = 73% So, 73% = 141036 ⇒ 1%=141036/73 ⇒ 100%=141036/73×100 = Rs. 193200 170. Vaishali spent Rs. 31897 on the air conditioner for the home, Rs. 38789 on buying plasma television and the remaining 23% of the total amount she had as cash with her. What was the total amount? Answer: Here, money spent on buying air conditioner = Rs. 31897 Money spent on buying plasma television = Rs. 38789 ∴ Total money spent = 31897 + 38789 = Rs. 70686 Now, she has left with 23% of total cash Hence, 77% = 70686 ⇒ 1% =70686/77 ⇒ 100%=70686/77×100 = 918 × 100 = Rs. 91800 171. Beena spend Rs. 44668 on her air tickets, Rs. 56732 on buying gifts for the family members and the remaining 22% of the total amount she had as cash with her. What was the total amount? Answer: Money spent on air tickets = Rs. 44668 Money spent on buying gifts = Rs. 56732 Total amount = Rs. 101400 This is equal to = (100 - 22)% = 78% of total money So, 78% = 101400 ⇒ 1% =101400/78 ∴ 100%=101400/78×100 = Rs. 130000 172. A sum of Rs. 731 is divided among A, B and C, such that A receive 25% more than B and B receives 25% less than C. What is C’s share in the amount?

Page 28

GovernmentAdda.com

QUANTITATIVE APTITUDE – 250 WORD PROBLEMS

Like My Facebook Page

173. Mr Giridhar spends 50% of his monthly income on household items and out of the remaining, he spends 50% on transport, 25% on entertainment, 10% on sports and remaining amount of Rs. 900 is saved. What is Mr Giridhar’s monthly income? Answer: Let Giridhar’s monthly income be Rs. 100 Then, money spent on household’s item =100×50/100 = Rs. 50 Remaining amount = 100 - 50 = Rs. 50 Money spent on transport =50×50/100 = Rs. 25 Money spent on entertainment =50×25/100 = Rs. 12.5 Money spent on sports =50×10/100 = Rs. 5 ∴ Last remaining amount = 100 - (50 + 25 + 12.5 + 5) 100 - (92.5) = Rs. 7.5 ∴ Rs. 7.5 is saved, when total income is Rs. 100 ∴ Rs. 1 is saved, when total income =100/7.5 Now, Rs. 900 is saved, when total income = 100/7.5×900 = 100 × 120 = Rs. 12000 174. Mr X spends 20% of his monthly income on household expenditure. Out of the remaining 25% he spends on children’s education, 15% on transport, 15% on medicine and 10% on entertainment. He is left with Rs. 9800 after incurring all these expenditures. What is his monthly income? Answer: Let Mr X monthly income be Rs. 100 Then, money spent on household expenditure =100×20/100 = Rs. 20 ∴ Remaining amount = 100 - 20 = Rs. 80 Money spent on children’s education =80×25/100 = Rs. 20 Money spent on transport =80×15/100 = Rs. 12 Money spent on medicine =80×15/100 = Rs. 12 Money spent on entertainment =80×10/100 = Rs. 8 ∴ Last remaining amount = 100 - (20 + 20 + 12 + 12 + 8) = 100 - 72 = Rs. 28 Now, Rs. 28 is left, when total income is Rs. 100 Rs. 1 is left, when total income =100/28 ∴ Rs. 9800 is left, when total income =100/28×9800 = Rs. 35000 175. In a class of 35 students and 6 teachers, each student got sweets that are 20% of the total number of students and each teacher got sweets that are 40% of the total number of students. How many sweets were there? Answer: Here, sweets that are got by each student =20/100×35=7 ∴ Total number of sweets distributed students = 35 × 7 = 255 Now, sweets that are got by each teacher =40/100×35=14 ∴ Total number of sweets distributed to teachers = 6 × 14 = 84 So, total number of sweets = 255 + 84 = 339 176. In a class of 80 students and 5 teachers, each student got sweets that are 15% of the total number of students and each teacher got sweets that are 25% of the total number of students. How many sweets were there? Answer: Here, number of sweets got by each student = 80×15/100=12 So, total number of sweets got by all students = 12 × 80 = 960 Number of sweets got by each teacher =80×25/100=20 So, total number of sweets got by all teachers = 20 × 5 = 100 ∴ Total number of sweets which are distributed to teachers and students = 960 + 100 = 1060 177. 405 sweets were distributed equally among children in such a way that the number of sweets received by each child is 20% of the total number of children. How many sweets did each child receive?

Page 29

GovernmentAdda.com

QUANTITATIVE APTITUDE – 250 WORD PROBLEMS

Like My Facebook Page

Answer: Let total number of children be x Then, each child gets (x×20/100) sweets Now, x/5×x=405 ⇒ x^2=405×5 ⇒ x^2=81×25 ⇒ x = 9 × 5 = 45 sweets Per child = 405/45 = 9 178. A candidate appearing for an examination has to secure 35% marks to pass. But he secured only 40 marks and failed by 30 marks. What would be the maximum marks to test? Answer: Here, passing marks of nay candidate = 40 + 30 = 70 Let the total marks be x Then, x=35/100 = 70 x = 200 179. In an election between two candidates, one got 52% of total valid votes. 25% of the total votes were invalid. The total number of votes were 8400. How many valid votes did the other person get? Answer: Answer: Here, total number of votes = 8400 Invalid votes =8400×25/100 = 2100 Valid votes = 8400 - 2100 = 6300 Votes got by one candidate = 6300×52/100=3276 Number of votes got by other candidate = 6300 - 3276 = 3024 180. The ratio of students in school A, B and C is 5 : 4 : 7 respectively. If number of students in schools are increased by 20%, 25% and 20% respectively, then what will be the ratio of students in school A, B and C respectively?

181. Population of a country increases every year by 10%. If the population in January 2006 was 15.8 lakh, what will be the population in January 2008?

182. The price of rice decreases by 6.25% and because of this reduction, Vandana is able to buy 1 kg more for Rs. 120. Find the reduced rate of rice.

183. Jitendra's age is three times the sum of the ages of his two sons. Two years ago, his age was six years less than four times the sum of the ages of his sons. What is the present age of Jitendra? Answer: Let the sum of Jitendra's sons be x years Then, Jitendra's age = 3x Again, 4(x - 2) – 6 = 3x - 2 or, 4x – 8 - 6 =3x – 2 or, x = 12 years

Page 30

GovernmentAdda.com

QUANTITATIVE APTITUDE – 250 WORD PROBLEMS

Like My Facebook Page

Present age of Jitendra = 3 x 12 = 36 years 184. An amount of Rs. 6996 is divided among Raju, Babu and Shyam in such a way that if their shares be reduced by Rs. 8, Rs. I2 and Rs. 16 respectively, the remainders shall be in the ratio of 7 : 8 : 9. Find the share of Babu. Answer: The amount which is divided among them = 6996 - (8 + 12+ 16) = 6996 - 36 = 6960 Now, Babu's share = 6960 x (8/24) +12 = 2320 + 12 = Rs. 2332 185. The weights of two persons Rahul and Rupesh are in the ratio of 4 : 5. Rupesh's weight increased by 20% and the total weight of Rahul and Rupesh together became 135 kg with an increase of 25%. By what per cent did the weight of Rahul increase? Answer: Rahul's + Rupesh's weight = 135 (100/125) Rahul's weight = 135(4/5)(4/9) = 48 kg Rupesh's weight = 48 (5/4) = 60 kg Now, after increase Rupesh's weight = 60x (120/100) = 72 kg After increase Rahul's weight = 135 - 72 = 63 Reqd % increase =[(63-48) / 48] x 100 = (15/48) x 100 = 31.25% 186. A boat takes 3 hours to travel from place A to place B downstream and back from B to A upstream. If the speed of the boat in still water is 4 kmph what is the distance between the two places? Answer: Let the distance from place A to B be x km and the speed of current be y km/hr. Now, [x/(4+y)] + [x/(4-y)] = 3 Or, (4x – xy + xy + 4x) / [(4 - y) (4 + y)] = 3 Or, 3 (16 – y2) = 8x Or, 48 – y2 = 8x So, we can’t find the distances 187. A train travelling at 57 km/hr passes another train half of its length travelling in the opposite direction at 33 km/hr in 18 seconds. If it passes a railway platform in 1.2 minutes, what is the length of the platform? Answer: Distance travelled with relative speed 57 + 33 = 90 km/hr in 18 seconds = 90 (5/18) × 18 = 450m Ratio of lengths = First : Second train = 2 : 1 Length of first train = 300m Now, distance travelled by 1st train at 57 km/hr in 72 seconds = 57 (5/18)×72 = 1140m Length of platform = 1140 – 300 = 840m 188. In a school there are 30 more boys than girls. If the number of boys is increased by 10% and the number of girls is also increased by 45%, there would be nine more girls than boys. What is the number of students in the school? Answer: Let there be ‘a’ boys and ‘b’ girls a-b=30 1.45b-1.1a=9 on solving a=150,b=120 189. The simple interest accured on Rs 36500 at the end of five years is Rs. 21900.What would be compound interest accured on the same amount for same time period(Approx) Answer: 36500*R*5/100=21900 R=12% Now 5 CI=36500*((1+12/100) -1)=36500*(1.76-1)=Rs 27740 190. Two pipes A and B can fill a cistern in 40 minute and 50 minutes respectively. If both the pipes are opened together, then after how much time should B be closed so that the cistern is full in 30 minutes? Answer: Let pipe B be close after x minutes X(1/40+1/50)+30-X(1/40)=1 9X/200-5X/200=1-30/40 4X/200=10/40 X=12.5 191. The approximate compound interest accured on Rs 27000 at the end of three years is Rs. 7012.What would be simple interest accured on the same amount for same time period(Approx)

Page 31

GovernmentAdda.com

QUANTITATIVE APTITUDE – 250 WORD PROBLEMS

Like My Facebook Page

Answer: CI=27000*((1+R/100)3-1) 3 7012/27000=((1+R/100) -1) 3 (1+R/100) =1.26 R=8 S.I=27000*8*3/100=6480 192. The average weight of boys in a class of students is 58 kg, while that of girls is 50 kg. The average weight of the entire class is 53 kg. The number of girls is approximately what per cent of the number of boys in the class? Answer: Let the number of boys in the class be x and that of girls be y. Then, (x × 58 + 50 × y) / (x + y) = 53 or, 58x + 50y = 53x + 53y or, 5x = 3y x/y = 3/5 Reqd % = (5/3) × 100 = (500/3) = 166 2/3% = 167% 193. A bag contains 5 red balls, 6 blue balls, 2 green balls and 7 white balls. If 2 balls are picked up at random, what is the probability that both the balls are white in colour? Answer: Total number of balls = 5 + 6 + 2 + 7 =20. 20 n(S) = C2= (19x20)/2 = 190 Probability that both balls are white 7 n(E) = C2= (7x6) / (1x2) = 21 P(E) = n(E) / n(s) = 21/190 194. A can complete a given task in 24 days, while B is twice as efficient as he. A started on the work initially, and was joined by B after a few days. If the whole work was completed in10 days, after how many days, from the time A started working, did B join A? Answer: A can complete the work in 24 days Efficiency of B is twice that of A. B can complete the work in 24×(1/2) = 12 days According to the question, the work is completed in 10 days. LCM of 24 and 12 = 24 units. Let the total work be 24 units. A can do in one day (24/24) = 1 unit And B can do in one day = 24/12 = 2 units Now, A works for 10 days. Total work done by A in 10 days = 10 x 1 = 10 units Remaining work = 24 -10 = 14 units Now, 14 units of work is done by B in (14/2) = 7 days Hence B joined the work after (10-7 =) 3 days 195. The angles of a quadrilateral are in the ratio of 9 : 8 : 12 : 7. The second largest angle of the quadrilateral is the part of a triangle, the base and hypotenuse of which are 15 cm and 17 cm respectively. What is the height of the triangle? Answer: 0 Sum of angles of a quadrilateral = 360 Let the angles be 9x, 8x, 12x and 7x. 0 Then, 9x + 8x + 12x + 7x = 360 or, 36x = 360 à x = 100 0 Thus, second largest angle = 9 x 10 = 90 Thus, the triangle is a right-angled triangle. Now, ABC makes a right-angled triangle.

2

2

2

2

Height of the triangle (AB) = √(AC) - (BC) = √(17) - (15) = √(289 – 225) = √64 = 8 cm

Page 32

GovernmentAdda.com

QUANTITATIVE APTITUDE – 250 WORD PROBLEMS

Like My Facebook Page

196. When the price of rice was increased by 17% a family reduced its consumption in such a way that the expenditure on rice was 8% more than before. If 13 kg was consumed per month earlier, find the new monthly consumption. Answer: Reqd monthly consumption = (108/117) × 13 = 12 kg 197. There are 52 students in a hostel. 312 toffees are distributed among them so that each boy gets 9 toffees and each girls gets 5 toffees. Find the number of boys and girls in that hostel. Answer: Mean value of toffee per student = 312/52 = 6 toffees

Boys : girls = 1 : 3 Number of boys = [52 / (1+ 3)] ×1 = 13 And number of girls = 52 -13 = 39 198. The batting average of 40 innings of a cricket player is 70 runs. His highest score exceeds his lowest score by 170 runs. If these two innings are excluded, the average of the remaining 38 innings is 68 runs. What is his highest score? Answer: Let the highest score be x. And the lowest score be y. Then, x + y = 40 x 70 - 38 x 68 = 2800 - 2584 - 216 x+y=216 Again, x - y = 170 (ii) Adding (i) and (ii), we get (x+y=216) +( x - y=170) = 2x = 386 x =386/2 = 193 y = 216 - 193 = 23 Therefore the highest score = 193 199. A water tank is 20m long, 12m wide and 30m deep. It is made up of iron sheet which is 2m wide. The tank is open at top. If the cost of the iron sheet is Rs.18 per metre, then what is the total cost of the iron sheet required to build the tank? Answer: Surface area of the open tank = 2(l x w + w x d + l x d) - (1 x w) = 2[20 x 12 + 12 x 30 + 20 x 30] - 20 x 12 = 2[240 + 360 + 600] - 240 2 = 2400 -240 = 2160m Length of iron sheet = 2160/2 = 1080m Total cost of iron sheet = 1080 × 18 = Rs.19440 200. Pipe A can fill a cistern in 24 minutes and B in 36 minutes. If both the pipes are open together, after how long should pipe B be closed so that the cistern becomes full in 18 minutes? Answer: Let the capacity of the tank = LCM of 24 and 36 = 72 Now, Pipe A can fill the tank (72/24=) 3 units in a minute Pipe B can fill the tank (72/36 =) 2 units in a minute Now, A fills the tank in 18 minutes = (18 × 3) = 54 units Remaining units = 72 - 54 = 18 units So, 18 units will be filled by B in (18/2 =) 9 minutes 201. A work which can be completed by 18 men in 26 days can also be done by 20 women in 33 days. 13 men start doing the work and complete one-third of the work. If they are now replaced by 22 women, in how many days the total work will be completed? Answer: 18 m in 26 days, so 13 men do in 18*26/13 = 36 days. They complete 1/3rd of work. So number of days required by 13 men to complete that work is 36* 1/3= 12 days. Now: 20 w in 33 days, so 22 w do in 20*33/22 = 30 days. They complete 2/3rd of work. So number of days required by 22 women to complete that work is 30* 2/3 = 20 days. So total 12 +20 = 32 days

Page 33

GovernmentAdda.com

QUANTITATIVE APTITUDE – 250 WORD PROBLEMS

Like My Facebook Page

202. A profit of Rs 1200 is made by selling an article if one-third of it is sold at 9% profit and the remaining at 3% loss. What is the cost price of the article? Answer: It can be calculated as: CP = 1200 *100 / [1/3 *9 + 2/3 * (-3)] = 1,20,000 * 1/3rd sold at 9% profit, 2/3rd sold at 3% loss 203. There are 3 blue balls, 4 red, and 5 green balls. 3 balls are drawn at random. What is the probability of all blue or all green balls? Answer: 3 12 Probability of all blue = C3 / C3 = 1/220 5 12 Probability of all green = C3 / C3 = 10/220 So probability of all blue or all green = 1/220 + 10/220 = 11/220 204. On a certain sum of money, compound interest obtained is Rs 3,520 after 2 years at 20% per annum. What will be the simple interest obtained at the same rate and for the same time. Answer: If P is the principal, SI for 2 years = P*20*2/100 = 2P/5 So, SI for 1 year = P/5 CI for 2 years = P/5 + (P/5 + 20/100 * P/5) = 2P/5 + P/25 = 11P/25 Now, 11P/25 = 3520, so P = 8,000 So SI = 2*8000/5 = 3200 205. A person whose monthly salary is Rs 10,000 has expenditure of Rs 6,000. In the next month, his salary increases by 10% and so he increases his expenditure by 20%. What is the percentage change in his savings made? Answer: Savings = 10,000 – 6000 = 4000 Income becomes = (110/100)*10,000 = 11,000 Exp. becomes = (120/100)*6,000 = 7,200 So savings now = 11000 – 7200 = 3800 So % decrease in savings = (4000-3800)/4000 * 100 = 5% 206. A circle whose area is 3850 sq. cm has circumference double the perimeter of a rectangle of breadth 30 cm. Find the area (in sq cm) of rectangle. Answer: ᴨr^2 = 3850, so r = 35 Now perimeter of rect. = (1/2)*2ᴨr = 110 So 2(l+30) = 110, so l = 25 So area = 25*30 207. There are 2 mixtures of milk and water such that mixture A contains 25% water and mixture B contains 10% water. Equal quantities of both mixtures are taken and put in a bottle. Find the final ratio of milk to water Answer: Let x litres taken from both, so milk : water = 75% of x + 90% of x : 25% of x + 10% of x 208. Ratio of ages of A and B is 2 : 3 and that of A and C is 4 : 9. If the difference in the ages of B and C is 15 years, find the age of C. Answer: B/A = 3/2 and A/C = 4/9 So B : A : C = 3*4 : 2*4 : 2*9 = 6 : 4 : 9 A = 6x, B = 4x, C = 9x So 9x – 6x = 15, this gives x = 5 So age of C = 9x = 45 209. An article which was sold for Rs 540 was marked at Rs 750. If two successive discounts were given with first being 20%, find the second discount given? Answer: Total discount% given = (750-540)/750 * 100 = 72% So by successive formula -20 – x + (20*x)/100 = -72 210. Thirty men can complete a work in 16 days. They started work and after 6 days, ten more men joined. Find the number of days in which the remaining work will get completed? Answer: 30 m in 16 days, so 40 men in (30*16)/40 = 12 days So

Page 34

GovernmentAdda.com

QUANTITATIVE APTITUDE – 250 WORD PROBLEMS

Like My Facebook Page

(1/16)*6 + (1/12)*x = 1 Solve, x = 7.5 days 211. Rajeev's age after 15 years will be 5 times his age 5 years back. What is the present age of Rajeev ? Answer: Let Rajeev's present age be x years. Then, Rajeev's age after 15 years = (x + 15) years. Rajeev's age 5 years back = (x - 5) years. Therefore x + 15 = 5 (x - 5) x + 15 = 5x - 25 4x = 40 x = 10. Hence, Rajeev's present age = 10 years. 212. The ages of two persons differ by 16 years. If 6 years ago, the elder one be 3 times as old as the younger one, find their present ages. Answer: Let the age of the younger person be x years. Then, age of the elder person = (x + 16) years. Therefore 3 (x - 6) = (x + 16 - 6) 3x -18 = x + 10 2x = 28 x = 14. Hence, their present ages are 14 years and 30 years. 213. The product of the ages of Ankit and Nikita is 240. If twice the age of Nikita is more than Ankit's age by 4 years, what is Nikita's age? Answer: Let Ankit's age be x years. Then, Nikita's age = 240/x years. 2 * (240 /x ) – x = 4 480 – x2 = 4x x2 + 4x – 480 = 0 ( x+24)(x-20) = 0 x = 20. Hence, Nikita's age = 240/x = 240/20 years = 12 years. 214. The present age of a father is 3 years more than three times the age of his son. Three years hence, father's age will be 10 years more than twice the age of the son. Find the present age of the father. Answer: Let the son's present age be x years. Then, father's present age = (3x + 3) years (3x + 3 + 3) = 2 (x + 3) + 10 3x + 6 = 2x + 16 x = 10. Hence, father's present age = (3x + 3) = ((3 * 10) + 3) years = 33 years. 215. Rohit was 4 times as old as his son 8 years ago. After 8 years, Rohit will be twice as old as his son. What are their present ages? Answer: Let son's age 8 years ago be x years. Then, Rohit's age 8 years ago = 4x years. Son's age after 8 years = (x + 8) + 8 = (x + 16) years. Rohit's age after 8 years = (4x + 8) + 8 = (4x+ 16) years. 2 (x + 16) = 4x + 16 2x = 16 => x = 8. Hence, son's 'present age = (x + 8) = 16 years. Rohit's present age = (4x + 8) = 40 years. 216. One year ago, the ratio of Gaurav's and Sachin's age was 6: 7 respectively. Four years hence, this ratio would become 7: 8. How old is Sachin ? Answer: Let Gaurav's and Sachin's ages one year ago be 6x and 7x years respectively. Then, Gaurav's age 4 years hence = (6x + 1) + 4 = (6x + 5) years. Sachin's age 4 years hence = (7x + 1) + 4 = (7x + 5) years. (6x+5) : (7x + 5) = 7 : 8 8(6x+5) = 7 (7x + 5) 48x + 40 = 49x + 35 x = 5. Hence, Sachin's present age = (7x + 1) = 36 years.

Page 35

GovernmentAdda.com

QUANTITATIVE APTITUDE – 250 WORD PROBLEMS

Like My Facebook Page

217. Abhay's age after six years will be three-seventh of his father’s age. Ten years ago the ratio of their ages was 1: 5. What is Abhay's father's age at present? Answer: Let the ages of Abhay and his father 10 years ago be x and 5x years respectively. Then, Abhay's age after 6 years = (x + 10) + 6 = (x + 16) years. Father's age after 6 years = (5x + 10) + 6 = (5x + 16) years. (x + 16): (5x + 16) = 3:7 7(x + 16) = 3 (5x + 16) 7x + 112 = 15x + 48 8x = 64 => x = 8. Hence, Abhay's father's present age = (5x + 10) = 50 years. 218. The Ratio of Ages of Mona and Sona is 4:5. Twelve Years hence, their ages will be in the ratio of 5:6. What will be Sona's age after 6 years ? Answer: Let their present ages be 4x & 6x Then (4x + 12)/(5x + 12) = 5/6 or x=12 Sona's age after 6 years = (5x +6) = 66 years 219. Ramu was 4 times as old as his son 8 years ago. After 8 years, Ramu will be twice as old as his son. What their present ages ? Answer: Let son's age 8 years ago be x years Then Ramu's age at that time = 4x years Son's age after 8 years = (x +8) + 8 = (x + 16) years Ramu's age after 8 years = (4x + 8) + 8 = (4x + 16) years 2(x + 16) = 4x + 16 or x=8 Son's present age = (x + 8) = 16 years Ramu's present age= (4x + 8) = 40 years 220. A man is four times as old as his son. Five years ago, the man was nine times as old his son was at that time. What is the present age of a man ? Answer: Let son's age = x, then man's age =4x. 9(x - 5) = (4x-5) or x=8. Man's present age = (4x + 7) = 35 years 221. 1.The average of 8 numbers is 20.The average of first two numbers is 31/2 and that of the next three is 21⅓. If the sixth number is less than seventh and eighth number by 4 and 7 respectively, then eighth number is? Answer: let the eighth number be x. Then, sixth number= x-7 Seventh number=(x-7)+4 So, (2×31/2)+(3×21⅓)+(x-7)+(x-3)+x=8×20 Or, x= 25 222. The price of a car is Rs.3,25,000. It was insured to 85% of its price. The car was damaged completely in an accident and the insurance company paid 90% of the insurance. What was the difference between the price of the car and the amount received? Answer: Amount paid to the card owner =90% of 85% of 325000=Rs.248625 So, the required difference= Rs. (325000-248625)=Rs. 76375 223. Three containers have their volumes in the ratio 3:4:5.They are full of mixtures of milk and water in the ratio (4:1), (3:1) and (5:2) respectively. The contents of all these buckets are poured into a fourth container. The ratio of milk and water in the fourth container is? Answer: Let the containers contain 3x, 4x and 5x litres of mixture respectively Milk in first mix= (3x ×4/5)=12x/5 litres Water in first mix=(3x-12x/5)=3x/5 litres Milk in second mix=(4x × 3/4) = 3x litres Water in second mix=(4x-3x)= x litres Milk in third mix=(5x × 5/7)=25x/7 litres Water in third mix=(5x-25/7)=10x/7 litres Total milk in final mix=314x/35 litres Total water in final mix=106x/35 litres Required ratio of milk and water=314x/35 : 106x/35=157:53

Page 36

GovernmentAdda.com

QUANTITATIVE APTITUDE – 250 WORD PROBLEMS

Like My Facebook Page

224. Two pipes A and B can fill a tank in 48min and 16min respectively. If both the pipes are opened simultaneously, after how much time B should be closed so that the tank is full in 18 minutes? Answer: let B be closed after x min Then, part filled by A+B in x min+part filled by A in (18-x)min=1 x(1/48+1/16)+(18-x)×1/48=1 or, x=10 225. The speed of a train in the onward journey is 25% more than that in the return journey. The train halts for 1hour on reaching the destination. The total time taken for the total to and fro journey is 17 hours, covering a distance of 800km. The speed of the train in the onward journey is ? Answer: let the speed in return journey be x kmph Then speed in onward journey =125x/100=5/4x kmph Average speed=10x/9 kmph Therefore, 800×9/10x=16 So, x=45kmph So, speed in onward journey=(5/4 ×45)=56.25kmph 226. A train running at a 54kmph takes 20seconds to pass a platform. Next it takes 12seconds to pass a man walking at 6kmph in the same direction in which the train is going. Find the length of the train and the length of the platform? Answer: let the length of the train be x mt and the length of the platform be y mt. Speed of the train relative to man=54-6=48kmph or 40/3 m/s In passing a man, train covers its own length with relative speed So, length of the train =Relative speed × time=40/3 × 12=160 m Also, speed of the train=54 × 5/18=15m/s Therefore, x+y/15=20 Or, y=140m 227. Speed of boat in the standing water is 9kmph and the speed of stream is 1.5kmph. A man rows to a place to a distance of 105km and comes back to the starting point. The total time taken by the man is? Answer: speed upstream=7.5kmph Speed downstream=10.5kmph Therefore, total time taken= (105/7.5+105/10.5) hrs. = 24hrs 228. A jar full of whisky contains 40% alcohol. A part of this whisky is replaced by another containing 19%alcohol and now the percentage of alcohol was found to be 26%. The quantity of whisky replaced is? Answer: Since strength of the first jar=40% Strength of second jar= 19% Mean strength =26% So, using the rule of allegation, the ratio between the two quantities is=7:14=1:2 Therefore, required quantity replaced is=2/3 229. 8litres are draw from a cask full of wine and replaced with water. This operation is performed three more times. The ratio of quantity of wine now left in the cask to that of water is 16:65. How much wine did the cask hold originally? Answer: let the qty of wine in the cask originally be = x litres Then, quantity of cask left in the wine after 4 operations=[x (1-8/4)^4] litres Therefore, x (1-8/4)^4/x=16/81 =>x=24 230. What annual installment (in approximate figure) will discharge a debt of Rs.2000 due in 3 years at 15% simple interest? Answer: let each installment be Rs. X So, [x+x×15/100] + [x+x×15×2/100] + x=2000 Solving we get x~Rs.580 231. In a 20 km Tunnel connecting 2 villages X and Y, there are three gutters. The distance between gutters 1 and 2 is half the distance between gutters 2 and 3. The distance from village X to its nearest gutter, gutter 1 is equal to the distance of Village Y from gutter 3. On a particular day, the hospital in village X receives information that an accident has happened at the third gutter. The victim can be saved only if an operation is started within 40 minutes. An ambulance started from village X at 30 kmph and crossed the first gutter after 5 minutes. If the driver had

Page 37

GovernmentAdda.com

QUANTITATIVE APTITUDE – 250 WORD PROBLEMS

Like My Facebook Page

doubled his speed after that, what is the maximum time the doctor would get to attend the patient at hospital? Assume 1 min is elapsed for taking the patient into and out of the ambulance. Answer: XG1=YG3=30*(5/60)=2.5 km G1G3=(20-2.5-2.5)=15km VILLAGE X G1 G2 G3 VILLAGE Y (2.5km) (2km) G1G2:G2G3=1:2 G1G2=5 km and G2G3= 10 km Now time taken for reaching X to G3 and back to X from X to G1=5 min (given) From G1 to G3=(15/60)*60=15 min From G3 to X=(17.5/60)*60=17.5 min Time elapsed=1 min Total time taken= 5+15+17.5+1=38.5 Remaining time=40-38.5=1.5 min 232. Subham gets on the lift at the 11th floor of a building and rides up at speed of 57 floors per minute. At the same time, Sonalin gets on a lift at the 51st floor of the same building and rides down at the rate of 63 floors per minute. If they continue travelling at these rates, then at which floor will their paths cross ? Answer: Suppose their paths cross after x minutes. Then, 11 + 57x = 51 - 63x 120x = 40 X=1/3 number of floors covered in 1/3 min by David= 57/3=19 So, their paths cross at (11 +19) i.e., 30th floor. 233. A tap can fill a tank in 4 hours. After half the tank is filled, two more similar taps are opened. What is the total time taken to fill the tank completely? Answer: A tap can fill a tank in 44 hours. Therefore, the tap can fill half the tank in 22 hours. Remaining part =12=12 After half the tank is filled, two more similar taps are opened. Hence, total number of taps becomes 33. Part filled by one tap in 1 hour =14=14 Part filled by three taps in 1 hour =3×14=34=3×14=34 Time taken to fill 1212 tank by 33 pipes=(12)(34)=46=(12)(34)=46 hour =40=40 minutes Total time taken=2hour + 40 minute =2 hour 40 minutes. 234. What is the difference between the compound interests on Rs. 5000 for 1 ½ years at 4% per annum compounded yearly and half-yearly? Answer: CI when interest compounded yearly = Rs [5000x[1+(4/100)x(1+2)/100]=Rs 5304 CI when interest is compounded half yearly = Rs [5000 {(1+2)/100}^3]= Rs 5306.04 Difference = Rs. (5306.04 - 5304) = Rs. 2.04 235. Binod got 30% of the maximum marks in an examination and failed by 10 marks. However, Sumit who took the same examination got 40% of the total marks and got 15 marks more than the passing marks. What were the passing marks in the examination? Answer: Let maximum marks of the examination =x Marks that Binod got =30% of x =30x/100 Given that Binod failed by 10 marks. => pass mark =( 30x/100)+10 ---------- 1 Marks that Sumit got =40% of x =40x/100 Given that Sumit got 15 marks more than the passing marks. => pass mark =(40x/100)−15 ------ 2 From (1)and (2) (30x/100)+10=(40x/100)−15 =>10x/100=25 =>x/10=25 =>x=10×25=250 pass mark=(30x/100)+10=(30×250/100)+10=75+10=85

Page 38

GovernmentAdda.com

QUANTITATIVE APTITUDE – 250 WORD PROBLEMS

Like My Facebook Page

236. A train travels for 7 hours at the speed of 27 km/hr. and for 9 hours at the speed of 38 km/hr. At the end of it driver finds he has covered 3/7th of total distance. At what speed the train should travel to cover the remaining distance in 24 hours? Answer: Let the total distance is ‘x’ = 3x/7 = (7×27) + (9×38)= 531 x = (531 × 7)/3 = 1239 km. Remaining distance = 1239 - 531 = 708 km. Speed = (708/24) = 29.5 km/hr 237. How many different words can be formed with the letters of the word "TRANSFER" so that the words begin with 'T'? Answer: First letter ‘T’ is fixed, so remaining ‘7’ letters can be filled in 7!/2 ways as the letter ‘R’ comes twice. Total arrangements = 7!/2 = 2520 238. A bag contains 5 black and 3 white balls. A second bag contains 4 black and 2 white balls. One bag is selected at random. From the selected bag one ball is drawn. What is the probability that the drawn ball is black ? Answer: Probability of selecting first bag = 1/2 and probability of drawn ball is black is 5c1 / 8c1 = 5/8 P (E1) = (1/2) ×(5/8), similarly P (E2) = (1/2) ×(4/6) P (E) = (5/16) + (1/3) = (15+16)/48 = 31/48 239. Two pipes 'A' and 'B' would fill a tank in 36 hours and 45 hours respectively. If both pipes are opened together, find when the first pipe must be closed so that the tank may be just filled in 30 hours ? Answer: Let the first pipe is closed after ‘t’ hours. (t/36) + (30/45) = 1, (t/36) = 1- (2/3) = 1/3 t=36× (1/3) = 12 hours. 240. A shopkeeper buys 5 tables and 8 chairs for Rs.5000. He sells the tables at a profit of 12% and chairs at a loss of 8%. If his total gain is Rs.80 then what price does he pay for a table and a chair ? Answer: Let the price of a table is ‘x’ and chair is ‘y’ 5x + 8y = 5000 (I) 12% of 5x = 5x × (12/100) = 3x/5 and 8% of 8y = (16y/25) = (3x/5) – (16y/25) = 80 15x – 16y = 2000 (II) Solving equn (I) and (II) x = 480 and y = 325 241. Population of a city is 1.2 lakh. If the population of male increases by 5% and the female by 10%, the population will be 1.2835 lakh. What is the number of female in the city ? Answer: Let the population of female is ‘x’. Population of male = 1.2 – x = (110x/100) + [(105/100) (1.2 - x)] = 1.2835 110 x + 126 - 105x = 128.35 5x = 128.35 - 126 = 2.35 x = (2.35/5) = 0.47lakh = 47000 242. A shopkeeper marks his goods 20% above the cost price but give 11% discount on it. If he sells the article for Rs.1575.30 then what is the cost price ? Answer: Let the cost price is ‘x’ x ×(120/ 100) × (89/100) = 1575.3 x = (157530 × 100) / (120×89) = 1475 243. If Rs. 6200 amounts to Rs. 8804 in 3 years 6 months, what will Rs. 7800 amount to in 4 years 6 months at the same rate percent per annum ? Answer: S.I. = 8804 - 6200 = 2604 r = (2604 × 100) / (6200 × 3.5) = 12% p.a Now for Rs. 7800, S.I. = (7800 × 4.5 × 12)/100 = 4212 Req. amount = 7800 + 4212 = 12012

Page 39

GovernmentAdda.com

QUANTITATIVE APTITUDE – 250 WORD PROBLEMS

Like My Facebook Page

244. The compound interest on a certain sum of money for two years at 8% p.a. is Rs. 499.20. What will be the simple interest at the same rate and for the same time period ? Answer: 2 P[1 +( 8/100)] – P = 499.20 2

P (27/25) – P = 499.20 P (729-625) / 625 = 499.20 P = (499.20 × 625) / 104 = 3000 S.I = (3000 × 8 × 2) / 100 = 480 245. Certain number of persons can do a work in 50 days. If there were 7 persons more the work could be finished in 14 days less. How many persons were there initially ? Answer: Let the original number of men ‘x’ 7 person (50 - 14 = 36) days work = x persons 14 days work x = (7 × 36)/14 = 18 2 246. Find the area of a circle whose radius is equal to the side of an equilateral triangle of area 9 √3 cm (find approximate area)Answer: 2 area of equilateral ∆ = √3/4×x where, ( x = side) 2 √3/4×x = 9√3 2 x = 36 x=6 à x = 6 cm. 2 à area of circle = πr , (where, r = x) 2

π× 6 = 113.04 247. In an election between two candidates, one got 55% of the total valid votes, 20% of votes were invalid. If the number of votes was 7500, what was the number of valid votes, 2nd candidate got? Answer: Valid votes = 80% of 7500 = 6000 2nd candidates got = 45% of 6000 = 2700 248. A bag contains 2 yellow, 3 green and 2 blue balls. Two balls are drawn at random, what is the probability that none of the balls drawn is blue? Answer: Total Balls = 2+3+2 = 7 5 7 2 balls drawn should not be blue à except blue, total is 5, so c2 out of total C2 5 7 so required probability = ( c2)/( C2) = 10/21 249. The ratio of height and diameter of a cylinder is 2 : 3. Find the ratio of its volume and curved surface area of radius 6 cm? Answer: h:d = 2:3 => h:2r = 2:3 => h/12 = 2/3 = 8 => Volume/C.S.A = (πr2h)/(2πrh) => 6/2 = 3/1 or 3:1 250. How many bricks are needed to complete a wall 15 m × 12 m × 10 cm. using bricks 24 cm × 25 cm × 10 cm. thick if 1/3 rd of the wall is already built? Answer: (1500×1200×10)/(24×25×10) = 3000 Since 1/3 rd is built, so required bricks = (2/3) × 3000 = 2000

Page 40

Brian Galvin Chris Kane

Data Sufficiency GovernmentAdda.com

Authors

Brian Galvin Chris Kane

Co-founders

Markus Moberg Chad Troutwine

Contributing Writers

David Newland Ashley Newman-Owens

Contributing Editor

Jodi Brandon

Cover Design

Nick Mason

Interior Design

Tom Ahn Dennis Anderson

GovernmentAdda.com ALL RIGHTS RESERVED. Printed in the U.S.A. Third Edition, Copyright © 2013 by Veritas Prep, LLC. GMAT® is a registered trademark of the Graduate Management Admissions Council, which is not affiliated with this book. No part of this publication may be reproduced, stored in a retrieval system, or transmitted in any form or by any means, electronic, mechanical, photocopying, recording, or otherwise without the prior written permission of Veritas Prep, LLC. All the materials within are the exclusive property of Veritas Prep, LLC. © 2013. Print Batch 2013.1

This book is dedicated to Veritas Prep’s instructors, whose enthusiasm and experience have contributed mightily to our educational philosophy and our students’ success. It is also dedicated to the teachers who inspired Veritas Prep’s instructors. The lesson that follows was only made possible by a lifelong love of learning and of undertaking educational challenges; we have teachers around the world to thank for that. Finally and most importantly, this book is dedicated to our thousands of students, who have taught us more about teaching and learning than they will ever know. And to you, the reader, thank you for adding yourself to that group.

Personal Dedications Veritas Prep is a community of educators, students, and support staff, and these books would not be possible without our cast of thousands. We thank you all, but would like to specifically acknowledge the following people for their inspiration:

GovernmentAdda.com

Clay Christensen (Harvard Business School), Tom Cotner (Plymouth-Salem High School), David Cromwell (Yale School of Management), Henry Grubb (Fort Osage High School), Dana Jinaru (Beat the GMAT), Steven Levitt (University of Chicago), Walter Lewin (Massachusetts Institute of Technology), Lawrence Rudner (Graduate Management Admissions Council), Jeff Stanzler (University of Michigan), and Robert Weber (Kellogg School of Management).

GovernmentAdda.com

Since 2002, more than 50,000 students have chosen Veritas Prep to help them prepare for the GMAT.

GovernmentAdda.com

Veritas Prep offers GMAT courses in more than 60 cities around the world, plus online classes available everywhere.

See how we can help you. >>

TABLE OF CONTENTS PREVIEW . . . . . . . . . . . . . . . . . . . . . . . . . . . . . . . . . . . . . . . . . . . . . . . . . . . . . . . . . . . . . . . . . . 7 How This Book Is Structured. . . . . . . . . . . . . . . . . . . . . . . . . . . . . . . . . . . . . . . . . . . . . . . . . . . 8 SKILLBUILDER . . . . . . . . . . . . . . . . . . . . . . . . . . . . . . . . . . . . . . . . . . . . . . . . . . . . . . . . . . . . 13 LESSON. . . . . . . . . . . . . . . . . . . . . . . . . . . . . . . . . . . . . . . . . . . . . . . . . . . . . . . . . . . . . . . . . . 35 Data Sufficiency: Resource Management . . . . . . . . . . . . . . . . . . . . . . . . . . . . . . . . . . . . . . 35 Data Sufficiency and the Veritas Prep Pyramid. . . . . . . . . . . . . . . . . . . . . . . . . . . . . . . . . 36 SECTION 1: DATA SUFFICIENCY FUNDAMENTALS . . . . . . . . . . . . . . . . . . . . . . 37 How to Approach Each Question................................................................................37 The Two Types of Data Sufficiency.............................................................................39 Data Sufficiency Fundamentals Summary...............................................................47 SECTION 2: THE DATA SUFFICIENCY TOOLKIT. . . . . . . . . . . . . . . . . . . . . . . . . . 49 1. Manipulate Algebraically...........................................................................................49 2. Use Conceptual Understanding..............................................................................50 3. Play Devil’s Advocate and Pick Numbers.............................................................51 4. Just Do It.........................................................................................................................52 The Data Sufficiency Toolkit Summary.....................................................................69

GovernmentAdda.com

SECTION 3: LEARNING TO PLAY THE GAME . . . . . . . . . . . . . . . . . . . . . . . . . . . . 71 The Two Wrong Answers...............................................................................................73 Sufficiency Drill: How to Leverage Assets and Play Devil’s Advocate.............74 Two Statements Are Better than One........................................................................75 Statements as Hints.........................................................................................................77 Playing Chess, Not Checkers.........................................................................................79 Learning To Play The Game Summary.......................................................................81

TABLE OF CONTENTS

SECTION 4: THE POWER OF CONSTRUCT THINKING . . . . . . . . . . . . . . . . . . . . 83 Why Are You Here?...........................................................................................................83 Additional Constructs.....................................................................................................87 Understanding the Reward System........................................................................ 100 What Makes Data Sufficiency So Hard?.................................................................. 101 The Power Of Construct Thinking Summary....................................................... 103 SECTION 5: YOU OUGHTA KNOW . . . . . . . . . . . . . . . . . . . . . . . . . . . . . . . . . . . . . 105 Don’t Contradict Yourself........................................................................................... 105 No News Is Good News............................................................................................... 107 HOMEWORK . . . . . . . . . . . . . . . . . . . . . . . . . . . . . . . . . . . . . . . . . . . . . . . . . . . . . . . . . 111 Challenge Problems..................................................................................................... 199 SOLUTIONS . . . . . . . . . . . . . . . . . . . . . . . . . . . . . . . . . . . . . . . . . . . . . . . . . . . . . . . . . 221

GovernmentAdda.com

ANSWER KEY. . . . . . . . . . . . . . . . . . . . . . . . . . . . . . . . . . . . . . . . . . . . . . . . . . . . . . . . 295

CR E ATING Think Like the Testmaker Creating is the top of the pyramid in Bloom’s Taxonomy. When you have completely mastered the GMAT, you are able to Think Like the Testmaker. You are on top of the pyramid looking down! You don’t just have good content knowledge and lots of practice with GMAT problems; you understand how a problem has been made, what makes it hard, and how to break it down. When you Think Like the Testmaker you can: 1.

Quickly recognize what the problem is actually asking,

2.

Discover hidden information and manipulate it to make it useful,

3.

Recognize and see through trap answers, and

4.

Create your own plan of attack for any problem.

APPLYING Skills Meet Strategy What makes the GMAT difficult is not so much the underlying skills and concepts, but rather the way those skills and concepts are tested. On the GMAT, what you know is only as valuable as what you can do with that knowledge. The Veritas Prep curriculum emphasizes learning through challenging problems so that you can:

GovernmentAdda.com

1.

Learn how to combine skills and strategies to effectively solve any GMAT problem,

2.

Most effectively utilize the classroom time you spend with a true GMAT expert, and

3.

Stay focused and engaged, even after a long day in the office.

R E M E M BE R ING Skillbuilder In order to test higher-level thinking skills, testmakers must have some underlying content from which to create problems. On the GMAT, this content is primarily: •

Math curriculum through the early high school level, and



Basic grammar skills through the elementary school level.

To succeed on the GMAT you must have a thorough mastery of this content, but many students already have a relatively strong command of this material. For each content area, we have identified all core skills that simply require refreshing and/or memorizing and have put them in our Skillbuilder section. By doing this: 1.

Students who need to thoroughly review or relearn these core skills can do so at their own pace, and

2.

Students who already have a solid command of the underlying content will not become disengaged because of a tedious review of material they’ve already mastered.

PREVIEW

LESSON PREVIEW PREVIEW

The educational philosophy at Veritas Prep is based on the multi-tiered Bloom’s Taxonomy of Educational Objectives, which classifies different orders of thinking in terms of understanding and complexity. At the base of Bloom’s pyramid are “Remembering” and “Understanding,” and everything above that - including “Analyzing,” “Applying,” and “Creating” - is considered Higher-Order Thinking. The GMAT is a self-professed test of Higher-Order Thinking, which is essentially the ability to take information and do something more with it. Consequently, our curriculum will force you to not just remember and understand the content that drives the test but also learn to apply it to the higher-level reasoning required for high GMAT scores. To achieve a high score on the GMAT, it is essential that you understand the test from the top of the pyramid. On the pages that follow, you will learn specifically how to achieve that goal and how this lesson in particular relates to the Veritas Prep Pyramid.

GovernmentAdda.com

7

How This Book Is Structured Our Curriculum Is Designed to Maximize The Value Of Your Time The Veritas Prep Teaching Philosophy: Learning by Doing Business schools have long featured the Case Method of education, providing students with real-world problems to solve by applying the frameworks they have studied. The Veritas Prep Learning by Doing method is similar. In the lesson portion of this book, you will find that many concepts and strategies are covered by leading with a practice question first, so that you review content and learn strategy while already engaged in the necessity of solving a problem. The Case Method in business school maximizes student engagement and develops higher-order thinking skills, because students must apply and create, not just remember. Similarly, the Learning by Doing philosophy maximizes the value of your study. An important note on Learning by Doing: In business school, your goal with a business case is not to simply master the details of a particular company’s historical situation, but rather to develop broader understanding of how to apply frameworks to real situations. In this course, you should be certain to reflect on each question not simply through that narrow lens (Did you answer correctly? What key word made the difference?), but rather as an example of larger GMAT strategy (How could the exam bait you with a similar trap? How deeply do you need to understand the content to solve this genre of problem more efficiently?).

GovernmentAdda.com

8

PREVIEW How This Book Is Structured

Throughout the Veritas Prep curriculum, you will be exposed to several important and recurring themes that you will see in most GMAT problems:

• • • • • • •

Abstraction Reverse-Engineering Large or Awkward Numbers Exploiting Common Mistakes Selling the Wrong Answer and Hiding the Correct Answer Misdirection Content-Specific Themes

SKILLS M E ET STR ATEGY • • •

Guiding Principles Problem-Solving Strategies Leveraging Assets

GovernmentAdda.com

9

PREVIEW

TH IN K LIKE TH E TESTMAKE R

Don’t mistake activity for achievement! Focus on recurring themes, not just underlying content. REMEMBER:

Each book in the Veritas Prep curriculum contains four distinct sections: 1. Skillbuilder. We strongly suggest that you complete each Skillbuilder lesson at your own pace, and return to the Skillbuilder when you recognize a content deficiency through practice tests and GMAT homework problem sets. The Skillbuilder section will: •

Cover content that is vital to your success on the GMAT, but is best learned at your own pace outside the classroom.



Allow you to review and/or relearn the skills, facts, formulas, and content of the GMAT. Each student will have his own set of skills that are “rusty” or even brand-new, and will find other items that come back quickly.



Vary in length significantly for each book, based on the number of underlying concepts. (For instance, the Advanced Verbal lesson does not have a Skillbuilder because you are already building on the concepts introduced in three previous lessons.)

GovernmentAdda.com

2. Lesson. The lessons are designed to provide students with maximum value added from GMAT experts by: •

Engaging in challenging problems (Learning by Doing), and



Analyzing those problems for the recurring takeaways.



With each problem, there will be a detailed explanation that will help you understand how the problem is testing a particular concept or series of concepts, what makes the problem hard, and what underlying skills are required to solve it.



When relevant, there will be particular boxes for Think Like the Testmaker, Skills Meet Strategy, and Skillbuilder when you should be focused on particular .

  N O T E : When doing in-class and homework problems, you should do your work below the problem, and you should not circle the answer on the actual question (just note it on the bottom of the page). That way, if you want to redo problems, you can simply cover up your work and proceed as if you had never done it.

10

PREVIEW How This Book Is Structured

3. You Oughta Know. The You Oughta Know sections will round out each lesson and cover: Obscure topics that arise infrequently.



More advanced topics that are not common on the GMAT but do get tested.



While these uncommon content areas do not warrant in-class time in the Veritas Prep classroom syllabus, it is still important that you have some exposure to these topics before taking the GMAT. Therefore you should complete these sections before moving to the homework problems. As with the Skillbuilders, the length of these will vary depending on their importance.

PREVIEW



4. Homework Problems. In many ways, the homework problems are the most important part of each book. After refreshing core content in the Skillbuilder and then applying that knowledge in the lesson, you must reinforce your understanding with more problems.

GovernmentAdda.com Each question is accompanied by a detailed explanation at the back of the book, as well as a quick-reference answer key on the last page. A majority of questions are above the 50th percentile in difficulty, and they are arranged in approximate order of difficulty (easiest to most difficult). By completing all of the homework problems, you will learn all of the different iterations of how concepts and skills are tested on the GMAT.

Homework problems are designed to be challenging, so do not despair if you are answering questions incorrectly as you practice! Your goal should be to learn from every mistake. Students can miss a significant percentage of questions in each book and still score extremely high on the GMAT, provided that they learn from each problem. Embrace the challenge of hard problems and the notion that every mistake you make in practice is one that you will know to avoid on the GMAT when every question counts.

11

GovernmentAdda.com

12

SKILLBUILDER

SKILLBUILDER

SKILLBUILDER

Data Sufficiency questions represent almost half of the Quantitative section, but for many students they make up much more than 50% of their concern. Why? Data Sufficiency questions are unique to the GMAT. Most people have completed thousands of multiple choice math questions in their academic career, but very few people have ever seen a Data Sufficiency question until their GMAT preparation. Data Sufficiency is challenging in large part because it’s unique, but keep this in mind: it’s unique and challenging for everyone, so by picking up a book and putting in the time to master the genre of Data Sufficiency, you’re gaining a substantial competitive advantage. In this Data Sufficiency lesson, you will learn to master much more than the simple Data Sufficiency structure. This lesson will teach you to: Skillbuilder •

Know the Data Sufficiency answer choices & the Decision Tree elimination approach



Understand the two types of Data Sufficiency questions: Yes/No and What is the Value

GovernmentAdda.com



Understand what constitutes “sufficiency” and how thin that line can be

Lesson •

Leverage the Data Sufficiency Toolkit to efficiently solve problems



“Play the Game” of Data Sufficiency, using clues embedded within the question to allocate time effectively and avoid trap answers



Recognize the Power of Construct Thinking to see the “Reward System” of Data Sufficiency

Homework •

Learn by Doing – build from basic problems, which allow you to practice the Decision Tree and become comfortable with the answer choices, to advanced problems, which will require you to Leverage Assets, Play Devil’s Advocate, and win the game of Data Sufficiency

13

Introduction – The Basics A NOTE TO SELF-STUDY STUDENTS This lesson appears halfway through the quantitative section of the Veritas Prep course, at a point at which students have reviewed and practiced the core skills related to Arithmetic, Algebra, Geometry, and basic Statistics, Probability, and Word Problems. This lesson focuses on strategy specific to Data Sufficiency, and will at times assume familiarity with the core math content of the GMAT. Problem explanations will review those core skills, but you will return the most value from this lesson if you’ve already begun reviewing that core content on your own. 

To begin understanding Data Sufficiency, it’s helpful to look at the official directions and answer choices for these problems, courtesy of the Graduate Management Admissions Council: Directions This Data Sufficiency problem consists of a question and two statements, labeled (1) and (2), in which certain data are given. You have to decide whether the data given in the statements are sufficient for answering the question. Using the data given in the statements plus your knowledge of mathematics and everyday facts (such as the number of days in July or the meaning of counterclockwise), you must indicate whether: (A) Statement (1) ALONE is sufficient, but statement (2) alone is not sufficient to answer the question asked; (B) Statement (2) ALONE is sufficient, but statement (1) alone is not sufficient to answer the question asked;

GovernmentAdda.com

(C) Both statements (1) and (2) TOGETHER are sufficient to answer the question asked; but NEITHER statement ALONE is sufficient. (D) EACH statement ALONE is sufficient to answer the question asked; (E) Statements (1) and (2) TOGETHER are NOT sufficient to answer the question asked, and additional data specific to the problem are needed.

You should see from the answer choices that the name of the game is “is the data sufficient?”, so let’s spend some time discussing what constitutes sufficient information.  

14

SKILLBUILDER

Understanding Sufficiency Data Sufficiency questions hinge on whether a statement is sufficient to answer a question. A statement is sufficient when it guarantees exactly one answer to that question. For example, in the question: Is integer x positive? The statement “x > 9” would be sufficient, as any number greater than 9 is also greater than 0 and therefore positive.

SKILLBUILDER

The statement “x2 > 81”, however, would not be sufficient, as there are two potential values of x: 9 (which gives the answer “yes, x is positive”) and -9 (which gives the answer “no, x is not positive”). Your job, then, is to determine when a statement is sufficient to provide exactly one answer to the overarching question. Try the following drill questions to get a feel for sufficiency: 1. Question: Is integer y positive? Statement: y3 > 27

GovernmentAdda.com

2. Question: What is the volume of cube W?

Statement: Cube W has a surface area of 96 square inches. 3. Question: What is the volume of rectangular box Z? Statement: Rectangular box Z has a surface area of 96 square inches. 4. Question: If all apples cost $A and all pears cost $P, what is the price of two apples and three pears? Statement: Together, nine apples and six pears cost $60. 5. Question: If all apples cost $A and all pears cost $P, what is the price of two apples and three pears? Statement: Together, six apples and nine pears cost $60.



15

Understanding Sufficiency – Solutions 1. Sufficient. Because 3 is an odd exponent, there is no chance of y being a negative integer. The cube root of 27 is 3, so this statement essentially tells you that y > 3, meaning all potential values of y are greater than 3. Because all potential values will give the same answer – yes – the statement is sufficient. 2. Sufficient. Because all sides of a cube are the same, a cube with a surface area of 96 will have six equal sides with area 16, meaning that each side has a length of 4. And a cube with a side of 4 has an area of 43, which is 64. Because this statement provides exactly one answer to the question – the cube has a volume of 64 – the statement is sufficient. 3. Not Sufficient. While this problem may look similar to the previous problem, the fact that this shape is a rectangle and not necessarily a square makes all the difference. There are multiple combinations of length, width, and height that could satisfy that surface area, which for a rectangle equals 2LW + 2LH + 2WH. Since this problem leaves three variables and can therefore lead to several different answers, the statement is not sufficient.

GovernmentAdda.com

4. Not sufficient. This problem asks you to solve for 2A + 3P, and gives the fact that 9A + 6P = 60. These two variables cannot be isolated in just one equation, and even simplifying the equation by dividing all terms by 3 to yield 3A + 2P = 20 does not allow you to solve for 2A + 3P. It could be that A = 6 and P = 1 (which means that 2A + 3P = 15) or it could be that A = 4 and P = 4 (which means that 2A + 3P = 20). Since the given information allows for at least two different answers to the question, this statement is not sufficient. 5. Sufficient. While this problem may look similar to #4, there is a key difference: When you simplify 6A + 9P = 60 by dividing all terms by 3, you get 2A + 3P = 20. And that’s exactly what the question asks you to solve for – what is 2A + 3P? Since this guarantees that the answer is 20, this statement is sufficient. It is important to pay particular attention to the exact question being asked! While you cannot solve for A or B individually, this question asks for the one particular combination – 2 of A, 3 of B – that you have exactly enough information to solve.

16

SKILLBUILDER

More important than those solutions, however, are these takeaways: A statement is sufficient when it guarantees exactly one (and only one) answer to the question.



This means that in a Yes/No question, you have sufficient information if the answer is “Definitely Yes” or if the answer is “Definitely No”. You do not have sufficient information when the answer is “Sometimes Yes but Sometimes No” (or “Maybe”).



This means that in a “What is the Value?” question, you have sufficient information when you can pin down exactly one value for the question, but you do not have sufficient information when more than one value is possible.



Data Sufficiency questions require attention to detail – the drills on the previous page came in pairs, and to the untrained eye each pair might have seemed the same. But subtle differences in what was given or asked – variable squared vs. variable cubed; cube vs. rectangular box; 2A + 3P vs. 3A + 2P – can make all the difference. The GMAT testmakers prey on this precision in wording frequently, so read carefully.

SKILLBUILDER



GovernmentAdda.com

17

Anatomy of a Data Sufficiency question While the genre of Data Sufficiency is unique, the structure is not – all Data Sufficiency questions are structured exactly the same, with three key elements: the question stem (which may or may not contain important facts), the statements (always two statements), and the answer choices (which are always exactly the same). Consider this example to see what a Data Sufficiency question will look like: QUESTION STEM STATEMENTS

By what percent was the price of a certain candy bar increased? (1) The price of the candy bar was increased by 5 cents. (2) The price of the candy bar after the increase was 45 cents.

(A) Statement (1) ALONE is sufficient, but statement (2) alone is not sufficient. (B) Statement (2) ALONE is sufficient, but statement (1) alone is not sufficient. (C) BOTH statements TOGETHER are sufficient, but NEITHER statement ALONE is sufficient.

GovernmentAdda.com

ANSWER CHOICES

(D) EACH statement ALONE is sufficient.

(E) Statements (1) and (2) TOGETHER are NOT sufficient.

18

SKILLBUILDER

If you’re relatively new to Data Sufficiency, the most striking feature of this question is likely the answer choices – they’re not numbers that might answer the questions, they’re more logical descriptions of when you might be able to answer the question. The good news is that the answer choices are always the same, so you have plenty of opportunities to practice with them. By the time you take the test, you shouldn’t even have to glance down at the answer choices because you’ll already know them so well, but that takes practice. To internalize the answer choices and have a system to attack them, you should use a system.



Is the information in statement (1) alone enough to answer the question?



Is the information in statement (2) alone enough to answer the question?



Can I answer the question if I combine the information from statements (1) and (2)? (Only ask this of yourself if neither statement alone was enough to answer the question.)

SKILLBUILDER

For every Data Sufficiency question, ask yourself the following questions (if starting with statement (1)):

GovernmentAdda.com

Data Sufficiency Decision Tree

Assess each statement to determine whether it is sufficient or not, and this tree will lead you to the correct answer:

YES

YES

Choose D

NO

Choose A

YES

Choose B

Statement (2) sufficient?

Statement (1) sufficient?

NO

Statement (2) sufficient?

YES NO

Choose C

Both (1) and (2) together sufficient? NO

Choose E

19

The Decision Tree is a helpful flowchart to ensure that your calculations and decisions lead you to the proper letter choice. To apply this system to the sample question: The question asks a “What is the Value?” question, asking specifically for the percent increase of the price of a candy bar. The typical components of a percent increase are the change in price and the original price. Statement 1 provides the percent change, but offers no way to solve for the original price, so statement 1 is not sufficient. This means that you can eliminate choices A and D, leaving only B, C, and E. Statement 2 ALONE provides the new price, but does not (ALONE) offer any insight as to the original price or the percent change, so statement 2 alone is not sufficient and you can eliminate choice B. Taking the statements together, you can combine them to see that if the increase is 5 cents and the new price is 45 cents, then the original price is 40 cents. The percent change can be calculated then by dividing the increase (5) by the original price (40) and multiplying by 100. But keep in mind this, too – you don’t have to! Since you know that by finishing these calculations you will arrive at one, exact answer, you’ve already proven that the statements together (but not alone) are sufficient, so you can save yourself the calculation and immediately choose answer choice C.

GovernmentAdda.com

That’s the Decision Tree in action, and keep in mind that once you’ve used it on a handful of questions it should start to sink in. Many students find it helpful to run through this process quickly by simply jotting down AD / BCE on their noteboard and eliminating one side of the slash after statement 1, then progressing through the elimination from there.



20

SKILLBUILDER

Decision Tree at Work Use the Decision Tree to work toward the correct answer on these problems: How much is 15% of a certain number? (1) 5% of the number is 15. (2) 30% of twice the number is 180. Choose D

NO

Choose A

YES

Choose B

SKILLBUILDER

YES

YES

Statement (2) sufficient?

Statement (1) sufficient?

NO

Statement (2) sufficient?

YES

Choose C

GovernmentAdda.com NO

Both (1) and (2) together sufficient? NO

Choose E

Statement 1 is sufficient. as you can set up the equation .05x = 15, allowing you to solve for x (the number in question). Eliminate B, C, and E

YES

YES

Choose D

NO

Choose A

YES

Choose B

Statement (2) sufficient?

Statement (1) sufficient?

NO

Statement (2) sufficient?

YES NO

Choose C

Both (1) and (2) together sufficient? NO

Choose E

Statement 2 is sufficient, as you can set up the equation 0.3(2x) = 180, again allowing you to solve for x. This means that the answer must be D.

21

YES

YES

Choose D

NO

Choose A

YES

Choose B

Statement (2) sufficient?

Statement (1) sufficient?

NO

Statement (2) sufficient?

YES NO

Choose C

Both (1) and (2) together sufficient? NO

Choose E

Note here, also, that once you had linear equations for the single variable x, you could confidently answer “sufficient” without finishing the math. Since your goal is to determine whether the information is sufficient to find x, you do not have to finish the steps to find x once you’ve proven that you will, indeed, arrive at exactly one value for x.

GovernmentAdda.com  

22

SKILLBUILDER

Anatomy of a Data Sufficiency Question: Note the Question Stem

If xy ≠ 0, what is the value of x? (1) yx2 + 4xy + 4y = 0

QUESTION STEM STATEMENTS

(2) y = 6

GovernmentAdda.com (A) Statement (1) ALONE is sufficient, but statement (2) alone is not sufficient to answer the question asked; (B) Statement (2) ALONE is sufficient, but statement (1) alone is not sufficient to answer the question asked; (C) BOTH statements (1) and (2) TOGETHER are sufficient to answer the question asked, but NEITHER statement ALONE is sufficient;

ANSWER CHOICES

(D) EACH statement ALONE is sufficient to answer the question asked; (E) Statements (1) and (2) TOGETHER are NOT sufficient to answer the question asked, and additional data are needed



23

SKILLBUILDER

By the time you take the GMAT, you shouldn’t have to read the answer choices – you should know exactly what they mean and have a system to arrive at the correct one (you’ll cover that soon). Importantly, though, you should also spend lots of time unpacking the information in the question stem. Everyone pays attention to the statements (as well they should) because the answer choices refer directly to them. The question stem? People often read it too quickly and miss crucial pieces of information embedded within it. Often the most important information is cleverly hidden in the question stem (or explicitly given there), not in the statements. To illustrate this, consider one more “Anatomy of a Data Sufficiency Question” example:

As you learn about Data Sufficiency, the answer choices and the decision tree should have a disproportionately high share of your attention. But as you consider the three elements of a Data Sufficiency question – Question Stem; Statements; Answer Choices – be sure not to rush through the Question Stem, which is often the ‘”sneakiest” part of the problem. Here that little caveat that xy ≠ 0 may not seem altogether useful, but it’s the lynchpin of this question. As you work through statement 1, you can factor it to: y(x + 2)(x + 2) = 0 Which should leave you with two options to solve that equation. Either x = -2, or y = 0. But that caveat that xy ≠ 0 tells you that neither x nor y can equal 0, so once you know that y is not 0, you can divide both sides of the statement 1 equation by y, and arrive simply at: (x + 2)2 = 0, so statement 1 is sufficient: x must be -2. You can eliminate B, C, and E, and since statement 2 provides no information about x (later in the lesson you’ll learn how to use these obviously-insufficient statements to your advantage), statement 2 is not sufficient and the answer is A.

GovernmentAdda.com

Your early practice with Data Sufficiency will almost certainly focus on the Decision Tree and mastering the answer choices – this is what we want you to do in the beginning. But a common study mistake of those shooting for 700+ scores who fall short and end up in the high 500s or low 600s is that they overvalue the importance of knowing and eliminating answer choices. The decision tree and the answer choices – which today may seem confusing, but by test day will be as natural as tying your shoes or brushing your teeth – are important but don’t neglect the information hidden in the question stem itself.

24

SKILLBUILDER

Sufficiency Drills As you’ve noted to this point, Data Sufficiency questions are all about when you have enough information to definitively answer a question. The Decision Tree is a useful tool to help you employ proper process-of-elimination and decision-making once you’ve begun to assess that sufficiency, but ultimately your success on Data Sufficiency questions will depend on how well you can determine whether statements are sufficient.

SKILLBUILDER

So before you dive too deeply into full Data Sufficiency problems and the answer choices, first complete this drill to become more confident with what constitutes sufficiency on these questions. In the following drills, you will see a single Data Sufficiency question stem for each exercise, followed by several individual statements. Your job: to determine which statements (ALONE) are sufficient, and which are not sufficient. Please note that, on full Data Sufficiency questions, the statements can be used together and therefore will never contradict each other. On this drill, however, the goal is simply to practice leveraging one statement at a time to think about sufficiency, so some of the statements may contradict one another.

GovernmentAdda.com

From this drill, you should focus on: •

What constitutes sufficiency.



How to leverage information in the question stem and statements to determine whether a statement is sufficient.



How to avoid assumptions.



What makes a statement tricky or difficult.

Directions: For each of the following questions, determine which of the statements, ALONE, would be sufficient to answer the question. Please note that there can be multiple sufficient statements for each question. 1. Is x > 0? (A) 3x is an integer (B) x2 < x (C) x2 – 3x + 2 = 0 (D) x is a prime number (E) x2 – 3x – 4 = 0

25

Solutions (A) Not sufficient. x could be a positive integer (e.g., 32 = 9, an integer, so x could be 2), but x could be 0 (30 = 1, also an integer). If x is 0, the answer is no, but for all other potential values of x (including 2), the answer is yes. Therefore, this statement is not sufficient. (B) Sufficient. This statement guarantees that x is between 0 and 1. For all negative numbers, their square is positive and therefore greater than x. For 0, its square equals 0 (and is therefore not less than 0). So negatives and 0 are not possible numbers given this statement, proving the answer yes. (C) Sufficient. Factoring this quadratic, you will find that (x – 2)(x – 1) = 0, so x = 2 or x = 1. Both are positive, so the answer must be yes. Note, also, that you could make this decision quickly, as in the form ax2 + bx + c, if b is negative and c is positive, then the potential values for x must be positive in order to satisfy the equation. (A negative x would mean that all three terms would be positive, making it impossible for their sum to equal 0.) (D) Sufficient. All prime numbers are, by definition, positive.

GovernmentAdda.com

(E) Not sufficient. This quadratic factors to (x – 4)(x + 1) = 0, meaning that x could be 4 or -1. Because 4 supplies the answer yes and -1 supplies the answer no, we cannot conclude a definitive answer to the question. Note also that this statement looks quite similar to statement (C). Be careful with quadratics and Yes or No questions. You will often need to do some work to determine whether the multiple solutions support the same answer or not. Exercise takeaways: The first statement shows that “Is x > 0?” is a completely different question from “is x < 0?” As zero is the dividing line between positive and negative, it can play an interesting role in Data Sufficiency questions. Statement (A) does not allow for any negative numbers, so you might think that it guarantees that x is positive, but it only promises that x is non-negative, a small but significant distinction. Beware the power of 0 in Data Sufficiency! Also note the similarity and difference between the two quadratics in statements (C) and (E). In Yes or No questions, a statement that allows for multiple values can be sufficient or not sufficient, depending on whether all the values give the same answer to the overall question.

26

SKILLBUILDER

2. In isosceles triangle ABC, what is the measure of angle A? (A) Angle B = 30 degrees (B) Angle C = 120 degrees (C) Angle A is one-fourth the measure of angle C (D) Angles A and B add to half the measure of angle C

SKILLBUILDER

— (E) The length of side BC = 2 √ 2

GovernmentAdda.com

27

Solutions (A) Not sufficient. You know that the sum of A + B + C is 180 degrees, and that either A = B, B = C, or A = C. But this statement does not allow us to determine whether, say, A = B = 30 and C = 120, or A = 120 and B = C = 30. (B) Sufficient. Because the sum of all angles must be 180, it is not possible for both C and another angle to match at 120 each. So if C is 120, then A and B must combine for the other 60, and since two sides must be equal, those are the equal sides, at 30 each. (C) Not sufficient. This statement still allows for either A to equal B (both 30 degrees, ¼ of 120 so that all three angles add up to 180), or for B to equal C (both 80, with A filling in the remaining 20). (D) Sufficient. This statement guarantees that C is the largest angle, meaning that A must equal B. This allows you to calculate A + B + C = 180, and A = B, and C + ½(C) = 3 180. So (C) = 180, meaning that C = 120, and A and B are each equal 30. 2

(E) Not sufficient. While this statement may follow the trend of an isosceles right — triangle (in which the side ratios are x, x, x√ 2 ), the problem gives no indication that this is a right triangle—just this one “symptom.”

GovernmentAdda.com

Exercise Takeaways: Notice that the question stem embeds information (the triangle is isosceles) that needs to be “unpacked” with the various statements. It’s not quite enough information to state an algebraic fact (you know that one angle equals another, but you can’t yet make that an equation), but it’s information that will undoubtedly be important, so you do need to find a way to leverage it with each statement. Also notice that statements (A) and (B) seem to be extremely similar statements, but are wholly different; and that statements (C) and (D) may also look fairly similar, but are substantially different. Data Sufficiency statements are often drawn “to the limit”; they’re often edging up against that line between sufficiency and not, and you need to read carefully and apply the given information in a few ways to ensure that you make the right decision.

28

SKILLBUILDER

3. Is integer x divisible by 12? (A) x is the product of three consecutive positive integers (B) x is the product of three consecutive even integers (C) x is the product of three consecutive prime numbers (D) x2 is divisible by 36

SKILLBUILDER

(E) x2 is divisible by 72

GovernmentAdda.com

29

Solutions (A) Not sufficient. x could be the product of 1, 2, and 3 (x = 6), but it could also be the product of 12, 13, and 14 (and therefore clearly divisible by 12). This statement does tell us that x is divisible by 6 (in any set of three consecutive integers, at least one will be even and at least one will be divisible by 2), but it is not sufficient to say whether x is divisible by 12. (B) Sufficient. In any set of three consecutive even integers, at least one will be divisible by 3. Algebraically, you can see this by noting these integers as 2a • (2a + 2) • (2a + 4). Factor out the 2s: 2(a) • 2(a + 1) • 2(a + 2), and you’ll see that you have as factors 23 • three consecutive integers. Clearly, then, one of those three consecutive integers will be divisible by 3, and you will have at least four factors of 2 in that group. This is sufficient to prove that the product will contain the prime factorization of 12: 2 • 2 • 3. (C) Sufficient. This is one of the cases (they’re rare but significant) in which the statement is enough to prove “no” as the answer to the question. In order to be divisible by 12, a number needs to have the prime factors 2 • 2 • 3. Only one prime number, 2, has a factor of 2, so this statement is enough to prove that x will be missing at least one of the required factors of 2.

GovernmentAdda.com

(D) Not sufficient. This statement tells us that x is divisible by 6, but is not enough to determine whether x is divisible by 12. At a minimum, x = 2 • 3, but “is divisible by 6” could include other factors (for example, 12 is divisible by 6). (E) Sufficient. This is a fairly challenging statement for many examinees. If x2 = 2 • 2 • 2 • 3 • 3 • (something) and x is an integer, then the odd 2 after you’ve broken apart the pairs of prime factors must have a pair of its own. Otherwise, x wouldn’t be — an integer, as that 2 would divide into √ 2 , an irrational decimal. Remember: That “(something)” in the factorization is there to show that x is divisible by, but not limited to, 72. The definition that x is an integer guarantees that the prime factors of x2 will form pairs so that x does not carry a radical after that root is taken. Therefore, x2 must have 2 • 2 • 2 • 2 • 3 • 3, meaning that x is divisible by at least 2 • 2 • 3. Exercise Takeaways: The first three statements all rely on definitions, and require you to unpack and apply those definitions to more concretely see the parameters of the problem. Many Data Sufficiency questions will offer “light” information that only has substance when you apply what you’re given. From statement (C), note that in a Yes or No question, the answer no is sufficient; we’ll discuss this more later, as it’s only used in smaller percentage of Data Sufficiency problems (don’t think that half the time the answer is no), but when it is it can be a hurdle for many test-takers.

30

SKILLBUILDER

4. If a = 5, what is value of b + c? (A) a is the average of the set {a, b, c} (B) ab + ac = 10 (C) a is the median of the set {a, b, c} (D) ba + bc = 5

SKILLBUILDER

(E) a + b = b + c

GovernmentAdda.com

31

Solutions a+b+c

(A) Sufficient. This statement tells us that 3 We know that a = 5, so b + c must equal 10.

= 5, which means that a + b + c = 15.

(B) Sufficient. Knowing that a = 5, this statement tells us that 5b + 5c = 10. Divide both sides by 5 to find that b + c = 2. (C) Not sufficient. While this statement looks quite similar to statement (A), it is quite different. The median is just the middle number in a set, which only means that one of b or c is 5 or less, and the other is 5 or greater. But the sets could include: {4, 5, 6} and {-10, 5, 7}, for example. This statement is not sufficient. (D) Not sufficient. While this statement looks similar to statement (B), it is also quite different. Once you plug in 5 for a, our only known variable, you still have one equation with two variables, and cannot manipulate them to get a value for b + c. 5b + bc = 5; which can also be phrased as b(5 + c) = 5. Either way, you cannot isolate b + c. You might also consider plugging in numbers to find different values. The statement is satisfied, for example, if b = 1 and c = 0; or if b = ½ and c = 5. Because you can get different answers, the statement is not sufficient.

GovernmentAdda.com

(E) Not sufficient. This statement allows you to solve for c (subtract b from both sides and you’ll find that a = c, and we know that a is 5). That also means that b could be absolutely anything, so the statement is not sufficient. Exercise Takeaways: Even when multiple variables are given in a problem, you need not always solve for all variables if the question asks for a combination of them (here, what is b+c?). Again, notice the similarity between some of the statements, but how the slight differences make some sufficient and others not. Learn to read carefully.

32

SKILLBUILDER

5. A pet store sells only dogs and cats. If the ratio of dogs to cats is 3:2, how many dogs are in the pet store? (A) If the number of cats were to double, the ratio would be 3:4. (B) If 3 cats were added, there would be more cats than dogs. (C) If 5 cats and 3 dogs were added, there would be more cats than dogs. (D) If 4 cats and 6 dogs were added, there would be no change to the ratio.

SKILLBUILDER

(E) If 4 cats were added, there would be twice as many cats as before.

GovernmentAdda.com

33

Solutions (A) Not sufficient. This statement adds no new information; the current totals are 3x dogs and 2x cats, so doubling the number of cats would just make it 2(2x) = 4x cats. We already know that that new ratio would be 3:4. (B) Not sufficient. But close! With a ratio of 3:2, the possibilities are 3 dogs and 2 cats; 6 dogs and 4 cats; 9 dogs and 6 cats; etc. In each of the first two cases, three more cats would tip the ratio so that there were more cats: 3 dogs and 2+3 cats  more cats. And 6 dogs and 4 + 3 cats = more cats. Because there are two potential solutions, the statement is insufficient. (C) Sufficient. Using the same ratios as in the previous solution: 3 + 3 dogs < 2 + 5 cats, but 6 + 3 dogs = 4 + 5 cats. Only with a starting value of 3 dogs and 2 cats does this statement hold true, so the statement guarantees that the store has 3 dogs. (D) Not sufficient. Like in statement (A), we already know this. We currently have 3 dogs for every 2 cats, so adding 6 dogs and 4 cats simply keeps the ratio the same. (3x + 6 dogs and 2x + 4 cats  3(x+2) dogs and 2(x+2) cats, algebraically.)

GovernmentAdda.com

(E) Sufficient. This statement tells us that 2x + 4 = 2(2x). (We start with 2x for the number of cats; adding four is the same as multiplying by 2.) That lets us solve for the multiplier (x = 2), which we can plug back into the total for dogs (3x, if x = 2 then there are 6 dogs). Exercise Takeaways: Ratio problems in Data Sufficiency can be quite difficult, as the GMAT authors have quite a few ways to obscure information. Here, nearly all the statements should look to be similar in scope, but notice the difference particularly between statements (C) and (D): Only the numbers themselves change—and not that dramatically—but the answer is completely different. Throughout this drill we have included these subtle changes to statements to show you that you need to read carefully, interpret information effectively, and do some work to unpack the information you’re given. In the lesson that follows, you will have plenty of opportunities to practice.

34

LESSON Data Sufficiency: Resource Management

LESSON Data Sufficiency: Resource Management Data Sufficiency is a question form unique to the GMAT. While you will not see Data Sufficiency questions on any other exam, you will see plenty on the GMAT, which has increased its pool of Data Sufficiency questions to now constitute close to half of the quantitative section. What makes Data Sufficiency such an integral part of the GMAT? Data Sufficiency cuts to the core of what MBAs will need to do, testing primarily your ability to efficiently and effectively manage resources. To truly understand what Data Sufficiency is all about, view answer choice C (and remember that the answer choices are fixed; they will always say the exact same things): (C) BOTH statements TOGETHER are sufficient, but NEITHER statement ALONE is sufficient.

GovernmentAdda.com LESSON

Data Sufficiency questions require you to know exactly when you have enough information to make a decision. If you use extra information that you don’t need, or make a premature decision without enough information, you will be wrong. To succeed on Data Sufficiency questions, you must maximize the value of each statement to glean as much information as possible, but you must also be careful not to overvalue a statement. Proper Data Sufficiency technique requires efficient “resource management” of information in each statement. Fortunately, the question type comes with rules and restrictions that actually work in your favor. As you work through this lesson, you will learn the common mistakes that the question format baits examinees into making, and you will learn the parameters of the question format that will help you astutely play the game as though you are playing chess (or poker) against the authors of the GMAT. To succeed, you will rely on a balanced mix of your logical reasoning skills and the math skills that you have developed thus far.

35

Data Sufficiency and the Veritas Prep Pyramid Because this lesson is primarily about the question type itself, the underlying content will not be the focus. Almost all of the takeaways from this lesson relate to the top of the pyramid. If there is one question type on the GMAT in which you must Think Like the Testmaker to succeed, it is Data Sufficiency. There will be a series of themes relating to the construct of Data Sufficiency that represent the very top of the pyramid. There will also be a series of guiding strategies from the middle of the pyramid—the Data Sufficiency Toolkit—that will help you decide how to behave on Data Sufficiency questions relating to certain content types. Learning how and when to use these important tools is another key takeaway from this lesson. The following skills and/or takeaways will be particularly highlighted in the different sections of this book: “Core Skills” from Skillbuilder • Decision Tree • Understanding Sufficiency “Skills Meet Strategy” Takeaways from the Lesson Section • The Data Sufficiency Toolkit 1. Manipulate algebraically. 2. Use conceptual understanding. 3. Pick numbers and play devil’s advocate. 4. Just do it. • Leveraging Assets • Learning by Doing

GovernmentAdda.com “Think Like the Testmaker” Takeaways from the Lesson Section • Construct Thinking • Misdirection • Selling the Wrong Answer

36

1

SECTION 1: DATA SUFFICIENCY FUNDAMENTALS How to Approach Each Question

SECTION 1: DATA SUFFICIENCY FUNDAMENTALS

How to Approach Each Question The following is an outline of the core approach that you should use every time you answer a Data Sufficiency question: 1. Read the question carefully and assess all information that is provided (or not provided) in the question stem. Organize this information so that you understand exactly what you will need to sufficiently answer the question. Note: On many Data Sufficiency questions, the most important information is cleverly hidden in the question stem itself.

GovernmentAdda.com

3. Make a quick judgment on which statement is easier to assess and start with that one. The order in which statements are analyzed does not matter. By starting with the easier statement, you simplify the decision tree and leverage easier information first. 4. Use the decision tree to carefully consider each piece of information separately and then together if necessary. For each question ask yourself the following questions (if starting with statement (1)): •

Is the information in statement (1) alone enough to answer the question?



Is the information in statement (2) alone enough to answer the question?



Can I answer the question if I combine the information from statements (1) and (2)? Only ask this of yourself if neither statement alone was enough to answer the question.

37

LESSON

2. Avoid careless assumptions. Do not assume anything that is not explicitly provided in the question stem or the statements that follow. For instance, do not assume that x and y are integers unless it is explicitly given or can be deduced from the question stem or statements. Unless instructed otherwise, assume that fractions, negatives, and zero are all included in the set of potential values.

GovernmentAdda.com

38

1

SECTION 1: DATA SUFFICIENCY FUNDAMENTALS The Two Types of Data Sufficiency

The Two Types of Data Sufficiency Questions •

Yes or No



What Is the Value?

Regardless of the type you are dealing with, your job is the same: If you can use a statement (or statements) to guarantee exactly one answer to the main question, then that information is sufficient. For Yes or No questions, if a statement provides a definitive yes or no to the question, it is sufficient. For What Is the Value? questions, if a statement provides exactly one value, then it is sufficient. For Yes or No questions, two common traps exist that tend to work against novice test-takers. 1. A statement allows for multiple values, but all values provide the same answer to the Yes or No question.

GovernmentAdda.com Consider a few drills to highlight these pitfalls and decide if the one statement is sufficient:

Yes or No Drills Is x > 5? (1) x2 = 16



Is x greater than 5? (1) 2x – 15 = 17(x – 15) + 171

39

LESSON

2. The statement only allows you to obtain the answer of no—but since always no is a consistent answer, that means that the statement is sufficient (even though the answer is not yes.) Remember: You are not trying to prove that the answer is yes—only that it is either yes or no, and not both.

In the second drill on the previous page, you saw that it is easy to do too much work on a Data Sufficiency question. Typically the bigger problem is that people do not do enough work to properly analyze the information given. Beware of the possibility of information that looks sufficient but upon further inspection is not, as you will see in first What Is the Value? example below. For What Is the Value? questions, most of the pitfalls relate to assumptions that people make about the problem or restrictions that people miss in the problem. In a What Is the Value? question the following mistakes are most common: 1. People assume that values must be integers and/or positive. With that assumption(s), a statement appears to be sufficient when there are actually multiple values possible. 2. People miss restrictions in the problem that do guarantee that the numbers involved are, for instance, integers, so they think there are multiple possibilities, when indeed that statement proves one exact value.

GovernmentAdda.com

40

1

SECTION 1: DATA SUFFICIENCY FUNDAMENTALS The Two Types of Data Sufficiency

Consider two drills that highlight some of these common pitfalls.

What Is the Value? Drills

What is the value of x? (1) x2 = 9x

What is the value of x? (1) x2 - 19x = 5x - 144

41

LESSON

GovernmentAdda.com

Detailed Solutions to Drills Yes/No Drill #1  While x could be 4 or -4, the answer is always “No, x is not greater than 5.” “No” means “sufficient” if it is a consistent conclusion. Remember that no answers are not common in Yes/No questions, but when they are utilized it is usually in tricky cases such as this. Yes/No Drill #2  Here you know that you have a linear equation that will simplify to one value of x. Once you have that value, you will know whether or not it is greater than 5. You don’t need to do the work, as you can see that simply taking each step will always produce one exact value and therefore one distinct answer. What Is the Value? Drill #1 Solution  It is easy to look at this quadratic and assume that x must be 9. However, if you move 9x to the left side of the equation, you see that x2 – 9x = 0 and x (x - 9) = 0, so x= 0 or 9. You can avoid this assumption by properly factoring the quadratic or just remembering about 0, one of the most important numbers on the GMAT. What Is the Value? Drill #2 Solution  In the first drill, you had to remember that there are two solutions with most quadratic equations. Here you might just be tempted to think that there are two solutions so it won’t be sufficient. However, when you factor the quadratic, you realize that it is a perfect square: x2 – 24x + 144 = 0  (x – 12)2 = 0 and x is 12, so the statement is sufficient.

GovernmentAdda.com

42

1

SECTION 1: DATA SUFFICIENCY FUNDAMENTALS The Two Types of Data Sufficiency

Yes/ No Review Problem







1. Is the triangle above equilateral? (1) x = y (2) z = 50

(B) Statement (2) ALONE is sufficient, but statement (1) alone is not sufficient to answer the question asked (C) BOTH statements (1) and (2) TOGETHER are sufficient to answer the question asked, but NEITHER statement ALONE is sufficient (D) EACH statement ALONE is sufficient to answer the question asked (E) Statements (1) and (2) TOGETHER are NOT sufficient to answer the question asked, and additional data are needed

Discussion Questions 1. What is the answer and why? 2. What makes this hard? 3. What is the answer if the question was “Is the triangle above isosceles?”

43

LESSON

GovernmentAdda.com (A) Statement (1) ALONE is sufficient, but statement (2) alone is not sufficient to answer the question asked

LE AR N ING BY DO ING No Is the Same as Yes The primary difficulty in this problem is that people forget about the no answer. If a statement proves that the triangle could never be an equilateral triangle, then that statement is sufficient. In statement (1), you learn that two of the angles are identical, so the triangle must be isosceles, but it does not have to be equilateral. Statement (1) provides a maybe answer to the question, so it is not sufficient. People have a tendency to immediately dismiss statement (2) as insufficient because it says nothing about the other angles. But remember: If you know that one angle is 50 degrees, then you are 100% sure that the triangle could never be equilateral. Therefore statement (2) is sufficient by itself, and answer choice B is correct.

SKILLS MEET STR ATEGY Understand What Constitutes Sufficiency Remember: On a Yes or No question, either “definitely yes” or “definitely no” leads to an answer of sufficient. The only answer that is not sufficient is maybe.

Discussion Question Answers: 1.

The answer is B as explained above – because it is necessary in an equilateral triangle for ALL angles to be 60 degrees, if just one angle is not 60 degrees you know that this is not an equilateral triangle.

2.

What makes this problem difficult is that your mind wants to know something about all three angles. Your inclination might well be “but this statement doesn’t tell me anything about angles x or y!”. Data Sufficiency is made difficult in many ways because of statements like this that don’t tell the entire story, but give just enough information to answer the specific question being asked.

3.

This problem is also difficult because, as you saw in the previous drills, the definitive answer to the question is “no”, which in your mind often corresponds directly with “eliminate / cross-out statement 2”. Keep in mind that a definitive “no” answer means that the information is sufficient.

4.

If the question asked whether the triangle was isosceles, statement 1 alone would have been sufficient but not statement 2, making the answer A. For an isosceles triangle, only two angles need to be equal, and statement 1 proves that that is true. The lesson? Precision in wording matters. Changing that one word “equilateral” to “isosceles” flipped statement 1 from not sufficient to sufficient and statement 2 from sufficient to not sufficient. Pay careful attention to the specific wording of each question and statement!

GovernmentAdda.com

44

1

SECTION 1: DATA SUFFICIENCY FUNDAMENTALS The Two Types of Data Sufficiency

What Is the Value? Review Problem 2. What is the value of x? (1) 6 < 2x < 10 (2) x2 = 16

(A) Statement (1) ALONE is sufficient, but statement (2) alone is not sufficient to answer the question asked (B) Statement (2) ALONE is sufficient, but statement (1) alone is not sufficient to answer the question asked (C) BOTH statements (1) and (2) TOGETHER are sufficient to answer the question asked, but NEITHER statement ALONE is sufficient (D) EACH statement ALONE is sufficient to answer the question asked

Discussion Questions 1. What is the answer? 2. What are the traps? 3. How close is the answer to being answer choice D? 4. What if the question stem were to begin “x represents the number of children on a field trip?” or “x denotes the volume, in gallons, of water in a tank?”

45

LESSON

GovernmentAdda.com (E) Statements (1) and (2) TOGETHER are NOT sufficient to answer the question asked, and additional data are needed

LE AR N ING BY DO ING Don’t Make Assumptions The question on the previous page highlights several traps specific to the Data Sufficiency format. While they are easy to notice here, it is essential that you keep them in mind on test day. Testmakers know that you tend to think via “counting numbers” unless told specifically otherwise (quick: Pick a number 1 through 10. We bet you didn’t pick 6.5, or pi. You probably chose an integer). But on the GMAT, all numbers are possible unless specifically ruled against; that’s why we advise that you play devil’s advocate when picking numbers so that the “trap” numbers, usually nonintegers and negative numbers, work in your favor.

SKILLS MEET STR ATEGY Understand What Constitutes Sufficiency Remember: On a What Is the Value? question, a sufficient statement will lead to exactly one and only one value. If a statement leaves the potential for even just a second possible value, it is not sufficient.

This question has three major traps embedded within it: 1. Statement (1) wants you to assume that x is an integer, but it need not be. x could be 4—but it could also be 4.99. 2. Statement (2) wants you to assume that x is positive, but it could be negative. x could be 4—but it could also be -4.

GovernmentAdda.com

3. Statement (2) wants you to “remember” statement (1). If you’ve already decided via Statement (1) that x is greater than 3, then you may feel justified in eliminating -4 as an option. But you don’t know Statement (1)—yet. Remember: Answer choice B requires that Statement (2) ALONE is sufficient. You cannot use Statement (1) until you’ve considered Statement (2) by itself. As you can see now, to solve this you need to have BOTH statements, so answer choice C is correct, but it is easy to think that one or each of the statements ALONE is sufficient. Discussion Question Answers: 1.

As outlined above, the correct answer is C.

2.

Statement 1 tries to trap you into thinking that x must be an integer (4), and statement 2 tries to trap you into thinking that x must be positive (eliminating the other potential value, -4) – either because you assume d it, or because you remembered that facet of statement 1.

3.

Had the question only allowed for positive integers, each statement alone would be sufficient to prove that x is 4. But as you saw in the traps, this was not the case – nonintegers and negative numbers were perfectly allowable. Make sure that you consider nonintegers, negative numbers, 0, and “numbers nearing infinity” when you’re assessing potential values in Data Sufficiency questions. Often the traps on these problems are built upon our human nature to think primarily about “counting numbers” – small, positive integers.

4.

Either of those question stems would have made statement 2 sufficient – neither case allows for a negative number. And the first question stem – “x represents the number of children…” – would make statement 1 also sufficient, as you cannot have a noninteger number of children. The lesson? Pay attention to the “backstory” on Data Sufficiency questions, as often the seemingly-mundane elements of a word problem will guarantee that the variable in question has to have certain properties (positive, integer, etc.).

46

SECTION 1: DATA SUFFICIENCY FUNDAMENTALS

1

Data Sufficiency Fundamentals Summary

Data Sufficiency Fundamentals Summary As you have seen in this section, it is essential that you understand the following fundamental strategies and rules relating to Data Sufficiency: Use a consistent approach to each Data Sufficiency question. Always organize the question first and make sure you have leveraged every piece of information that is given (or not given!) in the question before you move to the statements. When you go to the statements, start with the easier statement, as it simplifies the decision tree and allows you to leverage the easiest information first.



Completely master the decision tree. If you are still making mistakes with the decision tree, practice 20–30 easy Data Sufficiency questions to memorize the structure.



Understand the two different types of Data Sufficiency questions and what constitutes sufficiency for each one. For Yes or No questions, a statement is sufficient if it provides either a definitive yes or no answer. For What Is the Value? questions, a statement is sufficient if it provides exactly one value for the question.

GovernmentAdda.com

As tricky as Data Sufficiency can look at first, the way in which they are constructed ensures that you can attack them systematically. As you know, the answer choices are always the same. And you should also know that there are only two types of question stems that the GMAT can ask:

1. Yes or No Questions •

A statement gives multiple solutions, but they all give the same answer.



A statement provides a no answer instead of a yes answer.

2. What Is the Value? Questions





A statement appears to be giving one value because you have assumed properties of the number that were not actually given (positive, integers, etc.).



Restrictions were placed in the problem that you did not properly leverage (for instance, the problem is asking for the number of children, which must be an integer and cannot be negative).

Avoid assumptions. Every time you approach a Data Sufficiency problem, you must actively consider any assumptions that you may have been baited into making. Avoiding assumptions is perhaps the most important skill in all with Data Sufficiency.

47

LESSON



GovernmentAdda.com

48

2

SECTION 2: THE DATA SUFFICIENCY TOOLKIT Manipulate Algebraically

SECTION 2: THE DATA SUFFICIENCY TOOLKIT With Data Sufficiency, there are numerous ways you might attack a problem: You could consider possible values or think about the problem conceptually; you might manipulate algebraically; you might just do some math. Certain strategies work best for certain types of Data Sufficiency questions, so when you are attacking a Data Sufficiency question, you should be prepared to use any of the following four tools from the Data Sufficiency Toolkit (or some combination thereof ), depending on which is the most efficient:

1. Manipulate Algebraically

One of the biggest mistakes that students make with Data Sufficiency is that they always try to solve conceptually or with number picking. Algebraic manipulation and mirroring are the most efficient way to solve many Data Sufficiency questions. Why burden your brain with deep conceptual thought or time-consuming number picking when algebraic manipulation can give you a definitive answer? Importantly, do not forget that algebraic manipulation should be used not only on the statements, but also on the question (and, of course, it might involve both).

GovernmentAdda.com Drills Is 2x = 3y + 2z? (1) x – z = 3y 2

For integers a, b, and c, a = 1. What is the value of b–c ? b–c b (1) a = 3 b 5

49

LESSON

As the Veritas Prep Algebra lesson highlights, many GMAT problems contain “an inconvenient truth”—information that is sufficient to solve a problem but comes in an inconvenient form that needs to be manipulated. Accordingly, when you see algebra (or when you construct your own equation in a word problem) on a Data Sufficiency question, you should try to make the statement look like the question, or vice versa.

SKILLS MEET STR ATEGY Algebraic Manipulation and Mirroring

2. Use Conceptual Understanding Many Data Sufficiency questions hinge on your ability to understand deeply the underlying concepts being tested. For these questions, algebra may not work well, and number picking might be too time-consuming or won’t work at all. Many Arithmetic problems are best done with this conceptual approach, and that is especially true in Data Sufficiency. Importantly, though, if you do not fully understand a problem conceptually, you should try to use algebra or number picking to help you determine sufficiency.

Drill What is the ratio of the number of cats in a pet store to the number of dogs in a pet store?

SKILLS MEET STR ATEGY Conceptual Understanding One of the keys to success on Data Sufficiency questions is figuring out which of the approaches presented in this section to use on any given problem. There is no magic rule, and you need to be flexible, but generally speaking arithmetic problems are often best solved conceptually (and some can only be solved conceptually). It is very important to note that attacking a problem conceptually (particularly complicated word problems and inequalities) is a dangerous business. You may think you know a concept deeply, but the problem you are doing involves a particular exception or trick that you don’t recognize or understand. As noted, only answer problems conceptually when you are sure that you truly understand the underlying concept or if there is no other way to approach it. If you have any doubt, do some math or number picking to confirm your answer.

GovernmentAdda.com

(1) If there were twice as many cats in the store and five fewer dogs, the ratio of cats to dogs would be 5 to 2.

50

2

SECTION 2: THE DATA SUFFICIENCY TOOLKIT Play Devils’s Advocate and Pick Numbers

3. Play Devil’s Advocate and Pick Numbers Often to solidify your conceptual understanding or even begin a confusing Data Sufficiency problem, you will need to pick numbers. Be careful: Number picking without a goal can be extremely time-consuming and often leads to an incorrect answer. When you do choose to pick numbers, remember that your goal is to play devil’s advocate. Don’t pick the same types of numbers over and over, but rather pick the types of numbers (negatives, nonintegers, 0, large numbers) that are likely to give “the other answer” in a Yes or No question. For instance, if the statement appears to be giving you a yes answer for all the numbers you are picking, you should be actively looking for that one unusual case which would give you a no answer. In a “What is the Value?” question your goal with number picking is simply to show that there are two or more possible solutions from that piece of information, as that automatically results in insufficiency.

SKILLS MEET STR ATEGY Number Picking

GovernmentAdda.com Example: Is xy < 0?



Positive/negative

(1) |x-y| > |x|-|y|



Odd/even



Nonintegers



Prime factors



0 and 1

51

LESSON

Number picking should not be used as a “frontline” approach on most Data Sufficiency problems. It is rarely required to solve questions, but when it is, you need to know how to do it effectively (and sometimes it is the best approach). As you learned above, you need to number pick with the purpose of playing devil’s advocate. To do that, it is important that you keep in mind five common number properties when testing numbers.

4. Just Do It While a great many (safe to say most) Data Sufficiency problems are logic puzzles, some of them remain math problems in a tricky form. If you don’t see a “game” being played, you can begin doing the problem just as a math problem and see where the answers lead you. Often the GMAT will construct these problems so that you have to use one of the above three techniques or you’ll waste plenty of precious time. But some of questions are still best solved the old-fashioned way, so sometimes you have to “just do it” as you would in a problem-solving question.

Drill Is x > 4? (1) x2 – 7x + 6 = 0

SKILLS MEET STR ATEGY Some Data Sufficiency Problems Are Just About the Math While most Data Sufficiency problems have a strong logical basis and require a “game theory” approach (which you will learn shortly), some are just math problems. Certainly, if a problem can be solved conceptually or with good reasoning skills, you should not turn it into a time-consuming, “problemsolving style” math exercise. However, often a purely mathematical approach is required and/or preferred (and safer!), so don’t be afraid to just do the math in order to get a definitive answer. This is particularly true for many content areas in Data Sufficiency, such as quadratic equations or inequalities.

GovernmentAdda.com

52

2

SECTION 2: THE DATA SUFFICIENCY TOOLKIT Just Do It

Detailed Solutions to Drills 1a. By taking the equation in the statement and multiplying through by 2, you get 2x – 2z = 3y, and then adding 2z to both sides rephrases the question as a fact. Yes, 2x = 3y + 2z, so this statement is sufficient. This is called mirroring. Whenever a statement exactly mirrors the question stem, it is sufficient. 1b. Note how important it is here to manipulate the question instead of the statement. If you manipulate the given relationship a to reveal that a = b – b–c b–c c, then the question “What is the value of ” becomes, more simply, “What b a a 3 is the value of ”? That information is given directly in the statement = . b b 5 Therefore this statement is sufficient to answer the question.

GovernmentAdda.com

3. In an absolute value problem with multiple variables, performing the algebra takes on too many permutations to be efficient). But picking numbers— particularly in this case, in which the question is asking about a positive/ negative number property!—can be quite effective. (However, algebra is the preferred approach for most inequality problems.) If you were to try two positive numbers, say 8 and 6, you would get:

|8 – 6|= |8| – |6|  You cannot try those two positive numbers, because they do not satisfy statement (1).



Now your goal is clearer. Can you find a case in which the left-hand side does not equal the right? You may look at the left-hand side and see that, if you make y negative, then you would actually add on the left-hand side (x minus a negative…) whereas on the right, the negative within the absolute value would still represent a positive absolute value. So try a case in which x is negative and y is positive:

53

LESSON

2. Here you should not have to set up the algebra to recognize that, without a baseline number, you cannot tell the impact of five fewer dogs on the ratio. Therefore this statement is not sufficient. If there were over 1,000 dogs, losing five dogs wouldn’t make much of an impact, but if there were only 10 dogs to begin with, reducing the number by five would drastically change the proportion. Generally speaking, adding to or subtracting from a ratio is not sufficient information to gauge its impact on the ratio, unless you have a starting or ending total number. The main exception? If you add or subtract amounts in line with the current ratio (example: The cat/dog ratio is 2:1 and you subtract 2 cats and one dog. The ratio doesn’t change.).



|8 – (–6)| > |8| – |–6|  This works, as 14 is greater than 8 – 6. So as of now the only numbers we’ve been able to use are a positive x and a negative y, which give us the answer yes to the overall question.



Now you have one more goal: Can you get the answer no? We’ve already ruled out positive/positive, recognizing that to make the left-hand side different from the right, y needs to be negative. Let’s try negative/negative:



|(–8) – (–6)| = |–8| – |–6| This yields 2 = 2, so we can’t use both negative numbers. This means that the answer must be yes, because we can only use a positive x and a negative y. By number picking, we were able to hone in on our goal and choose appropriate numbers quickly.



Seems like it’s sufficient, right? Looking back, though, you might have missed one case that will give a no answer. What if they are both positive but y is bigger than x? Then |(6) – (8)| > |6| – |8| so both can be positive and you have a no answer. Therefore this statement is not sufficient. This highlights the difficulty of number picking and why it should generally be used as a last resort.

GovernmentAdda.com

4. Here you need to know the values of x in order to answer the question, and to get those answers you need to factor out the quadratic. Conceptual understanding can take you only so far (the numbers multiply to a positive 6 but add to a negative 7, so both parentheticals must be in the form x – y), but to ensure a correct answer you should finish the math. The quadratic factors to (x – 6)(x – 1) = 0, so x = 6 or x = 1. Because there is a yes and a no, the statement is not sufficient. Just knowing that you will get two different values for x is not enough to know that this problem is not sufficient. You have to know that at least one value is greater than 4 and at least one value is less than or equal to 4. If the two values for x had been 1 and – 6, then this statement would be sufficient, since both of those values are less than 4.

54

2

SECTION 2: THE DATA SUFFICIENCY TOOLKIT Just Do It

Now that you have seen the Data Sufficiency Toolkit, let’s apply the different strategies to some difficult GMAT problems.

Manipulate Algebraically 3. What is the value of x2 – y2? (1) x + y = 0 (2) x + y = 2x

(A) Statement (1) ALONE is sufficient, but statement (2) alone is not sufficient to answer the question asked (B) Statement (2) ALONE is sufficient, but statement (1) alone is not sufficient to answer the question asked

(D) EACH statement ALONE is sufficient to answer the question asked (E) Statements (1) and (2) TOGETHER are NOT sufficient to answer the question asked, and additional data are needed

55

LESSON

GovernmentAdda.com (C) BOTH statements (1) and (2) TOGETHER are sufficient to answer the question asked, but NEITHER statement ALONE is sufficient

LE AR N ING BY DOING Manipulate Statements and Questions By the time you take the GMAT you should recognize the algebraic “Difference of Squares” rule quickly. When you see the question stem (What is the value of x2 – y2), you should already see it as two questions: What is the value of x2 – y2, and what is the value of (x + y)(x – y)? With that in mind, statement (1) is clearly sufficient. If x + y = 0, then the second, manipulated question reads: “What is the value of 0(x – y)?” Clearly the answer is 0. With statement (2), manipulating the given equation will also show that it is sufficient. If x + y = 2x, then by subtracting x from both sides you find that y = x or x – y = 0. Again, this proves that the expression in the question stem equals 0, so answer choice D is correct, because each statement alone is sufficient. Remember that any one algebraic expression or equation can tell you multiple things: x + y = 7 also means x = 7 – y and y = 7 – x. When something is given to you in one form in Data Sufficiency, look at it in multiple forms to see which one might unlock the problem.

GovernmentAdda.com SKILLS ME ET STR ATEGY Don’t Forget to Change Questions to Match Statements

People’s natural inclination in Data Sufficiency is to leave the question alone and try to make the statements match it. One of the most important guiding strategies in Data Sufficiency, particularly on hard problems, is to change the question to match the statements. Often that manipulation is difficult or tedious, but once you do it, the answer is obvious. Of course, as you learned with this problem, don’t forget to properly manipulate any statements as well!

56

2

SECTION 2: THE DATA SUFFICIENCY TOOLKIT Just Do It

THINK LIKE THE TESTMAKER Selling the Wrong Answer Every Data Sufficiency question is designed for people to pick one particular incorrect answer choice. On this problem, many people (even those with high-level algebra skills) will pick answer choice A. Why? Because the testmakers have set you up for failure with the way the information is presented. After you manipulate the question, you see clearly that you are on the right track when you go to statement (1). You feel good about yourself because you have quickly recognized the difference of squares and substituted 0 for the x + y portion of the manipulated question. Your natural inclination with statement (2) is simply to take the value of x + y (which is now 2x) and plug that back into the question, as you did with 0 on the previous statement. That seems to leave all kinds of variables in the expression, so you think the statement is not sufficient and pick answer choice A. The problem, of course, is that you did not also try to manipulate statement (2) and get it in a convenient form. Don’t take things at face value in Data Sufficiency; always look to apply all of the strategies outlined in this section.

SKILLBUILDE R • Common algebraic equations • Algebraic manipulation

57

LESSON

GovernmentAdda.com

GovernmentAdda.com

58

2

SECTION 2: THE DATA SUFFICIENCY TOOLKIT Just Do It

Think Conceptually 4. Is integer k a prime number? (1) k = 10! + m, where 1 < m < 8 (2) k is a multiple of 7

(A) Statement (1) ALONE is sufficient, but statement (2) alone is not sufficient to answer the question asked; (B) Statement (2) ALONE is sufficient, but statement (1) alone is not sufficient to answer the question asked; (C) BOTH statements (1) and (2) TOGETHER are sufficient to answer the question asked, but NEITHER statement ALONE is sufficient; (D) EACH statement ALONE is sufficient to answer the question asked;

59

LESSON

GovernmentAdda.com (E) Statements (1) and (2) TOGETHER are NOT sufficient to answer the question asked, and additional data are needed.

LE AR N ING BY DOING Understanding the Number Line On a problem such as this, you must use conceptual understanding of the number line and divisibility to answer the question. Algebraic manipulation will not help you. Number picking will not help you. And certainly just doing it will not help you; calculating the number would be impossible! The goal from each statement is to prove whether k is prime or importantly not prime. It is impossible without computer or calculator assistance to determine whether a large number is actually prime (there are too many numbers you would have to check for divisibility), but it is actually quite easy to prove that a number is not prime. If you can prove that k is divisible by anything other than 1 and itself, you have proven that k is not prime. It is on this type of problem that you should be looking to disprove the question and find a no answer. In statement (1), you learn that k = 10! + either 2, 3, 4, 5, 6, or 7. To review, 10! (10 factorial) represents the product of all positive numbers from 1 to 10, inclusive: 10 x 9 x 8 x 7 x 6 x 5 x 4 x 3 x 2 x 1. As you can see, the number 10! is a multiple of each of 2, 3, 4, 5, 6, and 7. For demonstration, say that m were 7. 10! + 7 will then definitely be divisible by 7. If you find any multiple of 7 on the number line and add another 7 to it, it will always still be divisible by 7. Take the number 63, a multiple of 7. If you add 7 to 63, you get 70, another multiple of 7. This is then true for any potential value of m. If m were 2, then 10! (an even number) + 2 will remain divisible by 2. If m were 3, then 10! (a multiple of 3) + 3 will remain divisible by 3. Statement (1) thus proves that k is not prime, as whatever value of m (2 through 7) we add to 10!, k will remain divisible by that number, so it cannot be a prime number. Statement (1) is sufficient.

GovernmentAdda.com

Be careful with statement (2). If you carry some information with you from statement (1)—the fact that k is a large multiple of 7—you might think that statement (2) is also sufficient. In other words, you might think, if k is a multiple of 7, then it could never be prime, as it will be divisible by 7. However, remember that there is one multiple of 7 that is prime: 7 itself. Statement (2) is not sufficient, because k could be prime (7) or it could be any of the infinite set of multiples of 7 that are not prime. The answer to this question is answer choice A.

60

2

SECTION 2: THE DATA SUFFICIENCY TOOLKIT Just Do It

THINK LIKE THE TESTMAKER “No” Answers in Data Sufficiency In Yes or No Data Sufficiency questions, if a statement is sufficient, it almost always gives an affirmative yes answer. However, in certain types of questions, testmakers will cleverly insert statements that give a definitive no answer. This question is a classic example: You are desperately trying to figure out how to determine that k is prime when you should be trying to show that it is not prime. Remember also the equilateral triangle problem from the previous section. Testmakers know when you will forget to disprove things, and they cleverly create problems that exploit this weakness.

SKILLS ME ET STR ATEGY Don’t Carry Information

SKILLBUILDE R • Prime numbers • Divisibility

61

LESSON

GovernmentAdda.com

One of the common traps used by testmakers is to create questions in a way that you will carry some small but important piece of information from statement (1) to statement (2). Always make sure that you are properly assessing each statement, independent of the other, with only the information provided in the question stem and that one statement. You will learn shortly the art form of leveraging hints from the other statement, but you must not use any information from the other statement when actually determining sufficiency.

GovernmentAdda.com

62

2

SECTION 2: THE DATA SUFFICIENCY TOOLKIT Just Do It

Just Do It 5. Is y a positive number? (1) 2x + y > 27 (2) x – 3y < 24

(A) Statement (1) ALONE is sufficient, but statement (2) alone is not sufficient to answer the question asked (B) Statement (2) ALONE is sufficient, but statement (1) alone is not sufficient to answer the question asked (C) BOTH statements (1) and (2) TOGETHER are sufficient to answer the question asked, but NEITHER statement ALONE is sufficient

GovernmentAdda.com (E) Statements (1) and (2) TOGETHER are NOT sufficient to answer the question asked, and additional data are needed

63

LESSON

(D) EACH statement ALONE is sufficient to answer the question asked

LE AR N ING BY DO ING Just Do the Math Some inequality problems, particularly those that ask whether a value is > 0 or < 0, lend themselves well to conceptual understanding. But most of time you will find, as with this problem, that there is too much “action” in the statements to warrant a quick conceptual estimate of what the statements mean. Whenever you are in doubt, it’s generally a good idea to “just do it”—just perform the mathematical operations to solve for a variable or complete a calculation. In many cases you will be able to stop short of the finish line once a few steps have made the picture clearer, but regardless there will be times when you simply need to do the math. Here, it should be quickly apparent that neither statement alone is sufficient, as each equation allows for any possible value of x. But together, the statements allow you to do the math. Arrange the equations such that the inequalities face the same direction: 2x + y > 27 24 > x – 3y

SKILLS MEET STR ATEGY Do the Algebra on Most Inequality Problems Inequality problems in Data Sufficiency form are one of the most problematic question types for GMAT students. Why? Because students try to solve them with awkward, tedious number picking or with suspect conceptual approaches. As for all problems on the GMAT, you must be flexible; sometimes you will use number picking on these problems and sometimes you will use a conceptual approach. But for well over 80% of these problems, they are best solved by algebraic manipulation. Either you will be able to isolate the required variable or perhaps mirror the inequality that is given in the question stem. Regardless, to use this important strategic approach, you must understand deeply how to properly manipulate inequalities. When you are given inequality problems in Data Sufficiency form, go to algebra first, and mix in number picking and/or conceptual understanding when needed.

GovernmentAdda.com

Then manipulate the second equation to get the variables on the same side: 3y – x > -24

Then double the bottom equation to allow for an elimination of the x term: 6y – 2x > -48 Then you can combine the inequalities: y + 2x > 27 6y – 2x > -48 7y > -21 y > -3 You’ll find that y could be negative (it could be -2 or -1) or positive (all positive numbers are greater than -3), so the statements together are not sufficient, and answer choice E is correct.

SKILLBUILDE R • Inequalities • Algebraic manipulation

64

2

SECTION 2: THE DATA SUFFICIENCY TOOLKIT Just Do It

Play Devil’s Advocate/Pick Numbers 6. If x is a positive integer less than 30, is x odd? (1) When x is divided by 3, the remainder is 2. (2) When x is divided by 5, the remainder is 2.

(A) Statement (1) ALONE is sufficient, but statement (2) alone is not sufficient to answer the question asked (B) Statement (2) ALONE is sufficient, but statement (1) alone is not sufficient to answer the question asked (C) BOTH statements (1) and (2) TOGETHER are sufficient to answer the question asked, but NEITHER statement ALONE is sufficient (D) EACH statement ALONE is sufficient to answer the question asked

65

LESSON

GovernmentAdda.com (E) Statements (1) and (2) TOGETHER are NOT sufficient to answer the question asked, and additional data are needed

LE AR N ING BY DOING Smart Number Picking While it’s possible to solve this question with a conceptual understanding, it is much easier to put some numbers to work for you. When you do employ numbers, remember that your goal is to play devil’s advocate. Your goal is to determine whether x is an odd number, so you will likely start with an odd number that satisfies statement (1). 5 works 5 here, as 5 divided by 3, as = 1 remainder 2. So x could be odd. Now that you’ve found 3 an odd value of x—the answer yes to the overall question—your goal should change. You want to find an even value, because that would show that the statement is not sufficient. If you try everything you can think of and cannot find an even value of x, then you can conclude that it is sufficient. You want to play devil’s advocate to ensure that either x must be odd, or conclude that the statement is not sufficient. With that in 8 mind, you might try 8: 8 divided by 3 provides a remainder of 2 ( = 2 remainder 2). So 3 now you have an even potential x—and the answer no to conclude that statement (1) is not sufficient. The same process works for statement (2). 7 is an odd number that does the same, so x could still be odd, providing a yes answer. But 12 is an even number that satisfies statement (2), so you can get the answer no, and the answer is thus still maybe. Statement (2) is not sufficient. Taken together, the statements provide a bit more information, as now you know that x provides a remainder of 2 when divided by 3 and when divided by 5. You might recognize 17 as such a number, noting that 15 is the least common multiple of 3 and 5, so 17 will divide out that 15 and leave 2 remaining. Here’s where you really need to play devil’s advocate: If you chart out the values that work with each statement and look for matches between them, you may well conclude that 17 is the only such value less than 30:

GovernmentAdda.com Remainder 2 when divided by 3

Remainder 2 when divided by 5



7

5

8

12

11

17

14

22

17 (MATCH; x could be odd)

27

20

32

23

37

26

26

29 32 (MATCH; x could be even—but not even and less than 30)

66

2

SECTION 2: THE DATA SUFFICIENCY TOOLKIT Just Do It

But still play devil’s advocate. Is there any even number that could fit the bill? There is, but it’s very hard to find unless you remember what happens when you divide a smaller number by a larger number. 2 also works. When 2 is divided by 3, the quotient is 0 and the remainder is 2. When 2 is divided by 5, the quotient is 0, and the remainder is 5. 2 is the even counterpart, and although it may not be as readily clear as 17, if you force yourself to play devil’s advocate and consider the entire range of numbers available to you, you will often find that “catch” upon which correct answers often depend. The correct answer to this problem is E, but the authors of the question are betting that you will forget to consider 2 and therefore fall into the trap of selecting C.

SKILLS ME ET STR ATEGY When to Number Pick

THINK LIKE THE TESTMAKER Exploiting Common Mistakes

GovernmentAdda.com

Quotient/remainder problems such as this are deeply rooted in divisibility and how the number line works. If you understand those concepts well, you can avoid lots of number picking by leveraging your understanding of repeating patterns with divisibility on the number line. Still, there are many difficult quotient/remainder problems in which number picking is the only reasonable way to efficiently solve the problem. Remember: While number picking is often a tedious and inefficient approach, on some problem types, such as this, it is the best way to show whether certain information is sufficient.

SKILLBUILDE R • Division • Divisibility and the number line • Number properties

67

LESSON

Clearly this is a very difficult question as it exploits an oddity of arithmetic that even math PhDs might have forgotten: When you divide a smaller number by a larger number, the quotient is always 0 and the remainder is always the dividend. This unusual property of division has been used to create many tricky Data Sufficiency questions, because there is always that one value that people forget (for this problem, it is 2). As you prepare for the GMAT, file away these common mistakes so that you do not make them again if they happen to show on test day. While this is not an extremely common mistake, it is certainly one that has been used to make some particularly difficult questions.

GovernmentAdda.com

68

2

SECTION 2: THE DATA SUFFICIENCY TOOLKIT The Data Sufficiency Toolkit Summary

The Data Sufficiency Toolkit Summary To succeed on Data Sufficiency, you must break down questions and statements so that you get as much information as possible from them. To do that effectively, you should use one of the four tools in the Data Sufficiency Toolkit: 1. Manipulate algebraically. 2. Use conceptual understanding. 3. Play devil’s advocate and pick numbers. 4. Just do it. These four strategies from the toolkit are summarized in the following list: Manipulate Algebraically •



GovernmentAdda.com Since algebra “tells the truth,” this is often the safest and most efficient way to show sufficiency. Don’t waste time with number picking or conceptual thinking if you can prove something algebraically. Don’t forget that it is often more important to manipulate the question than the statements.

Use Conceptual Understanding •

Conceptual thinking is the best approach for most Arithmetic problems. Because problems about ratios, the number line, percents, etc. are more about the concept than actual calculations, doing math and or algebra can often be avoided.



If you are not sure conceptually, then you should prove sufficiency by doing some math or picking numbers.



Testmakers are good at finding exceptions to concepts that you think you understand well, so be careful when solving Data Sufficiency problems on a purely conceptual basis.

69

LESSON



Any time you are given algebra that can be manipulated, look to do that strategically. Often you will be able to make the statement mirror the question or vice versa.

Play Devil’s Advocate and Pick Numbers •

Number picking is an important strategy, but it is one that should only be used when necessary, as it can be time-consuming and make you error-prone.



If you have to number pick, make sure that you pick with the purpose of playing devil’s advocate and finding the exception. Smart number picking is key!



Certain questions types and scenarios lend themselves to number picking. Quotient/remainder problems and scenario-driven min/max word problems are great examples of problems in which you should number pick.

Just Do It •

You should treat many Data Sufficiency problems as problem-solving questions and just do the math until you can see the answer.



Quadratics, inequalities, and many word problems are examples of question types in which you must solve to prove sufficiency.

GovernmentAdda.com

70

3

SECTION 3: LEARNING TO PLAY THE GAME

SECTION 3: LEARNING TO PLAY THE GAME At this point in the lesson, you have learned the fundamentals of data sufficiency and been given a set of tools to use in assessing statements and questions. But data sufficiency is much more than just a question format with rules. It is a highly sophisticated game in which you are going head-to-head with the testmaker. Success on Data Sufficiency questions relies heavily on a “game theory” approach in which you leverage any hints given by the testmaker and you learn how to use the question format to your advantage. To play the game well, it is important to note that there are really only two mistakes you can make on Data Sufficiency questions. To consider this, look at the following variations two drills: 1. A recipe for mixed nuts includes only whole peanuts and cashews and calls for a strict peanut:cashew ratio of 7:3.  How many peanuts are in a bag? (1) The packaging facility guarantees that each bag will contain no fewer than 95 and no more than 105 nuts.

GovernmentAdda.com (2) The packaging facility guarantees that each bag will contain no less than 30 ounces and no more than 33 ounces of nuts.

In the first example you have to leverage the fact you are dealing with integers and that there are 10 total parts in the ratio. Because you need a whole-number multiplier, and because the number of total nuts is between 95 and 105, then there must be a total of 100 nuts and thus 70 peanuts. People get this wrong because they do not leverage all the mathematical assets given in the problem. In the second example, particularly after doing the first one, you may think that it is a similar situation. There are still 10 total parts, and the total amount is greater than or equal to 30 but less than 33. It has to be 30 total ounces, right? Not in this case, because you can have a fractional multiplier with weight. You do not need to have integer ounces and the multiplier is not known from that information. People get this wrong because they overvalue information and do not play devil’s advocate. These “two wrong answers” are summarized on the following page:

71

LESSON

2. A recipe for mixed nuts includes only whole peanuts and cashews and calls for a ratio of 7 ounces of peanuts for every 3 ounces of cashews.  How many ounces of peanuts are in a bag?

GovernmentAdda.com

72

3

SECTION 3: LEARNING TO PLAY THE GAME The Two Wrong Answers

The Two Wrong Answers 1. You think you don’t have enough information, but you do. Examples: •

You miss an inference that you should draw (for example, “x represents the number of children on a field trip”; you can’t have a negative or fractional number of children!).



One statement implies the other (the classic “C Trap,” which we will discuss in a few pages).

Your strategy: Leverage your assets. Get as much value out of each statement (plus question stem) as you can. 2. You think you have enough information, but you don’t. Examples: •

You assume something that isn’t explicitly given (that x is an integer, or that it’s positive).



You include statement (1) in your understanding of statement (2) (or vice versa).

Your strategy: Play devil’s advocate. Make sure you’re not missing something.

And that’s it. That’s the testmakers’ playbook against you. If you can correctly assess how much information you have, and how much is needed to answer the question, you will be correct each time. That is where the game is truly played. The testmakers will be clever about disguising the amount of information you’ve truly been given. But you have built-in advantages, too. In the pages that follow, you will learn how to play the game by taking advantage of the Data Sufficiency format and the hints that are given in that format.

73

LESSON

GovernmentAdda.com

Sufficiency Drill: How to Leverage Assets and Play Devil’s Advocate In the preceding sections, you completed several drills with only one statement. Did you notice that analyzing one statement is actually harder than analyzing two. Why? Because the other statement is often a hint or a piece of information that you can leverage to figure out what is tricky in the problem. Think about what you learned in Sentence Correction with “Decision Points”: The answer choices are your ally because you can leverage differences between the sentences (which are the hints in Sentence Correction). Remember how hard it was to simply analyze one sentence and decide whether it was correct? The same holds true in Data Sufficiency; it is much harder to decide in a vacuum if a statement is sufficient than when a second statement is present. To highlight this fact, please assess whether the individual statement is sufficient to answer the question that precedes it. After you have completed these, you will see clearly the advantage of having a second statement: 1. How many integers x exist such that a < x < b?

GovernmentAdda.com (1) b – a = 7

2. A nursery has 30 trees, carrying only pines, oaks, and maples. How many oaks does the nursery have? (1) The ratio of pines to maples is 14:9.

3. Is a > c ? b

(1) a > bc

74

SECTION 3: LEARNING TO PLAY THE GAME

3

Two Statements Are Better Than One

Two Statements Are Better Than One Now let’s add a statement (2) to each of the previous drill questions. Care to change any of your answers? To understand construct thinking you must first realize and accept that the statements are actually assets, not liabilities. 1. How many integers x exist such that a < x < b? (1) b – a = 7 (2) a and b are integers

2. A nursery has 30 trees, carrying only pines, oaks, and maples. How many oaks does the nursery have? (1) The ratio of pines to maples is 14:9. (2) The ratio of pines to oaks is 2:1.

3. Is a > c ? b

(1) a > bc (2) b > 0

Solutions to Sufficiency Drill: 1. C; 2. A; 3. C 75

LESSON

GovernmentAdda.com

LE AR N ING BY DO ING Leverage the Statements In the preceding section, you completed several drills with only one statement. Did you notice that analyzing one statement is actually harder than analyzing two? Why is that? Because the other statement is often a hint or a piece of information that you can leverage to figure out what is tricky in the problem. In many ways this is similar to Sentence Correction, in which you can use the differences between answer choices to alert you to grammatical problems that you might not have seen reading just one sentence (this is the basis for the Veritas Prep “Decision Points” strategy). Remember – Data Sufficiency statements are not created in a vacuum; the author of the question carefully crafts each statement as part of the entire problem, often using a statement to tempt you into a bad decision or to reward you for recognizing a clue. Therefore, if you can learn to recognize clues and traps embedded within “the other” statement, you have a competitive advantage on Data Sufficiency problems. Consider the first drill example. It’s not at all uncommon for someone to interpret the question and then test a few numbers: If b = 8 and a = 1, there are six integers in between (2, 3, 4, 5, 6, 7). Maybe try one as a negative to see if that reacts differently: b = 4, a = -3, and you still have six integers in between (-2, -1, 0, 1, 2, 3).

SKILLS MEET STR ATEGY Think of Statements as Hints

Perhaps the biggest takeaway in this section is the idea of thinking of statements as hints. Statements are assets, not liabilities, so you need to learn how to leverage information that is given or play devil’s advocate before you pick your final answer. One of the most important skills on all GMAT questions is your ability to find hints in the question and/or answer choices, and leverage those hints.

GovernmentAdda.com

But if you haven’t played your game of devil’s advocate well enough, statement (2) suggests that you need to consider “what if I didn’t know this information?” Well, if you don’t know that a and b are integers, then you have to account for nonintegers, and if b = 8.5 and a = 1.5, then there are seven integers (a different answer) in between: (2, 3, 4, 5, 6, 7, and 8). Here, statement (2) should clue you in to a consideration that you might not have made on your own. Answer choice C is correct. In the second example, you learn from statement (1) that the ratio of pines to maples is 14:9. Since it does not tell you anything about oaks, it can’t be sufficient, right? Remember that you were given a total in the question stem of 30 trees. If the ratio is 14:9 for pines to maples then the multiplier must be 1, as a higher multiplier would make the total over 30 trees. Since you have proved the multiplier is 1, then there must be 7 oaks, and statement (1) is sufficient without needing what is given in statement (2). The correct answer choice is A. For the third drill, it seems that statement (1) is a simple mirroring of the question and thus sufficient. But remember one of the most important rules for inequalities: You cannot divide or multiply by a variable unless you know the sign of that variable. Thus you need to know that b is either positive or negative so that you can manipulate the statement to match the question. Statement (2) gives you that information, so answer choice C is correct.

76

3

SECTION 3: LEARNING TO PLAY THE GAME Statement as Hints

Statement as Hints Consider one more example of how you can use statements to your advantage: 7.

What is the value of m + n? (1) jm + kn + nj + km = 36 (2) j + k = 12

(A) Statement (1) ALONE is sufficient, but statement (2) alone is not sufficient to answer the question asked (B) Statement (2) ALONE is sufficient, but statement (1) alone is not sufficient to answer the question asked (C) BOTH statements (1) and (2) TOGETHER are sufficient to answer the question asked, but NEITHER statement ALONE is sufficient

GovernmentAdda.com (E) Statements (1) and (2) TOGETHER are NOT sufficient to answer the question asked, and additional data are needed

77

LESSON

(D) EACH statement ALONE is sufficient to answer the question asked

LE AR N ING BY DO ING Take the Hint! If you were given statement (1) alone, it is clearly insufficient, and you would have no idea what to do with it. It is a whole bunch of variables with no clear organization. Statement (2) is also clearly insufficient as the question asks about m + n, and the statement tells you about j + k. For our purposes, though, it serves as a fantastic clue: If you can get the j and k terms together, you can replace them with a number, leaving just m and n. So that’s your catalyst to do some more algebraic manipulation: (1) jm + kn + nj + km = 36 Rearrange the order to get like terms together: jm + km + kn + nj = 36 Factor out the m and n: m(j + k) + n (k + j) = 36 and complete the factoring to show that: (m + n)(j + k) = 36

THINK LIKE THE TESTMAKER Selling the Wrong Answer

At first glance, it seems that you could never find values for m and n from the information given in the statements; there are just too many unknowns and not enough equations. But if you look closely, you realize that question is not asking for the value of an individual variable, but for the sum of two variables. This should be your first clue that you might be able to do it. As you learned earlier, the second clue is from statement (2), which gives a hint for how to factor statement (1). In Data Sufficiency, there are many tricks that testmakers can use to make it seem like you don’t have enough information, when you really do. Every time you see one of those tricks, you should file it away in your mind. On this problem, the trick is that when the question stem asks not for individual variables, but for a sum or a difference, you often need less information than you think.

GovernmentAdda.com

Using the information from statement (2), replace the (j + k) terms with 12: 12 (m + n) = 36 Divide by 12, and we have an answer: (m + n) = 3 If you don’t use statement (2) as a hint, you could easily miss this and pick answer choice E, as the testmakers want you to do. While it seems like there is not enough information to solve this, there really is, but you will only discover that if you leverage statement (2). Once you do, you will see that answer choice C is correct.

SKILLBUILDE R • Algebraic manipulation • Factoring

78

3

SECTION 3: LEARNING TO PLAY THE GAME Statement as Hints

Playing Chess, Not Checkers At this point you may ask: “But aren’t we supposed to only look at one statement at a time?” The simple answer is yes, as a common trap on these questions is that the GMAT gets you to use information before you really have it. The testmakers want to embed something about statement (1) in your mind that lingers when you look at statement (2). To truly become a master of Data Sufficiency, you need to be able to think in two parts: 1. I’m looking at this statement and only using the information that is specifically given in this statement and in the question stem. 2.  But, I can’t ignore that the testmakers have already shown me the other statement, so I should at least use it as a clue.

GovernmentAdda.com

On Data Sufficiency questions, the “other statement” is that move-then-retreat. It tells you something about what the testmakers are setting up. So while you’re falling for a beginner’s trap if you use the information in statement (1) when you assess statement (2) ALONE, you’re falling for almost as novice a trap if you wear blinders to each statement and fail to consider the question as a whole.

79

LESSON

In many ways, Data Sufficiency is a chess match between you and the testmakers.So consider this situation: You’re playing chess. Your opponent picks up her bishop, moves it five spaces across the board, and sets it on a particular square for a few seconds, letting her finger linger on the piece before deciding to move it back to where it began and select an entirely different move. True, after her move you have to play the game based on where the pieces actually do lie, but you’d be a poor player if you didn’t consider “what was she trying to set up with that move?” and “where did she see that she was vulnerable when she ultimately decided to retreat?”

GovernmentAdda.com

80

SECTION 3: LEARNING TO PLAY THE GAME

3

Learning To Play The Game Summary

Learning To Play The Game Summary The goal of this section has been to get you to focus on the following core strategies and thought processes. •

There are only two ways that you can make mistakes in Data Sufficiency: 1. You think you don’t have enough information, but you do. 2. You think you do have enough information, but you don’t. •

In the first case, the mistake is that you have not leveraged all the assets given in the statement and/or the question stem. Always maximize all the resources given.



In the second case, the mistake is that you overvalued the information that was given and you did not play “Devil’s Advocate” to find the exception or the misleading information. Always be skeptical of information, particularly when it seems too good to be true.



Be clear about the rules for Data Sufficiency. You cannot actually use the information from the other statement when assessing sufficiency in the first, but you can use that information as a guide for how to behave.



Playing the game properly and using all assets in the problem is the key to success in Data Sufficiency.

81

LESSON

GovernmentAdda.com In a nutshell, success in Data Sufficiency relies on a mix of these two core skills: leveraging your assets and playing devil’s advocate. Once you understand how to use those two skills, you must view the statements as hints. Remember: Statements give you important clues as to how to properly leverage information and how to play devil’s advocate.

GovernmentAdda.com

82

4

SECTION 4:THE POWER OF CONSTRUCT THINKING Why Are You Here?

SECTION 4: THE POWER OF CONSTRUCT THINKING In the previous section you learned how to the play the game by leveraging all your assets, playing devil’s advocate, and using the statements as hints. In the pages that follow, you will learn to recognize specific Data Sufficiency constructs that appear over and over. Once you unlock what these constructs mean, you will have an advantage in Data Sufficiency that cannot be overstated. The most important of the Data Sufficiency constructs hinges on asking this question:

Why Are You Here? This construct is present any time you are given a statement that is obviously insufficient. When this scenario presents itself, it is essential that you take notice of this statement and ask: “Why are you here?” In doing that, you should consider two possibilities: 1. The information is giving you something that you need in order to make the other statement sufficient. (In this case, answer choice C is correct.) In other words, the information is important and it is a hint to an assumption you may have made in the other statement.

GovernmentAdda.com OR

2. The information is something that you think you need, but you really don’t or you already know from the other statement. (In this case, answer choice A or B will be correct.) In other words, you are being tempted with information that you don’t need. Simply put, if a Data Sufficiency statement is clearly not sufficient, you need to ask: “Why are you here?” As an example of this construct, consider this textbook example from the previous drill problems: How many integers x exist such that a < x < b? (1) b – a = 7 (2) a and b are integers

83

LESSON



Clearly, statement (2) could never be sufficient by itself, so you should ask why that statement is there. Remember how easy it was in this drill to think that statement (1) was sufficient when that’s all you had? However, statement (2) is screaming loudly at you the assumption you probably made in the first. Remember: Statements are also hints, and to do well in Data Sufficiency you should always be trying to leverage those hints. Just remember when asking “Why are you here?” that you may need the information or you may not. What matters is that you are actively and consciously making that decision. The following are two different examples of how to leverage statements in this construct.

8.

Is x between 0 and 1? (1) x2 < x (2) x is positive.

GovernmentAdda.com

(A) Statement (1) ALONE is sufficient, but statement (2) alone is not sufficient to answer the question asked (B) Statement (2) ALONE is sufficient, but statement (1) alone is not sufficient to answer the question asked (C) BOTH statements (1) and (2) TOGETHER are sufficient to answer the question asked, but NEITHER statement ALONE is sufficient (D) EACH statement ALONE is sufficient to answer the question asked (E) Statements (1) and (2) TOGETHER are NOT sufficient to answer the question asked, and additional data are needed

84

4

SECTION 4:THE POWER OF CONSTRUCT THINKING Why Are You Here?

9. Set J consists of terms {2, 7, 12, 17, a}. Is a > 7? (1) a is the median of set J. (2) Set J does not have a mode.

(A) Statement (1) ALONE is sufficient, but statement (2) alone is not sufficient to answer the question asked (B) Statement (2) ALONE is sufficient, but statement (1) alone is not sufficient to answer the question asked (C) BOTH statements (1) and (2) TOGETHER are sufficient to answer the question asked, but NEITHER statement ALONE is sufficient (D) EACH statement ALONE is sufficient to answer the question asked (E) Statements (1) and (2) TOGETHER are NOT sufficient to answer the question asked, and additional data are needed

85

LESSON

GovernmentAdda.com

LE AR N ING BY DO ING Clearly Insufficient Statements Are There for a Reason In each of these problems, you are given pieces of information that could never be sufficient on their own. In example problem #8 it is statement (2): If x is positive, that does not tell you if it is between 0 and 1, so it is clearly insufficient. (In example problem #9, it is also statement (2): If set J does not have a mode, that does not tell you anything about a.) For each one you need to figure out why that information is there, and whether you need the information with other statement or not. In the first problem, the “Why Are You Here?” statement is clever; it’s a trap! Statement (1) already implies that x is positive. Any number squared (x2) will be greater than (or equal to) 0, so if x is negative then x2 will always be positive. Negative numbers simply do not satisfy statement (1), so we don’t need statement (2); we already knew that. Here, statement (2) is a trap; the authors of the GMAT know that you tend to feel more comfortable when you have more information. So remember: If one statement ALONE is sufficient, you don’t get to use both. The correct answer choice is A.

THINK LIKE THE TESTMAKER Selling the Wrong Answer If you missed one of these two questions, it was probably the first one. Why? Because the “Why Are You Here?” temptation construct (in which you are tempted with something you don’t need) is much harder psychologically than the traditional “Why Are You Here?” construct (in which you need the other information given). In other words, it is much harder to say no to something that fits so nicely with the other statement (but that you don’t need) than it is to say yes to something that you do need with the other statement. As a result, many of the harder “Why Are You Here” questions are tempting you with something you don’t need. Remember: “Why Are You Here” questions are mainly about isolating the statement that could never be sufficient on its own and figuring out why the testmakers have put it there. Sometimes you will need that information and sometimes you won’t.

GovernmentAdda.com

In #9, you learn in statement (1) that a is the median of the set, so then the set must look like this in order: {2, 7, a, 12, 17}. It would be very easy from this statement alone to think that it is sufficient. Doesn’t a have to be between 7 and 12 for it to be the middle term in this set? No. Remember that a could be any number between 7 and 12 inclusive. If a is 7, then it is still the median of the set, so you actually get a maybe answer to the question from statement (1) alone. (a could be greater than 7, but it does not have to be.) Remember: If you were about to make the mistake of picking answer choice A, you should ask: “Why are they telling me that set J has no mode?” That should help remind you that the median could be a repeating number and that you also need the information in statement (2). Answer choice C is correct.

SKILLBUILDE R • Statistics • Arithmetic number properties

86

4

SECTION 4:THE POWER OF CONSTRUCT THINKING Additional Constructs

Additional Constructs On the following pages you will learn several other important Data Sufficiency constructs. As you just saw, the “Why Are You Here?” construct relates primarily to two important strategies that we have emphasized throughout the lesson: •

Leverage assets.



Play devil’s advocate.

The remaining constructs still hinge on these two essential components, but they also relate much more to another important “mantra” that is essential on the GMAT (and in business!): “Don’t Be the Sucker.”

GovernmentAdda.com

In many respects, this is just another piece of information to leverage in the game of Data Sufficiency. To highlight this, consider the following construct problem: Last year, did company X produce more trucks than company Y? (1) Last year, company X produced 205,000 trucks and company Y produced 150,000 trucks. (2) ??????

87

LESSON

In business, if something seems too good to be true, it almost always is! In Data Sufficiency, if an answer choice seems obvious, it is almost certainly the incorrect “sucker” choice. Remember: The GMAT is a very challenging test that is trying to differentiate between highly intelligent and well-educated test-takers. Easy answer choices will only be correct well below the 50th percentile, so don’t pick them!

What must be true about statement (2) in this question? Statement (2), regardless of what it is, must be difficult, or this problem is well below the 20th percentile! Everyone taking the GMAT can recognize that Statement (1) is sufficient. So, if you look at statement (2) and it appears to be sufficient, it probably isn’t. If you look at statement (2) and it does not appear to be sufficient, it probably is. Using this thought process can’t give you the answer outright, but it will help you realize that you need to either play devil’s advocate on statement (2) or leverage some assets given in the question or the statement. If you simply look at statement (2) and accept it at face value, then you are “the sucker” because it is almost surely counterintuitive! On the pages that follow, questions will come first followed by a summary of the construct. As you try each question, try to isolate the sucker answer and then leverage that to find the correct answer. Remember: Construct thinking cannot give you the answer outright, but it can help you attack the problem efficiently and avoid cleverly created sucker choices.

GovernmentAdda.com

88

4

SECTION 4:THE POWER OF CONSTRUCT THINKING Additional Constructs

10. Julie opened a lemonade stand and sold lemonade in two different sizes, a 52-cent (12 oz.) size and a 58-cent (16 oz.) size. How many 52-cent (12 oz.) lemonade drinks did Julie sell? (1) Julie sold a total of 9 lemonades. (2) The total value of the lemonade drinks Julie sold was $4.92.

(A) Statement (1) ALONE is sufficient, but statement (2) alone is not sufficient to answer the question asked (B) Statement (2) ALONE is sufficient, but statement (1) alone is not sufficient to answer the question asked (C) BOTH statements (1) and (2) TOGETHER are sufficient to answer the question asked, but NEITHER statement ALONE is sufficient (D) EACH statement ALONE is sufficient to answer the question asked (E) Statements (1) and (2) TOGETHER are NOT sufficient to answer the question asked, and additional data are needed.

89

LESSON

GovernmentAdda.com

LE AR N ING BY DOING Beware of the C Trap Another common Data Sufficiency construct is one that baits you with an “easy C.” The testmakers know this about you: You like to make decisions using all available information, so if both statements TOGETHER are clearly sufficient but neither alone jumps out as obviously sufficient, you’re quite likely to take the bait. Remember, however, how answer choice C is written: “BOTH statements TOGETHER but neither statement ALONE.” If one statement will do on its own, you’re not allowed to use both. Perhaps Tupac Shakur said it best with his song title “U Can’t C Me” or his lyric from “I Get Around”: “I don’t want it if it’s that easy.” If a statement is too easy, don’t take it without further investigation. It’s likely that either: · One statement alone is sufficient if you rearrange the algebra or find a “hidden gem” of information embedded beneath the surface, OR · Both statements actually say the same thing in different ways (or one cleverly says nothing at all), and you actually don’t have two pieces of information.

GovernmentAdda.com

This problem follows the first construct. Here, again, you can use one statement to tell you something about the other. Which statement gives you more information? Currently you have: (1) x + y = 9 (2) 52x + 58y = 492 Statement (2) seems to contain a bit more information, as there are likely to be fewer combinations of 52x and 58y that will end in exactly 492. But you don’t have to test all the combinations! Look at what statement (1) is telling you: that there are exactly nine lemonades. With that piece of information and statement (2), you can easily solve the problem (two unique linear equations, two variables). Do you really need them to tell you that x + y = 9? Could it be anything else? Say you wanted there to be only eight lemonades sold. By reducing the number sold, to stay at the $4.92 revenue number, she would have to sell higher-priced items to compensate for the fewer units sold. Even eight of the most expensive lemonades (8 • 58 = 464) does not come close to the $4.92 figure, and any other combination of eight small and large lemonades will reduce the total revenue. So she cannot have sold eight or fewer. Could she have sold 10 or more?

90

4

SECTION 4:THE POWER OF CONSTRUCT THINKING Additional Constructs

Selling 10, she’d have to sell the cheaper items to have a chance at keeping the revenue at $4.92. Even 10 of the cheapest (10 • 52 = 520) are too many; she cannot sell 10 or more. So without statement (1) even having to tell us that x + y = 9, you can come up with statement (1) on our own using just the information in statement (2). Statement (2) alone is sufficient, so answer choice B is correct. Most importantly, recognize the strategy here: The testmakers already gave you the clue that x + y = 9, so you don’t have to create that information out of thin air. Statement (1) gave you a target; with just statement (2) by itself, with no other context, you’re not that likely to prove that it’s sufficient in two minutes or less. But with statement (1) there as a clue, you have a head start on what you should try to prove. That’s the Data Sufficiency “game.”

GovernmentAdda.com

The C trap is a great example of the mantra presented earlier: “Don’t be the sucker.” If an answer choice is handed to you on a gold platter, don’t take it. The beauty of the sucker choice is that it gives you a second chance. In many ways, it is logically identical to the hint in the “Why Are You Here?” construct. If you are about to pick answer choice C and it seems easy, you should ask yourself whether you can do it from one statement alone (you have not leveraged enough information) or whether you can’t do it at all (you have overvalued information and not played devil’s advocate). While the easy answer might be the correct answer, it rarely is and using the mantra of “don’t be the sucker” will help you to get many difficult problems correct.

SKILLBUILDE R • Algebraic manipulation • Mixture problems

91

LESSON

THINK LIKE THE TESTMAKER Don’t Be the Sucker

11. What is the value of x? (1) 6x + 4y = 30 (2) x = – 2 y + 5 3

(A) Statement (1) ALONE is sufficient, but statement (2) alone is not sufficient to answer the question asked (B) Statement (2) ALONE is sufficient, but statement (1) alone is not sufficient to answer the question asked (C) BOTH statements (1) and (2) TOGETHER are sufficient to answer the question asked, but NEITHER statement ALONE is sufficient (D) EACH statement ALONE is sufficient to answer the question asked (E) Statements (1) and (2) TOGETHER are NOT sufficient to answer the question asked, and additional data are needed

GovernmentAdda.com

92

4

SECTION 4:THE POWER OF CONSTRUCT THINKING Additional Constructs

12. The ratio of television sets to radios at an electronics store before a new shipment arrives is 12:7. If no television sets or radios leave the store, and the only television sets and radios that arrive are in the new shipment, what is the ratio of television sets to radios after the new shipment arrives? (1) The new shipment contains 132 television sets. (2) The new shipment contains 77 radios.

(A) Statement (1) ALONE is sufficient, but statement (2) alone is not sufficient to answer the question asked (B) Statement (2) ALONE is sufficient, but statement (1) alone is not sufficient to answer the question asked (C) BOTH statements (1) and (2) TOGETHER are sufficient to answer the question asked, but NEITHER statement ALONE is sufficient

(E) Statements (1) and (2) TOGETHER are NOT sufficient to answer the question asked, and additional data are needed

93

LESSON

GovernmentAdda.com (D) EACH statement ALONE is sufficient to answer the question asked

LE AR N ING BY DOING No Easy Es/Prove E In the previous two questions, you have seen examples of problems that come down to answer choice C vs. answer choice E. At first, you may have found problem #12 to be similar to the “lemonade problem” from the earlier example, in that the two statements each gave you a linear equation, leaving you with two equations/two variables. Here, however, statements (1) and (2) express the exact same relationship. Neither is sufficient alone, and neither adds new value to the other, so answer choice E is correct—with a “trap” of answer choice C. This further cements the point made earlier: When answer choices seem obvious, you need to investigate more closely. Example problem #12 demonstrates the inverse. In colloquial terms, make sure that you recognize that answer choice E does not mean “I can’t do it”; it means, more specifically, “It cannot be done.” Much like you should be leery of an “Easy C,” be just as wary of an “Easy E.” If you accept that Data Sufficiency is, above all else, a test of your ability to manage resources, you should recognize that the reward system is twofold. You should be rewarded for:

GovernmentAdda.com

· Cleverly extracting as much value as possible out of each piece of information, and · Effectively playing devil’s advocate when the information looks to point in one direction but has some particular limitation. Because of that, when a decision comes down to answer choice C vs. answer choice E, you must prove answer choice E in order to select it. Lackadaisically selecting answer choice E is fraught with error. The testmakers have every incentive to reward you for taking information that seemingly isn’t sufficient and for adeptly leveraging it to solve a problem. When answer choice E is correct, it is usually correct because, by selecting it, you have been able to demonstrate a keen ability to say: “Not so fast—this conclusion still has one potential flaw.” In example #11, that one potential flaw is that the two statements actually say the same exact thing, even though at first glance they are totally different. In example #12, all signs point to “not sufficient.” Clearly neither is sufficient on its own, and even together you are given an initial ratio and then two hard numbers, and then you are asked to comment on the new ratio. This is almost always impossible. With the initial ratio you don’t know whether the store has 12 TVs or 1,200 TVs, so gauging the impact of adding 132 of them on the ratio is impossible. But that’s where your ability to play the GMAT game comes into play. Answer choice E is the obvious choice here, but

94

4

SECTION 4:THE POWER OF CONSTRUCT THINKING Additional Constructs

further investigation shows that: The initial ratio is 12:7. You add 132, or 12(11) TVs, and 77, or 7(11) radios. If the initial number of units is 12x TVs + 7x radios, and you add 12(11) TVs and 7(11) radios, then you’re adding: 12x + 12(11) TVs + 7x + 7(11) radios You can factor the 12s and 7s to get: 12(x+11) TVs and 7(x+11) radios The ratio is still 12:7.

GovernmentAdda.com

SKILLS ME ET STR ATEGY Think Counter-Intuitively As you have seen in this lesson, Data Sufficiency is about resource management. The problems are difficult because you often think you have enough information, but you don’t, and vice versa. Since that is the game, it is incredibly helpful to think counterintuitively and, in a phrase, be skeptical. This is particularly true in questions that come down to answer choice C or answer choice E. If everything in the question seems to point to answer choice E, there is a good chance that it is answer choice C, and you should try to come up with a way to prove sufficiency and show you can do it with both. If it feels like answer choice C, then it’s probably answer choice E, and you should try to come up with the exception or the information that would prove answer choice E. While of course you can get too paranoid using this thought process, it is essential to success on harder Data Sufficiency problems.

SKILLBUILDE R • Algebraic manipulation • Ratios

95

LESSON

Is that a clear decision on the surface? Probably not. For most people, 132 isn’t an obvious multiple of 12 (or of 11). But knowing that answer choice E is a little too easy here, and knowing that when answer choice E is correct there’s usually a moment of “a-ha, that’s why it can’t be solved” that is missing here, you know to play with the information for just a few more seconds before reluctantly accepting answer choice E. That extra push toward using the information more efficiently should direct you to the correct answer: answer choice C.

13. For isosceles triangle LMN, what is the measure of angle N? (1) The lengths of all three sides of triangle LMN are the same. (2) Angle L measures 60 degrees.

(A) Statement (1) ALONE is sufficient, but statement (2) alone is not sufficient to answer the question asked (B) Statement (2) ALONE is sufficient, but statement (1) alone is not sufficient to answer the question asked (C) BOTH statements (1) and (2) TOGETHER are sufficient to answer the question asked, but NEITHER statement ALONE is sufficient (D) EACH statement ALONE is sufficient to answer the question asked (E) Statements (1) and (2) TOGETHER are NOT sufficient to answer the question asked, and additional data are needed

GovernmentAdda.com

96

4

SECTION 4:THE POWER OF CONSTRUCT THINKING Additional Constructs

14. A street  vendor sells only apples and pineapples, and all apples weigh 6.5 ounces, while all pineapples weigh 13 ounces. If she sells twice as many apples as pineapples, how many apples does she sell? (1) She sells 8 more apples than pineapples. (2) She sells an equal amount, by weight, of apples and pineapples.

(A) Statement (1) ALONE is sufficient, but statement (2) alone is not sufficient to answer the question asked (B) Statement (2) ALONE is sufficient, but statement (1) alone is not sufficient to answer the question asked (C) BOTH statements (1) and (2) TOGETHER are sufficient to answer the question asked, but NEITHER statement ALONE is sufficient (D) EACH statement ALONE is sufficient to answer the question asked (E) Statements (1) and (2) TOGETHER are NOT sufficient to answer the question asked, and additional data are needed

97

LESSON

GovernmentAdda.com

LE AR N ING BY DOING Dealing with Obviously Sufficient Statements With the “Why Are You Here?” construct, you learned how to deal with statements that are obviously insufficient; they are hints for the other statement, and you must decide if the statement is important or not. What about problems in which there is a statement that is clearly sufficient—the statement is easy and everyone knows it? As you learned in the introduction to these constructs, it must mean that the other statement is tricky or the problem would be too easy. You can use this fact to your advantage. The other statement must be counter-intuitive, so realize that and attack the problem accordingly. In example problem #13, statement (1) is clearly sufficient. If all sides are the same, then the triangle is equilateral and all angles must be 60 degrees. The second is a bit counter-intuitive to many, though. Because it only gives the measure of one angle, you might think that it’s not sufficient, as you don’t know whether angle L “matches” angle N, making N 60 degrees also, or whether L is the non-matching side, making N = M when we don’t know about either M or N. But wait! Because the given angle is 60, it doesn’t matter. Either:

GovernmentAdda.com L = N = 60

L = M = 60, so L + M = 120 and N is responsible for the remaining part of 180 (which is 60). OR L = 60 and M = N, and M + N = 120 (the portion of 180 not represented by L). In that case, N = 60, as well. So statement (2), although it seems a bit light on information, actually does guarantee that N = 60. Your weapon here is to recognize the construct: Statement (1) required very little work, if any, so statement (2) is where the difficulty simply has to lie. Knowing that should urge you to work a little harder to find a bit of ingenuity or difficulty in statement (2). With that in mind you should derive the above and correctly come to answer choice D. In problem #14, the word problem format begs you to set up equations. The given information states that, of the fruit that she sold, A = 2P. Statement (1) should again be pretty straightforward: If she sells 8 more apples than pineapples, then A = 8 + P. Using both equations together, we can plug in 2P for A to get 2P = 8 + P, and solve for P. Statement (2) also seems to offer the same type of information—a second equation to pair with the given information that A = 2P. But wait! Your senses should be heightened for the counter-intuitive statement (2) now that statement (1) has proven to be a bit

98

4

SECTION 4:THE POWER OF CONSTRUCT THINKING What Makes Data Sufficiency So Hard?

too easy. Statement (2), as an equation, is that 6.5A = 13P. Divide by 6.5, and you’ll find that A = 2P—the same equation that we already have! Statement (2) offers no new information and is therefore not sufficient (making answer choice A correct here). Having been on guard for a sneaky other statement after a straightforward first statement, you should be looking for that clever restatement of already-known information and avoid this trap. For ease of teaching, the recent examples used the construct “Statement (1) is obvious; (2) is counterintuitive.” There’s nothing magic about statement (1), and the GMAT could well feature the statements of any of these questions in opposite order, baiting you into handling the easier statement (2) first and then being caught unaware on statement (1). You will see examples of this in the homework to follow; just know that when one statement is a little too easy, there’s a high likelihood that the other has some sneaky difficulty built into it that you should anticipate. NOTE:

SKILLS MEET STR ATEGY Leverage Assets in the Question Stem

GovernmentAdda.com

THINK LIKE THE TESTMAKER Misdirection

In example problem #14, you saw a common trick used by testmakers: provide something that seems important but that you already know. This trick is present in many different GMAT question types (remember those Critical Reasoning Strengthen questions in which an answer choice is just a clever rewording of a premise that already exists in the argument?) and is particularly important in Data Sufficiency. If one of the statements simply repeats information already known from the question stem, it is not sufficient. Also, if a statement gives you something that you already know from the other statement, the answer will never be answer choice C. Keep your eyes open for statements that are cleverly repeating information that is already given.

SKILLBUILDE R • Triangles • Mixture problems • Algebraic manipulation

99

LESSON

As mentioned previously in this lesson, it is essential that you leverage all the information in the question stem. One of the favorite tricks used by testmakers is to hide important information there, because people tend to forget about it. If you missed example problem #13, it is almost surely because you did not leverage the fact that the triangle is given as isosceles in the question stem. In #14, you need to realize that one of the statements is simply repeating information that is already given, so it cannot be sufficient. In either case, success relies on reading the question carefully and leveraging every piece of information given there.

Understanding the Reward System At this point you have learned to recognize the important constructs, which can all be derived from a look at this chart: D

A



Each statement alone is sufficient. B

Only one statement alone is sufficient.



C

Neither statement alone is sufficient, but both together are.



E

No combination of statements is sufficient.

If you look a level above and below the “obvious” answer choice, you will probably see the construct that the GMAT author is using. For example: “Why Are You Here?”

It looks like A or B, but it’s actually C.

“Why Are You Here?—Temptation”

It looks like C, but it’s really A or B.

“The C Trap”

It looks like C, but it’s really A or B.

“…the Other Is Counterintuitive”

It looks like A or B, but is really D.



It looks like D, but is really A or B.

Prove E

It looks like E, but it’s really C.

No Easy Cs

It looks like C, but it’s really E.

GovernmentAdda.com Regardless of the construct, the most important takeaways are these: Before you pick D, you better make sure it’s not A or B. Before you pick A, you better make sure it’s not D or C. Before you pick B, you better make sure it’s not D or C. Before you pick C, you better make sure it’s not A/B or E. Before you pick E, you better make sure it’s not C.

These considerations represent the essential building blocks of construct thinking: If you ask yourself those questions every time and leverage the hints given in the construct, your improvement in Data Sufficiency will be dramatic.

100

4

SECTION 4:THE POWER OF CONSTRUCT THINKING The Power of Construct Thinking Summary

What Makes Data Sufficiency So Hard? 15. a, b, c, and d are consecutive integers such that the product abcd = 5,040.  What is the value of d? (1) d is prime (2) a > b > c > d

(A) Statement (1) ALONE is sufficient, but statement (2) alone is not sufficient to answer the question asked (B) Statement (2) ALONE is sufficient, but statement (1) alone is not sufficient to answer the question asked (C) BOTH statements (1) and (2) TOGETHER are sufficient to answer the question asked, but NEITHER statement ALONE is sufficient

GovernmentAdda.com (E) Statements (1) and (2) TOGETHER are NOT sufficient to answer the question asked, and additional data are needed

101

LESSON

(D) EACH statement ALONE is sufficient to answer the question asked

LE AR N ING BY DO ING Assumptions and the Power of Suggestion This problem is hard. Interestingly, it comes down mainly to a simple assumption that you have learned to avoid throughout this lesson: Don’t assume that numbers are positive. So how did you get baited into that on this question? The power of suggestion! Data Sufficiency authors are masters of human psychology; they know that your mind wants to organize information by archiving things you already know, so they know that they can often embed a piece of information deep enough in your psyche that you’ll carry it with you even if it’s not something you’re allowed to use. In the question stem, it takes some work to unravel the factors of 5,040, but once you do statement (1) works beautifully. 5,040 should jump out at you with two pretty-clear factors: It’s definitely divisible by 10 (it ends in 0), and it’s definitely divisible by 9 (the sum of the digits is 9). If you take out 9 and 10, you have 9 • 10 • 56, leaving you with the consecutive factors 7, 8, 9, and 10. Since 7 is the only prime in that set, statement (1) is sufficient. Now, note your assumptions when you attack statement (2). You’ve already proven 7, 8, 9, and 10, and probably realized with a bit of trial and error or conceptual understanding that you can’t come up with any other (positive) sets of consecutive factors. If you “repackage” 10 as a 2 and a 5, you cannot reallocate those without making one of the other factors much larger and another much smaller (say, 7, 8, 18, and 5). So knowing that 7 is the smallest, you might quickly determine that statement (2) is sufficient as well. But wait! There is one more set of consecutive integers a, b, c, and d that multiply to 5,040: -7, -8, -9, and -10. The problem does not rule out negatives! By asking about only primes in statement 1, the question embedded some lingering assumptions in your mind. It likely got you thinking only in terms of positive numbers and tricked you into taking your eye off the ball. The correct answer is A, but the authors know that many savvy test-takers will fall for trap answer D.

THINK LIKE THE TESTMAKER Misdirection There is one other thing that makes this problem hard: Incorrect answer choice D does not feel like a sucker choice. People who are good at game theory and higher-order thinking will only be satisfied if they feel they have done enough work to expose a sucker choice and see that the problem is hard. Because you have to do some clever factoring and a fair bit of work to pick answer choice D, you are satisfied that many other people could not do that and that you have a gotten a difficult problem correct. Alas, there is still one more trick lurking, and if you are not truly the “devil’s advocate,” it is unlikely you will notice that mistake.

GovernmentAdda.com

Beware the power of suggestion—and the best way to do this to play devil’s advocate. Only use the information that you know you have. For everything else you should push the boundaries. Ask “Am I allowed to…?” while trying different concepts or numbers. If you don’t find an explicit “no,” then err on the side of “yes.” SKILLBUILDE R • Prime numbers • Factors

102

5

SECTION 5: YOU OUGHTA KNOW Don’t Contradict Yourself

The Power of Construct Thinking Summary Construct thinking itself will not get you to the top of the pyramid. But without it, you are not leveraging one of the most important assets in Data Sufficiency. In analyzing Data Sufficiency constructs you should always consider the following: •

Construct thinking is based mainly in game theory and in understanding how testmakers can make problems hard. At its core, construct thinking helps you figure you out how and when to leverage information and play devil’s advocate. It also teaches you how to recognize and avoid sucker choices.



Perhaps the most important construct is what we call “Why Are You Here?” This applies any time there is an obviously insufficient statement. Whenever this construct presents itself there are two possibilities: 1. The information is giving you something that you need in order to make the other statement sufficient, OR

2. The information is something that you think you need, but you really don’t.



The “C trap” is another common construct and an important example of “Don’t Be the Sucker.” Whenever an answer choice is obvious, you should look to either leverage more information or play devil’s advocate. If the two statements together are obviously sufficient, you should be suspicious of answer choice C and look to answer choice A, B, or E as the likely correct answer.



Choices between C and E are particularly hard on the GMAT. Why? Because there are no more hints to leverage from the other statement. Your best hope in choosing between answer choices C and E on hard problems is construct thinking. Be wary of ever picking answer choice E when it just seems like the information is not sufficient, but you have not proven that is not sufficient. And if it seems to be answer choice C, you should be playing devil’s advocate by trying to prove answer choice E.

103

LESSON

GovernmentAdda.com



Any time one of the two statements is easily sufficient, you are being given a very helpful hint: The other statement must be difficult. It is almost surely counter-intuitive, so you should automatically be skeptical of how the statement appears at face value. If it appears sufficient, then it probably is not and you have not played “devil’s advocate” well enough. If it does not appear sufficient, then it probably is and you have not leveraged your assets well enough.



Regardless of whether you have found one of the common constructs, make sure you understand the reward system. Every answer in Data Sufficiency has a trap on one side (for answer choices D and E) or on both sides (for answer choices A, B, and C). Always check to see that you are not falling for one of those traps before you finalize your answer.

GovernmentAdda.com

104

5

SECTION 5: YOU OUGHTA KNOW No News Is Good News

SECTION 5: YOU OUGHTA KNOW In the following pages, you will see a few more Data Sufficiency structures and rules that, while not core to effective GMAT strategy, will likely prove helpful in your mastery of the discipline and could show up in one of your test questions.

Don’t Contradict Yourself In a Data Sufficiency question, the two statements are factual premises that must be true. Because of this fact, they cannot directly contradict each other, or the entire construct of Data Sufficiency would be illogical. In other words, statement (1) cannot say that x > 5 while statement (2) says that x < 3. These statements cannot co-exist or the question would be flawed. How can you use this to your advantage?

GovernmentAdda.com

16. For non-negative integers x, y, and z, is x z odd? (1) The product x z is odd. (2) x = 2y

(A) Statement (1) ALONE is sufficient, but statement (2) alone is not sufficient to answer the question asked (B) Statement (2) ALONE is sufficient, but statement (1) alone is not sufficient to answer the question asked (C) BOTH statements (1) and (2) TOGETHER are sufficient to answer the question asked, but NEITHER statement ALONE is sufficient (D) EACH statement ALONE is sufficient to answer the question asked (E) Statements (1) and (2) TOGETHER are NOT sufficient to answer the question asked, and additional data are needed

105

LESSON

Very carefully. Remember that you cannot use information from one statement while examining the other statement individually. But as we’ve discussed previously, you can use clues from one statement to save yourself from bad decisions on the other. If you are completely sure that statement (1) is giving you a yes answer to the question at hand, it would be impossible for statement (2) to give a definitive answer of no. Statement (2) could be ambiguous (an answer of maybe) or give a definitive answer of yes, but it cannot directly contradict the first. Consider a problem:

LE AR N ING BY DOING Statements Must Be True In this problem, you hopefully found statement (1) to be sufficient, without too much effort. If the product xz is odd, then both integers must be odd. And if x, the base of xz, is odd, then when raised to any integer it will be odd. Statement (1) guarantees that xz is odd. But statement (2) seems to guarantee that x is even, as 2 to any exponent is even. Right? There’s one glaring exception: 20 = 1—an important property for many exponent problems. So while most potential values of x, given statement (2), are even, one does exist where it’s odd. Statement (2) is not sufficient, and the correct answer is A. Here’s where you can use some higher-level strategy. If you were certain (as you should be) that statement (1) is sufficient with the answer “odd,” then as soon as you see a potential “even” with statement (2) you know that statement (2) is not sufficient! Why? Because the rules of the game dictate that if statement (1) gives you the answer “x must be odd,” then statement (2) cannot say “x must be even.” The only options are “x must be odd” or “x could be either odd or even.” Once you’ve found that “even” answer, then you know the “maybe” part must be coming, based on the rules of the game.

GovernmentAdda.com

SKILLBUILDER • Arithmetic number properties • Exponents

106

No News Is Good News If the two statements in a Data Sufficiency problem provide the same information in a different form, or if algebraic transformations show that one of the statements equals the other, then by the rules of the game, the correct answer must be either answer choice D or E. In other words, if you discover that each statement supplies the same information, there are only two possible scenarios: Each statement alone is sufficient, or both statements taken together are insufficient. After all, if statement (1) is the same as statement (2), then there’s no added value in using them together, and one cannot be sufficient without the other also being sufficient. Consider this example to see how you can use this concept to your advantage: 17. All attendees at a university gathering are faculty or alumni of the university. Are any of the attendees both faculty and alumni? (1)   3 of the attendees are members of the university faculty. 5

(2) 40% of the attendees are not members of the university faculty.

(A) Statement (1) ALONE is sufficient, but statement (2) alone is not sufficient to answer the question asked (B) Statement (2) ALONE is sufficient, but statement (1) alone is not sufficient to answer the question asked (C) BOTH statements (1) and (2) TOGETHER are sufficient to answer the question asked, but NEITHER statement ALONE is sufficient (D) EACH statement ALONE is sufficient to answer the question asked (E) Statements (1) and (2) TOGETHER are NOT sufficient to answer the question asked, and additional data are needed

107

LESSON

GovernmentAdda.com

LE AR N ING BY DOING Precision in Wording with Overlapping Sets In the Skills Meet Strategy box at the right, you’ll see that many test-takers quickly pick answer choice C, but the construct of the problem should warn you that C may be a trap. Two techniques from the Veritas Prep Word Problems lesson can help you conquer this problem and avoid the trap altogether. Venn Diagram If you’re using a Venn Diagram for this problem, remember that the visual representation of the diagram includes the entire circle for “member of the group” and the crescent shape for “member of this group only”. In this case, statement (1) tells you that 60% of the attendees are members of the faculty group, and statement (2) tells you that 40% of the attendees are not. Because there is no “neither” group, this means that those 40% must be in the “alumni only” group. And if you’re keeping disciplined accounting with the Venn Diagram, you’ll put the 60% Faculty number above the “Group A” circle, and the 40% Alumni Only number inside the “Group B” circle

GovernmentAdda.com FACULTY 60%

ALUMNI

40%

Notice that this does not allow you to determine the amount of the overlap “both” group using either of the Venn formulas: Group A + Group B – Both + Neither = Total In this formula, you know that Group A = 60, Neither = 0, and Total = 100. But you don’t have either Group B or Both alone: all you have is that (Group B – Both) = 40, but that doesn’t allow you to isolate either variable. Group A Only + Group B Only + Both + Neither = Total Here, again, you’re left with two variables. You have Group B Only = 40, Neither = 0 and Total = 100, but Group A Only and Both are both variables, and therefore you cannot solve for either. That is why the answer is E.

108

Matrix Box Because this situation allows for exactly two categories, each with exactly two options (Alumni and Not Alumni; Faculty and Not Faculty), you could use a Matrix Box to plug in the values from the statements to see if you can solve for the “Both” box (Alumni and Faculty):

Faculty

Not Faculty

Total

Alumni ?? Not Alumni Total 100%

Using the information from both statements together, you have:

Faculty

Not Faculty

Total

Alumni ??

SKILLS MEET STR ATEGY Using Identical Statements as Construct Thinking Upon initial inspection, many will look at this problem, 3 see that = 60%, and then 5 notice that 60% and 40% add up to 100%. Therefore, in their minds, 40% of the attendees are alumni, 60% are faculty, and there is no overlap. This looks like a pretty quick “C” answer, which means you need to investigate further to avoid what might well be a trap.

GovernmentAdda.com

Not Alumni Total

60% 40% 100%

Notice that your goal is to find the number in the “Faculty and Alumni” box at the upper left, but given the information from either statement you cannot plug in for any of the alumni boxes, so you cannot solve for that box and therefore the answer must be E.

So your two clues that C is a trap are: 1) C came a little too easy, and that’s usually a warning sign. And 2) the statements give the exact same information, meaning the answer must be either D or E. And since neither statement alone is sufficient – neither tells you anything about the alumni as a percentage of the total – the answer must be E.

109

LESSON

The fact that the two statements are identical should also be a clue. Think about it: if, as statement (1) states, 60% of the attendees are faculty, then of course 40% are not faculty. So statement (2) doesn’t actually add any new information. Note that it does not say “40% of the attendees are alumni” but rather just “40% are not faculty.” We already know that!

GovernmentAdda.com

110

HOMEWORK

HOMEWORK Learning by Doing: The strategies outlined in this lesson take practice, and like any “game” Data Sufficiency more than any other question type requires repetition so that you can get a feel for the Decision Tree, the Data Sufficiency Toolkit, the Reward System, and Construct Thinking.  The next 100+ pages include practice problems arranged generally by level of difficulty.  The first 25-30 problems are a great opportunity for you to become more comfortable with the Decision Tree and Toolkit.  By the last 40-50 problems, you will frequently need to also leverage Construct Thinking and the Reward System.  And should you want additional practice, visit the Veritas Prep Question Bank, a free online resource containing hundreds of practice problems arranged by question type. 18. Currently, there are 40 cars in each row of the lot at a car dealership. If the parking spaces are to be widened and lengthened so that only 30 cars fit in each row and fewer rows fit in the lot, how many cars will then fit in the entire lot?

GovernmentAdda.com (1) There will be 3 fewer rows of cars.

(2) Currently there are 10 rows of cars.

HOMEWORK

(A) Statement (1) ALONE is sufficient, but statement (2) alone is not sufficient to answer the question asked (B) Statement (2) ALONE is sufficient, but statement (1) alone is not sufficient to answer the question asked (C) BOTH statements (1) and (2) TOGETHER are sufficient to answer the question asked, but NEITHER statement ALONE is sufficient (D) EACH statement ALONE is sufficient to answer the question asked (E) Statements (1) and (2) TOGETHER are NOT sufficient to answer the question asked, and additional data are needed

111

19. Is William taller than Jane? (1) William is taller than Anna. (2) Anna is not as tall as Jane.

(A) Statement (1) ALONE is sufficient, but statement (2) alone is not sufficient to answer the question asked (B) Statement (2) ALONE is sufficient, but statement (1) alone is not sufficient to answer the question asked (C) BOTH statements (1) and (2) TOGETHER are sufficient to answer the question asked, but NEITHER statement ALONE is sufficient (D) EACH statement ALONE is sufficient to answer the question asked (E) Statements (1) and (2) TOGETHER are NOT sufficient to answer the question asked, and additional data are needed

GovernmentAdda.com

112

HOMEWORK

20. Is 11x < 7x ? 23 13 (1) x is an integer. (2) x > 0

(A) Statement (1) ALONE is sufficient, but statement (2) alone is not sufficient to answer the question asked (B) Statement (2) ALONE is sufficient, but statement (1) alone is not sufficient to answer the question asked (C) BOTH statements (1) and (2) TOGETHER are sufficient to answer the question asked, but NEITHER statement ALONE is sufficient (D) EACH statement ALONE is sufficient to answer the question asked (E) Statements (1) and (2) TOGETHER are NOT sufficient to answer the question asked, and additional data are needed

GovernmentAdda.com HOMEWORK

113

21. How many years old will Fred be y years from now? (1) Doris is 12 years older than Fred. (2) The sum of the ages of Doris and Fred is y years.

(A) Statement (1) ALONE is sufficient, but statement (2) alone is not sufficient to answer the question asked (B) Statement (2) ALONE is sufficient, but statement (1) alone is not sufficient to answer the question asked (C) BOTH statements (1) and (2) TOGETHER are sufficient to answer the question asked, but NEITHER statement ALONE is sufficient (D) EACH statement ALONE is sufficient to answer the question asked (E) Statements (1) and (2) TOGETHER are NOT sufficient to answer the question asked, and additional data are needed

GovernmentAdda.com

114

HOMEWORK

22. What is the value of 4x2 – 3x + 1? (1) (x – 1)(x + 2) = 0 (2) x = 1

(A) Statement (1) ALONE is sufficient, but statement (2) alone is not sufficient to answer the question asked (B) Statement (2) ALONE is sufficient, but statement (1) alone is not sufficient to answer the question asked (C) BOTH statements (1) and (2) TOGETHER are sufficient to answer the question asked, but NEITHER statement ALONE is sufficient (D) EACH statement ALONE is sufficient to answer the question asked (E) Statements (1) and (2) TOGETHER are NOT sufficient to answer the question asked, and additional data are needed

GovernmentAdda.com HOMEWORK

115

23. What was the final score in a game between Team X and Team Y? (1) Team X scored 14 more points during the second half of the game than it scored during the first half. (2) At the end of the first half of the game, the score was tied, but team Y won the game by 4 points.

(A) Statement (1) ALONE is sufficient, but statement (2) alone is not sufficient to answer the question asked (B) Statement (2) ALONE is sufficient, but statement (1) alone is not sufficient to answer the question asked (C) BOTH statements (1) and (2) TOGETHER are sufficient to answer the question asked, but NEITHER statement ALONE is sufficient (D) EACH statement ALONE is sufficient to answer the question asked

GovernmentAdda.com

(E) Statements (1) and (2) TOGETHER are NOT sufficient to answer the question asked, and additional data are needed

116

HOMEWORK

24. If ŧ is an operation, is the value of b ŧ c greater than 10? (1) x ŧ y = x2 + y2 for all x and y (2) b = 2 and c = 1

(A) Statement (1) ALONE is sufficient, but statement (2) alone is not sufficient to answer the question asked (B) Statement (2) ALONE is sufficient, but statement (1) alone is not sufficient to answer the question asked (C) BOTH statements (1) and (2) TOGETHER are sufficient to answer the question asked, but NEITHER statement ALONE is sufficient (D) EACH statement ALONE is sufficient to answer the question asked (E) Statements (1) and (2) TOGETHER are NOT sufficient to answer the question asked, and additional data are needed

GovernmentAdda.com HOMEWORK

117

B

A

C

D

25. In parallelogram ABCD above, what is the measure of angle ADC?

(1) The measure of angle ADC is greater than 90 degrees. (2) The measure of angle BCD is 70 degrees.

(A) Statement (1) ALONE is sufficient, but statement (2) alone is not sufficient to answer the question asked

GovernmentAdda.com

(B) Statement (2) ALONE is sufficient, but statement (1) alone is not sufficient to answer the question asked (C) BOTH statements (1) and (2) TOGETHER are sufficient to answer the question asked, but NEITHER statement ALONE is sufficient (D) EACH statement ALONE is sufficient to answer the question asked (E) Statements (1) and (2) TOGETHER are NOT sufficient to answer the question asked, and additional data are needed

118

HOMEWORK

26. Is the area of circle X greater than the area of circle Y?

(1) The circumference of circle X is greater than the circumference of circle Z, and the circumference of circle Z is less than the circumference of circle Y. (2) The radius of circle X is greater than the radius of circle Y.

(A) Statement (1) ALONE is sufficient, but statement (2) alone is not sufficient to answer the question asked (B) Statement (2) ALONE is sufficient, but statement (1) alone is not sufficient to answer the question asked (C) BOTH statements (1) and (2) TOGETHER are sufficient to answer the question asked, but NEITHER statement ALONE is sufficient (D) EACH statement ALONE is sufficient to answer the question asked;

GovernmentAdda.com (E) Statements (1) and (2) TOGETHER are NOT sufficient to answer the question asked, and additional data are needed

HOMEWORK

119

45°

J 45°

K

L

27. In triangle JKL shown above, what is the length of segment JL? — (1) JK = 2√ 2 (2) KL = 2

(A) Statement (1) ALONE is sufficient, but statement (2) alone is not sufficient to answer the question asked (B) Statement (2) ALONE is sufficient, but statement (1) alone is not sufficient to answer the question asked

GovernmentAdda.com

(C) BOTH statements (1) and (2) TOGETHER are sufficient to answer the question asked, but NEITHER statement ALONE is sufficient (D) EACH statement ALONE is sufficient to answer the question asked (E) Statements (1) and (2) TOGETHER are NOT sufficient to answer the question asked, and additional data are needed

120

HOMEWORK

28. Maria wants to send a package overseas. She can either pay shipping company A an amount dependent on the distance of the delivery, or pay shipping company B an amount dependent on the weight of the package. Which option is less expensive? (1) Company A charges $3.00 plus $0.01 per mile of shipment, and company B charges $5.00 plus $8.50 per pound. (2) The person who will receive the packages lives 3,900 miles away from Maria.

(A) Statement (1) ALONE is sufficient, but statement (2) alone is not sufficient to answer the question asked (B) Statement (2) ALONE is sufficient, but statement (1) alone is not sufficient to answer the question asked (C) BOTH statements (1) and (2) TOGETHER are sufficient to answer the question asked, but NEITHER statement ALONE is sufficient

GovernmentAdda.com (D) EACH statement ALONE is sufficient to answer the question asked

(E) Statements (1) and (2) TOGETHER are NOT sufficient to answer the question asked, and additional data are needed

HOMEWORK

121

29. A stenographer is tasked with typing a certain letter. How many words long is the letter? (1) It will take two minutes less time to type the letter at an average speed of 80 words per minute than at an average speed of 60 words per minute. (2) It will take 6 minutes to type the first half of the letter at an average speed of 40 words per minute.

(A) Statement (1) ALONE is sufficient, but statement (2) alone is not sufficient to answer the question asked (B) Statement (2) ALONE is sufficient, but statement (1) alone is not sufficient to answer the question asked (C) BOTH statements (1) and (2) TOGETHER are sufficient to answer the question asked, but NEITHER statement ALONE is sufficient

GovernmentAdda.com

(D) EACH statement ALONE is sufficient to answer the question asked (E) Statements (1) and (2) TOGETHER are NOT sufficient to answer the question asked, and additional data are needed

122

HOMEWORK

30. Is the positive square root of a an integer? (1) a = b4 and b is an integer (2) a = 81

(A) Statement (1) ALONE is sufficient, but statement (2) alone is not sufficient to answer the question asked (B) Statement (2) ALONE is sufficient, but statement (1) alone is not sufficient to answer the question asked (C) BOTH statements (1) and (2) TOGETHER are sufficient to answer the question asked, but NEITHER statement ALONE is sufficient (D) EACH statement ALONE is sufficient to answer the question asked (E) Statements (1) and (2) TOGETHER are NOT sufficient to answer the question asked, and additional data are needed

GovernmentAdda.com HOMEWORK

123

31. A certain jewelry store sells gold necklaces in 18-inch and 28-inch lengths, and all necklaces of the same length sell for the same price per necklace regardless of the number of necklaces purchased. What is the price of a 28-inch necklace at this jewelry store? (1) The total price of an 18-inch and a 28-inch gold necklace is $68. (2) The total price of two 18-inch necklaces and one 28-inch necklace is $96.

(A) Statement (1) ALONE is sufficient, but statement (2) alone is not sufficient to answer the question asked (B) Statement (2) ALONE is sufficient, but statement (1) alone is not sufficient to answer the question asked (C) BOTH statements (1) and (2) TOGETHER are sufficient to answer the question asked, but NEITHER statement ALONE is sufficient

GovernmentAdda.com

(D) EACH statement ALONE is sufficient to answer the question asked (E) Statements (1) and (2) TOGETHER are NOT sufficient to answer the question asked, and additional data are needed

124

HOMEWORK

32. Is x > y? (1) x = y + 2 (2) x = y – 1 2

(A) Statement (1) ALONE is sufficient, but statement (2) alone is not sufficient to answer the question asked (B) Statement (2) ALONE is sufficient, but statement (1) alone is not sufficient to answer the question asked (C) BOTH statements (1) and (2) TOGETHER are sufficient to answer the question asked, but NEITHER statement ALONE is sufficient (D) EACH statement ALONE is sufficient to answer the question asked (E) Statements (1) and (2) TOGETHER are NOT sufficient to answer the question asked, and additional data are needed

GovernmentAdda.com HOMEWORK

125

33. Is the average (arithmetic mean) of a and b less than 40? (1) The average (arithmetic mean) of 3a and 3b is 117. (2) b = 5a

(A) Statement (1) ALONE is sufficient, but statement (2) alone is not sufficient to answer the question asked (B) Statement (2) ALONE is sufficient, but statement (1) alone is not sufficient to answer the question asked (C) BOTH statements (1) and (2) TOGETHER are sufficient to answer the question asked, but NEITHER statement ALONE is sufficient (D) EACH statement ALONE is sufficient to answer the question asked (E) Statements (1) and (2) TOGETHER are NOT sufficient to answer the question asked, and additional data are needed

GovernmentAdda.com

126

HOMEWORK

34. Penny does consulting work part-time and is paid on an hourly basis. During week W, how much was the total amount of money Penny earned for her work with companies X and Y? (1) During week W, Penny was paid $36 per hour for work performed for company X, and $29 per hour for work performed for company Y. (2) During week W, Penny worked for a total of 60 hours.

(A) Statement (1) ALONE is sufficient, but statement (2) alone is not sufficient to answer the question asked (B) Statement (2) ALONE is sufficient, but statement (1) alone is not sufficient to answer the question asked (C) BOTH statements (1) and (2) TOGETHER are sufficient to answer the question asked, but NEITHER statement ALONE is sufficient

GovernmentAdda.com (D) EACH statement ALONE is sufficient to answer the question asked

(E) Statements (1) and (2) TOGETHER are NOT sufficient to answer the question asked, and additional data are needed

HOMEWORK

127

35. Is x a positive number? (1) –5x – 3 > -2x (2) x2 is positive.

(A) Statement (1) ALONE is sufficient, but statement (2) alone is not sufficient to answer the question asked (B) Statement (2) ALONE is sufficient, but statement (1) alone is not sufficient to answer the question asked (C) BOTH statements (1) and (2) TOGETHER are sufficient to answer the question asked, but NEITHER statement ALONE is sufficient (D) EACH statement ALONE is sufficient to answer the question asked (E) Statements (1) and (2) TOGETHER are NOT sufficient to answer the question asked, and additional data are needed

GovernmentAdda.com

128

HOMEWORK

36. Julie runs a small art museum. How many paintings does her museum have? (1) Her next acquisition of 4 Monet paintings will increase the number of paintings in the museum’s collection by 10%. (2) The museum currently exhibits 3 impressionist paintings for every 2 nonimpressionist paintings.

(A) Statement (1) ALONE is sufficient, but statement (2) alone is not sufficient to answer the question asked (B) Statement (2) ALONE is sufficient, but statement (1) alone is not sufficient to answer the question asked (C) BOTH statements (1) and (2) TOGETHER are sufficient to answer the question asked, but NEITHER statement ALONE is sufficient (D) EACH statement ALONE is sufficient to answer the question asked

GovernmentAdda.com (E) Statements (1) and (2) TOGETHER are NOT sufficient to answer the question asked, and additional data are needed

HOMEWORK

129

37. If n is an integer, then n is divisible by how many positive integers? (1) n is the product of a prime number and a non-prime positive integer. (2) n and 20 are each divisible by the same number of positive integers.

(A) Statement (1) ALONE is sufficient, but statement (2) alone is not sufficient to answer the question asked (B) Statement (2) ALONE is sufficient, but statement (1) alone is not sufficient to answer the question asked (C) BOTH statements (1) and (2) TOGETHER are sufficient to answer the question asked, but NEITHER statement ALONE is sufficient (D) EACH statement ALONE is sufficient to answer the question asked (E) Statements (1) and (2) TOGETHER are NOT sufficient to answer the question asked, and additional data are needed

GovernmentAdda.com

130

HOMEWORK

38. What is the remainder when integer k is divided by integer j? (1) j and k each have the same number of unique factors. (2) 1 < j < k < 10

(A) Statement (1) ALONE is sufficient, but statement (2) alone is not sufficient to answer the question asked (B) Statement (2) ALONE is sufficient, but statement (1) alone is not sufficient to answer the question asked (C) BOTH statements (1) and (2) TOGETHER are sufficient to answer the question asked, but NEITHER statement ALONE is sufficient (D) EACH statement ALONE is sufficient to answer the question asked (E) Statements (1) and (2) TOGETHER are NOT sufficient to answer the question asked, and additional data are needed

GovernmentAdda.com HOMEWORK

131

39. If production on line A increased 5% from 2006 to 2007, and if production on line B increased 10% in the same period, how many units did line A produce in 2006? (1) The two lines combined produced 100,000 units in 2006. (2) The two lines combined produced 107,500 units total in 2007.

(A) Statement (1) ALONE is sufficient, but statement (2) alone is not sufficient to answer the question asked (B) Statement (2) ALONE is sufficient, but statement (1) alone is not sufficient to answer the question asked (C) BOTH statements (1) and (2) TOGETHER are sufficient to answer the question asked, but NEITHER statement ALONE is sufficient (D) EACH statement ALONE is sufficient to answer the question asked

GovernmentAdda.com

(E) Statements (1) and (2) TOGETHER are NOT sufficient to answer the question asked, and additional data are needed

132

HOMEWORK

40. In a retail store, the average (arithmetic mean) sale for month M was d dollars. Was the average (arithmetic mean) sale for month J at least 20 percent higher than that for month M? (1) For month M, total revenue from sales was $3,500. (2) For month J, total revenue from sales was $6,000.

(A) Statement (1) ALONE is sufficient, but statement (2) alone is not sufficient to answer the question asked (B) Statement (2) ALONE is sufficient, but statement (1) alone is not sufficient to answer the question asked (C) BOTH statements (1) and (2) TOGETHER are sufficient to answer the question asked, but NEITHER statement ALONE is sufficient (D) EACH statement ALONE is sufficient to answer the question asked

GovernmentAdda.com (E) Statements (1) and (2) TOGETHER are NOT sufficient to answer the question asked, and additional data are needed

HOMEWORK

133

41. If x and y are positive integers, is the product xy divisible by 9? (1) The product xy is divisible by 6. (2) x and y are perfect squares.

(A) Statement (1) ALONE is sufficient, but statement (2) alone is not sufficient to answer the question asked (B) Statement (2) ALONE is sufficient, but statement (1) alone is not sufficient to answer the question asked (C) BOTH statements (1) and (2) TOGETHER are sufficient to answer the question asked, but NEITHER statement ALONE is sufficient (D) EACH statement ALONE is sufficient to answer the question asked (E) Statements (1) and (2) TOGETHER are NOT sufficient to answer the question asked, and additional data are needed

GovernmentAdda.com

134

HOMEWORK

42. A certain company paid bonuses of $125 to each of its executive employees and $75 to each of its non-executive employees. If 100 of the employees were non-executives, how many of the employees were executives? (1) The company has a total of 120 employees. (2) The total amount that the company paid in bonuses to its employees was $10,000.

(A) Statement (1) ALONE is sufficient, but statement (2) alone is not sufficient to answer the question asked (B) Statement (2) ALONE is sufficient, but statement (1) alone is not sufficient to answer the question asked (C) BOTH statements (1) and (2) TOGETHER are sufficient to answer the question asked, but NEITHER statement ALONE is sufficient

GovernmentAdda.com (D) EACH statement ALONE is sufficient to answer the question asked

(E) Statements (1) and (2) TOGETHER are NOT sufficient to answer the question asked, and additional data are needed

HOMEWORK

135

43. If x = y2, what is the value of y – x? (1) x = 4 (2) x + y = 2

(A) Statement (1) ALONE is sufficient, but statement (2) alone is not sufficient to answer the question asked (B) Statement (2) ALONE is sufficient, but statement (1) alone is not sufficient to answer the question asked (C) BOTH statements (1) and (2) TOGETHER are sufficient to answer the question asked, but NEITHER statement ALONE is sufficient (D) EACH statement ALONE is sufficient to answer the question asked (E) Statements (1) and (2) TOGETHER are NOT sufficient to answer the question asked, and additional data are needed

GovernmentAdda.com

136

HOMEWORK

44. Exactly how many show dogs does Sheila have? (1) Of Sheila’s show dogs, exactly 3 have won prizes of at least $500. (2) Of Sheila’s show dogs, exactly 40% have not won a prize of $500 or more.

(A) Statement (1) ALONE is sufficient, but statement (2) alone is not sufficient to answer the question asked (B) Statement (2) ALONE is sufficient, but statement (1) alone is not sufficient to answer the question asked (C) BOTH statements (1) and (2) TOGETHER are sufficient to answer the question asked, but NEITHER statement ALONE is sufficient (D) EACH statement ALONE is sufficient to answer the question asked (E) Statements (1) and (2) TOGETHER are NOT sufficient to answer the question asked, and additional data are needed

GovernmentAdda.com HOMEWORK

137

45. Each M-type unit will increase the base memory capacity of a certain computer by 3 megabytes. What is the base memory capacity, in megabytes, of the computer? (1) 2 M-type memory units will increase the computer’s base memory by 300 percent. (2) The memory capacity of the computer after 2 M-type memory units are added to the base memory capacity is 1.6 times the memory capacity after 1 M-type memory unit is added to the base memory capacity.

(A) Statement (1) ALONE is sufficient, but statement (2) alone is not sufficient to answer the question asked (B) Statement (2) ALONE is sufficient, but statement (1) alone is not sufficient to answer the question asked (C) BOTH statements (1) and (2) TOGETHER are sufficient to answer the question asked, but NEITHER statement ALONE is sufficient

GovernmentAdda.com

(D) EACH statement ALONE is sufficient to answer the question asked (E) Statements (1) and (2) TOGETHER are NOT sufficient to answer the question asked, and additional data are needed

138

HOMEWORK

46. Is a2 > 3a – b4? (1) 3a – b4 = -5 (2) a > 5 and b > 0

(A) Statement (1) ALONE is sufficient, but statement (2) alone is not sufficient to answer the question asked (B) Statement (2) ALONE is sufficient, but statement (1) alone is not sufficient to answer the question asked (C) BOTH statements (1) and (2) TOGETHER are sufficient to answer the question asked, but NEITHER statement ALONE is sufficient (D) EACH statement ALONE is sufficient to answer the question asked (E) Statements (1) and (2) TOGETHER are NOT sufficient to answer the question asked, and additional data are needed

GovernmentAdda.com HOMEWORK

139

47. If x is a positive integer, is x even? (1) 5x is even. (2) x – 5 is odd.

(A) Statement (1) ALONE is sufficient, but statement (2) alone is not sufficient to answer the question asked (B) Statement (2) ALONE is sufficient, but statement (1) alone is not sufficient to answer the question asked (C) BOTH statements (1) and (2) TOGETHER are sufficient to answer the question asked, but NEITHER statement ALONE is sufficient (D) EACH statement ALONE is sufficient to answer the question asked (E) Statements (1) and (2) TOGETHER are NOT sufficient to answer the question asked, and additional data are needed

GovernmentAdda.com

140

HOMEWORK

48. Is x2 > x3? (1) x3 > 0 (2) x ≠ 1

(A) Statement (1) ALONE is sufficient, but statement (2) alone is not sufficient to answer the question asked (B) Statement (2) ALONE is sufficient, but statement (1) alone is not sufficient to answer the question asked (C) BOTH statements (1) and (2) TOGETHER are sufficient to answer the question asked, but NEITHER statement ALONE is sufficient (D) EACH statement ALONE is sufficient to answer the question asked (E) Statements (1) and (2) TOGETHER are NOT sufficient to answer the question asked, and additional data are needed

GovernmentAdda.com HOMEWORK

141

49. In a taste test conducted for coffee brands X and Y, if each of the 1,600 people polled voted for either X or Y (but not both), what percent of the males voted for brand Y? (1) Forty percent of the females polled voted for brand X. (2) Sixty-five percent of the males voted for brand X.

(A) Statement (1) ALONE is sufficient, but statement (2) alone is not sufficient to answer the question asked (B) Statement (2) ALONE is sufficient, but statement (1) alone is not sufficient to answer the question asked (C) BOTH statements (1) and (2) TOGETHER are sufficient to answer the question asked, but NEITHER statement ALONE is sufficient (D) EACH statement ALONE is sufficient to answer the question asked

GovernmentAdda.com

(E) Statements (1) and (2) TOGETHER are NOT sufficient to answer the question asked, and additional data are needed

142

HOMEWORK

50. Jeff’s average (arithmetic mean) monthly utility bill for the months of June, July, and August was $75. During which month did he receive the highest bill? (1) The lowest bill was received in July. (2) The total of his bills for July and August was $160.

(A) Statement (1) ALONE is sufficient, but statement (2) alone is not sufficient to answer the question asked (B) Statement (2) ALONE is sufficient, but statement (1) alone is not sufficient to answer the question asked (C) BOTH statements (1) and (2) TOGETHER are sufficient to answer the question asked, but NEITHER statement ALONE is sufficient (D) EACH statement ALONE is sufficient to answer the question asked (E) Statements (1) and (2) TOGETHER are NOT sufficient to answer the question asked, and additional data are needed

GovernmentAdda.com HOMEWORK

143

51. What is the value of two-digit integer x? (1) The product of the two digits is 14. (2) x is divisible by 9.

(A) Statement (1) ALONE is sufficient, but statement (2) alone is not sufficient to answer the question asked (B) Statement (2) ALONE is sufficient, but statement (1) alone is not sufficient to answer the question asked (C) BOTH statements (1) and (2) TOGETHER are sufficient to answer the question asked, but NEITHER statement ALONE is sufficient (D) EACH statement ALONE is sufficient to answer the question asked (E) Statements (1) and (2) TOGETHER are NOT sufficient to answer the question asked, and additional data are needed

GovernmentAdda.com

144

HOMEWORK

52. If x + y = 6, then what does x – y equal? (1) x2 – y2 = 12 (2) 2y + x = 8

(A) Statement (1) ALONE is sufficient, but statement (2) alone is not sufficient to answer the question asked (B) Statement (2) ALONE is sufficient, but statement (1) alone is not sufficient to answer the question asked (C) BOTH statements (1) and (2) TOGETHER are sufficient to answer the question asked, but NEITHER statement ALONE is sufficient (D) EACH statement ALONE is sufficient to answer the question asked (E) Statements (1) and (2) TOGETHER are NOT sufficient to answer the question asked, and additional data are needed

GovernmentAdda.com HOMEWORK

145

53. If the two floors in a certain building are 9 feet apart, how many steps are there in a set of stairs that extends from the first floor to the second floor of the building? (1) Each step is ¾ foot high. (2) Each step is 1 foot wide.

(A) Statement (1) ALONE is sufficient, but statement (2) alone is not sufficient to answer the question asked (B) Statement (2) ALONE is sufficient, but statement (1) alone is not sufficient to answer the question asked (C) BOTH statements (1) and (2) TOGETHER are sufficient to answer the question asked, but NEITHER statement ALONE is sufficient (D) EACH statement ALONE is sufficient to answer the question asked

GovernmentAdda.com

(E) Statements (1) and (2) TOGETHER are NOT sufficient to answer the question asked, and additional data are needed

146

HOMEWORK

54. If a, b, and c are integers, is 2a – b + c greater than a – b – 2c? (1) a is positive. (2) c is positive.

(A) Statement (1) ALONE is sufficient, but statement (2) alone is not sufficient to answer the question asked (B) Statement (2) ALONE is sufficient, but statement (1) alone is not sufficient to answer the question asked (C) BOTH statements (1) and (2) TOGETHER are sufficient to answer the question asked, but NEITHER statement ALONE is sufficient (D) EACH statement ALONE is sufficient to answer the question asked (E) Statements (1) and (2) TOGETHER are NOT sufficient to answer the question asked, and additional data are needed

GovernmentAdda.com HOMEWORK

147

55. A beer company spent $100,000 last year on hops, yeast, and malt. How much of the total expenditure was for hops? (1) The expenditure for yeast was 20% greater than the expenditure for malt. (2) The total expenditure for yeast and malt was equal to the expenditure for hops.

(A) Statement (1) ALONE is sufficient, but statement (2) alone is not sufficient to answer the question asked (B) Statement (2) ALONE is sufficient, but statement (1) alone is not sufficient to answer the question asked (C) BOTH statements (1) and (2) TOGETHER are sufficient to answer the question asked, but NEITHER statement ALONE is sufficient (D) EACH statement ALONE is sufficient to answer the question asked

GovernmentAdda.com

(E) Statements (1) and (2) TOGETHER are NOT sufficient to answer the question asked, and additional data are needed

148

HOMEWORK

56. If xyz ≠ 0, what is the value of

x5y4z2 ? z2y4x2

(1) x = 1 (2) y = 1

(A) Statement (1) ALONE is sufficient, but statement (2) alone is not sufficient to answer the question asked (B) Statement (2) ALONE is sufficient, but statement (1) alone is not sufficient to answer the question asked (C) BOTH statements (1) and (2) TOGETHER are sufficient to answer the question asked, but NEITHER statement ALONE is sufficient (D) EACH statement ALONE is sufficient to answer the question asked (E) Statements (1) and (2) TOGETHER are NOT sufficient to answer the question asked, and additional data are needed

GovernmentAdda.com HOMEWORK

149

57. A certain 5-liter solution of floor cleaner consists of x liters of concentrated cleaner and y liters of water. How many liters of concentrated cleaner does the solution contain? (1)

x 1 = y 8

(2)

x 1 = 5 9

(A) Statement (1) ALONE is sufficient, but statement (2) alone is not sufficient to answer the question asked (B) Statement (2) ALONE is sufficient, but statement (1) alone is not sufficient to answer the question asked (C) BOTH statements (1) and (2) TOGETHER are sufficient to answer the question asked, but NEITHER statement ALONE is sufficient

GovernmentAdda.com

(D) EACH statement ALONE is sufficient to answer the question asked (E) Statements (1) and (2) TOGETHER are NOT sufficient to answer the question asked, and additional data are needed

150

HOMEWORK

58. If a rope is cut into three pieces of unequal length, what is the length of the shortest of these pieces of rope? (1) The combined length of the longer two pieces of rope is 12 meters. (2) The combined length of the shorter two pieces of rope is 11 meters.

(A) Statement (1) ALONE is sufficient, but statement (2) alone is not sufficient to answer the question asked (B) Statement (2) ALONE is sufficient, but statement (1) alone is not sufficient to answer the question asked (C) BOTH statements (1) and (2) TOGETHER are sufficient to answer the question asked, but NEITHER statement ALONE is sufficient (D) EACH statement ALONE is sufficient to answer the question asked (E) Statements (1) and (2) TOGETHER are NOT sufficient to answer the question asked, and additional data are needed

GovernmentAdda.com HOMEWORK

151

59. If a, b, and c are integers, is the number 3(a + b) + c divisible by 3? (1) a + b is not divisible by 3. (2) c is divisible by 3.

(A) Statement (1) ALONE is sufficient, but statement (2) alone is not sufficient to answer the question asked (B) Statement (2) ALONE is sufficient, but statement (1) alone is not sufficient to answer the question asked (C) BOTH statements (1) and (2) TOGETHER are sufficient to answer the question asked, but NEITHER statement ALONE is sufficient (D) EACH statement ALONE is sufficient to answer the question asked (E) Statements (1) and (2) TOGETHER are NOT sufficient to answer the question asked, and additional data are needed

GovernmentAdda.com

152

HOMEWORK

60. What is the value of x – y? (1) x – y = y – x (2) x – y = x2 – y2

(A) Statement (1) ALONE is sufficient, but statement (2) alone is not sufficient to answer the question asked (B) Statement (2) ALONE is sufficient, but statement (1) alone is not sufficient to answer the question asked (C) BOTH statements (1) and (2) TOGETHER are sufficient to answer the question asked, but NEITHER statement ALONE is sufficient (D) EACH statement ALONE is sufficient to answer the question asked (E) Statements (1) and (2) TOGETHER are NOT sufficient to answer the question asked, and additional data are needed

GovernmentAdda.com HOMEWORK

153

61. Is n an integer less than 5? (1) 5n is a positive integer. n (2)   is a positive integer. 5

(A) Statement (1) ALONE is sufficient, but statement (2) alone is not sufficient to answer the question asked (B) Statement (2) ALONE is sufficient, but statement (1) alone is not sufficient to answer the question asked (C) BOTH statements (1) and (2) TOGETHER are sufficient to answer the question asked, but NEITHER statement ALONE is sufficient (D) EACH statement ALONE is sufficient to answer the question asked (E) Statements (1) and (2) TOGETHER are NOT sufficient to answer the question asked, and additional data are needed

GovernmentAdda.com

154

HOMEWORK

62. Every man in a certain class either belongs to group A, belongs to group B, or belongs to both groups. 20% of group A consists of men and 65% of group B consists of men. What percentage of the two groups together is made up of men? (1) Group A contains 50 people. (2) Group B contains 100 people.

(A) Statement (1) ALONE is sufficient, but statement (2) alone is not sufficient to answer the question asked (B) Statement (2) ALONE is sufficient, but statement (1) alone is not sufficient to answer the question asked (C) BOTH statements (1) and (2) TOGETHER are sufficient to answer the question asked, but NEITHER statement ALONE is sufficient

GovernmentAdda.com (D) EACH statement ALONE is sufficient to answer the question asked

(E) Statements (1) and (2) TOGETHER are NOT sufficient to answer the question asked, and additional data are needed

HOMEWORK

155

63. If a and b are consecutive negative integers, is b greater than a? (1) a + 1 and b – 1 are consecutive negative integers. (2) b is an odd number.

(A) Statement (1) ALONE is sufficient, but statement (2) alone is not sufficient to answer the question asked (B) Statement (2) ALONE is sufficient, but statement (1) alone is not sufficient to answer the question asked (C) BOTH statements (1) and (2) TOGETHER are sufficient to answer the question asked, but NEITHER statement ALONE is sufficient (D) EACH statement ALONE is sufficient to answer the question asked (E) Statements (1) and (2) TOGETHER are NOT sufficient to answer the question asked, and additional data are needed

GovernmentAdda.com

156

HOMEWORK

64. Is x2 equal to xy? (1) x2 + y2 = -(x2 – y2) (2) x = y

(A) Statement (1) ALONE is sufficient, but statement (2) alone is not sufficient to answer the question asked (B) Statement (2) ALONE is sufficient, but statement (1) alone is not sufficient to answer the question asked (C) BOTH statements (1) and (2) TOGETHER are sufficient to answer the question asked, but NEITHER statement ALONE is sufficient (D) EACH statement ALONE is sufficient to answer the question asked (E) Statements (1) and (2) TOGETHER are NOT sufficient to answer the question asked, and additional data are needed

GovernmentAdda.com HOMEWORK

157

65. If 2.5 < x < 4.5, is the tenths digit of the decimal representation of x equal to 4? (1) x + 0.006 < 4.5 (2) x + 0.06 > 4.5

(A) Statement (1) ALONE is sufficient, but statement (2) alone is not sufficient to answer the question asked (B) Statement (2) ALONE is sufficient, but statement (1) alone is not sufficient to answer the question asked (C) BOTH statements (1) and (2) TOGETHER are sufficient to answer the question asked, but NEITHER statement ALONE is sufficient (D) EACH statement ALONE is sufficient to answer the question asked (E) Statements (1) and (2) TOGETHER are NOT sufficient to answer the question asked, and additional data are needed

GovernmentAdda.com

158

HOMEWORK

a

b

0 66. If the successive tick marks shown on the number line above are equally spaced and if a and b are the numbers designating the end points of intervals as shown, what is the value of b ?

1 4 1 (2) b – a = 3 (1) a =

GovernmentAdda.com (A) Statement (1) ALONE is sufficient, but statement (2) alone is not sufficient to answer the question asked (B) Statement (2) ALONE is sufficient, but statement (1) alone is not sufficient to answer the question asked

HOMEWORK

(C) BOTH statements (1) and (2) TOGETHER are sufficient to answer the question asked, but NEITHER statement ALONE is sufficient (D) EACH statement ALONE is sufficient to answer the question asked (E) Statements (1) and (2) TOGETHER are NOT sufficient to answer the question asked, and additional data are needed

159

67. A certain ice cream factory makes two flavors of ice cream: chocolate and caramel. Each flavor comes in two varieties: one with over 20 percent milk and 2 one with under 20 percent milk. Did more than of all the ice cream made in 5 July contain over 20 percent milk?

(1) Exactly 80 percent of the caramel ice cream made in July contained over 1 10 percent milk, and of this amount, contained over 20 percent milk. 4

(2) Exactly 1,000 gallons of the chocolate ice cream made in July contained over 20 percent milk.

(A) Statement (1) ALONE is sufficient, but statement (2) alone is not sufficient to answer the question asked (B) Statement (2) ALONE is sufficient, but statement (1) alone is not sufficient to answer the question asked

GovernmentAdda.com

(C) BOTH statements (1) and (2) TOGETHER are sufficient to answer the question asked, but NEITHER statement ALONE is sufficient (D) EACH statement ALONE is sufficient to answer the question asked (E) Statements (1) and (2) TOGETHER are NOT sufficient to answer the question asked, and additional data are needed

160

HOMEWORK

68. If x ≠ 0, is y greater than 0? (1) xy = 24 (2) x – y = 5

(A) Statement (1) ALONE is sufficient, but statement (2) alone is not sufficient to answer the question asked (B) Statement (2) ALONE is sufficient, but statement (1) alone is not sufficient to answer the question asked (C) BOTH statements (1) and (2) TOGETHER are sufficient to answer the question asked, but NEITHER statement ALONE is sufficient (D) EACH statement ALONE is sufficient to answer the question asked (E) Statements (1) and (2) TOGETHER are NOT sufficient to answer the question asked, and additional data are needed

GovernmentAdda.com HOMEWORK

161

69. If

a 4 = , is a less than b? 3 b

(1) b ≥ 4 (2) b ≤ 5

(A) Statement (1) ALONE is sufficient, but statement (2) alone is not sufficient to answer the question asked (B) Statement (2) ALONE is sufficient, but statement (1) alone is not sufficient to answer the question asked (C) BOTH statements (1) and (2) TOGETHER are sufficient to answer the question asked, but NEITHER statement ALONE is sufficient (D) EACH statement ALONE is sufficient to answer the question asked

GovernmentAdda.com

(E) Statements (1) and (2) TOGETHER are NOT sufficient to answer the question asked, and additional data are needed

162

HOMEWORK

70. Does x = 2? (1) x is a number such that x2 – 3x + 2 = 0. (2) x is a number such that x2 – x – 2 = 0.

(A) Statement (1) ALONE is sufficient, but statement (2) alone is not sufficient to answer the question asked (B) Statement (2) ALONE is sufficient, but statement (1) alone is not sufficient to answer the question asked (C) BOTH statements (1) and (2) TOGETHER are sufficient to answer the question asked, but NEITHER statement ALONE is sufficient (D) EACH statement ALONE is sufficient to answer the question asked (E) Statements (1) and (2) TOGETHER are NOT sufficient to answer the question asked, and additional data are needed

GovernmentAdda.com HOMEWORK

163

71. Is y less than 2.4? (1) y < 2.5 (2) y < 2.3

(A) Statement (1) ALONE is sufficient, but statement (2) alone is not sufficient to answer the question asked (B) Statement (2) ALONE is sufficient, but statement (1) alone is not sufficient to answer the question asked (C) BOTH statements (1) and (2) TOGETHER are sufficient to answer the question asked, but NEITHER statement ALONE is sufficient (D) EACH statement ALONE is sufficient to answer the question asked (E) Statements (1) and (2) TOGETHER are NOT sufficient to answer the question asked, and additional data are needed

GovernmentAdda.com

164

HOMEWORK

72. Ted owns an electronics business and is calculating the price of a certain group of microwaves in the inventory he purchased. If the total price of m equally priced microwaves was $18,000, what was the price per microwave in the inventory? (1) If the price per microwave in Ted’s inventory had been $10 more, the total price of the m microwaves would have been $5,000 more. (2) If the price per microwave in Ted’s inventory had been $9 less, the total price of the m microwaves would have been 25 percent less.

(A) Statement (1) ALONE is sufficient, but statement (2) alone is not sufficient to answer the question asked (B) Statement (2) ALONE is sufficient, but statement (1) alone is not sufficient to answer the question asked (C) BOTH statements (1) and (2) TOGETHER are sufficient to answer the question asked, but NEITHER statement ALONE is sufficient

GovernmentAdda.com (D) EACH statement ALONE is sufficient to answer the question asked

(E) Statements (1) and (2) TOGETHER are NOT sufficient to answer the question asked, and additional data are needed

HOMEWORK

165

73. A store sold 6 bicycles with an average sale price of $1,000. What was the price of the most expensive bicycle? (1) The median price was $1,000. (2) The range of prices was $600.

(A) Statement (1) ALONE is sufficient, but statement (2) alone is not sufficient to answer the question asked (B) Statement (2) ALONE is sufficient, but statement (1) alone is not sufficient to answer the question asked (C) BOTH statements (1) and (2) TOGETHER are sufficient to answer the question asked, but NEITHER statement ALONE is sufficient (D) EACH statement ALONE is sufficient to answer the question asked (E) Statements (1) and (2) TOGETHER are NOT sufficient to answer the question asked, and additional data are needed

GovernmentAdda.com

166

HOMEWORK

74. What is the value of j – k? (1) 2j + 4n = 2k + 4n – 6 (2) n = 3

(A) Statement (1) ALONE is sufficient, but statement (2) alone is not sufficient to answer the question asked (B) Statement (2) ALONE is sufficient, but statement (1) alone is not sufficient to answer the question asked (C) BOTH statements (1) and (2) TOGETHER are sufficient to answer the question asked, but NEITHER statement ALONE is sufficient (D) EACH statement ALONE is sufficient to answer the question asked (E) Statements (1) and (2) TOGETHER are NOT sufficient to answer the question asked, and additional data are needed

GovernmentAdda.com HOMEWORK

167

75. If 20x + 30y = 280, what is the value of x? (1) 4x = 56 – 6y (2) y2 = 16

(A) Statement (1) ALONE is sufficient, but statement (2) alone is not sufficient to answer the question asked (B) Statement (2) ALONE is sufficient, but statement (1) alone is not sufficient to answer the question asked (C) BOTH statements (1) and (2) TOGETHER are sufficient to answer the question asked, but NEITHER statement ALONE is sufficient (D) EACH statement ALONE is sufficient to answer the question asked (E) Statements (1) and (2) TOGETHER are NOT sufficient to answer the question asked, and additional data are needed

GovernmentAdda.com

168

HOMEWORK

76. If y is an integer, is y2 divisible by 4? (1) y is even. (2) y3 is divisible by 4.

(A) Statement (1) ALONE is sufficient, but statement (2) alone is not sufficient to answer the question asked (B) Statement (2) ALONE is sufficient, but statement (1) alone is not sufficient to answer the question asked (C) BOTH statements (1) and (2) TOGETHER are sufficient to answer the question asked, but NEITHER statement ALONE is sufficient (D) EACH statement ALONE is sufficient to answer the question asked (E) Statements (1) and (2) TOGETHER are NOT sufficient to answer the question asked, and additional data are needed

GovernmentAdda.com HOMEWORK

169

77. Is xy < 8? (1) x < 2 and y < 4 (2) 0 < x < ½ and y2 < 225

(A) Statement (1) ALONE is sufficient, but statement (2) alone is not sufficient to answer the question asked (B) Statement (2) ALONE is sufficient, but statement (1) alone is not sufficient to answer the question asked (C) BOTH statements (1) and (2) TOGETHER are sufficient to answer the question asked, but NEITHER statement ALONE is sufficient (D) EACH statement ALONE is sufficient to answer the question asked (E) Statements (1) and (2) TOGETHER are NOT sufficient to answer the question asked, and additional data are needed

GovernmentAdda.com

170

HOMEWORK

P

Q

T

R

S

78. In the figure above, QRS is a straight line and QR = PR. Is it true that lines TR and PQ are parallel? (1) Length PQ = Length PR (2) Line TR bisects angle PRS.

GovernmentAdda.com (A) Statement (1) ALONE is sufficient, but statement (2) alone is not sufficient to answer the question asked (B) Statement (2) ALONE is sufficient, but statement (1) alone is not sufficient to answer the question asked

HOMEWORK

(C) BOTH statements (1) and (2) TOGETHER are sufficient to answer the question asked, but NEITHER statement ALONE is sufficient (D) EACH statement ALONE is sufficient to answer the question asked (E) Statements (1) and (2) TOGETHER are NOT sufficient to answer the question asked, and additional data are needed

171

79. If each of the 20 bolts of fabric on a shelf is either 100 percent cotton, 100 percent wool, or a mixture of cotton and wool, how many bolts contain both cotton and wool? (1) Of the 20 bolts, 18 contain some wool and 14 contain some cotton. (2) Of the 20 bolts, 6 are 100 percent wool.

(A) Statement (1) ALONE is sufficient, but statement (2) alone is not sufficient to answer the question asked (B) Statement (2) ALONE is sufficient, but statement (1) alone is not sufficient to answer the question asked (C) BOTH statements (1) and (2) TOGETHER are sufficient to answer the question asked, but NEITHER statement ALONE is sufficient (D) EACH statement ALONE is sufficient to answer the question asked

GovernmentAdda.com

(E) Statements (1) and (2) TOGETHER are NOT sufficient to answer the question asked, and additional data are needed

172

HOMEWORK

80. If x, y, and z are lengths of three sides of a triangle, is x < 3? (1) z = y + 3 (2) y = 3 and z = 6

(A) Statement (1) ALONE is sufficient, but statement (2) alone is not sufficient to answer the question asked (B) Statement (2) ALONE is sufficient, but statement (1) alone is not sufficient to answer the question asked (C) BOTH statements (1) and (2) TOGETHER are sufficient to answer the question asked, but NEITHER statement ALONE is sufficient (D) EACH statement ALONE is sufficient to answer the question asked (E) Statements (1) and (2) TOGETHER are NOT sufficient to answer the question asked, and additional data are needed

GovernmentAdda.com HOMEWORK

173

81. Is the positive integer x an even number? (1) If x is divided by 3, the remainder is 2. (2) If x is divided by 5, the remainder is 2.

(A) Statement (1) ALONE is sufficient, but statement (2) alone is not sufficient to answer the question asked (B) Statement (2) ALONE is sufficient, but statement (1) alone is not sufficient to answer the question asked (C) BOTH statements (1) and (2) TOGETHER are sufficient to answer the question asked, but NEITHER statement ALONE is sufficient (D) EACH statement ALONE is sufficient to answer the question asked (E) Statements (1) and (2) TOGETHER are NOT sufficient to answer the question asked, and additional data are needed

GovernmentAdda.com

174

HOMEWORK

82. The surface area of a rectangular field was changed so that the length of one of the dimensions was reduced by 10 feet and the length of the other dimension was increased by 20 feet. What was the surface area before these changes were made? (1) After the changes were made, the surface area was 2,500 square feet. (2) The length and width of the field were equal after the changes were made.

(A) Statement (1) ALONE is sufficient, but statement (2) alone is not sufficient to answer the question asked (B) Statement (2) ALONE is sufficient, but statement (1) alone is not sufficient to answer the question asked (C) BOTH statements (1) and (2) TOGETHER are sufficient to answer the question asked, but NEITHER statement ALONE is sufficient

GovernmentAdda.com (D) EACH statement ALONE is sufficient to answer the question asked

(E) Statements (1) and (2) TOGETHER are NOT sufficient to answer the question asked, and additional data are needed

HOMEWORK

175

83. In a certain store, item X sells for 10 percent less than item Y. What is the ratio of the store’s revenue from the sales of item X to that from the sales of item Y? (1) The store sells 20 percent more units of item Y than of item X. (2) The store’s revenue from the sales of item X is $6,000 and from the sales of item Y is $8,000.

(A) Statement (1) ALONE is sufficient, but statement (2) alone is not sufficient to answer the question asked (B) Statement (2) ALONE is sufficient, but statement (1) alone is not sufficient to answer the question asked (C) BOTH statements (1) and (2) TOGETHER are sufficient to answer the question asked, but NEITHER statement ALONE is sufficient (D) EACH statement ALONE is sufficient to answer the question asked

GovernmentAdda.com

(E) Statements (1) and (2) TOGETHER are NOT sufficient to answer the question asked, and additional data are needed

176

HOMEWORK

84. During a three-year period, the profits of company X changed by what percent from the second year to the third year? (1) The increase in profits of company X from the first year to the second year was the same as the increase from the first year to the third year. (2) For company X, the profits for the first year were $13,800 and the profits for the third year were $15,900.

(A) Statement (1) ALONE is sufficient, but statement (2) alone is not sufficient to answer the question asked (B) Statement (2) ALONE is sufficient, but statement (1) alone is not sufficient to answer the question asked (C) BOTH statements (1) and (2) TOGETHER are sufficient to answer the question asked, but NEITHER statement ALONE is sufficient

GovernmentAdda.com (D) EACH statement ALONE is sufficient to answer the question asked

(E) Statements (1) and (2) TOGETHER are NOT sufficient to answer the question asked, and additional data are needed

HOMEWORK

177

85. A pyramid-shaped box to protect a plant is constructed with 4 lateral faces and an open bottom. What is the lateral area of the box? (1) The base of the pyramid is a polygon with all sides of equal length, and the perimeter of the base is 1 meter. (2) The lateral faces are isosceles triangles that have the same size and shape.

(A) Statement (1) ALONE is sufficient, but statement (2) alone is not sufficient to answer the question asked (B) Statement (2) ALONE is sufficient, but statement (1) alone is not sufficient to answer the question asked (C) BOTH statements (1) and (2) TOGETHER are sufficient to answer the question asked, but NEITHER statement ALONE is sufficient (D) EACH statement ALONE is sufficient to answer the question asked

GovernmentAdda.com

(E) Statements (1) and (2) TOGETHER are NOT sufficient to answer the question asked, and additional data are needed

178

HOMEWORK

86. If x and y are integers and their sum is 23, is y ≥ 9? (1) x – 6 < 9 (2) x3 = 2,744

(A) Statement (1) ALONE is sufficient, but statement (2) alone is not sufficient to answer the question asked (B) Statement (2) ALONE is sufficient, but statement (1) alone is not sufficient to answer the question asked (C) BOTH statements (1) and (2) TOGETHER are sufficient to answer the question asked, but NEITHER statement ALONE is sufficient (D) EACH statement ALONE is sufficient to answer the question asked (E) Statements (1) and (2) TOGETHER are NOT sufficient to answer the question asked, and additional data are needed

GovernmentAdda.com HOMEWORK

179

87. A farmer has a total of 60 pigs, cows, and horses on his farm. How many pigs does he have? (1) The ratio of horses to cows is 2:9. (2) He has more than 36 cows.

(A) Statement (1) ALONE is sufficient, but statement (2) alone is not sufficient to answer the question asked (B) Statement (2) ALONE is sufficient, but statement (1) alone is not sufficient to answer the question asked (C) BOTH statements (1) and (2) TOGETHER are sufficient to answer the question asked, but NEITHER statement ALONE is sufficient (D) EACH statement ALONE is sufficient to answer the question asked (E) Statements (1) and (2) TOGETHER are NOT sufficient to answer the question asked, and additional data are needed

GovernmentAdda.com

180

HOMEWORK

88. What is the value of x + y? (1)

4x2 – 4y2 = 2x – 2y 2(x+y)

 

(2) 3x + 2y = 24

(A) Statement (1) ALONE is sufficient, but statement (2) alone is not sufficient to answer the question asked (B) Statement (2) ALONE is sufficient, but statement (1) alone is not sufficient to answer the question asked (C) BOTH statements (1) and (2) TOGETHER are sufficient to answer the question asked, but NEITHER statement ALONE is sufficient (D) EACH statement ALONE is sufficient to answer the question asked

GovernmentAdda.com (E) Statements (1) and (2) TOGETHER are NOT sufficient to answer the question asked, and additional data are needed

HOMEWORK

181

89. What is the value of the two-digit number x? (1) The sum of the two digits is 4. (2) The difference between the two digits is 2.

(A) Statement (1) ALONE is sufficient, but statement (2) alone is not sufficient to answer the question asked (B) Statement (2) ALONE is sufficient, but statement (1) alone is not sufficient to answer the question asked (C) BOTH statements (1) and (2) TOGETHER are sufficient to answer the question asked, but NEITHER statement ALONE is sufficient (D) EACH statement ALONE is sufficient to answer the question asked (E) Statements (1) and (2) TOGETHER are NOT sufficient to answer the question asked, and additional data are needed

GovernmentAdda.com

182

HOMEWORK

—–– 90. Is √ 7ab an integer? (1) a = 7 (2) b is equal to an integer raised to the third power.

(A) Statement (1) ALONE is sufficient, but statement (2) alone is not sufficient to answer the question asked (B) Statement (2) ALONE is sufficient, but statement (1) alone is not sufficient to answer the question asked (C) BOTH statements (1) and (2) TOGETHER are sufficient to answer the question asked, but NEITHER statement ALONE is sufficient (D) EACH statement ALONE is sufficient to answer the question asked (E) Statements (1) and (2) TOGETHER are NOT sufficient to answer the question asked, and additional data are needed

GovernmentAdda.com HOMEWORK

183

91. Is xy > 24? (1) y – 2 < x (2) 2y > x + 8

(A) Statement (1) ALONE is sufficient, but statement (2) alone is not sufficient to answer the question asked (B) Statement (2) ALONE is sufficient, but statement (1) alone is not sufficient to answer the question asked (C) BOTH statements (1) and (2) TOGETHER are sufficient to answer the question asked, but NEITHER statement ALONE is sufficient (D) EACH statement ALONE is sufficient to answer the question asked (E) Statements (1) and (2) TOGETHER are NOT sufficient to answer the question asked, and additional data are needed

GovernmentAdda.com

184

HOMEWORK

92. Is ab a prime number? (1) a is a prime number. (2) b is not a prime number.

(A) Statement (1) ALONE is sufficient, but statement (2) alone is not sufficient to answer the question asked (B) Statement (2) ALONE is sufficient, but statement (1) alone is not sufficient to answer the question asked (C) BOTH statements (1) and (2) TOGETHER are sufficient to answer the question asked, but NEITHER statement ALONE is sufficient (D) EACH statement ALONE is sufficient to answer the question asked (E) Statements (1) and (2) TOGETHER are NOT sufficient to answer the question asked, and additional data are needed

GovernmentAdda.com HOMEWORK

185

93. A rectangular floor that is 4 meters wide is to be completely covered with non-overlapping square tiles, each with side of length 0.25 meter, with no portion of any tile remaining. What is the least number of such tiles that will be required? (1) The length of the floor is three times the width. (2) The area of the floor is 48 square meters.

(A) Statement (1) ALONE is sufficient, but statement (2) alone is not sufficient to answer the question asked (B) Statement (2) ALONE is sufficient, but statement (1) alone is not sufficient to answer the question asked (C) BOTH statements (1) and (2) TOGETHER are sufficient to answer the question asked, but NEITHER statement ALONE is sufficient

GovernmentAdda.com

(D) EACH statement ALONE is sufficient to answer the question asked (E) Statements (1) and (2) TOGETHER are NOT sufficient to answer the question asked, and additional data are needed

186

HOMEWORK

94. Is x > 3? (1) The sum of x and the square of x is 12. (2) x2 > 9

(A) Statement (1) ALONE is sufficient, but statement (2) alone is not sufficient to answer the question asked (B) Statement (2) ALONE is sufficient, but statement (1) alone is not sufficient to answer the question asked (C) BOTH statements (1) and (2) TOGETHER are sufficient to answer the question asked, but NEITHER statement ALONE is sufficient (D) EACH statement ALONE is sufficient to answer the question asked (E) Statements (1) and (2) TOGETHER are NOT sufficient to answer the question asked, and additional data are needed

GovernmentAdda.com HOMEWORK

187

—–– 95. If a, b, and c are distinct positive integers where a < b < c and √ abc = c, what is the value of a? (1) c = 8 (2) The average of a, b, and c is

14 . 3

(A) Statement (1) ALONE is sufficient, but statement (2) alone is not sufficient to answer the question asked (B) Statement (2) ALONE is sufficient, but statement (1) alone is not sufficient to answer the question asked (C) BOTH statements (1) and (2) TOGETHER are sufficient to answer the question asked, but NEITHER statement ALONE is sufficient (D) EACH statement ALONE is sufficient to answer the question asked (E) Statements (1) and (2) TOGETHER are NOT sufficient to answer the question asked, and additional data are needed

GovernmentAdda.com

188

HOMEWORK

96. If x is an integer, what is the value of x? (1)

1 1 1 < < 5 1+x 2

(2) (x – 3)(x – 4) = 0

(A) Statement (1) ALONE is sufficient, but statement (2) alone is not sufficient to answer the question asked (B) Statement (2) ALONE is sufficient, but statement (1) alone is not sufficient to answer the question asked (C) BOTH statements (1) and (2) TOGETHER are sufficient to answer the question asked, but NEITHER statement ALONE is sufficient (D) EACH statement ALONE is sufficient to answer the question asked (E) Statements (1) and (2) TOGETHER are NOT sufficient to answer the question asked, and additional data are needed

GovernmentAdda.com HOMEWORK

189

Y

x° P

Z

97. Cars Y and Z travel side-by-side at the same rate of speed along parallel roads as shown above. When car Y reaches point P, it forks to the left at angle x°, changes speed, and continues to stay even with car Z as shown by the dotted line. The speed of car Y beyond point P is what percent of the speed of car Z? (1) The speed of car Z is 50 miles per hour. (2) x = 45

(A) Statement (1) ALONE is sufficient, but statement (2) alone is not sufficient to answer the question asked

GovernmentAdda.com

(B) Statement (2) ALONE is sufficient, but statement (1) alone is not sufficient to answer the question asked (C) BOTH statements (1) and (2) TOGETHER are sufficient to answer the question asked, but NEITHER statement ALONE is sufficient (D) EACH statement ALONE is sufficient to answer the question asked (E) Statements (1) and (2) TOGETHER are NOT sufficient to answer the question asked, and additional data are needed

190

HOMEWORK

98. For integers a, b, and c,

(1)

a b–c = 1. What is the value of  ? b–c b

a 3 = b 5

(2) a and b have no common factors greater than 1.

(A) Statement (1) ALONE is sufficient, but statement (2) alone is not sufficient to answer the question asked (B) Statement (2) ALONE is sufficient, but statement (1) alone is not sufficient to answer the question asked (C) BOTH statements (1) and (2) TOGETHER are sufficient to answer the question asked, but NEITHER statement ALONE is sufficient (D) EACH statement ALONE is sufficient to answer the question asked

GovernmentAdda.com (E) Statements (1) and (2) TOGETHER are NOT sufficient to answer the question asked, and additional data are needed

HOMEWORK

191

99. What is the value of x – y? (1) (x + y)2 = 4xy (2) x2 – y2 = 0

(A) Statement (1) ALONE is sufficient, but statement (2) alone is not sufficient to answer the question asked (B) Statement (2) ALONE is sufficient, but statement (1) alone is not sufficient to answer the question asked (C) BOTH statements (1) and (2) TOGETHER are sufficient to answer the question asked, but NEITHER statement ALONE is sufficient (D) EACH statement ALONE is sufficient to answer the question asked (E) Statements (1) and (2) TOGETHER are NOT sufficient to answer the question asked, and additional data are needed

GovernmentAdda.com

192

HOMEWORK

100. For positive integer a, is the product (a)(a + 1)(a + 2) divisible by 48? (1) a is even. (2) 4a is divisible by 32.

(A) Statement (1) ALONE is sufficient, but statement (2) alone is not sufficient to answer the question asked (B) Statement (2) ALONE is sufficient, but statement (1) alone is not sufficient to answer the question asked (C) BOTH statements (1) and (2) TOGETHER are sufficient to answer the question asked, but NEITHER statement ALONE is sufficient (D) EACH statement ALONE is sufficient to answer the question asked (E) Statements (1) and (2) TOGETHER are NOT sufficient to answer the question asked, and additional data are needed

GovernmentAdda.com HOMEWORK

193

Q

P

R

T

S

101. In rectangular region PQRS above, T is a point on side PS. If PS = 4, what is the area of region PQRS?

(1) rQTR is equilateral. (2) Segments PT and TS have equal length.

GovernmentAdda.com

(A) Statement (1) ALONE is sufficient, but statement (2) alone is not sufficient to answer the question asked (B) Statement (2) ALONE is sufficient, but statement (1) alone is not sufficient to answer the question asked (C) BOTH statements (1) and (2) TOGETHER are sufficient to answer the question asked, but NEITHER statement ALONE is sufficient (D) EACH statement ALONE is sufficient to answer the question asked (E) Statements (1) and (2) TOGETHER are NOT sufficient to answer the question asked, and additional data are needed

194

HOMEWORK

102. If xy ≠0, is

1 1 + = 16? x y

(1) x + y = 16xy (2) x = y

(A) Statement (1) ALONE is sufficient, but statement (2) alone is not sufficient to answer the question asked (B) Statement (2) ALONE is sufficient, but statement (1) alone is not sufficient to answer the question asked (C) BOTH statements (1) and (2) TOGETHER are sufficient to answer the question asked, but NEITHER statement ALONE is sufficient (D) EACH statement ALONE is sufficient to answer the question asked (E) Statements (1) and (2) TOGETHER are NOT sufficient to answer the question asked, and additional data are needed

GovernmentAdda.com HOMEWORK

195

103. How many students in the senior class take both French and Spanish? (1) In the senior class, 100 students take French and 125 students take Spanish. (2) There are 200 students in the senior class.

(A) Statement (1) ALONE is sufficient, but statement (2) alone is not sufficient to answer the question asked (B) Statement (2) ALONE is sufficient, but statement (1) alone is not sufficient to answer the question asked (C) BOTH statements (1) and (2) TOGETHER are sufficient to answer the question asked, but NEITHER statement ALONE is sufficient (D) EACH statement ALONE is sufficient to answer the question asked (E) Statements (1) and (2) TOGETHER are NOT sufficient to answer the question asked, and additional data are needed

GovernmentAdda.com

196

HOMEWORK

104. Is x > 0? (1) x2 = 9x (2) |x| = – x

(A) Statement (1) ALONE is sufficient, but statement (2) alone is not sufficient to answer the question asked (B) Statement (2) ALONE is sufficient, but statement (1) alone is not sufficient to answer the question asked (C) BOTH statements (1) and (2) TOGETHER are sufficient to answer the question asked, but NEITHER statement ALONE is sufficient (D) EACH statement ALONE is sufficient to answer the question asked (E) Statements (1) and (2) TOGETHER are NOT sufficient to answer the question asked, and additional data are needed

GovernmentAdda.com HOMEWORK

197

105. What is the average of the terms in set J? (1) The sum of any three terms in set J is 21. (2) Set J consists of 12 total terms.

(A) Statement (1) ALONE is sufficient, but statement (2) alone is not sufficient to answer the question asked (B) Statement (2) ALONE is sufficient, but statement (1) alone is not sufficient to answer the question asked (C) BOTH statements (1) and (2) TOGETHER are sufficient to answer the question asked, but NEITHER statement ALONE is sufficient (D) EACH statement ALONE is sufficient to answer the question asked (E) Statements (1) and (2) TOGETHER are NOT sufficient to answer the question asked, and additional data are needed

GovernmentAdda.com

198

HOMEWORK Challenge Questions

CHALLENGE PROBLEMS 106. Line M is tangent to a circle, which is centered on point (3, 4). Does Line M run through point (6, 6)? (1) Line M runs through point (-8, 6). (2) Line M is tangent to the circle at point (3, 6).

(A) Statement (1) ALONE is sufficient, but statement (2) alone is not sufficient to answer the question asked (B) Statement (2) ALONE is sufficient, but statement (1) alone is not sufficient to answer the question asked (C) BOTH statements (1) and (2) TOGETHER are sufficient to answer the question asked, but NEITHER statement ALONE is sufficient

GovernmentAdda.com (D) EACH statement ALONE is sufficient to answer the question asked

(E) Statements (1) and (2) TOGETHER are NOT sufficient to answer the question asked, and additional data are needed

HOMEWORK

199

107. When positive integer x is divided by 7 the quotient is q and the remainder is 1. What is the remainder when x is divided by 10? (1) When x is divided by 5 the quotient is q and the remainder is 1. (2) x is less than 50.

(A) Statement (1) ALONE is sufficient, but statement (2) alone is not sufficient to answer the question asked (B) Statement (2) ALONE is sufficient, but statement (1) alone is not sufficient to answer the question asked (C) BOTH statements (1) and (2) TOGETHER are sufficient to answer the question asked, but NEITHER statement ALONE is sufficient (D) EACH statement ALONE is sufficient to answer the question asked (E) Statements (1) and (2) TOGETHER are NOT sufficient to answer the question asked, and additional data are needed

GovernmentAdda.com

200

HOMEWORK Challenge Questions

108. Of the 60 families in a certain neighborhood, 38 have a cat. How many families in this neighborhood have a dog? (1) 28 of the families in this neighborhood have a cat but not a dog. (2) The number of families in the neighborhood who have a dog and a cat is the same as the number of families who have neither a cat nor a dog.

(A) Statement (1) ALONE is sufficient, but statement (2) alone is not sufficient to answer the question asked (B) Statement (2) ALONE is sufficient, but statement (1) alone is not sufficient to answer the question asked (C) BOTH statements (1) and (2) TOGETHER are sufficient to answer the question asked, but NEITHER statement ALONE is sufficient (D) EACH statement ALONE is sufficient to answer the question asked

GovernmentAdda.com (E) Statements (1) and (2) TOGETHER are NOT sufficient to answer the question asked, and additional data are needed

HOMEWORK

201

109. If x ≠ 0, is

5x – 2 5x – 1 – >0? 3 4

(1) x > 1 (2) x = | x |

(A) Statement (1) ALONE is sufficient, but statement (2) alone is not sufficient to answer the question asked (B) Statement (2) ALONE is sufficient, but statement (1) alone is not sufficient to answer the question asked (C) BOTH statements (1) and (2) TOGETHER are sufficient to answer the question asked, but NEITHER statement ALONE is sufficient (D) EACH statement ALONE is sufficient to answer the question asked (E) Statements (1) and (2) TOGETHER are NOT sufficient to answer the question asked, and additional data are needed

GovernmentAdda.com

202

HOMEWORK Challenge Questions

110. If x and y are positive integers, what is the greatest common factor of x and y? (1) When x is divided by y, the remainder is 1. (2) x2 – 2xy + y2 = 1

(A) Statement (1) ALONE is sufficient, but statement (2) alone is not sufficient to answer the question asked (B) Statement (2) ALONE is sufficient, but statement (1) alone is not sufficient to answer the question asked (C) BOTH statements (1) and (2) TOGETHER are sufficient to answer the question asked, but NEITHER statement ALONE is sufficient (D) EACH statement ALONE is sufficient to answer the question asked (E) Statements (1) and (2) TOGETHER are NOT sufficient to answer the question asked, and additional data are needed

GovernmentAdda.com

HOMEWORK

203

111. Is (y – 10)2 > (x + 10)2? (1) –y > x + 5 (2) x > y

(A) Statement (1) ALONE is sufficient, but statement (2) alone is not sufficient to answer the question asked; (B) Statement (2) ALONE is sufficient, but statement (1) alone is not sufficient to answer the question asked; (C) BOTH statements (1) and (2) TOGETHER are sufficient to answer the question asked, but NEITHER statement ALONE is sufficient; (D) EACH statement ALONE is sufficient to answer the question asked; (E) Statements (1) and (2) TOGETHER are NOT sufficient to answer the question asked, and additional data are needed

GovernmentAdda.com

204

HOMEWORK Challenge Questions

112. What is the value of y? (1) x2 – y2 = 5 (2) x and y are each positive integers.

(A) Statement (1) ALONE is sufficient, but statement (2) alone is not sufficient to answer the question asked (B) Statement (2) ALONE is sufficient, but statement (1) alone is not sufficient to answer the question asked (C) BOTH statements (1) and (2) TOGETHER are sufficient to answer the question asked, but NEITHER statement ALONE is sufficient (D) EACH statement ALONE is sufficient to answer the question asked (E) Statements (1) and (2) TOGETHER are NOT sufficient to answer the question asked, and additional data are needed

GovernmentAdda.com

HOMEWORK

205

113. If a and b are nonzero integers, is ab an integer? (1) ba is negative. (2) ab is negative.

(A) Statement (1) ALONE is sufficient, but statement (2) alone is not sufficient to answer the question asked (B) Statement (2) ALONE is sufficient, but statement (1) alone is not sufficient to answer the question asked (C) BOTH statements (1) and (2) TOGETHER are sufficient to answer the question asked, but NEITHER statement ALONE is sufficient (D) EACH statement ALONE is sufficient to answer the question asked (E) Statements (1) and (2) TOGETHER are NOT sufficient to answer the question asked, and additional data are needed

GovernmentAdda.com

206

HOMEWORK Challenge Questions

114. For nonnegative integers x and y, what is the remainder when x is divided by y? x = 13.8 y (2) The numbers x and y have a combined total of less than 5 digits. (1)

(A) Statement (1) ALONE is sufficient, but statement (2) alone is not sufficient to answer the question asked (B) Statement (2) ALONE is sufficient, but statement (1) alone is not sufficient to answer the question asked (C) BOTH statements (1) and (2) TOGETHER are sufficient to answer the question asked, but NEITHER statement ALONE is sufficient (D) EACH statement ALONE is sufficient to answer the question asked (E) Statements (1) and (2) TOGETHER are NOT sufficient to answer the question asked, and additional data are needed

GovernmentAdda.com

HOMEWORK

207

Y 4 X q r

Z

115. If arc XYZ above is a semicircle, what is its length? (1) q = 2 (2) r = 8

(A) Statement (1) ALONE is sufficient, but statement (2) alone is not sufficient to answer the question asked (B) Statement (2) ALONE is sufficient, but statement (1) alone is not sufficient to answer the question asked

GovernmentAdda.com

(C) BOTH statements (1) and (2) TOGETHER are sufficient to answer the question asked, but NEITHER statement ALONE is sufficient (D) EACH statement ALONE is sufficient to answer the question asked (E) Statements (1) and (2) TOGETHER are NOT sufficient to answer the question asked, and additional data are needed

208

HOMEWORK Challenge Questions

116. If y is an odd integer and the product of x and y equals 222, what is the value of x? (1) x is a prime number. (2) y is a 3 digit number.

(A) Statement (1) ALONE is sufficient, but statement (2) alone is not sufficient to answer the question asked (B) Statement (2) ALONE is sufficient, but statement (1) alone is not sufficient to answer the question asked (C) BOTH statements (1) and (2) TOGETHER are sufficient to answer the question asked, but NEITHER statement ALONE is sufficient (D) EACH statement ALONE is sufficient to answer the question asked (E) Statements (1) and (2) TOGETHER are NOT sufficient to answer the question asked, and additional data are needed

GovernmentAdda.com

HOMEWORK

209

117. If x and y are positive integers, is

x an integer? y

(1) Every factor of y is also a factor of x. (2) Every factor of x is also a factor of y.

(A) Statement (1) ALONE is sufficient, but statement (2) alone is not sufficient to answer the question asked (B) Statement (2) ALONE is sufficient, but statement (1) alone is not sufficient to answer the question asked (C) BOTH statements (1) and (2) TOGETHER are sufficient to answer the question asked, but NEITHER statement ALONE is sufficient (D) EACH statement ALONE is sufficient to answer the question asked (E) Statements (1) and (2) TOGETHER are NOT sufficient to answer the question asked, and additional data are needed

GovernmentAdda.com

210

HOMEWORK Challenge Questions

118. The infinite sequence a1, a2, …, an, … is such that a1 = x, a2 = y, a3 = z, a4 = 3 and an = an-4 for n > 4. What is the sum of the first 98 terms of the sequence? (1) x = 5 (2) y + z = 2

(A) Statement (1) ALONE is sufficient, but statement (2) alone is not sufficient to answer the question asked (B) Statement (2) ALONE is sufficient, but statement (1) alone is not sufficient to answer the question asked (C) BOTH statements (1) and (2) TOGETHER are sufficient to answer the question asked, but NEITHER statement ALONE is sufficient (D) EACH statement ALONE is sufficient to answer the question asked

GovernmentAdda.com (E) Statements (1) and (2) TOGETHER are NOT sufficient to answer the question asked, and additional data are needed

HOMEWORK

211

119. If x is a positive integer less than 10, is 14,743 + x prime? x is odd. 2 (2) x2 = 36 (1)

(A) Statement (1) ALONE is sufficient, but statement (2) alone is not sufficient to answer the question asked (B) Statement (2) ALONE is sufficient, but statement (1) alone is not sufficient to answer the question asked (C) BOTH statements (1) and (2) TOGETHER are sufficient to answer the question asked, but NEITHER statement ALONE is sufficient (D) EACH statement ALONE is sufficient to answer the question asked, (E) Statements (1) and (2) TOGETHER are NOT sufficient to answer the question asked, and additional data are needed

GovernmentAdda.com

212

HOMEWORK Challenge Questions

120. Steve works at an apple orchard and is paid by the bushel for the apples he harvests each day. If Steve harvests 42 bushels or less per day, he is paid y dollars per bushel. If Steve harvests more than 42 bushels per day, he receives y dollars per bushel for the first 42 bushels and is paid 1.5 times that amount for each additional bushel. How many bushels of apples did Steve harvest yesterday? (1) Yesterday, Steve was paid $180 for the apples that he harvested. (2) Today, Steve harvested 10 more bushels of apples than yesterday and was paid a total $240.

(A) Statement (1) ALONE is sufficient, but statement (2) alone is not sufficient to answer the question asked (B) Statement (2) ALONE is sufficient, but statement (1) alone is not sufficient to answer the question asked

GovernmentAdda.com (C) BOTH statements (1) and (2) TOGETHER are sufficient to answer the question asked, but NEITHER statement ALONE is sufficient (D) EACH statement ALONE is sufficient to answer the question asked (E) Statements (1) and (2) TOGETHER are NOT sufficient to answer the question asked, and additional data are needed

HOMEWORK

213

121. If yz ≠ 0, is

x–y+z x y x < – – ? 2z 2z 2z y

x 1 <– y 2 (2) xy < 0 (1)

(A) Statement (1) ALONE is sufficient, but statement (2) alone is not sufficient to answer the question asked (B) Statement (2) ALONE is sufficient, but statement (1) alone is not sufficient to answer the question asked (C) BOTH statements (1) and (2) TOGETHER are sufficient to answer the question asked, but NEITHER statement ALONE is sufficient (D) EACH statement ALONE is sufficient to answer the question asked (E) Statements (1) and (2) TOGETHER are NOT sufficient to answer the question asked, and additional data are needed

GovernmentAdda.com

214

HOMEWORK Challenge Questions

122. In the first hour of a bake sale, students sold either chocolate chip cookies, which sold for $1.30, or brownies, which sold for $1.50. What was the ratio of chocolate chip cookies sold to brownies sold during that hour? (1) The average price for the items sold during that hour was $1.42. (2) The total price for all items sold during that hour was $14.20.

(A) Statement (1) ALONE is sufficient, but statement (2) alone is not sufficient to answer the question asked (B) Statement (2) ALONE is sufficient, but statement (1) alone is not sufficient to answer the question asked (C) BOTH statements (1) and (2) TOGETHER are sufficient to answer the question asked, but NEITHER statement ALONE is sufficient (D) EACH statement ALONE is sufficient to answer the question asked

GovernmentAdda.com (E) Statements (1) and (2) TOGETHER are NOT sufficient to answer the question asked, and additional data are needed

HOMEWORK

215

123. Set T is a finite set of positive consecutive multiples of 14. How many of these integers are also multiples of 21? (1) Set T consists of 30 integers. (2) The smallest integer in set T is a multiple of 21.

(A) Statement (1) ALONE is sufficient, but statement (2) alone is not sufficient to answer the question asked (B) Statement (2) ALONE is sufficient, but statement (1) alone is not sufficient to answer the question asked (C) BOTH statements (1) and (2) TOGETHER are sufficient to answer the question asked, but NEITHER statement ALONE is sufficient (D) EACH statement ALONE is sufficient to answer the question asked (E) Statements (1) and (2) TOGETHER are NOT sufficient to answer the question asked, and additional data are needed

GovernmentAdda.com

216

HOMEWORK Challenge Questions

124. If r, s, and t are positive integers and rst = 343, what is the value of t? (1) r < s < t (2) rs = 7

(A) Statement (1) ALONE is sufficient, but statement (2) alone is not sufficient to answer the question asked (B) Statement (2) ALONE is sufficient, but statement (1) alone is not sufficient to answer the question asked (C) BOTH statements (1) and (2) TOGETHER are sufficient to answer the question asked, but NEITHER statement ALONE is sufficient (D) EACH statement ALONE is sufficient to answer the question asked (E) Statements (1) and (2) TOGETHER are NOT sufficient to answer the question asked, and additional data are needed

GovernmentAdda.com

HOMEWORK

217

125. If n and a are positive integers, what is the units digit of n4a+2 – n8a? (1) n = 3 (2) a is odd.

(A) Statement (1) ALONE is sufficient, but statement (2) alone is not sufficient to answer the question asked (B) Statement (2) ALONE is sufficient, but statement (1) alone is not sufficient to answer the question asked (C) BOTH statements (1) and (2) TOGETHER are sufficient to answer the question asked, but NEITHER statement ALONE is sufficient (D) EACH statement ALONE is sufficient to answer the question asked (E) Statements (1) and (2) TOGETHER are NOT sufficient to answer the question asked, and additional data are needed

GovernmentAdda.com

218

HOMEWORK Challenge Questions

GovernmentAdda.com

HOMEWORK

219

GovernmentAdda.com

220

SOLUTIONS

SOLUTIONS 18. C Question Type: What Is the Value? This question asks how many cars will fit in the parking lot after it is made larger. Given information from the question stem: The number of cars in each row will be decreased from 40 to 30. However, there will be fewer total rows in the lot as well. In order to answer the question “How many total cars will fit in the entire lot?” you will need to know how many rows times how many cars per row. You already know how many cars per row (30), so this question will hinge on whether you can determine the number of total rows in the new lot. Statement 1: The new configuration will have 3 fewer rows than the current lot does. However there is no indication of how many rows were originally there. This is not sufficient. Therefore choices A and D are eliminated. Statement 2: Before the change there are 10 rows of cars. Yet this does not tell you how many rows there will be after the change. Remember: Answer choice B is that “Statement 2 ALONE is sufficient.” This problem is a good opportunity to practice looking at Statement 2 alone and forgetting what you learned from Statement 1. Since Statement 2 alone is also not sufficient, choice B is eliminated.

GovernmentAdda.com Together: The statements allow for the calculation that there are 7 rows after the changes to the parking lot. 7 rows @ 30 cars per row = 210 cars. This is sufficient and the answer is C. 19. E Question Type: Yes/No. This question asks whether William is taller than Jane. Given information from the question stem: The question asks whether William is taller than Jane. If you can show that William is DEFINITELY taller than Jane or that William is DEFINITELY NOT taller than Jane, the statement (or statements) is sufficient.

SOLUTIONS

Statement 1: You can write this as W > A (William taller than Anna). This is not sufficient because we do not know anything yet about Jane. Choices A and D are eliminated. Statement 2: You can write this as A < J (Anna is not as tall as Jane). This is not sufficient because it provides no information about William. Choice B is eliminated.

221

Together: Taken together, you can write the statements as: W > A, and J > A Note that it’s important to get the inequalities pointed in the same direction (Statement 2 was given as A < J, but that also means that J > A). This helps you to notice that both William and Jane are taller than Anna, but that is all you can conclude. You still have no way to establish the relationship between William and Jane. Because you do not have enough information even taken together, the answer is E. 20. B Question Type: Yes/No. This type asks whether

11x 23

is less than

7x 13

.

Given information from the question stem: You can think of the question stem 11x 7x 11 1 7 as asking whether 23 is less than 13 . 23 is slightly less than 2 and 13 is slightly 1 more than 2 . Since both numbers are multiplied by the same variable, x, it would 11x 7x appear that the answer will be “yes” 23 is less than 13 , at least if x is a positive integer. However, if x is a negative number or a fraction it may not be the case.

GovernmentAdda.com

Statement 1: “x is an integer.” With this information you know that x is not a 7x fraction. However, x could still be negative. If x is negative then 13 will be “more negative” (further to the left on the number line) and therefore a smaller 11x number than 23 . This would give the answer of “no.” You can also get a “yes” from Statement 1 if x is a positive integer. For this reason Statement 1 is not sufficient. Eliminate choices A and D. Statement 2: “x is positive.” This eliminates the possibility of x being negative (and also eliminates the possibility of x being 0, which would also have given a “no” answer to the question). However, Statement 2 does allow for x to be a noninteger—a fraction or decimal. Does it matter if x is a non-integer? It does not. 1 7x 11x Even if x is 2 , 13 will still be a larger number than 23 . The number property at work in this question is “positive/negative” and the fact that x cannot be negative and cannot be zero is enough to ensure that the answer to the question is always 11x 7x “yes” 23 is smaller than 13 . This statement is sufficient and the answer is B. The important takeaways from this question: When a problem deals with the combination of inequalities and variables and/or when a statement specifically defines a variable as positive (>0) or negative (<0), be certain to check positive/ negative number properties as part of your analysis.

222

SOLUTIONS

21. E Question Type: What Is the Value? This question asks how old Fred will be in y years. Given information from the question stem: This question is asking for a specific number for the age that Fred will be y years from now. You can represent that as F + y. Statement 1: Doris is 12 years older than Fred can be written “F + 12 = D.” This is clearly not sufficient to determine Fred’s age now or in the future. Eliminate choices A and D. Statement 2: The sum of the ages of Doris and Fred is y years can be written “F + D = y.” This statement only consists of three variables without any actual numbers, so it cannot give us a specific age for Doris or for Fred. This is not sufficient, so eliminate choice B. Together: If you take both statements together you can use the second equation “F+D = y” and substitute “F+12” for D. This yields the equation “F + F + 12 = y.” Without knowing either what y is or what F is this equation cannot be solved, as it is one equation with two variables. The answer is E.

GovernmentAdda.com

22. B Question Type: What Is the Value? This question asks for the value of the quadratic 4x2 – 3x + 1.

Given information from the question stem: This question asks for a specific value for the quadratic “4x2 – 3x + 1.” You might be tempted to factor this equation right away, but it is best to look at the statements first. You may need to factor but the question may be just as valuable in its current form.

SOLUTIONS

Statement 2: For this question Statement 2 is the clearer of the two statements, so we can evaluate this one first. Statement 2: Using conceptual understanding you know that if you have values for each variable in an equation or inequality that you will be able to arrive at a single value and that this information will be sufficient. In this case the only variable in the quadratic in the question stem is “x” and we have a value for x given by Statement 2. There is no need to insert x into the quadratic; Statement 2 is automatically sufficient. The answer will either be B or D. Statement 1: Statement 1 provides two values for x. Since either “(x – 1) = 0” or “(x+2) = 0,” x must either equal 1 or –2. At this point you have more than one value for x and this statement is “not sufficient” because it will yield more than one solution: If x = 1, the result of the quadratic will be 2; if x = –2, the result of the quadratic will be 23. The answer is B.

223

23. E Question Type: What Is the Value? This question asks for the final score of a game between two teams, X and Y. Given information from the question stem: This question is seeking a specific value for the final score in game between team X and team Y. Statement 1: This statement tells you that the first-half score for X + 14 = the Second Half Score for X. This statement is clearly not sufficient as it does not give you a score for either team X or Team Y. Eliminate choices A and D. Statement 2: This statement tells you that during the first half the two teams scored equal points but that during the second half team y scored 4 more points. This statement also does not give you any fixed value for the score of X or of Y. Also not sufficient: Eliminate choice B. Together: Even taken together these two statements do not yield enough information. Neither statement gives you an actual number. Playing Devil’s Advocate, you could simply choose 10 as the number of points each team scored in the first half. Then add 14 points to X’s first-half score to get the second-half score according to Statement 1, so x would score 10+24 = 34 points. Y scored 4 more than X (Statement 2) so that would mean 38 points for Y. But there is no reason why “10” should be the first-half score any more than “20” or “100.” This question allows for an infinite variety of final scores. The information is not sufficient. The answer is E.

GovernmentAdda.com

24. C Type of Question: Yes/No. This question asks whether the value of b (symbol) c is greater than 10. Given information from the question stem: This question asks you to determine “yes” or “no” is b symbol C is greater than 10. Now you are told that the symbol is an “operation” and you might think that this means addition, subtraction, multiplication, and division only. But it does not. The symbol can really require you to take any mathematical action. It could have you add a million and then subtract 2 million. So do not make any assumptions here. Statement 1: This statement tells you what the symbol represents. It says that for any two numbers “x symbol y” you square each of the numbers and then add them. This tells you what action the symbol requires. However, unless you know what the numbers are that you are squaring, you cannot get a definite answer. For example, if x and y each equal 1, then answer will be “no” because

224

SOLUTIONS

12 + 12 = 2. But if x and y are each large numbers like 10, then the answer will be “yes” because 102 + 102 =200. Because you don’t know the value of b and c, this statement is not sufficient. Statement 2: This statement gives you values for b and c. It may seem that this statement is sufficient because each standard operation—multiplication, division, addition, subtraction—will yield a result of less than 10. However, as mentioned above, the symbol does not necessarily represent one of these four operations and Statement 1 makes this possibility very clear. Since you do not know what to do with the values for b and c, you cannot determine what b symbol c will equal. This statement is not sufficient. Together: Taken together the two statements provide you with the values for b and c as well as the action that you should perform with these numbers. You take 22 + 12 which equals 3, so the answer to the question is “no” b symbol c is not larger than 10. Together the statements are sufficient. The answer is C. 25. B Question Type: What Is the Value? This question asks for the measure of angle ADC.

GovernmentAdda.com Given information from the question stem: This problem asks you for a specific number for the measure of angle ADC. The question stem indicates that this figure is a parallelogram. This information is valuable because there are properties of parallelograms that are very useful in determining the measure of an angle. The most important property for this problem is that opposite angles have equal measures. So angles BAD and BCD are equal, as are angles ABC and ADC. If you know the measure of any of the four angles then you can determine the measure of the other three. Statement 1: This statement does not give the measure of any of the angles. This statement is clearly not sufficient on its own. Answers A and D are eliminated.

SOLUTIONS

Statement 2: This statement gives you the measure of angle BCD. This angle measures 70 degrees. That means that angle BAD also measures 70 degrees. Since the angles of a parallelogram measure 360 in total that leaves 220 degrees for the remaining two angles. Since these angles are equal that means that angle ADC (as well as angle ABC) equals 110 degrees. Since this statement gives you a single measure for angle ADC it is sufficient. Answer Choice B is correct.

225

26. B Question Type: Yes/No. This question asks whether the area of circle X is greater than the area of circle Y. Given information from the question stem: This question asks you to determine whether circular region x has a larger area than does circular region y. Statement 2 seems easier to evaluate so you can start there. Statement 2: This statement gives you the relationship between the radius of circle x and that of circle y. Circle x has the larger radius. Since the formula for the area of a circle is πr2 and π is a constant, it is true that a larger radius definitely means a larger area of the circle. This statement is sufficient. The answer is either B or D. Statement 1: This statement compares the circumference of circles x and y with that of circle z. The statement can be rewritten as “circumference x > circumference z” and “circumference y > circumference z.” This does not allow for a comparison of circles x and y; it simply indicates that each is larger than circle z. This information is not sufficient. The correct answer is B.

GovernmentAdda.com

27. D Question Type: What Is the Value? This question asks you for a specific value for the side of a triangle—side JL. Given information from the question stem/diagram: The figure drawn above the question indicates that this is a 45 – 45 – 90, or isosceles right triangle. The — ratio of the sides of a 45–45–90 is x:x:x√ 2 . Note also that the specific angles are labeled, making JK the hypotenuse, and sides JL and KL the shorter sides. — Statement 1: This statement gives the measure of the hypotenuse JK = 2√ 2 . With this information you can determine that both JL and KL will equal 2. This is sufficient. The answer is either A or D. Statement 2: This statement gives the measure of side KL = 2. Side JL will have the same measure since this is an isosceles triangle. This statement is sufficient. The answer is D.

226

SOLUTIONS

28. E Question Type: Yes/No This question asks you to determine which of two shipping options is less expensive. This can be treated like a “yes/no” (Is Option A cheaper?) question since there are only two options. You may not even need to know the exact cost of each method as long as you can compare the two. Given information from the question stem: Company A’s charge is based on the distance of the delivery and Company B’s charge is based on the weight of the package. Statement 1: Statement 1 provides the scale at which the charges are applied. Company A charges a fixed amount plus .01 per mile and Company B charges a fixed amount plus $8.50 per pound. This statement provides necessary information to calculate the charges but does not provide the distance nor the weight, so this statement is not sufficient. The answer will be B, C, or E. Statement 2: This statement provides the distance that the package will travel. However it does not provide any way of calculating or even comparing the charges for companies A and B. This statement is not sufficient. Eliminate choice B. Together: Taken together the statements give an amount that Company A will charge for the package. Statement 1 provides the scale and Statement 2 the distance. However, the weight of the package remains unknown so that the charge from Company B cannot be calculated nor compared to Company A’s charge. These statements are not sufficient. The answer is E.

GovernmentAdda.com 29. D Question Type: What Is the Value? This question asks for a specific number indicating the length—in words—of the letter.

SOLUTIONS

Statement 1: This statement can be translated into the following equation: “(t – 2) 80 = t (60)” where t is the number of minutes it takes to type the letter at a rate of 60 words per minute. This equation then simplifies to “80t – 160 = 60t.” Subtract 60t from both sides and add 160 to each, and the equation becomes “20t = 160” and “t = 8.” Therefore 8 • 60 gives a total of 480 words. Of course the equation did not have to be solved. As soon as you got (t – 2) 80 = 60t you should see that this is a linear equation with just one variable and it can be solved for a single value. This is sufficient. The answer is either A or D. Statement 2: This statement can be translated into an equation using the formula w = rt (work = rate • time). The work is ½ of the letter. The rate is 40 words per 1 1 minute and the time is 6 minutes. The equation is “ 2 w = 40 • 6” or “ 2 w = 240” and “w = 480.” This statement is sufficient as well; the answer is D.

227

30. D Question Type: Yes/No This is a yes/no question asking you to determine whether the positive square root of a is an integer. Statement 1: This statement tells you that “b” is an integer and that “a = b4.” This means that the “square root of a” would equal “b2.” This tells you that “square root of a” must be an integer since multiplying an integer times itself always results in another integer. If “b” were, for example, 3, then “a” would be 81 and “the square root of a” would equal 9. There is no need to plug in numbers for this statement; it is sufficient. If b is an integer then so is b2. The answer will be either A or D. Statement 2: “a = 81”; the positive square root of “a” would then be 9. This is an integer. This statement is sufficient. The correct answer is D. 31. C Question Type: What Is the Value? This question asks you for a specific value for the price of the longer (28 inch) necklace. Given information from the question stem: Necklaces come in two lengths and there is no “volume discount” since all necklaces of the same length sell for the same price regardless of number purchased.

GovernmentAdda.com

Statement 1: Translated this becomes “x + y = $68” where x is 18 inch and y is 28 inch. This is two variables and just one equation. There is no way to distribute the $68 total. This is not sufficient. Eliminate choices A and D. Statement 2: Translated this becomes “2x + y = $96.” This is just like Statement 1 in that you have two variables and only one equation. This, too, is not sufficient. Eliminate choice B. Together: Taken together the statements give you two linear equations with the same two variables. As long as these equations are distinct this will be sufficient. To ensure you come to an answer, you can use the Elimination Method for multiple variables, subtracting Statement 1’s equations from Statement 2’s: 2x + y = 96 -x – y = -68 x = 28, and you can plug that in to the first equation to solve for y. If 28 + y = 68, then y = 40. These statements together allow you to solve for y, the price in question, so the correct answer is C.

228

SOLUTIONS

32. A Question Type: Yes/No This question asks: “Is x>y?” Statement 1: x = y + 2; this statement clearly indicates that x is larger than y. In words, this says “x is two greater than y.” This is sufficient. The correct answer is either A or D. x

Statement 2: 2 = y – 1; this statement can be investigated using the Play Devil’s Advocate technique. Inserting a value for x—for example, x = 2—will yield a 2 value for y. If x = 2, then y also equals 2, since 2 = 2 – 1. Since x and y are equal then the answer is “no,” x is not greater than y. Now you would be looking for values that give you a “yes,” values that make x > y and that work with Statement 2. Inserting a larger value for x, for example “x = 6”, will give you a value of “y=4” 6 since 2 = 4–1. In this case 6 > 4, and therefore x > y, yielding an answer of “yes.” Since this statement allows for values of x and y that give you answers of “yes” and of “no,” this statement is not consistent and is therefore not sufficient. The correct answer is A. 33. A Question Type: Yes/No. “is the average of a and b less than 40?”

GovernmentAdda.com Given information from the question stem: You’re dealing with averages. One helpful tool to use when asked about the average of a group of numbers is to use the formula “Average * number = total.” In this case if the average is 40 and A+B there are two numbers (a and b), then the question is really asking is 2 < 40 or is “A +B < 80.”

Statement 1: if the average of 3a and 3b is 117 this means that “3a + 3b = 234.” If you divide by sides by 3 then you get “a + b = 78.” This is sufficient to answer the question. “Yes, the average of a + b < 40.” This statement is sufficient. The correct answer is either A or D.

SOLUTIONS

Note: It would not be necessary to actually determine the value for “a + b.” Once you know that you can divide both sides of the equation “3a + 3b = 234” by 3 and get a value for “a + b” then you do not need to actually make the calculation. Either the average will be less than 40 or it will not be, but in either case you will have a consistent answer. Statement 2: This statement does not provide any fixed values for a or b and so it cannot be determined if the average of a + b will be less than 40. This statement is not sufficient. The answer is A.

229

34. E Question Type: What Is the Value? This question asks you for a specific number indicating the amount of money that Penny earned during week W. Given information from the question stem: Penny is paid hourly and works part time, and has two different employers, Company X and Company Y. Therefore, her total weekly pay will be (Hours Worked at X)(Pay Rate at X) + (Hours Worked at Y)(Pay Rate at Y). Statement 1: This statement provides the hourly rates for her work for the two companies, X and Y. However it does not provide a total number of hours worked for either company. This is not sufficient. Answers A and D are eliminated. Statement 2: Penny worked 60 hours total during week W. This means that she worked 60 hours for X and Y combined. However, nothing is indicated as to the rate that she was paid for this work. This statement is not sufficient and answer B is eliminated. Together: Taken together, you have the total number of hours worked and the hourly rate for each of the two companies. Yet, it is not possible to know how to divide the 60 hours between X and Y. Since she is paid a different amount by each of the two companies it is necessary to know precisely how many hours she worked for each. Therefore, the statements together are still not sufficient. The correct answer is E.

GovernmentAdda.com 35. A Question Type: Yes/No This question asks whether x is a positive number.

Statement 1: “-5x – 3 > -2x” can be simplified by adding 3 to both sides to yield “ -5x > -2x + 3” and then adding 2x to both sides to yield “-3x > 3.” Then, to isolate x, divide both sides by 3 and –x > 1. To change the sign of x, multiply both sides by a negative 1 and the statement becomes “x < -1.” This algebraic manipulation has answered the question directly. “No, x is a not positive. x is a negative number.” This statement is sufficient. The answer is either A or D. Statement 2: x2 is positive allows for x to be either positive or negative. Since the answer is not consistent this statement is not sufficient. The correct answer is A. One important takeaway here: Even though Statement 1 provided the answer “no,” it was indeed sufficient. A definite “no” means you have sufficient information to arrive at one exact answer. So while your instincts might tell you “No  Eliminate,” remember that “no” means “sufficient.”

230

SOLUTIONS

36. A Question Type: What Is the Value? This question asks you to provide a specific number of paintings in the museum. Statement 1: 4 new paintings will increase the number of paintings in the museum by 10%. This can be written as “4 = 0.1x” with x equaling the total number of paintings. Clearly this will be sufficient to provide a specific answer to the question and there is no need to actually solve this equation. This statement is sufficient. The answer will be either A or D. Statement 2: The ratio of impressionist to non-impressionist paintings is 3:2. This information does not allow you to arrive at a specific number since you do not know either the number of impressionist or the number of non-impressionist paintings. Not sufficient. The answer is A. 37. B Question Type: What Is the Value? This question asks for a specific number of positive integers that are factors of n. Given information from the question stem: n itself is an integer.

GovernmentAdda.com Statement 1: “n is the product of a prime and a non-prime positive integer.” From the Data Sufficiency toolkit, you can use “conceptual understanding” on this statement and see that it allows for too many possibilities for you to get a single answer, or you can “Play Devil’s Advocate” and use a couple of small numbers to prove to yourself that this statement is not sufficient. For example, you can simply choose 3 as the prime number and 4 as the nonprime positive integer. The result, 12, has six factors: 1, 2, 3, 4, 6, 12. However, if you choose 3 as the prime and 1,000,000,000 as the non-prime numbers then the result will clearly have many more than six factors. This statement yields more than one answer and is therefore not sufficient. The answer is B, C, or E.

SOLUTIONS

Statement 2: “n and 20 are each divisible by the same number of positive integers.” This statement is clearly sufficient. It is saying that when you determine the number of positive factors of 20 then you have the answer as n has the same number of factors. It is not necessary for you to determine the number of positive integers that 20 is divisible by; you know that it will be just one number and that is sufficient. The answer is B.

231

38. E Question Type: What Is the Value? This problem asks you to find the remainder when k is divided by j. Given information from the question stem: j and k are integers. Statement 1: This problem offers a great opportunity to pick numbers/Play Devil’s Advocate.” The easiest numbers to pick are likely prime numbers, since you know that by definition all primes have the same number of factors (two). 5 divided by 3 leaves a remainder of 2, so now you want to find an answer that gives you something other than 2. You want to Play Devil’s Advocate to arrive at an alternative answer to show that the statement is not sufficient. Typically the best way to do that is to try numbers with different properties, and 2—the only even prime—gives you just that opportunity. 3 divided by 2 leaves a remainder of 1, and since we’ve found two different answers (2 and 1) to this specific-value question, we can prove that this statement is not sufficient. Statement 2: Notice this: The numbers we chose for Statement 1 are also possibilities given Statement 2. So we can use the same numbers to show that Statement 2 alone is not sufficient, and that even taken together the statements are not sufficient. Accordingly, the correct answer is E.

GovernmentAdda.com

39. C Question Type: What Is the Value? This question asks you for the number of units that line A produced in 2006. Given information from the question stem: Production on line A increased by 5% from 2006 to 2007 and production on line B increased by 10%. Statement 1: This statement can be written as “A + B = 100,000 units.” This information is not sufficient since you do not know how many of the 100,000 units were produced by line A and how many by line B. Not sufficient. Eliminate answers A and D. Statement 2: This statement tells you the combined number of units in 2007. This can be written as 1.05A + 1.10B = 107,500. Similar to the above, this statement is not sufficient alone as you have two variables and one equation. Eliminate choice B as well. Together: Now you have two distinct linear equations with two variables and you can solve. There is no need to actually solve for A since you are certain that you can. Together the statements are sufficient and the answer is C.

232

SOLUTIONS

40. E Question Type: Yes/No This question asks whether the average sale for month J was at least 20 percent higher than for month M. Mathematically, that’s: J > 1.2M? Given information from the question stem: The average sale for month M was “d” dollars. Statement 1: Total revenue from sales in month M was $3,500. This statement provides no information as to the number of sales so there is no way to determine the size of the average sale. This statement is not sufficient alone. Eliminate choices A and D. Statement 2: Total revenue from sales in month J was $6,000. This statement is very similar to Statement 1 and is not sufficient for the same reason. Eliminate choice B. Together: Even together these two statements do not provide you the average sale for either month J or month M. These two statements are not sufficient together. The correct answer is E.

GovernmentAdda.com

41. C Question Type: Yes/No The question asks whether xy is divisible by 9. Another way to say this is “Is xy a multiple of 9?” and, further breaking down the question, “Does xy have prime factors that include 3 • 3?” By manipulating the question using conceptual understanding you are often in a stronger position to evaluate the answer choices. Now you know that if the information can guarantee that 32 is a factor of xy then that information is sufficient. Given information from the question stem: x and y are positive integers.

SOLUTIONS

Statement 1: The product xy is divisible by 6. This tells you that 3 and 2 are factors of xy. However this statement is not sufficient alone. Conceptual understanding will tell you that if 3 is a factor of xy then it is possible that 9 is also a factor of xy. However it is equally possible that 9 is not a factor of xy. If you are in doubt you can use numbers and Play Devil’s Advocate. xy could equal 6 or 12, both of which are divisible by 6 as the statement requires and are not multiples of 9. This would yield an answer of “no.” xy could also equal 36, which is a multiple of 6 and also of 9. This would yield the answer of “yes.” Since yes and no answers are both possible this statement is not consistent and is therefore not sufficient. Eliminate choices A and D. Statement 2: x and y are perfect squares. Conceptually this is clearly not

233

sufficient. While each of the number must be a perfect square this statement does not guarantee that the result will even be a multiple of 3, much less of 9. Eliminate choice B. Together: Statement 1 tells you that xy must include the prime factors of 2 and 3 (in order to be a multiple of 6). Since Statement 2 requires x and y to be perfect squares the only way to have a 2 and a 3 as prime factors of x and y is to have 22 and 32 as factors of x and y. In fact, the smallest values for x and y are 4 and 9. So the answer to the question “Is xy divisible by 9?” is “yes.” Together the statements yield one consistent answer and the correct answer is C. 42. D Question Type: What Is the Value? This question asks for a specific number of executives. Given information from the question stem: Bonuses were paid at $125 for executives and $75 to non-executives. There are 100 non-executive employees. Statement 1: Total employees = 120. This statement is very clearly sufficient. It is as simple as 120 – 100 = 20 (executives). Sufficient. The correct answer is either A or D.

GovernmentAdda.com

Statement 2: This statement gives you a total for the amount paid in bonuses. The total of $10,000 is divided between executives and non-executives at the rates given in the question stem. This information is sufficient. You can create the following equation: “$75(100) + $125E = $10,000.” If you solve the equation E will yield the number of executives. However there is no need to solve. Since it is very clear that you will get just one answer this statement is also sufficient. The correct answer is D.

43. C Question Type: What Is the value? This question asks for the value of y – x. Given information from the question stem: x = y2. While it may not be important for this question, you should train yourself to notice things like “x cannot be negative” here, as y2 has to be > 0. Statement 1: x = 4. This is not sufficient because even taken with the fact above it still results in two answers. Substitute x in the equation from the facts and you get 4 = y2. So y can equal 2 or – 2. These two values for y will give you different answers for “y – x” and are therefore not specific enough. You need a single value in order to be sufficient and you do not have one. The answer will either be B, C, or E.

234

SOLUTIONS

Statement 2: x + y = 2. This is not sufficient even if taken together with the information from the question stem. A quick example using Play Devil’s Advocate might be the simplest way to show that this is not sufficient. “x = 1 and y = 1”is compatible with the given information and with Statement 2. However, so is one of our examples from statement 1 “x = 4 and y = -2.” After all 4 + -2 = 2. These different values for x and y will result in different answers to “y – x = ?” This statement is also not sufficient. The answer is either C or E. Together: Taken together the statements are sufficient. Only one set of values will work with the two statements and the fact. “x = 4 and y = -2.” The correct answer is C. One important takeaway from this question: Do not assume that a variable is positive! When people answer this question incorrectly it’s usually because they only considered 2 as a value (and not -2) for y when they saw Statement 1. When a statement gives you a solution for y2, remember that y could be either positive or negative. “Forgetting about negatives” is one of the easiest (and most common) mistakes to make on Data Sufficiency problems. 44. C Question Type: What Is the Value? This question asks for the specific number of show dogs.

GovernmentAdda.com Statement 1: Exactly 3 of the dogs won prizes of at least $500. This statement is not sufficient alone. There is no way for you to determine the total number of show dogs. Eliminate choices A and D. Statement 2: 40% of the dogs have not won a prize of $500 or more. This statement is also not sufficient alone. As in Statement 1, there is no way to determine the total number of dogs. Eliminate choice B. Together: One of the best ways to solve a GMAT problem is to find a way to state the same thing in two different ways, allowing you to create an equation. You know from Statement 1 that 3 dogs have won prizes of $500 or more and you know from Statement 2 that 60% of the dogs have won a prize of $500 or more. So that means “3 dogs = 60% of the total dogs” or “3 = .6d” She has 5 show dogs total. This information is sufficient and the correct answer is C.

SOLUTIONS

235

45. D Question Type: What Is the Value? This question asks for a specific number for the base memory capacity of the computer. Given information from the question stem: Each M-type unit will increase base capacity by 3 megabytes. Statement 1: 2 M-types units will increase the computer’s base memory by 300%. From the fact we know that this is a total addition of 6 megabytes. So “6 = 3x,” where x is the current total base memory of the computer in megabytes. “x = 2.” This statement is sufficient, so the answer is either A or D. Statement 2: The ratio of total memory capacity after adding 2 M-types units to the total memory capacity after adding 1 M-type unit is “1.6: 1.” Therefore 6 (which is 2M-types units) + x (the current base memory) = 1.6 (3 +x). Therefore “6 + x = 4.8 + 1.6x.” You can see that you have just one variable and you will be able to solve for x, so there is no need to do so. This is sufficient as well. The correct answer is D. 46. D Question Type: Yes/No This question asks whether a2 > 3a – b4 .

GovernmentAdda.com

Statement 1: 3a – b4 = -5. This statement may not appear sufficient at first. You cannot manipulate it algebraically to mirror the question stem as might be the first impulse. However, the important thing is that “3a – b4 equals a negative number.” Remember that a2 cannot be less than zero, as a squared number cannot be negative. The answer, then, is “yes,” as you can absolutely conclude that a2 will be greater than -5. The correct answer is either A or D. Statement 2: a > 5 and b > 0. Again, this statement may not appear to be sufficient. It does not give specific values for a or b. However, if you “Just Do It” and plug in the numbers then you will see that it is sufficient. A good strategy for these “greater than” statements is to use the actual numbers given. Although the statement says a > 5 and b > 0 you can just use 5 and 0 in the inequality. The inequality becomes “25 > 15 – 0?” The answer is clearly “yes”: 25 > 15. And as you increase both “a” and “b” the result becomes a stronger “yes.” For example, if “a = 6 and b = 2” the inequality is “36 > 18 – 16?” Conceptually it looks like this, so long as a is greater than 3 then a2 will be greater than 3a. Whatever number “b” is can only take away from the 3a it cannot add to it. In fact, Statement 2 gives more information than strictly necessary; “a > 3” would be sufficient. The correct answer is D.

236

SOLUTIONS

47. D Question Type: Yes/No This question asks whether x is even. Given information from the question stem: x is a positive integer. Statement 1: 5x is even. If the product of multiplication is even then at least one of the numbers being multiplied must be even. 5 is odd so x must be even. This is sufficient. The answer is either A or D. Keep in mind here the importance of leveraging the “x is an integer” portion of the question stem. Had that not 2 2 been in place, a number like  5 would have also worked here, as 5( 5 ) = 2, an even integer. But the question limits you to only using integers, so you are allowed to employ even/odd number property concepts here. Statement 2: x – 5 is odd. In order to get an odd result from the addition or subtraction of integers you need to have one odd and one even number. Since 5 is odd x must be even. This is sufficient as well. The correct answer is D. 48. E Question Type: Yes/No Is x2 > x3?

GovernmentAdda.com Given information from the question stem: While no new information is explicitly provided, the question structure—exponents and inequalities—should already get your mind thinking in terms of positive/negative and integer/non-integer number properties. This problem will almost certainly test your ability to consider different types of numbers (negatives, fractions).

Statement 1: x3 is positive. Given that x3 is positive, x must be positive as well. However, x could still be a non-integer between 0 and 1. For example, if x =  1 1 1  then x2 = 4 and x3 =  8 . In this case x2 > x3 so that would be a “yes.” If x is a 2 positive integer like 2 then x3 > x2, which provides the answer “no.” Since this statement allows for both a “yes” and a “no” it is not sufficient. Eliminate choices A and D.

SOLUTIONS

Statement 2: x does not equal 1. Given the possibility of fractions between 1 and 0, this statement is clearly not sufficient on its own. Each of the values that was used in the analysis of Statement 1 works with this statement as well. This is also not sufficient. Eliminate choice B. Together: The same values work for both statements, giving you a “yes” and a “no” even with both statements together. If x is between 0 and 1 the answer is yes. If x is greater than 1 the answer is no. The correct answer choice is E.

237

49. B Question Type: What Is the Value? What percent of males in a certain poll voted for coffee Y? Given information from the question stem: A total of 1,600 people voted. Each voted for X or Y. No one voted for both. And because you’re dealing with people/ votes, the numbers for X and Y must be integers. Statement 1: 40% of females voted for brand X. This statement is not sufficient to tell you what percent of males voted for brand X. Not sufficient. The answer will either be B, C, or E. Statement 2: 65% of the males voted for X. This statement might appear to be insufficient as it does not even talk about brand Y. However, since you know that there are only two brands, X and Y, the percent of males who chose X + the percent who chose Y must equal 100%. So if 65% chose X, the other 35% voted for Y. And since the question asks for the percent, not the actual number of males, 35% is sufficient. The correct answer is B. 50. C Question Type: What Is the Value? What is the month when Jeff received the highest bill: June, July, or August?

GovernmentAdda.com

Given information from the question stem: The average for the 3 months was $75. It is a good idea to just multiply this right away to get the total amount for the 3 months = $225 total. Statement 1: The lowest bill was received in July. This means that the July bill must be lower than $75. However, the highest bill could still be for June or August. Since this is not specific it is not sufficient. Eliminate choices A and D. Statement 2: The total for July and August was $160. This gives you the exact amount for June. $225 – 160 = $65 for June. Be careful not to include the information from Statement 1 at this point. You do not know whether July or August is the highest bill. This statement alone is not sufficient. Eliminate choice B. Together: Taken together you know that the June bill is $65 and that the July bill is for less than this amount. Since the total amount of all three has to be $225, and June and July account for less than $130, the August bill must account for at least $95, and therefore be the highest total. This is sufficient. The answer is C.

238

SOLUTIONS

51. E Question Type: Yes/No What is the value of two-digit integer x? Given information from the question stem: x is a two-digit integer. For this question you can start with Statement 2 since that statement is more obviously not sufficient. Statement 2: x is divisible by 9. In other words, x is a multiple of 9. However, there are many two-digit multiples of 9, such as 18, 27, 36, etc. This is clearly not sufficient. Eliminate choices B and D. Statement 1: The product of the two digits is 14. What two-digit integers have a product of 14? 27 has a product of 14, as does 72. Because there is more than one value of x this statement is also not sufficient. Eliminate choice A. Together: Taken together the two values used in Statement 1 still work here. Both 27 and 72 are multiples of 9, and both multiply to 14. Taken together the statements are not sufficient. The correct answer is E. 52. D Question Type: What Is the Value? This question asks for the specific value of x – y.

GovernmentAdda.com Given information from the question stem: x + y = 6.

Statement 1: x2 – y2 = 12. You should recognize this as the difference of squares: x2 – y2 = (x + y)(x – y). Factoring this will make it fit the question and the information that you already have. You can rewrite this to say “(x + y) (x – y) = 12.” Given that you know that “x +y = 6” then it must be the case that “x – y = 2.” This statement is sufficient. The answer is either A or D.

SOLUTIONS

Statement 2: 2y + x = 8. From this statement and from the equation in the question stem, you can determine that y = 2. Either the substitution or elimination method (for solving multivariable equations) will work, but for this purpose let’s use elimination. If you take the equation 2y + x = 8 and subtract the equation x + y = 6, all that is left is y = 2. If you plug this number back into the equation in the fact, then x + 2 = 6. Therefore x = 4 and x – y = 2. This statement agrees with Statement 1 and is also sufficient. The correct answer is D.

239

53. A Question Type: What Is the Value? This question asks for the specific number of steps between the first and second floor. Given information from the question stem: The two floors are 9 vertical feet apart. 3

Statement 1: Each step is  4 foot high. This statement allows you to create the 3 equation “9 =  4 x” where x is the number of steps. This is sufficient to tell you the number of steps needed to climb 9 vertical feet. The correct answer is A or D. Statement 2: Each step is 1 foot wide. This statement is less helpful since you do not know the overall width of the entire staircase; you only know from the fact that it is 9 feet tall. There is no way to determine the number of steps from this information. This statement is not sufficient. The correct answer is A. And notice the trap here: In classic “Why Are You Here?” form, Statement 2 is designed to make you think you might need it. Since each stair has a height and a width, you might feel better knowing all dimensions. But you do not need the width; whether this is a long, processional staircase or a ladder, it needs to extend 9 vertical feet.

GovernmentAdda.com

54. C Question Type: Yes/No Is 2a – b + c > a – b – 2c? It will help to simplify this question through algebraic manipulation before moving forward. If you add b to both sides, you can streamline the question to “Is 2a + c > a – 2c?” If you then subtract a from both sides and subtract c from both sides, you get “Is a > - 3c?” This is important because now the statements are much easier to analyze. Given information from the question stem: a, b, and c are integers. Statement 1: a is positive. This is not sufficient given that “c” could still be a negative number and -3c could be larger than a. Eliminate choices A and D. Statement 2: c is positive. This statement indicates that the right side of the inequality is negative since c is positive. However, a could also be a negative number, and potentially more negative than c and therefore smaller. If a is a positive number it will clearly be larger than -3c. So the inequality might be true or not true. Therefore the statement is not sufficient. Eliminate choice B. Together: Taken together you know that the left side of the inequality will be positive and the right side will be negative. Any positive number is larger than any negative number, so “a > -3c.” Alternatively, you could add 3c to both sides to make the question “Is a + 3c > 0?” Clearly if a and c are both positive, this sum will be positive. Together the statements are sufficient. The correct answer is C.

240

SOLUTIONS

55. B Question Type: What Is the Value? This question asks for the dollar expenditure on hops. Given information from the question stem: H + Y + M = $100,000 (H = amount spent on hops, Y = amount spent on yeast, M = amount spent on malt). Statement 1: Y = 1.2M. This would be sufficient if the equation in the facts only included these two variables. But with three variables in the above equation, there is no way to distribute the $100,000. This statement is not sufficient. Eliminate choices A and D. Statement 2: Y + M = H. This statement seems at first glance to be like Statement 1. However, it contains some hidden information. You’re solving for H, and this statement allows you to plug in “H” for “Y + M” in the equation Y + M + H = 100,000. If you replace Y + M with H, you have H + H = 100,000, so H must be 50,000. Note here the opportunity to Think Like the Testmaker. C is too easy an answer, as it adds a third equation to the three-variable set-up. The testmaker is begging you to pick C, so if you initially went for that answer you should ask yourself whether you can Leverage Assets, and here if you spend a bit more time manipulating Statement 2 you should see that it is sufficient. Because Statement 2 is sufficient, the correct answer is B.

GovernmentAdda.com 56. A Question Type: What Is the Value? This statement asks for a specific value for the 5 4 2 fraction x y z . 2 4 2 zyx Given information from the question stem: The product xyz does not equal zero. This means that none of these three is equal to zero. At this point, it’s also helpful to streamline the given fraction. Since the same variables appear in both the numerator and denominator, you can “combine like terms” by factoring out terms with common bases: x: The numerator includes x5 and the denominator includes x2. You can divide out the x2 term to yield x3 in the numerator.

SOLUTIONS

y: Both the numerator and denominator include y4, so this term divides out entirely. z: Both the numerator and denominator include z2, so this term divides out entirely. So what you’re left with is the question “What is x3?”—a much easier question to answer!

241

Statement 1: x = 1. This is clearly sufficient given the work that you have done on the question stem. If x = 1 the entire question = 1. This is sufficient and the answer is either A or D. Statement 2: y = 1. This is not actually helpful, given that both the numerator and the denominator have y4 and this can therefore be eliminated. It literally does not matter what the value of y is. This statement is not sufficient. The correct answer is A. 57. D Question Type: What Is the Value? This question asks for a specific value for the number of liters (x) of concentrated cleaner in the solution. Given information from the question stem: 5 liters of solution = x liters of concentrated cleaner + y liters of water. Succinctly: x + y = 5. x 1 Statement 1: y = 8 . This statement means that the ratio of x:y = 1:8. That means 1 1 5 that 9 of the solution is concentrated cleaner. So 9 • 5 = x. So x = 9 of a liter. This is sufficient. The answer is either A or D. x 1 Statement 2: 5 = 9 . You may see immediately that this is sufficient, as it is one linear equation with one variable—and that variable, x, is what you’re asked to solve for. To see the math, you can cross-multiply to get 9x = 5. And if you divide 5 both sides by 9 you get x = 9 of a liter. This is also sufficient. The correct answer is D.

GovernmentAdda.com

58. E Question Type: What Is the Value? This question asks for the length of the shortest of three pieces of rope (s). Given information from the question stem: The three pieces of rope are each different lengths. (Let s = shortest, m= middle, and L = longest.) Statement 1: m + L = 12. This statement is not sufficient because you do not have the total length of rope and cannot say what the length of the shortest piece is exactly. Not sufficient. Eliminate choices A and D. Statement 2: s + m = 11. This statement is not sufficient because, although you have a total that includes the shorter piece, you have one equation with two variables and no way to say what the specific length of the shorter piece is. Eliminate choice B. Together: Together you have two equations, one from each statement. They are: m + L = 12 and s + m = 11. This allows you to determine that the longest piece

242

SOLUTIONS

is 1 foot longer than the shortest piece so that L – s = 1. However, you are still not able to fix an exact value to s, m, or L. The longest piece is longer than 6 feet and the shortest piece is shorter than 5.5 feet, but since the values do not have to be integers and since each of the three pieces can vary there is no way to get the specific values. These statements are not sufficient. The correct answer is E. 59. B Question Type: Yes/No Is the number 3(a + b) + c divisible by 3? Given information: a, b and c are integers. You can Use Conceptual Knowledge here. Given that each of these numbers is an integer, 3(a + b) will be a multiple of 3 regardless of the values of a and b. Therefore you are simply focused on the question “Is c a multiple of 3?” If c is a multiple of 3 then the whole thing is divisible by 3. If c is not a multiple of 3 then the whole thing is not divisible by 3. Statement 1: a + b is not divisible by 3. As mentioned above the value of a + b does not matter. So this statement is not sufficient alone. This is the advantage of doing your work early. When you truly understand the question you can work through the statements very efficiently. This statement is not sufficient. The correct answer is B, C, or E.

GovernmentAdda.com Statement 2: c is divisible by 3. This is an exact answer to the question that you developed by manipulating the question stem. If c is a multiple of 3 and since we know from algebra that 3(a + b) will also be a multiple of 3 then we also know that adding two multiples of 3 gives a result that is a multiple of 3. Try it. Make c = any multiple of 3. Then make a and b any integers at all. 3(a + b) will also be a multiple of 3 and “3(a + b) + c = a multiple of 3.” The answer is always “yes.” It is consistent and sufficient. The correct answer is B. 60. A Question Type: What Is the Value? This question asks for a specific value for x – y.

SOLUTIONS

Statement 1: x – y = y – x. This statement can be simplified to “2x = 2y” and finally to “x = y.” You can then finish the Algebraic Manipulation by subtracting y from both sides and you get “x – y = 0” This clearly gives you a specific value for “x – y.” This statement is sufficient. The answer is either A or D. Statement 2: x – y = x2 – y2. The left side of the equation is the difference of squares so we can rewrite this as “x – y = (x + y) (x – y).” This statement leaves you with two possibilities. In order for this to be true, either “x – y = 0” so that both sides of the equation = 0, or x + y = 1 so that both sides of the equation =

243

x – y. These two different possibilities allow infinite values for “x – y” since if “x + y = 1” “x – y” could equal anything. This statement is not sufficient. The correct answer is A. 61.

B Question Type: Yes/No Is n an integer less than 5? Notice that the question requires two components to provide the answer “yes”: n must be an integer and n must be less than 5. Statement 1: 5n is a positive integer. This statement is not sufficient. If you Play Devil’s Advocate you can get both integers and non-integers for n. For example, 1 n could equal 1, which is an integer less than 5. Or n could =   and 5n = 1. This 5 1 would give you a “no” since   is not an integer. 5 You could also manipulate the “algebra” on this one. If 5n = positive integer, (positive integer) 5 then n =   That allows n to equal something like   (which is 1) 5 5 1 or   (which is not an integer). 5 This statement is not sufficient. Eliminate choices A and D. n n Statement 2:   is a positive integer. In order for   to be a positive integer, n 5 5 5 must be at least equal to 5 and must be a multiple of 5. For example,   = 1 and  5 10 = 2, etc. So this statement is sufficient because the answer is “no.” n is an 5 integer but is not less than 5.

GovernmentAdda.com

n Again, you could also perform the “algebra” by saying that   = (positive integer). 5 This means that n = 5(positive integer). And since the lowest positive integer is 1, then n at a minimum is 5(1). Accordingly, n cannot be less than 5. Because you can answer definitively “no” to this question then this statement is sufficient and the answer is B. 62. E Question Type: What Is the Value? What percentage of the combined total of groups A and B is comprised of men? Given information from the question stem: Every man is either a member of Group A or Group B or both. So in Venn Diagram terms, the “Neither” category is empty, but there may very well be men who are in “Both” groups. 20% of Group A is men and 65% of Group B is men. Statement 1: Group A contains 50 people. So the number of men in group A is 50(20%). So 10 men are in Group A. This is not sufficient since you do not know the number of men in Group B. Eliminate choices A and D.

244

SOLUTIONS

Statement 2: Group B contains 100 people. The number of men in Group B is 100(65%) or 65 men. This is not sufficient since it does not tell you about group A. Eliminate choice B. Together: It may appear that these two statements are sufficient when taken together. There are 10 men from Group A and 65 from Group B for a total of 75 men, right? Not necessarily. It is possible for some of the people in groups A and B to be in both groups. It is possible, for example that all 10 men in Group A are also in Group B. So instead of 75 men total there could be as few as 65. This means that you cannot get one specific value for the percentage of men. This is not sufficient. So the answer is E. 63. A Question Type: Yes/No This question asks whether b > a. Given information from the question stem: a and b are consecutive negative integers. On this question you can start with Statement 2 first since that statement is easier to evaluate.

GovernmentAdda.com Statement 2: b is an odd number. It should not matter if b is odd when determining if “b > a.” You can quickly Play Devil’s Advocate with a couple of consecutive negative integers. b could = -5 and a could equal -6. In this case b > a. Of course b could also equal -5 and a could equal -4, in which case b is not greater than a. Since you can get both a “yes” and a “no” for this statement it is not sufficient. Eliminate choices B and D (since you did Statement 2 first.)

SOLUTIONS

Statement 1: a + 1 and b – 1 are consecutive negative integers. You already know from the facts that a and b are consecutive negative integers. Now when you add one to “a” and subtract one from “b” you also have consecutive negative integers. Playing Devil’s Advocate is a good strategy here to take the abstraction of variables and make it concrete with numbers. You can use the numbers from above: Let b = -5 and a = -6. Adding 1 to a and subtracting 1 from b changes it to b = -6 and a = -5, meaning that these are still consecutive negative numbers. So b can be greater (less negative) than a. Can the opposite be true? Can a be larger than b? Try using the reverse numbers. Make a = -5 and b = -6. Now when you add one to a and subtract one from b the result is a = -4 and b = - 7, so this will not work with Statement 2. a cannot be larger than b; b must be the larger number. The statement is sufficient. The correct answer is A.

245

64. D Question Type: Yes/No The question asks: “Is x2 = xy?” For this question it is easier to begin with Statement 2. Statement 2: x = y. If x = y then x2 will automatically equal xy. So this statement is sufficient. The answer is either B or D. Statement 1: x2 + y2 = - (x2 – y2). Generally with algebra you want to eliminate parentheses, so in this case you’ll want to multiply each term in (x2 – y2) by -1 to distribute that negative sign; the statement becomes x2 + y2 = -x2 + y2. If you then manipulate this statement algebraically by adding x2 to both sides and by subtracting y2 from both sides you get 2x2 = 0. For this to be true x must 0. And if x = 0, then 02 = 0(y) by definition—both sides of the equation would equal 0. This statement is also sufficient and the correct answer is D. 65. B Question Type: Yes/No This question asks: “Is the tenths digit of the decimal equal to 4?”

GovernmentAdda.com Given information from the question stem: 2.5 < x < 4.5.

Statement 1: x + 0.006 < 4.5. If you subtract 0.006 from both sides you get: x < 4.494. This statement is not very helpful in narrowing down the value for x. You already know from the fact that 2.5 < x < 4.5 and now you can say that 2.5 < x < 4.494. This leaves many possible values for x and many possibilities for the tenths digit of that decimal. This statement is not sufficient. Eliminate choices A and D. Statement 2: x + 0.06 > 4.5. Subtracting 0.06 from both sides makes the statement x > 4.44 combined with the fact above you get 4.44 < x < 4.5. Since x must be greater than 4.44 and less than 4.5, the tenths digit of x will definitely be a 4. This statement is sufficient to answer the question with a consistent “yes.” The correct answer is B. 66. D Question Type: What Is the Value? This question asks for a specific value for b. Given information from the question stem or diagram: The marks on the number line shown are equally spaced, and a and b represent specific marks where a is 3 3 marks above 0 and b is 7 marks above 0. Therefore, a is   as far away from 0 as 3 7 b is, so you can represent that mathematically as a =   b. 7

246

SOLUTIONS

1 Statement 1: a =   . If the numbers are evenly spaced then you can simply take 4 3 1 1 the fact that you discerned above that a = (   )b and replace a with   . So   =  7 4 4 3 b. This is sufficient to yield a specific value for b. The answer is either A or D. 7 1 Statement 2: b – a =   . Again, you can lean on the information you derived from 3 3 1 the question stem and diagram. If a = (   )b, then b – a =   can be rewritten as 7 3 3 1 4 1 b – (   )b =   , or (   )b =   . This allows you to solve for b. Like Statement 1, 7 3 7 3 this is sufficient to get a specific value for b. The correct answer is D. 67. E 2 Question Type: Yes/No Did more than   of the ice cream made in July contain 5 over 20% milk? Given information from the question stem: The factory makes two flavors of ice cream: chocolate and caramel. Each flavor comes in over 20% milk and under 20% milk. This means that there are four total possibilities: chocolate over 20%, chocolate under 20%, caramel over 20%, and caramel under 20%. You are looking for the combined fraction of the ice cream that is over 20% milk. Statement 1: 80% of the caramel ice cream contained over 20% milk. This statement is not sufficient because you know nothing about the chocolate ice cream. Eliminate choices A and D.

GovernmentAdda.com Statement 2: 1,000 gallons of chocolate ice cream contained over 20% milk. This statement does not tell you how many gallons of chocolate ice cream were made a as whole. It is not as useful as statement 1 and is certainly not sufficient alone. Eliminate choice B. 4 Together: You know that   of the caramel ice cream is over 20% milk, but you 5 do not know what the proportion is for the chocolate ice cream. The number “1,000 gallons” from Statement 2 is not particularly useful in this situation. Even with both statements together there is not enough information. The correct answer is E. 68. E Question Type: Yes/No “Is y greater than 0?” Or, in other words, “Is y positive?”

SOLUTIONS

Given information from the question stem: x does not equal 0. Statement 1: xy = 24. In order to get a positive result from multiplication you either need two positive or two negative numbers. It is possible for both x and y to be positive (giving you a “yes”) and for both to be negative (giving you a “no”).

247

This statement allows both yes and no and is not consistent and not sufficient. The answer will be choice B, C, or E. Statement 2: x – y = 5. This can be restated as “x = y + 5.” This is not sufficient alone since y could be a negative number. For example, x = -5 and y = -10. This statement allows x to be positive as well as negative so it is not sufficient. Eliminate choice B. Together: Taken together the statements tell you that “x is 5 greater than y” and that “xy = 24.” The question is, can you Play Devil’s Advocate and find both positive and negative values for y that work with both statements. First x and y can be positive. Let x = 8 and y = 3. This satisfies both statements and yields a “yes, y is positive.” Simply making x = -3 and y = -8 also works with both statements and gives you a “no, y is not positive.” Even taken together these statements are not sufficient. The correct answer is E. 69. A Question Type: Yes/No This question asks: “Is a less than b?” a 4 Given information from the question stem:   =   . If you cross-multiply this 3 b becomes ab = 12.

GovernmentAdda.com

Statement 1: b is greater than or equal to 4. If this is the case then both a and b must be positive and a must be less than or equal to 3. This means that “yes, a is less than b.” This statement is sufficient. The correct answer is either A or D. Statement 2: b is less than or equal to 5. This statement is not as helpful as Statement 1. First, you do not know if a and b are positive or negative. a could be -3 and b could be -4, which does work with the facts above and would yield a “no, a is not less than b.” Even if a and b are positive then a could equal 3 and b equal 4 in which case a would still be larger than b. Of course, b can be larger than a as well if b = 4 and a = 3. So Statement 2 allows for both a “yes” and a “no” and is not consistent and not sufficient. The correct answer is A.

70. C Question Type: Yes/ No. The question asks: “Does x = 2?” Given information in the question stem or diagram: No important information is given in the question stem. Statement 1: x2 – 3x + 2 = 0. In this case it is best for you to “Just Do It” and see what values you get for x. Factoring the quadratic yields (x – 2 )(x – 1 ) = 0. So x = 2 or x = 1. This gives you both a “yes” and a “no” answer to the question so it is

248

SOLUTIONS

not sufficient. Eliminate choices A and D. Statement 2: x2 – x – 2 = 0. Factoring this quadratic gives you (x – 2) (x + 1) = 0. So x either equals 2 or -1. This statement is also not sufficient alone since those values give a “no” and a “yes.” Eliminate choice B. Together: When taking these statements together you need to see if there is only one value that is allowed by BOTH of the statements. x = 2 is the only value that both statements allow so the correct answer is C. Note: This is only hard if you make a mistake with the factoring or get confused about what constitutes sufficiency when considering the two statements together. This would be trickier if one statement gave two values other than 2. Then that statement would be sufficient as the answer would be a definitive NO, even if there were two possibilities from the statement. As it stands, neither alone is sufficient and the answer is C. 71.

B Question Type: Yes/No This question asks: “Is y < 2.4?” Statement 1: y < 2.5. This is not sufficient because y could still be greater than 2.4 but less than 2.5, such as 2.49, and the answer would be “no.” However y could of course be equal to 1, and then the answer would be “yes.” This statement is not sufficient so you can eliminate answers A and D.

GovernmentAdda.com Statement 2: y < 2.3. If y is less than 2.3 then it must always be less than 2.4, so this statement will always give a “yes” answer to the question. The correct answer is thus B. Note: While not particularly difficult, this question is here to make sure people understand the importance of limits in this type of inequality question. Statement 1 sets a limit that creates a “maybe” answer but Statement 2 sets a limit that will create a definitive “yes” to the given question. 72. D Question Type: What Is the Value? This question asks for a specific value for the price per microwave (P).

SOLUTIONS

Given information in the question stem or diagram: The total price of M equally priced microwaves is $18,000 so MP = $18,000, where P is the price per microwave. Note: To get a definite value for P, you simply need some way to find the number of total microwaves or another unique equation with only M and P in it. Statement 1: This statement allows you to create the equation: M (P + 10) = 18,000 + 5,000, which simplifies to MP + 10M = $23,000. This equation, when combined with the equation in the facts, allows you to solve for M and P because you have

249

two unique equations with two unknowns. Since MP = 18,000 you can substitute directly for MP into the equation above to see that 18,000 + 10M = 23,000 and M= 50. This means P = 360. This is sufficient so the correct answer is either A or D. Note: This is much more difficult to conceptualize than it is to consider algebraically. The key is to make sure that the equation given in the statement is indeed unique from the one given in the question stem. When doing algebra on this type of question, you do not need to solve completely, but make sure that you can “see the finish line” and get exactly one value for P. Statement 2: This statement allows you to create the equation: M (P – 9) = 18,000 – .25(18,000), which simplifies to MP – 9M = 13,500. Again since you can substitute so easily for MP from the question stem, there is no need to do the rest of the math. Once you know that you will get just one value for M, which allows you to solve for P, then you know that the information is sufficient. The correct answer is D. 73. E Question Type: What Is the Value? The question asks for the price of the most expensive bicycle.

GovernmentAdda.com

Given information in the question stem or diagram: Total number of bicycles = 6; average price per bicycle = $1,000. This means that the total amount sold by the store was $6,000. Statement 1: The median price was $1,000. This means that the mean equals the median. However, this does not allow you to find the price of the most expensive bicycle. Depending on the prices of the other five bicycles, the most expensive one can have a wide range of prices. For instance, there could have been three $800 bikes and three $1,200 bikes that sold, and the most expensive would be $1,200. Or there could have been one $1,300 bike, four $1,000 bikes, and one $700 bike. In each case you have met the conditions in the statement and the question stem but gotten multiple values. Not sufficient, so eliminate choices A and D. Statement 2: The range of prices was $600. This information allows you to narrow down the range of values for the most expensive bicycle but does not allow you to establish this price. It is clearly not sufficient by itself so you can eliminate choice B. Together: It is much more difficult to make the choice between C and E on this question. With all the information in the question stem and the statements, you know that the six bicycles have a median and an average of $1,000 and a range of $600. Given this, many people will assume that the most expensive bike must be $1,300 and the least $700. For example, if the price of the most expensive is

250

SOLUTIONS

$1,300 that makes the price of the least expensive $700 and, as shown in the example previously, the middle four could all be $1,000. However, you could also make the most expensive be $1,400. Make the lowest price one equal to $800 (range has to be $600) and the remaining four could be $800, $1,000, $1,000, and $1,000. This means that multiple values will still work for the highest priced bicycle and so the information not sufficient. The correct answer is E. Note: This problem is about avoiding assumptions. People assume that because the mean and the median are the same, that it must be an evenly spaced set. This is not true as shown above. If you are dealing with an evenly spaced set, the mean always equals the median, but if the mean equals the median in a set, this does not prove it is evenly spaced. Remember to always prove E on a question like this by Playing Devil’s Advocate and doing your best to find multiple possibilities for the highest priced item. 74. A Question Type: What Is the Value? This question asks for the value of j – k. Given information in the question stem or diagram: There is no important information given in the question stem. Note: When you are asked for a difference versus an actual variable, you typically need less information than you think.

GovernmentAdda.com For this question you should start with Statement 2 because it is easier and clearly insufficient.

Statement 2: n = 3. n is not even part of the question so this statement is clearly not sufficient but creates the classic “Why Are You Here?” scenario. You need to consider very carefully why this statement might be important when considering Statement 1. Maybe you will need it or maybe you won’t! Eliminate choices B and D.

SOLUTIONS

Statement 1: 2j + 4n = 2k + 4n – 6. This statement also appears to be insufficient but you should do some manipulation to make sure that you cannot isolate j – k. Without Statement 2 you have no value for n and it appears that you need it in this equation. However, a careful look at the equation shows that the same element “+ 4n” is on both sides and will cancel out. You can simply subtract 4n from both sides and n disappears leaving you with 2j – 2k = -6. Now you can simplify by dividing both sides by 2 to see that j – k = -3. While you do not know the values of either j or k, you do know the difference and this is sufficient alone. The correct answer is A. Note: This is classic example of the “Why Are You Here?—Temptation” construct. Remember: Before you ever pick answer C in a case like this, make absolutely sure that you really need the information in the other statement.

251

75. E. Question Type: What Is the Value? This question asks for the value of x. Given information in the question stem or diagram: 20x + 30y = 280. This should be simplified before you move to the statements. Divide all terms by 10 to see that this equation is really 2x + 3y = 28. Note: On the GMAT, you should always simplify algebraically whenever you are given information that is not in simplified form. Statement 1: 4x = 56 – 6y. This statement appears to give you a second linear equation, which would be sufficient to solve for x when combined with the given information. However, before you do anything with this equation you should simplify it as you did with the question. If you add 6y to both sides of the equation and divide by 2, you see that this equation is really the same as the given equation: 2x + 3y = 28. These are not two distinct linear equations so this information is not sufficient. Eliminate choices A and D. Note: Be ready for this set-up in Data Sufficiency algebra questions: the “disguised” second equation that is really the same as the first.

GovernmentAdda.com

Statement 2: y2 = 16 or y = 4 or -4. The equation gives two different values for y, but it is important to glance at the original equation to make sure that the different values for y will actually result in different values for x. In this case they do as the original equation is a linear equation. Since this information gives you two different values for x it is not sufficient. Together: Given that the equation from Statement 1 is the same as in the given information, the analysis of the two statements together is the same as for Statement 2. Statement 1 is no help and since Statement 2 is not sufficient alone, the answer is E.

76. D. Question Type: Yes/No. This question asks: “Is y2 divisible by 4.” Given information in the question stem or diagram: y is an integer. Statement 1: y is even. With a conceptual understanding of factors, you know that if y is an even number then y2 will have to be a multiple of 4. If you want to quickly test this you can use some small even numbers. For example, if y = 2, then y2 = 4; yes, that is divisible by 4. If y = 4, then y2 = 16; this is also divisible by 4. Even 0, which is an even number, will work. 02 = 0; and yes, 0 is divisible by 4. This confirms what we already knew from our conceptual understanding: an even number when squared is a multiple of 4. The answer is either A or D.

252

SOLUTIONS

Statement 2: y3 is divisible by 4. For some, this may not seem sufficient at first glance. y3 might be divisible by 4 while y2 is not. For example, y could be the cube root of 4. However, if you leverage properly the information in the question stem (always so important!) then you know that “y is an integer.” So y cannot be the cube root of 4; it must be an integer. Once you have established that y must be an integer then the only way for y3 to be divisible by 4 is for y to be an even number. In other words, it gives you the same information that you already found sufficient in Statement 1. Thus, the correct answer is D. Note: This is another good example of “When one statement is easy, the other is hard/counterintuitive.” The first statement is relatively simple so you should be extra careful analyzing the second statement. As is often the case, the key with Statement 2 is to make sure you properly leverage the given information. 77. B. Question Type: Yes/ No. The question asks: “Is xy < 8?” Statement 1: x < 2 and y < 4. At first glance this might appear sufficient. When you multiply a positive number smaller than 2 by a positive number smaller than 4, the result is smaller than 8; however, you must consider negative numbers. x could equal -3, for example, and y could be -5. Together their product would equal 15. So this statement is not sufficient and you can eliminate choices A and D. Note: This statement is all about avoiding assumptions. Do not assume that x and y are positive as that is not given in the question stem!

GovernmentAdda.com Statement 2: 0 < x < ½ and y2 < 225. This means that x is a positive number between 0 and ½ and 15 > y > -15. When you multiply a positive number that is smaller than ½ by a number that is between 15 and -15, the result must be smaller than 8. Even if you took the values of ½ and 15 the product is 7.5. Unlike in Statement 1, it is okay when y is negative because x must be positive, so the product would be negative and less than 8. With this information, you know that xy must be smaller than 8 and this statement is sufficient. The answer is B.

SOLUTIONS

253

78. B Question Type: Yes/No. The question asks: “Are lines TR and PQ parallel?” Given information in the question stem or diagram: QRS is a straight line and QR = PR. If you leverage the information in the question stem, you can add several facts: Shape QPR is a triangle, and if sides QR and PR are equal, then it is an isosceles triangle with angle QPR and angle PQR each the same measure. If you call those angles each x, then the third angle – angle QRP will measure 180 – x. Statement 1: Length PQ = Length PR. This tells you that not only is the triangle QPR isosceles, it is equilateral, with all angles 60 degrees. So now you know just about everything you need to know about the triangle, but keep in mind that you know nothing about line TR. This statement is not sufficient, so eliminate choices A and D. Statement 2: Line TR bisects angle PRS. Here is where leveraging the information in the question stem—and labeling your diagram—will pay off. You already know that line QRS is a straight line, meaning that the three angles that meet at point R (angle QRP, angle PRT, and angle TRS) will have to sum to 180. And you know that angles PQR and QPR are equal, which you’ve labeled as x in the diagram. If line TR bisects angle PRS, then the two angles PRT and TRS are equal. You can call them each y, and then your diagram should now look like:

GovernmentAdda.com P

T

x

x Q

180 – 2x y y R

S

And since 180 – 2x + 2y has to equal 180 (for line QRS to be a straight line), then 2x has to equal 2y, and x = y, so angle TRS equals PQR, meaning that lines PQ and TR are, indeed, parallel. Statement 2 is sufficient, so the answer is B. Note this important takeaway from this problem: Geometry-based DS questions are quite often candidates for the Leverage Assets strategy. Be certain to label diagrams with all known information, as Geometry has a lot of potential for seemingly small pieces of information to leverage into sufficiency.

254

SOLUTIONS

79.

A Question Type: What Is the Value? This question asks for number of bolts that contain BOTH cotton and wool. Given information in the question stem or diagram: 20 bolts of fabric total; each is 100% cotton, or 100% wool, or a mixture of BOTH cotton and wool. This means that neither = 0 and you can set up your Venn diagram equation before you go to the statements: Total = Cotton (C) + Wool (W) – Both (B) or 20 = Cotton + Wool – Both. Remember that C and W represent the total sets not those with “Only Wool” or “Only Cotton.” Statement 1: Of the 20 bolts, 18 contain some wool and 14 contain some cotton. Plugging this information into the equation above you see that this statement is sufficient. 20 = 18 + 14 – B. B = 12. Or logically you know that there are 20 total bolts but 18 + 14 = 32. This means the difference must be the number of bolts that are double counted (those with both). The answer is either A or D. Statement 2: Of the 20 bolts 6 are 100% wool. As in any Venn Diagram problem, when you are given “only” information such as this, you must use a diagram or think logically (the equation above is for total set information, not “only” information). If “Wool Only” = 6 then you know that the total of cotton bolts (including those with both) is 14. However there is no way to determine the value of “Both” from this information so it is not sufficient. The correct answer is A. Note: Almost all Venn Diagram problems hinge on reading carefully and distinguishing between “Total Set” information and “Only” information. In this problem, it is the difference between bolts containing Wool or Cotton (which includes both) and bolts containing 100% Wool or Cotton or Only Wool or Cotton (which does not include both).

GovernmentAdda.com 80. D Question Type: Yes/No. This question asks: “Is x < 3?”

SOLUTIONS

Given information in the question stem or diagram: x, y, and z are the lengths of three sides of a triangle. Note: Even before going to the statements you should recognize that the third side rule of triangles will almost certainly come into play: The third side (i.e., any side) of a triangle is always between the sum and the difference of the other two sides. Statement 1: z = y + 3 OR z – y = 3. This statement does not give you the value of any of the sides and does not allow you to determine the actual value of side x. However, using the third side rule you know that x must be greater than the difference between z and y, which is given as 3 in this statement. Therefore x

255

must be greater than 3 and you can answer this question with a definitive “no” from this information. The information is sufficient so the correct answer is either A and D. Note: People tend to underleverage this piece of information because it does not lock down a value for x (which is not necessary) and because people have forgotten the third side rule. Statement 2: y = 3 and z = 6. This gives very similar information to statement 1 and also allows you to apply the third side rule. Since the explicit values are given you can see that x must be greater than the difference between x and y (which is again 3). Statement 2 is also sufficient and the answer is thus D. Note: The answer could never be C on this question. While this is not a common construct (and thus not mentioned in the lesson portion of this book), it is one that you will see occasionally. The first piece of information (z = y + 3) is automatically known from the second, so putting them together could never help you! It is either answer A, B, D, or E whenever one statement is known from the other, so C should be eliminated automatically here. 81. E Question Type: Yes/No. The question asks: “Is x an even number?”

GovernmentAdda.com Given information in the question stem or diagram: x is a positive integer.

Statement 1: If x is divided by 3, the remainder is 2. This statement can be easily addressed by considering numbers and Playing Devil’s Advocate. First attempt to get an even number from Statement 1: The number 8 gives you a remainder of 2 when divided by 3. This is an even number so you have a “yes” answer. Next choose a number exactly 3 higher or lower than 8: 11 is an odd number and also gives a remainder of 2 when divided by 3. So 11 works but gives you a “no” answer. This statement is not sufficient, so eliminate choices A and D. Statement 2: If x is divided by 5, the remainder is 2. This statement can be treated 12 in the same way as Statement 1. First get a “yes” answer. If x = 12, then   = 2 5 remainder 2. Next choose a number exactly 5 above or below 12. If x = 7, then 7   = 1 remainder 2. So this works as well and gives you a “no” answer. This 5 statement is also not sufficient, so eliminate choice B. Together: Taken together you know that x must be 2 greater than a multiple of both 5 and 3. So x must yield a remainder of 2 when divided by 15. So 17 17 17 would work.   = 3 remainder 2, and   = 5 remainder 2. 17 is an odd number 5 3 so you have a “no” answer. Now add 15 to 17 to get 32. 32 is an even number 32 32 and will give you a “yes” answer.   = 6 remainder 2 and   = 10 remainder 5 3 2. So 32 satisfies both statements as well. Note: Number picking is of course not

256

SOLUTIONS

required to answer this question. With a good conceptual understanding of the number line, and quotient/remainder problems in particular, you can see quite clearly that the answer will be E in this problem. As is often the case, Arithmetic problems in Data Sufficiency can be quickly and reliably answered if you have a deep conceptual understanding of the subject matter. 82. C Question Type: What Is the Value? This question asks for the surface area before the changes were made. Given information in the question stem or diagram: The surface of the rectangular field was changed so that one dimension was reduced by 10 feet and the other dimension was increased by 20 feet. Statement 1: After the changes, the surface area was 2,500 square feet. This statement is not sufficient because it does not allow you to determine the dimensions of the field either before or after the change. For example, the new dimensions might be 25 • 100, but which number was reduced by 10 and which was increased by 20? And 25 • 100 is just one example. Many other dimensions are possible for a 2,500 square foot field so this statement is clearly not sufficient. Eliminate choices A and D.

GovernmentAdda.com Statement 2: The length and width of the field were equal after the changes. Don’t be tempted to bring information from Statement 1! You do not yet have that information. Statement 2 alone is not sufficient because just knowing that the dimensions are equal does not tell you what those dimensions are. They could be 30 • 30 or 40 • 40, etc. Eliminate choice B.

SOLUTIONS

Together: Taken together you know that the area after the changes is 2,500, which means that after the changes the length and width are each 50 since they must be equal. One of these—either the length or width—used to be 10 longer and one used to be 20 shorter. It is not possible to determine which was longer and which was shorter, but it does not matter. It is because the sides are equal after the changes that you can solve this problem. So the dimensions before the change are 60 • 30 or 1,800 square feet. Together the statements give you a specific number and are sufficient. The answer is C. Note: This problem is designed to make you carry information between statements. Always be careful about polluting your mind from one of the statements, and make sure that you are only using the information from the question stem and the statement you are assessing.

257

83. D Question Type: What Is the Value? This question asks for the ratio of revenue from sales of x to revenues from sales of y. So “revenue x: revenue y = ?” Given information in the question stem or diagram: Item x sells for 10% less than item y. Statement 2 is very clear on this problem so you should begin there. Statement 2: The store’s revenue from sales of X = $6,000 and the revenue from sales of y = $8,000. This statement is clearly sufficient. The ratio is simply 6,000:8,000 or 3:4. The answer is either B or D. Note: This is a very good example of the construct “When one statement is very easy, the other will be hard and counterintuitive.” Everyone knows this is sufficient so there will be something tricky about Statement 1. Slow down and make sure you are not over- or underleveraging the given information in Statement 1. Statement 1: The store sells 20% more of item y than item x. This statement, when combined with the given information, is sufficient. 20% more of item y is sold at 10% higher prices. Revenue = (Unit Price)(Number of Units). Conceptual understanding will allow you to stop at this point and realize that although you do not have the actual number of units or the actual price per unit, you can calculate the ratio of revenue from x to that of revenue from y. This is sufficient and the answer is D. Note: Most people under-leverage this information because it does not give any hard number values. Using your hint from the other statement to dig deeper here, you should eventually realize that the percentage data is sufficient.

GovernmentAdda.com

84. A Question Type: What Is the Value? This question asks for the percent change in profits of company x from the second to third year. Statement 1: The increase in profits was the same from year 1 to 2 as from year 1 to 3. This is a tricky statement that most people under-leverage. At first glance, it appears insufficient but a careful analysis shows that this information proves that there was no change from year 2 to year 3. The only way that the change could be the same from year 1 to the year 2 as from year 1 to year 3 is with a 0% change from year 2 to year 3. This statement requires that you read very carefully and leverage the information given. The answer is A or D. Statement 2: The profits in year 1 were $13,800 and in year 3 were $15,900. This indicates that the increase in profits from year 1 to 3 was $2,100 and the

258

SOLUTIONS

percentage gain from year 1 to year 3 could be calculated. But it is not possible to know how much of this increase occurred between year 1 and 2 versus year 2 and 3 so it is not sufficient. The correct answer is then A. Note: People are apt to make two mistakes on this question. First, they may think that the hard numbers from Statement 2 are required with Statement 1 and pick choice C. This would be a classic “Why Are You here?—Temptation” set-up. They give you something that you do NOT need in the second statement but that is hard to say no to. Also, many people will pick choice E because they do not read the first statement carefully and realize that this information guarantees a 0% increase in that time period. 85. E Question Type: What Is the Value? This question asks for the lateral area of the pyramid-shaped box. The lateral area represents the area of the faces of the polygon added together. Given information in the question stem or diagram: The box is pyramid shaped and constructed with four lateral faces and an open bottom. Statement 1: The base is a polygon with all sides of equal length and the perimeter of the base is 1 meter. This statement does not provide enough information to determine the area of the faces of the pyramid. While you know that each side of the polygon at the base is .25 meters, you have no idea about the height of the pyramid and thus the height of each lateral face. This is not sufficient so you can eliminate choices A and D.

GovernmentAdda.com Statement 2: The lateral faces are isosceles triangles that have the same size and shape. This statement is clearly insufficient by itself, as you are given no hard numbers. The question, then, is whether this statement helps you when combined with Statement 1. The answer is C or E.

SOLUTIONS

Together: Taken together, the information still does not give you any idea about the height of the pyramid. If Statement 2 said that the lateral faces were equilateral triangles, then the answer would be C. However, this pyramid could be 3 meters high, or 10 meters high, greatly changing the area of the faces. Note: This question is a good example of the “prove E” concept. If you are going to choose answer E, as you should in this problem, do so because you can identify the missing information, not just because the problem seems difficult. Here you are still missing the height of the pyramid so the correct answer is indeed E.

259

86. D Question Type: Yes/No. The question asks: “Is y greater than or equal to 9?” Given information in the question stem or diagram: x and y are integers and x + y = 23. Note: When specific information like this is given in the question stem, make sure you carefully leverage it with each statement. Statement 1: x – 6 < 9. Adding 6 to both sides you see that x < 15. Since the question asks about y, substitute from the equation into the inequality. If x + y = 23, then x = 23 – y and, after substituting into the inequality, 23 – y < 15. Move y from the left to right and 15 from the right to left to see that y > 8. Since y must be an integer, this proves that y must be at least 9 and this information is sufficient. You could also do this logically, but algebraic manipulation leaves no doubt in your mind! This statement is sufficient so the answer is A or D. Note: In any problem that mixes equations and inequalities, do not forget that you can substitute from the equation into the inequality. Statement 2: x3 = 2744. It is tempting in this statement to try to find the cube root of 2,744 to get an exact value for x. However, since doing this will give you one value for x and since you can then subtract that value from 23 and get an exact value for y, this must be sufficient. Whatever value you find for y will either be “yes, greater than/ equal to 9” or “no, less than 9.” There is no need to do the math here and that is certainly the trick in this statement. The correct answer is D. Note: This second statement is testing exclusively if you understand the concept of sufficiency and how it applies to each of the two question types. If you are missing this type of question (or spending lots of extra time on it!) review the rules/drills relating to sufficiency at the beginning of the book.

GovernmentAdda.com

87. C Question Type: What Is the Value? This question asks for the number of pigs on the farm. Given information in the question stem or diagram: Pigs (P) + Cows (C) + Horses (H) = 60. P + C + H = 60. Statement 1: H:C = 2:9. This statement is not sufficient because it does not give an exact number of cows and horses, so it is not possible for you to determine the number of pigs. However, you should check to make sure that multiple cases are possible given a total of 60 animals. For instance, you could have 2 horses, 9 cows, and 49 pigs; or it could be 4 horses, 18 cows, and 38 pigs. Not sufficient, so you can eliminate answers A and D. Note: if the total was, for instance, 20 animals, then this statement would be sufficient because it would be impossible

260

SOLUTIONS

for there to be more than 2 horses and 9 cows (or total would be over 20); this would leave 9 pigs and a definite answer to the question. Statement 2: C > 36. This statement also does not give you specific numbers for any of the animals, so it is clearly insufficient and the answer is C or E. However, you should already be considering the “Why Are You Here?” strategy and wondering how this information might matter when combined with the first statement. Together: Taken together you know that C > 36 and H:C = 2:9. The hidden fact on many ratio questions is that the number of animals (or children or photocopiers) 1 must be an integer. There is no such thing as   of a horse. This means that you 3 have more information here than you might think. From the two statements together, you know that C must be a multiple of 9 and must be greater than 36. The next multiple of 9 is 45. If there are then 45 cows there are 10 horses because of the ratio in Statement 1. That leaves a total 5 pigs since P + C + H = 60. What about 54 cows? That is a multiple of 9. This is not possible, because 54 cows would mean 12 horses, and that is over 60 animals. Together the two statements are sufficient and there are 5 pigs on the farm. The correct answer is C. Note: People miss this because they under-leverage the information given in these statements and do not take the hint given in the second statement.

GovernmentAdda.com

88. E Question Type: What Is the Value? This question asks for the value of “x + y.”

Given information in the question stem or diagram: No important information is given in the question stem. On this question you should begin with Statement 2 as that statement is easier to evaluate. Statement 2: 3x + 2y = 24. Since there is no way to algebraically isolate x + y from this equation, it is not sufficient. However, if this equation had been 2x + 2y = 24 you could have divided by 2 to find x + y. However, since x and y have different coefficients, you cannot do that. This statement is not sufficient, so eliminate answers B and D.

SOLUTIONS

Statement 1: Clearly, this statement requires substantial algebraic manipulation before you can evaluate its sufficiency. Begin by simplifying the left side. You should see that the numerator 4x2 – 4y2 fits the Difference of Squares format and can be written as (2x – 2y)(2x + 2y). This works because the coefficients are perfect squares as well as the variables. So now the equation is  (2x – 2y) (2x +2y) = 2x – 2y. Simplified further it becomes 2x – 2y = 2x – 2y. This 2x + 2y

261

statement is not sufficient as there is an infinite set of solutions for x and y (i.e., it is true for any values of x and y—a useless equation!) Eliminate choice A. Together: Taken together the statements are still not sufficient because Statement 1 does not contribute anything when looking back at Statement 2. The correct answer is thus E. Note: This question is designed for people to pick answer C, as they believe you have two equations and two unknowns. However, by Playing Devil’s Advocate and manipulating the first statement properly, you can see that the first statement is a meaningless equation that doesn’t contribute any information to the problem. 89. E Question Type: What Is the Value? This question asks for the value of the twodigit number x. Given information in the question stem or diagram: x is a two-digit number. Statement 1: The sum of the two digits is 4. There are only four two-digit numbers that have digits that total 4. They are 13, 22, 31, and 40. Since that allows for more than one value for x, this statement is not sufficient. Eliminate choices A and D.

GovernmentAdda.com

Statement 2: The difference between the two digits is 2. This means that for the tens digit (T) and the units digit (U), either: T – U = 2, or U – T = 2. Many people confuse this statement and think that the tens digit must be larger, such as 64, where T – U = 2. However, 46 would also be acceptable since the difference between the digits is 2. Clearly this statement is not sufficient alone as there are many two-digit numbers where T – U = 2 or U – T = 2. Eliminate choice B. Together: When taking the statements together it is best to start with the more limiting statement. Statement 1 only allows four values: 13, 22, 31, and 40. How many of these values are compatible with Statement 2? Two of them: 31 is T – U = 2; and 13 is U – T = 2. They each have a difference of 2. Since there are still two possible values for the two-digit number x, the correct answer is E. Note: This is a classic C vs. E problem. Almost everyone gets it down to choice C or E, but many people forget to differentiate between 13 and 31. (They either miss one in their list of possibilities for the first statement or assume that it must be 31 for the reasons discussed above.) Remember to do your best to exhaust all possibilities before picking answer C.

262

SOLUTIONS

90. E Question Type: Yes/No. This question asks: “Is the square root of (7ab) an integer?” Given information in the question stem or diagram: No important information is given in the question stem. Statement 1: a = 7. This statement would change the question to: “Is the square root of 7 • 7 • b an integer?” This statement is not sufficient because while you know that 72 will still be an integer when the square root is taken, you do not know whether b is a perfect square. Not sufficient. Eliminate answers A and D. Statement 2: b = an integer raised to the third power. This is also not sufficient. It says nothing about a. In order for the square root of 7ab to be an integer you need either a or b to have 7 as a factor and for the other variable to be a perfect square. This statement does not guarantee either of these. Eliminate choice B. Together: Statement 1 brings the necessary 7 and takes care of that part. All that you need is for b to be a perfect square and then you would be sure that this square root is an integer. However, Statement 2 tells you that b is a perfect cube, not a perfect square. Many people will erroneously pick choice C at this point, thinking that this information proves that it could NEVER be an integer (which would give you a definitive “no” and be sufficient). However, there are many perfect cubes (1, 64, etc.) that are also perfect squares. Therefore together this information could result in either an integer or a non-integer and the answer is E. Note: This is another great example of the importance of Playing Devil’s Advocate. Your gut reaction is that the statements together prove that it could never be an integer, but you need to consider all other ways it COULD be an integer. Considering the number 1 for the value of b is probably the easiest way to show that there is at least one way it could be an integer.

GovernmentAdda.com

91. C Question Type: Yes/No. The question asks: “Is xy > 24?” Given information in the question stem or diagram: There is no important information given in the question stem.

SOLUTIONS

Statement 1: y – 2 < x. This statement is clearly not sufficient since it does not give you any fixed value for x or y. x could be any number that is greater than “y – 2,” easily making this product less than or greater than 24. Eliminate choices A and D. Statement 2: 2y > x + 8. Like Statement 1, this statement does not fix values for x or y but simply states them in relation to each other. This cannot be sufficient. Eliminate choice B.

263

Together: It is every easy to eliminate each of these statements individually so it is a classic C vs. E question. Inequality questions in Data Sufficiency form are one of the most difficult question types on the GMAT. The common mistake that most students make is to pick numbers and try to reason out a solution rather than to apply algebraic manipulation. Using your understanding of combining inequalities, it is possible to isolate x and y and learn more about them individually. First let’s eliminate x and isolate y: Step 1: Rewrite the inequalities to line up variables: y–x<2 2y – x > 8 Step 2: Multiply top inequality by -1 to get the signs pointing the same way and then combine to eliminate x: -y + x > -2 2y – x > 8

GovernmentAdda.com y>6

Repeat step 2 to eliminate y by multiplying the top inequality by -2 to get the signs pointing the same way and then combine: -2y + 2x > -4 2y - x > 8 x>4 If y > 6 and x > 4, then you know that the product of xy must be greater than 24 and the answer to the question is C. Attempting to do this with number picking is both time consuming and ineffective. Also, and importantly, you must do the necessary work to find the limit. If you did this work and found that x > 3 and y > 6, then the answer would be E, as xy might or might not be greater than 24. Many students make the mistake of assuming that if you have two inequalities with two variables, then you can definitely answer the question. That is only true when dealing with equations, not inequalities.

264

SOLUTIONS

92. E Question Type: Yes/No. This question stem asks: “Is ab a prime number?” Given information in the question stem or diagram: There is no important information given in the question stem. Statement 1: a is a prime number. This statement is not sufficient but it is tricky. Using conceptual knowledge, many people will assume that the product ab could never be prime. When you multiply any prime number with another positive integer, it could never be prime unless that integer is 1. However, since b could be 1 or any fraction, this statement is not sufficient, and you can eliminate 2 choices A and D. For instance, if a is 7 and b is 1 or   then the answer is “yes.” 7 However if b is any positive integer other than 1, then the answer is “no.” Note: This question is about avoiding assumptions! Do not assume that b is a whole number, and don’t forget about the very important number 1. Statement 2: b is not a prime number. If b is not prime then b could easily be a fraction or equal to 1. In either case it is very possible for ab to be a prime number and it is very easy to show that it might not be. Not sufficient. Eliminate answer B.

GovernmentAdda.com Together: Taken together you know that a is prime and b is not. But this leaves you in the same dilemma as with Statement 1 alone. Certainly a and b can multiply to equal a non-prime number such as “a = 2 and b = 6.” 12 is not a prime number. However, if b =1 then whatever prime number a equals will remain and ab will be that prime number. As in the first statement, the key in this problem is to avoid assumptions and remember that nothing limits b to an integer and nothing excludes the value of 1. Answer is E.

93. D Question Type: What Is the Value? This question asks for the least number of tiles required to cover a floor. Given information in question stem or diagram: Rectangular floor is 4 meters wide; floor is to be covered with non-overlapping square tiles; tiles are 0.25 meters on each side.

SOLUTIONS

Statement 1: L = 3W. This means that the length = 12 meters and the area is 48 square meters. This statement is sufficient. The number of tiles can be determined since you have the area of the tiles and the area of the floor. The answer is A or D. Statement 2: The area = 48 square meters. This gives the same information as in Statement 1 so the answer must be D or E. (This is the “no news is good news”

265

scenario referenced in the lesson. Answer choices A, B, and C are not possible if the statements give equivalent information.) Since each statement is sufficient alone the answer is D. Note: This problem is only difficult because people will forget to leverage the information in the question stem. If you look at both statements and forget that you know the width, then you will certainly pick answer C, as then you would need both statements. However, since you know the width, each one gives the necessary information. 94. A Question Type: Yes/No. This question asks: “Is x > 3?” Given information in question stem or diagram: No important information is given in the question stem. Statement 1: The first step in this statement is to translate the wording into the following equation: x + x2 = 12. Since this is a quadratic equation, you should set everything equal to zero so that x2 + x – 12 = 0. Factoring this, you see that (x + 4)(x - 3) = 0 and x would be -4 or 3. The difficulty in this statement is that many people assume that this information is not sufficient because there are two values, one negative and one positive. However, remember that to prove sufficiency in a yes or no question requires only a definitive answer, not one value. Since each of these values (-4 and 3) gives a “no” answer to the question, this statement is sufficient. The answer is either A or D.

GovernmentAdda.com

Statement 2: x2 > 9. If x2 > 9 then either x > 3, which gives you a “yes” answer, or x < 3, which gives you a “no” answer. For example x could be -5 (which when squared is > 9) or 5 (which when squared is also > 9). This statement is thus not sufficient, and the correct answer is A. Note: This question is created to prey on two common mistakes, one relating to Data Sufficiency itself and one relating to algebra: 1.) People (even those who have done lots of data sufficiency) tend to forget to look for the “no” answer in Yes/No questions and they often make mistakes about what is really required for sufficiency on Yes/No questions. 2.) People forget about the negative possibilities when dealing with squared variables in inequalities. 95. D Question Type: What Is the Value? This question asks for the specific value of positive integer a. Given information in the question stem or diagram: There is a lot of information

266

SOLUTIONS

to leverage from this question stem. a, b, and c are distinct positive integers; a < b < c; and the square root of abc = c. You should first manipulate the last one algebraically by squaring both sides to see that abc = c2. Divide both sides by c (you can do this because you know that c cannot be 0 from the question stem) and the equation becomes ab = c. So before you even go to the statements you know that ab = c and all of the variables are different positive integers. Statement 1: c = 8. Combined with what you learned from the question stem, this means that ab = 8. Since a and b are distinct positive integers and a < b, the only possibility is a = 2 and b= 4. You might consider a = 1 and b = 8 but since the integers must be distinct, you cannot have b = 8 since c = 8. This is sufficient but you will only see that if you properly leverage every piece of information given in the question stem. Remember: When you are given even a small piece of information in the question stem it is usually very important. The correct answer is A or D. 14 Statement 2: The average of a, b, and c is   . This means that the total of a 3 + b + c = 14. This statement is even trickier than the last but requires a similar leveraging of all available information. It may seem at first glance that there are many possibilities for the values of a, b, and c. However, the only way that ab = c and a + b + c = 14 is for a = 2, b= 4, and c = 8. There is no other way to have three distinct numbers add up to 14 and have ab = c. This statement is also sufficient and the correct answer is D. This question provides an excellent example of a phenomenon you will see often in Data Sufficiency: When a lot of information is given in the question stem, statements are usually sufficient with much less information than you might first think.

GovernmentAdda.com 96. C Question Type: What Is the Value? This question asks for the value of integer x. Given information in the question stem or diagram: You are told that x is an integer, so consider how that might be important with each statement. Statement 2 is much easier to consider on this problem, so you can start there.

SOLUTIONS

Statement 2: x = 3 or 4. Clearly this is not one specific value for x so it is not sufficient. Eliminate choices B and D. Since this statement is quite easy, consider how it might be important in relation to Statement 1. 1 1 1 Statement 1:   <   <   . This can be restated using only the denominators: 5 1+x 2 In order to have a smaller fraction (with the same numerator) you need a larger denominator so 5 > 1 + x > 2. Therefore x has to be < 4 and x has to be > 1. The

267

only integers between 4 and 1 are 3 and 2. So x could be 2 or 3. Thus Statement 1 is also not sufficient. Eliminate choice A. Together: When taken two statements that each give specific numbers, the key is the overlap. The only numbers that are acceptable when taking the statements together are those that satisfy the conditions in each of the two statements. In this case the only number that each statement allows is 3. Therefore x =3 and the correct answer is C. Note: Before you pick answer C on this problem, you should carefully make sure that you couldn’t do it from each of the statements alone. 97. B Type of question: What Is the Value? The question asks for “the speed of car y (beyond point p) as a percent of the speed of car z.” Note: This is the same as asking for Distance/Time of car y as a percent of the Distance/Time of car z. Given information in the question stem or diagram: The roads that car y and z are traveling on are parallel until point P. At point P the cars are even and then car y makes a turn of x degrees and continues to keep up with car z. The key information that you need to leverage from this given information is that the times will be the same for each car.

GovernmentAdda.com

Statement 1: “The speed of car z = 50 mph.” This statement does not give any information about the speed of car y, nor about the extra distance that car y must travel. This statement is not sufficient on its own. Eliminate choices A and D, and note that this is a classic “Why Are You Here?” set-up. This statement is clearly insufficient on its own so you will want to consider whether it is important in relation to Statement 2. Statement 2: “The measure of angle x = 45 degrees.” This statement may not appear sufficient until you realize that you do not actually need to know the speeds of cars y and z. Because the cars are keeping up with each other, the time that each car travels must be equal. That means that the increased percentage of distance that car y has traveled will be the same as the increased percentage of the rate of car y compared to car z. In other words, determining the ratio of the distance of y to the distance of z is enough to answer this question because the times are necessarily the same. While you do not need to do it, the actual mathematical solution is below. Also, note that this is a classic example of the “Why Are You Here?—Temptation” set-up that is used so often on harder problems. They have tempted you with a statement (the speed of car z) that is not necessary to determine sufficiency. Statement 2 is sufficient by itself. The road that car y travels on after point P forms a triangle with the road that y

268

SOLUTIONS

was traveling on (that road is parallel to the road z is traveling on). This triangle is a right triangle with the right angle formed by the dotted line. Statement 2 gives the measure of x at 45 degrees. That means you have a 45–45–90 triangle with the distance that y has traveled as the hypotenuse and the distance z has — traveled as one of the shorter sides. The hypotenuse is √ 2 bigger than the — shorter sides, so car y is √ 2 faster than car z. As noted above, this is sufficient and the correct answer is B. 98. A Question Type: What Is the Value? This question asks you for the specific value of   b–c . b Given information in the question stem or diagram: a, b, and c are integers, a and = 1. Before going to the statements it might be helpful to restate that b–c equation. If you multiply both sides by b – c, the equation becomes a = b – c. a Looking back to the question you can now phrase the question as “What is  ?” b Remember that often the important work in Data Sufficiency questions relates to manipulating the information in the question stem, not just in the statements. a 3 Statement 1: “  =  .” The work that you did on the question has paid off. b 5 a 3 This statement exactly answers the question:  =  . This is sufficient, and the b 5 answer is either A or D.

GovernmentAdda.com Statement 2: “a and b have no common factors greater than one.” This means that a and b are what is called “co-prime.” For example, 8 and 9 are “co-prime.” Neither has to be a prime number but in relation to each other they have no common factors greater than 1. This statement is not sufficient, as there are many co-prime numbers. Since this is clearly insufficient by itself, it is a “Why Are You Here?” set-up and you should wonder why testmakers are delivering this statement. Is it important? It would be important if you were trying to find the value of a or b individually, but since the question only asks for a ratio, the individual values of a and b are not important and the answer is A. 99. A Question Type: What Is the Value? This question asks for the specific value of x – y.

SOLUTIONS

Given information in the question stem or diagram: No important information is given in the question. Note that you only need to find the difference between the two variables. This will usually require less information than solving for the individual variables. Statement 1: This is a perfect example of using algebraic manipulation to find the value of x – y. By digging deep into your algebra toolkit and by using your

269

understanding of the common algebraic equations, you should manipulate accordingly: (x + y)2 = 4xy can be rephrased as x2 + 2xy + y2 = 4xy. Subtract 4xy from both sides to create the quadratic equation x2 – 2xy + y2 = 0. This can be factored to (x –y) (x – y) = 0. Therefore x – y = 0. This statement is sufficient, and the answer is either A or D. Statement 2: x2 – y2 = 0. This statement again requires proper algebraic manipulation and recognition of the difference of squares: x2 – y2 = 0 is the same as (x +y) (x –y) = 0. However, it is not clear whether x + y = 0 or x – y = 0 so this statement is not sufficient. The correct answer is A. Note: This problem demonstrates how important it is to have a high level of fluency in algebra, and with the common algebraic equations in particular. There is no way to prove sufficiency here without this fluency. 100. B Question type: Yes/No. The question asks: “Is the product (a) (a + 1) (a +2) divisible by 48?” You can think ahead of time that in order to be divisible by 48 this product must have the prime factors of 48: 24 • 3. So the question is really: Does this product contain at least four 2s and one 3?

GovernmentAdda.com

Given information in the question stem or diagram: “a is a positive integer.” Since a is a positive integer the rule that “in any three consecutive integers one of those integers will be a multiple of 3” applies. That means that before you even go to the statements you know that the product (a) (a + 1) (a +2) will be a multiple of 3. The question then can be simplified even more from above because you know that the factor of 3 will be present. The simplified question is: Does this product have 24 as a factor? It is essential that you always leverage all given information in the question before moving to statements. Also note that this problem (as is true for most arithmetic problems) is best solved with your conceptual understanding of factors and divisibility. While you could prove sufficiency/insufficiency with number picking, it would be cumbersome and risky in this example. Statement 1: a is even. If a is an even number, it means that a will contain at least one 2 as a factor. It also means that a + 2 will be even and that one of those two even numbers will be a multiple of 4. For example, if x = 2 then (x +2) = 4. This means that you have at least 23 as a factor. However, this statement is not sufficient as it only guarantees three 2s in the product and not the required four 2s. Eliminate choices A and D. Statement 2: If 4a is divisible by 32 then “a” must be divisible by 8. If a contains three 2’s as factors then this information is sufficient as you know that (a + 2) will

270

SOLUTIONS

have to contain at least one 2 as well. This statement is sufficient to prove that the product will contain 24 • 3 and the correct answer is B. 101. A Question Type: What Is the Value? This question asks for the area of the rectangle PQRS. Given information in the question stem or diagram: The diagram is given and importantly one side is defined with PS = 4, so both PS and QR = 4. To find the area you need to somehow determine either RS or PQ from the information in the statements. The question is really: What is RS or PQ? Statement 1: QTR is an equilateral triangle. If this is true, then with QR as the base, the height of the equilateral triangle is equivalent to the side that you are trying to determine (PQ or RS). Since the base of QTR is 4 you can determine the necessary height with your knowledge of 30–60–90 triangles or simply with your knowledge that if you know one thing about an equilateral triangle, then you know everything! While you do not need to calculate it, the height would — 1 be 2√ 3  as it is the long leg in a 30–60–90 triangle formed by  the base of QTR 2  — (2), the hypotenuse QT (4) and the height ( 2√ 3  ) which is equivalent to PQ or RS. Statement 1 is sufficient, so the answer is A or D.

GovernmentAdda.com Statement 2: In this difficult statement, the testmakers are playing with a common trick. They have polluted your brain with the first statement and want you to assume that if “segments PT and TS have equal lengths,” then QTR must again be equilateral. However, this statement does nothing to help you determine the length of the sides (QP and RS) because it only proves that QTR is isosceles. There is no limit put on the lengths of PQ and RS (because you do not know the angle of TQR and TRQ) with this statement, so it is not sufficient. Remember: One of the keys to success in Data Sufficiency is to consciously avoid assumptions, but that can be hard when you are set up so nicely to make assumptions with the other statement. Statement 2 is not sufficient, so the correct answer is A.

102. A 1 1 Question Type: Yes/No. The question asks: “Is +  = 16?” x y

SOLUTIONS

Given information in the question stem or diagram: xy does not equal 0. This is important only because x and y cannot be 0, as they are denominators. Importantly, you should manipulate the equation in question. As you learned with the Algebra Toolkit, in almost any equation involving fractions, the first step to simplification is to remove the denominators. Here that is done by multiplying the equation by xy to create the new question: Is y + x = 16xy?

271

Statement 1: If you have done the proper manipulation as shown above, it is clear that this statement is exactly the same as the changed question and thus sufficient. The answer is A or D. Note: Do not forget that on many harder Data Sufficiency questions it is easier to change the question to match the statements than vice versa. This provides an excellent example of this phenomenon and shows the importance of proper algebraic simplification in Data Sufficiency questions. Statement 2: If you take this information (x = y) and substitute it into the question above you get: Is 2x = 16x2? Or: Is 2y = 16y2? These questions are impossible to answer, as you do not know anything about the value of x or y. Statement 2 is not sufficient so the answer is A. Note: If you picked answer C on this question you have fallen for the “C Trap”! Most test-takers immediately see that this question can be answered with both statements together, as substituting x for y turns Statement 1 into a single variable equation, allowing you to solve for one variable, which we know from Statement 2 is exactly the same as the other. But in situations in which C seems “too easy,” one should take the more-comprehensive statement and try to exhaust it, as the GMAT is apt to reward you for being able to obtain more value from each resource. Statement 1, as shown earlier, is actually the same as the question, but this is only apparent after the proper manipulation shown above.

GovernmentAdda.com

103. E Question Type: What Is the Value? This is a Venn Diagram question and the question asks: “How many students take both French and Spanish?” Given information in the question stem or diagram: In any Venn Diagram problem you should draw out the diagram and remember that Total = Set 1 + Set 2 – Both + Neither. It is important to always consider “Neither” in these types of DS questions and to note the difference between total set information and “only” information (for instance, the difference between people who take “only French” and “French”). Statement 1: French = 100 and Spanish = 125. This can be written to say that French + Spanish – Both + Neither = Total or 100 + 125 – Both + Neither = Total. This statement is not sufficient alone because you do not know how many total students there are, nor how many students take neither French nor Spanish. Not sufficient. Eliminate choices A and D. Statement 2: Total students = 200. This is not sufficient alone since there is no way to allocate the 200 students among the categories. Eliminate choice B.

272

SOLUTIONS

Together: Many students will think that the answer is C because they have forgotten to consider “Neither.” You do not know how many students take neither French nor Spanish, so there is no way to determine “Both.” Without this information you only know that at least 25 students take BOTH (that is if Neither = 0) and as many as 100 might take BOTH if the neither category was 75. This is not sufficient and the correct answer is E. Note: In any choice between C and E such as this you must learn to Play Devil’s Advocate and consider anything you might be missing. In this example (as with many Venn problems) that is making sure you remember to consider the value of “Neither.”

104. B Question Type: Yes/No. The question asks: “Is x positive?” Given information in the question stem or diagram: No important information is given in the question stem. Statement 1: x2 = 9x. Many students will think that x = 9 after dividing both sides by x and thus believe that this statement is sufficient. However, it is important to consider all of the possibilities, and you are not allowed to divide by x if it is 0. Given this, the best way to approach this question is to subtract 9x from both sides to get x2 – 9x = 0. Factor out x and see that: x(x – 9) = 0. So clearly x can equal 9 or 0. Because this is a yes or no question, you must check each value to get a “yes” or a “no.” Just because you have two values for x does not make the statement automatically insufficient. x = 9 gives you a “yes” since x is positive. x = 0 gives you a “no” since zero is not greater than zero. So this statement is not sufficient and the answer is either B, C, or E. Note: This is a common and tricky algebraic set-up in data sufficiency. Mathematically, whenever you have a squared variable you should set the equation equal to zero and factor.

GovernmentAdda.com

SOLUTIONS

Statement 2: | x | = -x. This statement appears more difficult than it really is. The key of course is to figure out what this equation tells you about x. Hopefully you recognize that x must be negative OR zero given this statement, but if not, you can try some numbers to prove it. Try a positive number for x such as 9. That gives you | 9 | = -9, which means “9 = -9.” This is not true so you can see that any positive numbers are excluded by this statement. Try a negative number: x = -5. Now |-5 | = -(-5), which means 5 = 5. This is true so you can see that this statement allows for negative numbers. You will also want to try zero, but as in the last statement, many students forget about zero. | 0 | = -0. This is true (0 = 0) so negative numbers as well as 0 work for this statement. Taking these back to the question you see that any negative numbers give you a “no” answer. Zero

273

also gives you a “no” answer so there is a consistent “no” and this statement is sufficient. The answer is B. Note: The difficulty of this question lies in both the tricky algebra and in the fact that you might pick answer C even if you do the algebra properly. Why? Because people forget that “no” is just as good as “yes” in the second statement and they also see that 0 nicely overlaps between the statements, making it tempting to think “C.” If the question was asking “What is the value of x?” then the answer would be C. However, since this is a Yes/No question, the second statement gives you a definitive answer and is thus sufficient. 105. A Question Type: What Is the Value? This question asks for the average of the terms in set J. Given information in the question stem or diagram: No important information is given in the question stem. Statement 1: The sum of any three terms in the set is 21. This is a very difficult statement to consider. Clearly this WILL be true if all the terms are 7, but you need to make sure there are not any other possibilities. Will it work with any other sets, such as “6, 7, 8, 6, 7, 8”? Adding certain numbers in this set will give you 21, but not “any three terms.” The only ways to ensure that any three randomly chosen terms will sum to 21 are either to have only three terms (say, 6 + 7 + 8) or to have all terms in the set equal 7. In either case, the average must be 7, so this is sufficient information. The answer is A or D.

GovernmentAdda.com

Statement 2: Set J consists of 12 terms. This statement is not sufficient on its own since it does not give any values for the terms in the set. However, as is the case with any clearly insufficient statement such as this, you must consider whether it is important with the other statement. This statement was clearly designed to make choice C a more attractive option, but you do not need to know anything about the number of terms for the first statement to be sufficient. The correct answer is A. 106. B Question Type: Yes/No. The question asks whether line M runs through point (6,6). Given information in the question stem or diagram: Line M is tangent to a circle; the circle is centered on point (3,4). Statement 1: Line M runs through point (-8,6). This statement is not sufficient because there is no information about the size of the circle or the direction that

274

SOLUTIONS

line M is running. Simply having a single point on line M will not be sufficient to answer the question. Eliminate choices A and D. Statement 2: Line M is tangent to the circle at point (3,6). Many people will immediately combine this with Statement 1 in their minds and think that with both statements you know that M is the straight line y = 6, in which case line M would definitely run through point (6,6). However, before you jump to this stage, remember that you only get to combine statements when neither statement is sufficient alone. Before you pick choice C in a question like this, it is always essential that you take a closer look at each statement individually. The definition of tangent is very important for Statement 2 and it is what differentiates Statement 2 from Statement 1. A line that is tangent to a circle touches that circle in only one place and is perpendicular to that circle at that point. Given that the center of the circle is at (3,4) and the point of tangency is at (3,6), you can conclude that line M is perpendicular to the line x = 3. Therefore line M must be the line y = 6, and it does pass through (6,6). This statement is sufficient alone and the answer is B. Note: This is another great example of the “C trap” set-up in which testmakers give you two statements that fit together nicely but are not both required to answer the question. To combat this set-up, use the reward system from the lesson and always double-check each statement alone before picking answer C.

GovernmentAdda.com 107. A Question Type: What Is the Value? The question asks you for the remainder when x is divided by 10. Given information in the question stem or diagram: x is a positive integer. When x is divided by 7 the quotient is q and the remainder is 1. Number picking is an important and valid strategy in quotient/remainder questions, so you might even want to consider possibilities for x before moving to the statements. If the remainder is 1 when x is divided by 7, then x could be 1, 8, 15, 22, 29, 36, 43, etc., yielding different quotients for q.

SOLUTIONS

Statement 1: When x is divided by 5 the quotient is q and the remainder is 1. In the question stem, you learned that when x is divided by 7 the quotient is also q and the remainder is also 1. Most students understand that if you divide x by two different divisors and get the same remainder, x must equal the sum of the remainder and a multiple of the divisors’ LCM. For instance, if you divide a number by 5 and 7 and get a remainder of 1, then that can happen at 35 + 1; 70 + 1; 105 + 1; etc.

275

However, if you read carefully you see that the quotient is the same in both operations, a puzzling result. Clearly if you divide 36 by 7 it will give you a different quotient then when you divide 36 by 5. However, students will overlook this important fact (that q is the same in both cases) and believe that there are multiple possibilities for x and thus multiple possibilities for the remainder when you divide x by 10. The only way you can have the same quotient and the same remainder when you divide by two different numbers is if you are dividing a smaller number by a bigger number—in this case 1. Remember that when a smaller integer is divided by a larger integer, the quotient is always 0 and the remainder is the dividend itself. Here when 1 is divided by 7 the quotient is 0 and the remainder is 1, and when 1 is divided by 5 the quotient is also 0 and the remainder is 1. Statement 1 is sufficient, as x must be 1 and the remainder when x is divided by 10 is also 1. The answer is either A or D. Statement 2: x is less than 50. This is clearly not sufficient by itself. This statement is here for people who miss that the quotient is the same and think that x could be any (multiple of 35) + 1. By knowing that x is less than 50 it would seem to lock in the value at 36. However, from the discussion above it is clear that this would be incorrect. This is a very clever “C trap” because it makes the person choosing answer C feel like they have understood the problem while the answer is really A. The only way to get this right is by knowing this tricky and easily overlooked fact in Arithmetic and by reading the information closely (leveraging that q is the same when dividing by two different numbers—a puzzling result).

GovernmentAdda.com

108. B Question Type: What Is the Value? This question asks you for the number of families that have a dog. Given information in the question stem or diagram: There are a total of 60 families in the neighborhood. 38 of these families have a cat, or C = 38. Remember that you have been given the total number that have cats (not only cats) and you have been told nothing about how many of these total families have neither a cat nor a dog. Statement 1: This statement tells you that 28 families have only a cat. Since you know that there are 38 total families with a cat, this means that 10 families must have both. At this point, if you forget about neither, you might think that this is sufficient because the number of families with dogs would be 60 – 28 (those with only a cat), or 32. However, you do not know how many families have neither a cat nor a dog so perhaps fewer than 32 families have a dog. This is not sufficient, so eliminate choices A and D.

276

SOLUTIONS

Statement 2: In this statement you are told that “Both = Neither.” This information does not appear to be sufficient at first glance, but a closer look at the venn diagram formula for two set problems shows that it is. Total Familes = # of Families with a Cat (C) + # of Families with a Dog (D) – Families with Both (B) + Families with Neither (N) or with variables: T = C + D – B + N Combining this with the facts we know that T = 60. C = 38 and B = N, meaning that if you substitute “B” for “N” (they are equal) you get 38 + D – B + B = 60. Simplify and 38 + D = 60. D = 22. This statement is sufficient. The correct answer is B. Note: This is a very difficult example of the common “C trap” scenario. Together, you can clearly find the value for the number of families with dogs. Only with a very careful analysis of each statement alone will you realize that Statement 2 is sufficient. The fact that the two statements make it pretty easy to solve gives you a hint to look VERY carefully at each statement. 109. A 5x–2 5x–1 Question Type: Yes/No. The question asks whether  –  > 0. 3 4 Given information in the question stem or diagram: x does not equal 0, and you are given a question in inequality form that can be easily simplified. Before you move to the statements, you should simplify the question. Multiply both sides of the inequality by 12 (LCM of 3 and 4) to see that the question is really: Is 20x – 8 – (15x – 3) > 0? With a bit more simplification (being VERY careful with parentheses and negative/positive signs) the question becomes: Is 20x – 8 – 15x + 3 > 0? After combining like terms, it’s: Is 5x > 5? As it turns out this question is really asking: Is x > 1?

GovernmentAdda.com Statement 1: If you have properly simplified the question, it is clear that this statement exactly answers the question and is thus sufficient. If you have not, it will be both difficult and tedious to prove with number picking. Remember: With Data Sufficiency questions involving inequalities, manipulating algebraically is almost always the best way to prove sufficiency. Often this involves manipulating the question, not just the statements. Since this is sufficient, the answer is A or D.

SOLUTIONS

Statement 2: x = |x|. This indicates that x is a positive number (since you are told in the facts that x does not equal 0). However, this does not mean that x is larger than 1 so you have a “maybe” answer. (Numbers between 0 and 1 would give a “no” answer while all others would give a “yes” answer.) This statement is not sufficient. The correct answer is A. Note: This problem is very hard if you do not manipulate the question, but relatively easy if you do!

277

110. D Type of Question: What Is the Value? This question asks for the greatest common factor of x and y (in other words, the largest integer that will divide into both numbers x and y). Given information in the question stem or diagram: x and y are positive integers. Also note that any factor question such as this generally relies on your conceptual understanding of the underlying Arithmetic, not on manipulation or number picking. Statement 1: When x is divided by y the remainder is 1. Conceptually, this means that any factor of y other than 1 cannot divide into x, because it, too, will have a remainder of 1 when divided into x. Imagine as an example if x were 31 and y were 30. None of the factors of y greater than 1 (2, 15, 3, 10, 5, 6) can divide into 31; they must all necessarily yield a remainder of 1 as well. So if y = 10 and x = 31 there will be the same result. Therefore, if there is a remainder of 1 when x is divided by y then these numbers are “co-prime,” meaning that the only factor they can share is 1. Statement 1 is sufficient as the greatest common factor (GCF) must be 1. This would also be true if x is smaller than y, as x would have to be 1 and it would share only the factor of 1 with any other number. This statement is sufficient. The answer is A or D.

GovernmentAdda.com

Statement 2: x2 – 2xy + y2 = 1. This requires that you first do some algebraic manipulation. As with any of the common algebraic equations, if they are given to you in one form you should typically convert it to the other form. Here x2 – 2xy + y2 = 1 is the same as (x – y)2 = 1. You can determine from this equation that positive integers x and y must be consecutive numbers on the number line. For this equation to be true, the difference between x and y must be 1. For instance x could be 2 and y could be 3, or x could be 3 and y could be 2. Regardless of which it is, the greatest common factor of any consecutive integers on the number line is always 1 for the same reason as discussed in Statement 1: The factors of any positive integer x (when x is greater than 1, which is a factor of all numbers) cannot divide into a number directly adjacent to x on the number line. Therefore, this statement is also sufficient and the answer is D. Note: Like many arithmetic DS questions, this one requires a deep understanding of the underlying math. Data Sufficiency construct thinking will not help you on this problem, as it is more about the math than the Data Sufficiency question type.

278

SOLUTIONS

111. C Question Type: Yes/No. The question asks: “Is (y –10)2 > (x+10)2 ?” Given information in the question stem or diagram: While no information is given in the question stem, it is essential that you change/simplify the question. As in most difficult inequality problems, algebraic manipulation, not number picking, is the key to success. In this very difficult example, most of the algebraic manipulation has to be done with the question stem itself. To simplify the question, you should first expand the two common algebraic equations so the question looks like this: Is y2 – 20y + 100 > x2 + 20x + 100? Remove 100 from each side to get the question: Is y2 –20y > x2+ 20x? The next step is difficult and not obvious. By moving x2 from the right side to the left side in the question and moving 20y from the left side to the right side, you create the difference of squares on the left and an easily factored expression on the right: Is y2 – x2 > 20y + 20x? Factor each side to change the question to: Is (y + x)(y – x) > 20(y + x)?

GovernmentAdda.com At this point many students will incorrectly divide each side by (y + x) to cancel that expression. But remember that you cannot divide by variables in an inequality unless you know the signs of the variables involved. At this point you have simplified the question as much as possible, and you should look at each statement with your new question in front of you: Is (y + x)(y – x) > 20(y + x)?

Statement 1: “ -y > x +5” or, after rearranging, “y + x < -5.” Given the “new” question that you have just created above, this is a very important piece of information, as it says that y + x is negative. If you know the sign of y + x you can then again simplify the question: Is (y + x)(y – x) > 20(y + x)? Divide both sides by (y + x) and flip the sign because you know from Statement 1 that y + x is negative. Using Statement 1, then, the question becomes: Is y – x < 20?

SOLUTIONS

While you cannot answer this question yet, Statement 1 has helped get you closer by allowing more simplification of the question. Statement 1 is not sufficient. Eliminate choices A and D. Statement 2: “x > y.” This is clearly insufficient by itself. (With simple number picking or conceptual thinking you can see that you can get both a “yes” and a “no” from the original question.) This is not sufficient. Eliminate choice B.

279

Together: When you take the two statements together you know that the question is really: Is y – x < 20? AND you know that x > y. That second statement means that y – x must be a negative number (whenever you subtract a larger number from a smaller number you get a negative result) and you are sure that y – x will always be less than 20. The statements are sufficient together, and the answer is C. Note: This type of inequality question in data sufficiency form is a very difficult one for students. Ninety percent of the time, these questions require that you use algebraic manipulation to prove sufficiency, and this problem is no exception. 112. C Question Type: What Is the Value? This question asks you for the value of y. Given information in the question stem or diagram: No important information is given in the question stem. Statement 1: x2 – y2 = 5. This is not sufficient because x and y can be many pairs of — — real numbers. For instance x could be √15  and y could be √10 , or they could be negative integers such as x = -3 and y = -2, or positive integers such as x = 3 and y = 2. Not sufficient. Eliminate choices A and D. Note: If you assume that x and y are positive integers (which many people do!), then this would appear to be sufficient.

GovernmentAdda.com

Statement 2: x and y are each positive integers. This is clearly insufficient, as y could be any of the infinite set of positive integers. Eliminate choice B but note that is a perfect example of the “Why Are You Here?” construct. Together: Taken together, you should consider why Statement 2 (which is clearly insufficient by itself) might be important when analyzing Statement 1. In other words, consider what hints are being given by the second statement. If x and y are each positive integers, then x must be greater than y, as the difference is positive 5. Furthermore, there is only one set of perfect squares that can yield a difference of 5 and that is (3)2 – (2)2. As you move up the number line, the difference between any set of perfect squares increases to infinity, so there is only one place on the number line where the difference is 5. As a result, x must be 3 and y must be 2. If you take both statements together, they are sufficient. The correct answer is C.

113. E Question Type: Yes/No. The question asks: “Is ab an integer?” Given information in the question stem or diagram: a and b are nonzero integers. Note: The fact that they cannot be zero is likely important so make sure you leverage that as you look at each statement.

280

SOLUTIONS

Statement 1: “ba is negative.” This statement tells us that b is negative and that a is odd. (If a were even the result could not be negative.) Taking this information back to the question stem, you will find that it is not sufficient. For almost all values of ab, where b as negative, the result is a non-integer, as a negative exponent means that you are creating a fraction. But if the denominator of that fraction is 1 or -1 (which is allowed by this statement) then ab is either 1 or -1, and that means that ab is an integer. Statement 1 is therefore not sufficient, so eliminate choices A and D. This is a good illustration for how important it is to Play Devil’s Advocate when number picking. It is easy to show that ab can be a non-integer, but you must consider all possibilities to see that there is an exception. Statement 2: “ab is negative.” This isn’t particularly helpful, as it tells us only that b must be odd (positive or negative) and that a is negative. If you go back to the question stem you see that if b is negative, then you can have a fraction, but if b is positive then we’re guaranteed an integer. Together: Even taken together, the possibility still exists for ab to be -1, particularly if a were -1 and b were -1. This results in an integer. All other possibilities, such 1 as a = -3 and b = -1, which results in -  , yield nonintegers, but that one lone 3 catch of -1 makes statements together insufficient, leading to answer choice E. Note: This problem is all about Playing Devil’s Advocate. If you are forced to number pick or think conceptually, make sure you consider numbers like 1,-1, 0, and fractions to find those tricky exceptions.

GovernmentAdda.com 114. C Question Type: What Is the Value? This question asks for the value of the remainder when x is divided by y. Given information in the question stem or diagram: x and y are integers; x and y are not negative. Note: Anytime you are told that a variable is “nonnegative” you should always remember that it could be zero as well as positive when analyzing the statements (although it does not end up mattering in this example!). Start with Statement 2, the easier statement.

SOLUTIONS

Statement 2: “The numbers x and y have a combined total of fewer than 5 digits.” This is clearly insufficient on its own, as it allows for thousands of combinations of x and y. Eliminate choices B and D. x Statement 1:   = 13.8. This question, with Statement 1, tests your understanding y  of the relationship between a remainder and the decimal representation of a

281

quotient.  Perhaps the easiest way to recognize what information is conveyed by the decimal about the remainder is to express 13.8 as a mixed fraction: 4 13  . Still, this does not guarantee that our remainder is 4. We know only that 5 the remainder is a multiple of 4. 4 69 Next, express the mixed fraction 13  as an improper (top-heavy) fraction:  . 5 5 x 69 If  y =   , then perhaps x= 69 and y= 5, or perhaps x= 138 and y = 10.  In   5 x 69 138 207 other words,  y =   =   =  , etc. If x = 69 and y = 5, the remainder in 10   15   5 question is 4. But if x = 138 and y = 10, the remainder in question is 8. Statement 1 alone is not sufficient. Eliminate answer A. Together: What we learned from Statement 1 is that x is a multiple of 69 and y is the corresponding multiple of 5. For example, x and y could be the second positive multiples of 69 and 5, respectively: 138 and 10. Of course, from Statement 2, we know that 138 and 10 have too many total digits to be the values of x and y.  All bigger possible multiples of 69 and 5 would certainly have five or more total digits, as well. Thus, with both statements combined, we know that x = 69 and y = 5, and that the remainder in question is 4. The correct answer is C.

GovernmentAdda.com

Note: There are several ways to deal with remainder questions on the GMAT, and the specific form of the question should dictate what approach you take. If you are dealing with a decimal representation on a remainder question, converting the decimal to a mixed fraction, and then to an improper fraction, is a useful approach.  Note also that many remainder questions on the problem-solving format ask not for a precise value of a dividend, divisor, or remainder, but for a possible value of some piece (e.g., “Which of the following could be the value of the remainder?”). From the above explanation, you can see that these “could be” questions hinge on divisibility issues, or what the piece in question is a multiple of.

115. D Question Type: What Is the Value? This question asks for the length of the arc XYZ. This is really asking you for the diameter (q + r), as that will allow you to calculate the circumference. Given information in the question stem or diagram: You are told that arc XYZ is a semicircle and given all the geometric information in the diagram itself. You know that angle XYZ is measures 90 degrees; anytime a triangle is inscribed in a circle, and one side of the inscribed triangle is a diameter of the circle, that diameter is the hypotenuse of a right triangle. Before you even begin work on this one, recognize this: Choice C is much too easy an answer. If you know the diameter of a semicircle (which Statements 1

282

SOLUTIONS

and 2 together hand you on a silver platter), you can easily find the length of that semicircle (half the circumference). Don’t take the bait on choice C—or at least recognize that there’s a very, very high likelihood that the answer will be something other than C, so it’s worth your time to do some math here. Recognize also that statements 1 and 2 both give you the same kind of information: One segment of the diameter and the side of a smaller right triangle within the larger right triangle. So if one statement is sufficient, the other one will likely be sufficient too. Statement 1: This tells us that q = 2. And since we also have that height of 4, we can solve for line XY. But remember: Data Sufficiency is about whether you CAN SOLVE the problem, not about THE ANSWER itself. So save yourself the messy Pythagorean work and just call line XY “known.” It’s an ugly number (well, not that ugly—it’s the square root of 20), but you don’t need the exact number. Note now that that line XY is also part of one other Pythagorean calculation, for the large triangle (XYZ). That side squared plus side YZ squared will give us (2 + r)2. And our goal is to solve for r (because r + 2 is the diameter, which leads us to the circumference). So let’s think about other calculations we know for r. The right-hand side smaller triangle also includes r and side YZ. On that one, r2 + 42 = (YZ)2. So we have two equations with r:

GovernmentAdda.com (KNOWN) 2 + (YZ)2 = (2 + r)2 (YZ)2 = r2 + 42 Here, notice that because (YZ)2 is in two equations, we can substitute for it and get everything in terms of r. Substituting the second equation into the first, you have: (KNOWN)2 + r2 + 42 = (2 + r)2 And remember: The numbers themselves don’t really matter—just the fact that we’ll get numbers. So on the left-hand side: r2 + (a known number) = 4 + 4r + r2

SOLUTIONS

The r2 terms will subtract out, and now you have a linear equation with just 4r. You can prove here that you will get a value for r, so Statement 1 is sufficient. Also, these three triangles are all similar to each other and you can come to the same solution by leveraging that fact (and is much faster IF you see it). Statement 2: Since, as we’ve determined, each statement essentially gives you

283

the same amount and type of information, Statement 2 is sufficient, also. You do not want to go through the same process in the other direction; just know you can do it. The answer is D, but the takeaways may be more important: 1. When the math looks messy in a specific-value problem, know that you don’t need to actually worry about official numbers. 2. When choice C looks too good to be true, it probably is. 3. Geometry is about leveraging assets, so it’s quite possible for geometry based statements to be “sneaky sufficient.” Make sure that you leverage all your assets in geometry-based Data Sufficiency. 116. A Question Type: What Is the Value? This question asks you for the value of x. Given information in the question stem or diagram: y is an odd integer, and xy = 222. Statement 1: x is a prime number. When combined with the information from the question stem, this is sufficient. If xy is even number and if y is an odd integer, this means that x must be even (or a non-integer, but “x is prime” rules that out). Since x is a prime number, it can only be 2, the only even prime number. This gives you one specific value for x and is sufficient. The answer is either A or D.

GovernmentAdda.com

Statement 2: y is a 3 digit number. The mistake that students make here is that they carry information from Statement 1 to Statement 2. Your brain has been polluted by being told that x is a prime number in the first statement! You now assume that x is a positive integer but you don’t know that from the question stem. If x were a positive integer (as all prime numbers are), then the only way for y to be an odd three-digit number and xy = 222 is for x = 2 and y = 111. However, you do not know that x is positive nor that it is an integer. x = -2 and y = -111 222 would work, as would x =   and y = 999. Each of these variations fits both 999 the information from the question stem and Statement 2. There is not just one value for x, so this statement is not sufficient, and the correct answer is A. Note: It is essential that you always consciously ask yourself whether you are carrying information from the other statement. Make sure you “reset” your brain every time you move to the next statement. 117. A x Question Type: Yes/No. The question asks: “Is   an integer?” y   Given information in the question stem or diagram: x and y are positive integers.

284

SOLUTIONS

Statement 1: Every factor of y is also a factor of x. This means that whatever factors are in the denominator, those same factors are also in the numerator. So if y = 30 (with factors of 2 • 3 • 5) then x will have at least these factors. In other x words, x will be a multiple of 30. If x has all of the factors of y then   will always y   be an integer. This statement is sufficient, and the answer is either A or D. Statement 2: Every factor of x is also a factor of y. This statement may seem to be identical to Statement 1, but it is not! All of the factors of x will also be present in y, but y could contain other factors. For instance, x could be 30 and y could be 60, meeting all the conditions in this statement. Or they could each be equal to x 30. This statement allows for   to be the integer 1 but also many non-integers, y   so it is not sufficient and the correct answer is A. Note: In any question like this, where the statements appear to be the same, you should be highly suspicious of choice D as the correct answer! 118. E Question Type: What Is the Value? This question asks for the sum of the first 98 terms of the sequence.

GovernmentAdda.com Given information in question stem or diagram: There is a lot of information to digest before moving to the statements. a1 = x; a2 = y; a3 = z; a4 =3; an = an-4 for n > 4. This means that not only is a8 = a4 = 3, as each term is equal to the term four before it. This also means that a1 = a5 = a9; a2 = a6 = a10; a3 = a7 = a11; etc. The sequence begins: x, y, z, 3, x, y, z, 3, and carries on infinitely in that pattern. Therefore, if you can determine the values of the first four terms in the sequence you can extrapolate from those in cycles of four. If you divide 98 by 4 you get 24 with a remainder of 2. This means that the sequence goes through 24 cycles of x, y, z, 3, through the 96th term, and then terms 97 and 98 are x and y, respectively. This is important because in order to find the sum of the first 98 terms you need not only the sum of the four-term pattern (which you can multiply by 24 to get the sum of the first 96 terms) but also the sum of the extra x + y from the 97th and 98th terms. Leveraging all this, you can remove the sequence information from the question and realize that the question is really: What is 24 (x + y + z + 3) + x + y? Now it is just an algebra problem!

SOLUTIONS

Statement 1: x = 5. This statement is clearly not sufficient, as there is no way to determine the value of y or z. Eliminate choices A and D. Statement 2: y + z = 2. This statement is also clearly insufficient as, most conspicuously, it tells you nothing about x. But also notice that you need the individual value of y to fill in the algebra above, as you need 25y but only 24z. The statement is not sufficient. Eliminate choice B.

285

Together: Taken together with the question stem, the two statements allow you to get the sum of the first four terms: x + y + z + 3 = 10. Thus the sum of the first 96 terms is 24*10 or 240. However, you still need to be able to give a value for those last two numbers “x + y.” You have the value of x from Statement 1, but Statement 2 combines two terms in the sequence and therefore does not give you a clear value for the variable y, which represents a2 and therefore a98. Even together these statements are not sufficient. If the question had asked for sum of the first 96 or 97 terms, or the first 99 or 100 terms, you could have supplied that specific value. It is only because you cannot separate y and z—and because the 98th term falls directly between the two— that you cannot solve. The correct answer is thus E. Note: This is a very hard C vs. E choice. Almost everyone can get it down to those two choices, but it requires careful analysis to realize that the answer is E. As in so many DS questions, the key again is to simplify the question and all the given information before you move to the statements. 119. D Question Type: Yes/No. This question asks: “Is 14,743 + x prime?”

GovernmentAdda.com

Given information in the question stem or diagram: x is a positive integer and x < 10. That means that x = 1, 2, …, 9 and that 14743 + x will equal 14,744, 14,745, …, 14,752. Also note: It is difficult to confirm that a large prime number is prime. For instance there is no good way to determine whether 1,000,001 is prime. However, it is very easy to show that 1,000,011 is NOT prime, because you know it is divisible by 3 (as the sum of its digits is divisible by 3). So your strategy here should really be to try to prove that the possible values are not prime, by finding factors of each possible value. Statement 2 is easier because it gives you a specific value for x, so you should begin there. Statement 2: x2 = 36. Normally x = 6 or -6, but you have the fact that x is positive. So x must equal 6. This statement is sufficient because when you add 6 to 14,743 you will get a single value, and that number will either be a prime number or not; you do not really care which. You only need to know that the answer will be a consistent “yes” or a consistent “no” with only one number involved. If you are interested, 14,749 is not prime as it is clearly divisible by 7 so the answer is “no”! The answer is either B or D. x Statement 1: “   is odd.” With conceptual understanding you see that the only 2 numbers less than 10 that work with this statement are even numbers (so that

286

SOLUTIONS

they can still yield and integer when divided by 2) that are not multiples of 4 (so that the integer is not an even one). So x = 2 or x = 6. This means that you would be adding either 2 or 6 to 14,743. If you add 2 then you get 14,745, which is not prime since it ends in 5. If you add 6 you get 14, 749, which is not a multiple of 2, 3, or 5. However it is a multiple of 7. A quick check of division shows that 14,749 = 7*2,107. Since neither of the numbers are prime, you have a consistent “no” and this statement is also sufficient. Again, you should note that the only plausible answer to this question will be “no” or the question would be unfair. 14,747, for instance, is prime but there is no way in two minutes you could EVER prove that, so the testmakers could not have x be 4! The correct answer is D. 120. E Question Type: What Is the Value? This question asks for the specific number of bushels of apples harvested yesterday. Given information in the question stem or diagram: There is a lot of confusing information given in the question stem. If Steve harvests 42 or fewer bushels he is paid y dollars per bushel; if he harvests more than 42 per day then he is paid y dollars per bushel for the first 42 and 1.5y for each additional bushel.

GovernmentAdda.com After reading through this complicated word problem, it is fairly clear that each statement alone could not be sufficient. Statement 1: “Yesterday Steve was paid $180 for the apples he harvested.” This statement means that Steve might have only harvested one bushel if the rate of y was $180 per bushel or he could have harvested 30 bushels if the rate of y was $6 per bushel. This is clearly not sufficient. Eliminate choices A and D. Statement 2: “Today Steve harvested yesterday’s amount + 10 bushels and earned $240.” This statement alone only gives you information referencing the amount he harvested yesterday (which is unknown using only Statement 2). So, this is also not sufficient. This is a C vs. E question and the choice between those is very, very difficult.

SOLUTIONS

Together: Using the two statements together, it appears that you can determine one value for y. If Statement 2 tells you that the additional 10 bushels added $60 to his revenue, then the value of y appears to be $6. Using this value of y, you can see that he would have harvested 30 bushels yesterday for $180 and 40 bushels today for $240. Most students will then pick answer C and assume the answer is 30 for the amount of bushels bought yesterday. However, the question stem gave us two potential scenarios. Imagine if those

287

10 additional bushels that Steve harvested were already over the threshold of 42. Then the value of y would not be $6 (10y = 60); it would be $4 (10(1.5y) = 60), and y would be $4 per bushel. Looking back at Statement 1 and using y = $4 it is possible that Steve harvested 44 bushels yesterday. He would have earned $4 per bushel for the first 42 ($168) and then the increased rate of $6 per bushel for the remaining 2, which would be $12. Added together this gives the total value of $180. Since there are two possible scenarios for the number of bushels harvested yesterday, the answer is E. This question is the ultimate test of Resource Management skills and your ability to Play Devil’s Advocate. Did you realize that there were two possible scenarios given the stated information? Probably not, but for those who want to see them more clearly you can look below: Scenario 1: If the total number of bushels harvested on both days were less than 42, then y = $6. Yesterday: (30 bushels)($6 per bushel) = $180 Today:

(40 bushels)($6 per bushel) = $240

GovernmentAdda.com Answer to question = 30 bushels harvested yesterday.

Scenario 2: If the total number of bushels harvested on both days were more than 42, then y = $4 and he would have earned $6 per bushel for each one above 42. Yesterday: (42 bushels)($4 per bushel) = $168 + (2 bushels) ($6 per bushel) = $12 Total = $180 Today:

(42 bushels)($4 per bushel) = $168 + (12 bushels)($6 per bushel) = $72 Total = $240

Answer to question = 44 bushels harvested yesterday.

288

SOLUTIONS

121. A (x – y + z) x y x Question Type: Yes/No. This question asks: whether   <  –  –   .  2z 2z 2z y         Given information in question stem or diagram: yz does not equal zero. This is only given to assure that no denominators are equal to 0. Before moving to the statements, it is a good idea to use algebraic manipulation to try to simplify this very difficult inequality question. The first move is suggested by the fact that three of the four fractions have a denominator of 2z. While you might want to multiply both sides of the inequality by 2z, you are not allowed to do that as you do not know the sign of the variable z. However, you can get all of the 2z x x–y+z denominator terms on one side by subtracting   from   and adding   2z   2z y   to both sides. After you do that, you see that the new question is: 2z   x–y+z x y x Is   –  +  <-  ? 2z y    2z   2z   Now you can combine the numerators on the left side (the x’s and y’s each cancel) and put over one denominator: Is  

z x <-  ? 2z   y 

GovernmentAdda.com This can be further simplified by canceling the z’s to show that the question is really: 1 x Is   < –   ? You can transform this so that you’re not dealing with the negative 2  y  sign next to the variables. Multiply both sides by -1 (remember to flip the sign when doing so!) to make the question much more manageable: Is

x 1 <-  ? y  2 

Now that you’re down to only two variables and being asked about a simple x-to-y ratio, it’s time to attack the statements. x 1 Statement 1: < -   . You have done all of the work already in manipulating y  2 the question, as this statement perfectly matches the rephrased question. This statement is clearly sufficient, and the answer is A or D. Note: If you did not manipulate the question algebraically, this statement would be very difficult to incorporate and evaluate in a reasonable period of time.

SOLUTIONS

Statement 2: “xy < 0.” This is very close to being sufficient but not does not give x x enough information to find the limit for . This statement proves that must y  y 1   be negative, but it does not prove that it is less than -   . For instance, y could 2 be 4 and x could be -1, and this would give a “no” answer. Or x could be 4 and y could be -1 and then it would be a “yes” answer. This statement is not sufficient. The answer is A.

289

122. D Question Type: What Is the Value? This question asks for the ratio of chocolate chip cookies to brownies. Given information in question stem or diagram: Chocolate chip cookies = $1.30 each; brownies = $1.50 each Statement 1: The average price for the items sold was $1.42. This information allows you to determine the ratio of cookies to brownies using the principles of the weighted average. The relative distance of the price of each item to the overall average price is the inverse of the relative weight of each item. Of course, you do not need to do this to prove sufficiency (you know you CAN do it) but here is the process: First find the distance for each of the prices to the average price. Cookie price to average price is a distance of 12 cents and brownie price to average price is a distance of 8 cents. So the distance ratio is CC:B = 12:8. Now simply invert the numbers and simplify to see the actual ratio: CC:B = 8:12 and CC:B = 2:3. Since you have the average price and the prices of each of the items you can judge the relative weight of the items and create a ratio. This statement is sufficient and the answer is either A or D.

GovernmentAdda.com

Note: If the question had asked for the number of brownies or number of chocolate chip cookies, this statement would not have been sufficient. Statement 2: “The total price for all of the items = $14.20.” This information allows you to create the equation 1.3C + 1.5B = 14.20. This may seem like it is not sufficient since you have two variables with one equation. However, because C and B must be whole numbers, there is a limit put on the values of B and C that you must consider. The fact that the variables are only 20 cents apart and that the total value is a relatively small $14.20 means that it is likely that there is only one value for C and B. To prove that this statement is sufficient by itself, consider the limits of the total number of treats that could have been bought. Try 11 items first (because the first statement gives 10 total items this would be a logical place to test). If you buy 11 of the cheaper item (cookies), you have spent too much money! 11 cookies would total 11 • 1.30 = $14.30. This is too much; you know that the total is only $14.20. Therefore the total number of items must be less than 11. Now try 9 items. If you buy 9 of the more expensive items (brownies), you have not spent enough money! 9 brownies = 9 • $1.50 = $13.50, which is less than the $14.20 you know was spent. Therefore, the total number must be greater than 9. The total number of items must = 10, and so you actually have two equations. The equation 1.3C + 1.5B = 14.20 has embedded within it the equation C + B = 10. This statement is also sufficient, and the correct answer is D.

290

SOLUTIONS

Note: This is another classic “C trap” question. Choice C is absurdly easy on this question, so it is unlikely to be the correct answer. Even if you note the C trap and avoid picking choice C it is still likely you will pick A. Most people will miss the unique equation in the second statement that allows you to actually solve for the variables. The integer constraint implicit in the context of the question makes all the difference and provides a reward for those who take care to Leverage Assets. 123. A Question Type: What Is the Value? This question asks: “How many numbers in Set T are multiples of 21?” Given information in the question stem or diagram: Set T is a finite set of positive consecutive multiples of 14. It might be a good idea to think about the relationship between multiples of 14 and 21 before moving on. Multiples of 14 are multiples of 2 • 7, and multiples of 21 are multiples of 3 • 7. In order to be both, a number needs to be a multiple of 6 • 7, or 42. Indeed 42 is the smallest positive number that is a multiple of both 14 and 21. What you need to realize before moving to the statements is that every third multiple of 14 is also a multiple of 21.

GovernmentAdda.com Statement 1: Set T consists of 30 integers. If every third multiple of 14 is a multiple of 21, then there would be 10 multiples of 21 in Set T. A quick glance at Statement 2 indicates that it may be important to know if the first member of the set is a multiple of 21, which would make this statement alone not sufficient. However, since it is every third member of Set T that is a multiple of 21, and since 30 is a multiple of 3, there will be exactly 10 sets of 3, and each of those will have one multiple of 21. It does not matter if the multiple of 21 is the first number in each group of 3 or the last. This is because 30 is an exact multiple of 3. It the set contained 31 integers, then it would be important to know if the multiple of 21 was first, but in this case you do not need that information. This statement is sufficient alone. The answer is either answer A or D.

SOLUTIONS

Statement 2: The smallest integer in Set T is a multiple of 21. This statement is not sufficient alone since it does not indicate the number of members of the set. The correct answer is A. Note: This question is a very tricky example of the “Why Are You Here?—Temptation” construct. A good test-taker will very carefully analyze this second statement and prove whether it matters in relation to the first. In this case, it does not matter, so the answer is A, but in other cases it certainly will, and the answer would be C. The key here is that you take the time to make sure that the second statement does or does not matter.

291

124. D Question Type: What Is the Value? This questions asks for the value of the integer t. Given information in the question stem or diagram: r, s, and t are positive integers, and rst = 343. Statement 1: “r < s < t.” This statement not only puts the variables in ascending order but also subtly guarantees that the variables are all different numbers— no repeats. It would be a good idea at this point to get the prime factors of 343. 343 = 73. The only factors of 343 are 1, 7, 49, and 343. Even before you get to Statement 1, you might recognize from the question stem that the only possible sets of values for r, s, and t are 1 • 1 • 343, 1 • 7 • 49, and 7 • 7 • 7. With Statement 1 the only possible set is 1 • 7 • 49, so t must be 49.The answer is A or D. Statement 2: “rs = 7.” This means that 7t = 343. In order for rst to equal 343, t must = 49. This statement is clearly sufficient. The correct answer is D. Note: This is a great example of another important construct discussed in the lesson portion of this book. Statement 2 is a very easy statement: T is clearly equal to 49, and this is quite obviously sufficient. Whenever you have one clearly sufficient statement, the other one will almost always be hard and counterintuitive! The first statement does not seem sufficient at first glance, but with a careful analysis of factors, you see that it is indeed sufficient. The easy 2nd statement is a hint to dig deeper in the first statement and leverage every piece of information that is available.

GovernmentAdda.com

125. A Question Type: What Is the Value? This question asks for the unit’s digit of n4a + 2 – n8a. Given information in the question stem or diagram: n and a are positive integers. Statement 1: In the Algebra lesson, you learned that all numbers have a repeating pattern of units digits when they are raised to certain powers. For instance, here is the progression for 3, which is given as the value for n in Statement 1: Units Digit of 31 = 3 Units Digit of 32 = 9 Units Digit of 33 = 7 Units Digit of 34 = 1

292

SOLUTIONS

Units Digit of 35 = 3 Units Digit of 36 = 9 Units Digit of 37 = 7 Units Digit of 38 = 1 As you can see, the pattern repeats every 4. This Data Sufficiency question is asking whether you can determine the units digit of n(4a+2) – n(8a). If you can determine the units digit of each of those terms, then you can calculate the difference between their units digits. In Statement 1, you learn that n is 3, and it seems like you must also know something about a to answer the question. However, a closer look at the exponents shows that it does not matter what value a is. Remember that 3 raised to any multiple of 4 will always end in 1 (for instance, 312 or 324), and 3 raised to any multiple of 4 + 2 (for instance 316 or 326) will always end in 9. Therefore, regardless of what a is, n(4a+2) will end in 9 and n(8a) will end in 1. Therefore you can answer the question, and the answer must be A or D. Note: This is only possible because the numbers before the exponent a are multiples of 4.

GovernmentAdda.com Statement 2: A quick look at this statement and it is clearly insufficient. You must know something about n to answer the question. Since most people do not think that Statement 1 is sufficient, this is an important “Why Are You Here?” statement. The good test-taker will at this point try a few odd and even values for a to see if it makes a difference. In doing that, you will quickly prove to yourself that you do not need to know anything about a in order for statement 1 to be sufficient. This is another classic example of “Why Are You Here?—Temptation,” and if you play the Data Sufficiency game properly—leveraging hints and deciding if information is really important—you can get this problem correct every time!

SOLUTIONS

293

GovernmentAdda.com

294

ANSWER KEY

ANSWER KEY

LESSONS

HOMEWORK

1. B

10. B.

18. C

36. A

54. C

72. D

90. E

108. B

2. C

11. E

19. E

37. B

55. B

73. E

91. C

109. A

3. D

12. C

20. B

38. E

56. A

74. A

92. E

110. D

4. A

13. D

21. E

39. C

57. D

75. E

93. D

111. C

5. E

14. A

22. B

40. E

58. E

76. D

94. A

112. C

6. E

15. A

23. E

41. C

59. B

77. B

95. D

113. E

7. C

16. A

24. C

42. D

60. A

78. B

96. C

114. C

GovernmentAdda.com

8. A 9. C

17. E

25. B

43. C

61. B

79. A

97. B

115. D

26. B

44. C

62. E

80. D

98. A

116. A

27. D

45. D

63. A

81. E

99. A

117. A

28. E

46. D

64. D

82. C

100. B

118. E

29. D

47. D

65. B

83. D

101. A

119. D

30. D

48. E

66. D

84. A

102. A

120. E

31. C

49. B

67. E

85. E

103. E

121. A

32. A

50. C

68. E

86. D

104. B

122. D

33. A

51. E

69. A

87. C

105. A

123. A

34. E

52. D

70. C

88. E

106. B

124. D

35. A

53. A

71. B

89. E

107. A

125. A

295

GovernmentAdda.com

296

Daily Visit :

[GOVERNMENTADDA.COM]

300+ Mixed Reasoning Questions With Solution

GovernmentAdda.com

Governmentadda.com | IBPS SSC SBI RBI RRB FCI LIC RAILWAYS

1

Daily Visit :

1.Statements: a. Some sheets are rods. b. No sheet is a foil. c. All rods are marbles. Conclusions: I. No foil is a marble. II. Some sheets are marbles. III. Some rods are foils. IV. Some rods are not foils. a) Only I follows b) only II follows c) Either III or IV follows d) Only I and either III or IV follow e) Only II and IV follow

2.Statements: a. Some MLAs are not honest. b. Some MLAs are dishonest. c. No honest is a leader. Conclusions: I. Some MLAs are leaders. II. All dishonest are honest. III. Some dishonest are MLAs. IV. No leader is dishonest. a) Only IV follows b) Only III and II follow c) Only I follows d) Only I and IV follow e) None of these

[GOVERNMENTADDA.COM]

3.Statements: a. Some stones are not rods. b. All rods are black. c. No black is a crow. Conclusions: I. Some crows are rods. II. Some blacks are stones. III. Some crows are blacks. IV. Some stones are rods. a) Only I follows b) Only II follows c) Only I and IV follow d) Only IV follows e) None of these

4.Statements: a. Some garbages are money. b. All papers are garbages. c. All money are coins. Conclusions: I. Some papers are coins. II. Some garbages are coins. III. No money is a paper. Governmentadda.com | IBPS SSC SBI RBI RRB FCI LIC RAILWAYS

2

Daily Visit :

[GOVERNMENTADDA.COM]

IV. All coins are garbages. a) Only I follows b) Only I and III follow c) Only III follows d) Only II and III follow e) Only II follows

Step IV: again 36 early 7 we 17 morning in day 11 Step V: again 36 early 7 in we 17 morning day 11 Step VI: again 36 early 7 in 17 we morning day 11 Step VII: again 36 early 7 in 17 day we morning 11 Step VIII: again 36 early 7 in 17 day 11 we morning Step IX: again 36 early 7 in 17 day 11 morning we And Step IX is the last step. 6.If the following is the II step of an input what will be Vth step? Step II: After 89 she 38 wins 11 Olympic 22 the 7 a) after 89 she 7 the 22 Olympic 11 wins 38 b) after 89 Olympic she 38 wins 11 22 the 7 c) after 89 Olympic 7 she 38 the wins 11 22 d) after 89 Olympic 7 she 38 the 11 wins 22 e) None of these 7.Which of the following is the last step for the Input „eat 9 fast icecream 22 3 umbrella cat 5‟? a) cat eat 9 fast 5 icecream 22 umberlla 3 b) eat 22 icecream 3 umbrella 9 cat 5 fast c) eat 22 umbrella 3 icecream 9 cat 5 fast d) eat 22 icecream 3 umbrella 5 cat 9 fast e) None of these

5.Statements: a. Some pants are shirts. b. No face is a pant. c. No pant is a flower. Conclusions: I. No flower is a face. II. No face is a flower. III. Some shirts are not faces. IV. Some shirts are pants. a) Only I and II follow b) Only III and IV follow c) Either I or II follows d) Only IV follows e) None of these

8.Which step will be the last step for the Input „elephant 17 free open 41 27 danger 15‟? a) IV b) V c) VI d) VII e) None of these 9.Which word/number will be at 4th from the left in step V for the given input in above question number (8)? a) 41 b) danger c) open d) 15 e) None of these

Directions (Q.6-10): A word arrangement Machine, when given a particular input, rearranges it following a particular rule. The following is the illustration of the input and the steps of rearrangement. Input: we again 36 early 17 morning in day 7 11 Step I: again we 36 early 17 morning in day 7 11 Step II: again 36 we early 17 morning in day 7 11 Step III: again 36 early we 17 morning in day 7 11

10. Which word/number will be 3rd to the right of “41” in step IV for the given input in question number (8)? a) open b) danger c) 15 d) 17 e) None of these

Governmentadda.com | IBPS SSC SBI RBI RRB FCI LIC RAILWAYS

3

Daily Visit :

[GOVERNMENTADDA.COM]

Solutions (6-10): Word arrangement machine first arranges words having first letter vowel in alphabetical order, after that words having first letter consonant will be arranged in alphabetical order. Alternately the numbers are choosen such that – greatest, lowest, 2nd greatest, 2nd lowest and so on. 6. (c); Step II: after 89 she 38 wins 11 olympic 22 the 7 Step III: after 89 olympic she 38 wins 11 22 the 7 Step IV: after 89 olympic 7 she 38 wins 11 22 the Step V: after 89 olympic 7 she 38 the wins 11 22 7. (b) 8. (c) Input: elephant 17 free open 41 27 danger 15 Step I: elephant 41 17 free open 27 danger 15 Step II: elephant 41 open 17 free 27 danger 15 Step III: elephant 41 open 15 17 free 27 danger Step IV: elephant 41 open 15 danger 17 free 27 Step V: elephant 41 open 15 danger 27 17 free Step VI: elephant 41 open 15 danger 27 free 17 9. (d) 10. (b) Directions (11-15): Study the following information carefully and answer the given questions: Ten persons are sitting in two parallel rows containing 5 persons each in such a way that there is an equal distance between adjacent persons. In the 1st row E, F, G, H and I are sitting and all of them are facing south. In the 2nd row T, U, V, W and Z are sitting and all of them are facing towards north direction but not necessarily in the same order. In the given seating arrangement each member sitting in a row faces another member of the other row. The one who is an immediate neighbor of H is facing W. F does not sit at the extreme ends of the row. V and Z are immediate neighbors. Only one person sits between I and G. Z sits second to the left of the person who faces I. E faces the person who is on the immediate left of Z. T sits at the left end of the row. E and H are not immediate neighbours. Q11. Who among the following sits between G and I ? (a) H

(b) F (c) E (d) T (e) None of these Q12. Who among the following faces G ? (a) U (b) V (c) H (d) Z (e) None of these Q13. Who among the following sits second to the right of the person who faces U ? (a) F (b) I (c) W (d) V (e) None of these Q14. How many persons sits between E and H ? (a) One (b) Two (c) Three (d) Can‘t be determined (e) None of these Q15. Four of the following five are alike in a certain way based on the given seating arrangement and hence form a group. Which is the one that does not belong to that group ? (a) GZ (b) FV (c) WI (d) UH (e) EV Solutions (11-15):

S11. Ans.(b) Ans. S12. Ans.(d)

Governmentadda.com | IBPS SSC SBI RBI RRB FCI LIC RAILWAYS

4

Daily Visit :

[GOVERNMENTADDA.COM] (b) #7A (c) 5*K (d) W2I (e) 3@H

Ans. S13. Ans.(a) Ans. S14. Ans.(c) Ans. S15. Ans.(e) Ans.

Directions (1-5): These questions are based on the following letter/number/symbol arrangement. Study it carefully and answer the questions. Z$3H@8B1#A7CLJU5K*EWI2F%T Q1. If all the vowels in the above arrangement are replaced by the letter following it in the English alphabetic series, how many alphabets will appear twice in the given arrangement? (a) None (b) One (c) Two (d) Three (e) More than three Q2. Which of the following elements is the sixth to the right of fourteenth from the right? (a) 8 (b) # (c) J (d) E (e) None of these Q3. How many such numbers are there in the above arrangement, each of which is immediately preceded by a vowel and also immediately followed by a consonant? (a) None (b) One (c) Two (d) Three (e) More than three Q4. Four of the following five are alike in a certain way based on their position in the above arrangement and so form a group. Which is the one that does not belong to the group? (a) BA#

Q5. If the first eight digits/symbols/letters from the left are interchanged in a way that the first component is exchanged with the eighth, the second with the seventh and so on, which of the following would be the sixth from the left? (a) @ (b) 3 (c) H (d) B (e) None of these Solutions (1-5): S1. Ans.(d) Sol. Three S2. Ans.(e) Sol. None of these. S3. Ans.(d) Sol. Three S4. Ans.(a) Sol. BA# S5. Ans.(b) Sol. 3 Directions (6-10): In each of the questions below are given four statements followed by four conclusions numbered I, II, III and IV. You have to take the given statements to be true even if they seem to be at variance from commonly known facts. Read all the conclusions and then decide which of the given conclusions logically follows from the given statements disregarding commonly known facts. Q6. Statements: Some pens are rooms. All rooms are walls. Some walls are bricks. All bricks are slates. Conclusions: I. Some slates are walls. II. Some walls are pens. III. Some bricks are rooms. IV. Some slates are rooms. (a) Both I and III follow (b) Both II and III follow

Governmentadda.com | IBPS SSC SBI RBI RRB FCI LIC RAILWAYS

5

Daily Visit :

[GOVERNMENTADDA.COM]

(c) Both I and II follow (d) Both III and IV follow (e) None of these

Q7. Statements: Some chairs are pencils. Some pencils are bottles. Some bottles are bags. Some bags are books. Conclusions: I. Some books are pencils. II. Some bottles are chairs. III. No book is pencil. IV. Some bags are chairs. (a) Only I follows (b) Either I or III follows (c) Only III follows (d) Only IV follows (e) None of these

Q8. Statements: Some roads are buses. All buses are trains. Some trains are trucks. All trucks are kites. Conclusions: I. Some trucks are roads. II. Some kites are buses. III. Some trains are roads. IV. Some kites are trains. (a) None follows (b) Only I follows (c) Only II follows (d) Only III follows (e) None of these

Q9. Statements: All beads are rings. All rings are bangles. All bangles are tyres. All tyres are pendants. Conclusions: I. Some pendants are beads. II. Some tyres are rings. III. Some bangles are beads. IV. Some pendants are rings. (a) Both I and II follow (b) Only I, II and III follow (c) Only II, III and IV follow (d) Only I, III and IV follow (e) All follow

Q10. Statements: Some desks are fruits. All fruits are flowers. No flower is branch. Some branches are roots. Conclusions: I. Some roots are flowers. II. No desk is branch. III. Some flowers are desks. IV. Some branches are desks. (a) Only either II or IV follows. (b) Only III follows (c) Both Either II or IV and III follow (d) Both III and IV follow (e) None of these

Governmentadda.com | IBPS SSC SBI RBI RRB FCI LIC RAILWAYS

6

Daily Visit :

[GOVERNMENTADDA.COM]

Directions (11-15): Study the following information carefully and answer the given questions: In a certain code language „milk is very tasty‟ is written as ‗ta la ja sa‟, „tea is black‟ is written as „ha ja ka‟ and „sweet milk and tea‟ is written as „ha pa sa ra‟. Q11. What is the code for „milk‟? (a) ja (b) la (c) sa (d) pa (e) None of these

S11. Ans.(c) Sol. S12. Ans.(b) Sol. S13. Ans.(c) Sol. S14. Ans.(d) Sol.

Q12. „black tea‟ can be coded as (a) ha ja (b) ka ha (c) pa ha (d) Can‘t be determined (e) None of these

S15. Ans.(c) Sol.

Q13. Which of the following is coded as „ta‟? (a) very (b) sweet (c) Either tasty or very (d) black (e) None of these

T, U, V, W, X, Y and Z live on a seven-storey building but not necessarily in the same order. The lowermost floor of the building is numbered 1, and the topmost floor is numbered 7. Each of them belongs to different city, viz, Goa, Chandigarh, Delhi, Noida, Mumbai, Pune and Shimla.

Q14. Which of the following is the code for „black‟? (a) ra (b) pa (c) Either sa or ka (d) ka (e) None of these

T lives on an odd-numbered floor but not on floor number third. The one who belongs to Mumbai city live immediately above T. Two persons live between W and the one who belongs to Mumbai city. The one who belongs Chandigarh city live on one of the oddnumbered floors but above W. Only three persons live between V and the one who belongs to Chandigarh city. The one who belongs to Delhi lives immediately above V. The one who belongs Goa city live immediately above the one who belongs to Shimla city. Z lives on an odd-numbered floor. Only one person lives between U and X. U lives on one of the floors above X. Neither V nor T belongs to Noida city. X does not belong to Delhi.

Q15. „ja‟ is the code for (a) milk (b) and (c) is (d) tasty (e) None of these

Directions (1-5): Study the following information carefully and answer the questions given below:

Solutions (11-15): Governmentadda.com | IBPS SSC SBI RBI RRB FCI LIC RAILWAYS

7

Daily Visit :

[GOVERNMENTADDA.COM]

Q1. How many people live(s) above the floor on which Z lives? (a) One (b) No one. (c) Three (d) Four (e) Two Q2. Who among the following lives on floor number sixth? (a) Z (b) Y (c) V (d) W (e) U Q3. If X and V interchange their position then who lives immediately below the floor on which X lives? (a) W (b) Y (c) X (d) No one. (e) T Q4. Which of the following city does X belong? (a) Pune (b) Noida (c) Goa (d) Chandigarh (e) None of these Q5. If all person live in building according to alphabetical order from top to bottom, then position of how many remain unchanged? (a) Five (b) Two (c) Three (d) Four (e) None of these

S1. Ans. (b) S2. Ans. (e) S3. Ans.(d) S4. Ans. (c) S5. Ans. (b) Directions (6-10): Study the following information carefully and answer the questions that follow: A ÷ B means A is son of B A × B means A is sister of B A + B means A is brother of B A – B means A is mother of B Q6. How is G related to H in the expression „G × R + V † H‟? (a) Sister (b) Daughter (c) Son (d) Mother (e) None of these

Q7. Which of the following expressions represents „B is the husband of A‟? Governmentadda.com | IBPS SSC SBI RBI RRB FCI LIC RAILWAYS

8

Daily Visit :

[GOVERNMENTADDA.COM]

(a) A × I – E + B (b) A – I + E ÷ B (c) A + I ÷ E × B (d) A ÷ I × E + B (e) None of these

Q8. How is V related to T in the expression „T † R + V × N‟? (a) Niece (b) Father (c) Uncle (d) Aunt (e) Mother

Q9. How is P related to J in the expression „J × K † M – P‟? (a) Sister (b) Brother (c) Father (d) Either (a) or (b) (e) None of these

Q10. Which of the following expressions represents „J is wife of E‟? (a) E ÷ F × G + H – J (b) E × G ÷ H + F – J (c) J – H × G ÷ E + F (d) Both (a) and (b) (e) None of these

Directions (11-15): Study the following information carefully and answer the given questions: A, B, C, D, E, F and G are seven friends. They play three types of games, viz Hockey, Football and Cricket. Each game is played by at least two players. Each one of them has a favourite colour, viz Pink, Blue, White, Green, Yellow, Red and Black, but not necessarily in the same order. B likes Yellow and does not play Cricket. The one who likes Black plays the same game as E. C likes Blue and plays the same game as G. D plays Football only with the one who likes Pink. G plays neither Football nor Cricket. F does not like Black. G likes neither Green nor White. D does not like Green. E does not like Pink.

Governmentadda.com | IBPS SSC SBI RBI RRB FCI LIC RAILWAYS

9

Daily Visit :

[GOVERNMENTADDA.COM] S12. Ans.(d)

Q11. Which of the following groups plays Hockey? (a) B, G (b) A, B, C (c) B, C, G (d) D, G, B (e) None of these Q12. Who likes White Colour? (a) A (b) G (c) F (d) D (e) None of these Q13. Which of the following colours does A like? (a) White (b) Black (c) Pink (d) Either Black or Red (e) None of these Q14. Which of the following combinations is true? (a) A–Black–Cricket (b) G–Pink–Cricket (c) B–Yellow–Football (d) D–White–Hockey (e) None is true? Q15. Who likes Pink? (a) G (b) A (c) E (d) C (e) F

S11. Ans.(c)

S13. Ans.(b) S14. Ans.(a) S15. Ans.(e)

Directions (1-5): In each question below are given two/three statements followed by two conclusions numbered I and II. You have to take the given statements to be true even if they seem to be at variance with commonly known facts. Read all the conclusions and then decide which of the given conclusions logically follows from the given statements, disregarding commonly known facts. Give answer. (a) if only conclusion I follows. (b) if only conclusion II follows. (c) if either conclusion I or II follows. (d) if neither conclusion I nor II follows. (e) if both conclusions I and II follow. Q1. Statements: All plates are glasses. Some cups are glasses. Conclusions: I. At least some cups are plates. II. Some glasses are cups.

Q2. Statements: All trolleys are lamps. No lamp is a chair. Conclusions: I. At least some trolleys are chairs. II. Some chairs are definitely not trolleys. Governmentadda.com | IBPS SSC SBI RBI RRB FCI LIC RAILWAYS

10

Daily Visit :

[GOVERNMENTADDA.COM]

Q5. Statements: Some teachers are doctors. No doctor is a lawyer. Conclusions: I. Some teachers are not lawyers. II. Some lawyers are doctors.

Q3. Statements: Some clothes are shirts. All shirts are paints. Conclusions: I. All paints being clothes is a possibility. II. Some shirts are clothes.

Directions (6-10): Study the following information carefully and answer the questions given below: A, B, C, D, E, F, G and H are sitting around a circular table facing the centre. H is third to the right of C and second to the left of E. B is not an immediate neighbour of H or C. F is second to the right of D and is an immediate neighbour of C. G is not the neighbour of E.

Q4. Statements: No sand is a stone No sand is a tree. Conclusions: I. No stone is sand. II. No tree is a stone.

Q6. Who among the following is second to the right of C? (a) H (b) G (c) F (d) E (e) None of these Q7. Who among the following is an immediate neighbour of H and E(both)? (a) A (b) B (c) C (d) G (e) None of these Q8. In which of the following pairs the second person is sitting on the immediate right of the first person? (a) A, H (b) C, D (c) G, H Governmentadda.com | IBPS SSC SBI RBI RRB FCI LIC RAILWAYS

11

Daily Visit :

[GOVERNMENTADDA.COM]

(d) E, H (e) F, C

Sol.

Q9. Who among the following is second to the left of B? (a) C (b) H (c) F (d) A (e) None of these

Q11. If the letters of the word AMERICA are arranged in the English alphabetical order from left to right, the position of how many letters will remain unchanged? (a) None (b) One (c) Two (d) Three (e) None of these

Q10. Who among the following is opposite D? (a) A (b) G (c) H (d) E (e) None of these

Q12. In a certain code language TREAT is written as UBFSU and HABIT is written as UJCBI. How is AGREE written in that code language? (a) FSHBF (b) FSHFB (c) FFSHB (d) FFQBH (e) None of these

S6. Ans.(b) Sol. S7. Ans.(a) Sol. S8. Ans.(c) Sol. S9. Ans.(d) Sol. S10. Ans.(c)

Q13. Four of the following five are alike in a certain way and hence form a group. Which is the one that does not belong to that group? (a) EV (b) KP (c) IR (d) OL Governmentadda.com | IBPS SSC SBI RBI RRB FCI LIC RAILWAYS

12

Daily Visit :

[GOVERNMENTADDA.COM]

(e) CW

Directions (1-5): Study the following information carefully and answer the questions given below:

S13. Ans.(e) Sol. Q14. How many such pairs of letters are there in the word STARVATION each of which has as many letters between them in the word as in the English alphabet? (In both forward and backward directions) (a) None (b) One (c) Two (d) Three (e) More than three.

M, N, P, Q, R, S, T and U are eight family members. There are three generation in the family. Three couples are there in the family. They along with their children live in three different cities, viz Kolkata, Patna and Ranchi, but not necessarily in the same order. Q is mother of M and R. M is father of S and uncle of T. The one who is married with U lives in Kolkata. U is the only sister-in-law of N, who is daughter-in-law of P. S neither lives in Patna nor with her aunt U. T is son of U. Each couple lives in a same city with their child except P and Q. Q1. Who among the following is husband of Q? (a) U (b) N (c) P (d) Can‘t be determined (e) None of these Q2. How is R related to M? (a) Brother (b) Sister (c) Sister-in-law (d) Can‘t be determined (e) None of these

Q15. What should come in place of question mark (?) in the following series? JK, MN, QR, VW, ? (a) BC (b) XY (c) YZ (d) AB (e) None of these

Q3. How is S related to P? (a) Son (b) Granddaughter (c) Grandson (d) Wife (e) None of these Q4. How many female members are there in the family? (a) Three (b) Four (c) Five (d) Can‘t be determined (e) None of these Q5. Which of the following pairs is a pair of husband and wife? (a) M, T (b) N, S (c) R, U (d) P, T

Governmentadda.com | IBPS SSC SBI RBI RRB FCI LIC RAILWAYS

13

Daily Visit :

[GOVERNMENTADDA.COM] Q7. Statements: P ≤ K > R ≥ N = S < M Conclusions: I. K > S II. M > P

(e) None of these Solutions (1-5):

(8-9): Statements: L ≥ G > H, C = Q ≥ K, L = C Q8. Conclusions: I. C > H II. Q ≥ G Q9. Conclusions: I. K ≤ H II. G > K Q10. Statements: A ≤ Z = W < D = E > X Conclusions: I. Z > X II. E > A Solutions (6-10): S6. Ans.(d) Sol. Statement= M < P≤G ≥D=L Conclusions: I. P ≥ D ( Not True) II. L < M( Not True) S7. Ans.(a) Sol. Statement= P ≤ K > R ≥ N = S < M Conclusions: I. K > S ( True) II. M > P( Not True)

S1. Ans.(c) Sol. S2. Ans.(a) Sol.

S8. Ans.(e) Sol. Statement= K ≤ Q=C=L≥G>H Conclusions: I. C > H( True) II. Q ≥ G( True)

S3. Ans.(b) Sol. S4. Ans.(b) Sol.

S9. Ans.(d) Sol. Statement= K ≤ Q=C=L≥G>H Conclusions: I. K ≤ H ( Not True) II. G > K( Not True)

S5. Ans.(c) Sol. Directions (6-10): In these questions, a relationship between different elements is shown in the statements. The statements are followed by two conclusions. Give answer. (a) if only conclusion I is true. (b) if only conclusion II is true. (c) if either conclusion I or II is true. (d) if neither conclusion I nor II is true. (e) if both conclusion I and II are true. Q6. Statements: G ≥ D = L, M < P ≤ G Conclusions: I. P ≥ D II. L < M

S10. Ans.(b) Sol. Statement= A ≤ Z = W < D = E > X Conclusions: I. Z > X ( Not True) II. E > A( True) Directions (11-15): Study the following information carefully and answer the questions given below: P, Q, R, S, T and U are going to six different cities, viz Kolkata, Mumbai, Ranchi, Delhi, Jaipur and Pune, but not necessarily in the same order. All are going by three different modes of transport, viz Car, Train and

Governmentadda.com | IBPS SSC SBI RBI RRB FCI LIC RAILWAYS

14

Daily Visit :

[GOVERNMENTADDA.COM]

Aeroplane. At least two persons travel by each mode of transport. T goes to Jaipur by Car and P goes to Ranchi. Aeroplane is being used by the persons who are going to Mumbai and Pune. The one who is travelling by Train is not going to Delhi. Q is travelling by train and R is travelling by Car. P is travelling by train. Q11. Who among the following goes to Delhi? (a) S (b) R (c) U (d) P (e) None of these Q12. Which of the following pairs travels by aeroplane? (a) S, R (b) P, T (c) U, R (d) S, U (e) None of these Q13. Which of the following combinations is true for Q? (a) Jaipur–Car (b) Delhi–Car (c) Kolkata–Train (d) Pune–Aeroplane (e) None of these Q14. U travels by which of the following modes? (a) Aeroplane (b) Car (c) Train (d) Can‘t be determined (e) None of these Q15. Which of the following statements is definitely true? (a) U goes to Mumbai (b) R travels by train (c) S goes to Pune (d) All are true (e) None of these Solutions (11-15):

S11. Ans.(b) Sol. S12. Ans.(d) Sol. S13. Ans.(c) Sol. S14. Ans.(a) Sol. S15. Ans.(e) Sol.

Direction (1-5): Study the following information carefully and answer the given questions: Eight persons Monu, Sonu, Pinku, Mintu, Bantu, Chiru, Kalu and Sintu are sitting around a circular table but not necessarily in the same order. Three of them are facing opposite to center while other five are facing towards the centre of the table. Bantu sits third to the right of Pinku. Chiru is sitting third to the left of Bantu. Three persons are sitting between Chiru and Sonu. Kalu is sitting third to the right of Chiru who is not facing the centre. Mintu is sitting third to the right of Monu, who is not facing the centre. Q1. Who sits between Chiru and Monu ? (a) Sonu (b) Pinku (c) Bantu (d) Kalu (e) None of these Q2. Who among the following is second to the right of Sonu?

Governmentadda.com | IBPS SSC SBI RBI RRB FCI LIC RAILWAYS

15

Daily Visit :

[GOVERNMENTADDA.COM]

(a) Mintu (b) Pinku (c) Bantu (d) Chiru (e) None of these

S4. Ans.(a) S5. Ans.(c)

Q3. Which of the following statements is/are true with respect to Sintu? (a) Sintu is opposite of Kalu. (b) Sintu is fourth to the right of Kalu. (c) Sintu is fourth to the left of Kalu. (d) All are true (e) None of these Q4. Which of the following groups represents the group, of those facing away from the centre ? (a) Monu, Pinku, Chiru (b) Pinku, Chiru , Mintu (c) Sonu, Monu, Pinku (d) Mintu, Bantu, Chiru (e) None of these Q5. What is the position of Pinku with respect of Sintu ? (a) Pinku is third to the right of Sintu. (b) Pinku is second to the left of Sintu. (c) Pinku is third to the left of Sintu. (d) Pinku is in front of Sintu. (e) None of these

Directions (6-10): Study the following information to answer the given question: In a certain code language, 'always create new ideas' is written as "ba ri sha gi'. 'ideas and new thoughts' is written as 'fa gi ma ri'. 'Create thoughts and Insights' is written as 'ma jo ba fa' 'new and better solutions' is written as 'ki ri to fa'. Q6. What is the code for 'ideas'? (a) sha (b) ba (c) gi (d) ma (e) Cannot be determined Q7. What does 'fa' stand for? (a) thoughts (b) insights (c) new (d) and (e) solutions Q8. 'fa lo ba' could be a code for which of the following? (a)thoughts and action (b) create and innovate (c) ideas and thoughts (d) create new solutions (e) always better ideas Q9. What is the code for 'new'? (a) ki (b) ri (c) to (d) fa (e) ba

S1. Ans.(e) S2. Ans.(b)

Q10.Which of the following may represent 'insights thoughts always'? (a) jo ki to (b) ki to ri (c) sha jo ri (d) ma sha jo

S3. Ans.(d) Governmentadda.com | IBPS SSC SBI RBI RRB FCI LIC RAILWAYS

16

Daily Visit :

[GOVERNMENTADDA.COM]

(e) sha to ba

Q12.Statement: Militants are now attacking the police in several states of India in a bid to demoralise them and to choke the intelligence network. Courses of action: I. Government should formulate a plan to equip its policemen with sophisticated weapons and equipment besides making an effort to end militancy in a peaceful way. II. The police department should send strict instructions to all the police officers concerned that every policeman on duty should have proper gear like a bullet- proof vest and automatic weapons to counter any sudden attack.

Solution(6-10): Always----------------------- sha Create------------------------- ba New--------------------------- ri Ideas-------------------------- gi And--------------------------- fa Thought---------------------- ma Insights ---------------------- jo Better/solution--------------- ki/to S6.Ans.(c)

S12. Ans.(e) Sol. Both I and II will reduce the problem. Good equipment will keep the policemen alert whereas good weapons will make them strong to face the situation. Meaningful dialogue can change the situation dramatically. Therefore, both I and II are advisable.

S7.Ans.(d) S8.Ans.(b) S9.Ans.(b) S10.Ans.(d) Directions (11-15): In each question below is given a statement followed by two courses of action numbered I and II. A course of action is a step or administrative decision to be taken for improvement, follow-up or further action in regard to the problem, policy etc. on the basis of the information given in the statement. You have to assume everything in the statement to be true. Then decide which of the two given suggested courses of action logically follows for pursuing. Give answer (a) If only I follows. (b) If only II follows. (c) If either I or II follows. (d) If neither I nor II follows. (e) If both I and II follow. Q11.Statement: According to a report, energyguzzling technologies that we use at home, destroy the environment. Courses of action: I. An effort should be made to discourage use of energyguzzling technologies. II. A special tax should be imposed on the use of such technologies. S11. Ans.(a) Sol. I is advisable because it will reduce the problem. II is impractical. Hence, II is not advisable.

Q13.Statement: With more airlines crossing the Indian skies and air travelling become more affordable, the infrastructure facilities at airports have remained grossly inadequate. Courses of action: I. All airports in India should be privatised. II.A committee should be made to look into the matter and their recommendation should be sought to deal with the situation. S13. Ans.(b) Sol. I is not advisable. We need inspection to sort out the problem. II is advisable because it will be helpful to make strategies to sort out the problem. Q14.Statement: A number of travel agents in cities continue to harass and fleece the hapless travellers and cause damage to tourism industry. Courses of action: I. An effort should be made to punish all such travel agents and an instruction should be given to all travel agents to provide good services and treatment to travellers. II. An effort should be made to implement the rules and regulations effectively. S14. Ans.(e) Sol. Participation of travel agents is essential to make the situation conducive for tourists. The instruction given to them can make the situation comfortable for travelers. Hence, I is advisable. II is advisable because if rules and

Governmentadda.com | IBPS SSC SBI RBI RRB FCI LIC RAILWAYS

17

Daily Visit :

[GOVERNMENTADDA.COM]

regulations are followed properly, the number of incidents of harassing the tourists will reduce. Q15.Statement: Large areas in several towns of the state XYZ lay under waist-deep water as heavy rain lashed coastal areas of the XYZ state for the fourth day. Courses of action: I. The government should review the situation and the authorities should be asked to undertake enumeration of losses immediately to enable disbursement of relief. II. Efforts should be made to ferry people to safer places. S15. Ans.(e) Sol. Since the situation is alarming, rescue operation is indispensable. II will help in rescuing people. Hence, II is advisable. I is advisable to help people to deal with the situation.

Directions (1-5): Study the following information answer the questions that follow.

Q2. Which of the following institutes provides coaching for Banking? (a) P (b) T (c) U (d) T or U (e) None of these Q3. Which of the following combinations is correct? (a) Wednesday — Q — SSC (b) Wednesday — Q — Banking (c) Friday — U — CAT (d) Thursday — S— Medical (e) None of these Q4. Which institute provides coaching for Medical? (a) P (b) U (c) T (d) S (e) None of these

Seven institutes P, Q, R, S, T, U and V provide coaching for seven different competitive exams, viz. Engineering, NET, CAT, SSC, Banking, Medical and TET, but not necessarily in the same order. There is one day weekly, off in each institute from Monday to Sunday but not necessarily in the same order. No two institutes have the same weekly off day. Institute R provides coaching for NET and is neither closed on Friday nor on Wednesday. Institute S provides coaching for Engineering, and Thursday is its weekly off day. Institute T and U do not provide coaching for Banking and neither of these has Wednesday as weekly off day Institute Q provides coaching for SSC and remains closed on Sunday. The one which provides coaching for Medical has Tuesday as weekly off day. Institute V provides coaching for TET and remains closed on Monday. Institute T does not provide coaching for CAT.

Q5. On which of the following days does Institute U remain closed? (a) Tuesday (b) Wednesday (c) Sunday (d) Can‘t be determined (e) None of these

Q1. Institute R remains closed on which of the following days? (a) Sunday (b) Monday (c) Tuesday (d) Saturday (e) None of these

S1. Ans.(d) Sol.

Solutions (1-5):

S2. Ans.(a) Sol. S3. Ans.(c)

Governmentadda.com | IBPS SSC SBI RBI RRB FCI LIC RAILWAYS

18

Daily Visit :

[GOVERNMENTADDA.COM] Sol. Conclusion: I. Q ≥ N ( True) II. R > E ( Not True)

Sol. S4. Ans.(c) Sol.

S7. Ans.(d) Sol. Conclusions: I. G > P (Not True) II. S < R (Not True)

S5. Ans.(e) Sol. Directions (6-10): In these questions, a relationship between different elements is shown in the statements. The statements are following by two conclusions. Give answer (a) if only conclusion I is true (b) if only conclusion II is true (c) If either conclusion I or II is true. (d) if neither conclusion I nor II is true. (e) if both conclusions I and II are true. Question (6-7) Statement: Q ≥ P = R ≥ N < G ≥ E > S Q6. Conclusion: I. Q ≥ N II. R > E Q7. Conclusions: I. G > P II. S < R Q8. Statement: B=Y≥D Y II. D < N Question (9-10): Statement: JU=W U II. W < V Q10. Conclusions: I. J < W II. T > U Solutions (6-10): Solution (6-7):

S8. Ans.(b) Sol. Conclusions: I. M > Y (Not True) II. D < N (True) Solution (9-10): S9. Ans.(e) Sol. Conclusions: I. P > U (True) II. W < V (True) S10. Ans.(b) Sol. Conclusions: I. J < W (Not True) II. T > U (True) Directions (11-15): Study the following arrangement carefully and answer the given questions: K28%P#B3H$GTIAYE£⋆49LU@7C U Q11. If all the symbols are dropped from the above arrangement, which of the following will be tenth from the right end? (a) A (b) Y (c) 9 (d) G (e) None of these Q12. How many such consonants are there in the above arrangement each of which is not immediately preceded by a symbol? (a) None (b) One (c) Two (d) Three (e) More than three

S6. Ans.(a) Governmentadda.com | IBPS SSC SBI RBI RRB FCI LIC RAILWAYS

19

Daily Visit :

[GOVERNMENTADDA.COM]

Q13. How many such letters are there in the above arrangement each of which is immediately followed by a number? (a) One (b) Two (c) three (d) None (e) None of these Q14. How many such symbols are there in the above arrangement each of which is not immediately preceded by a consonant but followed by a vowel? (a) Two (b) Three (c) Four (d) None (e) None of these Q15. Which of the following element is 4th to left of 13th from left in the above given series? (a) N (b) ⋆ (c) L (d) K (e) H Solutions (11-15): S11. Ans.(a) Sol.

Renault, Jaguar, Honda City but not necessarily in the same order and the colour of all car also different viz. Red, Pink, Blue, White, Yellow, Orange and Violet but not necessarily in the same order. Only one person lives between the one who have Renault and the one who have Jaguar car. S does not have Renault. V does not have Ford. The one who have Orange colour car live immediate above the one who have Renault. T lives on one of the floors below Q, but does not live on the lowermost floor. The one who lives on 7th floor is immediate above the one, who lives immediate below the one who have Violet colour car. There are three persons between Q and T. The one who have Toyota car lives on the odd-numbered floors below U. P lives immediately above U and does not have Mercedes car. U does not have White and Pink colour car. The one who have Blue colour car lives, immediate above the one, who have Yellow colour car. Jaguar car is of Pink colour. Only one person lives between Q and the one who have Mercedes car. The one who have Yellow colour car lives below the one who have Red colour car. Only two persons live between V and the one who have Mercedes car. The one who have Audi car live on evennumbered floors and above the one who have Mercedes car. The one who have Honda City car lives immediately above V who does not have Jaguar car. S lives on one of the floors above R. V does not live on first and top floor. Q1. Who among following have White colour car? (a)R (b) The one who have Audi (c) The one who have Mercedes (d) The one who have Ford (e) Q

S12. Ans.(e) Sol. 3H, GT, AY, 9L, 7C S13. Ans.(b) Sol. K2, B3 S14. Ans.(d) Sol. None. S15. Ans.(e) Sol. 4th to left of 13th of left= 13-4=9th from left =H

Directions (1-5): Study the following information carefully and answer the question given below: Seven persons P, Q, R, S, T, U and V live on seven different floors of a building but not necessarily in the same order. The lowermost floor of the building is numbered one; the one above it is number two and so on till the topmost floor is number seven. Each of them have different car viz. Ford, Audi, Mercedes, Toyota,

Q2.Who lives on immediate above the one who have Pink colour car? (a)The one who have Mercedes (b) Both (a) and (e) (c) S (d) Q (e) The one who have Red colour car Q3.Who among following have Honda City car? (a)Q (b) P (c) V (d) T (e) None of these

Governmentadda.com | IBPS SSC SBI RBI RRB FCI LIC RAILWAYS

20

Daily Visit :

[GOVERNMENTADDA.COM]

Q4.How money persons live between the one who have Toyota and the one who have Pink colour car? (a) None (b) Three (c) Two (d) One (e) None of these Q5.Four of the following five are alike in a certain way and hence they form a group. Which one of the following does not belong to that group? (a) The one who have Toyota car (b) The one who have Orange colour car (c) The one who lives on 5th floor (d) The one who lives on 6th floor (e) The one who have Ford Solution(1 –5):

(a) if the data in statement I alone are sufficient to answer the question, while the data in statement II alone are not sufficient to answer the question. (b) if the data in statement II alone are sufficient to answer the question, while the data in statement I alone are not sufficient to answer the question. (c) if the data either in statement I alone or in statement II alone are sufficient to answer the question. (d) if the data in both statement I and II together are not sufficient to answer the question. (e) if the data in both statement I and II together are necessary to answer the question. Q6. Is Z brother in law of D? I.M and Z are brothers of B and E is wife of Z . II.D is mother of U and is wife of B‘s brother . Q7. Who has secured the maximum marks among six friends A, B, C, D, E and F? I. B secured less marks than A and F but not less than C, D and E. II. F secured more marks than B but not as much as A. Q8. Sanchit‟s flat is on which Floor of the five-floor apartment? I. His flat is exactly above Ganesh‘s flat whose flat is exactly above Nitin‘s first-floor flat. II. Jeevan‘s flat, which is adjacent to Sanchit‘s flat, who is exactly below Ahmed‘s flat, who is on fourth floor.

S1.Ans(b) Sol.

Q9. What is the code of „there‟ in the code language? I. In the code language ‗there is she‘ is written as ‗ka ma te‘ and ‗she is good‘ is written as ‗te ka ro‘. II. In the code language ‗there are you going‘ is written as ‗la ma pa je‘ and ‗there is the girl‘ is written as ‗cha fa ma te‘.

S2.Ans.(b) Sol. S3.Ans.(d) Sol.

Q10. In a row of boys facing South who is immediate left to Rinku? I. Suresh is immediate right to Chandrakant, who is fourth to the right of Rinku. II. Suresh is third to the right to Rinku, and Naresh is second to the right of Suresh.

S4.Ans.(c) Sol. S5.Ans.(d) Sol. Directions (6-10): Each of the questions below consists of a question and two statements numbered I and II given below it. You have to decide whether the data provided in the statements are sufficient to answer the question. Read both the statements and give answer

Solution(6-10): S6. Ans.(e) From 1-

Governmentadda.com | IBPS SSC SBI RBI RRB FCI LIC RAILWAYS

21

Daily Visit :

[GOVERNMENTADDA.COM] As per the rules followed in the above steps, find out in each of the following questions the appropriate step for the given input. Input: 17 as 32 of 12 now 29 to 7 push 4 for

From 2-

Q11. In which step the elements „32 to as‟ found in the same order? (a) Step I (b) Step II (c) Step III (d) Step V (e)Step VI

From combining 1 and 2We can say that Z is brother in law of D. S7. Ans.(e) Sol. From both statement I and II- A > F > B > C/D/E A has secured maximum mark. S8. Ans.(c) Sol. We can get the answer either from the statement I or II- Sanchit‘s flat is on third floor. S9. Ans.(c) Sol. We can get the answer either from the statement I or II- there=ma S10. Ans.(d) Sol. If the data in both statement I and II together are not sufficient to answer the question. Directions (11-15): Study the given information and answer the questions: A word and number arrangement machine when given an input line of words and numbers rearranges them following a particular rule in each step. The following is an illustration of an input and rearrangement. Input: 19 eyes 24 are 18 now 25 on 32 meet 9 between Step I: 10 19 eyes 24 18 now 25 on 32 meet between are Step II: between 10 eyes 24 18 now 25 on 32 meet are 20 Step III: 26 between 10 24 18 now on 32 meet are 20 eyes Step IV: meet 26 between 10 24 now on 32 are 20 eyes 17 Step V: 23 meet 26 between 10 on 32 are 20 eyes 17 now Step VI: on 23 meet 26 between 10 are 20 eyes 17 now 31 And Step VI is the last step of the rearrangement of the above input.

Q12. In step V, which of the following element would be at 3rd to the left of 5th from the right end? (a) 8 (b) to (c) push (d) 4 (e) as Q13. Which of the following is last but one step? (a) III (b) IV (c) VI (d) VII (e) V Q14. Which of the following would be the step III after arrangement? (a) for 30 8 32 of 12 to push 4 as 18 now (b) 30 for 8 32 of 12 to push 4 as now18 (c) 30 for 8 32 of to12 push 4 as 18 now (d) 30 for 8 32 of 12 to push 4 as 18 now (e) None of these Q15. In step VI, „to‟ is related to „of‟ and „now‟ is related to „push‟. In the same way „for‟ is related to? (a) to (b) 11 (c) of (d) as (e) None of these Solution (11-15): Students let us understand the Logic behind this Question and let‘s understand how to solve it. When we see the each step, then we can find that there is both number and words are arranged in each step. 1) For words arrangement- Words are arranged according to ascending order given in English

Governmentadda.com | IBPS SSC SBI RBI RRB FCI LIC RAILWAYS

22

Daily Visit :

[GOVERNMENTADDA.COM]

dictionary. In first step the word which come first according to English dictionary arranged first to extreme right. And in second step next word is arranged to extreme left. And this process is continued in further step. 2) For number arrangement- Numbers are arranged according to ascending order. There are three odd numbers and three even numbers. In first three step odd numbers are arranged in ascending order first to extreme left end and in second step to extreme right end and this process is continued in further steps. And in last three step even number are arranged in ascending order. (Each odd number is added by 1 while they are arranged and each even number is subtracted by 1 while they are arranged). Input: 17 as 32 of 12 now 29 to 7 push 4 for Step I: 8 17 32 of 12 now 29 to push 4 for as Step II: for 8 32 of 12 now 29 to push 4 as 18 Step III: 30 for 8 32 of 12 to push 4 as 18 now Step IV: of 30 for 8 32 12 to push as 18 now 3 Step V: 11 of 30 for 8 32 to as 18 now 3 push Step VI: to 11 of 30 for 8 as 18 now 3 push 31 S11. Ans. (d) Sol. S12. Ans. (a) Sol.

Italian, French and Korean, but not necessarily in the same order. Only two persons sit between the one who speaks Chinese who sits at an extreme end and E. P who sits in the middle of the row is not an immediate neighbour of Q who doesn‘t speak Korean. T sits at an extreme end. E speaks Russian and also sits on the immediate right of the person who speaks Spanish . E faces the immediate neighhour of Q. B is not sitting at the extreme left end. P does not speak French. Only one person sits between C and D who speaks Chinese. S speaks Japanese. S is an immediate neighbour of the one who speaks Korean. S does not face the person who speaks English. R who speaks Arabic is an immediate neighbour of the person who speaks French. The person who speaks French faces the immediate neighbour of the one who speaks Russian. There are two persons between the one who speaks Hausa and the one who speaks English. B does not speak Hausa. T does not speak Italian. D does not sit at extreme right end. Q1.A speaks which of the following language? (a) Italian (b) Hausa (c) English (d) None of these (e) Can't be determined Q2.Who speaks Italian? (a) R (b) B (c) T (d) P (e) Can't be determined

S13. Ans.(e) Sol. S14. Ans.(d) Sol. S15. Ans. (d) Sol.

Directions (1-5): Read the following information carefully and answer the questions that follow. Ten persons are sitting in two parallel rows containing five persons each. In row 1, A, B, C, D and E are sitting and all of them are facing south. In row 2, P, Q, R, S and T are sitting and all of them are facing north. In the given seating arrangement, each member seated in a row faces another member of the other row. Moreover, each of them is speaking different language i.e. Chinese, Spanish, English, Arabic, Russian, Japanese, Hausa,

Q3.„Chinese‟ is related to ' Korean' in a certain way, based on their sitting arrangement. Then “Hausa” is related to which, following the same seating positions? (a) Italian (b) French (c) Spanish (d) Japanese (e) Arabic Q4.Four of the following five are alike in a certain way based on their seating positions and so form a group. Which of the following is different from the group? (a) Russian (b) Hausa

Governmentadda.com | IBPS SSC SBI RBI RRB FCI LIC RAILWAYS

23

Daily Visit :

[GOVERNMENTADDA.COM]

(c) Korean (d) Arabic (e) Japanese

"has asked him stay " is coded as"SR6 KC10 ML6 AX8"

Q5.Which of the following statements is/are definitely false? (a) B speaks English. (b) There are two persons sitting between the one who speaks Japanese and the one who speaks French. (c) The person who speaks Spanish faces the person who speaks Italian. (d) The person who speaks French sits opposite the person who speaks Chinese. (e) All are true Solution(1-5):

Q9.What may be the possible code for „political‟ in the given code language? (a) KK18 (b) LK18 (c) LL18 (d) LK19 (e)None of these

S2.(d) Sol. S3.(d) Sol. S4.(c) Sol. S5.(d) Sol. Directions (6-10): Study the information and answer the following questions: In a certain code language

"parties have decided their" is coded as "RR14 VD8 CC14 EQ10 "

Q7.What is the code for „significant' in the given code language? (a) GS22 (b) GH22 (c) GS21 (d) HS22 (e)None of these Q8.What may be the possible code for „especially‟ in the given code language? (a) QX20 (b) PX20 (c) PX21 (d) EX20 (e)None of these

S1.(b) Sol.

"form panel forge consensus" is coded as "RL8 NK10 RD10 NR18 "

Q6.What is the code for „candidate‟ in the given code language? (a) ND18 (b) MD18 (c) ND19 (d) NE18 (e)None of these

Q10.What is the code for „factions' in the given code language? (a) DR16 (b) RR16 (c) CR16 (d)CR15 (e)None of these Solutions (6-10): These are the latest pattern of coding-decoding questions. In these questions we are applying following concept:-

Governmentadda.com | IBPS SSC SBI RBI RRB FCI LIC RAILWAYS

24

Daily Visit :

[GOVERNMENTADDA.COM] (d) Only (A) and (B) (e) None of these Q12. Statement: A large private bank has decided to retrench one-third of its employees in view of the losses incurred by it during the past three quarters. Courses of action A. The government should issue a notification to general public to immediately stop all transactions with the bank. B. The government should direct the bank to refrain from retrenching its employees. C. The government should ask the central bank of the country to initiate an enquiry into the bank‘s activities and submit its report. (a) None (b) Only (A) (c) Only (B) (d) Only (C) (e) A and (C)

S6. Ans.(a) Sol. S7. Ans.(a) Sol. S8. Ans.(b) Sol. S9. Ans.(b) Sol. S10. Ans.(c) Sol. Directions (11-12): In each questions below is given a statement followed by three courses of action numbered (A), (B) and (C). A course of action is a step or administrative decision to be taken for improvement, follow up or further action in regard to the problem, policy etc. On the basis of the information given in the statement, you have to assume everything in the statement to be true, then decide which of the suggested course of action logically follow(s) for pursing.

Solution(11-12):

Q11. Statement: The school dropout rate in many districts in the state has increased during the last few years as the parents of these children make them work in the fields owned by others to earn enough for them to get at least one meal a day. Courses of action A. The government should put up a mechanism to provide foodgrains to the poor people in these districts through public distribution system to encourage the parents to send their children to school. B. The government should close down some of these schools in the district and deploy the teachers of these schools to nearby schools and also ask remaining students to join these schools. C. The government should issue arrest warrants for all the parents who force their children to work in fields instead of attending classes. (a) Only (A) (b) Only (B) (c) Only (C)

Directions (13-15): In making decisions about important questions, it is desirable to be able to distinguish between “Strong” arguments and “Weak” arguments as they relate to the question. “Strong” arguments are those which are both important and directly related to the questions. “Weak” arguments are those which are of minor importance and also may not be directly related to the questions or may not be directly related to a trivial aspect of the questions. Each question below is followed by two arguments numbered I and II. You have to decide which of the arguments is a strong arguments and which is a weak agreement. Give answer (a) If only argument I is strong (b) If only argument II is strong (c) If either I or II is strong (d) If neither I nor II is strong (e) If both I and II are strong

S11. Ans.(a) Sol. Course of Action A logically follows. S12. Ans.(d) Sol. Course of action C logically follows, because government controls banking system with the help of Central Bank.

Governmentadda.com | IBPS SSC SBI RBI RRB FCI LIC RAILWAYS

25

Daily Visit :

[GOVERNMENTADDA.COM]

Q13. Statement Should there be only one rate of interest for term deposits of varying durations in banks? Arguments I. No, people will refrain from keeping money for longer duration resulting into reduction of liquidity level of banks. II. Yes, this will be much simple for the common people and they may be encouraged to keep more money in banks. Q14. Statements Should labour reforms be immediately introduced in India? Arguments I. Yes, this will help increase the productivity in all the sectors in general and in the public sector in particular. II. No, many other countries have not implemented this so far. Q15. Statements Should the system of paying minimum purchase price for farmers for wheat, rice and the like be scrapped in India? Arguments I. No, farmers who produce our staple food must get decent return on their investment and labour in a welfare state like India. II. Yes, it is an outdated practice which we must discard. Solution(13-15): S13. Ans.(a) Sol. Higher rate of interest attracts people to deposit money for longer duration. If higher rate of interest is available for short duration, then people will refrain from keeping money for longer duration. So, arguments I is strong while II is not. S14. Ans.(a) Sol. Argument I is strong because labour reforms will encourage the labour to work more efficiently, which will be helpful in incrasing the productivity. Argument II is not strong because basic needs of India are different from other countries. S15. Ans.(a) Sol. Argument I is strong because of the welfare state theory. Argument II is not strong because it does not give a proper reason.

Directions (1-5): Study the following information carefully and answer the question given below:

K, L, M, P, Q, R, S and T are sitting around a square table in such a way that four of them sit at four corner of the square while four sit in the middle of each of the four sides. The ones who sit at the four corners face outside while those who sit in the middle of the sides face the centre of the table. P sits third to the right of S. S faces the centre. Q sits third to the left of M. M does not sit in the middle of the sides. Only one person sits between Q and R. R is not an immediate neighbour of M. T faces the centre. K is not an immediate neighbour of R. Q1. What is position of M with respect to L? (a) Third to the right (b) M and L sit diagonally opposite to each other (c) Second to the right (d) Second to the left (e) Fifth to the right Q2. Who sits exactly between Q and R, when counted from right of Q? (a) T (b) P (c) K (d) M (e) S and K Q3. Which of the following pairs represents the persons seated in the middle of the sides and face each other? (a) S, Q (b) K, L (c) M, P (d) R, T (e) T, Q Q4. If K is made to face the opposite direction, who would sit to his immediate right? (a) R (b) Q (c) P (d) T (e) S Q5. Four of the following five are alike in a certain way and so form a group. Which is the one that does not belong to that group? (a) L (b) M (c) K (d) P

Governmentadda.com | IBPS SSC SBI RBI RRB FCI LIC RAILWAYS

26

Daily Visit :

[GOVERNMENTADDA.COM]

(e) R

(b) One (c) Two (d) Three (e) More than three

Solution(1-5):

Q8. How many such 5s there in the above arrangement each of which is immediately preceded and followed by an odd digit? (a) None (b) One (c) Two (d) Three (e) More than three Q9. Which of the following is third to the left of the eighteenth digit from the left end of the above arrangement? (a) 8 (b) 3 (c) 4 (d) 5 (e) 1

S1. Ans.(d) Sol. S2. Ans.(b) Sol. S3. Ans.(e) Sol. S4. Ans.(b) Sol. S5. Ans.(e) Sol. Direction (6-10): Study the following arrangement carefully and answer the questions given below.

Q10. If all the even digits are deleted from the above arrangement, which of the following will be ninth from the right end of the arrangement? (a) 9 (b) 3 (c) 1 (d) 5 (e) 7 Solution(6-10):

761792415649234125858483127526 72953

S6. Ans.(e) Sol. 25, 27, 26, 29

Q6. How many 2s are there in the above arrangement, each of which is immediately followed by a digit which has a numerical value of more than four? (a) None (b) one (c) Two (d) Three (e) More than three

S7. Ans. (c ) Sol. 41, 41

Q7. How many such 1s are there in the above arrangement each of which is immediately preceded by a perfect square? (a) None

S10. Ans.(d) Sol. 5

S8. Ans.(b) Sol. 953 S9. Ans.(c ) Sol. third to the left of the eighteenth digit from the left end= 18-3= 15th from left=4

Governmentadda.com | IBPS SSC SBI RBI RRB FCI LIC RAILWAYS

27

Daily Visit :

[GOVERNMENTADDA.COM]

Directions (11-15): In each question below, are given a statement followed by two conclusion numbered I and II. On the basis of the information given in the statement, you have to assume everything in the statement to be true, and then decide which of the following conclusion follow(s) Give answer: (a) if only I follows. (b) if only II follows. (c) if either I or II follows. (d) if neither I nor II follows. (e) if both I and II follow. Q11. Statement : T < P ≤ U; L > U ≥K; P ≤R Conclusions: I. K > R II. L > R Q12. Statement: H = I ≤R; M ≥R < S Conclusions : I. M = I II. M > I Q13. Statement : D > H > N; S > I ≤H Conclusions : I. N ≤S II. N < D Q14. Statement : P ≤ O < I; P > Y > W Conclusions : I. Y ≤I II. O > W Q15. Statement: A ≤B > C ≤ F; Z < C ≤ D < E Conclusions: I. A > Z II. F < E Solution(11-15): S11. Ans.(d) Sol.Conclusions: I. K > R ( Not True ) II. L > R ( Not True ) S12. Ans.(c) Sol.Conclusions : I. M = I ( Not True ) II. M > I ( Not True ) S13. Ans.(b) Sol.Conclusions : I. N≤S ( Not True ) II. N < D ( True ) S14. Ans.(b) Sol.Conclusions : Y≤ I ( Not True ) II. O > W ( True )

S15. Ans.(d) Sol.Conclusions: I. A > Z ( Not True ) II. F < E ( Not True )

Directions (1-5):Study the following information and answer the given questions: Nine people – P, Q, R, S, T, U, V, W, and X live on separate floors of a 9-floor building. Ground floor is numbered 1, first floor is numbered 2 and so on until the topmost floor is numbered 9. Each one of them is suffering from different diseases viz. Asthma, Cancer, Diabetes, Lupus, Migraine, Narcolepsy, Ulcers, Stroke, and Obesity but not necessarily in the same order. Only five people live above the floors on which P lives. Only one person lives between P and the one who is suffering from Narcolepsy. U lives immediately below the one who is suffering from Cancer. The one who is suffering from Cancer lives on an even numbered floor. Only three people live between the one who is suffering from Narcolepsy and the one who is suffering from Diabetes. T lives immediately above R. T is not suffering from Diabetes. Only two people live between Q and the one who is suffering from Migraine. The one who is suffering from Migraine lives below the floor on which Q lives. W is suffering from Stroke and lives on floor no. 8. The one who is suffering from Asthma does not live immediately above or immediately below Q. S does not live immediately above or immediately below P. X is suffering from Obesity and lives immediately above W. V does not suffer from Lupus. Q1. Which of the following is true with respect to V as per the given information? (a)The one, who lives immediately below V, is suffering from Cancer. (b)V lives on floor no. 7. (c)V lives immediately below T. (d)V lives on the lowermost floor. (e)V is suffering from Narcolepsy. Q2.Who amongst the following lives on floor no. 3 ? (a)The one who is suffering from Lupus (b)The one who is suffering from Migraine (c)R (d) V (e) T Q3.Who lives on the floor immediately above T ? (a) P (b) Q

Governmentadda.com | IBPS SSC SBI RBI RRB FCI LIC RAILWAYS

28

Daily Visit :

[GOVERNMENTADDA.COM]

(c) S (d) V (e) U

"second day month long" is coded as ―15M 15G 1Z 5M ‖ "resign after her speech" is coded as ―16X 5V 6V 5T ‖

Q4.S is suffering from which disease? (a)Cancer (b) Narcolepsy (c) Diabetes (d) Migraine (e) Lupus

"gave berth after the" is coded as ―8S 6V 5G 1E ‖

Q5.How many people live between the floors on which S and the one who is suffering from Obesity live ? (a) None (b) Two (c) One (d) More than three (e) Three Solution(1-5):

Q6. What is the code for „road'? (a) 15Z (b) 10Z (c) 15X (d) 25Z (e) None of these. Q7. What is the code for „train'? (a) 18R (b) 18S (c) 8R (d) 28R (e) None of these. Q8. What is the code for „temple'? (a) 5L (b)15O (c) 5P (d) 5O (e) None of these. Q9. What is the code for „minister'? (a) 19V (b) 9C (c) 10V (d) 9V (e) None of these.

S1. Ans.(a) Sol. S2. Ans.(b) Sol.

Q10. What is the code for „travel'? (a) 18V (b) 28V (c) 18W (d) 18X (e) None of these.

S3. Ans.(e) Sol. S4. Ans.(e) Sol. S5. Ans.(c) Sol. Directions (6-10): Study the information and answer the following questions: In a certain code language

Solution (6-10): These are the latest pattern of coding-decoding questions. In these questions we are applying following concept:-

Governmentadda.com | IBPS SSC SBI RBI RRB FCI LIC RAILWAYS

29

Daily Visit :

[GOVERNMENTADDA.COM] I. Very large number of devotees may visit the temple on the main festival day. II. People travelling to the areas near the temple may postpone their journey by a day unless they have very urgent work in that areas. Q13. Statement: The government has instructed all the private school in the city to maintain the current fees for at least two more years. Assumptions I. The authorities of private schools may not follow the government instructions as they are not dependent on government funds. II. The parents of the students of private school of the city may still be eager to pay higher fees.

S6. Ans. (a) Sol. S7. Ans. (a) Sol. S8. Ans. (d) Sol.

Solution(11-13): S11. Ans.(b) Sol. In such cases of illness, the patient is rushed to hospital, irrespective of how the relatives of patients may behave. So, assumption I is not implicit. Assumption II is implicit because affected people were rushed to the nearby hospitals, that means nearby hospitals may be able to attend all the affected people.

S9. Ans. (d) Sol. S10. Ans. (a) Sol. Directions (11-13): In each question below is given a statement followed by two assumptions numbered I and II. An assumption is something supposed or taken for granted. You have to consider the statement and the following assumptions and decide which of the assumptions is/are implicit in the statement. Give answer (a) if only Assumption I is implicit (b) if only Assumption II is implicit (c) if either Assumption I nor II is implicit (d) if neither Assumption I nor II is implicit (e) if both Assumption I and II are implicit Q11. Statements: Many people fell ill after consuming meal at a wedding reception and were rushed to the nearby government and private hospitals. Assumption I. The relatives of the affected people may refuse to take them all to the government hospitals. II. The nearly hospitals may be able to attend all the affected people. Q12. Statements: The municipal authority blocked movement of traffic in and around the temple on the main festival day. Assumptions:

S12. Ans.( e) Sol. It is expected that a large number of devotees may visit the temple on main festival day. So municipal authority is blocking the movement of traffic in that area. So, assumption I is implicit. People travelling to that areas may postpone their journey because of blocked movement. So assumption II is also implicit. S13. Ans.(d) Sol.I is not a correct assumption, as while instructing the schools the government must have assumed that instructions will be followed and parents never want to pay more fees. So, neither assumption I nor II is implicit. Directions (14-15): In each question below is given a statement followed by two courses of action numbered I and II. On the basis of the information given in the statement, you have to assume everything in the statement to be true, then decide which of the suggested courses of action logically follow (s) for pursuing. Give answer: (a) if only I follows. (b) if only II follows. (c) if either I or II follows. (d) if neither I nor II follows.

Governmentadda.com | IBPS SSC SBI RBI RRB FCI LIC RAILWAYS

30

Daily Visit :

[GOVERNMENTADDA.COM]

(e) if both I and II follow. Q14. Statement: Many complaints have been registered lately by commuters regarding unavailability of railway tickets during the peak travelling season. Course of action I. The commuters should be advised strictly to defer their travel plane till the peak season gets over. II. Railways should try to accommodate as many commuters as possible by adding extra coaches to the trains. Q15. Statements: The police reported that two armed terrorists had entered the city a couple of days ago. Course of action I. Police should immediately circulate the photographs of the terrorists using electronic and print media in order to take public‘s helps in nabbing them. II. Public should be informed to stay indoors until the terrorists are arrested. Solution(14-15): S14. Ans.(b) Sol. Course of action I does not follow because of the word strictly. Course of action II follows as extra coaches should be added to trains so, that more people can travel. S15. Ans.(a) Sol. Course of action I follows because circulation the phonograph of terrorist in electronic and print media will be helpful in nabbing them. Course of action II does not follow because it is not feasible and a coward step.

Directions (1-5): Study the sets of numbers given below and answer the questions, given below:

Q2. If five is subtracted from each of the numbers, which of the following numbers will be the difference between the second digit of second highest number and the second digit of the highest number? (a) Zero (b) 3 (c) 1 (d) 4 (e) 2 Q3. If in each number the first and the second digits are interchanged, which will be the third highest number? (a) 489 (b) 541 (c) 654 (d) 953 (e) 783 Q4. Which of the following numbers will be obtained if the first digit of lowest number is subtracted from the second digit of highest number after adding one to each of the numbers? (a) 1 (b) 2 (c) 3 (d) 4 (e) 5 Q5. If in each number, the first and the last digits are interchanged, which of the following will be the second highest number? (a) 489 (b) 541 (c) 654 (d) 953 (e) 783 Solution(1-5):

489 541 654 953 783 Q1. If in each number, all the three digits are arranged in ascending order, which of the following will be the lowest number? (a) 489 (b) 541 (c) 654 (d) 953 (e) 783

S1. Ans.(b) Sol.

S2. Ans.(b) Sol.

Governmentadda.com | IBPS SSC SBI RBI RRB FCI LIC RAILWAYS

31

Daily Visit :

[GOVERNMENTADDA.COM] Q7. Statements : H © W, W $ M, M @ B Conclusions : I. B* H II. M % H Q8. Statements : D % B, B T, T $ M Conclusions : I. T © D II. M © D

S3. Ans.(d) Sol.

9. Statements : M T, T @ K, K © N Conclusion : I. N T II. N M

S4. Ans.(a) Sol.

10. Statements : R $ J, J % D, D F Conclusions : I. D $ R II. D @ R

S5. Ans.(c ) Sol.

Solution(6-10):

Directions (6-10): In the following questions, the symbols @, ©, $, %, and are used with the following meanings as illustrated below: ‗P © Q‘ means ‗P is not greater than Q‘. ‗P % Q‘ means ‗P is not smaller than Q‘. ‗P Q‘ means ‗P is neither smaller than nor equal to Q.‘ ‗P @ Q‘ means ‗P is neither greater than nor equal to Q‘. ‗P $ Q‘ means ‗P is neither greater than nor smaller than Q.‘ Now in each of the following questions, assuming the given statement to be true, find which of the two conclusions I and II given below them is/are definitely true. Given answer (a) If only Conclusion I is true. (b) If only conclusion II is true (c) If either conclusion I or II is true (d) if neither Conclusion I nor II is true. (e) if both Conclusions I and II are true. Q6. Statements : K @ V, V © N, N % F Conclusions : I. F @ V II. K @ N

S6. Ans.(b) Sol. Conclusions : I. F @ V ( Not True ) II. K @ N ( True ) S7. Ans.(e) Sol. Conclusions : I. B H ( True ) II. M % H ( True ) S8. Ans.(d) Sol. Conclusions : I. T © D ( Not True ) II. M © D ( Not True ) S9. Ans.(a) Sol. Conclusion : I. N T ( True ) II. N M ( Not True ) S10. Ans.(c ) Sol. Conclusions : I. D $ R ( Not True ) II. D @ R ( Not True )

Governmentadda.com | IBPS SSC SBI RBI RRB FCI LIC RAILWAYS

32

Daily Visit :

[GOVERNMENTADDA.COM] (e) More than three

Directions (11-15): Read the following information and answer the question that follow: Six lectures-A, B, C, D, E and F-are to be delivered from Monday to Sunday one lecture every day. (i) Lecture C cannot be delivered on Friday. (ii) Lecture A is Delivered immediately after lecture D. (iii) There should be a gap of two days between the lectures B and F. (iv) There is one holiday except Saturday. Lecture F is delivered on the next day of holiday. (v) Lecture E is delivered on Wednesday . (vi) Lecture E is not delivered on immediately after the Lecture F. Q11. On which day was lecture D Delivered? (a) Friday (b) Saturday (c) Sunday (d) Thursday (e) None of these

Solution(11-15):

S11. Ans.(b) Sol. S12. Ans.(d) Sol. S13. Ans.(a) Sol. S14. Ans.(a) Sol.

Q12. On which day was holiday? (a) Sunday (b) Friday (c) Monday (d) Thursday (e) None of these

S15. Ans.(a)

Q13. How many lectures were delivered between F and D? (a) None (b) one (c) two (d) Three (e) None of these Q14. Which of the following was the last lecture? (a) A (b) C (c) B (d) Can‘t be determined (e) None of these Q15. How many lectures scheduled before the day on which C delivered? (a) No one. (b) One (c) Two (d) Three

Directions (1-5): Study the following information to answer the given questions. Eight persons, viz A, B, C, D, E, F, G and H are parking their car in a straight line facing towards north direction but not necessarily in the same order. Each of them has different car viz. Tata, Maruti, Toyota, Renault, Honda ,Ford, Audi and Mercedes but not necessarily in the same order. G is parking his car third to the right of the one who has Honda car. Mercedes car is parked second to the right of G. A and E are parking their car adjacent to each other. Neither A nor E has either Honda or Mercedes car. Neither A nor E park their car adjacent to G. H parks his car third to the right of the one who has Tata car. Neither A nor E has Tata car. H does not have Mercedes car. Only two persons park their car between E and the one who has Audi car. Maruti car is parked on the immediate left of the D. Only one person parks his car between E and B. C parks his car second to left of one who has Audi car. E does not have Toyota car. The one who has Ford car parks his car at the extreme end of the line.

Governmentadda.com | IBPS SSC SBI RBI RRB FCI LIC RAILWAYS

33

Daily Visit :

[GOVERNMENTADDA.COM]

Q1.Who among the following parks his car exactly between E and B? (a)The one who has Honda car (b)The pone who has Tata car (c)D (d)A (e)The one who has Mercedes car Q2.„H‟ is related to „Audi‟ in a certain way based on the above arrangement. „B‟ is related to „Renault‟ following the same pattern. „_____‟ is related to „Honda‟ following the same pattern. (a)F (b)G (c)A (d)D (e)C Q3.Which of the following is true regarding D? (a)Only two people park their car left of D. (b)D parks his car second to the right of the one who has Audi car. (c)E and B both park their car adjacent to D. (d)D has Mercedes car. (e)None is true Q4.How many people park their car between C and the one who has Maruti car? (a)None (b)One (c)Two (d)Three (e)Four Q5.Who among the following are parking their car at extreme ends of the line? (a)A and the one who has Mercedes car (b)The one who has Honda car and E (c)C and G (d)The one who has Toyota car and the one who has Ford car (e)None of these Solutions(1-5):

S1. Ans.(a) Sol.

S2. Ans.(a) Sol. S3. Ans.(d) Sol. S4. Ans.(d) Sol. S5. Ans.(a) Sol. Directions (6-10): In the following question @, $, #, % and & are used according to following meaning. ‗P # Q‘ means, ‗P is not greater than Q‘. ‗P % Q‘ means, ‗P is neither greater nor smaller than Q‘. ‗P & Q‘ means, ‗P is neither greater nor equal to Q‘. ‗P $ Q‘ means, ‗P is not smaller than Q‘. ‗P @ Q‘ means, ‗P is neither smaller nor equal to Q‘. Now according to the following statement if they are true, judge their Conclusions I, II and III follow definitely true. Q6. Statements: Z $ X, X @ W, W % V Conclusions: I. Z @ V II. X % V III. V # Z (a) Only I is true (b) Only II is true (c) Only III is true (d) I and III are true (e) None of these Q7. Statements: B @ X, X # F, F $ H Conclusions: I. F @ B II. H % X III. H # B (a) Only I is true (b) I and III are true (c) Only II is true (d) None is true (e) None of these Q8. Statements: Z & S, S @ T, T $ U Conclusions: I. U # S II. T & Z III. S @ U (a) Either I or III is true

Governmentadda.com | IBPS SSC SBI RBI RRB FCI LIC RAILWAYS

34

Daily Visit :

[GOVERNMENTADDA.COM]

(b) Only I is true (c) Only III is true (d) I and II are true (e) None of the above Q9. Statements: K % H, H # G, G & I Conclusions: I. H & I II. G $ K III. K # I (a) Only I is true (b) I and II are true (c) Only III is true (d) None is true (e) All of the above Q10. Statements: T @ Y, Y $ G, G # W Conclusions: I. G % T II. T # G III. T @ G (a) Either I or II is true (b) Either II or III is true (c) Only III is true (d) None is true (e) None of the above Solution(6-10): S6. Ans.(a) Sol. I. Z >V ( True ) II. X = V (Not True ) III. V ≤ Z (Not True ) S7. Ans.(d) Sol. I. F >B (Not True ) II. H = X (Not True ) III. H ≤B (Not True ) S8. Ans.(c) Sol. I. U ≤ S (Not True ) II. T U ( True ) S9. Ans.(b) Sol. I. H < I ( True ) II. G ≥K ( True )

III. K ≤ I (Not True ) S10. Ans.(c) Sol. I. G = T (Not True ) II. T ≤ G (Not True ) III. T > G ( True ) Directions (11-15): In each of the questions below are given four statements followed by four conclusions numbered I, II, III and IV. You have to take the given statements to be true even if they seem to be at variance from commonly known facts. Read all the conclusions and then decide which of the given conclusions logically follows from the given statements disregarding commonly known facts. Q11. Statements: All door are paint. All paint are windows. All windows are walls.. Some windows are water. Conclusions: I. Some walls are windows. II. All door are walls. III. Some windows are door. IV. Some water are windows. (a) Only I, II and III follow (b) Only II, III and IV follow (c) Only I, III and IV follow (d) Only I, II and IV follow (e) All are true. Q12. Statements: Some boy are girls. Some girls are womens. All womens are aunt. Some aunt are uncle. Conclusions: I. Some uncle are girls. II. Some girls are aunt. III. Some aunt are womens. IV. Some womens are boy. (a) None follows (b) Only I follows (c) Only II follows (d) Only II and III follow (e) Only IV follows Q13. Statements: All bench are desk. Governmentadda.com | IBPS SSC SBI RBI RRB FCI LIC RAILWAYS

35

Daily Visit : Some desk are rods. All rods are papers. Some papers are windows. Conclusions: I. Some windows are bench. II. Some papers are desk. III. Some desk are bench. IV. Some papers are bench. (a) Only I and II follow (b) Only II and III follow (c) Only III and IV follow (d) Only I and III follow (e) None of these Q14. Statements: Some cool are warm. Some warm are winter. Some winter are hands. Some hands are ears. Conclusions: I. Some ears are winter. II. Some winter are cool. III. Some hands are warm. IV. Some cool is ears. (a) None follows (b) Only II follows (c) Only IV follows (d) Only either II or IV follows (e) Only III follows Q15. Statements: All bad are cart. Some cart are yard. All yard are seat. Some seat are tables. Conclusions: I. Some seat are cart. II. Some tables are seat. III. Some cart are bad. IV. Some seat are yard. (a) Only II, III and IV follow (b) Only I, II and III follow (c) Only I, III and IV follow (d) All follow (e) None of these Solution(11-15): S11. Ans.(e) Sol.

[GOVERNMENTADDA.COM]

S12. Ans.(d) Sol.

S13. Ans.(b) Sol.

S14. Ans.(a) Sol.

S15. Ans.(d) Sol.

Directions (1-5): Study the following information carefully and answer the questions given below: A, B, C, D, E, F, G and H are eight persons who live in an eight-storey building. The ground floor is numbered one and the topmost floor is numbered eight. Each of Governmentadda.com | IBPS SSC SBI RBI RRB FCI LIC RAILWAYS

36

Daily Visit :

[GOVERNMENTADDA.COM]

them have different qualifications viz. B.TECH, MD, MS, CA, M.TECH, M.SC, MCA and MBBS, but not necessarily in the same order. There is only one floor between A and the floor on which person who has only MBBS degree . The one who has only MBBS degree does not live on floor number 1. D lives just below B. The one who has B.TECH degree lives on an evennumbered floor and just above the floor on which one who has M.TECH degree. The person who has M.SC degree lives on an even-numbered floor but not on the 8th floor. Neither D nor H lives on the 1st floor. Only one person lives between the one who has MCA degree and the D. A lives on an odd-numbered floor and E lives just above A. B lives on the fourth floor. Only two persons live between the one who has M.SC degree and A. F lives just below the one who has M.TECH degree. D has neither M.TECH nor MBBS degree. The person who has MS degree does not live on an odd-numbered floor. G does not has MD degree. There are two floors between the floor on which H lives and the floor on which E lives. Only two persons live between the one who has CA degree and the one who has MS degree. Q1. Who among the following has been applied for MD? (a) D (b) C (c) F (d) E (e) None of these Q2. How many persons are there between E and B? (a) One (b) Two (c) Three (d) Four (e) None of these Q3. Who among the following lives on the topmost floor? (a) The one who applied in B.TECH (b) The one who applied in M.SC (c) The one who applied in MS (d) The one who applied in MD (e) None of these Q4. Which of the following combinations is/are true? (a) Floor no. 2 – D – CA (b) Floor no. 5 – F – MBBS (c) Floor no. 1 – C – MD

(d) Floor no. 8 – E – B.TECH (e) None of these Q5. A is applying in which of the following examinations? (a) B.TECH (b) MBBS (c) M.SC (d) M.TECH (e) None of these Solution(1-5):

S1. Ans.(b) Sol. S2. Ans.(c) Sol. S3. Ans.(a) Sol. S4. Ans.(d) Sol. S5. Ans.(d) Sol. Directions (6-10): Study the information and answer the following questions: In a certain code language "keeping the interest rate" is coded as " 8#V 16#G 9@G 21@P " "second consecutive year that" is coded as " 12#W 33@X 8#I 8#G"

Governmentadda.com | IBPS SSC SBI RBI RRB FCI LIC RAILWAYS

37

Daily Visit :

[GOVERNMENTADDA.COM]

"labour ministry has raised" is coded as" 12#I 16#B 9@S 12#W" Q6.What is the code for „latest tussle‟ in the given code language? (a) 12@G 12@V (b) 12#G 12#V (c) 12#G 12@V (d) 12@G 12#V (e)None of these Q7.What is the code for „surplus over‟ in the given code language? (a) 21@H 8#I (b) 20@H 8#I (c) 21#H 8#I (d) 21@H 8@I (e)None of these Q8.What may be the possible code for „finance ministry‟ in the given code language? (a) 21@U 16#B (b) 16@V 16@B (c) 16#V 16@B (d) 16@V 16#B (e)None of these Q.9What may be the possible code for „inception‟ in the given code language? (a) 25@R (b) 27#R (c) 25#R (d) 27@R (e)None of these Q10.What is the code for „estimates' in the given code language? (a) 27@V (b) 21#V (c) 20@V (d) 21@S (e)Noneofthese Solution(6-10):

S6. Ans.(b) Sol. S7. Ans.(a) Sol. 21@H 8#I S8. Ans.(a) Sol. 21@U 16#B S9. Ans.(d) Sol. 27@R S10. Ans.(a) Sol. 27@V Q11. Of the five poles P, Q, R, S and T situated close to each other, P is to the west of Q, R is to the south of P and T is to north of Q and S is to the east of T. Then R is in which direction with respect to S? (a) North-west (b) South-east (c) South-west (d) Data inadequate (e) None of these Q12. A man starts walking from point P. He goes 90 metres in the East before turning to his right. He goes 20 metres before turning to his right again and walk 30 metres and reaches point Q. From point Q, he goes 100 metres to the North and reaches point R. What is shortest distance between P and R? (a) 80 metres (b) 100 metres

Governmentadda.com | IBPS SSC SBI RBI RRB FCI LIC RAILWAYS

38

Daily Visit :

[GOVERNMENTADDA.COM]

(c) 140 metres (d) 260 metres (e) None of these Q13. If „A $ B‟ means „A is father of B‟, „A # B‟ means „A is daughter of B‟, „A @ B‟ means „A is sister of B‟, then how is K related to M in H @ K $ L #M? (a) Husband (b) Uncle (c) Father (d) Cannot be determined (e) None of these Q14. If „P $ Q‟, means „P is father of Q‟; „P # Q‟ means „P is mother of Q‟; „P * Q‟ means „P is sister of Q‟, then how is D related to N in N # A $ B * D? (a) Nephew (b) Grandson (c) Granddaughter (d) Cannot be determined (e) None of these

S12. Ans.(b) Sol.

Q15. In a class six students M, N, O, P, Q and R are the top six Rank holders, not necessarily in the same order. O did not get the 4th rank. M‟s rank is higher than R‟s and O‟s but lower than N‟s. Among these six rankers, there are four students whose ranks are lower than P‟s rank and five students whose ranks are above that of Q. Who is ranked 5th in the class? (a) R (b) O (c) Q (d)N (d) None of these Solution(11-15): S11. Ans.(c) Sol.

S13. Ans.(a) Sol.

Governmentadda.com | IBPS SSC SBI RBI RRB FCI LIC RAILWAYS

39

Daily Visit :

[GOVERNMENTADDA.COM]

S14. Ans.(d) Sol.

N. S sits second to the right of R. M likes Black and he does not research on Heart or Plastic surgery. P does not research on Plastic Surgery. O does not sit in the middle. The one who research on Autism sits opposite to the one who likes yellow. O does not likes orange or Red. The one who likes orange faces outside. O does not research on obesity. Q1. Who among the following likes yellow? (a) M (b) S (c) N (d) R (e) P Q2. Who among the following research on Obesity? (a) R (b) S (c) M (d) T (e) None of these

S15. Ans.(b) Sol. N>P>M>R>O>Q

Directions (1-5): Study the following information carefully and answer the following questions. There are eight persons – M, N, O, P, Q, R, T and S – who are sitting around a square table such that four of them sit on the middle side of table and facing away from the center and other four of them sit at the corner of table and they are facing towards the center of the table. They all are doing research in different field viz. Heart surgery, Obesity, Cancer, Autism, Depression, Plastic Surgery, Sleep and Drug Abuse but not necessarily in the same order. They like different colour‘s viz. Red, Green, Black, White, Yellow, Violet, Blue and Orange but not necessarily in the same order. The one who likes violet, researchers on Depression and he does not faces inside. N sits third to the right of T who like Green. Q sits second to the right of P . Q researches on Sleep. S researches on Drug abuse and he does not like white or red or orange. The one who researches on Plastic surgery sits opposite to the one who is doing his research on Heart surgery. R is doing his research on Cancer and sits third to the left P who likes Blue.P faces inside. P is an immediate neighbour of

Q3. Who among the following sits opposite the one who research on Depression? (a) Both c and d (b) S (c) R (d) The one who research on Cancer (e) The one who research on obesity Q4. P is doing research in which field? (a) Obesity (b) Plastic surgery (c) Heart Surgery (d) Cancer (e) None of these Q5. Who among the following sits second to the right of the one who is doing research on Depression? (a) S (b) R (c) Q (d) O (e) M Solutions (1-5):

Governmentadda.com | IBPS SSC SBI RBI RRB FCI LIC RAILWAYS

40

Daily Visit :

[GOVERNMENTADDA.COM] Q6. Among P, Q, R, S and T each having a different height who among following is taller than only the shortest? I. Q is taller than T and P. S is shorter than only R. II. Q is shorter than only two persons. S is taller than Q and T. P is neither the shortest nor the tallest. Q7. How is „how‟ written in a code language? I. ‗you do this‘ is written as ‗pa da na‘ in some code language and ‗do this again‘ is written as ‗na ka da‘ in code language. II. ‗how do this‘ is written as ‗da na sx‘ in that code language. Q8. Y is which direction with respect to X? I. F is to the West of Q, which is to the West of M. M is north of X. Y is south of M. II. M is to the east of Q. X is south of M. F is south of Q. Y is west of F.

S1.Ans.(b) Sol. S2.Ans.(d) Sol.

Q9. V, W, X, Y and Z are sitting in a row. If Y is sitting middle of the row, who is sitting extreme left? (All are facing north.) I. Z is sitting immediate right of Y, but on the left of W. X does not sit any of the extreme end of line. II. X always sits at either of the extreme ends. Neither Y nor W is an immediate neighbour of X.

S3.Ans.(a) Sol. S4.Ans.(c) Sol. S5.Ans.(e) Sol. Directions (6-10): Each of the questions below consists of a question and two statements numbered I and II given below it. You have to decide whether the data provided in the statements are sufficient to answer the question. Read both the statements and give the answer (a) if the data in statement I alone are sufficient to answer the question, while the data in statement II alone are not sufficient to answer the question. (b) if the data in statement II alone are sufficient to answer the question, while the data in statement I alone are not sufficient to answer the question. (c) if the data either in statement I alone or in statement II alone are sufficient to answer the question. (d) if the data in both the statements I and II together are not sufficient to answer the question. (e) if the data in both statements I and II together are necessary to answer the question.

Q10. How is Z related to A? I. X is father of Y. Z is only sister of Y. A is mother of Y. II. X is mother of Y. Y is father of Z. A is grandfather of Z. Solutions (6-10): S6. Ans.(e) Sol. If the data in both the statements I and II are together necessary to answer the question. From both Statement I and IIR>S>Q>P>T S7. Ans.(e) Sol. If the data in both the statements I and II are together necessary to answer the question. From both Statement I and IIhow=sx S8. Ans.(d) Sol. If the data in both the statements I and II together are not sufficient to answer the question.

Governmentadda.com | IBPS SSC SBI RBI RRB FCI LIC RAILWAYS

41

Daily Visit :

[GOVERNMENTADDA.COM]

S9. Ans.(a) Sol. From 1- V is in on the extreme left.

From 2- We cannot decide. S10. Ans. (a) Sol. If the data in statement I alone are sufficient to answer the question, while the data in statement II alone are not sufficient to answer the question.

Direction (11-12): In each questions below is given a statement following by three courses of action numbered I, II and III. A course of action is a step or administrative decision to be taken for improvement, follow up or further action in regard to the problem, policy etc. On the basis of the information given in the statement, you have to assume everything in the statement to be true, then decide which of the suggested coursed of action logically follow(s) pursuing Q11. Statements: Many students of the local school fell ill for the fourth time in a row in the last six months after consuming food prepared by the school canteen. Course of action I. The School management should immediately terminate the contract of the canteen and ask for compensation. II. The school management should advice all the students not to eat food articles from the canteen. III. The owner of the canteen should immediately be arrested for negligence. (a) None follows (b) Only II follows (c) Only III follows (d) I and II follow (e) II and II follow

Q12. Statements: Many school buses have fitted CNG kit without observing the safety guidelines properly. This results into some instances of these buses catching fire due to short circuit and endangering the lives of the school children. Course of action I. The regional transport authority should immediately carry out checks of all the school buses fitted with CNG kit. II. The management of all the schools should stop hiring buses fitted with CNG kit. III. The government should issue a notification banning school buses for use of CNG kit. (a) Only I follows (b) Only II follows (c) Only III follows (d) I and III follows (e) None of these Directions (13-15): Below in each question are given two statements (A) and (B). These statements may be either independent causes or may be effects of independent causes or of a common cause. One of these statements may be the effect of the other statement. Read both the statements and decide which of the following answer choices correctly depicts the relationship between these two statements. Mark answer (a) if statement (A) is the cause and statement (B) is its effect. (b) if statement (B) is the cause and statement (A) is its effect. (c) if both the statement (A) and (B) are independent causes. (d) if both the statements (A) and (B) are effects of independent causes. (e) if both the statements (A) and (B) are effects of some common cause. Q13. (A) The association of management colleges conducted a combined admission exam for all the states this year. (B) As the dates for entrance exam for many management colleges clashed last year, many candidates complained that they could not appear for a number of entrance exams. Q14. (a) The conditions of all the major roads in the city have deteriorated causing hardship to motorists. (b) The municipal authority has sanctioned significant amount to repair all the major roads in the city.

Governmentadda.com | IBPS SSC SBI RBI RRB FCI LIC RAILWAYS

42

Daily Visit :

[GOVERNMENTADDA.COM]

Q15. (a) The price of aircraft fuel has risen during the past few months. (b) Many passenger airlines in India have been forced to cut their air fares by about 10 percent. Solutions (11-15): S11. Ans.(d) Sol. As the incident is repeated again and again, so the contract of the canteen should be terminated and compensation should be asked. Students should be advised not to eat food articles from the canteen. So, courses of action I and II follow. Course of action III is too harsh, so it does not follow. S12. Ans.(a) Sol. As safety guidelines are not observed by many school buses, so regional transport authority should carry out checks of all the buses fitted with CNG Kit. So, course of action I follows. CNG is a pollution free fuel and its sources are more than petrol and diesel So, it should be encouraged but with safety norms. So, course of action II and III do not follow. S13. Ans.(b) Sol. The association of management colleges conducted a combined admission exam for all the institutes this year because last year date entrance exam of many management colleges clashed and many student could appear for a number of entrance exam. So, statement (B) is the cause and (A) is its effect. S14. Ans.(a) Sol. Statement (A) is the cause while (B) is it effect. S15. Ans.(d) Sol. Both statements (A) and (B) are effects of independent causes.

Directions (1-5): Read the following information carefully and answer the questions which follow. There are eight employees A, B, C, D, E, F, G, and H are living on eight storey building in USA. The ground floor is numbered one and next floor is number two and so on upto last floor is numbered eight. They all work in different MNC viz. Birla, Tata, Infosys, Wipro, Airtel, Hcl, Mahindra and Mindtree but not necessarily in the same order. They earns different annual income viz. $18L, $25L, $35L, $75L, $56L, $31L, $99L and $89L but not necessarily in the same order.

There are three floors between the A and the one who earns $89L, both of them living on even number floor but none of them living on top floor. F lives on 3rd floor and works in Hcl . H works in Birla and lives immediately below A. B works in Tata. There are gap of two floors between the floors on which B and H lives. There is only one floor between B and G, who works in Infosys. C lives immediately above D, who earns $99L. There are gap of two floors between the one who earn $99L and the one who earn $31L. The one who earns $56L works in Mindtree but he does not live on odd number floor. The one who works in Airtel lives on one of the floor below on which the person who earns $18L lives. There are as many as person lives between the one who works in Hcl and the one who earns $75L and as between the one who earns $75L and the one who earns $35L. The one who earns $25L live one of the floor below on which the person who works in Airtel lives. There are as many as floor between C and E who works in Wipro and as between A and who earns $18L. Q1. Who among following lives on top floor? (a) B (b) E (c) G (d) D (e) C Q2. Who among following earns $25L? (a) E (b) B (c) F (d) A (e) H Q3. Who among following works in Mahindra? (a) E (b) A (c) G (d) D (e) None of these Q4. If C is relate to $18L and E is related to $75L, then in the same way B is related to? (a) $18L (b) $99L (c) $89L (d) $25L (e) None of these Q5. E lives on which of the following floor?

Governmentadda.com | IBPS SSC SBI RBI RRB FCI LIC RAILWAYS

43

Daily Visit :

[GOVERNMENTADDA.COM] Q7.What is the code for „statement‟ in the given code language? (a) 39#I (b) 3#F (c) 39@F (d) 39#F (e)None of these

(a) Third (b) Fourth (c) Fifth (d) Sixth (e)Second Solutions (1-5):

Q8.What is the code for „profile‟ in the given code language? (a) 21#H (b) 21#I (c) 22#H (d) 1#I (e)None of these S1. Ans. (e) Sol. .

Q9.What is the code for „constraint‟ in the given code language? (a) 23@K (b) 3@K (c) 23#K (d) 23@L (e)None of these

S2. Ans. (e) Sol. S3. Ans. (d) Sol. S4. Ans. (d) Sol. S5. Ans.(c) Sol. Directions (6-10): Study the information and answer the following questions: In a certain code language

Q10.What is the code for „nice‟ in the given code language? (a) 1@Q (b) 19@Q (c) 19@L (d) 19#Q (e)None of these Solutions (6-10):

"high growth potential and " is coded as " 16@Q 15@H 28#K 5#L " " debt burden over the " is coded as " 24@U 16@E 33@D 25#R " " credit with stable outlook " is coded as " 23@H 31@Q 24@F 26#E "

S6. Ans.(d) Sol.

Q6.What is the code for „expectation‟ in the given code language? (a) 19#C (b) 19@B (c) 19#D (d) 19#B (e)None of these

S7. Ans.(d) Sol. S8. Ans.(a) Sol. S9. Ans.(a) Sol.

Governmentadda.com | IBPS SSC SBI RBI RRB FCI LIC RAILWAYS

44

Daily Visit :

[GOVERNMENTADDA.COM]

S10. Ans.(b) Sol. Q11. If ‗A $ B‘ means ‗A is father of B‘, ‗A # B‘ means ‗A is daughter of B‘, ‗A @ B‘ means ‗A is sister of B‘, then how is K related to M in H @ K $ L # M ? (a) Husband (b) Uncle (c) Father (d) Cannot be determined (e) None of these S11. Ans.(a) Sol.

Q12. If ‗P $ Q‘, means ‗P is father of Q‘; ‗P # Q‘ means ‗P is mother of Q‘; ‗P * Q‘ means ‗P is sister of Q‘, then how is D related to N in N # A $ B * D? (a) Nephew (b) Grandson (c) Granddaughter (d) Cannot be determined (e) None of these S12. Ans.(d) Sol.

Q13. Pole P is 13 km towards the East of Pole Q. Siddharth, starts from Pole Q, travels 8 km towards West and takes a right turn. After taking the right turn, he travels 5 km and reaches Pole B. From Pole B , Siddharth takes a right turn again, travels 21 km and reaches Pole C. How far and towards which direction must the Siddharth travel to reach Pole P? (a) 5 km towards South (b) 5 km towards West (c) 21 km towards South (d)13 km towards South (e) None of these

S13. Ans.(a) Sol.

Q(14-15): These questions are based on the following information: Each of six students P, Q, R, S, T, and U gets different marks in their annual examination. Only two students gets less mark than T. S gets more mark than U but gets less marks than P. P does not get highest mark. R gets Governmentadda.com | IBPS SSC SBI RBI RRB FCI LIC RAILWAYS

45

Daily Visit :

[GOVERNMENTADDA.COM]

less mark than U. The student who got second highest marks gets 70 percent marks. Q14. If the addition of percentage of marks of P and T is 120 percent, then what may be the percentage mark of S? (a) 60% (b) 80% (c) 45% (d) 40% (e) 49% Q15. If the addition of percentage of marks of P and T is 120 percent ,then how many marks probably obtained by U? (a) 55% (b) 60% (c) 45% (d) 65% (e) 70# Solution(14-15): Q > P(70%) >S > T > U > R S14. Ans.(a) Sol. S15. Ans.(c)

Governmentadda.com | IBPS SSC SBI RBI RRB FCI LIC RAILWAYS

46

GovernmentAdda.com

501 CHALLENGING LOGIC AND REASONING PROBLEMS

GovernmentAdda.com

GovernmentAdda.com

501 CHALLENGING LOGIC AND REASONING PROBLEMS

2nd Edition

®

NEW

YORK

GovernmentAdda.com

Copyright © 2005 LearningExpress, LLC. All rights reserved under International and Pan-American Copyright Conventions. Published in the United States by LearningExpress, LLC, New York. Library of Congress Cataloging-in-Publication Data: 501 challenging logic & reasoning problems. p. cm.—(LearningExpress skill builders practice) Includes bibliographical references. ISBN 1-57685-534-1 1. Logic—Problems, exercises, etc. 2. Reasoning—Problems, exercises, etc. 3. Critical thinking—Problems, exercises, etc. I. LearningExpress (Organization) II. Title: 501 challenging logic and reasoning problems. III. Series. BC108.A15 2006 160'.76—dc22 2005057953 Printed in the United States of America 9 8 7 6 5 4 3 2 1 Second Edition ISBN 1-57685-534-1 For information or to place an order, contact LearningExpress at: 55 Broadway 8th Floor New York, NY 10006 Or visit us at: www.learnatest.com

GovernmentAdda.com

Contents

INTRODUCTION

vii

QUESTIONS

1

ANSWERS

99

v

GovernmentAdda.com

GovernmentAdda.com

Introduction

T

his book—which can be used alone, with other logic and reasoning texts of your choice, or in combination with LearningExpress’s Reasoning Skills Success in 20 Minutes a Day—will give you practice dealing with the types of multiple-choice questions that appear on standardized tests assessing logic, reasoning, judgment, and critical thinking. It is designed to be used by individuals working on their own and by teachers or tutors helping students learn, review, or practice basic logic and reasoning skills. Practice on 501 logic and reasoning questions will go a long way in alleviating test anxiety, too! Maybe you’re one of the millions of people who, as students in elementary or high school, never understood the necessity of having to read opinion essays and draw conclusions from the writer’s argument. Or maybe you never understood why you had to work through all those verbal analogies or number series questions. Maybe you were one of those people who could never see a “plan of attack” when working through logic games or critical thinking puzzles. Or perhaps you could never see a connection between everyday life and analyzing evidence from a series of tedious reading passages. If you fit into one of these groups, this book is for you. First, know you are not alone. It is true that some people relate more easily than do others to number series questions, verbal analogies, logic games, and reading passages that present an argument. And that’s okay; we all have unique talents. Still, it’s a fact that for most jobs today, critical thinking skills—including analytical and logical reasoning—are essential. The good news is that these skills can be developed with practice. Learn by doing. It’s an old lesson, tried and true. And it’s the tool this book is designed to give you. The 501 logic and reasoning questions that follow will provide you with lots of practice. As you work through each set of questions, you’ll be gaining a solid understanding of basic analytical and logical reasoning skills—all without memorizing! The purpose of this book is to help you improve your critical thinking through encouragement, no frustration.

vii

GovernmentAdda.com – INTRODUCTION –



An Over view

Working on Your Own

If you are working alone to improve your logic skills or prepare for a test in connection with a job or school, you will probably want to use this book in combination with its companion text, Reasoning Skills Success in 20 Minutes a Day, 2nd Edition, or with some other basic reasoning skills text. If you’re fairly sure of your basic logic and reasoning abilities, however, you can use 501 Challenging Logic and Reasoning Problems by itself. Use the answer key at the end of the book not only to find out if you got the right answer, but also to learn how to tackle similar kinds of questions next time. Every answer is explained. Make sure you understand the explanations—usually by going back to the questions—before moving on to the next set.

501 Challenging Logic and Reasoning Problems is divided into 37 sets of questions: Sets 1–4: Number Series Sets 5–6: Letter and Symbol Series Sets 7–8: Verbal Classification Sets 9–11: Essential Part Sets 12–17: Analogies Sets 18–19: Artificial Language Set 20: Matching Definitions Set 21: Making Judgments Set 22: Verbal Reasoning Sets 23–27: Logic Problems Sets 28–31: Logic Games Sets 32–37: Analyzing Arguments

Tutoring Others

This book will work well in combination with almost any analytical reasoning or logic text. You will probably find it most helpful to give students a brief lesson in the particular operation they’ll be learning— number series, verbal classification, artificial language, logic problems, analyzing arguments—and then have them spend the remainder of the session actually answering the questions in the sets. You will want to stress the importance of learning by doing and of checking their answers and reading the explanations carefully. Make sure they understand a particular set of questions before you assign the next one.

Each set contains between 5–20 questions, depending on their length and difficulty. The book is specifically organized to help you build confidence as you further develop your logic and reasoning skills. 501 Challenging Logic and Reasoning Problems begins with basic number and letter series questions, and then moves on to verbal classification, artificial language, and matching definition items. The last sets contain logic problems, logic games, and logical reasoning questions. By the time you reach the last question, you’ll feel confident that you’ve improved your critical thinking and logical reasoning abilities.

 

Additional Resources

How to Use This Book Answering the 501 logic and reasoning questions in this book will give you lots of practice. Another way to improve your reasoning ability is to read and study on your own and devise your own unique methods of attacking logic problems. Following is a list of logic and reasoning books you may want to buy or take out of the library:

Whether you’re working alone or helping someone brush up his or her critical thinking and reasoning skills, this book will give you the opportunity to practice, practice, practice!

viii

GovernmentAdda.com – INTRODUCTION –

REASONING

CRITICAL THINKING

Reasoning Skills Success in 20 Minutes a Day (2nd Edition) by LearningExpress Critical Reasoning: A Practical Introduction by Anne Thomson (Routledge) Attacking Faulty Reasoning: A Practical Guide to Fallacy-Free Arguments by T. Edward Damer (Wadsworth) Thinking Critically: Techniques for Logical Reasoning by James H. Kiersky and Nicholas J. Caste (Wadsworth)

Critical Thinking by Alec Fisher (Cambridge University Press) Brainplay: Challenging Puzzles & Thinking Games by Tom Werneck (Sterling) Challenging Critical Thinking Puzzles by Michael A. Dispezio and Myron Miller (Sterling) Becoming a Critical Thinker: A User-Friendly Manual by Sherry Diestler (Prentice Hall) ANALOGIES

501 Word Analogy Questions by LearningExpress Analogies for Beginners by Lynne Chatham (Dandy Lion Publications) Cracking the MAT (3rd Edition) by Marcia Lerner (Princeton Review)

LOGIC

Essential Logic: Basic Reasoning Skills for the Twenty-First Century by Ronald C. Pine (Oxford University Press) Increase Your Puzzle IQ: Tips and Tricks for Building Your Logic Power by Marcel Danesi (Wiley) Amazing Logic Puzzles by Norman D. Willis (Sterling) Challenging Logic Puzzles by Barry R. Clarke (Sterling)

ix

GovernmentAdda.com

GovernmentAdda.com

Questions

R

eady to test your mental abilities? Your 501 challenging logic and reasoning problems begin on the next page. They’re grouped together in sets of 5–20 questions with a common theme. You can work through the sets in order or jump around, whichever you choose. When you finish a set, check your answers beginning on page 99.

1

GovernmentAdda.com – QUESTIONS –



Set 1

4. Look at this series: 544, 509, 474, 439, . . . What number should come next? a. 404 b. 414 c. 420 d. 445

(Answers begin on page 99.)

Start off with these simple series of numbers. Number series questions measure your ability to reason without words. To answer these questions, you must determine the pattern of the numbers in each series before you will be able to choose which number comes next. These questions involve only simple arithmetic. Although most number series items progress by adding or subtracting, some questions involve simple multiplication or division. In each series, look for the degree and direction of change between the numbers. In other words, do the numbers increase or decrease, and by how much?

5. Look at this series: 201, 202, 204, 207, . . . What number should come next? a. 205 b. 208 c. 210 d. 211 6. Look at this series: 8, 22, 8, 28, 8, . . . What number should come next? a. 9 b. 29 c. 32 d. 34

1. Look at this series: 2, 4, 6, 8, 10, . . . What number should come next? a. 11 b. 12 c. 13 d. 14

7. Look at this series: 80, 10, 70, 15, 60, . . . What number should come next? a. 20 b. 25 c. 30 d. 50

2. Look at this series: 58, 52, 46, 40, 34, . . . What number should come next? a. 26 b. 28 c. 30 d. 32

8. Look at this series: 36, 34, 30, 28, 24, . . . What number should come next? a. 20 b. 22 c. 23 d. 26

3. Look at this series: 40, 40, 47, 47, 54, . . . What number should come next? a. 40 b. 44 c. 54 d. 61

9. Look at this series: 22, 21, 23, 22, 24, 23, . . . What number should come next? a. 22 b. 24 c. 25 d. 26

2

GovernmentAdda.com – QUESTIONS –

10. Look at this series: 3, 4, 7, 8, 11, 12, . . . What number should come next? a. 7 b. 10 c. 14 d. 15

16. Look at this series: 7, 10, 8, 11, 9, 12, . . . What number should come next? a. 7 b. 10 c. 12 d. 13

11. Look at this series: 31, 29, 24, 22, 17, . . . What number should come next? a. 15 b. 14 c. 13 d. 12

17. Look at this series: 14, 28, 20, 40, 32, 64, . . . What number should come next? a. 52 b. 56 c. 96 d. 128

12. Look at this series: 21, 9, 21, 11, 21, 13, . . . What number should come next? a. 14 b. 15 c. 21 d. 23

18. Look at this series: 1.5, 2.3, 3.1, 3.9, . . . What number should come next? a. 4.2 b. 4.4 c. 4.7 d. 5.1

13. Look at this series: 53, 53, 40, 40, 27, 27, . . . What number should come next? a. 12 b. 14 c. 27 d. 53

19. Look at this series: 5.2, 4.8, 4.4, 4, . . . What number should come next? a. 3 b. 3.3 c. 3.5 d. 3.6

14. Look at this series: 2, 6, 18, 54, . . . What number should come next? a. 108 b. 148 c. 162 d. 216

20. Look at this series: 2, 1, 12, 14, . . . What number should come next? a. b. c. d.

15. Look at this series: 1,000, 200, 40, . . . What number should come next? a. 8 b. 10 c. 15 d. 20

3

1  3 1  8 2  8 1 1 6

GovernmentAdda.com – QUESTIONS –



Set 2

24. 18 21 25 18 29 33 18 a. 43 18 b. 41 44 c. 37 18 d. 37 41 e. 38 41

(Answers begin on page 101.)

This set contains additional, and sometimes more difficult, number series questions. Again, each question has a definite pattern. Some of the number series may be interrupted by a particular number that appears periodically in the pattern. For example, in the series 14, 16, 32, 18, 20, 32, 22, 24, 32, the number 32 appears as every third number. Sometimes, the pattern contains two alternating series. For example, in the series 1, 5, 3, 7, 5, 9, 7, the pattern is add 4, subtract 2, add 4, subtract 2, and so on. Look carefully for the pattern, and then choose which pair of numbers comes next. Note also that you will be choosing from five options instead of four.

25. 9 11 33 13 15 33 17 a. 19 33 b. 33 35 c. 33 19 d. 15 33 e. 19 21 26. 2 8 14 20 26 32 38 a. 2 46 b. 44 50 c. 42 48 d. 40 42 e. 32 26

21. 84 78 72 66 60 54 48 a. 44 34 b. 42 36 c. 42 32 d. 40 34 e. 38 32

27. 28 25 5 21 18 5 14 a. 11 5 b. 10 7 c. 11 8 d. 5 10 e. 10 5

22. 3 8 13 18 23 28 33 a. 39 44 b. 38 44 c. 38 43 d. 37 42 e. 33 38

28. 9 12 11 14 13 16 15 a. 14 13 b. 18 21 c. 14 17 d. 12 13 e. 18 17

23. 20 20 17 17 14 14 11 a. 8 8 b. 11 11 c. 11 14 d. 8 9 e. 11 8

29. 75 65 85 55 45 85 35 a. 25 15 b. 25 85 c. 35 25 d. 85 35 e. 25 75

4

GovernmentAdda.com – QUESTIONS –

30. 1 10 7 20 13 30 19 a. 26 40 b. 29 36 c. 40 25 d. 25 31 e. 40 50

36. 9 16 23 30 37 44 51 a. 59 66 b. 56 62 c. 58 66 d. 58 65 e. 54 61

31. 10 20 25 35 40 50 55 a. 70 65 b. 60 70 c. 60 75 d. 60 65 e. 65 70

37. 8 22 12 16 22 20 24 a. 28 32 b. 28 22 c. 22 28 d. 32 36 e. 22 26

32. 40 40 31 31 22 22 13 a. 13 4 b. 13 5 c. 4 13 d. 9 4 e. 4 4

38. 6 20 8 14 10 8 12 a. 14 10 b. 2 18 c. 4 12 d. 2 14 e. 14 14

33. 17 17 34 20 20 31 23 a. 26 23 b. 34 20 c. 23 33 d. 27 28 e. 23 28

39. 11 16 21 26 31 36 41 a. 47 52 b. 46 52 c. 45 49 d. 46 51 e. 46 52

34. 2 3 4 5 6 4 8 a. 9 10 b. 4 8 c. 10 4 d. 9 4 e. 8 9

40. 8 11 21 15 18 21 22 a. 25 18 b. 25 21 c. 25 29 d. 24 21 e. 22 26

35. 61 57 50 61 43 36 61 a. 29 61 b. 27 20 c. 31 61 d. 22 15 e. 29 22

5

GovernmentAdda.com – QUESTIONS –



Set 3

46. 14 14 26 26 38 38 50 a. 60 72 b. 50 62 c. 50 72 d. 62 62 e. 62 80

(Answers begin on page 102.)

This set will give you additional practice dealing with number series questions. 41. 44 41 38 35 32 29 26 a. 24 21 b. 22 19 c. 23 19 d. 29 32 e. 23 20

47. 8 12 9 13 10 14 11 a. 14 11 b. 15 12 c. 8 15 d. 15 19 e. 8 5

42. 6 10 14 18 22 26 30 a. 36 40 b. 33 37 c. 38 42 d. 34 36 e. 34 38

48. 4 7 26 10 13 20 16 a. 14 4 b. 14 17 c. 18 14 d. 19 13 e. 19 14

43. 34 30 26 22 18 14 10 a. 8 6 b. 6 4 c. 14 18 d. 6 2 e. 4 0

49. 3 8 10 15 17 22 24 a. 26 28 b. 29 34 c. 29 31 d. 26 31 e. 26 32

44. 2 44 4 41 6 38 8 a. 10 12 b. 35 32 c. 34 9 d. 35 10 e. 10 52

50. 17 14 14 11 11 8 8 a. 8 5 b. 5 2 c. 8 2 d. 5 5 e. 5 8

45. 32 29 26 23 20 17 14 a. 11 8 b. 12 8 c. 11 7 d. 32 29 e. 10 9

51. 13 29 15 26 17 23 19 a. 21 23 b. 20 21 c. 20 17 d. 25 27 e. 22 20

6

GovernmentAdda.com – QUESTIONS –

52. 16 26 56 36 46 68 56 a. 80 66 b. 64 82 c. 66 80 d. 78 68 e. 66 82

57. 11 14 14 17 17 20 20 a. 23 23 b. 23 26 c. 21 24 d. 24 24 e. 24 27

53. 7 9 66 12 14 66 17 a. 19 66 b. 66 19 c. 19 22 d. 20 66 e. 66 20

58. 17 32 19 29 21 26 23 a. 25 25 b. 20 22 c. 23 25 d. 25 22 e. 27 32

54. 3 5 35 10 12 35 17 a. 22 35 b. 35 19 c. 19 35 d. 19 24 e. 22 24

59. 10 34 12 31 14 28 16 a. 25 18 b. 30 13 c. 19 26 d. 18 20 e. 25 22

55. 36 31 29 24 22 17 15 a. 13 11 b. 10 5 c. 13 8 d. 12 7 e. 10 8

60. 32 31 32 29 32 27 32 a. 25 32 b. 31 32 c. 29 32 d. 25 30 e. 29 30

56. 42 40 38 35 33 31 28 a. 25 22 b. 26 23 c. 26 24 d. 25 23 e. 26 22

7

GovernmentAdda.com – QUESTIONS –



Set 4

65. Look at this series: 72, 76, 73, 77, 74, __, 75, . . . What number should fill the blank? a. 70 b. 71 c. 75 d. 78

(Answers begin on page 103.)

This set contains additional number series questions, some of which are in Roman numerals. These items differ from Sets 1, 2, and 3 because they ask you to find the number that fits somewhere into the middle of the series. Some of the items involve both numbers and letters; for these questions, look for a number series and a letter series. (For additional practice in working letter series questions, see Set 5.)

66. Look at this series: 70, 71, 76, __, 81, 86, 70, 91, . . . What number should fill the blank? a. 70 b. 71 c. 80 d. 96

61. Look at this series: 8, 43, 11, 41, __, 39, 17, . . . What number should fill in the blank? a. 8 b. 14 c. 43 d. 44

67. Look at this series: 664, 332, 340, 170, __, 89, . . . What number should fill the blank? a. 85 b. 97 c. 109 d. 178

62. Look at this series: 15, __, 27, 27, 39, 39, . . . What number should fill the blank? a. 51 b. 39 c. 23 d. 15

68. Look at this series: 0.15, 0.3, __, 1.2, 2.4, . . . What number should fill the blank? a. 4.8 b. 0.006 c. 0.6 d. 0.9

63. Look at this series: 83, 73, 93, 63, __, 93, 43, . . . What number should fill the blank? a. 33 b. 53 c. 73 d. 93

69. Look at this series: 19, 13, 1, __, 9, . . . What number should fill the blank? a. 23 b. 3 c. 6 d. 27

64. Look at this series: 4, 7, 25, 10, __, 20, 16, 19, . . . What number should fill the blank? a. 13 b. 15 c. 20 d. 28

70. Look at this series: U32, V29, __, X23, Y20, . . . What number should fill the blank? a. W26 b. W17 c. Z17 d. Z26

8

GovernmentAdda.com – QUESTIONS –

71. Look at this series: J14, L16, __, P20, R22, . . . What number should fill the blank? a. S24 b. N18 c. M18 d. T24

74. Look at this series: XXIV, XX, __, XII, VIII, . . . What number should fill the blank? a. XXII b. XIII c. XVI d. IV

72. Look at this series: F2, __, D8, C16, B32, . . . What number should fill the blank? a. A16 b. G4 c. E4 d. E3

75. Look at this series: VI, 10, V, 11, __, 12, III, . . . What number should fill the blank? a. II b. IV c. IX d. 14

73. Look at this series: V, VIII, XI, XIV, __, XX, . . . What number should fill the blank? a. IX b. XXIII c. XV d. XVII

9

GovernmentAdda.com – QUESTIONS –



Set 5

81. CMM EOO GQQ _____ KUU a. GRR b. GSS c. ISS d. ITT

(Answers begin on page 104.)

Another type of sequence question involves a series of letters in a pattern. Usually, these questions use the letters’ alphabetical order as a base. To make matters more complicated, sometimes subscript numbers will be thrown into the letter sequence. In these series, you will be looking at both the letter pattern and the number pattern. Some of these questions ask you to fill the blank in the middle of the series; others ask you to add to the end of the series.

82. QAR RAS SAT TAU _____ a. UAV b. UAT c. TAS d. TAT 83. DEF DEF2 DE2F2 _____ D2E2F3 a. DEF3 b. D3EF3 c. D2E3F d. D2E2F2

76. QPO NML KJI _____ EDC a. HGF b. CAB c. JKL d. GHI

84. SCD TEF UGH ____ WKL a. CMN b. UJI c. VIJ d. IJT

77. JAK KBL LCM MDN _____ a. OEP b. NEO c. MEN d. PFQ

85. FAG GAF HAI IAH ____ a. JAK b. HAL c. HAK d. JAI

78. B2CD _____ BCD4 B5CD BC6D a. B2C2D b. BC3D c. B2C3D d. BCD7

86. BCB DED FGF HIH ___ a. JKJ b. HJH c. IJI d. JHJ

79. ELFA GLHA ILJA _____ MLNA a. OLPA b. KLMA c. LLMA d. KLLA

87. ZA5 Y4B XC6 W3D _____ a. E7V b. V2E c. VE5 d. VE7

80. P5QR P4QS P3QT _____ PQV a. PQW b. PQV2 c. P2QU d. PQ3U 10

GovernmentAdda.com – QUESTIONS –



Set 6 (Answers begin on page 105.)

93.

This set contains sequence questions that use a series of nonverbal, nonnumber symbols. Look carefully at the sequence of symbols to find the pattern.

a.

b.

c.

d.

88. 94.

a.

b.

c.

d. a.

89.

b.

c.

d.

95.

a.

b.

c.

d. a.

b.

c.

d.

90. 96.

a.

b.

c.

d. a.

b.

c.

d.

91. 97.

a.

b.

c.

d. a.

b.

c.

d.

92. 98.

a.

b.

c.

a.

d.

11

b.

c.

d.

GovernmentAdda.com – QUESTIONS –

99.

a.

101.

b.

c.

d. a.

100.

a.

b.

c.

d.

12

b.

c.

d.

GovernmentAdda.com – QUESTIONS –



Set 7

106. Which word does NOT belong with the others? a. tulip b. rose c. bud d. daisy

(Answers begin on page 106.)

The next two sets contain verbal classification questions. For these questions, the important thing (as the name “verbal classification” indicates) is to classify the words in the four answer choices. Three of the words will be in the same classification; the remaining one will not be. Your answer will be the one word that does NOT belong in the same classification as the others.

107. Which word does NOT belong with the others? a. tire b. steering wheel c. engine d. car

102. Which word does NOT belong with the others? a. leopard b. cougar c. elephant d. lion

108. Which word does NOT belong with the others? a. parsley b. basil c. dill d. mayonnaise

103. Which word does NOT belong with the others? a. couch b. rug c. table d. chair

109. Which word does NOT belong with the others? a. branch b. dirt c. leaf d. root

104. Which word does NOT belong with the others? a. tape b. twine c. cord d. yarn

110. Which word does NOT belong with the others? a. unimportant b. trivial c. insignificant d. familiar

105. Which word does NOT belong with the others? a. guitar b. flute c. violin d. cello

111. Which word does NOT belong with the others? a. book b. index c. glossary d. chapter

13

GovernmentAdda.com – QUESTIONS –

112. Which word does NOT belong with the others? a. noun b. preposition c. punctuation d. adverb

116. Which word does NOT belong with the others? a. street b. freeway c. interstate d. expressway

113. Which word does NOT belong with the others? a. cornea b. retina c. pupil d. vision

117. Which word does NOT belong with the others? a. dodge b. flee c. duck d. avoid

114. Which word does NOT belong with the others? a. rye b. sourdough c. pumpernickel d. loaf

118. Which word does NOT belong with the others? a. heading b. body c. letter d. closing

115. Which word does NOT belong with the others? a. inch b. ounce c. centimeter d. yard

14

GovernmentAdda.com – QUESTIONS –



Set 8

124. Which word does NOT belong with the others? a. evaluate b. assess c. appraise d. instruct

(Answers begin on page 123.)

Here’s another set of classification questions. Remember, you are looking for the word that does NOT belong in the same group as the others. Sometimes, all four words seem to fit in the same group. If so, look more closely to further narrow your classification.

125. Which word does NOT belong with the others? a. eel b. lobster c. crab d. shrimp

119. Which word does NOT belong with the others? a. core b. seeds c. pulp d. slice

126. Which word does NOT belong with the others? a. scythe b. knife c. pliers d. saw

120. Which word does NOT belong with the others? a. unique b. beautiful c. rare d. exceptional

127. Which word does NOT belong with the others? a. two b. three c. six d. eight

121. Which word does NOT belong with the others? a. biology b. chemistry c. theology d. zoology

128. Which word does NOT belong with the others? a. peninsula b. island c. bay d. cape

122. Which word does NOT belong with the others? a. triangle b. circle c. oval d. sphere

129. Which word does NOT belong with the others? a. seat b. rung c. cushion d. leg

123. Which word does NOT belong with the others? a. excite b. flourish c. prosper d. thrive 15

GovernmentAdda.com – QUESTIONS –

130. Which word does NOT belong with the others? a. fair b. just c. equitable d. favorable

134. Which word does NOT belong with the others? a. acute b. right c. obtuse d. parallel

131. Which word does NOT belong with the others? a. defendant b. prosecutor c. trial d. judge

135. Which word does NOT belong with the others? a. wing b. fin c. beak d. rudder

132. Which word does NOT belong with the others? a. area b. variable c. circumference d. quadrilateral

136. Which word does NOT belong with the others? a. aorta b. heart c. liver d. stomach

133. Which word does NOT belong with the others? a. mayor b. lawyer c. governor d. senator

16

GovernmentAdda.com – QUESTIONS –



Set 9

141. election a. president b. voter c. November d. nation

(Answers begin on page 108.)

In the next three sets, you will be looking for the essential part of something. Each question has an underlined word followed by four answer choices. You will choose the word that is a necessary part of the underlined word. A good way to approach this type of question is to say the following sentence: “A ______ could not exist without ______.” Put the underlined word in the first blank. Try each of the answer choices in the second blank to see which choice is most logical.

142. diploma a. principal b. curriculum c. employment d. graduation 143. swimming a. pool b. bathing suit c. water d. life jacket

For questions 137 through 151, find the word that names a necessary part of the underlined word. 137. book a. fiction b. pages c. pictures d. learning

144. school a. student b. report card c. test d. learning

138. guitar a. band b. teacher c. songs d. strings

145. language a. tongue b. slang c. writing d. words

139. shoe a. sole b. leather c. laces d. walking

146. desert a. cactus b. arid c. oasis d. flat

140. respiration a. mouth b. circulation c. oxygen d. carbon monoxide

147. lightning a. electricity b. thunder c. brightness d. rain

17

GovernmentAdda.com – QUESTIONS –

148. monopoly a. corrupt b. exclusive c. rich d. gigantic

150. gala a. celebration b. tuxedo c. appetizer d. orator

149. harvest a. autumn b. stockpile c. tractor d. crop

151. pain a. cut b. burn c. nuisance d. hurt

18

GovernmentAdda.com – QUESTIONS –



Set 10

157. antique a. rarity b. artifact c. aged d. prehistoric

(Answers begin on page 109.)

Remember, you are looking for the essential part of something. If you had trouble with Set 9, go back through the items and study each answer explanation. Then work through this set of more difficult necessary part questions.

158. itinerary a. map b. route c. travel d. guidebook

For questions 152 through 166, find the word that names a necessary part of the underlined word. 152. infirmary a. surgery b. disease c. patient d. receptionist

159. orchestra a. violin b. stage c. musician d. soloist

153. facsimile a. picture b. image c. mimeograph d. copier

160. knowledge a. school b. teacher c. textbook d. learning

154. domicile a. tenant b. dwelling c. kitchen d. house

161. dimension a. compass b. ruler c. inch d. measure

155. culture a. civility b. education c. agriculture d. customs

162. sustenance a. nourishment b. water c. grains d. menu

156. bonus a. reward b. raise c. cash d. employer

163. ovation a. outburst b. bravo c. applause d. encore

19

GovernmentAdda.com – QUESTIONS –

166. purchase a. trade b. money c. bank d. acquisition

164. vertebrate a. backbone b. reptile c. mammal d. animal 165. provisions a. groceries b. supplies c. gear d. caterers

20

GovernmentAdda.com – QUESTIONS –



172. wedding a. love b. church c. ring d. marriage

Set 11 (Answers begin on page 111.)

Here is one more set of necessary part questions. This set is somewhat more difficult than the previous two sets, and it should give you practice in mastering this particular type of question. Remember: A good way to approach this type of question is to use the following sentence: “A ______ could not exist without ______.”

173. faculty a. buildings b. textbooks c. teachers d. meetings

For questions 167 through 181, find the word that names a necessary part of the underlined word. 167. dome a. rounded b. geodesic c. governmental d. coppery

174. cage a. enclosure b. prisoner c. animal d. zoo

168. recipe a. desserts b. directions c. cookbook d. utensils

175. directory a. telephone b. listing c. computer d. names

169. hurricane a. beach b. cyclone c. damage d. wind

176. contract a. agreement b. document c. written d. attorney

170. autograph a. athlete b. actor c. signature d. pen

177. saddle a. horse b. seat c. stirrups d. horn

171. town a. residents b. skyscrapers c. parks d. libraries

178. vibration a. motion b. electricity c. science d. sound

21

GovernmentAdda.com – QUESTIONS –

181. glacier a. mountain b. winter c. prehistory d. ice

179. cell a. chlorophyll b. nucleus c. nerve d. human 180. champion a. running b. swimming c. winning d. speaking

22

GovernmentAdda.com – QUESTIONS –



Set 12

185. Window is to pane as book is to a. novel. b. glass. c. cover. d. page.

(Answers begin on page 113.)

Here is the first of several sets of analogies. Analogies test your ability to see relationships between words, objects, or concepts. There are many different types of analogy relationships: use or function, part-to-whole, classification, proportion or degree, cause and effect, similarity or difference. In each of these verbal analogies, you will be given a set of two related words, followed by a third word and four answer choices. Of the four choices, you must identify the one that would best complete the second set so that it expresses the same relationship as the first set. A good way to figure out the relationship in a given question is to make up a sentence that describes the relationship between the first two words. Then, try to use the same sentence to find out which of the answer choices completes the same relationship with the third word.

186. Secretly is to openly as silently is to a. scarcely. b. impolitely. c. noisily. d. quietly. 187. Artist is to painting as senator is to a. attorney. b. law. c. politician. d. constituents. 188. Play is to actor as concert is to a. symphony. b. musician. c. piano. d. percussion.

182. Cup is to coffee as bowl is to a. dish. b. soup. c. spoon. d. food.

189. Careful is to cautious as boastful is to a. arrogant. b. humble. c. joyful. d. suspicious.

183. Exercise is to gym as eating is to a. food. b. dieting. c. fitness. d. restaurant.

190. Pride is to lion as school is to a. teacher. b. student. c. self-respect. d. fish.

184. Oar is to rowboat as foot is to a. running. b. sneaker. c. skateboard. d. jumping.

191. Guide is to direct as reduce is to a. decrease. b. maintain. c. increase. d. preserve.

23

GovernmentAdda.com – QUESTIONS –

192. Yard is to inch as quart is to a. gallon. b. ounce. c. milk. d. liquid.

197. Odometer is to mileage as compass is to a. speed. b. hiking. c. needle. d. direction.

193. Reptile is to lizard as flower is to a. petal. b. stem. c. daisy. d. alligator.

198. Optimist is to cheerful as pessimist is to a. gloomy. b. mean. c. petty. d. helpful.

194. Elated is to despondent as enlightened is to a. aware. b. ignorant. c. miserable. d. tolerant.

199. Sponge is to porous as rubber is to a. massive. b. solid. c. elastic. d. inflexible.

195. Marathon is to race as hibernation is to a. winter. b. bear. c. dream. d. sleep.

200. Candid is to indirect as honest is to a. frank. b. wicked. c. truthful. d. untruthful.

196. Embarrassed is to humiliated as frightened is to a. terrified. b. agitated. c. courageous. d. reckless.

201. Pen is to poet as needle is to a. thread. b. button. c. sewing. d. tailor.

24

GovernmentAdda.com – QUESTIONS –



203.

Set 13 (Answers begin on page 115.)

Now that you have some practice working basic analogies, try these picture analogies, which will give you practice with nonverbal reasoning. Solve these picture analogies in the same way you solved the word analogies. For each item, you will be presented with a set of two pictures that are related to each other in the same way. Along with this pair, you’ll be given a third picture and four answer choices, which are also pictures. Of the four choices, choose the picture that would go in the empty box so that the two bottom pictures are related in the same way as the top two are related.

a.

202.

b.

c.

d.

204.

a.

b.

c.

d. a.

25

b.

c.

d.

GovernmentAdda.com – QUESTIONS –

205.

207.

a.

b.

c.

d.

208. a.

b.

c.

d.

206.

a.

a.

b.

c.

d.

26

b.

c.

d.

GovernmentAdda.com – QUESTIONS –

209.

211.

a. a.

b.

c.

d.

b.

c.

d.

212.

210.

a.

a.

b.

c.

d.

27

b.

c.

d.

GovernmentAdda.com – QUESTIONS –

213.

215.

a.

b.

c.

d. a.

b.

c.

d.

214. 216.

a.

b.

c.

d.

a.

28

b.

c.

d.

GovernmentAdda.com – QUESTIONS –

217.

219.

a.

b.

c.

d. a.

b.

c.

d.

218. 220.

Soa p a.

b.

c.

d. a.

29

b.

c.

d.

GovernmentAdda.com – QUESTIONS –

T

221.

ONE CEN

LI

RTY BE

a.

b.

c.

d.

30

GovernmentAdda.com – QUESTIONS –



Set 14

223.

(Answers begin on page 116.)

Here are more picture analogies for you to master. There is essentially no difference between verbal and picture analogies, except that you have to take an extra first step by naming each picture. Make sure you understand the relationship between the first set of pictures before you attempt to choose an answer. Make up a sentence that describes this relationship. From the four answer choices, choose the picture that would go in the empty box so that the two bottom pictures are related in the same way as the top two are related.

222.

a.

b.

c.

d.

c.

d.

224.

a.

b.

c.

d.

a.

31

b.

GovernmentAdda.com – QUESTIONS –

225.

a.

227.

b.

c.

d. a.

b.

c.

d.

226. 228.

a.

b.

c.

d. a.

32

b.

c.

d.

GovernmentAdda.com – QUESTIONS –

229.

231.

a. a.

b.

c.

b.

c.

d.

d.

232. 230.

a.

b.

c.

d.

a.

33

b.

c.

d.

GovernmentAdda.com – QUESTIONS –

233.

a.

235.

b.

c.

d.

a.

234.

a.

b.

c.

d.

c.

d.

236.

b.

c.

a.

d.

34

b.

GovernmentAdda.com – QUESTIONS –

237.

239.

a. a.

b.

c.

b.

c.

d.

d. 240.

238.

a.

b.

c.

a.

d.

35

b.

c.

d.

GovernmentAdda.com – QUESTIONS –

241.

a.

b.

c.

d.

36

GovernmentAdda.com – QUESTIONS –



Set 15

246. meal

(Answers begin on page 117.)

shelter palace a. mansion b. hallway c. protection d. haven

This set contains another type of verbal analogy questions. In each, the words in the top row are related in some way. To help you discover this relationship, make up a sentence based on the top three words. The words in the bottom row are related in the same way as the words in the top row. For each item, find the word that completes the bottom row of words. 242. ant hamster a. spider b. mouse c. rodent d. cat 243. carpenter

fly

bee

squirrel

_______

pediatrician stethoscope a. thermometer b. baby c. doctor d. illness 244. table shirt a. sewing b. dress c. cotton d. tree 245. rule doze a. snore b. govern c. awaken d. hibernate

247. fence

_______

249. snow

oak

cloth

_______

warmth a. sand b. swim c. sunburn d. vacation 250. candle

command sleep

easel

textbook lesson plan a. artist b. teacher c. report card d. paint

nails

wood

wall

path alley a. ramp b. passageway c. airfield d. pedestrian 248. palette

saw

banquet

hut a. tent b. city c. dwelling d. house

dictate _______

37

feast _______

boundary _______

brush _______

mountain

ski

lake

_______

lamp

floodlight

cottage

_______

GovernmentAdda.com – QUESTIONS –

251. apples

fruit

novel book a. bookstore b. magazine c. vegetable d. shopping 252. tadpole lamb a. animal b. wool c. farm d. mammal 253. walk toss a. swerve b. hurl c. jump d. dance

254. honeybee

supermarket _______

frog

amphibian

sheep

_______

skip

run

pitch

_______

kangaroo a. mermaid b. possum c. grasshopper d. sprinter 255. daisy bungalow a. building b. cottage c. apartment d. city

38

angel

bat

rabbit

_______

flower

plant

house

_______

GovernmentAdda.com – QUESTIONS –



Set 16

260. SIAMESE : CAT a. type : breed b. dog : puppy c. mark : spot d. romaine : lettuce e. collar : leash

(Answers begin on page 118.)

The next two sets will give you more practice with analogies. Every one of the following questions consists of a related pair of words, followed by five pairs of words labeled a through e. Choose the pair that best represents a similar relationship to the one expressed in the original pair of words. Remember, the best way to approach an analogy question is to make up a sentence that describes the relationship between the first two words. Then, find the pair that has a similar relationship.

261. PEDAL : BICYCLE a. inch : yardstick b. walk : skip c. tire : automobile d. buckle : belt e. oar : canoe

256. PETAL : FLOWER a. salt : pepper b. tire : bicycle c. base : ball d. sandals : shoes e. puppy : dog

262. PULSATE : THROB a. walk : run b. tired : sleep c. examine : scrutinize d. ballet : dancer e. find : lose

257. BRISTLE : BRUSH a. arm : leg b. stage : curtain c. recline : chair d. key : piano e. art : sculpture

263. ELEPHANT : PACHYDERM a. mantis : rodent b. poodle : feline c. kangaroo : marsupial d. zebra : horse e. tuna : mollusk

258. FISH : SCHOOL a. wolf : pack b. elephant : jungle c. beagle : clan d. herd : peacock e. cow : farm

264. DEPRESSED : SAD a. neat : considerate b. towering : cringing c. rapid : plodding d. progressive : regressive e. exhausted : tired

259. ODOMETER : DISTANCE a. scale : weight b. length : width c. inch : foot d. mileage : speed e. area : size

265. PSYCHOLOGIST : NEUROSIS a. ophthalmologist : cataract b. dermatologist : fracture c. infant : pediatrician d. rash : orthopedist e. oncologist : measles

39

GovernmentAdda.com – QUESTIONS –

266. BINDING : BOOK a. criminal : gang b. display : museum c. artist : carpenter d. nail : hammer e. frame : picture

271. WAITRESS : RESTAURANT a. doctor : diagnosis b. actor : role c. driver : truck d. teacher : school e. author : book

267. EXPLORE : DISCOVER a. read : skim b. research : learn c. write : print d. think : relate e. sleep : wake

272. FINCH : BIRD a. frog : toad b. elephant : reptile c. Dalmatian : dog d. collie : marsupial e. ant : ladybug

268. COTTON : BALE a. butter : churn b. wine : ferment c. grain : shock d. curd : cheese e. beef : steak

273. RAIN : DRIZZLE a. swim : dive b. hop : shuffle c. juggle : bounce d. walk : run e. run : jog

269. DIVISION : SECTION a. layer : tier b. tether : bundle c. chapter : verse d. riser : stage e. dais : speaker

274. SKEIN : YARN a. squeeze : lemon b. fire : coal c. ream : paper d. tree : lumber e. plow : acre

270. PASTORAL : RURAL a. metropolitan : urban b. harvest : autumn c. agrarian : benevolent d. sleepy : nocturnal e. wild : agricultural

275. TAILOR : SUIT a. scheme : agent b. edit : manuscript c. revise : writer d. mention : opinion e. implode : building

40

GovernmentAdda.com – QUESTIONS –



Set 17

280. INTEREST : OBSESSION a. mood : feeling b. weeping : sadness c. dream : fantasy d. plan : negation e. highlight : indication

(Answers begin on page 119.)

Now try this last set of analogies, which are somewhat more difficult than the previous set. Remember that the first step in solving an analogy is to make up a sentence that describes the relationship between the first two words. Sometimes, your sentence may fit more than one answer choice. In these cases, be prepared to revise your original sentence. Each of the following questions consists of a related pair of words, followed by five pairs of words labeled a through e. Choose the pair that best represents a similar relationship to the one expressed in the original pair of words.

281. MONK : DEVOTION a. maniac : pacifism b. explorer : contentment c. visionary : complacency d. rover : wanderlust e. philistine : culture

276. CONDUCTOR : ORCHESTRA a. jockey : mount b. thrasher : hay c. driver : tractor d. skipper : crew e. painter : house

282. SLAPSTICK : LAUGHTER a. fallacy : dismay b. genre : mystery c. satire : anger d. mimicry : tears e. horror : fear

277. JAUNDICE : LIVER a. rash : skin b. dialysis : kidney c. smog : lung d. valentine : heart e. imagination : brain

283. VERVE : ENTHUSIASM a. loyalty : duplicity b. devotion : reverence c. intensity : color d. eminence : anonymity e. generosity : elation

278. COBBLER : SHOE a. jockey : horse b. contractor : building c. mason : stone d. cowboy : boot e. potter : paint

284. SOUND : CACOPHONY a. taste : style b. touch : massage c. smell : stench d. sight : panorama e. speech : oration

279. PHOBIC : FEARFUL a. finicky : thoughtful b. cautious : emotional c. envious : desiring d. shy : familiar e. asinine : silly

285. CONVICTION : INCARCERATION a. reduction : diminution b. induction : amelioration c. radicalization : estimation d. marginalization : intimidation e. proliferation : alliteration

41

GovernmentAdda.com – QUESTIONS –

286. DELTOID : MUSCLE a. radius : bone b. brain : nerve c. tissue : organ d. blood : vein e. scalpel : incision

292. DIRGE : FUNERAL a. chain : letter b. bell : church c. telephone : call d. jingle : commercial e. hymn : concerto

287. UMBRAGE : OFFENSE a. confusion : penance b. infinity : meaning c. decorum : decoration d. elation : jubilance e. outrage : consideration

293. FERAL : TAME a. rancid : rational b. repetitive : recurrent c. nettlesome : annoying d. repentant : honorable e. ephemeral : immortal

288. PROFESSOR : ERUDITE a. aviator : licensed b. inventor : imaginative c. procrastinator : conscientious d. overseer : wealthy e. moderator : vicious

294. SPY : CLANDESTINE a. accountant : meticulous b. furrier : rambunctious c. lawyer : ironic d. shepherd : garrulous e. astronaut : opulent

289. DEPENDABLE : CAPRICIOUS a. fallible : cantankerous b. erasable : obtuse c. malleable : limpid d. capable : inept e. incorrigible : guilty

295. DOMINANCE : HEGEMONY a. romance : sympathy b. furtherance : melancholy c. independence : autonomy d. tolerance : philanthropy e. recompense : hilarity

290. FROND : PALM a. quill : porcupine b. blade : evergreen c. scale : wallaby d. tusk : alligator e. blade : fern

296. AERIE : EAGLE a. capital : government b. bridge : architect c. unit : apartment d. kennel : veterinarian e. house : person

291. METAPHOR : SYMBOL a. pentameter : poem b. rhythm : melody c. nuance : song d. slang : usage e. analogy : comparison

42

GovernmentAdda.com – QUESTIONS –



Set 18

299. Here are some words translated from an artificial language. moolokarn means blue sky wilkospadi means bicycle race moolowilko means blue bicycle

(Answers begin on page 120.)

Now try some reasoning questions that ask you to translate English words into an artificial language. First, you will be given a list of three “nonsense” words and their English word meanings. The question(s) that follow will ask you to reverse the process and translate an English word into the artificial language. Your best approach to this type of question is to look for elements (parts) of the “nonsense” words that repeat. This is the best way to translate from the imaginary language to English. For example, if you know that linsmerk means oak tree and linsdennel means oak table, then you know that lins means oak. And, if lins means oak, merk must mean tree, and dennel must mean table. When you discover what a word element means in English, write it down. Then, look for the word elements that appear both on the list and in the answer choices.

Which word could mean “racecar”? a. wilkozwet b. spadiwilko c. moolobreil d. spadivolo 300. Here are some words translated from an artificial language. daftafoni means advisement imodafta means misadvise imolokti means misconduct Which word could mean “statement”? a. kratafoni b. kratadafta c. loktifoni d. daftaimo

297. Here are some words translated from an artificial language. granamelke means big tree pinimelke means little tree melkehoon means tree house

301. Here are some words translated from an artificial language. dionot means oak tree blyonot means oak leaf blycrin means maple leaf

Which word could mean “big house”? a. granahoon b. pinishur c. pinihoon d. melkegrana

Which word could mean “maple syrup”? a. blymuth b. hupponot c. patricrin d. crinweel

298. Here are some words translated from an artificial language. lelibroon means yellow hat plekafroti means flower garden frotimix means garden salad Which word could mean “yellow flower”? a. lelifroti b. lelipleka c. plekabroon d. frotibroon

43

GovernmentAdda.com – QUESTIONS –

305. Here are some words translated from an artificial language. tamceno means sky blue cenorax means blue cheese aplmitl means star bright

302. Here are some words translated from an artificial language. agnoscrenia means poisonous spider delanocrenia means poisonous snake agnosdeery means brown spider Which word could mean “black widow spider”? a. deeryclostagnos b. agnosdelano c. agnosvitriblunin d. trymuttiagnos

Which word could mean “bright sky”? a. cenotam b. mitltam c. raxmitl d. aplceno 306. Here are some words translated from an artificial language. gorblflur means fan belt pixngorbl means ceiling fan arthtusl means tile roof

303. Here are some words translated from an artificial language. myncabel means saddle horse conowir means trail ride cabelalma means horse blanket

Which word could mean “ceiling tile”? a. gorbltusl b. flurgorbl c. arthflur d. pixnarth

Which word could mean “horse ride”? a. cabelwir b. conocabel c. almamyn d. conoalma

307. Here are some words translated from an artificial language. hapllesh means cloudburst srenchoch means pinball resbosrench means ninepin

304. Here are some words translated from an artificial language. godabim means kidney stones romzbim means kidney beans romzbako means wax beans

Which word could mean “cloud nine”? a. leshsrench b. ochhapl c. haploch d. haplresbo

Which word could mean “wax statue”? a. godaromz b. lazbim c. wasibako d. romzpeo

44

GovernmentAdda.com – QUESTIONS –

308. Here are some words translated from an artificial language. migenlasan means cupboard lasanpoen means boardwalk cuopdansa means pullman Which word could mean “walkway”? a. poenmigen b. cuopeisel c. lasandansa d. poenforc

45

GovernmentAdda.com – QUESTIONS –



Set 19

312. Here are some words translated from an artificial language. krekinblaf means workforce dritakrekin means groundwork krekinalti means workplace

(Answers begin on page 121.)

Here is another set of questions that ask you to translate from an imaginary language into English. Remember, the best way to approach these questions is to translate each word element. When you discover what a word element means in English, write it down. Then, look for the word elements that appear both on the list and in the answer choices.

Which word could mean “someplace”? a. moropalti b. krekindrita c. altiblaf d. dritaalti

309. Here are some words translated from an artificial language. morpirquat means birdhouse beelmorpir means bluebird beelclak means bluebell

313. Here are some words translated from an artificial language. plekapaki means fruitcake pakishillen means cakewalk treftalan means buttercup

Which word could mean “houseguest”? a. morpirhunde b. beelmoki c. quathunde d. clakquat

Which word could mean “cupcake”? a. shillenalan b. treftpleka c. pakitreft d. alanpaki

310. Here are some words translated from an artificial language. slar means jump slary means jumping slarend means jumped

314. Here are some words translated from an artificial language. peslligen means basketball court ligenstrisi means courtroom oltaganti means placement test

Which word could mean “playing”? a. clargslarend b. clargy c. ellaclarg d. slarmont

Which word could mean “guest room”? a. peslstrisi b. vosefstrisi c. gantipesl d. oltastrisi

311. Here are some words translated from an artificial language. briftamint means militant uftonel means occupied uftonalene means occupation Which word could mean “occupant”? a. elbrifta b. uftonamint c. elamint d. briftalene 46

GovernmentAdda.com – QUESTIONS –

318. Here are some words translated from an artificial language. aptaose means first base eptaose means second base lartabuk means ballpark

315. Here are some words translated from an artificial language. jalkamofti means happy birthday moftihoze means birthday party mentogunn means goodness

Which word could mean “baseball”? a. buklarta b. oseepta c. bukose d. oselarta

Which word could mean “happiness”? a. jalkagunn b. mentohoze c. moftihoze d. hozemento

319. Here are some words translated from an artificial language. relftaga means carefree otaga means careful fertaga means careless

316. Here are some words translated from an artificial language. mallonpiml means blue light mallontifl means blueberry arpantifl means raspberry

Which word could mean “aftercare”? a. zentaga b. tagafer c. tagazen d. relffer

Which word could mean “lighthouse”? a. tiflmallon b. pimlarpan c. mallonarpan d. pimldoken 317. Here are some words translated from an artificial language. gemolinea means fair warning gerimitu means report card gilageri means weather report

320. Here are some words translated from an artificial language. malgauper means peach cobbler malgaport means peach juice moggagrop means apple jelly

Which word could mean “fair weather?” a. gemogila b. gerigeme c. gemomitu d. gerimita

Which word could mean “apple juice”? a. moggaport b. malgaauper c. gropport d. moggagrop

47

GovernmentAdda.com – QUESTIONS –



Set 20

322. It is appropriate to compensate someone if you have damaged his or her property in some way. This is called Restitution. Which situation below is the best example of Restitution? a. Jake borrows Leslie’s camera and the lens shatters when it falls on the ground because he fails to zipper the case. When Jake returns the camera, he tells Leslie that he will pay for the repair. b. Rebecca borrows her neighbor’s car, and when she returns it, the gas tank is practically empty. She apologizes profusely and tells her neighbor she will be more considerate the next time. c. Aaron asks Tom to check in on his apartment while he is out of town. When Tom arrives, he discovers that a pipe has burst and there is a considerable amount of water damage. He calls a plumber to repair the pipe. d. Lisa suspects that the pothole in her company’s parking lot caused her flat tire. She tells her boss that she thinks the company should pay for the repair.

(Answers begin on page 122.)

The questions in this set ask you to match definitions to particular situations. For each question, you will be given a definition and four possible answer choices. Read each definition and all four choices carefully, and find the answer that provides the best example of the given definition. Answer each question solely on the basis of the definition given.

321. Violating an Apartment Lease occurs when a tenant does something prohibited by the legally binding document that he or she has signed with a landlord. Which situation below is the best example of Violating an Apartment Lease? a. Tim has decided to move to another city, so he calls his landlord to tell him that he is not interested in renewing his lease when it expires next month. b. Valerie recently lost her job and, for the last three months, has neglected to pay her landlord the monthly rent they agreed upon in writing when she moved into her apartment eight months ago. c. Mark writes a letter to his landlord that lists numerous complaints about the apartment he has agreed to rent for two years. d. Leslie thinks that her landlord is neglecting the building in which she rents an apartment. She calls her attorney to ask for advice.

323. People speculate when they consider a situation and assume something to be true based on inconclusive evidence. Which situation below is the best example of Speculation? a. Francine decides that it would be appropriate to wear jeans to her new office on Friday after reading about “Casual Fridays” in her employee handbook. b. Mary spends thirty minutes sitting in traffic and wishes that she took the train instead of driving. c. After consulting several guidebooks and her travel agent, Jennifer feels confident that the hotel she has chosen is first-rate. d. When Emily opens the door in tears, Theo guesses that she’s had a death in her family.

48

GovernmentAdda.com – QUESTIONS –

326. Embellishing the Truth occurs when a person adds fictitious details or exaggerates facts or true stories. Which situation below is the best example of Embellishing the Truth? a. Isabel goes to the theater, and the next day, she tells her coworkers she thought the play was excellent. b. The realtor describes the house, which is eleven blocks away from the ocean, as prime waterfront property. c. During the job interview, Fred, who has been teaching elementary school for ten years, describes himself as a very experienced teacher. d. The basketball coach says it is likely that only the most talented players will get a college scholarship.

324. A Guarantee is a promise or assurance that attests to the quality of a product that is either (1) given in writing by the manufacturer or (2) given verbally by the person selling the product. Which situation below is the best example of a Guarantee? a. Melissa purchases a DVD player with the highest consumer ratings in its category. b. The salesperson advises Curt to be sure that he buys an air conditioner with a guarantee. c. The local auto body shop specializes in refurbishing and selling used cars. d. Lori buys a used digital camera from her coworker who says that she will refund Lori’s money if the camera’s performance is not of the highest quality. 325. Reentry occurs when a person leaves his or her social system for a period of time and then returns. Which situation below best describes Reentry? a. When he is offered a better paying position, Jacob leaves the restaurant he manages to manage a new restaurant on the other side of town. b. Catherine is spending her junior year of college studying abroad in France. c. Malcolm is readjusting to civilian life after two years of overseas military service. d. After several miserable months, Sharon decides that she can no longer share an apartment with her roommate Hilary.

327. Applying for Seasonal Employment occurs when a person requests to be considered for a job that is dependent on a particular season or time of year. Which situation below is the best example of Applying for Seasonal Employment? a. The ski instructors at Top of the Peak Ski School work from December through March. b. Matthew prefers jobs that allow him to work outdoors. c. Lucinda makes an appointment with the beach resort restaurant manager to interview for the summer waitressing position that was advertised in the newspaper. d. Doug’s ice cream shop stays open until 11 p.m. during the summer months.

49

GovernmentAdda.com – QUESTIONS –

330. In the Maple Hill school district, a Five-Day Suspension occurs when a student is not permitted to attend school for five days for (1) physically assaulting another student, a teacher, or a school employee or (2) willfully destructing or defacing school property. Which situation below is the best example of a Five-Day Suspension? a. Lillian gets caught cheating on a math test for the second time and is suspended from school. b. Marc is asked to leave the classroom due to his constant disruptions. c. Franny uses spray paint to write derogatory comments on the locker room wall and she is given a suspension. d. Ms. Farmer tells her class that students who fail the midterm exam will be expected to stay after school for tutoring help.

328. An Informal Gathering occurs when a group of people get together in a casual, relaxed manner. Which situation below is the best example of an Informal Gathering? a. The book club meets on the first Thursday evening of every month. b. After finding out about his promotion, Jeremy and a few coworkers decide to go out for a quick drink after work. c. Mary sends out 25 invitations for the bridal shower she is giving for her sister. d. Whenever she eats at the Mexican restaurant, Clara seems to run into Peter. 329. The rules of baseball state that a batter Legally Completes His Time at Bat when he is put out or becomes a base runner. Which situation below is the best example of a batter Legally Completing His Time at Bat? a. Jared’s blooper over the head of the shortstop puts him in scoring position. b. The umpire calls a strike, even though the last pitch was way outside. c. The pitcher throws his famous knuckleball, Joe swings and misses, and the umpire calls a strike. d. The count is two balls and two strikes as Mario waits for the next pitch.

331. A Tiebreaker is an additional contest or period of play designed to establish a winner among tied contestants. Which situation below is the best example of a Tiebreaker? a. At halftime, the score is tied at 28. b. Mary and Megan have each scored three goals in the game. c. The referee tosses a coin to decide which team will have possession of the ball first. d. The Sharks and the Bears each finished with 14 points, and they are now battling it out in a five-minute overtime.

50

GovernmentAdda.com – QUESTIONS –

334. Posthumous Publication occurs when a book is published after the author’s death. Which situation below is the best example of Posthumous Publication? a. Richard’s illness took his life before he was able to enjoy the amazing early reviews of his novel. b. Melissa’s publisher cancels her book contract after she fails to deliver the manuscript on time. c. Clarence never thought he’d live to see the third book in his trilogy published. d. Elizabeth is honored with a prestigious literary award for her writing career and her daughter accepts the award on behalf of her deceased mother.

332. Establishing a Power of Attorney occurs when a legal document is created that gives one individual the authority to act for another. Which situation below is the best example of Establishing a Power of Attorney? a. Louise is selling her house and she hires a lawyer to review the contract. b. Simone’s mother can no longer get to the bank to cash her checks and make deposits, so she has taken legal steps to enable Simone to do these things for her. c. Jack’s father is elderly and Jack thinks he is no longer able to make decisions for himself. d. At her daughter’s urging, Mrs. Lenox opens up a retirement account with the local bank. 333. Erratic Behavior occurs when an individual acts in a manner that lacks consistency, regularity, and uniformity. Which situation below is the best example of Erratic Behavior? a. Julia cannot contain her anger whenever the subject of local politics is discussed. b. Martin has just been told that he is being laid off. Before leaving his supervisor’s office, he punches a hole in the door. c. Rhonda has visited the dealership several times, but she still cannot decide which car to buy. d. In the past month, Jeffrey, who has been a model employee for three years, has repeatedly called in sick, forgotten important meetings, and been verbally abusive to colleagues.

51

GovernmentAdda.com – QUESTIONS –



Set 21

consists of some of the wealthiest and most powerful citizens in the county. b. Lindy recently graduated at the top of her class from one of the best dental hygiene programs in the state. Prior to becoming a dental hygienist, Lindy spent two years working in a day care center. c. James has worked as a dental hygienist for three years in a public health clinic. He is very interested in securing a position in a private dental office. d. Kathy is an experienced and highly recommended dental hygienist who is also finishing up a degree in early childhood education, which she hopes will get her a job as a preschool teacher. She is eager to find a job in a pediatric practice, since she has always wanted to work with children.

(Answers begin on page 124.)

This set contains additional situations. Each question presents a situation and asks you to make a judgment regarding that particular circumstance. Answer each one solely on the basis of the information given. 335. The school principal has received complaints from parents about bullying in the school yard during recess. He wants to investigate and end this situation as soon as possible, so he has asked the recess aides to watch closely. Which situation should the recess aides report to the principal? a. A girl is sitting glumly on a bench reading a book and not interacting with her peers. b. Four girls are surrounding another girl and seem to have possession of her backpack. c. Two boys are playing a one-on-one game of basketball and are arguing over the last basket scored. d. Three boys are huddled over a handheld video game, which isn’t supposed to be on school grounds.

337. Mrs. Jansen recently moved to Arizona. She wants to fill her new backyard with flowering plants. Although she is an experienced gardener, she isn’t very well-versed in what plants will do well in the Arizona climate. Also, there is a big tree in her backyard making for shady conditions and she isn’t sure what plants will thrive without much direct sunlight. Her favorite gardening catalog offers several backyard seed packages. Which one should Mrs. Jansen choose? a. The Rainbow Collection is ideal for Northeast gardens. It includes a variety of colorful perennials that thrive in cool, moist conditions. b. The Greenhouse Collection will blossom year after year if planted in brightly lit locations and watered regularly. c. The Treehouse Collection will provide lush green plants with delicate colorful flowers

336. Dr. Miller has a busy pediatric dentistry practice and she needs a skilled, reliable hygienist to keep things running smoothly. The last two people she hired were recommended by top dentists in the area, but they each lasted less than one month. She is now in desperate need of a hygienist who can competently handle the specific challenges of her practice. Which one of the following candidates should Dr. Miller consider most seriously? a. Marilyn has been a hygienist for fifteen years, and her current employer, who is about to retire, says she is the best in the business. The clientele she has worked with

52

GovernmentAdda.com – QUESTIONS –

less professionals who use the train to commute to their offices each day b. a little shop three blocks away from the town’s main street, located across the street from an elementary school and next door to an ice cream store c. a stand-alone storefront on a quiet residential street ten blocks away from the town’s center d. a storefront in a small strip mall located on the outskirts of town that is also occupied by a pharmacy and a dry cleaner

that thrive in shady and partially shady locations. d. The Oasis Collection includes a variety of perennials that thrive in dry climates and bright sunlight. 338. Eileen is planning a special birthday dinner for her husband’s 35th birthday. She wants the evening to be memorable, but her husband is a simple man who would rather be in jeans at a baseball game than in a suit at a fancy restaurant. Which restaurant below should Eileen choose? a. Alfredo’s offers fine Italian cuisine and an elegant Tuscan décor. Patrons will feel as though they’ve spent the evening in a luxurious Italian villa. b. Pancho’s Mexican Buffet is an all-you-caneat family style smorgasbord with the best tacos in town. c. The Parisian Bistro is a four-star French restaurant where guests are treated like royalty. Chef Dilbert Olay is famous for his beef bourguignon. d. Marty’s serves delicious, hearty meals in a charming setting reminiscent of a baseball clubhouse in honor of the owner, Marty Lester, a former major league baseball allstar.

340. Rita, an accomplished pastry chef who is well known for her artistic and exquisite wedding cakes, opened a bakery one year ago and is surprised that business has been so slow. A consultant she hired to conduct market research has reported that the local population doesn’t think of her shop as one they would visit on a daily basis but rather a place they’d visit if they were celebrating a special occasion. Which of the following strategies should Rita employ to increase her daily business? a. making coupons available that entitle the coupon holder to receive a 25% discount on wedding, anniversary, or birthday cakes b. exhibiting at the next Bridal Expo and having pieces of one of her wedding cakes available for tasting c. placing a series of ads in the local newspaper that advertise the wide array of breads, muffins, and cookies offered at her shop d. moving the bakery to the other side of town

339. Mark is working with a realtor to find a location for the toy store he plans to open in his town. He is looking for a place that is either in, or not too far from, the center of town and one that would attract the right kind of foot traffic. Which of the following locations should Mark’s realtor call to his attention? a. a storefront in a new high-rise building near the train station in the center of town whose occupants are mainly young, child-

53

GovernmentAdda.com – QUESTIONS –

2. Take the employee back to the human resources office and have him complete a general information questionnaire, a healthcare insurance form, and a tax form. 3. Take a photograph of the employee for his identification card. 4. Issue the employee a temporary identification card. 5. Walk the employee to the department in which he will be working and introduce him to his colleagues. 6. Bring the employee to his office or cubicle.

341. Mrs. Carson took a taxi to meet her three friends for lunch. They were waiting for her outside the restaurant when she pulled up in the car. She was so excited to see her friends that she left her tote bag in the taxi. As the taxi pulled away, she and her friends took notice of the license plate number so they would be able to identify the car when they called the taxi company. The four license plate numbers below represent what each of the four women thinks she saw. Which one is most likely the license plate number of the taxi? a. JXK 12L b. JYK 12L c. JXK 12I d. JXX 12L

343. It is Kate Milford’s first day of work as a paralegal at the law firm of Jasper, Jenkins & Mead. Taylor Franklin, the human resources manager, greets Kate in the reception area and gets her a cup of tea as they walk back to the human resources office. Taylor asks Kate to sit at a table and fill out three forms. While Kate completes the forms, Taylor checks her messages and asks her secretary to confirm a meeting she has for later that morning. Taylor then takes a photograph of Kate that will be used on her company identification card. As Taylor walks Kate over to the paralegal department, she tells her that the identification card should be ready in a couple of days. Taylor introduces Kate to her new colleagues, who all greet her quite warmly, and then shows her to her new cubicle, jots down her phone extension, and says that Kate should call her if she has any questions at all. Taylor Franklin’s actions were a. proper, because she is the human resources manager and knows how to greet a new employee. b. improper, because she did not spend enough time making sure that Kate was comfortable. c. proper, because she told Kate that she should feel free to call her if she had questions. d. improper, because she did not issue a temporary identification card.

342. Zachary has invited his three buddies over to watch the basketball game on his wide-screen television. They are all hungry, but no one wants to leave to get food. Just as they are arguing about who should make the food run, a commercial comes on for a local pizzeria that delivers. The phone number flashes on the screen briefly and they all try to remember it. By the time Zachary grabs a pen and paper, each of them recollects a different number. Which of the numbers is most likely the telephone number of the pizzeria? a. 995-9266 b. 995-9336 c. 995-9268 d. 995-8266 Answer question 343 solely on the basis of the following information. When a new employee is hired at the law firm, a human resources representative should take the following steps on the employee’s first day of work. 1. Greet the employee in the reception area and offer him coffee or tea. 54

GovernmentAdda.com – QUESTIONS –

344. Marcus and Cynthia Howard arrive at Smithfield Realty for their appointment with realtor Patricia Russo. Ms. Russo leads the couple to a comfortable sofa in her office and gets them both a cup of coffee. Ms. Russo asks Marcus and Cynthia what kind of house they are looking for and it becomes clear that they have very particular ideas. Most importantly, they are looking for a house that is in walking distance of the train station. They also want a newer house, preferably one built after 1970. They must have four bedrooms and central air conditioning. A finished basement would be a welcome bonus. Ms. Russo discusses price range with her new clients, and before the discussion is finished, they hand her a letter from their mortgage company that indicates that they have been preapproved for a mortgage. Together, the three of them browse the listings on Ms. Russo’s computer screen and information is printed out for four houses that the couple would like to see. Ms. Russo determines that Marcus and Cynthia are free for another few hours, so the three of them head to her car to begin looking at potential new homes. Based on the company guidelines, the actions taken by Ms. Russo were a. improper, because she was only able to find four houses that Marcus and Cynthia wanted to see. b. proper, because she obtained all the necessary information from the clients. c. improper, because she failed to get any details about the client’s current living circumstances. d. proper, because she didn’t try to persuade the clients to consider houses that didn’t meet all of their criteria.

Answer question 344 solely on the basis of the following information. When a client comes in looking for a new home, the real estate agency requires its realtors to follow some specific guidelines during the first meeting. The realtor is expected to do the following. 1. Be sure the client is comfortably seated and has been offered a drink. 2. Get background information on the client’s current living circumstances. 3. Ask the client what qualities she is looking for in a house. 4. Discuss the price range that the client has in mind and determine whether or not she has been preapproved for a mortgage. 5. With the computer screen facing the client, browse the current house listings and print out information for any of the houses that the client would like to see in person. 6. Ask the client if she is available to look at some of the houses immediately, and if not, make an appointment to show her the houses as soon as possible.

55

GovernmentAdda.com – QUESTIONS –

346. The neighborhood block association has received many complaints about people knocking on doors and soliciting money for an unknown charity organization even though door-to-door solicitation is prohibited by local laws. Three residents have provided descriptions of individuals who have come to their door asking for money.

345. The film director wants an actress for the lead role of Lucy who perfectly fits the description that appears in the original screenplay. He is not willing to consider actresses who do not resemble the character as she is described in the screenplay, no matter how talented they are. The screenplay describes Lucy as an average-sized, fortysomething redhead, with deep brown eyes, very fair skin, and a brilliant smile. The casting agent has four actresses in mind.

Solicitor #1 is a white male, 20–25 years old, 5'9", 145 pounds, with very short brown hair. He was wearing a dark blue suit and carrying a brown leather briefcase.

Actress #1 is a stunning red-haired beauty who is 5'9" and in her mid-twenties. Her eyes are brown and she has an olive complexion.

Solicitor #2 is a white male, 25–30 years old, 6'2", 200 pounds, with a shaved-head. He was wearing a red T-shirt and jeans.

Actress #2 has red hair, big brown eyes, and a fair complexion. She is in her mid-forties and is 5'5".

Solicitor #3 is a white male, approximately 23 years old, 5'10", slight build, with short brown hair. He was wearing a blue suit.

Actress #3 is 5'4" and of medium build. She has red hair, brown eyes, and is in her early forties.

Three days after the block association meeting, a resident noticed a man knocking on doors in the neighborhood and phoned the police to report the illegal activity. This solicitor was described as follows:

Actress #4 is a blue-eyed redhead in her early thirties. She’s of very slight build and stands at 5'.

Solicitor #4 is a white male, 22 years old, 140 pounds, about 5'10", with short brown hair. He was carrying a briefcase and wearing a dark suit.

Which two actresses should the casting agent send to meet the director? a. 1, 2 b. 2, 3 c. 1, 4 d. 2, 4

Based on this description, which of the three solicitations was also likely carried out by Solicitor #4? a. #1, #2, and #3 b. #1, but not #2 and #3 c. #1 and #3, but not #2 d. #1 and #2, but not #3

56

GovernmentAdda.com – QUESTIONS –



Set 22

349. The Pacific yew is an evergreen tree that grows in the Pacific Northwest. The Pacific yew has a fleshy, poisonous fruit. Recently, taxol, a substance found in the bark of the Pacific yew, was discovered to be a promising new anticancer drug. a. Taxol is poisonous when taken by healthy people. b. Taxol has cured people from various diseases. c. People should not eat the fruit of the Pacific yew. d. The Pacific yew was considered worthless until taxol was discovered.

(Answers begin on page 125.)

Here’s another type of verbal reasoning question. For each item in this set, you will be given a short, informational paragraph and four answer choices. Look for the statement that must be true according to the given information. The best way to approach this problem is to read the answer choices in turn, going back each time to look for that exact information in the short passage. For questions 347 through 357, find the statement that must be true according to the given information.

350. On weekends, Mr. Sanchez spends many hours working in his vegetable and flower gardens. Mrs. Sanchez spends her free time reading and listening to classical music. Both Mr. Sanchez and Mrs. Sanchez like to cook. a. Mr. Sanchez enjoys planting and growing vegetables. b. Mr. Sanchez does not like classical music. c. Mrs. Sanchez cooks the vegetables that Mr. Sanchez grows. d. Mrs. Sanchez enjoys reading nineteenthcentury novels.

347. Erin is twelve years old. For three years, she has been asking her parents for a dog. Her parents have told her that they believe a dog would not be happy in an apartment, but they have given her permission to have a bird. Erin has not yet decided what kind of bird she would like to have. a. Erin’s parents like birds better than they like dogs. b. Erin does not like birds. c. Erin and her parents live in an apartment. d. Erin and her parents would like to move. 348. Last summer, Mike spent two weeks at a summer camp. There, he went hiking, swimming, and canoeing. This summer, Mike looks forward to attending a two-week music camp, where he hopes to sing, dance, and learn to play the guitar. a. Mike’s parents want him to learn to play the guitar. b. Mike prefers music to outdoor activities. c. Mike goes to some type of camp every summer. d. Mike likes to sing and dance.

57

GovernmentAdda.com – QUESTIONS –

353. Seahorse populations have declined everywhere that seahorses are fished. During the past five years, seahorse populations have decreased by 50%. Last year, biologists met to discuss what might be done to reverse this trend. a. Seahorses are likely to become extinct within five years. b. One way to increase seahorse populations is to ban the fishing of seahorses. c. Biologists from all over the world are working to save the seahorses. d. Seahorse fishermen have spoken out against the biologists.

351. Tim’s commute never bothered him because there were always seats available on the train and he was able to spend his 40 minutes comfortably reading the newspaper or catching up on paperwork. Ever since the train schedule changed, the train has been extremely crowded, and by the time the doors open at his station, there isn’t a seat to be found. a. Tim would be better off taking the bus to work. b. Tim’s commute is less comfortable since the train schedule changed. c. Many commuters will complain about the new train schedule. d. Tim will likely look for a new job closer to home.

354. Vincent has a paper route. Each morning, he delivers 37 newspapers to customers in his neighborhood. It takes Vincent 50 minutes to deliver all the papers. If Vincent is sick or has other plans, his friend Thomas, who lives on the same street, will sometimes deliver the papers for him. a. Vincent and Thomas live in the same neighborhood. b. It takes Thomas more than 50 minutes to deliver the papers. c. It is dark outside when Vincent begins his deliveries. d. Thomas would like to have his own paper route.

352. When they heard news of the hurricane, Maya and Julian decided to change their vacation plans. Instead of traveling to the island beach resort, they booked a room at a fancy new spa in the mountains. Their plans were a bit more expensive, but they’d heard wonderful things about the spa and they were relieved to find availability on such short notice. a. Maya and Julian take beach vacations every year. b. The spa is overpriced. c. It is usually necessary to book at least six months in advance at the spa. d. Maya and Julian decided to change their vacation plans because of the hurricane.

355. Georgia is older than her cousin Marsha. Marsha’s brother Bart is older than Georgia. When Marsha and Bart are visiting with Georgia, all three like to play a game of Monopoly. Marsha wins more often than Georgia does. a. When he plays Monopoly with Marsha and Georgia, Bart often loses. b. Of the three, Georgia is the oldest. c. Georgia hates to lose at Monopoly. d. Of the three, Marsha is the youngest.

58

GovernmentAdda.com – QUESTIONS –

357. Sara lives in a large city on the East Coast. Her younger cousin Marlee lives in the Midwest in a small town with fewer than 1,000 residents. Marlee has visited Sara several times during the past five years. In the same period of time, Sara has visited Marlee only once. a. Marlee likes Sara better than Sara likes Marlee. b. Sara thinks small towns are boring. c. Sara is older than Marlee. d. Marlee wants to move to the East Coast.

356. Ten new television shows appeared during the month of September. Five of the shows were sitcoms, three were hour-long dramas, and two were news-magazine shows. By January, only seven of these new shows were still on the air. Five of the shows that remained were sitcoms. a. Only one of the news-magazine shows remained on the air. b. Only one of the hour-long dramas remained on the air. c. At least one of the shows that was cancelled was an hour-long drama. d. Television viewers prefer sitcoms over hour-long dramas.

59

GovernmentAdda.com – QUESTIONS –



Set 23

360. All the tulips in Zoe’s garden are white.

(Answers begin on page 126.)

All the pansies in Zoe’s garden are yellow. The next three sets contain short logic problems. Each problem consists of three statements. Based on the first two statements, the third statement may be true, false, or uncertain. Logic problems may appear daunting at first. However, solving these problems can be done in the most straightforward way. Simply translate the abstract relationships in the questions into real-world relationships, so you can see the facts more clearly. For example, if the problem is comparing the ages of three people, make a chart and list the names of the people and their possible ages according to the information given. Or, create a diagram using symbols to represent phrases like “older than” or “greater than.”

All the flowers in Zoe’s garden are either white or yellow. If the first two statements are true, the third statement is a. true. b. false. c. uncertain. 361. Blueberries cost more than strawberries. Blueberries cost lest than raspberries. Raspberries cost more than both strawberries and blueberries. If the first two statements are true, the third statement is a. true. b. false. c. uncertain.

358. Tanya is older than Eric. Cliff is older than Tanya. Eric is older than Cliff. If the first two statements are true, the third statement is a. true. b. false. c. uncertain.

362. All the offices on the 9th floor have wall-towall carpeting. No wall-to-wall carpeting is pink. None of the offices on the 9th floor has pink wall-to-wall carpeting.

359. During the past year, Josh saw more movies than Stephen.

If the first two statements are true, the third statement is a. true. b. false. c. uncertain.

Stephen saw fewer movies than Darren. Darren saw more movies than Josh. If the first two statements are true, the third statement is a. true. b. false. c. uncertain.

363. Class A has a higher enrollment than Class B. Class C has a lower enrollment than Class B. Class A has a lower enrollment than Class C. If the first two statements are true, the third statement is a. true. b. false. c. uncertain.

60

GovernmentAdda.com – QUESTIONS –

367. The Kingston Mall has more stores than the Galleria.

364. Rover weighs less than Fido. Rover weighs more than Boomer.

The Four Corners Mall has fewer stores than the Galleria.

Of the three dogs, Boomer weighs the least. If the first two statements are true, the third statement is a. true. b. false. c. uncertain.

The Kingston Mall has more stores than the Four Corners Mall. If the first two statements are true, the third statement is a. true. b. false. c. uncertain.

365. All the trees in the park are flowering trees. Some of the trees in the park are dogwoods. All dogwoods are flowering trees.

368. Mara runs faster than Gail.

If the first two statements are true, the third statement is a. true. b. false. c. uncertain.

Lily runs faster than Mara. Gail runs faster than Lily. If the first two statements are true, the third statement is a. true. b. false. c. uncertain.

366. Apartments in the Riverdale Manor cost less than apartments in The Gaslight Commons. Apartments in the Livingston Gate cost more than apartments in the The Gaslight Commons. Of the three apartment buildings, the Livingston Gate costs the most. If the first two statements are true, the third statement is a. true. b. false. c. uncertain.

61

GovernmentAdda.com – QUESTIONS –



Set 24

371. The temperature on Monday was lower than on Tuesday.

(Answers begin on page 127.)

Some of the logic questions in this set ask you to determine the direction of a particular place in relation to other places. For these problems, instead of making a chart or grid, draw a very simple map and label North, South, East, and West to help you see where the places are located in relation to each other.

The temperature on Wednesday was lower than on Tuesday. The temperature on Monday was higher than on Wednesday. If the first two statements are true, the third statement is a. true. b. false. c. uncertain.

369. Oat cereal has more fiber than corn cereal but less fiber than bran cereal. Corn cereal has more fiber than rice cereal but less fiber than wheat cereal.

372. Spot is bigger than King and smaller than Sugar.

Of the three kinds of cereal, rice cereal has the least amount of fiber.

Ralph is smaller than Sugar and bigger than Spot.

If the first two statements are true, the third statement is a. true. b. false. c. uncertain.

King is bigger than Ralph. If the first two statements are true, the third statement is a. true. b. false. c. uncertain.

370. On the day the Barton triplets are born, Jenna weighs more than Jason. Jason weighs less than Jasmine.

373. A fruit basket contains more apples than lemons.

Of the three babies, Jasmine weighs the most. If the first two statements are true, the third statement is a. true. b. false. c. uncertain.

There are more lemons in the basket than there are oranges. The basket contains more apples than oranges. If the first two statements are true, the third statement is a. true. b. false. c. uncertain.

62

GovernmentAdda.com – QUESTIONS –

377. Battery X lasts longer than Battery Y.

374. The Shop and Save Grocery is south of Greenwood Pharmacy.

Battery Y doesn’t last as long as Battery Z.

Rebecca’s house is northeast of Greenwood Pharmacy.

Battery Z lasts longer than Battery X. If the first two statements are true, the third statement is a. true. b. false. c. uncertain.

Rebecca’s house is west of the Shop and Save Grocery. If the first two statements are true, the third statement is a. true. b. false. c. uncertain.

378. Martina is sitting in the desk behind Jerome. Jerome is sitting in the desk behind Bryant. Bryant is sitting in the desk behind Martina.

375. Joe is younger than Kathy.

If the first two statements are true, the third statement is a. true. b. false. c. uncertain.

Mark was born after Joe. Kathy is older than Mark. If the first two statements are true, the third statement is a. true. b. false. c. uncertain.

379. Middletown is north of Centerville. Centerville is east of Penfield. Penfield is northwest of Middletown.

376. All spotted Gangles have long tails.

If the first two statements are true, the third statement is a. true. b. false. c. uncertain.

Short-haired Gangles always have short tails. Long-tailed Gangles never have short hair. If the first two statements are true, the third statement is a. true. b. false. c. uncertain.

63

GovernmentAdda.com – QUESTIONS –



Set 25

383. Tom puts on his socks before he puts on his shoes.

(Answers begin on page 128.)

Here’s your last set of “true-false-uncertain” problems. Remember, the best way to answer questions like this is usually to draw a quick diagram or take notes.

He puts on his shirt before he puts on his jacket. Tom puts on his shoes before he puts on his shirt.

380. Taking the train across town is quicker than taking the bus.

If the first two statements are true, the third statement is a. true. b. false. c. uncertain.

Taking the bus across town is slower than driving a car. Taking the train across town is quicker than driving a car.

384. Three pencils cost the same as two erasers.

If the first two statements are true, the third statement is a. true. b. false. c. uncertain.

Four erasers cost the same as one ruler. Pencils are more expensive than rulers. If the first two statements are true, the third statement is a. true. b. false. c. uncertain.

381. All Lamels are Signots with buttons. No yellow Signots have buttons. No Lamels are yellow.

385. A jar of jelly beans contains more red beans than green.

If the first two statements are true, the third statement is a. true. b. false. c. uncertain.

There are more yellow beans than red. The jar contains fewer yellow jelly beans than green ones. If the first two statements are true, the third statement is a. true. b. false. c. uncertain.

382. The hotel is two blocks east of the drugstore. The market is one block west of the hotel. The drugstore is west of the market. If the first two statements are true, the third statement is a. true. b. false. c. uncertain.

64

GovernmentAdda.com – QUESTIONS –

388. At a parking lot, a sedan is parked to the right of a pickup and to the left of a sport utility vehicle.

386. Cloudy days tend to be more windy than sunny days. Foggy days tend to be less windy than cloudy days.

A minivan is parked to the left of the pickup.

Sunny days tend to be less windy than foggy days.

The minivan is parked between the pickup and the sedan.

If the first two statements are true, the third statement is a. true. b. false. c. uncertain.

If the first two statements are true, the third statement is a. true. b. false. c. uncertain. 389. A toothpick is useful.

387. The bookstore has a better selection of postcards than the newsstand does.

Useful things are valuable.

The selection of postcards at the drugstore is better than at the bookstore.

A toothpick is valuable. If the first two statements are true, the third statement is a. true. b. false. c. uncertain.

The drugstore has a better selection of postcards than the bookstore or the newsstand. If the first two statements are true, the third statement is a. true. b. false. c. uncertain.

65

GovernmentAdda.com – QUESTIONS –



Set 26

392. Fact 1: All chickens are birds. Fact 2: Some chickens are hens. Fact 3: Female birds lay eggs.

(Answers begin on page 129.)

The logic problems in this set present you with three true statements: Fact 1, Fact 2, and Fact 3. Then, you are given three more statements (labeled I, II, and III), and you must determine which of these, if any, is also a fact. One or two of the statements could be true; all of the statements could be true; or none of the statements could be true. Choose your answer based solely on the information given in the first three facts.

If the first three statements are facts, which of the following statements must also be a fact? I. All birds lay eggs. II. Hens are birds. III. Some chickens are not hens. a. II only b. II and III only c. I, II, and III d. None of the statements is a known fact.

390. Fact 1: Jessica has four children. Fact 2: Two of the children have blue eyes and two of the children have brown eyes. Fact 3: Half of the children are girls.

393. Fact 1: Most stuffed toys are stuffed with beans. Fact 2: There are stuffed bears and stuffed tigers. Fact 3: Some chairs are stuffed with beans.

If the first three statements are facts, which of the following statements must also be a fact? I. At least one girl has blue eyes. II. Two of the children are boys. III. The boys have brown eyes. a. II only b. I and III only c. II and III only d. None of the statements is a known fact.

If the first three statements are facts, which of the following statements must also be a fact? I. Only children’s chairs are stuffed with beans. II. All stuffed tigers are stuffed with beans. III. Stuffed monkeys are not stuffed with beans. a. I only b. II only c. II and III only d. None of the statements is a known fact.

391. Fact 1: All hats have brims. Fact 2: There are black hats and blue hats. Fact 3: Baseball caps are hats. If the first three statements are facts, which of the following statements must also be a fact? I. All caps have brims. II. Some baseball caps are blue. III. Baseball caps have no brims. a. I only b. II only c. I, II, and III d. None of the statements is a known fact.

66

GovernmentAdda.com – QUESTIONS –

397. Fact 1: All drink mixes are beverages. Fact 2: All beverages are drinkable. Fact 3: Some beverages are red.

394. Fact 1: Pictures can tell a story. Fact 2: All storybooks have pictures. Fact 3: Some storybooks have words.

If the first three statements are facts, which of the following statements must also be a fact? I. Some drink mixes are red. II. All beverages are drink mixes. III. All red drink mixes are drinkable. a. I only b. II only c. I and III d. None of the statements is a known fact.

If the first three statements are facts, which of the following statements must also be a fact? I. Pictures can tell a story better than words can. II. The stories in storybooks are very simple. III. Some storybooks have both words and pictures. a. I only b. II only c. III only d. None of the statements is a known fact.

398. Fact 1: Eyeglass frames cost between $35 and $350. Fact 2: Some eyeglass frames are made of titanium. Fact 3: Some eyeglass frames are made of plastic.

395. Fact 1: Robert has four vehicles. Fact 2: Two of the vehicles are red. Fact 3: One of the vehicles is a minivan. If the first three statements are facts, which of the following statements must also be a fact? I. Robert has a red minivan. II. Robert has three cars. III. Robert’s favorite color is red. a. I only b. II only c. II and III only d. None of the statements is a known fact.

If the first three statements are facts, which of the following statements must also be a fact? I. Titanium eyeglass frames cost more than plastic frames. II. Expensive eyeglass frames last longer than cheap frames. III. Only a few eyeglass frames cost less than $35. a. II only b. I and II only c. II and III only d. None of the statements is a known fact.

396. Fact 1: Islands are surrounded by water. Fact 2: Maui is an island. Fact 3: Maui was formed by a volcano. If the first three statements are facts, which of the following statements must also be a fact? I. Maui is surrounded by water. II. All islands are formed by volcanoes. III. All volcanoes are on islands. a. I only b. III only c. I and II only d. None of the statements is a known fact.

67

GovernmentAdda.com – QUESTIONS –

401. Fact 1: All dogs like to run. Fact 2: Some dogs like to swim. Fact 3: Some dogs look like their masters.

399. Fact 1: Some pens don’t write. Fact 2: All blue pens write. Fact 3: Some writing utensils are pens.

If the first three statements are facts, which of the following statements must also be a fact? I. All dogs who like to swim look like their masters. II. Dogs who like to swim also like to run. III. Dogs who like to run do not look like their masters. a. I only b. II only c. II and III only d. None of the statements is a known fact.

If the first three statements are facts, which of the following statements must also be a fact? I. Some writing utensils don’t write. II. Some writing utensils are blue. III. Some blue writing utensils don’t write. a. II only b. I and II only c. II and III only d. None of the statements is a known fact. 400. Fact 1: Mary said, “Ann and I both have cats.” Fact 2: Ann said, “I don’t have a cat.” Fact 3: Mary always tells the truth, but Ann sometimes lies. If the first three statements are facts, which of the following statements must also be a fact? I. Ann has a cat. II. Mary has a cat. III. Ann is lying. a. II only b. I and II only c. I, II, and III d. None of the statements is a known fact.

68

GovernmentAdda.com – QUESTIONS –



Set 27

404. Four friends in the sixth grade were sharing a pizza. They decided that the oldest friend would get the extra piece. Randy is two months older than Greg, who is three months younger than Ned. Kent is one month older than Greg. Who should get the extra piece of pizza? a. Randy b. Greg c. Ned d. Kent

(Answers begin on page 130.)

Here is yet another set of logic questions. The logic problems in this set are somewhat more complex than the ones in the previous sets, but your approach should be the same. Make a chart or grid so that you can order the given information. 402. Children are in pursuit of a dog whose leash has broken. James is directly behind the dog. Ruby is behind James. Rachel is behind Ruby. Max is ahead of the dog walking down the street in the opposite direction. As the children and dog pass, Max turns around and joins the pursuit. He runs in behind Ruby. James runs faster and is alongside the dog on the left. Ruby runs faster and is alongside the dog on the right. Which child is directly behind the dog? a. James b. Ruby c. Rachel d. Max

405. A four-person crew from Classic Colors is painting Mr. Field’s house. Michael is painting the front of the house. Ross is in the alley behind the house painting the back. Jed is painting the window frames on the north side, Shawn is on the south. If Michael switches places with Jed, and Jed then switches places with Shawn, where is Shawn? a. in the alley behind the house b. on the north side of the house c. in front of the house d. on the south side of the house

403. Nurse Kemp has worked more night shifts in a row than Nurse Rogers, who has worked five. Nurse Miller has worked fifteen night shifts in a row, more than Nurses Kemp and Rogers combined. Nurse Calvin has worked eight night shifts in a row, less than Nurse Kemp. How many night shifts in a row has Nurse Kemp worked? a. eight b. nine c. ten d. eleven

406. In a four-day period—Monday through Thursday—each of the following temporary office workers worked only one day, each a different day. Ms. Johnson was scheduled to work on Monday, but she traded with Mr. Carter, who was originally scheduled to work on Wednesday. Ms. Falk traded with Mr. Kirk, who was originally scheduled to work on Thursday. After all the switching was done, who worked on Tuesday? a. Mr. Carter b. Ms. Falk c. Ms. Johnson d. Mr. Kirk

69

GovernmentAdda.com – QUESTIONS –

407. The high school math department needs to appoint a new chairperson, which will be based on seniority. Ms. West has less seniority than Mr. Temple, but more than Ms. Brody. Mr. Rhodes has more seniority than Ms. West, but less than Mr. Temple. Mr. Temple doesn’t want the job. Who will be the new math department chairperson? a. Mr. Rhodes b. Mr. Temple c. Ms. West d. Ms. Brody

410. Four defensive football players are chasing the opposing wide receiver, who has the ball. Calvin is directly behind the ball carrier. Jenkins and Burton are side by side behind Calvin. Zeller is behind Jenkins and Burton. Calvin tries for the tackle but misses and falls. Burton trips. Which defensive player tackles the receiver? a. Burton b. Zeller c. Jenkins d. Calvin

408. Four people witnessed a mugging. Each gave a different description of the mugger. Which description is probably right? a. He was average height, thin, and middle-aged. b. He was tall, thin, and middle-aged. c. He was tall, thin, and young. d. He was tall, of average weight, and middle-aged.

411. Danielle has been visiting friends in Ridgewood for the past two weeks. She is leaving tomorrow morning and her flight is very early. Most of her friends live fairly close to the airport. Madison lives ten miles away. Frances lives five miles away, Samantha, seven miles. Alexis is farther away than Frances, but closer than Samantha. Approximately how far away from the airport is Alexis? a. nine miles b. seven miles c. eight miles d. six miles

409. As they prepare for the state championships, one gymnast must be moved from the Level 2 team to the Level 1 team. The coaches will move the gymnast who has won the biggest prize and who has the most experience. In the last competition, Roberta won a bronze medal and has competed seven times before. Jamie has won a silver medal and has competed fewer times than Roberta. Beth has won a higher medal than Jamie and has competed more times than Roberta. Michele has won a bronze medal, and it is her third time competing. Who will be moved to the Level 1 team? a. Roberta b. Beth c. Michele d. Jamie

412. Ms. Forest likes to let her students choose who their partners will be; however, no pair of students may work together more than seven class periods in a row. Adam and Baxter have studied together seven class periods in a row. Carter and Dennis have worked together three class periods in a row. Carter does not want to work with Adam. Who should be assigned to work with Baxter? a. Carter b. Adam c. Dennis d. Forest

70

GovernmentAdda.com – QUESTIONS –

413. At the baseball game, Henry was sitting in seat 253. Marla was sitting to the right of Henry in seat 254. In the seat to the left of Henry was George. Inez was sitting to the left of George. Which seat is Inez sitting in? a. 251 b. 254 c. 255 d. 256

71

GovernmentAdda.com – QUESTIONS –



Set 28

414. If the legislature decides to fund the agricultural subsidy program, national radio, and the small business loan program, the only other single program that can be funded is a. hurricane preparedness. b. harbor improvements. c. school music program. d. senate office building remodeling. e. international airport.

(Answers begin on page 131.)

Questions that involve analytical reasoning—better known as “logic games”—tend to inspire fear in most test takers. These games give the most trouble to test takers who haven’t defined a specific method for solving these problems. The best way to attack logic games is to have a plan. When solving the problems in this set, try the following strategy:

415. If the legislature decides to fund the agricultural subsidy program, national radio, and the small business loan program, what two other programs could they fund? a. harbor improvements and international airport b. harbor improvements and school music program c. hurricane preparedness and school music program d. hurricane preparedness and international airport e. harbor improvements and hurricane preparedness

1. Know the rules of the logic game and what each rule means. 2. Draw up an easy-to-reference diagram that includes all of the game’s information. 3. Look for common elements in the rules; you can combine these to make deductions. 4. Read the questions carefully; be sure you know what is being asked before you try to answer the question. Now, try solving the logic games in this set. Answer questions 414 through 416 on the basis of the information below.

416. Senators from urban areas are very concerned about assuring that there will be funding for a new international airport. Senators from rural areas refuse to fund anything until money for agricultural subsidies is appropriated. If the legislature funds these two programs, on which of the following could they spend the rest of the money? a. the school music program and national radio b. hurricane preparedness c. harbor improvements and the school music program d. small business loan program e. national radio and senate office building remodeling

The government of an island nation is in the process of deciding how to spend its limited income. It has $7 million left in its budget and eight programs to choose among. There is no provision in the constitution to have a surplus, and each program has requested the minimum amount they need; in other words, no program may be partially funded. The programs and their funding requests are: ■ ■ ■ ■ ■ ■ ■ ■

Hurricane preparedness: $2.5 million Harbor improvements: $1 million School music program: $0.5 million Senate office building remodeling: $1.5 million Agricultural subsidy program: $2 million National radio: $0.5 million Small business loan program: $3 million International airport: $4 million 72

GovernmentAdda.com – QUESTIONS –

Answer questions 417 through 418 on the basis of the information below.

Answer questions 419 through 421 on the basis of the information below.

A weekly television show routinely stars six actors, J, K, L, M, N, and O. Since the show has been on the air for a long time, some of the actors are good friends and some do not get along at all. In an effort to keep peace, the director sees to it that friends work together and enemies do not. Also, as the actors have become more popular, some of them need time off to do other projects. To keep the schedule working, the director has a few things she must be aware of:

A cinema complex with six movie theaters never shows the same movie in more than one theater. None of the theaters is the same size as any other, with number 1 being the largest and going in order to number 6, the smallest. The theater also has the following rules:













J will only work on episodes on which M is working. N will not work with K under any circumstances. M can only work every other week, in order to be free to film a movie At least three of the actors must appear in every weekly episode.







417. In a show about L getting a job at the same company J already works for and K used to work for, all three actors will appear. Which of the following is true about the other actors who may appear? a. M, N, and O must all appear. b. M may appear and N must appear. c. M must appear and O may appear. d. O may appear and N may appear. e. Only O may appear.

It will never show more than two R-rated movies at once. It will always show at least one G-rated movie and one PG-rated movie in the two middlesized theaters (theaters 3 and 4). It will never show more than one foreign film at a time and never in the biggest theater. The starting times of movies will be staggered by fifteen minutes and will always be on the quarter hour. Employees need twenty minutes between showings to clean the theaters.

The cinema has the following films to choose from this particular week: Shout, rated R Que Pasa, a Spanish film rated PG Abra Cadabra, rated G Lessons, rated R Jealousy, rated PG Mist, a Swedish film rated R Trek, rated NC-17 Fly, rated G

418. Next week, the show involves N’s new car and O’s new refrigerator. Which of the following is true about the other actors who may appear? a. M, J, L, and K all may appear. b. J, L, and K must appear. c. Only K may appear. d. Only L may appear. e. L and K must appear. 73

GovernmentAdda.com – QUESTIONS –

Answer questions 422 through 423 on the basis of the information below.

419. Which one of the following is an acceptable listing of films to show this week? a. Shout, Mist, Trek, Que Pasa, Fly, and Jealousy b. Shout, Mist, Trek, Fly, Jealousy, and Abra Cadabra c. Que Pasa, Lessons, Mist, Shout, Abra Cadabra, and Trek d. Shout, Lessons, Mist, Trek, Fly, and Jealousy e. Shout, Fly, Trek, Lessons, Abra Cadabra, and Mist

The six top songs (not in order) of 1968 were: “People Got to Be Free” by The Rascals “Sittin’ on the Dock of the Bay” by Otis Redding “Honey” by Bobby Goldsboro “Sunshine of Your Love” by Cream “Love Is Blue” by Paul Mauriat & His Orchestra “Hey Jude” by The Beatles Here are some rules about the order of the songs:

420. If Shout starts at 8:30, Mist at 8:15, Trek at 8:00, Fly at 7:45, Jealousy at 7:30, and Abra Cadabra at 7:15, and each movie is exactly two hours long, at what time will the next showing of Trek start? a. 10:00 b. 10:15 c. 10:30 d. 10:45 e. 11:00



■ ■



The Beatles and Cream do not appear next to each other on the list. The number 1 song is not “Love Is Blue.” The songs by individual artists are numbers 3 and 4. The Rascals appear right before Cream and right after Otis Redding.

422. Which of the following is true? a. Song #3 is “Honey” by Bobby Goldsboro. b. Song #6 is “Hey Jude” by the Beatles. c. Song #1 is “Sittin’ on the Dock of the Bay” by Otis Redding. d. Song #1 is “Sunshine of Your Love” by Cream. e. Song #3 is “Sittin’ on the Dock of the Bay” by Otis Redding.

421. The movies this week are showing in the following theaters: Theater 1: Shout Theater 2: Trek Theater 3: Abra Cadabra Theater 4: Jealousy Theater 5: Fly Theater 6: Mist Shout is doing the most business, followed by Trek and, to the management’s surprise, Mist. The management wants to move Mist to a larger theater. Which theater is the most logical? a. theater 1 b. theater 2 c. theater 3 d. theater 4 e. theater 5 74

GovernmentAdda.com – QUESTIONS –

423. Which of the following is the correct order of songs? a. “Honey,” “Love Is Blue,” “People Got to Be Free,” “Sunshine of Your Love,” “Sittin’ on the Dock of the Bay,” “Hey Jude” b. “Love Is Blue,” “Hey Jude,” “Honey,” “Sittin’ on the Dock of the Bay,” “People Got to Be Free,” “Sunshine of Your Love” c. “Sunshine of Your Love,” “People Got to Be Free,” “Sittin’ on the Dock of the Bay,” “Honey,” “Love Is Blue,” “Hey Jude” d. “Hey Jude,” “Love Is Blue,” “Honey,” “Sittin’ on the Dock of the Bay,” “People Got to Be Free,” “Sunshine of Your Love” e. “Honey,” “Sittin’ on the Dock of the Bay,” “Hey Jude,” “Sunshine of Your Love,” “People Got to Be Free,” “Love Is Blue”

75

GovernmentAdda.com – QUESTIONS –



Set 29

Answer questions 426 through 428 on the basis of the information below.

(Answers begin on page 132.)

If you’re having trouble after the first set of logic games, there’s a bonus for you—a complete explanation of questions 424 and 425 in the answers section, with a step-by-step explanation of how to set up a table to answer the questions.

Evan is a waiter in a café. After he turns in orders for the six people sitting at the counter—each of whom is eating alone and is sitting in chairs numbered 1 through 6—the cook opens a window in the kitchen and the order slips get messed up. Here’s what Evan remembers about the orders:

Answer questions 424 and 425 on the basis of the information below.



At a Halloween party where people were asked to dress as an object that represented their professions, Quentin, Rachel, Sarah, Thomas, and Ulysses were among the guests. The costumes included a flower, a pencil, a spoon, a camera, and a thermometer. The professions included a photographer, a florist, a doctor, an accountant, and a chef. ■ ■

■ ■ ■









Quentin is an accountant. Neither Rachel nor Sarah was dressed as a spoon. None of the men is a doctor. Thomas is dressed as a camera. Sarah is a florist.



424. Which person is dressed as a thermometer? a. Quentin b. Rachel c. Sarah d. Thomas e. Ulysses

The entree orders are: fried eggs, a hamburger, a cheeseburger, a vegetable burger, soup, and a ham sandwich. The two people who did not order sandwiches are sitting at chairs 3 and 4. The person who ordered the cheeseburger and the one who ordered the hamburger are not sitting next to each other. The person in chair number 5 is a regular. She will not sit next to anyone who is eating ham. The person eating the vegetable burger is not sitting in chair 2, but is sitting between the person who ordered fried eggs and the one who ordered a cheeseburger. The customer who ordered the hamburger is not sitting next to the customer who ordered soup.

426. To which customer should Evan serve the vegetable burger? a. the customer in chair 2 b. the customer in chair 3 c. the customer in chair 4 d. the customer in chair 5 e. the customer in chair 6

425. What is Ulysses’s profession? a. photographer b. florist c. doctor d. accountant e. chef

76

GovernmentAdda.com – QUESTIONS –

427. To which customer should Evan serve the soup? a. the customer in chair 1 b. the customer in chair 2 c. the customer in chair 3 d. the customer in chair 4 e. the customer in chair 5

429. With which entrée does the potato salad belong? a. soup b. hamburger c. cheeseburger d. fried eggs e. ham sandwich

428. To which customer should Evan serve the ham sandwich? a. the customer in chair 1 b. the customer in chair 2 c. the customer in chair 4 d. the customer in chair 5 e. the customer in chair 6

430. With which entrée do the french fries belong? a. soup b. cheeseburger c. hamburger d. fried eggs e. ham sandwich Answer questions 431 through 433 on the basis of the information below.

Use the additional information below, along with the information before question 426, to answer questions 429 and 430.

At a small company, parking spaces are reserved for the top executives: CEO, president, vice president, secretary, and treasurer—with the spaces lined up in that order. The parking lot guard can tell at a glance if the cars are parked correctly by looking at the color of the cars. The cars are yellow, green, purple, red, and blue, and the executives’ names are Alice, Bert, Cheryl, David, and Enid.

Now Evan has to decide who gets which side dish. Here is what he remembers, in addition to the previous information, about the orders, which were: cole slaw, hash browns, onion rings, potato salad, french fries, and lettuce salad. ■









The person who ordered soup did not order french fries, hash browns, onion rings, or a lettuce salad. The person who ordered onion rings is sitting in chair 6. The person who ordered potato salad is sitting between the person who ordered cole slaw and the one who ordered hash browns. The person who ordered a vegetable burger ordered a lettuce salad. The hash browns were ordered by the customer who ordered fried eggs.

■ ■

■ ■ ■ ■ ■



77

The car in the first space is red. A blue car is parked between the red car and the green car. The car in the last space is purple. The secretary drives a yellow car. Alice’s car is parked next to David’s. Enid drives a green car. Bert’s car is parked between Cheryl’s and Enid’s. David’s car is parked in the last space.

GovernmentAdda.com – QUESTIONS –

433. Who is the secretary? a. Enid b. David c. Cheryl d. Bert e. Alice

431. What color is the vice president’s car? a. green b. yellow c. blue d. purple e. red 432. Who is the CEO? a. Alice b. Bert c. Cheryl d. David e. Enid

78

GovernmentAdda.com – QUESTIONS –



Set 30

436. What team is in Fulton? a. Panthers b. Gazelles c. Whippets d. Kangaroos e. Antelopes

(Answers begin on page 134.)

Remember, the best way to answer these logic game questions is to attack the information systematically. Make a diagram outlining all the given information. There’s always at least one fact that can serve as your starting point, the place to begin eliminating possibilities.

Answer questions 437 through 439 on the basis of the information below.

Answer questions 434 through 436 on the basis of the information below.

Henri delivers flowers for a local florist. One lovely day, he left the windows open on the delivery van and the cards all blew off the bouquets. He has to figure out who gets which flowers. He has five bouquets, each of which has only one kind of flower: daisies, roses, carnations, iris, and gladioli. He has five cards with names on them: a birthday card for Inez, a congratulations-onyour-promotion card for Jenny, a graduation card for Kevin, an anniversary card for Liz, and a housewarming card for Michael. Here’s what Henri knows:

Five towns—Fulton, Groton, Hudson, Ivy, and Jersey—which are covered by the same newspaper, all have excellent soccer teams. The teams are named the Panthers, the Whippets, the Antelopes, the Kangaroos, and the Gazelles. The sports reporter, who has just started at the newspaper, has to be careful not to get them confused. Here is what she knows: ■



■ ■

The team in Fulton has beaten the Antelopes, Panthers, and Kangaroos. The Whippets have beaten the teams in Jersey, Hudson, and Fulton. The Antelopes are in Groton. The team in Hudson is not the Kangaroos.





■ ■

434. Where are the Whippets? a. Fulton b. Groton c. Hudson d. Ivy e. Jersey



Roses are Jenny’s favorite flower and what her friends always send. Gladioli are traditionally sent for a housewarming. Kevin is allergic to daisies and iris. Liz is allergic to daisies and roses. Neither Liz nor Inez has moved recently.

437. Which flowers should be delivered to Kevin? a. carnations b. iris c. gladioli d. daisies e. roses

435. Where are the Panthers? a. Ivy b. Jersey c. Fulton d. Groton e. Hudson

79

GovernmentAdda.com – QUESTIONS –

438. Who should get the housewarming gladioli? a. Jenny b. Kevin c. Liz d. Michael e. Inez

440. Which city is in the desert? a. Last Stand b. Mile City c. New Town d. Olliopolis e. Polberg

439. Which flowers should be delivered to Liz? a. gladioli b. daisies c. roses d. carnations e. iris

441. Which city got the most rain? a. Last Stand b. Mile City c. New Town d. Olliopolis e. Polberg 442. How much rain did Mile City get? a. 12 inches b. 27 inches c. 32 inches d. 44 inches e. 65 inches

Answer questions 440 through 443 on the basis of the information below. Five cities all got more rain than usual this year. The five cities are: Last Stand, Mile City, New Town, Olliopolis, and Polberg. The cities are located in five different areas of the country: the mountains, the forest, the coast, the desert, and in a valley. The rainfall amounts were: 12 inches, 27 inches, 32 inches, 44 inches, and 65 inches. ■

■ ■ ■

■ ■

443. Where is Olliopolis located? a. the mountains b. the coast c. in a valley d. the desert e. the forest

The city in the desert got the least rain; the city in the forest got the most rain. New Town is in the mountains. Last Stand got more rain than Olliopolis. Mile City got more rain than Polberg, but less rain than New Town. Olliopolis got 44 inches of rain. The city in the mountains got 32 inches of rain; the city on the coast got 27 inches of rain.

80

GovernmentAdda.com – QUESTIONS –



446. What occupation does Jarrod hold? a. a moderator b. a pilot c. a writer d. an attorney e. an explorer

Set 31 (Answers begin on page 135.)

Here’s one more set of logic games. Remember, mapping out the game using all the given information is the most efficient way to attack this type of question. Answer questions 444 through 447 on the basis of the information below.

447. Who is the writer? a. Gary b. Heloise c. Jarrod d. Kate e. Lane

Eleanor is in charge of seating the speakers at a table. In addition to the moderator, there will be a pilot, a writer, an attorney, and an explorer. The speakers’ names are Gary, Heloise, Jarrod, Kate, and Lane. ■

■ ■ ■







Answer question 448 on the basis of the information below.

The moderator must sit in the middle, in seat #3. The attorney cannot sit next to the explorer. Lane is the pilot. The writer and the attorney sit on either side of the moderator. Heloise, who is not the moderator, sits between Kate and Jarrod. The moderator does not sit next to Jarrod or Lane. Gary, who is the attorney, sits in seat #4.

Zinnia has a small container garden on her balcony. Each spring, she only has room to plant three vegetables. Because five vegetables are her favorites, she has worked out a schedule to plant each at least once every two years. The vegetables are: beans, cabbage, carrots, peppers, and tomatoes. ■



444. Who is the moderator? a. Lane b. Gary c. Heloise d. Kate e. Jarrod





445. Where does Jarrod sit? a. seat #1 b. seat #2 c. seat #3 d. seat #4 e. seat #5

81

Tomatoes are her favorites and she plants them every year. Each year, she plants only one vegetable that begins with the letter “C.” She never plants carrots and peppers in the same year. She will plant cabbage in the second year.

GovernmentAdda.com – QUESTIONS –

448. In what order does she plant the vegetables in the next two years? a. first year: tomatoes, carrots, cabbage second year: tomatoes, peppers, beans b. first year: tomatoes, peppers, beans second year: cabbage, carrots, tomatoes c. first year: tomatoes, carrots, peppers second year: tomatoes, cabbage, beans d. first year: tomatoes, carrots, beans second year: tomatoes, cabbage, peppers e. first year: tomatoes, cabbage, peppers second year: carrots, cabbage, beans

449. When does Sally do the dusting? a. Friday b. Monday c. Tuesday d. Wednesday e. Thursday 450. What task does Terry do on Wednesday? a. vacuuming b. dusting c. mopping d. sweeping e. laundry

Answer questions 449 through 453 on the basis of the information below.

451. What day is the vacuuming done? a. Friday b. Monday c. Tuesday d. Wednesday e. Thursday

Five roommates—Randy, Sally, Terry, Uma, and Vernon—each do one housekeeping task— mopping, sweeping, laundry, vacuuming, or dusting—one day a week, Monday through Friday. ■



■ ■





452. What task does Vernon do? a. vacuuming b. dusting c. mopping d. sweeping e. laundry

Vernon does not vacuum and does not do his task on Tuesday. Sally does the dusting, and does not do it on Monday or Friday. The mopping is done on Thursday. Terry does his task, which is not vacuuming, on Wednesday. The laundry is done on Friday, and not by Uma. Randy does his task on Monday.

453. What day does Uma do her task? a. Monday b. Tuesday c. Wednesday d. Thursday e. Friday

82

GovernmentAdda.com – QUESTIONS –



Set 32

455. It is well known that the world urgently needs adequate distribution of food, so that everyone gets enough. Adequate distribution of medicine is just as urgent. Medical expertise and medical supplies need to be redistributed throughout the world so that people in emerging nations will have proper medical care.

(Answers begin on page 136.)

Each of the questions in this set contains a short paragraph, and each paragraph presents an argument. Your task is to read the paragraph carefully and determine the main point the author is trying to make. What conclusion can be drawn from the argument? Each paragraph is followed by five statements. One statement supports the author’s argument better than the others do. The best way to approach these questions is to first read the paragraph and then restate the author’s main argument, or conclusion, in your own words.

This paragraph best supports the statement that a. the majority of the people in the world have never been seen by a doctor. b. food production in emerging nations has slowed during the past several years. c. most of the world’s doctors are selfish about giving time and money to the poor. d. the medical-supply industry should step up production of its products. e. many people who live in emerging nations are not receiving proper medical care.

454. If you’re a fitness walker, there is no need for a commute to a health club. Your neighborhood can be your health club. You don’t need a lot of fancy equipment to get a good workout either. All you need is a well-designed pair of athletic shoes. This paragraph best supports the statement that a. fitness walking is a better form of exercise than weight lifting. b. a membership in a health club is a poor investment. c. walking outdoors provides a better workout than walking indoors. d. fitness walking is a convenient and valuable form of exercise. e. poorly designed athletic shoes can cause major foot injuries.

456. The criminal justice system needs to change. The system could be more just if it allowed victims the opportunity to confront the person who has harmed them. Also, mediation between victims and their offenders would give the offenders a chance to apologize for the harm they have done. This paragraph best supports the statement that victims of a crime should a. learn to forgive their offenders. b. have the right to confront their offenders. c. learn the art of mediation. d. insist that their offenders be punished. e. have the right to impose a sentence on their offenders.

83

GovernmentAdda.com – QUESTIONS –

459. One of the warmest winters on record has put consumers in the mood to spend money. Spending is likely to be the strongest in thirteen years. During the month of February, sales of existing single-family homes hit an annual record rate of 4.75 million.

457. In the past, consumers would rarely walk into an ice cream store and order low-fat ice cream. But that isn’t the case today. An increasing health consciousness combined with a much bigger selection of tasty low-fat foods in all categories has made low-fat ice cream a very profitable item for ice cream store owners.

This paragraph best supports the statement that a. consumer spending will be higher thirteen years from now than it is today. b. more people buy houses in the month of February than in any other month. c. during the winter months, the prices of single-family homes are the lowest. d. there were about 4 million homes for sale during the month of February. e. warm winter weather is likely to affect the rate of home sales.

This paragraph best supports the statement that a. low-fat ice cream produces more revenue than other low-fat foods. b. ice cream store owners would be better off carrying only low-fat ice cream. c. ice cream store owners no longer think that low-fat ice cream is an unpopular item. d. low-fat ice cream is more popular than other kinds of ice cream. e. consumers are fickle and it is impossible to please them.

460. One New York publisher has estimated that 50,000 to 60,000 people in the United States want an anthology that includes the complete works of William Shakespeare. And what accounts for this renewed interest in Shakespeare? As scholars point out, his psychological insights into both male and female characters are amazing even today.

458. A few states in this country are considering legislation that would prohibit schools from using calculators before the sixth grade. Other states take a different position. Some states are insisting on the purchase of graphing calculators for every student in middle school.

This paragraph best supports the statement that a. Shakespeare’s characters are more interesting than fictional characters today. b. people today are interested in Shakespeare’s work because of the characters. c. academic scholars are putting together an anthology of Shakespeare’s work. d. New Yorkers have a renewed interested in the work of Shakespeare. e. Shakespeare was a psychiatrist as well as a playwright.

This paragraph best supports the statement that in this country a. there are at least two opinions about the use of calculators in schools. b. calculators are frequently a detriment to learning math. c. state legislators are more involved in education than ever before. d. the price of graphing calculators is less when schools buy in bulk. e. the argument against calculators in schools is unfounded.

84

GovernmentAdda.com – QUESTIONS –

463. Today’s high school students spend too much time thinking about trivial and distracting matters such as fashion. Additionally, they often dress inappropriately on school grounds. Rather than spending time writing another detailed dress policy, we should make school uniforms mandatory. If students were required to wear uniforms, it would increase a sense of community and harmony in our schools and it would instill a sense of discipline in our students. Another positive effect would be that teachers and administrators would no longer have to act as clothing police, freeing them up to focus on more important issues.

461. Today’s workforce has a new set of social values. Ten years ago, a manager who was offered a promotion in a distant city would not have questioned the move. Today, a manager in that same situation might choose family happiness instead of career advancement. This paragraph best supports the statement that a. most managers are not loyal to the corporations for which they work. b. businesses today do not understand their employees’ needs. c. employees’ social values have changed over the past ten years. d. career advancement is not important to today’s business managers. e. companies should require their employees to accept promotions.

This paragraph best supports the statement that a. inappropriate clothing leads to failing grades. b. students who wear school uniforms get into better colleges. c. teachers and administrators spend at least 25% of their time enforcing the dress code. d. students are not interested in being part of a community. e. school uniforms should be compulsory for high school students.

462. Generation Xers are those people born roughly between 1965 and 1981. As employees, Generation Xers tend to be more challenged when they can carry out tasks independently. This makes Generation Xers the most entrepreneurial generation in history. This paragraph best supports the statement that Generation Xers a. work harder than people from other generations. b. have a tendency to be self-directed workers. c. have an interest in making history. d. tend to work in jobs that require risk-taking behavior. e. like to challenge their bosses’ work attitudes.

85

GovernmentAdda.com – QUESTIONS –



Set 33

465. There are no effective boundaries when it comes to pollutants. Studies have shown that toxic insecticides that have been banned in many countries are riding the wind from countries where they remain legal. Compounds such as DDT and toxaphene have been found in remote places like the Yukon and other Arctic regions.

(Answers begin on page 138.)

For more practice with logical reasoning, try another set of questions that contain short paragraphs that make a specific argument. Remember, you are looking for the statement that is best supported by the information given in the passage. 464. Critical reading is a demanding process. To read critically, you must slow down your reading and, with pencil in hand, perform specific operations on the text. Mark up the text with your reactions, conclusions, and questions. When you read, become an active participant.

This paragraph best supports the statement that a. toxic insecticides such as DDT have not been banned throughout the world. b. more pollutants find their way into polar climates than they do into warmer areas. c. studies have proven that many countries have ignored their own antipollution laws. d. DDT and toxaphene are the two most toxic insecticides in the world. e. even a worldwide ban on toxic insecticides would not stop the spread of DDT pollution.

This paragraph best supports the statement that a. critical reading is a slow, dull, but essential process. b. the best critical reading happens at critical times in a person’s life. c. readers should get in the habit of questioning the truth of what they read. d. critical reading requires thoughtful and careful attention. e. critical reading should take place at the same time each day.

466. The Fourth Amendment to the Constitution protects citizens against unreasonable searches and seizures. No search of a person’s home or personal effects may be conducted without a written search warrant issued on probable cause. This means that a neutral judge must approve the factual basis justifying a search before it can be conducted. This paragraph best supports the statement that the police cannot search a person’s home or private papers unless they have a. legal authorization. b. direct evidence of a crime. c. read the person his or her constitutional rights. d. a reasonable belief that a crime has occurred. e. requested that a judge be present.

86

GovernmentAdda.com – QUESTIONS –

469. Human technology developed from the first stone tools about two and a half million years ago. At the beginning, the rate of development was slow. Hundreds of thousands of years passed without much change. Today, new technologies are reported daily on television and in newspapers.

467. During colonial times in America, juries were encouraged to ask questions of the parties in the courtroom. The jurors were, in fact, expected to investigate the facts of the case themselves. If jurors conducted an investigation today, we would throw out the case. This paragraph best supports the statement that a. juries are less important today than they were in colonial times. b. jurors today are less interested in court cases than they were in colonial times. c. courtrooms today are more efficient than they were in colonial times. d. jurors in colonial times were more informed than jurors today. e. the jury system in America has changed since colonial times.

This paragraph best supports the statement that a. stone tools were not really technology. b. stone tools were in use for two and a half million years. c. there is no way to know when stone tools first came into use. d. In today’s world, new technologies are constantly being developed. e. none of the latest technologies is as significant as the development of stone tools.

468. Mathematics allows us to expand our consciousness. Mathematics tells us about economic trends, patterns of disease, and the growth of populations. Math is good at exposing the truth, but it can also perpetuate misunderstandings and untruths. Figures have the power to mislead people. This paragraph best supports the statement that a. the study of mathematics is dangerous. b. words are more truthful than figures. c. the study of mathematics is more important than other disciplines. d. the power of numbers is that they cannot lie. e. figures are sometimes used to deceive people.

87

GovernmentAdda.com – QUESTIONS –

471. In the 1966 Supreme Court decision Miranda v. Arizona, the court held that before the police can obtain statements from a person subjected to an interrogation, the person must be given a Miranda warning. This warning means that a person must be told that he or she has the right to remain silent during the police interrogation. Violation of this right means that any statement that the person makes is not admissible in a court hearing.

470. Obesity is a serious problem in this country. Research suggests that obesity can lead to a number of health problems including diabetes, asthma, and heart disease. Recent research has even indicated that there may be a relationship between obesity and some types of cancer. Major public health campaigns that increase awareness and propose simple lifestyle changes that will, with diligence and desire, eliminate or least mitigate the incidence of obesity are a crucial first step in battling this critical problem.

This paragraph best supports the statement that a. police who do not warn persons of their Miranda rights are guilty of a crime. b. a Miranda warning must be given before a police interrogation can begin. c. the police may no longer interrogate persons suspected of a crime unless a lawyer is present. d. the 1966 Supreme Court decision in Miranda should be reversed. e. persons who are interrogated by police should always remain silent until their lawyer comes.

This paragraph best supports the statement that a. public health campaigns that raise consciousness and propose lifestyle changes are a productive way to fight obesity. b. obesity is the leading cause of diabetes in our country. c. people in our country watch too much television and do not exercise enough. d. a decline in obesity would radically decrease the incidence of asthma. e. fast-food restaurants and unhealthy school lunches contribute greatly to obesity.

88

GovernmentAdda.com – QUESTIONS –

473. Yoga has become a very popular type of exercise, but it may not be for everyone. Before you sign yourself up for a yoga class, you need to examine what it is you want from your fitness routine. If you’re looking for a high-energy, fast-paced aerobic workout, a yoga class might not be your best choice.

472. Walk into any supermarket or pharmacy and you will find several shelves of products designed to protect adults and children from the sun. Additionally, a host of public health campaigns have been created, including National Skin Cancer Awareness Month, that warn us about the sun’s damaging UV rays and provide guidelines about protecting ourselves. While warnings about the sun’s dangers are frequent, a recent survey found that fewer than half of all adults adequately protect themselves from the sun.

This paragraph best supports the statement that a. yoga is more popular than high-impact aerobics. b. before embarking on a new exercise regimen, you should think about your needs and desires. c. yoga is changing the world of fitness in major ways. d. yoga benefits your body and mind. e. most people think that yoga isn’t a rigorous form of exercise.

This paragraph best supports the statement that a. children are better protected from the sun’s dangerous rays than adults. b. sales of sun protection products are at an all-time high. c. adults are not heeding the warnings about the dangers of sun exposure seriously enough. d. more adults have skin cancer now than ever before. e. there is not enough information disseminated about the dangers of sun exposure.

89

GovernmentAdda.com – QUESTIONS –



Set 34

475. During the last six years, the number of practicing physicians has increased by about 20%. During the same time period, the number of healthcare managers has increased by more than 600%. These percentages mean that many doctors have lost the authority to make their own schedules, determine the fees that they charge, and decide on prescribed treatments.

(Answers begin on page 140.)

Here’s one more set of questions based on short paragraphs that make a specific argument. You will sometimes have to use inference—reading between the lines—to see which statement is best supported by the passage. 474. For too long, school cafeterias, in an effort to provide food they thought would be appetizing to young people, mimicked fast-food restaurants, serving items such as burgers and fries, pizza, hot dogs, and fried chicken. School districts nationwide are now addressing this trend by incorporating some simple and inexpensive options that will make cafeteria lunches healthier while still appealing to students.

This paragraph best supports the statement that doctors a. resent the interference of healthcare managers. b. no longer have adequate training. c. care a great deal about their patients. d. are less independent than they used to be. e. are making a lot less money than they used to make.

This paragraph best supports the statement that a. school cafeterias have always emphasized nutritional guidelines over any other considerations. b. young people would rather eat in a school cafeteria than a local fast-food restaurant. c. school lunch menus are becoming healthier due to major new initiatives on the part of school districts. d. it is possible to make school lunches both healthier and appealing without spending a great deal of money and undertaking a radical transformation. e. vegetarian lunch options would greatly improve the nutritional value of the school lunch program.

476. By the time they reach adulthood, most people can perform many different activities involving motor skills. Motor skills involve such diverse tasks as riding a bicycle, threading a needle, and cooking a dinner. What all these activities have in common is their dependence on precision and timing of muscular movement. This paragraph best supports the statement that a. most adults have not refined their motor skills. b. all adults know how to ride a bicycle. c. refined motor skills are specifically limited to adults. d. children perform fewer fine motor activities in a day than adults do. e. threading a needle is a precise motor skill.

90

GovernmentAdda.com – QUESTIONS –

479. Originating in the 1920s, the Pyramid scheme is one of the oldest con games going. Honest people are often pulled in, thinking the scheme is a legitimate investment enterprise. The first customer to “fall for” the Pyramid scheme will actually make big money and will therefore persuade friends and relatives to join also. The chain then continues with the con artist who originated the scheme pocketing, rather than investing, the money. Finally, the pyramid collapses, but by that time, the scam artist will usually have moved out of town, leaving no forwarding address.

477. Close-up images of Mars by the Mariner 9 probe indicated networks of valleys that looked like the stream beds on Earth. These images also implied that Mars once had an atmosphere that was thick enough to trap the sun’s heat. If this were true, something happened to Mars billions of years ago that stripped away the planet’s atmosphere. This paragraph best supports the statement that a. Mars now has little or no atmosphere. b. Mars once had a thicker atmosphere than Earth does. c. the Mariner 9 probe took the first pictures of Mars. d. Mars is closer to the sun than Earth is. e. Mars is more mountainous than Earth is.

This paragraph best supports the statement that a. it is fairly easy to spot a Pyramid scheme in the making. b. the first customer of a Pyramid scheme is the most gullible. c. the people who set up Pyramid schemes are able to fool honest people. d. the Pyramid scheme had its heyday in the 1920s, but it’s making a comeback. e. the Pyramid scheme got its name from its structure.

478. Forest fires feed on decades-long accumulations of debris and leap from the tops of young trees into the branches of mature trees. Fires that jump from treetop to treetop can be devastating. In old-growth forests, however, the shade of mature trees keeps thickets of small trees from sprouting, and the lower branches of mature trees are too high to catch the flames. This paragraph best supports the statement that a. forest fire damage is reduced in old-growth forests. b. small trees should be cut down to prevent forest fires. c. mature trees should be thinned out to prevent forest fires. d. forest fires do the most damage in oldgrowth forests. e. old-growth forests have a larger accumulation of forest debris.

91

GovernmentAdda.com – QUESTIONS –

481. The image of a knitter as an older woman sitting in a comfortable, old-fashioned living room with a basket of yarn at her feet and a bun in her hair is one of the past. As knitting continues to become more popular and increasingly trendy, it is much more difficult to describe the average knitter. Knitters today might be 18, 28, 40, or 65. They might live in a big urban center and take classes in a knitting shop that doubles as a café or they may gather in suburban coffee shops to support one another in knitting and other aspects of life. They could be college roommates knitting in their dorm room or two senior citizens knitting in a church hall. Even men are getting in the act. It would be incredibly difficult to come up with an accurate profile of a contemporary knitter to replace that image of the old woman with the basket of yarn!

480. Most Reality TV centers on two common motivators: fame and money. The shows transform waitresses, hairdressers, investment bankers, counselors, and teachers, to name a few, from obscure figures to household names. A lucky few successfully parlay their fifteen minutes of fame into celebrity. The luckiest stars of Reality TV also reap huge financial rewards for acts including eating large insects, marrying someone they barely know, and revealing their innermost thoughts to millions of people. This paragraph best supports the statement that a. the stars of Reality TV are interested in being rich and famous. b. Reality TV is the best thing that has happened to network television in a long time. c. for Reality TV stars, fame will last only as long as their particular television show. d. traditional dramas and sitcoms are being replaced by Reality TV programming at an alarming rate. e. Reality TV shows represent a new wave of sensationalistic, low quality programming.

This paragraph best supports the statement that a. people are returning to knitting in an attempt to reconnect with simpler times. b. knitting is now more of a group activity, as opposed to an individual hobby. c. creating an accurate profile of a particular type of person depends on the people in this group having traits and characteristics in common. d. today’s knitters are much less accomplished than knitters of the past. e. young people are turning to knitting in record numbers.

92

GovernmentAdda.com – QUESTIONS –



482. A person seeking to refute the argument might argue that a. all doctors charge too much money and should lower their fees. b. medical practices are more expensive to maintain in large cities than in small towns and rural areas. c. doctors who owe student loans should charge more than other doctors. d. medical care from small-town doctors is better than medical care from large-city doctors. e. certain medical specialists should charge more than others.

Set 35 (Answers begin on page 141.)

A typical logical reasoning question presents an argument and asks you to analyze it. You may be asked to draw further conclusions from the argument, determine what strengthens or weakens the argument, find flaws in the argument, or justify the argument. Success with these types of questions depends on your being able to understand the structure of the argument. Remember that every argument has a point of view. Every argument draws a conclusion and is generally supported with evidence. Study each passage to determine how each sentence contributes to the argument the speaker is trying to make. Then make sure you understand the question that is being asked before you choose from the five answer options.

483. A major flaw in the argument is that the speaker assumes that a. all doctors are specialists. b. all patients carry health insurance. c. all doctors have huge student loans. d. all patients take too much time. e. all doctors see the same number of patients.

Answer questions 482 and 483 on the basis of the information below. According to last week’s newspaper, doctors in large cities make more money than doctors in small towns or rural areas. It does not seem fair that just because a doctor’s office is in a fancy building or at a fancy address, he or she can charge the patients more. Of course, some medical schools cost more than others, but basically all doctors spend a lot of money and a long time in school. There’s no proof that graduates of the more expensive schools practice in big cities and graduates of the less expensive schools practice in small towns. All doctors should charge the same. Whether a patient goes to a doctor in a big city or small town, the cost should be the same.

Answer questions 484 and 485 on the basis of the information below. English ought to be the official language of the United States. There is no reason for the government to spend money printing documents in several different languages, just to cater to people who cannot speak English. The government has better ways to spend our money. People who come to this country should learn to speak English right away.

93

GovernmentAdda.com – QUESTIONS –

484. Which of the following, if true, would make the speaker’s argument stronger? a. There is currently a law that says the government must provide people with documents in their native language. b. Most people in the United States who do not speak English were born here. c. Immigration rates have decreased in recent years. d. Many other countries have an official language. e. Canada has two official languages.

486. Which of the following best expresses the main point of the passage? a. Only veterans care about the flag-burning issue. b. Flag burning almost never happens, so outlawing it is a waste of time. c. Flag burning will be a very important issue in the next election. d. To outlaw flag burning is to outlaw what the flag represents. e. Burning the flag should only be illegal when it is done in foreign countries.

485. Which of the following, if true, would make the speaker’s argument weaker? a. The government currently translates official documents into more than twenty languages. b. English is the most difficult language in the world to learn. c. Most people who immigrate to the United States learn English within two years of their arrival. d. Making English the official language is a politically unpopular idea. e. People who are bilingual are usually highly educated.

487. Which of the following, if true, would weaken the speaker’s argument? a. An action is not considered a part of freedom of speech. b. People who burn the flag usually commit other crimes as well. c. The flag was not recognized by the government until 1812. d. State flags are almost never burned. e. Most people are against flag burning. 488. Which of the following is similar to the argument made by the speaker? a. The rich should not be allowed to “buy” politicians, so the Congress should enact campaign finance reform. b. The idea of freedom of religion also means the right not to participate in religion, so mandated school prayer violates freedom of religion. c. The Constitution guarantees freedom to own property, so taxes should be illegal. d. Convicted felons should not have their convictions overturned on a technicality. e. In order to understand what may be constitutional today, one needs to look at what the laws were when the Constitution was enacted.

Answer questions 486 through 488 on the basis of the information below. Some groups want to outlaw burning the flag. They say that people have fought and died for the flag and that citizens of the United States ought to respect that. But I say that respect cannot be legislated. Also, most citizens who have served in the military did not fight for the flag, they fought for what the flag represents. Among the things the flag represents is freedom of speech, which includes, I believe, the right for a citizen to express displeasure with the government by burning the flag in protest.

94

GovernmentAdda.com – QUESTIONS –



Set 36

Answer questions 490 and 491 on the basis of the information below.

(Answers begin on page 142.)

Some logical reasoning questions ask you to determine the method the speaker is using when he or she presents the argument. Method-of-argument questions ask you to demonstrate an understanding of how a speaker’s argument is put together. To determine the method of argument, again focus on the conclusion and on the evidence presented. What method does the speaker use to link the two?

A recent study on professional football players showed that this new ointment helps relieve joint pain. My mother has arthritis, and I told her she should try it, but she says it probably won’t help her. 490. What argument should the mother use to point out why the ointment probably will not help her arthritis? a. The ointment was just experimental. b. The ointment is expensive. c. Football players’ joint pain is not the result of arthritis. d. She has already tried another ointment and it didn’t work. e. Football players are generally younger than she is.

Answer question 489 on the basis of the information below. I know that our rules prohibit members from bringing more than one guest at a time to the club, but I think there should be an exception to the rule on Tuesdays, Wednesdays, and Thursdays. Members should be allowed to bring multiple guests on those days, since the majority of members use the club facilities on the other four days of the week.

491. Which of the following, if true, would strengthen the speaker’s argument? a. The mother used to be a professional bowler. b. Football players’ injuries are rarely painful. c. The mother’s arthritis only flares up in bad weather. d. The mother finds exercise helps her arthritis. e. Football players who are injured tend to develop arthritis.

489. The rules restricting the number of guests a member can bring to the club probably are intended to a. assure that members are not crowded by the presence of guests. b. provide extra income for the club on slow days. c. allow members to bring guests to the club for special events. d. restrict guests to public areas of the club. e. control the exact number of people in the club at any time.

Answer questions 492 through 494 on the basis of the information below. Giving children computers in grade school is a waste of money and teachers’ time. These children are too young to learn how to use computers effectively and need to spend time on learning the basics, like arithmetic and reading. After all, a baby has to crawl before she can walk.

95

GovernmentAdda.com – QUESTIONS –

Answer questions 495 and 496 on the basis of the information below.

492. Which of the following methods of argument is used in the previous passage? a. a specific example that illustrates the speaker’s point b. attacking the beliefs of those who disagree with the speaker c. relying on an analogy to prove the speaker’s point d. displaying statistics that back up the speaker’s point e. comparing different methods of learning

The corner of Elm and Third needs to have a stoplight. Children cross this intersection on the way to school, and sometimes, they do not check for traffic. I’ve seen several children almost get hit by cars at this corner. I know that stoplights are not cheap, and I know that children cannot be protected from every danger, but this is one of the worst intersections in town. There needs to be a stoplight here so that traffic will be slowed down and the children can walk more safely.

493. Which of the following, if true, would strengthen the speaker’s argument? a. studies showing computers are expensive b. research on the effect of computer games on children c. examples of high school students who use computers improperly d. proof that the cost of computers is coming down e. evidence that using computers makes learning to read difficult

495. Which of the following methods of argument is used in the above passage? a. analogy—comparing the intersection to something dangerous b. emotion—referring to the safety of children to get people interested c. statistical analysis—noting the number of children almost hit and the cost of a stop light d. personalization—telling the story of one child’s near accident at the intersection e. attack—pointing out that people who are against the stoplight do not care about children

494. Which of the following, if true, would weaken the speaker’s argument? a. a demonstration that computers can be used to teach reading and arithmetic b. analysis of the cost-effectiveness of new computers versus repairing old computers c. examples of adults who do not know how to use computers d. recent grade reports of students in the computer classes e. a visit to a classroom where computers are being used

496. Which of the following, if true, would weaken the speaker’s argument? a. Sometimes, cars run red lights. b. Fewer children are injured at corners that have stoplights. c. If parents teach their children basic traffic safety, then they might remember to look for cars. d. Children from this neighborhood used to take the bus to a school farther away. e. In the last year, there have only been three minor accidents at the intersection and none of them involved children.

96

GovernmentAdda.com – QUESTIONS –



Set 37

497. In what way does Lars’s comment relate to Frances’s? a. It weakens Frances’s argument by changing the focus of the discussion. b. It strengthens Frances’s argument by providing support for her premise. c. It states the logical outcome of Frances’s views. d. It cannot be true if Frances’s assertion about parental responsibility is true. e. It provides an argument that is the opposite of Frances’s views.

(Answers begin on page 143.)

Another type of logical reasoning question presents you with two different speakers talking about the same issue. Sometimes, the speakers’ arguments overlap; in other words, they support each other. Sometimes, the speakers are presenting opposing viewpoints. For these items, make sure you understand the conclusion of both speakers before you attempt to answer the questions. Answer questions 497 and 498 on the basis of the information below.

498. What main assumption underlies each statement? a. As teachers become more scarce, schools will have to learn to be more cost-effective in recruiting new teachers. b. In the information age, the equipment schools must purchase for their students is getting more expensive. c. The study about students and breakfast is inconclusive at best, and more studies should be conducted to find out if school breakfasts are healthy. d. Schools have never had the responsibility for supplying students with breakfast; rather, they spend their money on teachers, books, and other tangibles of education. e. Parents are not assuming enough responsibility for their children’s education and should become more involved in school issues.

Frances: Studies show that eating a healthy breakfast improves young children’s ability to learn. However, it is not the responsibility of the schools to provide this meal; it is the responsibility of each child’s parents. Lars: Although it would be nice if the schools could provide each child with a healthy breakfast, the cost of doing that takes money away from other, more important learning resources, such as the purchase of new computers. In the long run, children learn more when the schools concentrate on the services they traditionally provide and the parents do what they are supposed to do.

97

GovernmentAdda.com – QUESTIONS –

499. What is the point at issue between Quinn and Dakota? a. whether sixteen-year-olds should be required to take drivers’ education before being issued a license b. whether schools ought to provide drivers’ education to fourteen- and fifteen-year-old students c. whether the standards for issuing drivers’ licenses should become more stringent d. whether sixteen-year-olds are prepared to drive in today’s traffic conditions e. whether parents are able to do a good job teaching their children to drive

Answer questions 499 through 501 on the basis of the information below. Quinn: Our state is considering raising the age at which a person can get a driver’s license to eighteen. This is unfair because the age has been sixteen for many years and sixteen-year-olds today are no less responsible than their parents and grandparents were at sixteen. Many young people today who are fourteen and fifteen years old are preparing to receive their licenses by driving with a learner’s permit and a licensed driver, usually one of their parents. It would not be fair to suddenly say they have to wait two more years.

500. On what does Quinn rely in making her argument? a. statistics b. emotion c. fairness d. anecdotes e. actualities

Dakota: It is true that people have been allowed to receive a driver’s license at sixteen for generations. However, in recent years, the increase in traffic means drivers face more dangers than ever and must be ready to respond to a variety of situations. The fact that schools can no longer afford to teach drivers’ education results in too many young drivers who are not prepared to face the traffic conditions of today.

501. On what does Dakota rely in making her argument? a. statistics b. emotion c. fairness d. anecdotes e. actualities

98

GovernmentAdda.com

Answers



Set 1

8. b. This is an alternating number subtraction series. First, 2 is subtracted, then 4, then 2, and so on. 9. c. In this simple alternating subtraction and addition series; 1 is subtracted, then 2 is added, and so on. 10. d. This alternating addition series begins with 3; then 1 is added to give 4; then 3 is added to give 7; then 1 is added, and so on. 11. a. This is a simple alternating subtraction series, which subtracts 2, then 5. 12. c. In this alternating repetition series, the random number 21 is interpolated every other number into an otherwise simple addition series that increases by 2, beginning with the number 9. 13. b. In this series, each number is repeated, then 13 issubtracted to arrive at the next number. 14. c. This is a simple multiplication series. Each number is 3 times more than the previous number. 15. a. This is a simple division series. Each number is divided by 5.

(Page 2)

1. b. This is a simple addition series. Each number increases by 2. 2. b. This is a simple subtraction series. Each number is 6 less than the previous number. 3. c. This is an alternation with repetition series in which each number repeats itself and then increases by 7. 4. a. This is a simple subtraction series. Each number is 35 less than the previous number. 5. d. In this addition series, 1 is added to the first number; 2 is added to the second number; 3 is added to the third number; and so forth. 6. d. This is a simple addition series with a random number, 8, interpolated as every other number. In the series, 6 is added to each number except 8, to arrive at the next number. 7. a. This is an alternating addition and subtraction series. In the first pattern, 10 is subtracted from each number to arrive at the next. In the second, 5 is added to each number to arrive at the next.

99

GovernmentAdda.com – ANSWERS –

16. b. This is a simple alternating addition and subtraction series. In the first pattern, 3 is added; in the second, 2 is subtracted. 17. b. This is an alternating multiplication and subtracting series: First, multiply by 2 and then subtract 8. 18. c. In this simple addition series, each number increases by 0.8.

19. d. In this simple subtraction series, each number decreases by 0.4. 20. b. This is a simple division series; each number is one-half of the previous number.

100

GovernmentAdda.com – ANSWERS –



Set 2

(Page 4)

21. b. In this simple subtraction series, each number is 6 less than the previous number. 22. c. In this simple addition series, each number is 5 greater than the previous number. 23. e. This is a simple subtraction with repetition series. It begins with 20, which is repeated, then 3 is subtracted, resulting in 17, which is repeated, and so on. 24. d. This is a simple addition series with a random number, 18, interpolated as every third number. In the series, 4 is added to each number except 18, to arrive at the next number. 25. a. In this alternating repetition series, a random number, 33, is interpolated every third number into a simple addition series, in which each number increases by 2. 26. b. This is a simple addition series, which begins with 2 and adds 6. 27. a. This is an alternating subtraction series with the interpolation of a random number, 5, as every third number. In the subtraction series, 3 is subtracted, then 4, then 3, and so on. 28. e. This is a simple alternating addition and subtraction series. First, 3 is added, then 1 is subtracted, then 3 is added, 1 subtracted, and so on. 29. b. This is a simple subtraction series in which a random number, 85, is interpolated as every third number. In the subtraction series, 10 is subtracted from each number to arrive at the next. 30. c. Here, every other number follows a different pattern. In the first series, 6 is added to each number to arrive at the next. In the second series, 10 is added to each number to arrive at the next. 31. e. This is an alternating addition series, in which 10 is added, then 5, then 10, and so on. 101

32. a. This is a subtraction series with repetition. Each number repeats itself and then decreases by 9. 33. e. This is an alternating subtraction series with repetition. There are two different patterns here. In the first, a number repeats itself; then 3 is added to that number to arrive at the next number, which also repeats. This gives the series 17, 17, 20, 20, 23, and so on. Every third number follows a second pattern, in which 3 is subtracted from each number to arrive at the next: 34, 31, 28. 34. d. This is an alternating addition series with a random number, 4, interpolated as every third number. In the main series, 1 is added, then 2 is added, then 1, then 2, and so on. 35. e. This is an alternating repetition series, in which a random number, 61, is interpolated as every third number into an otherwise simple subtraction series. Starting with the second number, 57, each number (except 61) is 7 less than the previous number. 36. d. Here is a simple addition series, which begins with 9 and adds 7. 37. c. This is an alternating repetition series, with a random number, 22, interpolated as every third number into an otherwise simple addition series. In the addition series, 4 is added to each number to arrive at the next number. 38. d. This is an alternating addition and subtraction series. In the first pattern, 2 is added to each number to arrive at the next; in the alternate pattern, 6 is subtracted from each number to arrive at the next. 39. d. In this simple addition series, each number is 5 more than the previous number. 40. b. This is an alternating addition series, with a random number, 21, interpolated as every third number. The addition series alternates between adding 3 and adding 4. The number 21 appears after each number arrived at by adding 3.

GovernmentAdda.com – ANSWERS –



Set 3

(Page 6)

41. e. This is a simple subtraction series, in which 3 is subtracted from each number to arrive at the next. 42. e. This simple addition series adds 4 to each number to arrive at the next. 43. d. This is a simple subtraction series, in which 4 is subtracted from each number to arrive at the next. 44. d. Here, there are two alternating patterns, one addition and one subtraction. The first starts with 2 and increases by 2; the second starts with 44 and decreases by 3. 45. a. In this simple subtraction series, the numbers decrease by 3. 46. b. In this simple addition with repetition series, each number in the series repeats itself, and then increases by 12 to arrive at the next number. 47. b. This is an alternating addition and subtraction series, in which the addition of 4 is alternated with the subtraction of 3. 48. e. Two patterns alternate here, with every third number following the alternate pattern. In the main series, beginning with 4, 3 is added to each number to arrive at the next. In the alternating series, beginning with 26, 6 is subtracted from each number to arrive at the next. 49. c. This is an alternating addition series that adds 5, then 2, then 5, and so on. 50. d. In this simple subtraction with repetition series, each number is repeated, then 3 is subtracted to give the next number, which is then repeated, and so on. 51. b. Here, there are two alternating patterns, with every other number following a different pattern. The first pattern begins with 13 and adds 2 to each number to arrive at the next; the alternating pattern begins with 29 and subtracts 3 each time.

52. c. Here, every third number follows a different pattern from the main series. In the main series, beginning with 16, 10 is added to each number to arrive at the next. In the alternating series, beginning with 56, 12 is added to each number to arrive at the next. 53. a. This is an alternating addition series with repetition, in which a random number, 66, is interpolated as every third number. The regular series adds 2, then 3, then 2, and so on, with 66 repeated after each “add 2” step. 54. c. This is an alternating addition series, with a random number, 35, interpolated as every third number. The pattern of addition is to add 2, add 5, add 2, and so on. The number 35 comes after each “add 2” step. 55. e. This is an alternating subtraction series, which subtracts 5, then 2, then 5, and so on. 56. c. This is an alternating subtraction series in which 2 is subtracted twice, then 3 is subtracted once, then 2 is subtracted twice, and so on. 57. a. This is a simple addition series with repetition. It adds 3 to each number to arrive at the next, which is repeated before 3 is added again. 58. c. Here, there are two alternating patterns. The first begins with 17 and adds 2; the second begins with 32 and subtracts 3. 59. a. Two patterns alternate here. The first pattern begins with 10 and adds 2 to each number to arrive at the next; the alternating pattern begins with 34 and subtracts 3 each time. 60. a. This is an alternating repetition series. The number 32 alternates with a series in which each number decreases by 2.

102

GovernmentAdda.com – ANSWERS –



Set 4

(Page 8)

61. b. This is a simple alternating addition and subtraction series. The first series begins with 8 and adds 3; the second begins with 43 and subtracts 2. 62. d. In this simple addition with repetition series, each number in the series repeats itself, and then increases by 12 to arrive at the next number. 63. b. This is a simple subtraction series in which a random number, 93, is interpolated as every third number. In the subtraction series, 10 is subtracted from each number to arrive at the next. 64. a. Two series alternate here, with every third number following a different pattern. In the main series, 3 is added to each number to arrive at the next. In the alternating series, 5 is subtracted from each number to arrive at the next. 65. d. This series alternates the addition of 4 with the subtraction of 3.

66. a. In this series, 5 is added to the previous number; the number 70 is inserted as every third number. 67. d. This is an alternating division and addition series: First, divide by 2, and then add 8. 68. c. This is a simple multiplication series. Each number is 2 times greater than the previous number. 69. b. This is a multiplication series; each number is 3 times the previous number. 70. a. In this series, the letters progress by 1; the numbers decrease by 3. 71. b. In this series, the letters progress by 2, and the numbers increase by 2. 72. c. The letters decrease by 1; the numbers are multiplied by 2. 73. d. This is a simple addition series; each number is 3 more than the previous number. 74. c. This is a simple subtraction series; each number is 4 less than the previous number. 75. b. This is an alternating addition and subtraction series. Roman numbers alternate with Arabic numbers. In the Roman numeral pattern, each number decreases by 1. In the Arabic numeral pattern, each number increases by 1.

103

GovernmentAdda.com – ANSWERS –



Set 5

(Page 10)

76. a. This series consists of letters in a reverse alphabetical order. 77. b. This is an alternating series in alphabetical order. The middle letters follow the order ABCDE. The first and third letters are alphabetical beginning with J. The third letter is repeated as a first letter in each subsequent three-letter segment. 78. b. Because the letters are the same, concentrate on the number series, which is a simple 2, 3, 4, 5, 6 series, and follows each letter in order. 79. d. The second and forth letters in the series, L and A, are static. The first and third letters consist of an alphabetical order beginning with the letter E. 80. c. The first two letters, PQ, are static. The third letter is in alphabetical order, beginning with R. The number series is in descending order beginning with 5. 81. c. The first letters are in alphabetical order with a letter skipped in between each segment: C, E, G, I, K. The second and third letters are repeated; they are also in order with a skipped letter: M, O, Q, S, U.

82. a. In this series, the third letter is repeated as the first letter of the next segment. The middle letter, A, remains static. The third letters are in alphabetical order, beginning with R. 83. d. In this series, the letters remain the same: DEF. The subscript numbers follow this series: 1,1,1; 1,1,2; 1,2,2; 2,2,2; 2,2,3. 84. c. There are two alphabetical series here. The first series is with the first letters only: STUVW. The second series involves the remaining letters: CD, EF, GH, IJ, KL. 85. a. The middle letters are static, so concentrate on the first and third letters. The series involves an alphabetical order with a reversal of the letters. The first letters are in alphabetical order: F, G, H, I, J. The second and fourth segments are reversals of the first and third segments. The missing segment begins with a new letter. 86. a. This series consists of a simple alphabetical order with the first two letters of all segments: B, C, D, E, F, G, H, I, J, K. The third letter of each segment is a repetition of the first letter. 87. d. There are three series to look for here. The first letters are alphabetical in reverse: Z, Y, X, W, V. The second letters are in alphabetical order, beginning with A. The number series is as follows: 5, 4, 6, 3, 7.

104

GovernmentAdda.com – ANSWERS –



Set 6

(Page 11)

88. b. Look at each segment. In the first segment, the arrows are both pointing to the right. In the second segment, the first arrow is up and the second is down. The third segment repeats the first segment. In the fourth segment, the arrows are up and then down. Because this is an alternating series, the two arrows pointing right will be repeated, so option b is the only possible choice. 89. b. Notice that in each segment, the figures are all the same shape, but the one in the middle is larger than the two on either side. Also, notice that one of the figures is shaded and that this shading alternates first right and then left. To continue this pattern in the third segment, you will look for a square. Choice b is correct because this choice will put the large square between the two smaller squares, with the shading on the right. 90. c. This is an alternating series. In the first segment, the letter “E” faces right, then down, then right. In the second segment, the letters all face down. To follow this pattern, in the fourth segment, the letters must all face up. 91. c. In this series, the shaded part inside the circle gets larger and then smaller. 92. d. Look for opposites in this series of figures. The first and second segments are opposites of each other. The same is true for the third and fourth segments. 93. a. Look carefully at the number of dots in each domino. The first segment goes from five to three to one. The second segment goes from one to three to five. The third segment repeats the first segment. 94. c. All four segments use the same figures: two squares, one circle, and one triangle. In the first segment, the squares are on the outside of the circle and triangle. In the second segment, the squares are below the other two. In the third segment, the squares on are the

inside. In the fourth segment, the squares are above the triangle and circle. 95. a. Look at each segment. You will notice that in each, the figure on the right and the figure on the left are the same; the figure in between is different. To continue this pattern in the last segment, the diamond on the left will be repeated on the right. Choice a is the only possible answer. 96. b. Each arrow in this continuing series moves a few degrees in a clockwise direction. Think of these arrows as the big hand on a clock. The first arrow is at noon. The last arrow before the blank would be 12:40. Choice b, the correct answer, is at 12:45. 97. c. Study the pattern carefully. In the first segment, two letters face right and the next two face left. The first letter in the second segment repeats the last letter of the previous segment. The same is true for the third segment. But the forth segment changes again; it is the opposite of the first segment, so the last two letters must face right. 98. d. This sequence concerns the number of sides on each figure. In the first segment, the three figures have one side, and then two sides, and then three sides. In the second segment, the number of sides increases and then decreases. In the third segment, the number of sides continues to decrease. 99. a. In this series, the figures increase the amount of shading by one-fourth and, once a square is completely shaded, starts over with an unshaded square. In the second segment, you will notice that the figure goes from completely shaded to completely unshaded. This is why choice a is the correct choice. 100. d. This is an alternating series. The first and third segments are repeated. The second segment is simply upside down. 101. d. In each of the segments, the figures alternate between one-half and one-fourth shaded.

105

GovernmentAdda.com – ANSWERS –



Set 7

(Page 13)

102. c. A leopard, cougar, and lion all belong to the cat family; an elephant does not. 103. b. The couch, table, and chair are pieces of furniture; the rug is not. 104. a. The yarn, twine, and cord are all used for tying. The tape is not used in the same way. 105. b. The guitar, violin, and cello are stringed instruments; the flute is a wind instrument. 106. c. Tulip, rose, and daisy are all types of flowers. A bud is not. 107. d. Tire, steering wheel, and engine are all parts of a car. 108. d. Parsley, basil, and dill are types of herbs. Mayonnaise is not an herb. 109. b. A branch, leaf, and root are all parts of a tree. The dirt underneath is not a part of the tree. 110. d. The first three choices are all synonyms.

111. a. An index, glossary, and chapter are all parts of a book. Choice a does not belong because the book is the whole, not a part. 112. c. The noun, preposition, and adverb are classes of words that make up a sentence. Punctuation belongs in a sentence, but punctuation is not a class of word. 113. d. The cornea, retina, and pupil are all parts of the eye. 114. d. Rye, sourdough, and pumpernickel are types of bread. A loaf is not a bread type. 115. b. An ounce measures weight; the other choices measure length. 116. a. Freeway, interstate, and expressway are all highspeed highways; a street is for low-speed traffic. 117. b. Dodge, duck, and avoid are all synonyms meaning evade. Flee means to run away from. 118. c. Heading, body, and closing are all parts of a letter; the letter is the whole, not a part.

106

GovernmentAdda.com – ANSWERS –



Set 8

(Page 15)

119.d. The core, seeds, and pulp are all parts of an apple. A slice would be a piece taken out of an apple. 120.b. Unique, rare, and exceptional are all synonyms. Beautiful has a different meaning. 121. c. Biology, chemistry, and zoology are all branches of science. Theology is the study of religion. 122. a. A circle, oval, and sphere are all circular shapes with no angles. A triangle is a different kind of shape with angles and three straight sides. 123. a. Flourish, prosper, and thrive are all synonyms; excite does not mean the same thing. 124.d. Evaluate, assess, and appraise are all synonyms; instruct does not mean the same thing. 125. a. The lobster, crab, and shrimp are all types of crustaceans; an eel is a fish. 126. c. The scythe, knife, and saw are all cutting tools. Pliers are tools, but they are not used for cutting. 127.b. Two, six, and eight are all even numbers; three is an odd number.

128. c. A peninsula, island, and cape are all landforms; a bay is a body of water. 129. c. Seat, rung, and leg are all parts of a chair. Not all chairs have cushions. 130.d. Fair, just, and equitable are all synonyms meaning impartial. Favorable means expressing approval. 131. c. Defendant, prosecutor, and judge are all persons involved in a trial. A trial is not a person. 132.b. Area, circumference, and quadrilateral are all terms used in the study of geometry. Variable is a term generally used in the study of algebra. 133.b. The mayor, governor, and senator are all persons elected to government offices; the lawyer is not an elected official. 134.d. Acute, right, and obtuse are geometric terms describing particular angles. Parallel refers to two lines that never intersect. 135. c. The wing, fin, and rudder are all parts of an airplane. 136. a. The heart, liver, and stomach are all organs of the body. The aorta is an artery, not an organ.

107

GovernmentAdda.com – ANSWERS –



Set 9

(Page 17)

137.b. The necessary part of a book is its pages; there is no book without pages. Not all books are fiction (choice a), and not all books have pictures (choice c). Learning (choice d) may or may not take place with a book. 138.d. A guitar does not exist without strings, so strings are an essential part of a guitar. A band is not necessary to a guitar (choice a). Guitar playing can be learned without a teacher (choice b). Songs are byproducts of a guitar (choice c). 139. a. All shoes have a sole of some sort. Not all shoes are made of leather (choice b); nor do they all have laces (choice c). Walking (choice d) is not essential to a shoe. 140. c. A person or animal must take in oxygen for respiration to occur. A mouth (choice a) is not essential because breathing can occur through the nose. Choices b and d are clearly not essential and can be ruled out. 141.b. An election does not exist without voters. The election of a president (choice a) is a byproduct. Not all elections are held in November (choice c), nor are they nationwide (choice d). 142.d. A diploma is awarded at graduation, so graduation is essential to obtaining a diploma. Employment may be a byproduct (choice c). A principal and a curriculum (choices a and b) may play a role in the awarding of some diplomas, but they are not essential. 143. c. Water is essential for swimming—without water, there is no swimming. The other choices are things that may or may not be present.

144. a. Without students, a school cannot exist; therefore, students are the essential part of schools. The other choices may be related, but they are not essential. 145.d. Words are a necessary part of language. Slang is not necessary to language (choice b). Not all languages are written (choice c). Words do not have to be spoken in order to be part of a language (choice a). 146.b. A desert is an arid tract of land. Not all deserts are flat (choice d). Not all deserts have cacti or oases (choices a and c). 147. a. Lightning is produced from a discharge of electricity, so electricity is essential. Thunder and rain are not essential to the production of lightning (choices b and d). Brightness may be a byproduct of lightning, but it is not essential (choice c). 148.b. The essential part of a monopoly is that it involves exclusive ownership or control. 149.d. To harvest something, one must have a crop, which is the essential element for this item. Autumn (choice a) is not the only time crops are harvested. There may not be enough of a crop to stockpile (choice b), and you can harvest crops without a tractor (choice c). 150. a. A gala indicates a celebration, the necessary element here. A tuxedo (choice b) is not required garb at a gala, nor is an appetizer (choice c). A gala may be held without the benefit of anyone speaking (choice d). 151.d. Pain is suffering or hurt, so choice d is the essential element. Without hurt, there is no pain. A cut (choice a) or a burn (choice b) may cause pain, but so do many other types of injury. A nuisance (choice c) is an annoyance that may or may not cause pain.

108

GovernmentAdda.com – ANSWERS –



Set 10

(Page 19)

152. c. An infirmary is a place that takes care of the infirm, sick, or injured. Without patients, there is no infirmary. Surgery (choice a) may not be required for patients. A disease (choice b) is not necessary because the infirmary may only see patients with injuries. A receptionist (choice d) would be helpful but not essential. 153.b. A facsimile must involve an image of some sort. The image or facsimile need not, however, be a picture (choice a). A mimeograph and a copier machine (choices c and d) are just a two of the ways that images may be produced, so they do not qualify as the essential element for this item. 154.b. A domicile is a legal residence, so dwelling is the essential component for this item. You do not need a tenant (choice a) in the domicile, nor do you need a kitchen (choice c). A house (choice d) is just one form of a domicile (which could also be a tent, hogan, van, camper, motor home, apartment, dormitory, etc.). 155.d. A culture is the behavior pattern of a particular population, so customs are the essential element. A culture may or may not be civil or educated (choices a and b). A culture may be an agricultural society (choice c), but this is not the essential element. 156. a. A bonus is something given or paid beyond what is usual or expected, so reward is the essential element. A bonus may not involve a raise in pay or cash (choices b and c), and it may be received from someone other than an employer (choice d). 157. c. An antique is something that belongs to, or was made in, an earlier period. It may or may not be a rarity (choice a), and it does not have to be an artifact, an object produced or shaped by human craft (choice b). An antique is old but does not have to be prehistoric (choice d).

158.b. An itinerary is a proposed route of a journey. A map (choice a) is not necessary to have a planned route. Travel (choice c) is usually the outcome of an itinerary, but not always. A guidebook (choice d) may be used to plan the journey but is not essential. 159. c. An orchestra is a large group of musicians, so musicians are essential. Although many orchestras have violin sections, violins aren’t essential to an orchestra (choice a). Neither a stage (choice b) nor a soloist (choice d) is necessary. 160.d. Knowledge is understanding gained through experience or study, so learning is the essential element. A school (choice a) is not necessary for learning or knowledge to take place, nor is a teacher or a textbook (choices b and c). 161.d. A dimension is a measure of spatial content. A compass (choice a) and ruler (choice b) may help determine the dimension, but other instruments may also be used, so these are not the essential element here. An inch (choice c) is only one way to determine a dimension. 162. a. Sustenance is something, especially food, that sustains life or health, so nourishment is the essential element. Water and grains (choices b and c) are components of nourishment, but other things can be taken in as well. A menu (choice d) may present a list of foods, but it is not essential to sustenance. 163. c. An ovation is prolonged, enthusiastic applause, so applause is necessary to an ovation. An outburst (choice a) may take place during an ovation; “bravo” (choice b) may or may not be uttered; and an encore (choice d) would take place after an ovation. 164. a. All vertebrates have a backbone. Reptiles (choice b) are vertebrates, but so are many other animals. Mammals (choice c) are vertebrates, but so are birds and reptiles. All vertebrates (choice d) are animals, but not all animals are vertebrates.

109

GovernmentAdda.com – ANSWERS –

165.b. Provisions imply the general supplies needed, so choice b is the essential element. The other choices are byproducts, but they are not essential.

110

166.d. A purchase is an acquisition of something. A purchase may be made by trade (choice a) or with money (choice b), so those are not essential elements. A bank (choice c) may or may not be involved in a purchase.

GovernmentAdda.com – ANSWERS –



Set 11

174. a. A cage is meant to keep something surrounded, so enclosure is the essential element. A prisoner (choice b) or an animal (choice c) are two things that may be kept in cages, among many other things. A zoo (choice d) is only one place that has cages. 175.b. A directory is a listing of names or things, so choice b is the essential element. A telephone (choice a) often has a directory associated with it, but it is not essential. A computer (choice c) uses a directory format to list files, but it is not required. Names (choice d) are often listed in a directory, but many other things are listed in directories, so this is not the essential element. 176. a. An agreement is necessary to have a contract. A contract may appear on a document (choice b), but it is not required. A contract may be oral as well as written, so choice c is not essential. A contract can be made without an attorney (choice d). 177.b. A saddle is something one uses to sit on an animal, so it must have a seat (choice b). A saddle is often used on a horse (choice a), but it may be used on other animals. Stirrups (choice c) are often found on a saddle but may not be used. A horn (choice d) is found on Western saddles, but not English saddles, so it is not the essential element here. 178. a. Something cannot vibrate without creating motion, so motion is essential to vibration. 179.b. The essential part of a cell is its nucleus. Not all cells produce chlorophyll (choice a). Not all cells are nerve cells (choice c). All living things, not just humans (choice d), have cells. 180. c. Without a first-place win, there is no champion, so winning is essential. There may be champions in running, swimming, or speaking, but there are also champions in many other areas.

(Page 21)

167. a. A dome is a large rounded roof or ceiling, so being rounded is essential to a dome. A geodesic dome (choice b) is only one type of dome. Some, but not all domes, have copper roofs (choice d). Domes are often found on government buildings (choice c), but domes exist in many other places. 168.b. A recipe is a list of directions to make something. Recipes may be used to prepare desserts (choice a), among other things. One does not need a cookbook (choice c) to have a recipe, and utensils (choice d) may or may not be used to make a recipe. 169.d. A hurricane cannot exist without wind. A beach is not essential to a hurricane (choice a). A hurricane is a type of cyclone, which rules out choice b. Not all hurricanes cause damage (choice c). 170. c. Without a signature, there is no autograph. Athletes and actors (choices a and b) may sign autographs, but they are not essential. An autograph can be signed with something other than a pen (choice d). 171. a. Residents must be present in order to have a town. A town may be too small to have skyscrapers (choice b). A town may or may not have parks (choice c) and libraries (choice d), so they are not the essential elements. 172.d. A wedding results in a joining, or a marriage, so choice d is the essential element. Love (choice a) usually precedes a wedding, but it is not essential. A wedding may take place anywhere, so a church (choice b) is not required. A ring (choice c) is often used in a wedding, but it is not necessary. 173. c. A faculty consists of a group of teachers and cannot exist without them. The faculty may work in buildings (choice a), but the buildings aren’t essential. They may use textbooks (choice b) and attend meetings (choice d), but these aren’t essential either. 111

GovernmentAdda.com – ANSWERS –

181.d. A glacier is a large mass of ice and cannot exist without it. A glacier can move down a mountain, but it can also move across a valley or a plain, which rules out choice a. Glaciers exist in all seasons, which rules out choice b. There are many glaciers in the world today, which rules out choice c.

112

GovernmentAdda.com – ANSWERS –



Set 12

(Page 23)

182.b. Coffee goes into a cup and soup goes into a bowl. Choices a and c are incorrect because they are other utensils. The answer is not choice d because the word food is too general. 183.d. A gym is a place where people exercise. A restaurant is a place where people eat. Food (choice a) is not the answer because it is something people eat, not a place or location where they eat. The answer is not choice b or c because neither represents a place where people eat. 184. c. An oar puts a rowboat into motion. A foot puts a skateboard into motion. The answer is not choice a because running is not an object that is put into motion by a foot. Sneaker (choice b) is incorrect because it is something worn on a foot. Jumping (choice d) is incorrect because although you do need feet to jump, jumping is not an object that is put into motion by means of a foot. 185.d. A window is made up of panes, and a book is made up of pages. The answer is not choice a because a novel is a type of book. The answer is not choice b because glass has no relationship to a book. Choice c is incorrect because a cover is only one part of a book; a book is not made up of covers. 186. c. Secretly is the opposite of openly, and silently is the opposite of noisily. Choices a and b are clearly not the opposites of silently. Choice d means the same thing as silently. 187.b. An artist makes paintings; a senator makes laws. The answer is not choice a because an attorney does not make laws and a senator is not an attorney. Choice c is incorrect because a senator is a politician. Constituents (choice d) is also incorrect because a senator serves his or her constituents.

113

188.b. An actor performs in a play. A musician performs at a concert. Choices a, c, and d are incorrect because none is people who perform. 189. a. Careful and cautious are synonyms (they mean the same thing). Boastful and arrogant are also synonyms. The answer is not choice b because humble means the opposite of boastful. The answer is not choice c or d because neither means the same as boastful. 190.d. A group of lions is called a pride. A group of fish swim in a school. Teacher (choice a) and student (choice b) refer to another meaning of the word school. The answer is not choice c because self-respect has no obvious relationship to this particular meaning of school. 191. a. Guide and direct are synonyms, and reduce and decrease are synonyms. The answer is not choice b or d because neither means the same as reduce. Choice c is incorrect because increase is the opposite of reduce. 192.b. A yard is a larger measure than an inch (a yard contains 36 inches). A quart is a larger measure than an ounce (a quart contains 32 ounces). Gallon (choice a) is incorrect because it is larger than a quart. Choices c and d are incorrect because they are not units of measurement. 193. c. A lizard is a type of reptile; a daisy is a type of flower. Choices a and b are incorrect because a petal and a stem are parts of a flower, not types of flowers. Choice d is incorrect because an alligator is another type of reptile, not a type of flower. 194.b. Elated is the opposite of despondent; enlightened is the opposite of ignorant. 195.d. A marathon is a long race and hibernation is a lengthy period of sleep. The answer is not choice a or b because even though a bear and winter are related to hibernation, neither completes the analogy. Choice c is incorrect because sleep and dream are not synonymous.

GovernmentAdda.com – ANSWERS –

196. a. If someone has been humiliated, they have been greatly embarrassed. If someone is terrified, they are extremely frightened. The answer is not choice b because an agitated person is not necessarily frightened. Choices c and d are incorrect because neither word expresses a state of being frightened. 197.d. An odometer is an instrument used to measure mileage. A compass is an instrument used to determine direction. Choices a, b, and c are incorrect because none is an instrument. 198. a. An optimist is a person whose outlook is cheerful. A pessimist is a person whose outlook is gloomy. The answer is not choice b because a pessimist does not have to be mean. Choices c and d are incorrect because neither adjective describes the outlook of a pessimist.

114

199. c. A sponge is a porous material. Rubber is an elastic material. Choice a is incorrect because rubber would not generally be referred to as massive. The answer is not choice b because even though rubber is a solid, its most noticeable characteristic is its elasticity. Choice d is incorrect because rubber has flexibility. 200.d. Candid and indirect refer to opposing traits. Honest and untruthful refer to opposing traits. The answer is not choice a because frank means the same thing as candid. Wicked (choice b) is incorrect because even though it refers to a negative trait, it does not mean the opposite of honest. Choice c is incorrect because truthful and honest mean the same thing. 201.d. A pen is a tool used by a poet. A needle is a tool used by a tailor. The answer is not choice a, b, or c because none is a person and therefore cannot complete the analogy.

GovernmentAdda.com – ANSWERS –



Set 13

(Page 25)

202.d. A can of paint is to a paintbrush as a spool of thread is to a sewing needle. This is a relationship of function. Both show the tool needed to perform a task. 203. a. Grapes are to a pear as cheese is to butter. This relationship shows the grouping or category to which something belongs. Grapes and pears are fruit; cheese and butter are both dairy products. 204.d. An oar is to a canoe as a steering wheel is to a car. This is a functional relationship. The oar helps steer the canoe in the way that the steering wheel steers the car. 205. a. Cup is to bowl as vacuum cleaner is to broom. This is another relationship about function. The cup and bowl are both used for eating. The vacuum cleaner and broom are both used for cleaning. 206.d. Sheep are to sweater as pine trees are to log cabin. Wool comes from the sheep to make a sweater; wood comes from the trees to make the log cabin. 207. a. Hand is to ring as head is to cap. A ring is worn on a person’s hand; a cap is worn on a person’s head. 208. b. A palm tree is to a pine tree as a bathing suit is to a parka. This relationship shows an opposite— warm to cold. Palm trees grow in warm climates and pine trees grow in cold climates. Bathing suits are worn in warm weather; parkas are worn in cold weather. 209.d. Batteries are to a flashlight as telephone wires are to a telephone. The batteries provide power to the flashlight; the wires send power to the telephone. 210.d. A fish is to a dragonfly as a chicken is to corn. Fish eat insects; chickens eat corn. 211. a. A telephone is to a stamped letter as an airplane is to a bus. A telephone and letter are both forms of communication. An airplane and bus are both forms of transportation. 115

212. c. A trapeze performer is to a clown as swings are to a sliding board. This relationship shows a classification. Trapeze performers and clowns are found at circuses; swings and sliding boards are found on playgrounds. 213. c. Camera is to photograph as teakettle is to a cup of tea. The camera is used to make the photo; the teakettle is used to make the tea. 214.b. Hat and mittens are to desert as snorkel and flippers are to snow. This relationship shows an opposition. The hat and mittens are NOT worn in the desert; the snorkel and flippers are NOT worn in the snow. 215.d. Car is to horse and buggy as computer is to pen and ink. This relationship shows the difference between modern times and times past. 216. c. Leather boots are to cow as pearl necklace is to oyster. The leather to make the boots comes from a cow; the pearls to make the necklace come from oysters. 217.b. A toddler is to an adult as a caterpillar is to a butterfly. This relationship shows the young and the adult. The caterpillar is an early stage of the adult butterfly. 218.b. Towel is to bathtub as chest of drawers is to bed. The towel and bathtub are both found in a bathroom; the chest and the bed are both found in a bedroom. 219. a. A snow-capped mountain is to a crocodile as a cactus is to a starfish. This relationship shows an opposition. The crocodile does NOT belong on the mountain; the starfish does NOT belong in the desert. 220. c. A shirt is to a button as a belt is to a belt buckle. A button is used to close a shirt; a belt buckle is used to close a belt. 221. c. A penny is to a dollar as a small house is to a skyscraper. This relationship shows smaller to larger. A penny is much smaller than a dollar; a house is much smaller than a skyscraper.

GovernmentAdda.com – ANSWERS –



Set 14

(Page 31)

222.b. Guitar is to horn as hammer is to saw. This relationship is about grouping. The guitar and horn are musical instruments. The hammer and saw are carpentry tools. 223.d. Tree is to leaf as bird is to feather. This relationship shows part to whole. The leaf is a part of the tree; the feather is a part of the bird. 224. c. House is to tent as truck is to wagon. The house is a more sophisticated form of shelter than the tent; the truck is a more sophisticated mode of transportation than the wagon. 225. c. Scissors is to knife as pitcher is to watering can. This relationship is about function. The scissors and knife are both used for cutting. The pitcher and watering can are both used for watering. 226.b. A T-shirt is to a pair of shoes as a chest of drawers is to a couch. The relationship shows to which group something belongs. The T-shirt and shoes are both articles of clothing; the chest and couch are both pieces of furniture. 227.d. A bookshelf is to a book as a refrigerator is to a carton of milk. The book is placed on a bookshelf; the milk is placed in a refrigerator. 228.d. A squirrel is to an acorn as a bird is to a worm. A squirrel eats acorns; a bird eats worms. 229.b. An eye is to a pair of binoculars as a mouth is to a microphone. This relationship shows magnification. The binoculars help one see farther. The microphone helps one speak louder. 230. a. Knitting needles are to sweater as a computer is to a report. This relationship shows the tool needed to make a product. The knitting needles are used to create the sweater; the computer is used to write a report. 231.b. Bread is to knife as log is to ax. This relationship shows function. The knife cuts the bread; the ax chops the log.

116

232.b. Closet is to shirt as kitchen cabinets are to cans of food. The shirt is stored in the closet; the food is stored in the cabinets. 233. a. Pyramid is to triangle as cube is to square. This relationship shows dimension. The triangle shows one dimension of the pyramid; the square is one dimension of the cube. 234. c. Toothbrush is to toothpaste as butter knife is to butter. This relationship shows function. The toothbrush is used to apply the toothpaste to teeth; the knife is used to apply butter to bread. 235. c. Fly is to ant as snake is to lizard. The fly and ant are both insects; the snake and lizard are both reptiles. 236. a. Sail is to sailboat as pedal is to bicycle. The sail makes the sailboat move; the pedal makes the bicycle move. 237.d. Hose is to firefighter as needle is to nurse. This relationship shows the tools of the trade. A hose is a tool used by a firefighter; a needle is a tool used by a nurse. 238. c. A U.S. flag is to a fireworks display as a Halloween mask is to a pumpkin. This relationship shows symbols. The flag and fireworks are symbols of the Fourth of July. The mask and pumpkin are symbols of Halloween. 239.d. Newspaper is to book as trumpet is to banjo. The newspaper and book are to read; the trumpet and banjo are musical instruments to play. 240.b. Dishes are to kitchen sink as car is to hose. Dishes are cleaned in the sink; the car is cleaned with the hose. 241. a. The United States is to the world as a brick is to a brick house. This relationship shows part to whole. The United States is one part of the world; the brick is one part of the house.

GovernmentAdda.com – ANSWERS –



Set 15

(Page 37)

242.b. The three above the line are all insects. The hamster and squirrel are rodents, so the correct choice is b because the mouse is also a rodent. The other three choices are not rodents. 243. a. In the relationship above the line, the saw and the nails are tools a carpenter uses. In the relationship below the line, the stethoscope and thermometer are tools a pediatrician uses. 244. c. A table made of wood could come from an oak tree. A shirt made of cloth could come from a cotton plant. Choice a looks like a reasonable answer if you apply the same sentence: “A shirt made of cloth could come from sewing.” But this is not the same relationship as the one above the line. The oak and the cotton are both materials used to make the table and the shirt. 245.d. The words above the line show a continuum: Command is more extreme than rule, and dictate is more extreme than command. Below the line, the continuum is as follows: Sleep is more than doze, and hibernate is more than sleep. The other choices are not related in the same way. 246. a. A banquet and a feast are both large meals; a palace and a mansion are both large places of shelter. 247.b. A fence and a wall mark a boundary. A path and an alley mark a passageway. 248. c. The objects above the line are all things used by an artist. The objects below the line are all things used by a teacher. 249.b. The relationship above the line is that snow on a mountain creates conditions for skiing. Below the line, the relationship is that warmth at a lake creates conditions for swimming.

117

250.d. Above the line, the relationship shows a progression of sources of light. The relationship below the line shows a progression of types of housing, from smallest to largest. Choice a is incorrect because a tent is smaller than a house. Choices b and c are wrong because they are not part of the progression. 251. a. The relationship above the line is as follows; apples are a kind of fruit; fruit is sold in a supermarket. Below the line, the relationship is: a novel is a kind of book; books are sold in a bookstore. 252.d. The tadpole is a young frog; frogs are amphibians. The lamb is a young sheep; sheep are mammals. Animal (choice a) is incorrect because it is too large a grouping: Animals include insects, birds, mammals, reptiles, and amphibians. Choices b and c are incorrect because they are not part of the progression. 253.b. Walk, skip, and run represent a continuum of movement: Skipping is faster than walking; running is faster than skipping. Below the line, the continuum is about throwing: Pitch is faster than toss; hurl is faster than pitch. 254. c. The honeybee, angel, and bat all have wings; they are capable of flying. The kangaroo, rabbit, and grasshopper are all capable of hopping. 255. a. Above the line, the relationship is as follows: A daisy is a type of flower, and a flower is a type of plant. Below the line, the relationship is as follows: A bungalow is a type of house, and a house is a type of building.

GovernmentAdda.com – ANSWERS –



Set 16

(Page 39)

256.b. A petal is a part of a flower; a tire is a part of a bicycle. 257.d. A bristle is a part of a brush; a key is a part of a piano. 258. a. A group of fish is a school; a group of wolves is a pack. 259. a. An odometer measures distance; a scale measures weight. 260.d. Siamese is a kind of cat; romaine is a kind of lettuce. 261. e. A pedal propels a bicycle; an oar propels a canoe. 262. c. Pulsate and throb are synonyms, as are examine and scrutinize. 263. c. An elephant is a pachyderm; a kangaroo is a marsupial. 264. e. Depressed is an intensification of sad; exhausted is an intensification of tired. 265. a. A psychologist treats a neurosis; an ophthalmologist treats a cataract.

118

266. e. A binding surrounds a book; a frame surrounds a picture. 267.b. One explores to discover; one researches to learn. 268. c. Upon harvesting, cotton is gathered into bales; grain is gathered into shocks. 269. a. Division and section are synonyms; layer and tier are synonyms. 270. a. Pastoral describes rural areas; metropolitan describes urban areas. 271.d. A waitress works in a restaurant; a teacher works in a school. 272. c. A finch is a type of bird; a Dalmatian is a type of dog. 273. e. To drizzle is to rain slowly; to jog is to run slowly. 274. c. A skein is a quantity of yarn; a ream is a quantity of paper. 275.b. To tailor a suit is to alter it; to edit a manuscript is to alter it.

GovernmentAdda.com – ANSWERS –



Set 17

(Page 41)

276.d. A conductor leads an orchestra; a skipper leads a crew. 277. a. Jaundice is an indication of a liver problem; rash is an indication of a skin problem. 278.b. A cobbler makes and repairs shoes; a contractor builds and repairs buildings. 279. e. To be phobic is to be extremely fearful; to be asinine is to be extremely silly. 280. c. Obsession is a greater degree of interest; fantasy is a greater degree of dream. 281.d. Devotion is characteristic of a monk; wanderlust is characteristic of a rover. 282. e. Slapstick results in laughter; horror results in fear. 283.b. Verve and enthusiasm are synonyms; devotion and reverence are synonyms. 284. c. A cacophony is an unpleasant sound; a stench is an unpleasant smell. 285. a. A conviction results in incarceration; a reduction results in diminution.

119

286. a. The deltoid is a muscle; the radius is a bone. 287.d. Umbrage and offense are synonyms; elation and jubilance are synonyms. 288.b. Being erudite is a trait of a professor; being imaginative is a trait of an inventor. 289.d. Dependable and capricious are antonyms; capable and inept are antonyms. 290. a. A palm (tree) has fronds; a porcupine has quills. 291. e. A metaphor is a symbol; an analogy is a comparison. 292.d. A dirge is a song used at a funeral; a jingle is a song used in a commercial. 293. e. Feral and tame are antonyms; ephemeral and immortal are antonyms. 294. a. A spy acts in a clandestine manner; an accountant acts in a meticulous manner. 295. c. Hegemony means dominance; autonomy means independence. 296. e. An aerie is where an eagle lives; a house is where a person lives.

GovernmentAdda.com – ANSWERS –



Set 18

(Page 42)

297. a. Grana means big; melke means tree; pini means little; hoon means house. Therefore, granahoon means big house. 298.b. Leli means yellow; broon means hat; pleka means flower; froti means garden; mix means salad. Therefore, lelipleka means yellow flower. 299.d. From wilkospadi, you can determine that wilko means bicicyle and spadi means race. Therefore, the first part of the word that means racecar should begin with spadi. That limits your choices to b and d. Choice b, spadiwilko, is incorrect because we have already determined that wilko means bicycle. Therefore, the answer must be choice d, spadivolo. 300. a. Dafta means advise; foni is the same as the suffix –ment; imo is the same as the prefix mis–; lokti means conduct. Since the only word in the answer choices that hasn’t been defined is krata, it is reasonable to assume that krata means state. Therefore, kratafoni is the only choice that could mean statement. 301. c. In this language, the adjective follows the noun. From dionot and blyonot, you can determine that onot means oak. From blyonot and blycrin, you can determine that bly means leaf. Therefore, crin means maple. Because the adjective maple comes after the noun, patricrin is the only possible choice. 302. c. In this language, the noun appears first and the adjectives follow. Since agnos means spider and should appear first, choices a and d can be ruled out. Choice b can be ruled out because delano means snake.

303. a. Myn means saddle; cabel means horse; cono means trail; and wir means ride. Therefore, cabelwir is the correct answer. 304. c. In this language, the adjective follows the noun. From godabim and romzbim, you can determine that bim means kidney. From romzbim and romzbako, you can determine that romz means beans. Therefore, bako means wax. Because the adjective wax must come after the noun in this language, wasibako is the only choice. 305.b. Tam means sky; ceno means blue; rax means cheese; apl means star; and mitl means bright. So, mitltam means bright sky. 306.d. Gorbl means fan; flur means belt; pixn means ceiling; arth means tile; and tusl means roof. Therefore, pixnarth is the correct choice. 307.d. Hapl means cloud; lesh means burst; srench means pin; och means ball; and resbo means nine. Leshsrench (choice a) doesn’t contain any of the words needed for cloud nine. We know that och means ball, so that rules out choices b and c. When you combine hapl (cloud) with resbo (nine), you get the correct answer. 308.d. Migen means cup; lasan means board; poen means walk; cuop means pull; and dansa means man. The only possible choices, then, are choices a and d. Choice a can be ruled out because migen means cup.

120

GovernmentAdda.com – ANSWERS –



Set 19

(Page 46)

309. c. Morpir means bird; quat means house; beel means blue; clak means bell. Choice c, which begins with quat, is the only possible option. 310.b. According to this language, slar means jump. The suffix –ing is represented by –y. Since choice b is the only one that ends in the letter y, this is the only possible option. 311.b. Brift means the root word mili–; the suffix amint means the same as the English suffix –tant; the root word ufton– means occupy; el means the suffix –ied of occupied; and alene means the suffix –tion. (Because ufton means occupy, choices a, c, and d can be easily ruled out.) 312. a. Krekin means work; blaf means force; drita means ground; and alti means place. Drita means ground, so that rules out choices b and d. Choice c isn’t correct because blaf means force. That leaves choice a as the only possible answer. 313.d. Pleka means fruit; paki means cake; shillen means walk; treft means butter; and alan means cup. Therefore, alanpaki means cupcake. 314.b. Pesl means basketball; ligen means court; strisi means room; olta means placement; and ganti means test. Because strisi means room, it must be present in the answer, so that rules out choice c. Choices a and d are incorrect because pesl means basketball and olta means placement. That leaves choice b as the only possible answer.

121

315. a. Jalka means happy; mofti means birthday; hoze means party; mento means good; and gunn means the suffix –ness. We know the answer must include the suffix –ness. The only choice that uses that suffix is choice a. 316.d. Mallon means blue; piml means light; tifl means berry; and arpan means “rasp” in raspberry. The word piml, which means light, is required for the word lighthouse. That rules out choices a and c. Arpan in choice b means “rasp,” so that rules out choice b. That leaves choice d the only possible answer. 317. a. Gemo means fair; linea means warning; geri means report; mitu means card; and gila means weather. Thus, gemogila is the correct choice. 318.d. Apta means first; ose means base; epta means second; larta means ball; and buk means park. Thus, oselarta means baseball. 319. c. In this language, the root word taga, which means care, follows the affix (relf, o–, or fer–). Therefore, in the word aftercare, the root word and the affix would be reversed in the artificial language. The only choice, then, is tagazen, because tagafer would mean less care. 320. a. Malga means peach; uper means cobbler; port means juice; mogga means apple; and grop means jelly. Therefore, moggaport means apple juice.

GovernmentAdda.com – ANSWERS –



Set 20

(Page 48)

321.b. Valerie signed a legally binding document that requires her to pay a monthly rent for her apartment and she has failed to do this for the last three months. Therefore, she has violated her apartment lease. 322. a. Jake damaged Leslie’s camera while it was in his possession and he has agreed to compensate Leslie for the cost of the repair. 323.d. This is the only situation in which someone makes an assumption that is not based on conclusive evidence. Choices a and c reflect situations in which assumptions are made based on evidence. In choice b, Mary is not assuming anything to be true. She is simply wishing that she’d made a different decision. 324.d. Choices a, b, and c do not describe situations in which a product is guaranteed. Only choice d reflects a situation in which a seller attests to the quality of a product by giving the buyer a promise or assurance about its quality. 325. c. Malcolm is the only person returning to a social system that he has been away from for an extended period of time. 326.b. The realtor is using a clear exaggeration when she states that a house which is eleven blocks away from the ocean is prime waterfront property. 327. c. Although the ski instructors at Top of the Peak Ski School do work seasonally, choice a does not describe anyone applying for seasonal employment. In choice b, the statement that Matthew likes to work outdoors tells us nothing about seasonal employment or someone applying for it. And although choice d describes a business with seasonal hours, it does not describe a person applying for seasonal work. Choice c, on the other hand, very specifically depicts a person, Lucinda, who is applying for a job as a summer waitress at a beach resort, which is dependent upon a particular season of the year.

328.b. After getting some good news, Jeremy and a few friends casually get together for a drink after work, thereby having an informal gathering. Choices a and c describe more formal types of gatherings. Choice d describes a chance or coincidental kind of meeting. 329. a. The fact that Jared is in scoring position due to his blooper indicates that he has hit the ball and is now a base runner; therefore, he has legally completed his time at bat. Choices b and c both describe situations in which a strike is called, but they do not state that the batter has been put out or that he is now a base runner. Choice d describes a situation in which the batter, Mario, is still at the plate waiting for the next pitch. 330. c. Although choices a and c both describe suspensions, only choice c describes a suspension that is the result of one of the two scenarios given in the definition of a five-day suspension (physical assault or destructing or defacing school property). Therefore, we can assume that Franny’s suspension, which is the result of spray painting school property, will be a fiveday suspension. Since the definition doesn’t provide any information about suspensions for cheating, we can assume that Lillian’s suspension does not fall into the five-day suspension category. 331.d. This is the only choice that indicates that an additional period of play is taking place to determine the winner of a game that ended in a tie. 332.b. Simone’s mother has taken legal steps to allow another person to act on her behalf. Therefore, this is the only choice that indicates that a power of attorney has been established. 333.d. Jeffrey’s recent behavior is clearly inconsistent and irregular.

122

GovernmentAdda.com – ANSWERS –

334. a. Although choice d also mentions a writer who has died, it does not state that one of the writer’s books was published after her death, only that she received an award. Choice a states that Richard wasn’t around to see the early reviews of his novel, therefore implying that Richard died before the book was published. The other two options depict living writers.

123

GovernmentAdda.com – ANSWERS –



Set 21

(Page 52)

335.b. Seeing four girls surrounding another girl, while in possession of her backpack, is the most suspicious of the incidents described. 336.b. The situation described indicates that Dr. Miller’s practice presents some specific challenges, namely that it is a busy environment with a child clientele. There is also some indication that even highly recommended, experienced hygienists might not be cut out for Dr. Miller’s office. There is nothing to suggest that Marilyn (choice a) or James (choice c) would be a good fit for Dr. Miller’s practice. Kathy (choice d) has experience and she is also interested in working with children. However, the fact that she hopes to become a preschool teacher in the not-too-distant future indicates that she might not be the kind of committed, long-term employee that Dr. Miller needs. Lindy (choice b), with her hands-on experience working with children as well as a degree from a prestigious dental hygiene program, is the most attractive candidate for the position based on the situation described. 337. c. The Treehouse Collection is the only package that can thrive in shady locations. Choice a requires a Northeastern climate. Choices b and d require bright sunlight. 338.d. Since Eileen’s husband does not enjoy fancy restaurants, choices a and c can be ruled out. Choice b, although casual, doesn’t sound as though it would be the kind of special and memorable evening that Eileen is looking for. Choice d, which is owned by a former baseball star and is described as “charming” and “reminiscent of a baseball clubhouse,” sounds perfect for Eileen’s husband, who is described as a baseball fan and a man with simple tastes. 339.b. This option is both near the center of town and in a location (near a school and an ice cream store) where children and their parents are sure

340. c.

341. a.

342. a.

343.d.

344. c.

345.b.

346. c.

124

to be around. This is the only option that meets both of Mark’s requirements. This is the only option that would encourage people to think of the bakery as a shop they would visit regularly and not just on special occasions. The four women seem to agree that the plate starts out with the letter J. Three of them agree that the plate ends with 12L. Three of them think that the second letter is X, and a different three think that the third letter is K. The plate description that has all of these common elements is a. All of the men agree that the first three numbers are 995. Three of them agree that the fourth number is 9. Three agree that the fifth number is 2. Three agree that the sixth number is 6; three others agree that the seventh number is also 6. Choice a is the best choice because it is made up of the numbers that most of the men agree they saw. Step 4 clearly states that the human resources representative should issue the new employee a temporary identification card. Step 2 of the guidelines states that the realtor should get background information about the client’s current living circumstances. Ms. Russo failed to do this. Actresses #2 and #3 possess most of the required traits. They both have red hair and brown eyes, are average-sized, and are in their forties. Actress #1 is very tall and is only in her mid-twenties. She also has an olive complexion. Actress #4 is of very slight build and is in her early thirties. She also has blue eyes. The solicitor described as #2 has a shaved head and is much taller and heavier than the solicitors described as #1 and #3. Therefore, choices a and d, which include #2, can be ruled out. Solicitors #1, #3, and #4 have such similar descriptions that the correct answer is clearly choice c.

GovernmentAdda.com – ANSWERS –



Set 22

(Page 57)

347. c. Since Erin’s parents think a dog would not be happy in an apartment, we can reasonably conclude that the family lives in an apartment. We do not know if Erin’s parents dislike dogs (choice a) or if Erin dislikes birds (choice b). There is no support for choice d. 348.d. It is reasonable to conclude that Mike likes singing and dancing because he looks forward to doing these things at music camp. There is no information that supports any of the other three choices. 349. c. Given the information presented, the only statement that could be considered true is that the fruit should not be eaten because it is poisonous. There is no support that taxol is poisonous or that taxol has cured anyone (choices a and b). There is no support for choice d. 350. a. Because Mr. Sanchez spends many hours during the weekend working in his vegetable garden, it is reasonable to suggest that he enjoys this work. There is no information to suggest that he does not like classical music. Although Mrs. Sanchez likes to cook, there is nothing that indicates she cooks vegetables (choice c). Mrs. Sanchez likes to read, but there is no information regarding the types of books she reads (choice d). 351.b. The passage tells us that Tim’s commute didn't bother him because he was always able to sit down and comfortably read or do paperwork. Therefore, it is reasonable to assume that Tim’s commute has become less comfortable since the schedule change, because it is very crowded and he can no longer find a seat. There is no information given that supports choices a, c, and d.

352.d. The first sentence makes this statement true. There is no support for choice a. The passage tells us that the spa vacation is more expensive than the island beach resort vacation, but that doesn’t necessarily mean that the spa is overpriced; therefore, choice b cannot be supported. And even though the paragraph says that the couple was relieved to find a room on short notice, there is no information to support choice c, which says that it is usually necessary to book at the spa at least six months in advance. 353.b. Since the seahorse populations have declined as a result of fishing, their populations will increase if seahorse fishing is banned. There is no support for any of the other choices. 354. a. The fact that Vincent and Thomas live on the same street indicates that they live in the same neighborhood. There is no support for any of the other choices. 355.d. If Georgia is older than Marsha and Bart is older than Georgia, then Marsha has to be the youngest of the three. Choice b is clearly wrong because Bart is the oldest. There is no information in the paragraph to support either choice a or choice c. 356. c. If there were seven shows left and five were sitcoms, this means that only two of the shows could possibly be dramas. Choices a and b may be true, but there is no evidence to indicate this as fact. The fact that all of the sitcoms remained does not necessarily mean that viewers prefer sitcoms (choice d). 357. c. Since the paragraph states that Marlee is the younger cousin, Sara must be older than Marlee. There is no information to support the other choices.

125

GovernmentAdda.com – ANSWERS –



Set 23

(Page 60)

358.b. Because the first two statements are true, Eric is the youngest of the three, so the third statement must be false. 359. c. Because the first two sentences are true, both Josh and Darren saw more movies than Stephen. However, it is uncertain as to whether Darren saw more movies than Josh. 360. c. The first two statements give information about Zoe’s tulips and pansies. Information about any other kinds of flowers cannot be determined. 361. a. Because the first two statements are true, raspberries are the most expensive of the three. 362. a. If no wall-to-wall carpeting is pink and all the offices have wall-to-wall carpeting, none of the offices has pink wall-to-wall carpeting. 363.b. From the first two statements, we know that of the three classes, Class A has the highest enrollment, so the third statement must be false.

364. a. According to the first two statements, Fido weighs the most and Boomer weighs the least. 365. c. Although all of the trees in the park are flowering trees, it cannot be determined by the information given whether all dogwoods are flowering trees. 366. a. Since the Gaslight Commons costs more than the Riverdale Manor and the Livingston Gate costs more than the Gaslight Commons, it is true that the Livingston Gate costs the most. 367. a. From the first two statements, you know that the Kingston Mall has the most stores, so the Kingston Mall would have more stores than the Four Corners Mall. 368.b. We know from the first two statements that Lily runs fastest. Therefore, the third statement must be false.

126

GovernmentAdda.com – ANSWERS –



Set 24

(Page 62)

369. a. From the first statement, we know that bran cereal has more fiber than both oat cereal and corn cereal. From the second statement, we know that rice cereal has less fiber than both corn and wheat cereals. Therefore, rice cereal has the least amount of fiber. 370. c. We only know that Jasmine weighs more than Jason. There is no way to tell whether Jasmine also weighs more than Jenna. 371. c. We know from the first two statements that Tuesday had the highest temperature, but we cannot know whether Monday’s temperature was higher than Tuesday’s. 372.b. Spot is bigger than King, and Ralph is bigger than Spot. Therefore, King must be smaller than Ralph. 373. a. There are fewer oranges than either apples or lemons, so the statement is true.

374.b. Because the first two statements are true, Rebecca’s house is also northeast of the Shop and Save Grocery, which means that the third statement is false. 375. a. Joe is younger than Kathy and older than Mark, so Mark must be younger than Kathy. 376. c. We know only that long-tailed Gangles have spots. We cannot know for certain if long-tailed Gangles also have short hair. 377. c. The first two statements indicate that Battery Y lasts the least amount of time, but it cannot be determined if Battery Z lasts longer than Battery X. 378.b. Given the information in the first two statements, Bryant is sitting in front of both Jerome and Martina, so the third statement must be false. 379.b. Because the first two statements are true, Penfield is west of Centerville and southwest of Middletown. Therefore, the third statement is false.

127

GovernmentAdda.com – ANSWERS –



Set 25

(Page 64)

380. c. Both the car and the train are quicker than the bus, but there is no way to make a comparison between the train and the car. 381. a. We know that there are Signots with buttons, or Lamels, and that there are yellow Signots, which have no buttons. Therefore, Lamels do not have buttons and cannot be yellow. 382. a. The market is one block west of the hotel. The drugstore is two blocks west of the hotel, so the drugstore is west of the market. 383. c. There is not enough information to verify the third statement. 384.b. Rulers are the most expensive item.

385.b. The first two statements indicate there are more yellow jelly beans than red and green. 386. c. Cloudy days are the most windy, but there is not enough information to compare the wind on the foggy days with the wind on the sunny days. 387. a. Of the three, the drugstore has the best selection of postcards. 388.b. This is the order of the cars from left to right: minivan, pickup, sedan, sport utility vehicle. 389. a. To the extent that a toothpick is useful, it has value.

128

GovernmentAdda.com – ANSWERS –



Set 26

(Page 66)

390. a. Since one-half of the four children are girls, two must be boys. It is not clear which children have blue or brown eyes. 391.d. All baseball caps have brims, since baseball caps are hats (Fact 3) and all hats have brims (Fact 1). This rules out statement III—but it doesn’t follow that all caps, a category that may include caps that are not baseball caps, have brims (statement I). Statement II cannot be confirmed, either, since it is possible, given the information, that all baseball caps are black. 392.b. The first statement cannot be true because only female birds lay eggs. Statement II is true because hens are chickens and chickens are birds. Statement III is also true because if only some chickens are hens, then some must not be hens. 393.d. None of the three statements is supported by the known facts. 394. c. Statements I and II are not supported by the facts. Statement III is true because if all storybooks have pictures and only some have words, then some storybooks have both words and pictures. 395. d. There is not enough information to support any of the statements. Robert is known to have a minvan, but it is not known which of his vehicles is red. Robert may have a pickup or sport utility vehicle, so the second statement cannot be supported. There is no way to know if Robert’s favorite color is red (statement III).

396. a. Since Maui is an island and islands are surrounded by water, Maui must be surrounded by water. There is not enough information to support statements II and III. 397. c. If all drink mixes are beverages and some beverages are red, then some drink mixes are red (statement I). Since all beverages are drinkable and all drink mixes are beverages, then all red drink mixes must be drinkable (statement III). Statement II can be ruled out. 398.d. There is no information in the facts to support statements I or II. Statement III is clearly wrong because, according to Fact 1, no frames cost less than $35. 399.b. Since some pens don’t write, some writing utensils don’t write (statement I). Since there are blue pens and since pens are writing utensils, some writing utensils are blue (statement II). There is not enough information to support statement III. 400. c. If Mary always tells the truth, then both Ann and Mary have cats (statements I and II), and Ann is lying (statement III). 401.b. Statement II is the only true statement. Since all dogs like to run, then the ones who like to swim also like to run. There is no support for statement I or statement III.

129

GovernmentAdda.com – ANSWERS –



Set 27

(Page 69)

402.d. After all the switches were made, Max is directly behind the dog, James is alongside the dog on the left, Ruby is alongside the dog on the right, and Rachel is behind Max. 403.b. Nurse Kemp has worked more shifts in a row than Nurse Calvin; therefore, Kemp has worked more than eight shifts. The number of Kemp’s shifts plus the number of Rogers’s shifts (five) cannot equal fifteen or more, the number of Miller’s shifts. Therefore, Kemp has worked nine shifts in a row (5 + 9 = 14). 404. c. If Randy is two months older than Greg, then Ned is three months older than Greg and one month older than Randy. Kent is younger than both Randy and Ned. Ned is the oldest. 405. c. After all the switches were made, Shawn is in front of the house. Ross is in the alley behind the house, Michael is on the north side, and Jed is on the south. 406.d. After all the switches were made, Mr. Kirk worked on Tuesday. Mr. Carter worked on Monday, Ms. Johnson on Wednesday, and Ms. Falk on Thursday. 407. a. Mr. Temple has the most seniority, but he does not want the job. Next in line is Mr. Rhodes, who has more seniority than Ms. West or Ms. Brody.

408.b. Tall, thin, and middle-aged are the elements of the description repeated most often and are therefore the most likely to be accurate. 409.b. Beth won the biggest prize, described as a higher medal than Jamie’s, which we’ve been told was a silver medal. Roberta and Michele both won bronze medals, which are lower ranking medals than silver. Beth is also described as having competed more times than Roberta— who has competed seven times. Jamie is described as having competed fewer times than Roberta, and Michele has competed three times. Therefore, Beth has competed more times than the others and has won the biggest prize to date. 410. c. After all the switching was done, Jenkins was directly behind the receiver. Calvin and Burton had fallen. Zeller remained in the rear. 411.d. Alexis is farther away than Frances, who is five miles away, and closer than Samantha, who is seven miles away. 412. a. Baxter should be assigned to study with Carter. Baxter cannot be assigned with Adam, because they have already been together for seven class periods. If Baxter is assigned to work with Dennis, that would leave Adam with Carter, but Carter does not want to work with Adam. 413. a. If George is sitting at Henry’s left, George’s seat is 252. The next seat to the left, then, is 251.

130

GovernmentAdda.com – ANSWERS –



Set 28

(Page 72)

414.d. The total of the three programs (2 million + 0.5 million + 3 million) is 5.5 million. That leaves 1.5 million (7 million – 5.5 million), and the only single program needing that amount is the senate office building remodeling. 415.b. The only two programs that total 1.5 million dollars are the harbor improvements and school music program. 416. a. The total cost of the school music program and national radio is $1 million, the amount left after the international airport and agricultural subsidies are funded. 417. c. J will only work in episodes in which M is working and there are no restrictions on O’s schedule. However, N will not work with K, so M must appear and O may appear. 418.d. K will not work with N, so choices c and e are incorrect. M can only work every other week, so choice a is incorrect. Since M is not working, J will not work, so choice b is incorrect. 419.b. Only choice b contains no more than two R-rated movies (Shout and Mist), at least one G and one PG (Fly, Abra Cadabra, and Jealousy), and only one foreign film (Mist). 420. c. The first showing of Trek will be over at 10:00. Then, the employees will need 20 minutes to clean the theater, which is 10:20. Since the movies always start on the quarter hour, the second showing of Trek will be 10:30.

131

421. e. Since Shout is doing the most business and Trek the second most, they should remain in the two largest theaters. Also, the theater never shows a foreign film in the largest theater. Theaters 3 and 4 must show the movies that are rated G and PG, so the movies that are there must stay there. The most logical choice is to put Mist in theater 5 and Fly in theater 6. 422. a. “Honey” and “Sittin’ on the Dock of the Bay” are either 3 and 4 or 4 and 3. The Rascals appear on the list right after Otis Redding, who cannot be #3 (or he would be followed by Bobby Goldsboro), so “Honey” is #3 and “Sittin’ on the Dock of the Bay” is #4; therefore, choices c and e are incorrect. The Rascals are #5 (because they are right after Otis Redding), and Cream appears right after them, so choice d is incorrect. Since Cream has song #6, it cannot be “Hey Jude,” so choice b is incorrect. 423.d. In the previous question, it was determined that #3 is “Honey,” #4 is “Sittin’ on the Dock of the Bay,” #5 is “People Got to Be Free,” and #6 is “Sunshine of Your Love.” Since the #1 song is not “Love Is Blue,” #1 is “Hey Jude,” and #2 is “Love Is Blue.”

GovernmentAdda.com – ANSWERS –



Set 29

(Page 76)

Here’s a quick illustration of how to work “logic game” puzzles, using the situation in questions 424 and 425 as an example. First, read the paragraph. Then, construct a diagram or table like the one below. Write down the letters that represent the names of the people at the party. Next, add any other information that is given. You know that Quentin is an accountant and Sarah is a florist; you know which objects represent their type of work. You also know that Thomas is dressed as a camera, so he must be the photographer.

Q

accountant

pencil

S

florist

flower

T

photographer

camera

R

U

Since none of the men is a doctor, Rachel must be the doctor. That leaves Ulysses, who must be the chef. Once you’ve filled in your diagram and made the deductions, answering the questions is the easy part.

Q

accountant

pencil

R

doctor

thermometer

S

florist

flower

T

photographer

camera

U

chef

spoon

424.b. See the table above. The thermometer costume logically would be worn by the doctor. According to the information, none of the men is a doctor. Also, Sarah is a florist, so Rachel must be the doctor wearing the thermometer costume.

425. e. Ulysses cannot be a doctor, because that is Rachel. Quentin is an accountant, Thomas must be a photographer, and Sarah is a florist. That leaves chef for Ulysses. We also know the chef must be a man, because neither of the women is dressed as a spoon. 426.d. The person who ordered the vegetable burger cannot be sitting in chairs 1 or 6, because she is sitting between two people. She also cannot be sitting in chairs 3 or 4, because those customers did not order sandwiches. Since she is not sitting in chair 2, she must be in chair 5. 427. c. The customer who ordered soup must be in chair 3 or 4, where the non-sandwich orders go. The other non-sandwich order is fried eggs, and that person is sitting next to the customer in chair 5 (who ordered the vegetable burger), so the fried eggs go to chair 4 and the soup to chair 3. 428.b. The orders that go to chairs 3, 4, 5, and 6 are already determined, so the ham sandwich must go to chair 1 or 2. The customer who ordered the hamburger is not sitting next to the person who ordered the soup in chair 3, so the hamburger must go to chair 1 and the ham sandwich to chair 2. 429. a. The person who ordered potato salad cannot be in chair 1 or 6, since he is sitting between two people. The person who ordered fried eggs ordered hash browns and is sitting in chair 4. The person who ordered potato salad is on one side of chair 4, either 3 or 5. He cannot be in chair 5 and still be next to both the hash browns and the cole slaw, so he must be in chair 3, which is where the soup was ordered. 430. c. If the potato salad is with the soup and the hash browns are with the fried eggs, then the cole slaw must be with the ham sandwich, in chairs 2, 3, and 4. The lettuce salad is with the vegetable burger in chair 5. The onion rings belong to the cheeseburger in chair 6, leaving the french fries for the hamburger in chair 1.

132

GovernmentAdda.com – ANSWERS –

431. a. The vice president’s car cannot be red, because that is the CEO’s car, which is in the first space. Nor can it be purple, because that is the treasurer’s car, which is in the last space, or yellow, because that is the secretary’s. The president’s car must be blue, because it is parked between a red car (in the first space) and a green car, which must be the vice president’s. 432. c. The CEO drives a red car and parks in the first space. Enid drives a green car; Bert’s car is not in the first space; David’s is not in the first space, but the last. Alice’s car is parked next to David’s, so Cheryl is the CEO.

433. e. Cheryl cannot be the secretary, since she’s the CEO, nor can Enid, because she drives a green car, and the secretary drives a yellow car. David’s, the purple car, is in the last space. Alice is the secretary, because her car is parked next to David’s, which is where the secretary’s car is parked.

133

GovernmentAdda.com – ANSWERS –



Set 30

(Page 79)

434.d. The Whippets cannot be in Jersey, Hudson, or Fulton, since they have beaten those teams. The Antelopes are in Groton, so the Whippets are in Ivy. 435. e. The Panthers cannot be in Ivy or Groton, because the Whippets and Antelopes are there. Fulton has beaten the Panthers, so they cannot be in Fulton. Fulton has also beaten the Kangaroos, so the only town left for the Kangaroos is Jersey. That leaves Hudson for the Panthers. 436.b. Every team and town is matched up, except Fulton and the Gazelles, so the Gazelles must be in Fulton. 437. a. Kevin is allergic to daisies and iris; he’s not getting gladioli because it’s not his housewarming. The roses are going to Jenny, leaving the carnations for Kevin. 438.d. Jenny is getting roses and Kevin is getting carnations. Neither Liz nor Inez would be getting a housewarming present. Michael is getting gladioli.

439. e. The only flowers unassigned are iris and daisies. Liz is allergic to daisies, so she is getting the iris. 440. e. The city that got the least rain is in the desert. New Town is in the mountains. Last Stand got more rain than Olliopolis, so it cannot be the city with the least rain; also, Mile City cannot be the city with the least rain. Olliopolis got 44 inches of rain. Therefore, Polberg is in the desert and got 12 inches of rain. 441. a. Olliopolis got 44 inches of rain. Last Stand got more rain than that, so it got 65 inches, which is the most. 442.b. Olliopolis got 44 inches of rain, Last Stand got 65, and Polberg got 12. New Town is in the mountains, and the city in the mountains got 32 inches of rain. Therefore, Mile City got 27. 443. c. Olliopolis got 44 inches of rain, so it is not in the desert or the forest. The city in the mountains got 32 inches of rain; the coast 27. Therefore, Olliopolis is in a valley.

134

GovernmentAdda.com – ANSWERS –



Set 31

(Page 81)

444.d. The moderator sits in seat #3. It cannot, then, be Gary or Jarrod or Lane, who sit next to the moderator. Heloise is not the moderator; therefore, the moderator is Kate. 445. a. Jarrod cannot sit in seat #3 because he is not the moderator. Nor can he sit in seat #2 or #4, because he does not sit next to the moderator. Heloise cannot sit on an end, nor in seat #3 or #4, so she is in seat #2, between the moderator (Kate) and Jarrod, who must be in seat #1. 446. e. Jarrod sits in seat #1 and is not the moderator; nor is he the pilot or the attorney. The attorney sits in seat #4 and cannot sit next to the explorer. Therefore, the pilot, Lane, is in seat #5, and the explorer must be in seat #1, Jarrod’s seat. 447.b. Jarrod is the explorer, Lane is the pilot, Kate is the moderator, and Gary is the attorney. Heloise must be the writer. 448.d. Zinnia plants tomatoes each year, so choice e is incorrect. Each year, she plants either carrots or cabbage, but not both. She will plant cabbage in the second year, so she will plant carrots in the first. She never plants carrots and peppers together, so the first year is tomatoes, carrots, beans and the second is tomatoes, cabbage, peppers.

449. c. Dusting must be done on Tuesday, Wednesday, or Thursday. However, the mopping is done on Thursday, and Terry does his task on Wednesday. Therefore, Sally does the dusting on Tuesday. 450.d. Terry does not dust, mop, do laundry, or vacuum. Therefore, Terry does the sweeping on Wednesday. 451.b. Dusting is on Tuesday, sweeping is on Wednesday, mopping is on Thursday, and laundry is on Friday. Therefore, the vacuuming is done on Monday. 452. e. Vernon does not vacuum, dust, or sweep. Randy does the vacuuming, Sally does the dusting, Terry does the sweeping—leaving laundry and mopping for Uma and Vernon. Uma does not do laundry; therefore, she must mop, and Vernon does the laundry. 453.d. Uma does the mopping, which is done on Thursday.

135

GovernmentAdda.com – ANSWERS –



Set 32

(Page 83)

454.d. By stating that fitness walking does not require a commute to a health club, the author stresses the convenience of this form of exercise. The paragraph also states that fitness walking will result in a good workout. Choice a is incorrect because no comparison to weight lifting is made. Choice b may seem like a logical answer, but the paragraph only refers to people who are fitness walkers, so for others, a health club might be a good investment. Choice c is not in the passage. Although choice e seems logical, the paragraph does not indicate that the wrong shoes will produce major injuries. 455. e. This answer is implied by the statement that redistribution is needed so that people in emerging nations can have proper medical care. Choices a, b, and c are not mentioned in the passage. Choice d is also incorrect—the passage indicates that the distribution of medicine, not its production, is inadequate. 456.b. This answer is clearly stated in the first sentence of the paragraph. There is no support in the passage for choices a, d, or e. As for choice c, although mediation is mentioned, the statement does not indicate that victims should be the mediators. 457. c. This choice is supported as the best answer because the paragraph indicates that low-fat ice cream was once an unpopular item, but now, because consumers are more health conscious and because there is a wider array of tasty low-fat foods, low-fat ice cream is a profitable item for ice cream store owners. There is no indication that choices a, b, d, or e are true based on the information given.

458. a. The paragraph clearly states that there are two differing opinions with regard to the use of calculators in the classroom. Although some people may believe that choice b is true, the paragraph does not indicate this. Choice c has no relation to the paragraph. Choice d makes logical sense, but the paragraph says nothing about cost. Choice e is an opinion that is not given in the paragraph. 459. e. This is clearly the best answer because the paragraph directly states that warm weather affects consumers’ inclination to spend. It furthers states that the sales of single-family homes was at an all-time high. There is no support for choice a or c. Choice b is wrong because even though there were high sales for a particular February, this does not mean that sales are not higher in other months. Choice d presents a misleading figure of 4 million. The paragraph states that the record of 4.75 million was at an annual, not a monthly, rate. 460.b. The last sentence in the paragraph clearly gives support for the idea that the interest in Shakespeare is due to the development of his characters. Choice a is incorrect because the writer never makes this type of comparison. Choice c is wrong because even though scholars are mentioned in the paragraph, there is no indication that the scholars are compiling the anthology. Choice d is wrong because there is no support to show that most New Yorkers are interested in this work. There is no support for choice e either.

136

GovernmentAdda.com – ANSWERS –

461. c. A change in employee social values over the past ten years is implied in the whole paragraph, but particularly in the first sentence. Choice a is incorrect because the loyalty of the managers to their corporations is never discussed. There is no support for choice b. In choice d, perhaps career advancement is less important than it once was, but the paragraph does not indicate that advancement is unimportant to managers. Choice e is an opinion that is not supported. 462.b. The support for choice b is given in the second sentence of the paragraph. Generation Xers like to work independently, which means they are self-directed. No support is given for either choice a or choice c. Choice d is not related to the paragraph. Although the paragraph mentions that Generation Xers like to be challenged, it does not say they like to challenge their bosses’ attitudes; therefore, choice e can be ruled out.

463. e. The support for choice e is in the third sentence “. . . we should make school uniforms mandatory.” There is no evidence provided to support choices a, b, and d. And although we know that teachers and administrators are spending some of their time enforcing dress code, the paragraph does not quantify how much of their time is spent that way, so there is no support for choice c.

137

GovernmentAdda.com – ANSWERS –



Set 33

(Page 86)

464.d. This answer is implied by the whole paragraph. The author stresses the need to read critically by performing thoughtful and careful operations on the text. Choice a is incorrect because the author never says that reading is dull. Choices b, c, and e are not supported by the paragraph. 465. a. The support for this choice is in the second sentence, which states that in some countries, toxic insecticides are still legal. Choice b is incorrect because even though polar regions are mentioned in the paragraph, there is no support for the idea that warmer regions are not just as affected. There is no support for choice c. Choice d can be ruled out because there is nothing to indicate that DDT and toxaphene are the most toxic. Choice e is illogical. 466. a. The second and third sentence combine to give support to choice a. The statement stresses that there must be a judge’s approval (i.e., legal authorization) before a search can be conducted. Choices b and d are wrong because it is not enough for the police to have direct evidence or a reasonable belief—a judge must authorize the search for it to be legal. Choices c and e are not mentioned in the passage. 467. e. The paragraph focuses on the idea that the jury system is different from what it was in colonial times. There is no support given for choices a, b, and c. Choice d is incorrect because, even though jurors in colonial times were expected to investigate and ask questions, this does not necessarily mean that they were more informed than today’s jurors. 468. e. This answer is clearly stated in the last sentence of the paragraph. Choice a can be ruled out because there is no support to show that study-

ing math is dangerous. Words are not mentioned in the passage, which rules out choice b. Choice d is a contradiction to the information in the passage. There is no support for choice c. 469.d. The last sentence states that new technologies are reported daily, and this implies that new technologies are being constantly developed. There is no support for choice a. With regard to choice b, stone tools were first used two and a half million years ago, but they were not necessarily in use all that time. Choice c is clearly wrong since the paragraph states when stone tools first came into use. Although some may agree that choice e is true, the author of the paragraph does not give support for this opinion. 470. a. The support for this choice is in the last sentence, which states that major public health campaigns that increase awareness and propose lifestyle changes are important in our fight against obesity. Choice b can be ruled out because although the paragraph states that obesity can lead to diabetes, it doesn’t tell us that it is the leading cause of this disease. Choices c and e might sound reasonable and true, but they are not supported in the paragraph. And although we are told that obesity has been connected to asthma, this fact is not quantified in any way, so choice d is also not supported by the information given. 471. b. This answer is clearly supported in the second sentence. Nothing in the paragraph suggests that it is a crime not to give a Miranda warning, so choice a is incorrect. Choice c is also wrong because police may interrogate as long as a warning is given. There is no support given for either choice d or e.

138

GovernmentAdda.com – ANSWERS –

472. c. The last sentence gives direct support for this response. Although children might be better protected from the sun than adults, the paragraph does not specifically cite statistics about children, so we can’t know for sure, ruling out choice a. There is no evidence provided in the paragraph to support choices b and d. Choice e is incorrect since the last sentence tells us that warnings about the sun’s dangers are frequent.

473.b. The second sentence points out that people should examine what they want from a fitness routine before signing up for a new exercise class. There is no evidence to support choice a. Choice c might sound reasonable due to the fact that the paragraph tells us that yoga has become very popular, but this statement is not supported by the information provided in the paragraph. Choices d and e are also not supported since the paragraph doesn’t tell us whether yoga is good for both body and mind or what people think about it.

139

GovernmentAdda.com – ANSWERS –



Set 34

(Page 90)

474.d. The final sentence of the paragraph supports choice d. The other choices are not supported by the passage. Choice c may seem correct at first, but the paragraph states that the new initiatives are simple and inexpensive, not major. Choice e might seem to represent a truth, but vegetarian options are not discussed in this paragraph. 475.d. The author of this statement suggests that doctors are less independent. The author stresses that many doctors have lost authority. There is no support for the opinion that doctors resent the healthcare managers, however—which rules out choice a. The doctors’ training is never mentioned (choice b). Doctors may care about their patients (choice c), but this information is not part of the paragraph. Choice e is not mentioned. 476. e. The second sentence states that threading a needle involves motor skill. The other choices are not in the paragraph. 477. a. The paragraph states that Mars once had a thick atmosphere, but that it was stripped away. The other choices, true or not, cannot be found in the passage. 478. a. The last sentence provides direct support for choice a. The author never suggests that any trees should be cut down or thinned out, which eliminates choices b and c. Choice d contradicts the author’s opinion. The author suggests that old growth forests have less debris, which rules out choice e.

479. c. The fact that the Pyramid scheme is set up by a con artist suggests that the honest people who invest have been fooled. Choices a and b are contradicted in the passage. The paragraph says that the Pyramid scheme originated in the 1920s, but does not say it had its heyday then; thus, choice d is incorrect. Choice e is a fact, but it is not mentioned in the passage. 480. a. This is expressed in the first sentence. Choices b, d, and e are not supported by the passage. Choice c is incorrect because the paragraph states that some Reality TV stars manage to parlay their fifteen minutes of fame into celebrity. 481. c. The statement that it is difficult to create an accurate profile of a contemporary knitter comes immediately after a discussion about how different today’s knitters are from one another and from knitters of the past. Choices a and d are not supported by the paragraph. Although the paragraph does discuss knitting done in group settings, it does not specifically say that more of today’s knitting is done in groups; therefore, choice b is incorrect. Young people may be turning to knitting in record numbers, but again, that statement is not verified by the information provided in the paragraph, so choice e must be ruled out as well.

140

GovernmentAdda.com – ANSWERS –



Set 35

(Page 93)

482.b. If it is more expensive to run a medical practice in a large city than a small town, it would make sense for doctors to charge more in large cities. Choices a, c, and e are incorrect because the information in these statements is extraneous to the author’s argument. Choice d is wrong because it supports, rather than refutes, the author’s argument. 483. e. The passage states that “doctors in large cities make more money than doctors in small towns or rural areas.” The speaker then assumes that if doctors all charge the same, they will all earn the same, but if doctors in large cities see more patients, they will still earn more money. 484. a. The argument is based on the idea that the government spends a great deal of money translating documents into different languages. Choices b and e make the argument somewhat weaker. Choice c offers no support for the argument. Choice d may offer some support, but choice a makes the argument much stronger.

141

485. c. If most people learn English within a short period of time, making English the official language is unnecessary. 486.d. The speaker maintains that to burn a flag is an act of freedom of speech, which is among the things the flag represents. 487. a. If an action is not included under freedom of speech, the speaker’s main argument is incorrect. 488.b. This is the best choice because it relates to a situation where a proposed law would actually violate the part of the Constitution it is intended to protect.

GovernmentAdda.com – ANSWERS –



Set 36

(Page 95)

489. a. Because the speaker is arguing that multiple guests should be allowed when fewer members are present, the purpose of the rule is to make sure members are not crowded by the presence of guests. There is no support for choices b, c, or d. Choice e is attractive, but it is not the best choice because there is no way the club could control which members would be at the club at any one time. 490. c. Joint pain caused by physical activity and that caused by arthritis may not respond the same way to medication. 491. e. This would indicate that the conditions of the football players and the speaker’s mother are similar. 492. c. The speaker uses analogies to compare crawling with learning arithmetic and reading and to compare walking with using a computer. The speaker is making the point that, in both cases, a child needs to learn one before learning the other.

493. e. This evidence would back up the speaker’s contention that young students should learn the basics before learning computers. Choices a and d, which are both about cost, would have no effect on the argument. Choices b and c are too vague. 494. a. If computers enhance the learning of arithmetic and reading, the speaker’s argument is not as strong. 495.b. The speaker refers to the safety of children because most people are concerned about that. The speaker does not make a comparison (choice a). Choice c can be ruled out because the speaker does not give a specific number. Choices d and e are incorrect because the speaker doesn’t give an account of any specific child, nor does he or she use any method of attack. 496. e. Since the speaker is basing the argument on the safety of children, if there were only a few accidents and none involved children, the argument is weaker.

142

GovernmentAdda.com – ANSWERS –



Set 37

(Page 97)

497.b. Lars provides information that supports Frances’s more general statements. Both agree that schools should spend money on educating children, not on providing breakfast. Choices a, d, and e are incorrect because they all imply that Frances and Lars are arguing in opposition to each other. Choice c can be ruled out because Lars’s position does not give any outcomes. 498.d. Both speakers rely on the fact that schools do not traditionally have the responsibility for providing students with breakfast. 499.d. The speakers support their arguments in different ways, but both are concerned with whether sixteen-year-olds should continue to be allowed to receive drivers’ licenses.

500. c. Quinn discusses the fairness of changing the law and raising the age at which one can receive a driver’s license. Emotion (choice b) may be involved, but the argument relies on the fairness issue. 501. e. Dakota discusses the actualities of increased traffic and the decline in the teaching of drivers’ education. She doesn’t use statistics (choice a). Her argument is not emotion-filled, which rules out choice b. She doesn’t mention fairness (choice c) and doesn’t tell stories about specific situations (choice d).

143

GovernmentAdda.com

GovernmentAdda.com – NOTES –

GovernmentAdda.com – NOTES –

GovernmentAdda.com – NOTES –

GovernmentAdda.com – NOTES –

GovernmentAdda.com – NOTES –

GovernmentAdda.com – NOTES –

Reasoning Power Question Bank By Governmentadda.com.pdf ...

There was a problem loading more pages. Whoops! There was a problem previewing this document. Retrying... Download. Connect more apps... Try one of the apps below to open or edit this item. Reasoning Power Question Bank By Governmentadda.com.pdf. Reasoning Power Question Bank By Governmentadda.com.

17MB Sizes 13 Downloads 1915 Views

Recommend Documents

Quant Power Question Bank By Governmentadda.com.pdf ...
A system of moral ( = ethical ) principles applied in the commercial world”. B- Let's hear it (p.47). Listening Script: ( Interview with Mr. Karim ... of long- distance relationships of all kinds! hakuna. Page 3 of 3. Quant Power Question Bank By G

Question Bank
UNIT-1: Electronic Components and Signals. Short questions: 1 Define: a) Amplitude b) Frequency c) Wavelength d) waveform e) phase. 2.Draw the symbol of any four semiconductor. 3.Draw the voltage source and current source. 4.Draw different types of w

Computer Question Bank - BankExamsToday
c) Web design d) Database management e) None of these. 16. Fourth generation mobile technology provides enhanced capabilities allowing the transfer of both.

Computer Question Bank - Bank Exams Today
a) supercomputer b) personal computer c) Laptop d) PDA e) None of these. 3. ... 10. The software that is used to create text-based documents are referred to as ...

Computer Question Bank - Bank Exams Today
What feature adjusts the top and bottom margins so that the text is centered vertically on ... Fourth generation mobile technology provides enhanced capabilities .... A USB communication device that supports data encryption for secure wireless ...

The Power of Comparative Reasoning
given criterion (max in this case) we call the resulting fam- ily of hash functions ... Another aspect of this method merits a discussion. Our choice of K leads to ..... and 27 positions relative to the descriptor center, for a total of 108 dimension

The Power of Comparative Reasoning
art machine learning methods with complex optimization setups. For solving ... in about 10 lines of code in most languages (2 lines in MAT-. LAB), and does not ...

Question Bank NEW
Question134 :When the Ashoka Mehta Committee was constituted? 134 :Tшëк yةŒpـ кلyk¨tà qـutـ кzـ ~إ¤yـR qW? A: 1975 B: 1976. C: 1977 D: 1978. A: 1975 B: 1976. C: 1977 D: ...... Reservation for SCs and STs to be provided at all levels

reasoning pdf bank exam
There was a problem previewing this document. Retrying... Download. Connect more apps... Try one of the apps below to open or edit this item. reasoning pdf ...

Essential Reasoning for Bank & SSC.pdf
(4) New Zealand (5) Malagasy. 9. (1) Producer (2) Director (3) Investor. (4) Financier (5) Entrepreneur. CLASSIFICATION (ODDMAN OUT). TEST OF REASONING. For more free Video / Audio Tutorials & Study Material visit. www.ssc-cgl2014.in. Facebook Page -

Question bank on Punjab.pdf
To which 'misl' did Ranjit Singh belong ? (a) Bhangi. (b) Ahluwalia. (c) Nihang. (d) Sukerchakia. 7. Where did Ranjit Singh modern factories to manufacture ...

MCS Question bank- group 10 -
the performance of different units or departments of a business. .... provides a guiding orientation for a broad range of Apple's business decisions such as the ...

Question Bank-I YEar.pdf
Page 1 of 2. I YEAR. I.) 10-mark questions. 1. Write an essay on the Salient features of the Indus Valley Civilization. 2. Write in detail about the Vedic Corpus of Literature. 3. Write in detail about the Socio-Economic conditions of the early Vedic

LSS & USS QUESTION BANK FINAL.pdf
PDF. The Limits of the Criminal Sanction by Herbert Packer. PDF File: The Limits Of The Criminal Sanction 1. Page 1. Whoops! There was a problem loading this page. Retrying... Whoops! There was a problem loading this page. Retrying... LSS & USS QUEST

Question Bank Partial Differential Equations
Find the PDE of the family of planes, the sum of whose x,y,z intercepts is ... Form the partial differential equation by eliminating the arbitrary constants a and.

Question Bank in AC Circuits.pdf
The root mean square (r.m.s.) value of a.c. is the same as. A. instantaneous value C. effective value. B. average value D. maximum value. 39. The r.m.s. value of ...

The Deep-Level-Reasoning-Question Effect: The Role ...
tion asking in those environments and a subsection on the self-explanation effect. The second ... cal in the dialogue and monologue-like conditions on each topic. An sample ..... other applications, Xtrain (Hu, 1998) and Microsoft Agent 2.0.

QUESTION BANK 4.pdf
(a) Data flip flop to Toggle flip flop. (b) Toggle flip flop to Data flip flop. (c) Data flip flop to JK flip flop. (d) Toggle flip flop to JK flip flop. (e) JK flip flop to Data flip flop. (f) JK flip flop to Toggle flip flop. (g) SR flip flop to JK

racemic modification question bank .pdf
racemic modification question bank .pdf. racemic modification question bank .pdf. Open. Extract. Open with. Sign In. Main menu. Displaying racemic modification ...

Aristotle Prep Critical Reasoning Question Bank.pdf
There was a problem previewing this document. Retrying... Download. Connect more apps... Try one of the apps below to open or edit this item. Aristotle Prep ...

reasoning questions for bank exams with answers pdf in hindi ...
reasoning questions for bank exams with answers pdf in hindi. reasoning questions for bank exams with answers pdf in hindi. Open. Extract. Open with. Sign In.

reasoning questions pdf for bank exams
File: Reasoning questions pdf for bank. exams. Download now. Click here if your download doesn't start automatically. Page 1 of 1. reasoning questions pdf for bank exams. reasoning questions pdf for bank exams. Open. Extract. Open with. Sign In. Main